Вы находитесь на странице: 1из 645

MyPasTest

Main Navigation
Question Browser: MRCP 1
Home
Subscriptions Question Browser Exam Builder Saved Exams
» MRCP 1
• Question Browser
• Timed Test A 51-year-old man presents to A&E with altered consciousness, his blood pressure is
• Mock Exam 80/50 mmHg, his skin is pigmented and he has a past history of Hashimoto’s
thyroiditis. His family say he has been tired for several months and has been losing
• Past Papers weight and complaining of abdominal pain.
• Random Questions
Which of the following results is most likely to be found on investigation?
• My Performance
• Media Bank Laboratory glucose level of 12.3 mmol/l
• New Multimedia Serum sodium level of 116 mmol/l
Online Extras Peaked T-waves on ECG
Library Serum potassium level of 2.4 mmol/l
Community Serum urea level of 3.2 mmol/l
Help
612
PasTest Store

My Account

Profile
Newsletters
My Career and Exams
Order History
Learning Goals
Question Filters

Security

Change Password
Sign Out

file:///E|/Shakhawan/Endocrinology/14.htm[3/13/2012 7:02:07 PM]


MyPasTest

Main Navigation
Question Browser: MRCP 1
Home
Subscriptions Question Browser Exam Builder Saved Exams
» MRCP 1 Session Progress
• Question Browser
Questions Correct 10
• Timed Test A 51-year-old man presents to A&E with altered
consciousness, his blood pressure is 80/50 mmHg, his skin Questions Incorrect 3
• Mock Exam
is pigmented and he has a past history of Hashimoto’s Questions Total 13
• Past Papers thyroiditis. His family say he has been tired for several
Questions Percentage 76 %
• Random Questions months and has been losing weight and complaining of
abdominal pain. More
• My Performance
• Media Bank Which of the following results is most likely to be
found on investigation?
• New Multimedia
Online Extras Laboratory glucose level of 12.3
Library mmol/l
Community Serum sodium level of 116 mmol/l Correct answer
Help Peaked T-waves on ECG Your answer
PasTest Store Serum potassium level of 2.4
mmol/l
Serum urea level of 3.2 mmol/l
My Account

Profile
Newsletters
My Career and Exams The clinical picture is suggestive of Addison’s disease
with hypotension, pigmentation and a prodromal period Reference: Normal Values
Order History
with symptoms in keeping with glucocorticoid deficiency
Learning Goals in a man with a history of another autoimmune Click to open/closeHaematology
Question Filters endocrine disorder. Other disorders that can be Click to open/closeImmunoglobulins
associated are diabetes and pernicious anaemia. Click to open/closeBiochemistry
Hypoglycaemia, hyponatraemia, hyperkalaemia and an Click to open/closeDiabetes Click to
Security elevated urea are all present during a hypoadrenal crisis open/closeEndocrinology Click to
due to steroid deficiency and subsequent salt and water open/closeBlood gases Click to
Change Password loss. Although the potassium is elevated it is unusual to open/closeCSF
find peaked T-waves on the ECG in patients with
Sign Out
hypoadrenalism: the complexes tend to be small, low-
volume ones.

612

© 2011 PasTest Ltd | About Us | Contact Us | Help

file:///E|/Shakhawan/Endocrinology/14a.htm[3/13/2012 7:02:08 PM]


MyPasTest

Main Navigation
Question Browser: MRCP 1
Home
Subscriptions Question Browser Exam Builder Saved Exams
» MRCP 1 Session Progress
• Question Browser
Questions Correct 0
• Timed Test A 31-year-old man is referred to the local hypertension
clinic because of recently discovered hypertension that is Questions Incorrect 0
• Mock Exam
labile and difficult to control. Questions Total 0
• Past Papers
Which of the following features is most likely to Questions Percentage 0%
• Random Questions
suggest a genetic/familial syndrome is the cause for More
• My Performance his hypertension?
• Media Bank
• New Multimedia Serum potassium level of 3.9 mmol/l

Online Extras Random blood glucose level of 9 mmol/l


Reference: Normal Values
Library A serum calcium level of 1.5 mmol/l
A family history of unexplained death in childbirth Click to
Community open/closeHaematology
Help A family history of papillary thyroid neoplasia Click to
open/closeImmunoglobulins
PasTest Store 613 Click to
open/closeBiochemistry
My Account Click to open/closeDiabetes
Click to
Profile open/closeEndocrinology
Click to open/closeBlood
Newsletters gases Click to
My Career and Exams open/closeCSF
Order History
Learning Goals
Question Filters

Security

Change Password
Sign Out

© 2011 PasTest Ltd | About Us | Contact Us | Help

file:///E|/Shakhawan/Endocrinology/15.htm[3/13/2012 7:02:09 PM]


MyPasTest

Main Navigation
Question Browser: MRCP 1
Home
Subscriptions Question Browser Exam Builder Saved Exams
» MRCP 1 Session Progress
• Question Browser
Questions Correct 1
• Timed Test A 31-year-old man is referred to the local hypertension
clinic because of recently discovered hypertension that is Questions Incorrect 0
• Mock Exam
labile and difficult to control. Questions Total 1
• Past Papers
Which of the following features is most likely to Questions Percentage 100 %
• Random Questions
suggest a genetic/familial syndrome is the cause for More
• My Performance his hypertension?
• Media Bank
• New Multimedia Serum potassium level of 3.9 mmol/l

Online Extras Random blood glucose level of 9


mmol/l
Library
A serum calcium level of 1.5 mmol/l
Community
A family history of unexplained death Your answer
Help in childbirth
PasTest Store A family history of papillary thyroid
neoplasia

My Account

Profile
Newsletters Adrenal causes of hypertension are Conn’s syndrome
My Career and Exams (primary hyperaldosteronism ), Cushing’s syndrome
(from adrenal adenoma, carcinoma or adrenal nodular Reference: Normal Values
Order History
hyperplasia) and phaeochromocytoma. Hypokalaemia is
Learning Goals a feature of Conn’s and Cushing’s syndromes. Click to open/closeHaematology
Question Filters Hyperglycaemia can be present in Cushing’s syndrome Click to open/closeImmunoglobulins
or phaeochromocytoma. Unexplained death in childbirth Click to open/closeBiochemistry
and a family history of thyroid neoplasia raise the Click to open/closeDiabetes Click to
Security possibility of multiple endocrine neoplasia type-2 open/closeEndocrinology Click to
(phaeochromocytoma, medullary thyroid tumours and open/closeBlood gases Click to
Change Password hyperparathyroidism) as the cause of his hypertension. open/closeCSF
A raised serum calcium level would also point towards
Sign Out
MEN2 (multiple endocrine neoplasia – type 2) but
hypocalcaemia is not associated with adrenal causes of
hypertension.

613

© 2011 PasTest Ltd | About Us | Contact Us | Help

file:///E|/Shakhawan/Endocrinology/15a.htm[3/13/2012 7:02:11 PM]


MyPasTest

Main Navigation
Question Browser: MRCP 1
Home
Subscriptions Question Browser Exam Builder Saved Exams
» MRCP 1
• Question Browser
• Timed Test A 25-year-old woman presents to a reproductive endocrinology clinic with a history of
• Mock Exam being unable to conceive after 2 years of using no contraception. It is thought she may
have polycystic ovarian syndrome.
• Past Papers
• Random Questions Which of the following is most likely to be associated with this condition?
• My Performance
A 28 day menstrual cycle
• Media Bank
Elevated LH/FSH ratio
• New Multimedia
Normal free-androgen index
Online Extras
Low levels of circulating insulin
Library
Normal BMI (body mass index)
Community
Help 614

PasTest Store

My Account

Profile
Newsletters
My Career and Exams
Order History
Learning Goals
Question Filters

Security

Change Password
Sign Out

file:///E|/Shakhawan/Endocrinology/16.htm[3/13/2012 7:02:12 PM]


MyPasTest

Main Navigation
Question Browser: MRCP 1
Home
Subscriptions Question Browser Exam Builder Saved Exams
» MRCP 1 Session Progress
• Question Browser
Questions Correct 2
• Timed Test A 25-year-old woman presents to a reproductive
endocrinology clinic with a history of being unable to Questions Incorrect 0
• Mock Exam
conceive after 2 years of using no contraception. It is Questions Total 2
• Past Papers thought she may have polycystic ovarian syndrome.
Questions Percentage 100 %
• Random Questions
Which of the following is most likely to be associated More
• My Performance with this condition?
• Media Bank
• New Multimedia A 28 day menstrual cycle

Online Extras Elevated LH/FSH ratio Your answer

Library Normal free-androgen index

Community Low levels of circulating insulin

Help Normal BMI (body mass index)

PasTest Store

My Account
Polycystic ovarian syndrome is one of the commonest
Profile causes of anovulatory infertility. Patients can have a
normal menstrual cycle but are more likely to have
Newsletters oligomenorrhoea. It is associated with a number of
My Career and Exams biochemical abnormalities, including raised LH levels,
normal or elevated testosterone but with a low SHBG Reference: Normal Values
Order History
(sex-hormone-binding globulin) resulting in a high free-
Learning Goals androgen index. Androstenedione levels can either be Click to open/closeHaematology
Question Filters normal or raised. The underlying biochemical defect in Click to open/closeImmunoglobulins
patients with PCOS is recognised to be insulin Click to open/closeBiochemistry
resistance. This causes high circulating insulin levels Click to open/closeDiabetes Click to
Security due to peripheral insulin resistance: therefore open/closeEndocrinology Click to
hyperinsulinaemia, and not low insulin levels, is open/closeBlood gases Click to
Change Password characteristic of the condition. The insulin resistance open/closeCSF
has been shown in both lean and obese patients with
Sign Out
the condition.

614

© 2011 PasTest Ltd | About Us | Contact Us | Help

file:///E|/Shakhawan/Endocrinology/16a.htm[3/13/2012 7:02:13 PM]


MyPasTest

Main Navigation
Question Browser: MRCP 1
Home
Subscriptions Question Browser Exam Builder Saved Exams
» MRCP 1 Session Progress
• Question Browser
Questions Correct 2
• Timed Test A 37-year-old woman presents to the endocrine clinic with
a history of hirsutism, acne and oligomenorrhoea. She is Questions Incorrect 0
• Mock Exam
having difficulty losing weight and has searched the Questions Total 2
• Past Papers Internet and thinks she may have polycystic ovarian
Questions Percentage 100 %
• Random Questions syndrome. She wants to discuss the implications of this.
• My Performance More
Which of the following is the most important issue to
• Media Bank discuss with her at this stage of her life?
• New Multimedia
Exercise regimens
Online Extras Reference: Normal Values
Does she want to have children
Library
Her blood glucose level Click to
Community open/closeHaematology
Treatment for her hirsutism
Help Click to
Weight-reduction diets open/closeImmunoglobulins
PasTest Store
Click to
615 open/closeBiochemistry
My Account Click to open/closeDiabetes
Click to
Profile open/closeEndocrinology
Click to open/closeBlood
Newsletters gases Click to
My Career and Exams open/closeCSF
Order History
Learning Goals
Question Filters

Security

Change Password
Sign Out

© 2011 PasTest Ltd | About Us | Contact Us | Help

file:///E|/Shakhawan/Endocrinology/17.htm[3/13/2012 7:02:15 PM]


MyPasTest

Main Navigation
Question Browser: MRCP 1
Home
Subscriptions Question Browser Exam Builder Saved Exams
» MRCP 1
• Question Browser
• Timed Test A 37-year-old woman presents to the endocrine clinic with a history of hirsutism, acne
• Mock Exam and oligomenorrhoea. She is having difficulty losing weight and has searched the
Internet and thinks she may have polycystic ovarian syndrome. She wants to discuss
• Past Papers the implications of this.
• Random Questions
Which of the following is the most important issue to discuss with her at this
• My Performance stage of her life?
• Media Bank
• New Multimedia Exercise regimens

Online Extras Does she want to have children Your answer

Library Her blood glucose level

Community Treatment for her hirsutism

Help Weight-reduction diets

PasTest Store

My Account
All the above are relevant and each should be discussed. The hirsutism and acne
Profile can be very difficult to deal with and can cause distress from a cosmetic point of
view. There are a variety of treatment options but Dianette (cyproterone acetate)
Newsletters is probably the most effective, along with cosmetic treatments like waxing,
My Career and Exams shaving, plucking or electrolysis. Her future risk of type-2 diabetes and associated
cardiovascular risk is very important and she should be advised about the need for
Order History
lifestyle treatments and the need to lose weight and exercise regularly to reduce
Learning Goals the chance of this happening. It is now recommended that all patients with PCOS
Question Filters have their fasting blood glucose level measured annually to pick up diabetes at an
earlier stage. However, the most important issue in a woman of her age is fertility,
as women with PCOS frequently require assistance with conception. Her age is
Security against her if she is going to have problems with fertility and requires help to
conceive. The commonest treatment is to induce ovulation with clomifene. She
Change Password needs to be advised that pregnancy also carries an increased risk of gestational
diabetes.
Sign Out

615

file:///E|/Shakhawan/Endocrinology/17a.htm[3/13/2012 7:02:16 PM]


MyPasTest

Main Navigation
Question Browser: MRCP 1
Home
Subscriptions Question Browser Exam Builder Saved Exams
» MRCP 1 Session Progress
• Question Browser
Questions Correct 3
• Timed Test A 29-year-old woman presents to her GP with a history of
weight loss, heat intolerance, poor concentration and Questions Incorrect 0
• Mock Exam
palpitations. Questions Total 3
• Past Papers Which of the following is most likely to be associated
Questions Percentage 100 %
• Random Questions with a diagnosis of thyroiditis associated with viral
infection? More
• My Performance
• Media Bank Bilateral exophthalmos
• New Multimedia Diffuse, smooth goitre
Online Extras Reduced uptake on thyroid isotope scan Reference: Normal Values
Library Positive thyroid peroxidase antibodies
Click to
Community Pretibial myxoedema open/closeHaematology
Help Click to
616 open/closeImmunoglobulins
PasTest Store
Click to
open/closeBiochemistry
My Account Click to open/closeDiabetes
Click to
Profile open/closeEndocrinology
Click to open/closeBlood
Newsletters gases Click to
My Career and Exams open/closeCSF
Order History
Learning Goals
Question Filters

Security

Change Password
Sign Out

© 2011 PasTest Ltd | About Us | Contact Us | Help

file:///E|/Shakhawan/Endocrinology/18.htm[3/13/2012 7:02:17 PM]


MyPasTest

Main Navigation
Question Browser: MRCP 1
Home
Subscriptions Question Browser Exam Builder Saved Exams
» MRCP 1 Session Progress
• Question Browser
Questions Correct 4
• Timed Test A 29-year-old woman presents to her GP with a history of
weight loss, heat intolerance, poor concentration and Questions Incorrect 0
• Mock Exam
palpitations. Questions Total 4
• Past Papers Which of the following is most likely to be associated
Questions Percentage 100 %
• Random Questions with a diagnosis of thyroiditis associated with viral
infection? More
• My Performance
• Media Bank Bilateral exophthalmos
• New Multimedia Diffuse, smooth goitre
Online Extras Reduced uptake on thyroid isotope Your answer
Library scan
Community Positive thyroid peroxidase antibodies
Help Pretibial myxoedema
PasTest Store

My Account
Graves’ disease is an autoimmune thyroid disorder and
Profile is more common in women than men. It is associated
with positive TPO (thyroid peroxidase antibodies) and
Newsletters
TSH (thyroid stimulating hormone) receptor antibodies.
My Career and Exams The goitre in Graves’ disease is normally diffusely
Order History enlarged rather than nodular. Uptake of radio-isotope is Reference: Normal Values
increased in Graves’ disease. In other conditions
Learning Goals causing thyrotoxicosis, like postpartum thyroiditis and Click to open/closeHaematology
Question Filters viral thyroiditis, the uptake is reduced. Graves’ disease Click to open/closeImmunoglobulins
is associated with a number of other features not found Click to open/closeBiochemistry
in other causes of thyrotoxicosis, eg exophthalmos, Click to open/closeDiabetes Click to
Security pretibial myxoedema and thyroid acropachy (a condition open/closeEndocrinology Click to
that affects the nails and looks like finger clubbing). The open/closeBlood gases Click to
Change Password exophthalmos can be unilateral or bilateral. open/closeCSF
Sign Out

616

© 2011 PasTest Ltd | About Us | Contact Us | Help

file:///E|/Shakhawan/Endocrinology/18a.htm[3/13/2012 7:02:19 PM]


MyPasTest

Main Navigation
Question Browser: MRCP 1
Home
Subscriptions Question Browser Exam Builder Saved Exams
» MRCP 1 Session Progress
• Question Browser
Questions Correct 4
• Timed Test A 34-year-old woman is referred to the endocrine clinic
with a history of thyrotoxicosis. At her first appointment she Questions Incorrect 0
• Mock Exam
is found to have a smooth goitre, lid lag and bilateral Questions Total 4
• Past Papers exophthalmos with puffy eyelids and conjunctival injection.
Questions Percentage 100 %
• Random Questions She wants to discuss treatment of her thyroid problem as
she is keen to become pregnant. More
• My Performance
• Media Bank What is the most likely treatment you would advise?
• New Multimedia
12–18 months of carbimazole alone
Online Extras Reference: Normal Values
12–18 months of propylthiouracil alone
Library
A combination of antithyroid drugs and thyroxine Click to
Community open/closeHaematology
Radioactive iodine (iodine-131)
Help Click to
Thyroidectomy open/closeImmunoglobulins
PasTest Store
Click to
617 open/closeBiochemistry
My Account Click to open/closeDiabetes
Click to
Profile open/closeEndocrinology
Click to open/closeBlood
Newsletters gases Click to
My Career and Exams open/closeCSF
Order History
Learning Goals
Question Filters

Security

Change Password
Sign Out

© 2011 PasTest Ltd | About Us | Contact Us | Help

file:///E|/Shakhawan/Endocrinology/19.htm[3/13/2012 7:02:20 PM]


MyPasTest

Main Navigation
Question Browser: MRCP 1
Home
Subscriptions Question Browser Exam Builder Saved Exams
» MRCP 1 Session Progress
• Question Browser
Questions Correct 5
• Timed Test A 34-year-old woman is referred to the endocrine clinic
with a history of thyrotoxicosis. At her first appointment she Questions Incorrect 0
• Mock Exam
is found to have a smooth goitre, lid lag and bilateral Questions Total 5
• Past Papers exophthalmos with puffy eyelids and conjunctival injection.
Questions Percentage 100 %
• Random Questions She wants to discuss treatment of her thyroid problem as
she is keen to become pregnant. More
• My Performance
• Media Bank What is the most likely treatment you would advise?
• New Multimedia
12–18 months of carbimazole alone
Online Extras
12–18 months of propylthiouracil alone Your answer
Library
A combination of antithyroid drugs and
Community thyroxine
Help Radioactive iodine (iodine-131)
PasTest Store Thyroidectomy

My Account

Profile
All the above treatments are recognised treatments for
Newsletters thyrotoxicosis. Antithyroid drugs (carbimazole and
My Career and Exams propylthiouracil) can be used alone and the dose
titrated down or up to keep the patient euthyroid. Reference: Normal Values
Order History
Treatment is usually for 12–18 months. However
Learning Goals carbimazole has been associated with aplasia cutis (a Click to open/closeHaematology
Question Filters scalp defect) in babies born to mothers treated during Click to open/closeImmunoglobulins
pregnancy. The ‘block and replace’ regimen of thyroxine Click to open/closeBiochemistry
and anti-thyroid drugs is not used in pregnancy as Click to open/closeDiabetes Click to
Security thyroxine does not cross the placenta. Surgery is an open/closeEndocrinology Click to
option for people with a large goitre, or for people who open/closeBlood gases Click to
Change Password fail on other treatments but would not be first choice in open/closeCSF
a young otherwise healthy woman. Radio-iodine is a
Sign Out
very effective treatment for thyrotoxicosis but is
contraindicated in women of childbearing age who are
contemplating pregnancy within 6 months.

617

© 2011 PasTest Ltd | About Us | Contact Us | Help

file:///E|/Shakhawan/Endocrinology/19a.htm[3/13/2012 7:02:22 PM]


MyPasTest

Main Navigation
Question Browser: MRCP 1
Home
Subscriptions Question Browser Exam Builder Saved Exams
» MRCP 1 Session Progress
• Question Browser
Questions Correct 5
• Timed Test A 30-year-old man and his wife present to a reproductive
endocrinology clinic because of infertility. The man is tall Questions Incorrect 0
• Mock Exam
with bilateral gynaecomastia. Examination of the testes Questions Total 5
• Past Papers reveals bilateral, small, firm testes.
Questions Percentage 100 %
• Random Questions
Which of the following investigations is most helpful More
• My Performance in diagnosing a patient with Klinefelter’s syndrome?
• Media Bank
• New Multimedia CT scan of the pituitary gland

Online Extras Chromosomal analysis


Reference: Normal Values
Library Measurement of serum gonadotrophins
Measurement of serum testosterone Click to
Community open/closeHaematology
Help Semen analysis Click to
open/closeImmunoglobulins
PasTest Store 618 Click to
open/closeBiochemistry
My Account Click to open/closeDiabetes
Click to
Profile open/closeEndocrinology
Click to open/closeBlood
Newsletters gases Click to
My Career and Exams open/closeCSF
Order History
Learning Goals
Question Filters

Security

Change Password
Sign Out

© 2011 PasTest Ltd | About Us | Contact Us | Help

file:///E|/Shakhawan/Endocrinology/20.htm[3/13/2012 7:02:23 PM]


MyPasTest

Main Navigation
Question Browser: MRCP 1
Home
Subscriptions Question Browser Exam Builder Saved Exams
» MRCP 1 Session Progress
• Question Browser
Questions Correct 6
• Timed Test A 30-year-old man and his wife present to a reproductive
endocrinology clinic because of infertility. The man is tall Questions Incorrect 0
• Mock Exam
with bilateral gynaecomastia. Examination of the testes Questions Total 6
• Past Papers reveals bilateral, small, firm testes.
Questions Percentage 100 %
• Random Questions
Which of the following investigations is most helpful More
• My Performance in diagnosing a patient with Klinefelter’s syndrome?
• Media Bank
• New Multimedia CT scan of the pituitary gland

Online Extras Chromosomal analysis Your answer

Library Measurement of serum gonadotrophins

Community Measurement of serum testosterone

Help Semen analysis

PasTest Store

My Account
Klinefelter’s syndrome is a genetic disorder with an
Profile extra X chromosome, giving a genotype XXY. It is
usually diagnosed in late pubertal or early adult life
Newsletters because of delayed sexual development or infertility. It
My Career and Exams is associated with hypogonadism giving raised
gonadotrophin levels and low testosterone levels. Reference: Normal Values
Order History
Semen analysis would show azoospermia.
Learning Goals Gynaecomastia is often present. CT brain scan will be Click to open/closeHaematology
Question Filters normal. The other tests are appropriate for investigation Click to open/closeImmunoglobulins
of male infertility. There are other cause of infertility Click to open/closeBiochemistry
that would give high gonadotrophin and low Click to open/closeDiabetes Click to
Security testosterone levels; any primary testicular disorder open/closeEndocrinology Click to
would give this pattern, eg cryptorchidism, mumps open/closeBlood gases Click to
Change Password orchitis, haemochromatosis, myotonic dystrophy and open/closeCSF
alcohol abuse. The only test that is specific for
Sign Out
Klinefelter’s syndrome is a genetic test for the XXY
genotype.

618

© 2011 PasTest Ltd | About Us | Contact Us | Help

file:///E|/Shakhawan/Endocrinology/20a.htm[3/13/2012 7:02:24 PM]


MyPasTest

Main Navigation
Question Browser: MRCP 1
Home
Subscriptions Question Browser Exam Builder Saved Exams
» MRCP 1
• Question Browser
• Timed Test A 26-year-old man is referred for gastroscopy because of several months of
• Mock Exam dyspepsia. He has routine bloods checked and is found to have a serum calcium level
of 3.2 mmol/l with a venous bicarbonate level of 33 mmol/l. Renal and liver function
• Past Papers are both normal. Chest X-ray is normal.
• Random Questions
What is the most likely cause of his hypercalcaemia?
• My Performance
• Media Bank Myeloma
• New Multimedia Metastatic malignancy
Online Extras Milk–alkali syndrome
Library Primary hyperparathyroidism
Community Sarcoidosis
Help
619
PasTest Store

My Account

Profile
Newsletters
My Career and Exams
Order History
Learning Goals
Question Filters

Security

Change Password
Sign Out

file:///E|/Shakhawan/Endocrinology/21.htm[3/13/2012 7:02:26 PM]


MyPasTest

Main Navigation
Question Browser: MRCP 1
Home
Subscriptions Question Browser Exam Builder Saved Exams
» MRCP 1 Session Progress
• Question Browser
Questions Correct 7
• Timed Test A 26-year-old man is referred for gastroscopy because of
several months of dyspepsia. He has routine bloods Questions Incorrect 0
• Mock Exam
checked and is found to have a serum calcium level of 3.2 Questions Total 7
• Past Papers mmol/l with a venous bicarbonate level of 33 mmol/l. Renal
Questions Percentage 100 %
• Random Questions and liver function are both normal. Chest X-ray is normal.
• My Performance More
What is the most likely cause of his hypercalcaemia?
• Media Bank
• New Multimedia Myeloma

Online Extras Metastatic malignancy

Library Milk–alkali syndrome Your answer

Community Primary hyperparathyroidism

Help Sarcoidosis

PasTest Store

My Account
All the above diagnoses are causes of hypercalcaemia.
Profile Symptoms of hypercalcaemia are commonly thirst,
polyuria, dyspepsia, malaise, bone pain and
Newsletters constipation. Myeloma and metastatic malignancy cause
My Career and Exams hypercalcaemia either by direct lytic lesions of bone or
by the production of PTHrP (parathyroid hormone- Reference: Normal Values
Order History
related protein). Myeloma would be extremely rare in
Learning Goals this age group. There is nothing in the history to Click to open/closeHaematology
Question Filters suggest bone pain or a diagnosis of metastatic Click to open/closeImmunoglobulins
malignancy. Primary hyperparathyroidism is most Click to open/closeBiochemistry
commonly found by chance in elderly women. It can be Click to open/closeDiabetes Click to
Security part of the multiple endocrine neoplasia syndromes and open/closeEndocrinology Click to
therefore can be seen in younger people. The normal open/closeBlood gases Click to
Change Password chest X-ray suggests that sarcoidosis is not the open/closeCSF
diagnosis in this case. The age of the patient, the
Sign Out
dyspeptic symptoms and the raised bicarbonate level
suggest the most likely diagnosis is the milk–alkali
syndrome caused by the ingestion of antacids.

619

© 2011 PasTest Ltd | About Us | Contact Us | Help

file:///E|/Shakhawan/Endocrinology/21a.htm[3/13/2012 7:02:27 PM]


MyPasTest

Main Navigation
Question Browser: MRCP 1
Home
Subscriptions Question Browser Exam Builder Saved Exams
» MRCP 1 Session Progress
• Question Browser
Questions Correct 0
• Timed Test A 17-year-old young woman is referred to the endocrine
clinic with primary amenorrhoea. She is of normal height Questions Incorrect 0
• Mock Exam
and weight. She has moderate hirsutism. A male cousin Questions Total 0
• Past Papers was seen in the clinic at the age of 8 years with precocious
Questions Percentage 0%
• Random Questions puberty.
What is the most likely cause for her primary More
• My Performance
amenorrhoea?
• Media Bank
• New Multimedia Congenital adrenal hyperplasia
Online Extras Hyperprolactinaemia Reference: Normal Values
Library Polycystic ovarian syndrome
Turner’s syndrome Click to
Community open/closeHaematology
Help Testicular feminisation syndrome Click to
open/closeImmunoglobulins
PasTest Store 621 Click to
open/closeBiochemistry
My Account Click to open/closeDiabetes
Click to
Profile open/closeEndocrinology
Click to open/closeBlood
Newsletters gases Click to
My Career and Exams open/closeCSF
Order History
Learning Goals
Question Filters

Security

Change Password
Sign Out

© 2011 PasTest Ltd | About Us | Contact Us | Help

file:///E|/Shakhawan/Endocrinology/22.htm[3/13/2012 7:02:28 PM]


MyPasTest

Main Navigation
Question Browser: MRCP 1
Home
Subscriptions Question Browser Exam Builder Saved Exams
» MRCP 1 Session Progress
• Question Browser
Questions Correct 1
• Timed Test A 17-year-old young woman is referred to the endocrine
clinic with primary amenorrhoea. She is of normal height Questions Incorrect 0
• Mock Exam
and weight. She has moderate hirsutism. A male cousin Questions Total 1
• Past Papers was seen in the clinic at the age of 8 years with precocious
Questions Percentage 100 %
• Random Questions puberty.
What is the most likely cause for her primary More
• My Performance
amenorrhoea?
• Media Bank
• New Multimedia Congenital adrenal hyperplasia Your answer
Online Extras Hyperprolactinaemia
Library Polycystic ovarian syndrome
Community Turner’s syndrome
Help Testicular feminisation syndrome
PasTest Store

My Account
Turner’s syndrome (genotype XO) is associated with
Profile primary amenorrhoea and short stature.
Hyperprolactinaemia and PCOS are more commonly
Newsletters
associated with secondary amenorrhoea. Testicular
My Career and Exams feminisation causes primary amenorrhoea and is caused
Order History by either a partial or complete androgen-receptor Reference: Normal Values
defect. Affected patients are genotypically male (XY)
Learning Goals but phenotypically female. Congenital adrenal Click to open/closeHaematology
Question Filters hyperplasia is most commonly caused by 21- Click to open/closeImmunoglobulins
hydroxylase deficiency. Severe forms present in infancy Click to open/closeBiochemistry
with salt-losing crises and females can have ambiguous Click to open/closeDiabetes Click to
Security genitalia. Milder forms cause precocious puberty in boys open/closeEndocrinology Click to
and virilism, hirsutism and primary amenorrhoea in open/closeBlood gases Click to
Change Password girls. Diagnosis is by finding raised serum 17- open/closeCSF
Sign Out hydroxyprogesterone levels that show a
hyperresponsiveness to ACTH. Treatment is with
glucocorticoids.

621

© 2011 PasTest Ltd | About Us | Contact Us | Help

file:///E|/Shakhawan/Endocrinology/22a.htm[3/13/2012 7:02:30 PM]


MyPasTest

Main Navigation
Question Browser: MRCP 1
Home
Subscriptions Question Browser Exam Builder Saved Exams
» MRCP 1 Session Progress
• Question Browser
Questions Correct 0
• Timed Test An 81-year-old woman is referred to the thyroid clinic with
increasing size of a pre-existing goitre. She has had long- Questions Incorrect 0
• Mock Exam
standing hypothyroidism and has been on a dose of Questions Total 0
• Past Papers
thyroxine of 100 μg daily for many years. Questions Percentage 0%
• Random Questions
• My Performance Which of the following primary thyroid cancers is she More
most likely to have?
• Media Bank
• New Multimedia Anaplastic thyroid cancer
Online Extras Follicular thyroid cancer Reference: Normal Values
Library Medullary thyroid cancer
Click to
Community Papillary thyroid cancer
open/closeHaematology
Help Thyroid lymphoma Click to
open/closeImmunoglobulins
PasTest Store
623 Click to
open/closeBiochemistry
My Account Click to open/closeDiabetes
Click to
Profile open/closeEndocrinology
Click to open/closeBlood
Newsletters gases Click to
My Career and Exams open/closeCSF
Order History
Learning Goals
Question Filters

Security

Change Password
Sign Out

© 2011 PasTest Ltd | About Us | Contact Us | Help

file:///E|/Shakhawan/Endocrinology/23.htm[3/13/2012 7:02:31 PM]


MyPasTest

Main Navigation
Question Browser: MRCP 1
Home
Subscriptions Question Browser Exam Builder Saved Exams
» MRCP 1 Session Progress
• Question Browser
Questions Correct 1
• Timed Test An 81-year-old woman is referred to the thyroid clinic with
increasing size of a pre-existing goitre. She has had long- Questions Incorrect 0
• Mock Exam
standing hypothyroidism and has been on a dose of Questions Total 1
• Past Papers
thyroxine of 100 μg daily for many years. Questions Percentage 100 %
• Random Questions
• My Performance Which of the following primary thyroid cancers is she More
most likely to have?
• Media Bank
• New Multimedia Anaplastic thyroid cancer
Online Extras Follicular thyroid cancer
Library Medullary thyroid cancer
Community Papillary thyroid cancer
Help Thyroid lymphoma Your answer
PasTest Store

My Account
Primary thyroid lymphoma is strongly associated with
Profile lymphocytic thyroiditis (Hashimoto’s disease), which is
Newsletters present in 80% of cases. It is predominantly a disease
of elderly women. Up to 30% of patients have a history
My Career and Exams of goitre and may be taking thyroxine. Anaplastic
Order History thyroid cancer is commonly found in elderly patients Reference: Normal Values
and usually presents with a thyroid mass. It can occur
Learning Goals in a pre-existing goitre. It is usually rapidly progressive Click to open/closeHaematology
Question Filters and has the worst prognosis of the thyroid cancers. Click to open/closeImmunoglobulins
Medullary thyroid cancer arises from within the ‘C’ cells Click to open/closeBiochemistry
of the thyroid, cells which produce calcitonin. It is Click to open/closeDiabetes Click to
Security commonly associated with multiple endocrine neoplasia open/closeEndocrinology Click to
type-2 (MEN2) along with primary hyperparathyroidism open/closeBlood gases Click to
Change Password and phaeochromocytoma. Follicular thyroid cancer is open/closeCSF
Sign Out typically found in middle-aged to older individuals and
usually presents as a solitary asymptomatic thyroid
nodule. Eventually 20% of patients will develop distant
metastases. Papillary thyroid cancer is most commonly
found in the 30–50 age group but can occur at any age.
It most commonly metastasises to the local lymph
nodes within the neck. Prognosis is better than the
other thyroid cancers.

623

© 2011 PasTest Ltd | About Us | Contact Us | Help

file:///E|/Shakhawan/Endocrinology/23a.htm[3/13/2012 7:02:33 PM]


MyPasTest

Main Navigation
Question Browser: MRCP 1
Home
Subscriptions Question Browser Exam Builder Saved Exams
» MRCP 1 Session Progress
• Question Browser
Questions Correct 2
• Timed Test A 52-year-old woman sees her GP complaining of a 4-kg
weight gain, dry hair and skin, she feels slow and always Questions Incorrect 0
• Mock Exam
has the heating up high. She has a small diffuse goitre. Her Questions Total 2
• Past Papers TSH level is 11 U/l (0.5-4.5) and free thyroxine 5 pmol/l.
Questions Percentage 100 %
• Random Questions Thyroid peroxidase antibody is positive at high titres. She
has two sisters who have had thyroid disease. More
• My Performance
What is the most likely diagnosis?
• Media Bank
• New Multimedia DeQuervain’s thyroiditis
Online Extras Follicular carcinoma Reference: Normal Values
Library Hashimoto’s thyroiditis
Graves’ disease Click to
Community open/closeHaematology
Help Nodular goitre Click to
open/closeImmunoglobulins
PasTest Store 625 Click to
open/closeBiochemistry
My Account Click to open/closeDiabetes
Click to
Profile open/closeEndocrinology
Click to open/closeBlood
Newsletters gases Click to
My Career and Exams open/closeCSF
Order History
Learning Goals
Question Filters

Security

Change Password
Sign Out

© 2011 PasTest Ltd | About Us | Contact Us | Help

file:///E|/Shakhawan/Endocrinology/24.htm[3/13/2012 7:02:34 PM]


MyPasTest

Main Navigation
Question Browser: MRCP 1
Home
Subscriptions Question Browser Exam Builder Saved Exams
» MRCP 1 Session Progress
• Question Browser
Questions Correct 3
• Timed Test A 52-year-old woman sees her GP complaining of a 4-kg
weight gain, dry hair and skin, she feels slow and always Questions Incorrect 0
• Mock Exam
has the heating up high. She has a small diffuse goitre. Her Questions Total 3
• Past Papers TSH level is 11 U/l (0.5-4.5) and free thyroxine 5 pmol/l.
Questions Percentage 100 %
• Random Questions Thyroid peroxidase antibody is positive at high titres. She
has two sisters who have had thyroid disease. More
• My Performance
What is the most likely diagnosis?
• Media Bank
• New Multimedia DeQuervain’s thyroiditis
Online Extras Follicular carcinoma
Library Hashimoto’s thyroiditis Your answer
Community Graves’ disease
Help Nodular goitre
PasTest Store

My Account
Hashimoto’s thyroiditis is an autoimmune thyroid
Profile disorder found more often in women than men. It is
associated with positive thyroid antibodies and there
Newsletters
may be a goitre that tends to be diffuse rather than
My Career and Exams multinodular. The gland is infiltrated with lymphocytes
Order History and patients can become hypothyroid. DeQuervain’s Reference: Normal Values
thyroiditis is not associated with positive antibodies and
Learning Goals tends to present with painful swelling in the neck. Click to open/closeHaematology
Question Filters Graves’ disease is also an autoimmune thyroid disorder Click to open/closeImmunoglobulins
but patients usually present with thyrotoxicosis. Click to open/closeBiochemistry
Follicular carcinoma can present with a thyroid swelling Click to open/closeDiabetes Click to
Security but not hypothyroidism. open/closeEndocrinology Click to
open/closeBlood gases Click to
Change Password open/closeCSF
Sign Out 625

© 2011 PasTest Ltd | About Us | Contact Us | Help

file:///E|/Shakhawan/Endocrinology/24a.htm[3/13/2012 7:02:36 PM]


MyPasTest

Main Navigation
Question Browser: MRCP 1
Home
Subscriptions Question Browser Exam Builder Saved Exams
» MRCP 1
• Question Browser
• Timed Test A 45-year-old man presents to his GP with headaches. His BP is 166/94 mmHg.
• Mock Exam Routine investigations reveal sodium 142 mmol/l, potassium 2.6 mmol/l, chloride 101
mmol/l and normal urea and creatinine levels. Plasma renin is undetectable and
• Past Papers aldosterone levels are raised. What is the most likely cause for his
• Random Questions hypertension?
• My Performance
Cushing’s syndrome
• Media Bank
Conn's syndrome
• New Multimedia
Phaeochromocytoma
Online Extras
Renal artery stenosis
Library
Acromegaly
Community
Help 627
PasTest Store

My Account

Profile
Newsletters
My Career and Exams
Order History
Learning Goals
Question Filters

Security

Change Password
Sign Out

file:///E|/Shakhawan/Endocrinology/25.htm[3/13/2012 7:02:37 PM]


MyPasTest

Main Navigation
Question Browser: MRCP 1
Home
Subscriptions Question Browser Exam Builder Saved Exams
» MRCP 1 Session Progress
• Question Browser
Questions Correct 4
• Timed Test A 45-year-old man presents to his GP with headaches. His
BP is 166/94 mmHg. Routine investigations reveal sodium Questions Incorrect 0
• Mock Exam
142 mmol/l, potassium 2.6 mmol/l, chloride 101 mmol/l Questions Total 4
• Past Papers and normal urea and creatinine levels. Plasma renin is
Questions Percentage 100 %
• Random Questions undetectable and aldosterone levels are raised. What is
the most likely cause for his hypertension? More
• My Performance
• Media Bank Cushing’s syndrome
• New Multimedia Conn's syndrome Your answer
Online Extras Phaeochromocytoma
Library Renal artery stenosis
Community Acromegaly
Help
PasTest Store

My Account Conn’s syndrome is a condition where there is a benign


adrenal adenoma that is secreting aldosterone. The
Profile renin level is low, and hypokalaemia is one of the
common findings as well as hypertension. Cushing’s
Newsletters
syndrome, acromegaly and phaeochromocytoma are all
My Career and Exams associated with secondary hypertension, but not with
Order History low renin and elevated aldosterone levels. Renal artery Reference: Normal Values
stenosis has high renin levels as well as high
Learning Goals aldosterone levels and so causes secondary Click to open/closeHaematology
Question Filters hyperaldosteronism. Treatment of Conn’s syndrome is Click to open/closeImmunoglobulins
with surgical excision of the adenoma or with Click to open/closeBiochemistry
potassium-sparing diuretics. Click to open/closeDiabetes Click to
Security open/closeEndocrinology Click to
open/closeBlood gases Click to
Change Password 627 open/closeCSF
Sign Out

© 2011 PasTest Ltd | About Us | Contact Us | Help

file:///E|/Shakhawan/Endocrinology/25a.htm[3/13/2012 7:02:38 PM]


MyPasTest

Main Navigation
Question Browser: MRCP 1
Home
Subscriptions Question Browser Exam Builder Saved Exams
» MRCP 1
• Question Browser
• Timed Test You are called to see a 36-year-old woman on the surgical ward who is 2 days’ post-
• Mock Exam thyroidectomy. She is complaining of tingling around her mouth and in her hands and
has developed spasm of her hands.
• Past Papers
• Random Questions What immediate treatment can you give that is most likely to resolve her
symptoms?
• My Performance
• Media Bank Ask her to breathe into a paper bag
• New Multimedia Intravenous calcium
Online Extras Intravenous diazepam
Library Intravenous glucose
Community Intravenous potassium
Help
628
PasTest Store

My Account

Profile
Newsletters
My Career and Exams
Order History
Learning Goals
Question Filters

Security

Change Password
Sign Out

file:///E|/Shakhawan/Endocrinology/26.htm[3/13/2012 7:02:40 PM]


MyPasTest

Main Navigation
Question Browser: MRCP 1
Home
Subscriptions Question Browser Exam Builder Saved Exams
» MRCP 1 Session Progress
• Question Browser
Questions Correct 5
• Timed Test You are called to see a 36-year-old woman on the surgical
ward who is 2 days’ post-thyroidectomy. She is complaining Questions Incorrect 0
• Mock Exam
of tingling around her mouth and in her hands and has Questions Total 5
• Past Papers developed spasm of her hands.
Questions Percentage 100 %
• Random Questions
What immediate treatment can you give that is most More
• My Performance likely to resolve her symptoms?
• Media Bank
• New Multimedia Ask her to breathe into a paper bag

Online Extras Intravenous calcium Your answer

Library Intravenous diazepam

Community Intravenous glucose

Help Intravenous potassium

PasTest Store

My Account
Postoperative complications of thyroidectomy include
Profile recurrent laryngeal damage, haemorrhage and
inadvertent removal of the parathyroid glands, although
Newsletters actually less than 1% of patients suffer permanent
My Career and Exams hypocalcaemia. The symptoms described are typical of
hypocalcaemia, although they can be seen in patients Reference: Normal Values
Order History
who are hyperventilating. The treatment of choice to
Learning Goals relieve the acute symptoms is 10 ml of 10% calcium Click to open/closeHaematology
Question Filters gluconate. Hypocalcaemia can be transient after thyroid Click to open/closeImmunoglobulins
surgery. If it is permanent then the patient needs long- Click to open/closeBiochemistry
term therapy with alfacalcidol to maintain her calcium Click to open/closeDiabetes Click to
Security levels. open/closeEndocrinology Click to
open/closeBlood gases Click to
Change Password open/closeCSF
Sign Out 628

© 2011 PasTest Ltd | About Us | Contact Us | Help

file:///E|/Shakhawan/Endocrinology/26a.htm[3/13/2012 7:02:41 PM]


MyPasTest

Main Navigation
Question Browser: MRCP 1
Home
Subscriptions Question Browser Exam Builder Saved Exams
» MRCP 1 Session Progress
• Question Browser
Questions Correct 5
• Timed Test A 28-year-old woman who is 3 months’ postpartum comes
to the surgery complaining of tiredness, she has had no Questions Incorrect 0
• Mock Exam
periods since the baby was born and she has been unable Questions Total 5
• Past Papers to breast-feed because of a lack of milk production. You
Questions Percentage 100 %
• Random Questions notice in her case sheet that she required a blood
transfusion after delivery for postpartum haemorrhage. More
• My Performance
What is the most likely diagnosis?
• Media Bank
• New Multimedia Empty sella syndrome
Online Extras Nelson’s syndrome Reference: Normal Values
Library Prolactinoma
Sheehan’s syndrome Click to
Community open/closeHaematology
Help Sipple’s syndrome Click to
open/closeImmunoglobulins
PasTest Store 629 Click to
open/closeBiochemistry
My Account Click to open/closeDiabetes
Click to
Profile open/closeEndocrinology
Click to open/closeBlood
Newsletters gases Click to
My Career and Exams open/closeCSF
Order History
Learning Goals
Question Filters

Security

Change Password
Sign Out

© 2011 PasTest Ltd | About Us | Contact Us | Help

file:///E|/Shakhawan/Endocrinology/27.htm[3/13/2012 7:02:42 PM]


MyPasTest

Main Navigation
Question Browser: MRCP 1
Home
Subscriptions Question Browser Exam Builder Saved Exams
» MRCP 1 Session Progress
• Question Browser
Questions Correct 6
• Timed Test A 28-year-old woman who is 3 months’ postpartum comes
to the surgery complaining of tiredness, she has had no Questions Incorrect 0
• Mock Exam
periods since the baby was born and she has been unable Questions Total 6
• Past Papers to breast-feed because of a lack of milk production. You
Questions Percentage 100 %
• Random Questions notice in her case sheet that she required a blood
transfusion after delivery for postpartum haemorrhage. More
• My Performance
What is the most likely diagnosis?
• Media Bank
• New Multimedia Empty sella syndrome
Online Extras Nelson’s syndrome
Library Prolactinoma
Community Sheehan’s syndrome Your answer
Help Sipple’s syndrome
PasTest Store

My Account
Sheehan’s syndrome is hypopituitarism following
Profile pregnancy complicated by haemorrhage at the time of
delivery. The haemorrhage and associated hypotension
Newsletters
is thought to cause pituitary infarction. Patients fail to
My Career and Exams lactate and remain amenorrhoeic postdelivery, and also
Order History develop other pituitary hormone deficiencies of the Reference: Normal Values
thyroid and adrenal axis. Nelson’s syndrome is seen in
Learning Goals patients with pituitary-driven Cushing’s syndrome many Click to open/closeHaematology
Question Filters years after bilateral adrenalectomy. Such patients are Click to open/closeImmunoglobulins
deeply pigmented. Empty sella syndrome is associated Click to open/closeBiochemistry
with hypopituitarism, imaging of the pituitary fossa Click to open/closeDiabetes Click to
Security shows no obvious pituitary tissue. The cause is open/closeEndocrinology Click to
unknown. Sipple’s syndrome is MEN2 open/closeBlood gases Click to
Change Password (hyperparathyroidism, medullary thyroid tumours and open/closeCSF
Sign Out phaeochromocytoma).

629

© 2011 PasTest Ltd | About Us | Contact Us | Help

file:///E|/Shakhawan/Endocrinology/27a.htm[3/13/2012 7:02:44 PM]


MyPasTest

Main Navigation
Question Browser: MRCP 1
Home
Subscriptions Question Browser Exam Builder Saved Exams
» MRCP 1 Session Progress
• Question Browser
Questions Correct 6
• Timed Test A 34-year-old man presents to the diabetes clinic with a
history of thirst, polyuria and a recent 3.2 kg (7 lb) weight Questions Incorrect 0
• Mock Exam
loss. His urine contains a small amount of ketones. Questions Total 6
• Past Papers Which of the following would suggest he is most
Questions Percentage 100 %
• Random Questions likely to have type-2 rather than type-1 diabetes?
• My Performance More
A BMI of 23
• Media Bank
High circulating insulin level
• New Multimedia
HLA type DR-3
Online Extras Reference: Normal Values
Positive islet-cell antibodies
Library
Plasma bicarbonate level of 8 mmol/l Click to
Community open/closeHaematology
Help 630 Click to
open/closeImmunoglobulins
PasTest Store
Click to
open/closeBiochemistry
My Account Click to open/closeDiabetes
Click to
Profile open/closeEndocrinology
Click to open/closeBlood
Newsletters gases Click to
My Career and Exams open/closeCSF
Order History
Learning Goals
Question Filters

Security

Change Password
Sign Out

© 2011 PasTest Ltd | About Us | Contact Us | Help

file:///E|/Shakhawan/Endocrinology/28.htm[3/13/2012 7:02:45 PM]


MyPasTest

Main Navigation
Question Browser: MRCP 1
Home
Subscriptions Question Browser Exam Builder Saved Exams
» MRCP 1
• Question Browser
• Timed Test A 34-year-old man presents to the diabetes clinic with a history of thirst, polyuria and
• Mock Exam a recent 3.2 kg (7 lb) weight loss. His urine contains a small amount of ketones.
Which of the following would suggest he is most likely to have type-2 rather
• Past Papers than type-1 diabetes?
• Random Questions
• My Performance A BMI of 23

• Media Bank High circulating insulin level Your answer

• New Multimedia HLA type DR-3

Online Extras Positive islet-cell antibodies

Library Plasma bicarbonate level of 8 mmol/l

Community
Help
PasTest Store
Type-1 diabetes mellitus is an autoimmune disorder characterised by the presence,
in many patients, of autoantibodies to the islet cell (ICA), insulin (IAA) or glutamic
My Account acid dehydrogenase (GAD). It is associated with HLA DR-3 or DR-4. Patients with
type-2 diabetes more commonly have a high BMI > 25, as this form of diabetes is
Profile associated with insulin resistance and high insulin levels rather than low insulin
levels. Finally, patients with type-2 diabetes mellitus can have + or ++ ketones in
Newsletters
the urine, but don’t usually have severe acidosis represented by a bicarbonate
My Career and Exams level of 8 mmol/l. This is more common in type-1 diabetes, with diabetic
Order History ketoacidosis still a common and sometimes fatal complication.

Learning Goals
Question Filters 630

Security

Change Password
Sign Out

file:///E|/Shakhawan/Endocrinology/28a.htm[3/13/2012 7:02:46 PM]


MyPasTest

Main Navigation
Question Browser: MRCP 1
Home
Subscriptions Question Browser Exam Builder Saved Exams
» MRCP 1 Session Progress
• Question Browser
Questions Correct 0
• Timed Test A 67-year-old man is referred to the hospital diabetes clinic
with a new diagnosis of type-2 diabetes mellitus. He has a Questions Incorrect 0
• Mock Exam
BMI of 29. Creatinine level is 150 nmol/l and he has 1+ Questions Total 0
• Past Papers protein on urinalysis. He has a past history of heart failure.
Questions Percentage 0%
• Random Questions
Which of the following drugs are you most likely to More
• My Performance prescribe?
• Media Bank
• New Multimedia Chlorpropamide

Online Extras Gliclazide


Reference: Normal Values
Library Pioglitazone
Vildagliptin Click to
Community open/closeHaematology
Help Metformin Click to
open/closeImmunoglobulins
PasTest Store 631 Click to
open/closeBiochemistry
My Account Click to open/closeDiabetes
Click to
Profile open/closeEndocrinology
Click to open/closeBlood
Newsletters gases Click to
My Career and Exams open/closeCSF
Order History
Learning Goals
Question Filters

Security

Change Password
Sign Out

© 2011 PasTest Ltd | About Us | Contact Us | Help

file:///E|/Shakhawan/Endocrinology/29.htm[3/13/2012 7:02:48 PM]


MyPasTest

Main Navigation
Question Browser: MRCP 1
Home
Subscriptions Question Browser Exam Builder Saved Exams
» MRCP 1 Session Progress
• Question Browser
Questions Correct 0
• Timed Test A 67-year-old man is referred to the hospital diabetes clinic
with a new diagnosis of type-2 diabetes mellitus. He has a Questions Incorrect 1
• Mock Exam
BMI of 29. Creatinine level is 150 nmol/l and he has 1+ Questions Total 1
• Past Papers protein on urinalysis. He has a past history of heart failure.
Questions Percentage 0%
• Random Questions
Which of the following drugs are you most likely to More
• My Performance prescribe?
• Media Bank
• New Multimedia Chlorpropamide

Online Extras Gliclazide Correct answer

Library Pioglitazone

Community Vildagliptin Your answer

Help Metformin

PasTest Store

My Account
Gliclazide is a sulphonylurea and can be used alone or
Profile in combination therapy for type-2 diabetes mellitus. It
acts by increasing insulin release from the pancreatic β-
Newsletters
cell. It can be used in mild to moderate renal failure.
My Career and Exams Pioglitazone is a thiazolidinedione, which promotes
Order History insulin sensitivity by its action on the PPAR γ receptor Reference: Normal Values
Learning Goals (peroxisome proliferator activated receptor- γ ). It is
associated with fluid retention and is contraindicated in Click to open/closeHaematology
Question Filters heart failure. Chlorpropamide, a sulphonylurea, is rarely Click to open/closeImmunoglobulins
used now and is excreted by the kidney. Metformin is a Click to open/closeBiochemistry
biguanide and, although its mechanism of action is not Click to open/closeDiabetes Click to
Security entirely clear, it reduces insulin resistance and hepatic open/closeEndocrinology Click to
glucose production. It is thought to be able to cause open/closeBlood gases Click to
Change Password open/closeCSF
lactic acidosis in certain circumstances and its use is
Sign Out contraindicated in patients with renal, hepatic or cardiac
failure. Vildagliptin is not recommended in patients with
moderate or worse renal impairment.

631

© 2011 PasTest Ltd | About Us | Contact Us | Help

file:///E|/Shakhawan/Endocrinology/29a.htm[3/13/2012 7:02:49 PM]


MyPasTest

Main Navigation
Question Browser: MRCP 1
Home
Subscriptions Question Browser Exam Builder Saved Exams
» MRCP 1 Session Progress
• Question Browser
Questions Correct 0
• Timed Test A 27-year-old woman with type-1 diabetes mellitus attends
for her routine review and says she is keen on becoming Questions Incorrect 1
• Mock Exam
pregnant. Questions Total 1
• Past Papers
Which of the following is the factor most likely to Questions Percentage 0%
• Random Questions
make you ask her to defer pregnancy at this stage? More
• My Performance
• Media Bank Minor background retinopathy
• New Multimedia Hb A 1C 9.4%
Online Extras She hasn’t been taking folic acid Reference: Normal Values
Library Microalbumin level of 6-mg excretion in 24 hours
Click to
Community Sensory neuropathy open/closeHaematology
Help Click to
632 open/closeImmunoglobulins
PasTest Store
Click to
open/closeBiochemistry
My Account Click to open/closeDiabetes
Click to
Profile open/closeEndocrinology
Click to open/closeBlood
Newsletters gases Click to
My Career and Exams open/closeCSF
Order History
Learning Goals
Question Filters

Security

Change Password
Sign Out

© 2011 PasTest Ltd | About Us | Contact Us | Help

file:///E|/Shakhawan/Endocrinology/30.htm[3/13/2012 7:02:51 PM]


MyPasTest

Main Navigation
Question Browser: MRCP 1
Home
Subscriptions Question Browser Exam Builder Saved Exams
» MRCP 1 Session Progress
• Question Browser
Questions Correct 0
• Timed Test A 27-year-old woman with type-1 diabetes mellitus attends
for her routine review and says she is keen on becoming Questions Incorrect 2
• Mock Exam
pregnant. Questions Total 2
• Past Papers
Which of the following is the factor most likely to Questions Percentage 0%
• Random Questions
make you ask her to defer pregnancy at this stage? More
• My Performance
• Media Bank Minor background retinopathy
• New Multimedia Hb A 1C 9.4% Correct answer
Online Extras She hasn’t been taking folic acid
Library Microalbumin level of 6-mg Your answer
Community excretion in 24 hours
Help Sensory neuropathy
PasTest Store

My Account
Pregnancy in type-1 diabetes is still associated with a
Profile two- to threefold increase in congenital abnormalities
Newsletters when compared with the background population. It is
also associated with higher neonatal morbidity and
My Career and Exams mortality and higher operative delivery rates.
Order History Prepregnancy counselling with the aim of bringing the Reference: Normal Values
Hb A 1c to near-normal levels is associated with better
Learning Goals
pregnancy outcomes. Diabetic retinopathy can progress Click to open/closeHaematology
Question Filters during pregnancy and the eyes must be checked at least Click to open/closeImmunoglobulins
once each trimester; if there is progression that is Click to open/closeBiochemistry
sight-threatening then the patient should be referred to Click to open/closeDiabetes Click to
Security an ophthalmologist. Background retinopathy would not open/closeEndocrinology Click to
be a contraindication to pregnancy. The microalbumin open/closeBlood gases Click to
Change Password level is within normal limits. Nephropathy can also open/closeCSF
Sign Out progress during pregnancy, but microalbuminuria would
not be a contraindication to pregnancy and nor would
sensory neuropathy. It is important that patients with
diabetes take 5 mg folic acid daily prepregnancy (this
reduces neural tube defects) and this can be started
when the patient starts trying to conceive, although it
should ideally be taken for 3 months beforehand. The
most important thing this woman can do is to improve
her glycaemic control before trying to conceive.

632

© 2011 PasTest Ltd | About Us | Contact Us | Help

file:///E|/Shakhawan/Endocrinology/30a.htm[3/13/2012 7:02:52 PM]


MyPasTest

Main Navigation
Question Browser: MRCP 1
Home
Subscriptions Question Browser Exam Builder Saved Exams
» MRCP 1
• Question Browser
• Timed Test A 17-year-old young woman with poorly controlled diabetes mellitus presents with a
• Mock Exam temperature, dehydration and altered consciousness. Her initial biochemistry shows
sodium 130 mmol/l, potassium 4.5 mmol/l, bicarbonate 6 mmol/l, urea 11.2 mmol/l,
• Past Papers creatinine 135 nmol/l and hydrogen ion 80.
• Random Questions
What is the most important immediate treatment?
• My Performance
• Media Bank Intravenous antibiotics
• New Multimedia Intravenous bicarbonate
Online Extras Intravenous fluids
Library Intravenous insulin
Community Intravenous potassium
Help
633
PasTest Store

My Account

Profile
Newsletters
My Career and Exams
Order History
Learning Goals
Question Filters

Security

Change Password
Sign Out

file:///E|/Shakhawan/Endocrinology/31.htm[3/13/2012 7:02:53 PM]


MyPasTest

Main Navigation
Question Browser: MRCP 1
Home
Subscriptions Question Browser Exam Builder Saved Exams
» MRCP 1 Session Progress
• Question Browser
Questions Correct 0
• Timed Test A 17-year-old young woman with poorly controlled
diabetes mellitus presents with a temperature, dehydration Questions Incorrect 3
• Mock Exam
and altered consciousness. Her initial biochemistry shows Questions Total 3
• Past Papers sodium 130 mmol/l, potassium 4.5 mmol/l, bicarbonate 6
Questions Percentage 0%
• Random Questions mmol/l, urea 11.2 mmol/l, creatinine 135 nmol/l and
hydrogen ion 80. More
• My Performance
• Media Bank What is the most important immediate treatment?
• New Multimedia
Intravenous antibiotics
Online Extras
Intravenous bicarbonate
Library
Intravenous fluids Correct answer
Community
Intravenous insulin Your answer
Help
Intravenous potassium
PasTest Store

My Account

Profile The patient has diabetic ketoacidosis, a condition still


associated with mortality in patients with type-1
Newsletters diabetes mellitus. There is a total body deficit of fluid
My Career and Exams and electrolytes. The most important initial treatment is
intravenous saline as the hypovolaemia is the factor Reference: Normal Values
Order History
most likely to cause the patient to die. Intravenous
Learning Goals insulin is required at an early stage; and, although it is Click to open/closeHaematology
Question Filters normal, the potassium level will rapidly fall with fluid Click to open/closeImmunoglobulins
and insulin treatment and the patient will require Click to open/closeBiochemistry
potassium replacement. Intravenous bicarbonate should Click to open/closeDiabetes Click to
Security not be used immediately as it can cause massive fluid open/closeEndocrinology Click to
shifts and precipitate cerebral oedema. It can be used if open/closeBlood gases Click to
Change Password hydrogen ion is greater than 100 and the patient is not open/closeCSF
responding to initial measures, in which case small
Sign Out
volumes of a 1.26% bicarbonate solution should be
used.

633

© 2011 PasTest Ltd | About Us | Contact Us | Help

file:///E|/Shakhawan/Endocrinology/31a.htm[3/13/2012 7:02:55 PM]


MyPasTest

Main Navigation
Question Browser: MRCP 1
Home
Subscriptions Question Browser Exam Builder Saved Exams
» MRCP 1 Session Progress
• Question Browser
Questions Correct 0
• Timed Test A 54-year-old man, newly diagnosed with type-2 diabetes
mellitus, presents to the clinic for his first assessment. He Questions Incorrect 3
• Mock Exam
is found to have changes in his eyes on fundoscopy. Questions Total 3
• Past Papers Which of the following is most likely to need
Questions Percentage 0%
• Random Questions immediate referral to the ophthalmologist?
• My Performance More
A few dot and blot haemorrhages
• Media Bank
Some hard exudates > 1 disc diameter from the
• New Multimedia fovea
Online Extras Cataract Reference: Normal Values
Library New vessels on the disc
Click to
Community Two soft exudates in the temporal field open/closeHaematology
Help Click to
634 open/closeImmunoglobulins
PasTest Store
Click to
open/closeBiochemistry
My Account Click to open/closeDiabetes
Click to
Profile open/closeEndocrinology
Click to open/closeBlood
Newsletters gases Click to
My Career and Exams open/closeCSF
Order History
Learning Goals
Question Filters

Security

Change Password
Sign Out

© 2011 PasTest Ltd | About Us | Contact Us | Help

file:///E|/Shakhawan/Endocrinology/32.htm[3/13/2012 7:02:56 PM]


MyPasTest

Main Navigation
Question Browser: MRCP 1
Home
Subscriptions Question Browser Exam Builder Saved Exams
» MRCP 1
• Question Browser
• Timed Test A 54-year-old man, newly diagnosed with type-2 diabetes mellitus, presents to the
• Mock Exam clinic for his first assessment. He is found to have changes in his eyes on fundoscopy.
Which of the following is most likely to need immediate referral to the
• Past Papers ophthalmologist?
• Random Questions
• My Performance A few dot and blot haemorrhages

• Media Bank Some hard exudates > 1 disc diameter from the fovea

• New Multimedia Cataract

Online Extras New vessels on the disc Your answer

Library Two soft exudates in the temporal field

Community
Help
PasTest Store
Background diabetic retinopathy consists of dot and blot haemorrhages and hard
exudates. Patients do not need to be referred to the ophthalmologist unless these
My Account are within 1 disc diameter of the fovea. This can be monitored annually at the
routine clinic. Cataracts appear about 10 years earlier in type-2 diabetes than in
Profile non-diabetic patients. If the vision is significantly affected the patients warrant
routine and not urgent referral to the ophthalmologist. Soft exudates suggest
Newsletters
retinal ischaemia, which would require routine referral to the ophthalmologist. New
My Career and Exams vessels anywhere in the fundus are a feature of proliferative retinopathy and, as
Order History new vessels have a risk of haemorrhage and can threaten sight, they should be
referred urgently to the ophthalmologist.
Learning Goals
Question Filters
634

Security

Change Password
Sign Out

file:///E|/Shakhawan/Endocrinology/32a.htm[3/13/2012 7:02:57 PM]


MyPasTest

Main Navigation
Question Browser: MRCP 1
Home
Subscriptions Question Browser Exam Builder Saved Exams
» MRCP 1 Session Progress
• Question Browser
Questions Correct 1
• Timed Test A 78-year-old man is admitted to hospital with a left
hemiparesis and altered consciousness. He is on aspirin 75 Questions Incorrect 3
• Mock Exam
mg, bendrofluazide 2.5 mg, atorvastatin 10 mg and Questions Total 4
• Past Papers glibenclamide 15 mg daily. His wife says he has been
Questions Percentage 25 %
• Random Questions unwell for a couple of days and has been off his food. She
has still been giving him all his medication. More
• My Performance
• Media Bank Which of the following tests is going to be most
helpful in finding an immediately reversible cause for
• New Multimedia
his symptoms?
Online Extras Reference: Normal Values
Library Blood glucose level
CT brain scan Click to
Community open/closeHaematology
Help ECG Click to
Serum creatinine level open/closeImmunoglobulins
PasTest Store
Click to
Troponin level open/closeBiochemistry
My Account Click to open/closeDiabetes
635 Click to
Profile open/closeEndocrinology
Click to open/closeBlood
Newsletters gases Click to
My Career and Exams open/closeCSF
Order History
Learning Goals
Question Filters

Security

Change Password
Sign Out

© 2011 PasTest Ltd | About Us | Contact Us | Help

file:///E|/Shakhawan/Endocrinology/33.htm[3/13/2012 7:02:59 PM]


MyPasTest

Main Navigation
Question Browser: MRCP 1
Home
Subscriptions Question Browser Exam Builder Saved Exams
» MRCP 1 Session Progress
• Question Browser
Questions Correct 2
• Timed Test A 78-year-old man is admitted to hospital with a left
hemiparesis and altered consciousness. He is on aspirin 75 Questions Incorrect 3
• Mock Exam
mg, bendrofluazide 2.5 mg, atorvastatin 10 mg and Questions Total 5
• Past Papers glibenclamide 15 mg daily. His wife says he has been
Questions Percentage 40 %
• Random Questions unwell for a couple of days and has been off his food. She
has still been giving him all his medication. More
• My Performance
• Media Bank Which of the following tests is going to be most
helpful in finding an immediately reversible cause for
• New Multimedia
his symptoms?
Online Extras
Library Blood glucose level Your answer
Community CT brain scan
Help ECG
PasTest Store Serum creatinine level
Troponin level

My Account

Profile
Newsletters Hypoglycaemia in the elderly is a not uncommon
My Career and Exams problem, even if only treated with oral hypoglycaemic
agents. Glibenclamide has a long half-life and should Reference: Normal Values
Order History
therefore be avoided in the elderly. The reduced food
Learning Goals intake and the ongoing intake of medication in this Click to open/closeHaematology
Question Filters patient is likely to have caused hypoglycaemia, which Click to open/closeImmunoglobulins
can be associated with neurological symptoms in the Click to open/closeBiochemistry
elderly. The neurological symptoms will resolve Click to open/closeDiabetes Click to
Security promptly with intravenous dextrose. Although a open/closeEndocrinology Click to
myocardial infarction or a cerebrovascular accident open/closeBlood gases Click to
Change Password could have caused his symptoms, they would not be open/closeCSF
immediately reversible.
Sign Out

635

© 2011 PasTest Ltd | About Us | Contact Us | Help

file:///E|/Shakhawan/Endocrinology/33a.htm[3/13/2012 7:03:00 PM]


MyPasTest

Main Navigation
Question Browser: MRCP 1
Home
Subscriptions Question Browser Exam Builder Saved Exams
» MRCP 1 Session Progress
• Question Browser
Questions Correct 2
• Timed Test A 43-year-old woman presents with weight loss,
palpitations, diarrhoea and a cessation of periods. She has Questions Incorrect 3
• Mock Exam
been treated by her GP for anxiety. Examination reveals a Questions Total 5
• Past Papers single nodule on the left of her thyroid, about 1.5 cm in
Questions Percentage 40 %
• Random Questions diameter. Thyroid scanning with technetium shows
increased uptake within the nodule with reduced activity More
• My Performance
throughout the rest of the gland. Thyroid function tests
• Media Bank showed a free thyroxine of 30 pmol/l (9–25 pmol/l), TSH <
• New Multimedia 0.05 mU/l (0.5–5).

Online Extras Based upon these findings, what would be the Reference: Normal Values
Library definitive treatment?
Click to
Community Radioactive iodine therapy open/closeHaematology
Help Carbimazole Click to
open/closeImmunoglobulins
PasTest Store Surgical excision Click to
Propanolol therapy open/closeBiochemistry
My Account Click to open/closeDiabetes
High-dose carbimazole therapy with thyroxine Click to
replacement open/closeEndocrinology
Profile
Click to open/closeBlood
Newsletters 1227 gases Click to
My Career and Exams open/closeCSF
Order History
Learning Goals
Question Filters

Security

Change Password
Sign Out

© 2011 PasTest Ltd | About Us | Contact Us | Help

file:///E|/Shakhawan/Endocrinology/34.htm[3/13/2012 7:03:01 PM]


MyPasTest

Main Navigation
Question Browser: MRCP 1
Home
Subscriptions Question Browser Exam Builder Saved Exams
» MRCP 1
• Question Browser
• Timed Test A 43-year-old woman presents with weight loss, palpitations, diarrhoea and a
• Mock Exam cessation of periods. She has been treated by her GP for anxiety. Examination reveals
a single nodule on the left of her thyroid, about 1.5 cm in diameter. Thyroid scanning
• Past Papers with technetium shows increased uptake within the nodule with reduced activity
• Random Questions throughout the rest of the gland. Thyroid function tests showed a free thyroxine of 30
pmol/l (9–25 pmol/l), TSH < 0.05 mU/l (0.5–5).
• My Performance
• Media Bank Based upon these findings, what would be the definitive treatment?
• New Multimedia
Radioactive iodine therapy Your answer
Online Extras
Carbimazole
Library
Surgical excision
Community
Propanolol therapy
Help
High-dose carbimazole therapy with thyroxine replacement
PasTest Store

My Account

Profile Toxic thyroid nodules preferentially take up radioactive iodine. This makes them
particularly amenable to radioactive iodine treatment. A dose of 300–500 MBq is
Newsletters usually sufficient to cure the thyrotoxicosis. Radioiodine therapy is contraindicated
My Career and Exams in children and women who are lactating or pregnant, and where the safety of
cohabitants from the effects of radioactivity can’t be guaranteed. Surgery is the
Order History
next best option for patients in whom radioiodine is contraindicated or refused.
Learning Goals Carbimazole alone or in combination with thyroxine replacement is used in the
Question Filters medical treatment of Grave’s disease; Propanolol is a useful symptom reliever for
thyrotoxicosis.

Security
1227
Change Password
Sign Out

file:///E|/Shakhawan/Endocrinology/34a.htm[3/13/2012 7:03:03 PM]


MyPasTest

Main Navigation
Question Browser: MRCP 1
Home
Subscriptions Question Browser Exam Builder Saved Exams
» MRCP 1
• Question Browser
• Timed Test A 37-year-old woman presents to A&E after chasing thieves who were stealing her
• Mock Exam car. Her blood pressure is noted to be 185/110 mmHg on admission. She admits to
episodic headaches and feeling stressed and anxious. She is of normal appearance,
• Past Papers her serum calcium on admission is noted to be 2.95 mmol/l with normal renal
• Random Questions function, abdominal ultrasound reveals a possible adrenal mass.
• My Performance What is the most likely diagnosis?
• Media Bank
• New Multimedia Phaeochromocytoma

Online Extras Carcinoid syndrome

Library MEN-1

Community MEN-2a

Help MEN-2b

PasTest Store 1228

My Account

Profile
Newsletters
My Career and Exams
Order History
Learning Goals
Question Filters

Security

Change Password
Sign Out

file:///E|/Shakhawan/Endocrinology/35.htm[3/13/2012 7:03:04 PM]


MyPasTest

Main Navigation
Question Browser: MRCP 1
Home
Subscriptions Question Browser Exam Builder Saved Exams
» MRCP 1
• Question Browser
• Timed Test A 24-year-old adopted man presents with transient left-sided weakness of his arm,
• Mock Exam which resolves after a few hours. His only other history of note is a reduced libido and
inability to maintain erections. On examination he appears to have a spotty skin
• Past Papers pigmentation. You notice a heart murmur, and there is suggestion of a left atrial mass
• Random Questions on echo. His prolactin is elevated at 2000 µmol/l.
• My Performance What is the most likely diagnosis?
• Media Bank
• New Multimedia Left atrial myxoma

Online Extras Carney complex

Library Prolactinoma

Community Protein C deficiency

Help Somatisation disorder

PasTest Store 1229

My Account

Profile
Newsletters
My Career and Exams
Order History
Learning Goals
Question Filters

Security

Change Password
Sign Out

file:///E|/Shakhawan/Endocrinology/37.htm[3/13/2012 7:03:05 PM]


MyPasTest

Main Navigation
Question Browser: MRCP 1
Home
Subscriptions Question Browser Exam Builder Saved Exams
» MRCP 1 Session Progress
• Question Browser
Questions Correct 5
• Timed Test A 24-year-old adopted man presents with transient left-
sided weakness of his arm, which resolves after a few Questions Incorrect 3
• Mock Exam
hours. His only other history of note is a reduced libido and Questions Total 8
• Past Papers inability to maintain erections. On examination he appears
Questions Percentage 62 %
• Random Questions to have a spotty skin pigmentation. You notice a heart
murmur, and there is suggestion of a left atrial mass on More
• My Performance
echo. His prolactin is elevated at 2000 µmol/l.
• Media Bank
What is the most likely diagnosis?
• New Multimedia
Online Extras Left atrial myxoma
Library Carney complex Your answer
Community Prolactinoma
Help Protein C deficiency
PasTest Store Somatisation disorder

My Account

Profile
This man has spotty skin pigmentation, probable
Newsletters prolactinoma and a probable left atrial myxoma. Carney
My Career and Exams complex is diagnosable with two features out of spotty
skin pigmentation, myxoma, endocrine tumours Reference: Normal Values
Order History
(commonest being primary pigmented nodular
Learning Goals adrenocortical disease), but it is also associated with Click to open/closeHaematology
Question Filters Sertoli-cell tumours, growth hormone- or prolactin- Click to open/closeImmunoglobulins
producing pituitary adenomas, thyroid adenomas and Click to open/closeBiochemistry
ovarian cysts) and psammomatous melanotic Click to open/closeDiabetes Click to
Security schwannoma (PMS). It is also diagnosable on the open/closeEndocrinology Click to
presence of one feature and an affected first-degree open/closeBlood gases Click to
Change Password relative. It is an autosomal-dominant condition caused open/closeCSF
by an inactivating mutation of protein kinase A on
Sign Out
chromosome 17.

1229

© 2011 PasTest Ltd | About Us | Contact Us | Help

file:///E|/Shakhawan/Endocrinology/37a.htm[3/13/2012 7:03:07 PM]


MyPasTest

Main Navigation
Question Browser: MRCP 1
Home
Subscriptions Question Browser Exam Builder Saved Exams
» MRCP 1 Session Progress
• Question Browser
Questions Correct 5
• Timed Test A 28-year-old man presents to casualty with a sudden loss
of vision in his right eye. His only past history of note is a Questions Incorrect 3
• Mock Exam
previous cerebellar haemorrhage. On examination he has Questions Total 8
• Past Papers evidence of bilateral retinal angiomas, and a partial retinal
Questions Percentage 62 %
• Random Questions detachment in his right eye.
• My Performance More
What is the most likely diagnosis?
• Media Bank
• New Multimedia Simple traumatic retinal detachment

Online Extras Clotting disorder


Reference: Normal Values
Library Bleeding due to hypertension
von Hippel–Lindau disease Click to
Community open/closeHaematology
Help McCune–Albright syndrome Click to
open/closeImmunoglobulins
PasTest Store 1230 Click to
open/closeBiochemistry
My Account Click to open/closeDiabetes
Click to
Profile open/closeEndocrinology
Click to open/closeBlood
Newsletters gases Click to
My Career and Exams open/closeCSF
Order History
Learning Goals
Question Filters

Security

Change Password
Sign Out

© 2011 PasTest Ltd | About Us | Contact Us | Help

file:///E|/Shakhawan/Endocrinology/38.htm[3/13/2012 7:03:08 PM]


MyPasTest

Main Navigation
Question Browser: MRCP 1
Home
Subscriptions Question Browser Exam Builder Saved Exams
» MRCP 1 Session Progress
• Question Browser
Questions Correct 6
• Timed Test A 28-year-old man presents to casualty with a sudden loss
of vision in his right eye. His only past history of note is a Questions Incorrect 3
• Mock Exam
previous cerebellar haemorrhage. On examination he has Questions Total 9
• Past Papers evidence of bilateral retinal angiomas, and a partial retinal
Questions Percentage 66 %
• Random Questions detachment in his right eye.
• My Performance More
What is the most likely diagnosis?
• Media Bank
• New Multimedia Simple traumatic retinal detachment

Online Extras Clotting disorder

Library Bleeding due to hypertension

Community von Hippel–Lindau disease Your answer

Help McCune–Albright syndrome

PasTest Store

My Account
Von Hippel–Lindau disease is characterised by CNS and
Profile retinal haemangioblastomas (presenting feature in 40%
of patients), renal cysts and carcinomas (occurring
Newsletters later), phaeochromocytoma (20% of affected families,
My Career and Exams bilateral in 40% of patients) and pancreatic tumours
(50% non-functioning). The prevalence is 1 in 39,000 Reference: Normal Values
Order History
with a mean age at presentation of 27 years. Renal
Learning Goals cysts occur later, with 70% of patients having them by Click to open/closeHaematology
Question Filters age 60 years. Ideally, genetic testing in affected Click to open/closeImmunoglobulins
families should take place around the age of 5 years. Click to open/closeBiochemistry
Affected individuals then require yearly urinalysis, Click to open/closeDiabetes Click to
Security catecholamine screening, fluorescein angiography, with open/closeEndocrinology Click to
3-yearly brain MRI scanning. open/closeBlood gases Click to
Change Password open/closeCSF
Sign Out
1230

© 2011 PasTest Ltd | About Us | Contact Us | Help

file:///E|/Shakhawan/Endocrinology/38a.htm[3/13/2012 7:03:09 PM]


MyPasTest

Main Navigation
Question Browser: MRCP 1
Home
Subscriptions Question Browser Exam Builder Saved Exams
» MRCP 1 Session Progress
• Question Browser
Questions Correct 1
• Timed Test A 37-year-old woman presents to A&E after chasing
thieves who were stealing her car. Her blood pressure is Questions Incorrect 0
• Mock Exam
noted to be 185/110 mmHg on admission. She admits to Questions Total 1
• Past Papers episodic headaches and feeling stressed and anxious. She is
Questions Percentage 100 %
• Random Questions of normal appearance, her serum calcium on admission is
noted to be 2.95 mmol/l with normal renal function, More
• My Performance
abdominal ultrasound reveals a possible adrenal mass.
• Media Bank
What is the most likely diagnosis?
• New Multimedia
Online Extras Phaeochromocytoma
Library Carcinoid syndrome
Community MEN-1
Help MEN-2a Your answer
PasTest Store MEN-2b

My Account

Profile
MEN-2 (multiple endocrine neoplasia – type 2) is
Newsletters associated with medullary thyroid carcinoma (almost
My Career and Exams always), parathyroid chief-cell hyperplasia (10–25%)
and phaeochromocytoma (20–50%). MEN-2b is also Reference: Normal Values
Order History
associated with a marfanoid appearance, whereas MEN-
Learning Goals 2a is not. The cause is an autosomal-dominant inherited Click to open/closeHaematology
Question Filters genetic mutation on the long arm of chromosome 10.
Correct management of this patient includes appropriate
a- and β -blockade prior to surgical removal of the
Security phaeochromocytoma, screening for medullary thyroid
carcinoma with the iv pentagastrin test and calcitonin
Change Password assay, then likely thyroidectomy, and probable
Sign Out parathyroidectomy in the hands of an experienced
surgeon. The prognosis of MEN-2 is variable, but,
overall, the 10-year survival rate is around 65%.

1228

file:///E|/Shakhawan/Endocrinology/35a.htm[3/13/2012 7:03:11 PM]


MyPasTest

Main Navigation
Question Browser: MRCP 1
Home
Subscriptions Question Browser Exam Builder Saved Exams
» MRCP 1 Session Progress
• Question Browser
Questions Correct 0
• Timed Test A 32-year-old woman presents with collapse. She works in
an office environment and it has been a particularly hot Questions Incorrect 0
• Mock Exam
day. On examination in casualty she looks a little Questions Total 0
• Past Papers dehydrated. BP is 110/70 mmHg.
Questions Percentage 0%
• Random Questions
The following serum electrolyte results are obtained; More
• My Performance
Na + 134mmol/l
• Media Bank
K + 2.9mmol/l
• New Multimedia Mg ++ 0.57 mmol/l (0.75-1.05)
Online Extras HCO3 - 34mmol/l Reference: Normal Values
Library
What is the most likely diagnosis? Click to
Community open/closeHaematology
Help Bartter's syndrome Click to
open/closeImmunoglobulins
PasTest Store Gitelman's syndrome
Click to
Gordon's syndrome open/closeBiochemistry
My Account Conn's syndrome Click to open/closeDiabetes
Click to
Profile Liddle's syndrome open/closeEndocrinology
Click to open/closeBlood
Newsletters 1231 gases Click to
My Career and Exams open/closeCSF
Order History
Learning Goals
Question Filters

Security

Change Password
Sign Out

© 2011 PasTest Ltd | About Us | Contact Us | Help

file:///E|/Shakhawan/Endocrinology/3.8.htm[3/13/2012 7:03:12 PM]


MyPasTest

Main Navigation
Question Browser: MRCP 1
Home
Subscriptions Question Browser Exam Builder Saved Exams
» MRCP 1 Session Progress
• Question Browser
Questions Correct 1
• Timed Test A 32-year-old woman presents with collapse. She works in
an office environment and it has been a particularly hot Questions Incorrect 0
• Mock Exam
day. On examination in casualty she looks a little Questions Total 1
• Past Papers dehydrated. BP is 110/70 mmHg.
Questions Percentage 100 %
• Random Questions
The following serum electrolyte results are obtained; More
• My Performance
Na + 134mmol/l
• Media Bank
K + 2.9mmol/l
• New Multimedia Mg ++ 0.57 mmol/l (0.75-1.05)
Online Extras HCO3 - 34mmol/l
Library
What is the most likely diagnosis?
Community
Help Bartter's syndrome
PasTest Store Gitelman's syndrome Your answer
Gordon's syndrome
My Account Conn's syndrome

Profile Liddle's syndrome

Newsletters
My Career and Exams
Order History Reference: Normal Values
Gitelman’s syndrome is due to a mutation in the
Learning Goals thiazide-sensitive Na–Cl transporter in the distal Click to open/closeHaematology
Question Filters convoluted tubule. It is associated with hypokalaemia, Click to open/closeImmunoglobulins
hypomagnesaemia (a cardinal feature) and raised Click to open/closeBiochemistry
serum bicarbonate levels. There is also hypocalciuria. Click to open/closeDiabetes Click to
Security Treatment is with potassium and magnesium open/closeEndocrinology Click to
replacement with or without potassium-sparing open/closeBlood gases Click to
Change Password diuretics. Differentiation between Gitelman's and open/closeCSF
Sign Out Bartter's syndromes is made by measuring the urinary
excretion of magnesium (which is high in Gitelman
syndrome and normal in Bartter syndrome) and calcium
(which is high in Bartter syndrome and low in Gitelman
syndrome). Bartter’s syndrome presents earlier with
hypokalaemic alkalosis and hypercalciuria, due to a
mutation in the bumetanide-sensitive Na + –K + -2Cl -
transporter. Treatment is with potassium replacement,
non-steroidal anti-inflammatories and sometimes ACE
inhibitors. Liddle’s syndrome is due to a mutation in the
distal nephron sodium channel, and is associated with
hypokalaemic alkalosis and low renin and aldosterone
levels, but hypertension still occurs. Essentially,
Gordon’s syndrome is the opposite of Bartter’s
syndrome, presenting with hyperkalaemia. Conn’s
syndrome is associated with hypertension and is due to
aldosterone excess.

1231

© 2011 PasTest Ltd | About Us | Contact Us | Help

file:///E|/Shakhawan/Endocrinology/3.8a.htm[3/13/2012 7:03:14 PM]


MyPasTest

Main Navigation
Question Browser: MRCP 1
Home
Subscriptions Question Browser Exam Builder Saved Exams
» MRCP 1 Session Progress
• Question Browser
Questions Correct 1
• Timed Test A 45-year-old woman who works in a pharmacy presents
with episodes of tiredness and lethargy. Her blood pressure Questions Incorrect 0
• Mock Exam
is 115/75 mmHg. Her bloods reveal hypokalaemia and a Questions Total 1
• Past Papers raised serum bicarbonate level. Urine collection reveals
Questions Percentage 100 %
• Random Questions hypercalciuria. Otherwise the findings are unremarkable.
• My Performance More
What is the likely diagnosis?
• Media Bank
• New Multimedia Bartter’s syndrome

Online Extras Gitelman’s syndrome


Reference: Normal Values
Library Frusemide abuse
Conn’s syndrome Click to
Community open/closeHaematology
Help Liddle’s syndrome Click to
open/closeImmunoglobulins
PasTest Store 1232 Click to
open/closeBiochemistry
My Account Click to open/closeDiabetes
Click to
Profile open/closeEndocrinology
Click to open/closeBlood
Newsletters gases Click to
My Career and Exams open/closeCSF
Order History
Learning Goals
Question Filters

Security

Change Password
Sign Out

© 2011 PasTest Ltd | About Us | Contact Us | Help

file:///E|/Shakhawan/Endocrinology/39.htm[3/13/2012 7:03:15 PM]


MyPasTest

Main Navigation
Question Browser: MRCP 1
Home
Subscriptions Question Browser Exam Builder Saved Exams
» MRCP 1 Session Progress
• Question Browser
Questions Correct 1
• Timed Test A 45-year-old woman who works in a pharmacy presents
with episodes of tiredness and lethargy. Her blood pressure Questions Incorrect 1
• Mock Exam
is 115/75 mmHg. Her bloods reveal hypokalaemia and a Questions Total 2
• Past Papers raised serum bicarbonate level. Urine collection reveals
Questions Percentage 50 %
• Random Questions hypercalciuria. Otherwise the findings are unremarkable.
• My Performance More
What is the likely diagnosis?
• Media Bank
• New Multimedia Bartter’s syndrome Your answer

Online Extras Gitelman’s syndrome

Library Frusemide abuse Correct answer

Community Conn’s syndrome

Help Liddle’s syndrome

PasTest Store

My Account
This picture could fit Bartter’s syndrome, although
Profile Bartter’s syndrome is rare (1 per million), has an
autosomal-recessive pattern of inheritance and
Newsletters commonly presents below the age of 5 years. Features
My Career and Exams of Bartter’s syndrome include volume depletion,
seizures, tetany and muscle weakness. This woman’s Reference: Normal Values
Order History
occupation is the clue, which unfortunately makes
Learning Goals frusemide abuse very much more likely. Click to open/closeHaematology
Question Filters Click to open/closeImmunoglobulins
Click to open/closeBiochemistry
1232 Click to open/closeDiabetes Click to
Security open/closeEndocrinology Click to
open/closeBlood gases Click to
Change Password open/closeCSF
Sign Out

© 2011 PasTest Ltd | About Us | Contact Us | Help

file:///E|/Shakhawan/Endocrinology/39a.htm[3/13/2012 7:03:16 PM]


MyPasTest

Main Navigation
Question Browser: MRCP 1
Home
Subscriptions Question Browser Exam Builder Saved Exams
» MRCP 1
• Question Browser
• Timed Test A 42-year-old man is referred to the hypertension clinic for advice. He is currently
• Mock Exam taking atenolol, bendrofluazide and ramipril and his blood pressure is currently
165/105 mmHg. His potassium is 3.0 mmol/l, with a serum bicarbonate concentration
• Past Papers of 28 mmol/l.
• Random Questions
What is the best next management step?
• My Performance
• Media Bank Measure the aldosterone:renin ratio
• New Multimedia Wash out as many of his antihypertensive agents as is possible for a period of 2
Online Extras weeks, then review

Library Measure his 24-h blood pressure

Community Arrange 24-h urinary free-cortisol collection

Help Add in a further agent and review in 12 months

PasTest Store 1233

My Account

Profile
Newsletters
My Career and Exams
Order History
Learning Goals
Question Filters

Security

Change Password
Sign Out

file:///E|/Shakhawan/Endocrinology/40.htm[3/13/2012 7:03:18 PM]


MyPasTest

Main Navigation
Question Browser: MRCP 1
Home
Subscriptions Question Browser Exam Builder Saved Exams
» MRCP 1 Session Progress
• Question Browser
Questions Correct 2
• Timed Test A 42-year-old man is referred to the hypertension clinic for
advice. He is currently taking atenolol, bendrofluazide and Questions Incorrect 1
• Mock Exam
ramipril and his blood pressure is currently 165/105 mmHg. Questions Total 3
• Past Papers His potassium is 3.0 mmol/l, with a serum bicarbonate
Questions Percentage 66 %
• Random Questions concentration of 28 mmol/l.
• My Performance More
What is the best next management step?
• Media Bank
• New Multimedia Measure the aldosterone:renin ratio

Online Extras Wash out as many of his Your answer


antihypertensive agents as is possible
Library for a period of 2 weeks, then review
Community Measure his 24-h blood pressure
Help Arrange 24-h urinary free-cortisol
PasTest Store collection
Add in a further agent and review in
12 months
My Account

Profile
Newsletters
My Career and Exams The suspicion here with hypokalaemia and metabolic
alkalosis, and resistant hypertension on three agents, is Reference: Normal Values
Order History
that he has primary hyperaldosteronism. ACE inhibitors
Learning Goals and angiotensin II-receptor blockers, diuretics, calcium- Click to open/closeHaematology
Question Filters channel blockers, b -blockers all ideally require a Click to open/closeImmunoglobulins
washout period of 2 weeks to make the Click to open/closeBiochemistry
aldosterone:renin ratio assay meaningful. Whilst it may Click to open/closeDiabetes Click to
Security seem dangerous to wash out, a definitive diagnosis is open/closeEndocrinology Click to
impossible if you don't. Spironolactone requires a open/closeBlood gases Click to
Change Password washout period of 6 weeks. A high aldosterone:renin open/closeCSF
ratio is suggestive of primary hyperaldosteronism. The
Sign Out
blood sample should be taken in the morning, standing,
and with a normalised potassium concentration (using
supplementation) if possible. Urinary potassium
excretion > 30 mmol/24 h may be another useful
adjunct in making the diagnosis. Surgery is the
treatment of choice for Conn’s adenoma and leads to
resolution of hypertension in around 70% of patients,
mitotane may be useful for controlling symptoms of
adrenal carcinoma, spironolactone is the medical
treatment of choice for adrenal hyperplasia.

1233

© 2011 PasTest Ltd | About Us | Contact Us | Help

file:///E|/Shakhawan/Endocrinology/40a.htm[3/13/2012 7:03:19 PM]


MyPasTest

Main Navigation
Question Browser: MRCP 1
Home
Subscriptions Question Browser Exam Builder Saved Exams
» MRCP 1 Session Progress
• Question Browser
Questions Correct 2
• Timed Test A 42-year-old man is referred to the hypertension clinic for
advice. He is currently taking atenolol 100 mg, Questions Incorrect 1
• Mock Exam
bendrofluazide 2.5 mg and ramipril 10 mg, and his blood Questions Total 3
• Past Papers pressure is currently 165/105 mmHg. Examination is
Questions Percentage 66 %
• Random Questions otherwise unremarkable. His potassium is 3.0 mmol/l, with
a serum bicarbonate concentration of 28 mmol/l, creatinine More
• My Performance
85 µmol /l, glucose tolerance is normal.
• Media Bank
What is the most likely underlying diagnosis?
• New Multimedia
Online Extras Cushing’s disease Reference: Normal Values
Library Primary hyperaldosteronism
Click to
Community Essential hypertension open/closeHaematology
Help Renal artery stenosis Click to
open/closeImmunoglobulins
PasTest Store Phaeochromocytoma Click to
open/closeBiochemistry
1234 Click to open/closeDiabetes
My Account
Click to
Profile open/closeEndocrinology
Click to open/closeBlood
Newsletters gases Click to
My Career and Exams open/closeCSF
Order History
Learning Goals
Question Filters

Security

Change Password
Sign Out

© 2011 PasTest Ltd | About Us | Contact Us | Help

file:///E|/Shakhawan/Endocrinology/41.htm[3/13/2012 7:03:21 PM]


MyPasTest

Main Navigation
Question Browser: MRCP 1
Home
Subscriptions Question Browser Exam Builder Saved Exams
» MRCP 1 Session Progress
• Question Browser
Questions Correct 3
• Timed Test A 42-year-old man is referred to the hypertension clinic for
advice. He is currently taking atenolol 100 mg, Questions Incorrect 1
• Mock Exam
bendrofluazide 2.5 mg and ramipril 10 mg, and his blood Questions Total 4
• Past Papers pressure is currently 165/105 mmHg. Examination is
Questions Percentage 75 %
• Random Questions otherwise unremarkable. His potassium is 3.0 mmol/l, with
a serum bicarbonate concentration of 28 mmol/l, creatinine More
• My Performance
85 µmol /l, glucose tolerance is normal.
• Media Bank
What is the most likely underlying diagnosis?
• New Multimedia
Online Extras Cushing’s disease
Library Primary hyperaldosteronism Your answer
Community Essential hypertension
Help Renal artery stenosis
PasTest Store Phaeochromocytoma

My Account

Profile
There is evidence of hypokalaemic metabolic alkalosis,
Newsletters probably associated with primary hyperaldosteronism.
My Career and Exams His resistant hypertension in the presence of marked
hypokalaemia supports this. Primary Reference: Normal Values
Order History
hyperaldosteronism accounts for at least 2% and
Learning Goals perhaps up to 10% of hypertensive patients. The Click to open/closeHaematology
Question Filters aldosterone:renin ratio is likely to be raised if the Click to open/closeImmunoglobulins
patient is off antihypertensive medication, as is urinary Click to open/closeBiochemistry
potassium excretion. It is important, however, to do the Click to open/closeDiabetes Click to
Security blood testing in the morning, after a period of standing open/closeEndocrinology Click to
and wash out of this man’s antihypertensives for a open/closeBlood gases Click to
Change Password period of 2 weeks if possible. Hyperaldosteronism does open/closeCSF
respond to some extent to spironolactone, but surgery
Sign Out
is the treatment of choice for Conn’s adenoma. Bilateral
adrenal hyperplasia, glucocorticoid-suppressible
hyperaldosteronism and adrenal carcinoma are other
causes.

1234

© 2011 PasTest Ltd | About Us | Contact Us | Help

file:///E|/Shakhawan/Endocrinology/41a.htm[3/13/2012 7:03:22 PM]


MyPasTest

Main Navigation
Question Browser: MRCP 1
Home
Subscriptions Question Browser Exam Builder Saved Exams
» MRCP 1 Session Progress
• Question Browser
Questions Correct 3
• Timed Test A 24-year-old student has been recovering at home after a
period of intensive care and general medical admission for Questions Incorrect 1
• Mock Exam
meningococcal septicaemia. 4 days after discharge from Questions Total 4
• Past Papers hospital she visits her GP complaining of dizziness on
Questions Percentage 75 %
• Random Questions standing and profound tiredness. On examination she looks
tired and ‘washed out’ and does indeed have postural More
• My Performance
hypotension. Blood testing reveals a sodium concentration
• Media Bank of 121 mmol/l, potassium of 6.7 mmol/l and urea of 15.0
• New Multimedia mmol/l. She has mild normochromic, normocytic anaemia.

Online Extras What is the most likely underlying diagnosis? Reference: Normal Values
Library
Secondary adrenal insufficiency Click to
Community open/closeHaematology
Waterhouse–Friderichsen syndrome
Help Click to
Myalgic encephalitis (ME) open/closeImmunoglobulins
PasTest Store
Syndrome of inappropriate ADH Click to
open/closeBiochemistry
Hypothyroidism Click to open/closeDiabetes
My Account
Click to
Profile 1235 open/closeEndocrinology
Click to open/closeBlood
Newsletters gases Click to
My Career and Exams open/closeCSF
Order History
Learning Goals
Question Filters

Security

Change Password
Sign Out

© 2011 PasTest Ltd | About Us | Contact Us | Help

file:///E|/Shakhawan/Endocrinology/42.htm[3/13/2012 7:03:24 PM]


MyPasTest

Main Navigation
Question Browser: MRCP 1
Home
Subscriptions Question Browser Exam Builder Saved Exams
» MRCP 1 Session Progress
• Question Browser
Questions Correct 4
• Timed Test A 24-year-old student has been recovering at home after a
period of intensive care and general medical admission for Questions Incorrect 1
• Mock Exam
meningococcal septicaemia. 4 days after discharge from Questions Total 5
• Past Papers hospital she visits her GP complaining of dizziness on
Questions Percentage 80 %
• Random Questions standing and profound tiredness. On examination she looks
tired and ‘washed out’ and does indeed have postural More
• My Performance
hypotension. Blood testing reveals a sodium concentration
• Media Bank of 121 mmol/l, potassium of 6.7 mmol/l and urea of 15.0
• New Multimedia mmol/l. She has mild normochromic, normocytic anaemia.

Online Extras What is the most likely underlying diagnosis?


Library
Secondary adrenal insufficiency
Community
Waterhouse–Friderichsen syndrome Your answer
Help
Myalgic encephalitis (ME)
PasTest Store
Syndrome of inappropriate ADH
Hypothyroidism
My Account

Profile
Newsletters
My Career and Exams Waterhouse–Friderichsen syndrome is bilateral adrenal
haemorrhage occurring due to massive septicaemia, Reference: Normal Values
Order History
often associated with severe, life-threatening
Learning Goals meningococcal disease. It may present as in this patient Click to open/closeHaematology
Question Filters with tiredness, lethargy and postural hypotension a Click to open/closeImmunoglobulins
short period after discharge from the precipitating Click to open/closeBiochemistry
illness. Classically, laboratory findings associated with Click to open/closeDiabetes Click to
Security adrenal insufficiency are hyponatraemia, hyperkalaemia, open/closeEndocrinology Click to
elevated urea, anaemia, elevated ESR, eosinophilia and open/closeBlood gases Click to
Change Password mild hypercalcaemia. The commonest cause in the open/closeCSF
western world is autoimmune disease (70%), closely
Sign Out
followed by malignancy, although TB is a common
association in the developing world. Emergency
treatment of adrenal insufficiency involves fluid
replacement, iv hydrocortisone and glucose
supplementation, often even prior to determining the
underlying cause.

1235

© 2011 PasTest Ltd | About Us | Contact Us | Help

file:///E|/Shakhawan/Endocrinology/42a.htm[3/13/2012 7:03:25 PM]


MyPasTest

Main Navigation
Question Browser: MRCP 1
Home
Subscriptions Question Browser Exam Builder Saved Exams
» MRCP 1 Session Progress
• Question Browser
Questions Correct 0
• Timed Test A 38-year-old psychiatric patient who is being treated for
depression is admitted with increased confusion. His sodium Questions Incorrect 0
• Mock Exam
is 122 mmol/l, with a plasma osmolality of 235 mOsmol/kg. Questions Total 0
• Past Papers Urine osmolality is 300mOsm/kg. There is no evidence on
Questions Percentage 0%
• Random Questions examination of cardiac, renal or hepatic failure. Random
plasma cortisol and thyroid function are normal. More
• My Performance
• Media Bank Given the most likely diagnosis, which of the
following statements best fits this condition?
• New Multimedia
Online Extras Reduced renal sodium excretion is likely Reference: Normal Values
Library Treatment with IV saline is likely to be necessary
Click to
Community Renal sodium excretion is likely to be elevated open/closeHaematology
Help Psychiatric drugs are very unlikely to be related to Click to
the underlying condition open/closeImmunoglobulins
PasTest Store
Click to
Hypothyroidism is a possible contributor open/closeBiochemistry
My Account Click to open/closeDiabetes
1236 Click to
Profile open/closeEndocrinology
Click to open/closeBlood
Newsletters gases Click to
My Career and Exams open/closeCSF
Order History
Learning Goals
Question Filters

Security

Change Password
Sign Out

© 2011 PasTest Ltd | About Us | Contact Us | Help

file:///E|/Shakhawan/Endocrinology/43.htm[3/13/2012 7:03:27 PM]


MyPasTest

Main Navigation
Question Browser: MRCP 1
Home
Subscriptions Question Browser Exam Builder Saved Exams
» MRCP 1 Session Progress
• Question Browser
Questions Correct 0
• Timed Test A 38-year-old psychiatric patient who is being treated for
depression is admitted with increased confusion. His sodium Questions Incorrect 1
• Mock Exam
is 122 mmol/l, with a plasma osmolality of 235 mOsmol/kg. Questions Total 1
• Past Papers Urine osmolality is 300mOsm/kg. There is no evidence on
Questions Percentage 0%
• Random Questions examination of cardiac, renal or hepatic failure. Random
plasma cortisol and thyroid function are normal. More
• My Performance
• Media Bank Given the most likely diagnosis, which of the
following statements best fits this condition?
• New Multimedia
Online Extras Reduced renal sodium excretion is
Library likely
Community Treatment with IV saline is likely to
be necessary
Help
Renal sodium excretion is likely to Correct answer
PasTest Store be elevated
Psychiatric drugs are very unlikely
My Account to be related to the underlying
condition
Profile
Hypothyroidism is a possible Your answer
Newsletters contributor
My Career and Exams
Order History Reference: Normal Values
Learning Goals
Click to open/closeHaematology
Question Filters This is the syndrome of inappropriate antidiuretic
Click to open/closeImmunoglobulins
hormone (SIADH) secretion. Criteria for diagnosis are
Click to open/closeBiochemistry
hyponatraemia with an osmolality of less than 270
Click to open/closeDiabetes Click to
Security mOsmol/kg. Inappropriately raised urine osmolality (>
open/closeEndocrinology Click to
100 mOsmol/l). High urinary sodium of > 20 mmol/l
open/closeBlood gases Click to
Change Password (reflecting relative increased concentration due to
open/closeCSF
reduced water loss), and normal renal, adrenal and
Sign Out
thyroid function. Sodium handling by the kidney is not
affected in SIADH. Psychiatric drugs, particularly
monoamine oxidase inhibitors and phenothiazines may
be possible causes. Other possible causes include
tumours, such as small-cell lung cancer, other chest
pathology, intracerebral haemorrhage or trauma, drugs
such as carbamazepine or chlorpropamide and
metabolic causes such as hypothyroidism. Treatment is
with fluid restriction or tetracycline-like compounds that
promote water excretion.

Another possible diagnostic consideration in a


presentation of hyponatraemia, especially given the
psychiatric aspect here, is that of psychogenic
polydipsia. However, in this condition urine is maximally
dilute (urine osmolality < 100mOsm/kg), in contrast to
SIADH.

1236

© 2011 PasTest Ltd | About Us | Contact Us | Help

file:///E|/Shakhawan/Endocrinology/43a.htm[3/13/2012 7:03:28 PM]


MyPasTest

Main Navigation
Question Browser: MRCP 1
Home
Subscriptions Question Browser Exam Builder Saved Exams
» MRCP 1 Session Progress
• Question Browser
Questions Correct 0
• Timed Test A 44-year-old patient with hypomania is referred for
opinion. She is noted to have a sodium concentration of Questions Incorrect 1
• Mock Exam
142 mmol/l, with a urea of 12 mmol/l and a creatinine of Questions Total 1
• Past Papers 140 µmol/l. Urine osmolality is 250 mOsmol/l. Fasting
Questions Percentage 0%
• Random Questions plasma glucose is normal. The nurses have monitored her
urine output and found it to be 4.4 litres in 24 h. More
• My Performance
• Media Bank Which of the following statements best fits with her
condition?
• New Multimedia
Online Extras Psychiatric drugs are unlikely to have played a part in Reference: Normal Values
Library her condition
Urine osmolality rising to > 375 mOsmol/l 4 h after Click to
Community open/closeHaematology
desmopressin in the water deprivation test is a positive
Help result for cranial diabetes insipidus, it is unlikely to rise Click to
in this case open/closeImmunoglobulins
PasTest Store
Click to
Nephrogenic diabetes insipidus usually has a open/closeBiochemistry
dominant pattern of inheritance Click to open/closeDiabetes
My Account
Desmopressin in addition to her usual drugs is likely Click to
Profile to be effective open/closeEndocrinology
Click to open/closeBlood
Newsletters Indometacin is of no value gases Click to
My Career and Exams open/closeCSF
1237
Order History
Learning Goals
Question Filters

Security

Change Password
Sign Out

© 2011 PasTest Ltd | About Us | Contact Us | Help

file:///E|/Shakhawan/Endocrinology/44.htm[3/13/2012 7:03:29 PM]


MyPasTest

Main Navigation
Question Browser: MRCP 1
Home
Subscriptions Question Browser Exam Builder Saved Exams
» MRCP 1 Session Progress
• Question Browser
Questions Correct 1
• Timed Test A 44-year-old patient with hypomania is referred for
opinion. She is noted to have a sodium concentration of Questions Incorrect 1
• Mock Exam
142 mmol/l, with a urea of 12 mmol/l and a creatinine of Questions Total 2
• Past Papers 140 µmol/l. Urine osmolality is 250 mOsmol/l. Fasting
Questions Percentage 50 %
• Random Questions plasma glucose is normal. The nurses have monitored her
urine output and found it to be 4.4 litres in 24 h. More
• My Performance
• Media Bank Which of the following statements best fits with her
condition?
• New Multimedia
Online Extras Psychiatric drugs are unlikely to have
Library played a part in her condition
Community Urine osmolality rising to > 375 Your answer
mOsmol/l 4 h after desmopressin in the
Help water deprivation test is a positive
PasTest Store result for cranial diabetes insipidus, it is
unlikely to rise in this case
Nephrogenic diabetes insipidus usually
My Account has a dominant pattern of inheritance
Profile Desmopressin in addition to her usual
Newsletters drugs is likely to be effective

My Career and Exams Indometacin is of no value

Order History Reference: Normal Values


Learning Goals
Click to open/closeHaematology
Question Filters Click to open/closeImmunoglobulins
This patient has diabetes insipidus, probably renal in Click to open/closeBiochemistry
origin related to the use of lithium as treatment for Click to open/closeDiabetes Click to
Security manic depression. Decompensated diabetes insipidus open/closeEndocrinology Click to
has features of dehydration (as in this case), with an open/closeBlood gases Click to
Change Password inappropriately low urine osmolality. Obviously diabetes open/closeCSF
mellitus needs excluding as a cause of polyuria before a
Sign Out
diagnosis of diabetes insipidus can be made.
Psychogenic polydipsia is ruled out by the water
deprivation test, with urine osmolality rising
appropriately in response to the lack of oral intake. The
urine remains dilute in cases of nephrogenic diabetes
insipidus, despite a rising plasma osmolality and urine
failing to concentrate after desmopressin. Desmopressin
is unlikely to be effective in this case, indometacin may
be effective. However, if possible, the lithium should be
withdrawn in consultation with a psychiatrist, and
another drug such as lamotrigine substituted.
Nephrogenic diabetes insipidus has an autosomal or X-
linked recessive pattern of inheritance, whereas cranial
diabetes insipidus is inherited only with an autosomal-
dominant pattern.

1237

© 2011 PasTest Ltd | About Us | Contact Us | Help

file:///E|/Shakhawan/Endocrinology/44a.htm[3/13/2012 7:03:31 PM]


MyPasTest

Main Navigation
Question Browser: MRCP 1
Home
Subscriptions Question Browser Exam Builder Saved Exams
» MRCP 1 Session Progress
• Question Browser
Questions Correct 0
• Timed Test A 62-year-old man presents with bony pain that has been
present for some months, particularly affecting his left Questions Incorrect 0
• Mock Exam
femur, pelvis and lower back. Blood testing reveals a Questions Total 0
• Past Papers normal serum calcium level, but a raised alkaline
Questions Percentage 0%
• Random Questions phosphatase. X-rays of the femur and pelvis reveal mixed
lytic and sclerotic change, with accentuated trabecular More
• My Performance
markings. Chest X-ray is normal.
• Media Bank
What is the likely diagnosis?
• New Multimedia
Online Extras Secondary carcinoma Reference: Normal Values
Library Multiple myeloma
Click to
Community Hyperparathyroidism open/closeHaematology
Help Hypoparathyroidism Click to
open/closeImmunoglobulins
PasTest Store Paget’s disease Click to
open/closeBiochemistry
1238 Click to open/closeDiabetes
My Account
Click to
Profile open/closeEndocrinology
Click to open/closeBlood
Newsletters gases Click to
My Career and Exams open/closeCSF
Order History
Learning Goals
Question Filters

Security

Change Password
Sign Out

© 2011 PasTest Ltd | About Us | Contact Us | Help

file:///E|/Shakhawan/Endocrinology/45.htm[3/13/2012 7:03:32 PM]


MyPasTest

Main Navigation
Question Browser: MRCP 1
Home
Subscriptions Question Browser Exam Builder Saved Exams
» MRCP 1 Session Progress
• Question Browser
Questions Correct 1
• Timed Test A 62-year-old man presents with bony pain that has been
present for some months, particularly affecting his left Questions Incorrect 0
• Mock Exam
femur, pelvis and lower back. Blood testing reveals a Questions Total 1
• Past Papers normal serum calcium level, but a raised alkaline
Questions Percentage 100 %
• Random Questions phosphatase. X-rays of the femur and pelvis reveal mixed
lytic and sclerotic change, with accentuated trabecular More
• My Performance
markings. Chest X-ray is normal.
• Media Bank
What is the likely diagnosis?
• New Multimedia
Online Extras Secondary carcinoma
Library Multiple myeloma
Community Hyperparathyroidism
Help Hypoparathyroidism
PasTest Store Paget’s disease Your answer

My Account

Profile
Paget’s disease is thought to be present in 2% of the
Newsletters population above 55 years of age, with 90% being
My Career and Exams asymptomatic. It is more common in Caucasian
populations and rarer in populations of African descent. Reference: Normal Values
Order History
There are said to be three phases associated with the
Learning Goals time course of radiological changes seen in Paget’s Click to open/closeHaematology
Question Filters disease. Early disease is said to be primarily lytic, then Click to open/closeImmunoglobulins
there are mixed lytic and sclerotic changes, progressing Click to open/closeBiochemistry
later to primarily sclerotic change with increasing bony Click to open/closeDiabetes Click to
Security thickening. Goals of treatment are to normalise bone open/closeEndocrinology Click to
turnover, maintain the alkaline phosphatase level within open/closeBlood gases Click to
Change Password the normal range, minimise symptoms and prevent open/closeCSF
long-term complications. The mainstay of treatment for
Sign Out
this has been use of the bisphosphonates. These are
now often given as intermittent iv courses a few weeks
apart. Long-term complications include deafness (in up
to 50% of patients with skull-base Paget’s disease),
and vary rarely osteogenic sarcoma.

1238

© 2011 PasTest Ltd | About Us | Contact Us | Help

file:///E|/Shakhawan/Endocrinology/45a.htm[3/13/2012 7:03:34 PM]


MyPasTest

Main Navigation
Question Browser: MRCP 1
Home
Subscriptions Question Browser Exam Builder Saved Exams
» MRCP 1 Session Progress
• Question Browser
Questions Correct 1
• Timed Test A 32-year-old woman presents with amenorrhoea for 6
months. A pregnancy test is negative. Over the past few Questions Incorrect 0
• Mock Exam
months she has occasionally been leaking milk, and Questions Total 1
• Past Papers presents now as this has occurred more and more during
Questions Percentage 100 %
• Random Questions stimulation and intercourse and she is becoming distressed
by it. Thyroid function testing is normal. She is on no More
• My Performance
medication. Her serum prolactin level is 2400 mU/l and a
• Media Bank CT scan of the pituitary is unremarkable.
• New Multimedia
Which of the following best fits her condition?
Online Extras Reference: Normal Values
Library She is likely to have a macroprolactinoma
She should be observed for 12 months Click to
Community open/closeHaematology
Help Cabergoline is effective therapy Click to
Surgery is the best option here open/closeImmunoglobulins
PasTest Store
Click to
A visual field defect is likely open/closeBiochemistry
My Account Click to open/closeDiabetes
1239 Click to
Profile open/closeEndocrinology
Click to open/closeBlood
Newsletters gases Click to
My Career and Exams open/closeCSF
Order History
Learning Goals
Question Filters

Security

Change Password
Sign Out

© 2011 PasTest Ltd | About Us | Contact Us | Help

file:///E|/Shakhawan/Endocrinology/46.htm[3/13/2012 7:03:35 PM]


MyPasTest

Main Navigation
Question Browser: MRCP 1
Home
Subscriptions Question Browser Exam Builder Saved Exams
» MRCP 1 Session Progress
• Question Browser
Questions Correct 2
• Timed Test A 32-year-old woman presents with amenorrhoea for 6
months. A pregnancy test is negative. Over the past few Questions Incorrect 0
• Mock Exam
months she has occasionally been leaking milk, and Questions Total 2
• Past Papers presents now as this has occurred more and more during
Questions Percentage 100 %
• Random Questions stimulation and intercourse and she is becoming distressed
by it. Thyroid function testing is normal. She is on no More
• My Performance
medication. Her serum prolactin level is 2400 mU/l and a
• Media Bank CT scan of the pituitary is unremarkable.
• New Multimedia
Which of the following best fits her condition?
Online Extras
Library She is likely to have a
macroprolactinoma
Community
She should be observed for 12 months
Help
Cabergoline is effective therapy Your answer
PasTest Store
Surgery is the best option here
A visual field defect is likely
My Account

Profile
Newsletters
My Career and Exams She most likely has a microprolactinoma, which would
appear as a hypodense area on MRI scanning. Serum Reference: Normal Values
Order History
prolactin is often in the range 1500 mU/l to 3000 mU/l
Learning Goals in patients with microadenomas; levels are usually Click to open/closeHaematology
Question Filters above 3000 mU/l in those with macroadenomas. Click to open/closeImmunoglobulins
Although surgery in conjunction with dopamine-agonist Click to open/closeBiochemistry
therapy is the treatment of choice for patients with Click to open/closeDiabetes Click to
Security macroadenomas, microadenomas often respond well to open/closeEndocrinology Click to
cabergoline (83% normalisation of prolactin). A visual open/closeBlood gases Click to
Change Password field defect is unlikely. Observation in these patients is open/closeCSF
not usually preferable to normalisation of prolactin
Sign Out
levels with dopamine agonists, if this observational
approach is taken then adequate sex-hormone
replacement should be undertaken. Of course,
normalisation of prolactin may restore fertility, and the
patient should be warned of this possibility.

1239

© 2011 PasTest Ltd | About Us | Contact Us | Help

file:///E|/Shakhawan/Endocrinology/46a.htm[3/13/2012 7:03:36 PM]


MyPasTest

Main Navigation
Question Browser: MRCP 1
Home
Subscriptions Question Browser Exam Builder Saved Exams
» MRCP 1
• Question Browser
• Timed Test A 55-year-old woman presents with her husband to the endocrine clinic. She has
• Mock Exam distressing symptoms of sweating, and her husband noticed increased prominence of
her jaw when he was archiving photos from recent years. Last year she was diagnosed
• Past Papers with type-2 diabetes. Other past history of note is that she has recently been
• Random Questions operated on for carpal tunnel syndrome.
• My Performance Which of the following most likely to fit with her condition?
• Media Bank
• New Multimedia Random growth-hormone level is likely to be < 1 mU/l

Online Extras IGF-1 levels are likely to be normal

Library Growth-hormone levels are likely to remain above 2 mU/l after a 75-g glucose
load
Community
1-25-OH vitamin D level is invariably normal
Help
Hypertension coexists in 20% of patients with this condition
PasTest Store
1240
My Account

Profile
Newsletters
My Career and Exams
Order History
Learning Goals
Question Filters

Security

Change Password
Sign Out

file:///E|/Shakhawan/Endocrinology/47.htm[3/13/2012 7:03:38 PM]


MyPasTest

Main Navigation
Question Browser: MRCP 1
Home
Subscriptions Question Browser Exam Builder Saved Exams
» MRCP 1 Session Progress
• Question Browser
Questions Correct 3
• Timed Test A 55-year-old woman presents with her husband to the
endocrine clinic. She has distressing symptoms of sweating, Questions Incorrect 0
• Mock Exam
and her husband noticed increased prominence of her jaw Questions Total 3
• Past Papers when he was archiving photos from recent years. Last year
Questions Percentage 100 %
• Random Questions she was diagnosed with type-2 diabetes. Other past history
of note is that she has recently been operated on for carpal More
• My Performance
tunnel syndrome.
• Media Bank
Which of the following most likely to fit with her
• New Multimedia
condition?
Online Extras
Library Random growth-hormone level is likely
to be < 1 mU/l
Community
IGF-1 levels are likely to be normal
Help
Growth-hormone levels are likely to Your answer
PasTest Store remain above 2 mU/l after a 75-g
glucose load
My Account 1-25-OH vitamin D level is invariably
normal
Profile
Hypertension coexists in 20% of
Newsletters patients with this condition
My Career and Exams
Order History Reference: Normal Values
Learning Goals
Click to open/closeHaematology
Question Filters The diagnosis here is acromegaly. A random growth- Click to open/closeImmunoglobulins
hormone level of < 1 mU/l excludes the diagnosis, Click to open/closeBiochemistry
growth-hormone levels usually remain above 2 mU/l Click to open/closeDiabetes Click to
Security after an oral glucose tolerance test. There are increased open/closeEndocrinology Click to
levels of 1,25-OH vitamin D in some patients. IGF-1 is open/closeBlood gases Click to
Change Password invariably elevated in patients with acromegaly. open/closeCSF
Sweating is present in more than 80% of cases,
Sign Out
hypertension in 40%, impaired glucose tolerance in
40% and frank type-2 diabetes in 20%. Coarse facial
features including prognathism are often noticed by
patients on comparing old photographs, enlargement of
the hands and feet and soft tissue swelling are also
common features. Growth hormone-secreting pituitary
tumours are usually identified on MRI scanning and
trans-sphenoidal resection is the treatment of choice.

1240

© 2011 PasTest Ltd | About Us | Contact Us | Help

file:///E|/Shakhawan/Endocrinology/47a.htm[3/13/2012 7:03:39 PM]


MyPasTest

Main Navigation
Question Browser: MRCP 1
Home
Subscriptions Question Browser Exam Builder Saved Exams
» MRCP 1 Session Progress
• Question Browser
Questions Correct 3
• Timed Test A 32-year-old woman presents with extreme lethargy a
couple of weeks after the birth of her third child by Questions Incorrect 0
• Mock Exam
emergency caesarean section. The reason for the caesarean Questions Total 3
• Past Papers section was severe blood loss and hypotension. She
Questions Percentage 100 %
• Random Questions complained to the health visitor of increasing problems
some 7 days earlier, but was told that this was to be More
• My Performance
expected after the birth of her child. On admission via
• Media Bank casualty she was noted to have a sodium concentration of
• New Multimedia 127 mmol/l, a potassium concentration of 5.0 mmol/l and a
urea of 12 mmol/l.
Online Extras Reference: Normal Values
Library What is the likely diagnosis?
Click to
Community Sheehan’s syndrome open/closeHaematology
Help Hypothyroidism Click to
open/closeImmunoglobulins
PasTest Store Primary adrenal failure Click to
Postnatal depression open/closeBiochemistry
My Account Click to open/closeDiabetes
Dehydration Click to
Profile open/closeEndocrinology
1241 Click to open/closeBlood
Newsletters gases Click to
My Career and Exams open/closeCSF
Order History
Learning Goals
Question Filters

Security

Change Password
Sign Out

© 2011 PasTest Ltd | About Us | Contact Us | Help

file:///E|/Shakhawan/Endocrinology/48.htm[3/13/2012 7:03:40 PM]


MyPasTest

Main Navigation
Question Browser: MRCP 1
Home
Subscriptions Question Browser Exam Builder Saved Exams
» MRCP 1 Session Progress
• Question Browser
Questions Correct 4
• Timed Test A 32-year-old woman presents with extreme lethargy a
couple of weeks after the birth of her third child by Questions Incorrect 0
• Mock Exam
emergency caesarean section. The reason for the caesarean Questions Total 4
• Past Papers section was severe blood loss and hypotension. She
Questions Percentage 100 %
• Random Questions complained to the health visitor of increasing problems
some 7 days earlier, but was told that this was to be More
• My Performance
expected after the birth of her child. On admission via
• Media Bank casualty she was noted to have a sodium concentration of
• New Multimedia 127 mmol/l, a potassium concentration of 5.0 mmol/l and a
urea of 12 mmol/l.
Online Extras
Library What is the likely diagnosis?

Community Sheehan’s syndrome Your answer


Help Hypothyroidism
PasTest Store Primary adrenal failure
Postnatal depression
My Account Dehydration
Profile
Newsletters
My Career and Exams
She has suffered a period of hypotension and blood loss Reference: Normal Values
Order History
associated with her emergency caesarean section. This
Learning Goals has resulted in pituitary infarction, and she presents Click to open/closeHaematology
Question Filters now with symptoms of hypoadrenalism, which include Click to open/closeImmunoglobulins
hyponatraemia and a potassium at the upper limit of Click to open/closeBiochemistry
normal. She, of course, requires fluid rehydration and Click to open/closeDiabetes Click to
Security emergency steroid replacement with iv hydrocortisone. open/closeEndocrinology Click to
She will also have pituitary-dependent hypothyroidism open/closeBlood gases Click to
Change Password and require thyroxine replacement. Restoration of open/closeCSF
fertility is more difficult, pulsed delivery of pituitary sex-
Sign Out
axis hormones is usually required. Postnatal depression
and simple dehydration are somewhat unlikely with this
set of blood results.

1241

© 2011 PasTest Ltd | About Us | Contact Us | Help

file:///E|/Shakhawan/Endocrinology/48a.htm[3/13/2012 7:03:42 PM]


MyPasTest

Main Navigation
Question Browser: MRCP 1
Home
Subscriptions Question Browser Exam Builder Saved Exams
» MRCP 1
• Question Browser
• Timed Test A 25-year-old overweight woman presents with hirsutism and oligomenorrhoea. She
• Mock Exam has been unable to conceive for 18 months. The adrenals appear normal on
ultrasound scanning, but an ovarian ultrasound scan reveals numerous small cysts in
• Past Papers both ovaries.
• Random Questions
Which of the following is likely to fit best with her diagnosis?
• My Performance
• Media Bank The LH/FSH ratio is likely to be normal
• New Multimedia Sex hormone-binding globulin is low in 50% of sufferers with this condition
Online Extras Testosterone levels are usually normal
Library Fertility is usually unaffected by this condition
Community DHEAS is usually normal or low
Help
1242
PasTest Store

My Account

Profile
Newsletters
My Career and Exams
Order History
Learning Goals
Question Filters

Security

Change Password
Sign Out

file:///E|/Shakhawan/Endocrinology/49.htm[3/13/2012 7:03:43 PM]


MyPasTest

Main Navigation
Question Browser: MRCP 1
Home
Subscriptions Question Browser Exam Builder Saved Exams
» MRCP 1 Session Progress
• Question Browser
Questions Correct 5
• Timed Test A 25-year-old overweight woman presents with hirsutism
and oligomenorrhoea. She has been unable to conceive for Questions Incorrect 0
• Mock Exam
18 months. The adrenals appear normal on ultrasound Questions Total 5
• Past Papers scanning, but an ovarian ultrasound scan reveals numerous
Questions Percentage 100 %
• Random Questions small cysts in both ovaries.
• My Performance More
Which of the following is likely to fit best with her
• Media Bank diagnosis?
• New Multimedia
The LH/FSH ratio is likely to be normal
Online Extras
Sex hormone-binding globulin is low in Your answer
Library 50% of sufferers with this condition
Community Testosterone levels are usually normal
Help Fertility is usually unaffected by this
PasTest Store condition
DHEAS is usually normal or low

My Account

Profile
Newsletters This is polycystic ovarian syndrome (PCOS). The
My Career and Exams prevalence of PCOS is estimated to range from 8 to
22% of women. Oligo/amenorrhoea is present in 70%, Reference: Normal Values
Order History
hirsutism in 60%, obesity in 35% and infertility in 30%.
Learning Goals LH is usually markedly raised, as is the LH/FSH ratio. Click to open/closeHaematology
Question Filters Sex hormone-binding globulin is low in 50% of Click to open/closeImmunoglobulins
sufferers. due primarily to hyperinsulinaemia. Click to open/closeBiochemistry
Testosterone and DHEAS (dehydroepiandrosterone) Click to open/closeDiabetes Click to
Security levels are also usually raised. Up to 40% of women with open/closeEndocrinology Click to
PCOS may have impaired glucose tolerance, and up to open/closeBlood gases Click to
Change Password 10% frank type-2 diabetes mellitus. The insulin- open/closeCSF
resistant state is also associated with dyslipidaemia.
Sign Out
Management is usually with weight loss and lifestyle
advice. Metformin is a useful treatment for insulin
resistance and may help restore fertility, although
clomifene may also be required for ovulation induction.
Local creams and electrolysis may have a useful role to
play if the primary complaint is one of hirsutism.

1242

© 2011 PasTest Ltd | About Us | Contact Us | Help

file:///E|/Shakhawan/Endocrinology/49a.htm[3/13/2012 7:03:44 PM]


MyPasTest

Main Navigation
Question Browser: MRCP 1
Home
Subscriptions Question Browser Exam Builder Saved Exams
» MRCP 1 Session Progress
• Question Browser
Questions Correct 5
• Timed Test A 38-year-old woman presents to the clinic with difficult to
treat hypertension. She is on two agents and currently has Questions Incorrect 0
• Mock Exam
a BP of 155/95 mmHg. She has noted that her face has Questions Total 5
• Past Papers become more rounded over the years and she is having
Questions Percentage 100 %
• Random Questions increasing trouble with both acne and hirsutism. Fasting
blood glucose testing has revealed impaired glucose More
• My Performance
tolerance. There has also been increasing trouble with
• Media Bank abdominal obesity and she has noticed some purple
• New Multimedia stretchmarks appearing around her abdomen.

Online Extras What is the most likely diagnosis? Reference: Normal Values
Library
Phaeochromocytoma Click to
Community open/closeHaematology
Multiple endocrine neoplasia
Help Click to
Essential hypertension open/closeImmunoglobulins
PasTest Store
Simple obesity Click to
open/closeBiochemistry
Cushing’s syndrome Click to open/closeDiabetes
My Account
Click to
Profile 1243 open/closeEndocrinology
Click to open/closeBlood
Newsletters gases Click to
My Career and Exams open/closeCSF
Order History
Learning Goals
Question Filters

Security

Change Password
Sign Out

© 2011 PasTest Ltd | About Us | Contact Us | Help

file:///E|/Shakhawan/Endocrinology/50.htm[3/13/2012 7:03:46 PM]


MyPasTest

Main Navigation
Question Browser: MRCP 1
Home
Subscriptions Question Browser Exam Builder Saved Exams
» MRCP 1 Session Progress
• Question Browser
Questions Correct 6
• Timed Test A 38-year-old woman presents to the clinic with difficult to
treat hypertension. She is on two agents and currently has Questions Incorrect 0
• Mock Exam
a BP of 155/95 mmHg. She has noted that her face has Questions Total 6
• Past Papers become more rounded over the years and she is having
Questions Percentage 100 %
• Random Questions increasing trouble with both acne and hirsutism. Fasting
blood glucose testing has revealed impaired glucose More
• My Performance
tolerance. There has also been increasing trouble with
• Media Bank abdominal obesity and she has noticed some purple
• New Multimedia stretchmarks appearing around her abdomen.

Online Extras What is the most likely diagnosis?


Library
Phaeochromocytoma
Community
Multiple endocrine neoplasia
Help
Essential hypertension
PasTest Store
Simple obesity
Cushing’s syndrome Your answer
My Account

Profile
Newsletters
My Career and Exams Cushing’s syndrome is associated with a round,
plethoric, facial appearance and weight gain – in Reference: Normal Values
Order History
particular truncal obesity, buffalo hump and
Learning Goals supraclavicular fat pads, skin fragility, proximal muscle Click to open/closeHaematology
Question Filters weakness, mood disturbance, menstrual disturbance Click to open/closeImmunoglobulins
and reduced libido. Hypertension is present in more Click to open/closeBiochemistry
than 50% of patients, impaired glucose tolerance in Click to open/closeDiabetes Click to
Security 30%. Osteopenia and osteoporosis, and premature open/closeEndocrinology Click to
vascular disease are an inevitable consequence in open/closeBlood gases Click to
Change Password untreated Cushing’s. The annual incidence is open/closeCSF
approximately 2 per million and the disease is
Sign Out
commoner in women. The disease results from
hypercortisolaemia and the cause in 68% of cases is a
pituitary adenoma producing ACTH. Ectopic ACTH
production is the cause in 12%, adrenal adenoma in
10% and adrenal carcinoma in 8%. Diagnosis is made
on the results of the 24-h urinary free-cortisol assay.
Differentiation as to the cause is carried out with the
dexamethasone-suppression test and selective venous
sampling in cases when a discrete tumour cannot be
identified on contrast-CT scanning.

1243

© 2011 PasTest Ltd | About Us | Contact Us | Help

file:///E|/Shakhawan/Endocrinology/50a.htm[3/13/2012 7:03:47 PM]


MyPasTest

Main Navigation
Question Browser: MRCP 1
Home
Subscriptions Question Browser Exam Builder Saved Exams
» MRCP 1 Session Progress
• Question Browser
Questions Correct 6
• Timed Test A 24-year-old woman presents with 13 months of
amenorrhoea. For the past few months she has been Questions Incorrect 0
• Mock Exam
experiencing hot flushes, night sweats, mood changes and Questions Total 6
• Past Papers pain on intercourse. FSH has been > 40 mIU/l on two
Questions Percentage 100 %
• Random Questions separate occasions, and her serum estradiol level is low.
TSH is normal. Fasting blood glucose is normal. Pregnancy More
• My Performance
test is negative.
• Media Bank What is the most likely diagnosis?
• New Multimedia
Premature ovarian failure
Online Extras Reference: Normal Values
Polycystic ovarian syndrome (PCOS)
Library
Androgen-secreting adrenal tumour Click to
Community open/closeHaematology
Pituitary failure
Help Click to
Thyrotoxicosis open/closeImmunoglobulins
PasTest Store
Click to
1244 open/closeBiochemistry
My Account Click to open/closeDiabetes
Click to
Profile open/closeEndocrinology
Click to open/closeBlood
Newsletters gases Click to
My Career and Exams open/closeCSF
Order History
Learning Goals
Question Filters

Security

Change Password
Sign Out

© 2011 PasTest Ltd | About Us | Contact Us | Help

file:///E|/Shakhawan/Endocrinology/51.htm[3/13/2012 7:03:49 PM]


MyPasTest

Main Navigation
Question Browser: MRCP 1
Home
Subscriptions Question Browser Exam Builder Saved Exams
» MRCP 1 Session Progress
• Question Browser
Questions Correct 1
• Timed Test A 24-year-old woman presents with 13 months of
amenorrhoea. For the past few months she has been Questions Incorrect 0
• Mock Exam
experiencing hot flushes, night sweats, mood changes and Questions Total 1
• Past Papers pain on intercourse. FSH has been > 40 mIU/l on two
Questions Percentage 100 %
• Random Questions separate occasions, and her serum estradiol level is low.
TSH is normal. Fasting blood glucose is normal. Pregnancy More
• My Performance
test is negative.
• Media Bank What is the most likely diagnosis?
• New Multimedia
Premature ovarian failure Your answer
Online Extras
Polycystic ovarian syndrome (PCOS)
Library
Androgen-secreting adrenal tumour
Community
Pituitary failure
Help
Thyrotoxicosis
PasTest Store

My Account

Profile The blood picture of elevated FSH, low estradiol and


prolonged amenorrhoea fits the picture. The normal
Newsletters
TSH rules out thyrotoxicosis, and the raised FSH makes
My Career and Exams PCOS unlikely and this does not fit the picture of an
Order History androgen-secreting adrenal tumour. Autoimmune Reference: Normal Values
disease is responsible for 20% of cases of premature
Learning Goals ovarian failure, and is prevalent in 10% of women with Click to open/closeHaematology
Question Filters Addison’s disease and 25% of women with autoimmune Click to open/closeImmunoglobulins
thyroid disease. Spontaneous recovery of fertility is Click to open/closeBiochemistry
unlikely, and put at only 5%. Click to open/closeDiabetes Click to
Security open/closeEndocrinology Click to
open/closeBlood gases Click to
Change Password 1244 open/closeCSF
Sign Out

© 2011 PasTest Ltd | About Us | Contact Us | Help

file:///E|/Shakhawan/Endocrinology/51a.htm[3/13/2012 7:03:50 PM]


MyPasTest

Main Navigation
Question Browser: MRCP 1
Home
Subscriptions Question Browser Exam Builder Saved Exams
» MRCP 1 Session Progress
• Question Browser
Questions Correct 1
• Timed Test A 44-year-old woman has attended A&E on a number of
occasions this year because of renal tract stones. She has Questions Incorrect 0
• Mock Exam
also suffered depression during the past year or two. She is Questions Total 1
• Past Papers found to have a serum calcium of 3.10 mmol/l (2.4–2.6),
Questions Percentage 100 %
• Random Questions creatinine of 138 µmol/l and albumin of 40 g/l.
• My Performance More
What is her most likely underlying diagnosis?
• Media Bank
• New Multimedia Hyperparathyroidism

Online Extras Familial hypercalcaemic hypocalciuria


Reference: Normal Values
Library Multiple endocrine neoplasia
Hypoparathyroidism Click to
Community open/closeHaematology
Help Pseudohypoparathyroidism Click to
open/closeImmunoglobulins
PasTest Store 1245 Click to
open/closeBiochemistry
My Account Click to open/closeDiabetes
Click to
Profile open/closeEndocrinology
Click to open/closeBlood
Newsletters gases Click to
My Career and Exams open/closeCSF
Order History
Learning Goals
Question Filters

Security

Change Password
Sign Out

© 2011 PasTest Ltd | About Us | Contact Us | Help

file:///E|/Shakhawan/Endocrinology/52.htm[3/13/2012 7:03:52 PM]


MyPasTest

Main Navigation
Question Browser: MRCP 1
Home
Subscriptions Question Browser Exam Builder Saved Exams
» MRCP 1
• Question Browser
• Timed Test A 44-year-old woman has attended A&E on a number of occasions this year because
• Mock Exam of renal tract stones. She has also suffered depression during the past year or two.
She is found to have a serum calcium of 3.10 mmol/l (2.4–2.6), creatinine of 138
• Past Papers µmol/l and albumin of 40 g/l.
• Random Questions
What is her most likely underlying diagnosis?
• My Performance
• Media Bank Hyperparathyroidism Your answer
• New Multimedia Familial hypercalcaemic hypocalciuria
Online Extras Multiple endocrine neoplasia
Library Hypoparathyroidism
Community Pseudohypoparathyroidism
Help
PasTest Store

My Account This woman most likely has hyperparathyroidism. Hyperparathyroidism is a feature


in 95% of patients with MEN-1 (multiple endocrine neoplasia – type 1) and may
Profile coexist with prolactin- or growth hormone-producing pituitary adenomas,
pancreatic islet cell tumours, non-functioning adrenal adenomas or thyroid
Newsletters adenomas. As part of the MEN-2 syndrome, there is an association with
My Career and Exams phaeochromocytoma, Cushing’s syndrome and medullary carcinoma of the thyroid.
Short metacarpals (usually the fourth or fifth) are associated with
Order History
pseudohypoparathyroidism, which, in turn, is associated with hypocalcaemia due to
Learning Goals an abnormality of the G-protein receptor for parathyroid hormone. In the case of
Question Filters parathyroid hyperplasia or parathyroid adenoma, a surgical solution is the best
option where symptoms such as renal stones have begun to appear.

Security
1245
Change Password
Sign Out

file:///E|/Shakhawan/Endocrinology/52a.htm[3/13/2012 7:03:53 PM]


MyPasTest

Main Navigation
Question Browser: MRCP 1
Home
Subscriptions Question Browser Exam Builder Saved Exams
» MRCP 1 Session Progress
• Question Browser
Questions Correct 2
• Timed Test A 40-year-old man presents with a serum calcium
concentration of 3.05 mmol/l, and urinary calcium Questions Incorrect 0
• Mock Exam
excretion of 12 mg/24 h (100-250 mg/24hr). There is no Questions Total 2
• Past Papers history of renal stones, pancreatitis, depression or any prior
Questions Percentage 100 %
• Random Questions illness. He was referred by a particularly zealous GP and
underwent parathyroidectomy. More
• My Performance
• Media Bank Which of the following is true of the disease which
underlies this man's case history?
• New Multimedia
Online Extras He has hyperparathyroidism that is likely to have Reference: Normal Values
Library been cured by the surgery
Urinary calcium excretion is increased Click to
Community open/closeHaematology
Help Inheritance is autosomal dominant Click to
Acute pancreatitis is commonly associated with the open/closeImmunoglobulins
PasTest Store
condition Click to
open/closeBiochemistry
The body has an increased ability to sense raised Click to open/closeDiabetes
My Account calcium levels Click to
Profile open/closeEndocrinology
1246 Click to open/closeBlood
Newsletters gases Click to
My Career and Exams open/closeCSF
Order History
Learning Goals
Question Filters

Security

Change Password
Sign Out

© 2011 PasTest Ltd | About Us | Contact Us | Help

file:///E|/Shakhawan/Endocrinology/53.htm[3/13/2012 7:03:54 PM]


MyPasTest

Main Navigation
Question Browser: MRCP 1
Home
Subscriptions Question Browser Exam Builder Saved Exams
» MRCP 1 Session Progress
• Question Browser
Questions Correct 3
• Timed Test A 40-year-old man presents with a serum calcium
concentration of 3.05 mmol/l, and urinary calcium Questions Incorrect 0
• Mock Exam
excretion of 12 mg/24 h (100-250 mg/24hr). There is no Questions Total 3
• Past Papers history of renal stones, pancreatitis, depression or any prior
Questions Percentage 100 %
• Random Questions illness. He was referred by a particularly zealous GP and
underwent parathyroidectomy. More
• My Performance
• Media Bank Which of the following is true of the disease which
underlies this man's case history?
• New Multimedia
Online Extras He has hyperparathyroidism that is
Library likely to have been cured by the surgery
Community Urinary calcium excretion is increased
Help Inheritance is autosomal dominant Your answer
PasTest Store Acute pancreatitis is commonly
associated with the condition
The body has an increased ability to
My Account sense raised calcium levels
Profile
Newsletters
My Career and Exams
This is not hyperparathyroidism. This is familial Reference: Normal Values
Order History
hypocalciuric hypercalcaemia. It has an autosomal-
Learning Goals dominant pattern of inheritance with virtually complete Click to open/closeHaematology
Question Filters penetrance. The heterozygotic state is commonly Click to open/closeImmunoglobulins
asymptomatic, but homozygotes present with severe Click to open/closeBiochemistry
hypercalcaemia soon after birth and require Click to open/closeDiabetes Click to
Security parathyroidectomy. Occasionally a subgroup of adult open/closeEndocrinology Click to
heterozygotes does present with recurrent pancreatitis open/closeBlood gases Click to
Change Password and may require parathyroidectomy. The mutation open/closeCSF
causes a reduced ability for the calcium sensor to detect
Sign Out
hypercalcaemia, so the body tolerates levels of serum
calcium that would usually be said to be outside the
normal range.

1246

© 2011 PasTest Ltd | About Us | Contact Us | Help

file:///E|/Shakhawan/Endocrinology/53a.htm[3/13/2012 7:03:56 PM]


MyPasTest

Main Navigation
Question Browser: MRCP 1
Home
Subscriptions Question Browser Exam Builder Saved Exams
» MRCP 1 Session Progress
• Question Browser
Questions Correct 3
• Timed Test A 51-year-old woman presents to her GP with polyuria,
tiredness and a finger prick glucose level of 13.0 mmol/l. Questions Incorrect 0
• Mock Exam
According to the ADA criteria what should happen Questions Total 3
• Past Papers next?
Questions Percentage 100 %
• Random Questions
• My Performance She should be reassured that the result is normal More
• Media Bank She may have diabetes mellitus and requires a
fasting blood test the following day to confirm the
• New Multimedia diagnosis
Online Extras She probably has impaired glucose tolerance and Reference: Normal Values
Library should undergo a 2-h glucose tolerance test
A 2-h glucose tolerance test plasma glucose level of Click to
Community open/closeHaematology
10.5 mmol/l would confirm diabetes mellitus
Help Click to
A fasting plasma glucose of 7.2 mmol/l the following open/closeImmunoglobulins
PasTest Store day would suggest impaired fasting glucose Click to
open/closeBiochemistry
1409 Click to open/closeDiabetes
My Account
Click to
Profile open/closeEndocrinology
Click to open/closeBlood
Newsletters gases Click to
My Career and Exams open/closeCSF
Order History
Learning Goals
Question Filters

Security

Change Password
Sign Out

© 2011 PasTest Ltd | About Us | Contact Us | Help

file:///E|/Shakhawan/Endocrinology/54.htm[3/13/2012 7:03:57 PM]


MyPasTest

Main Navigation
Question Browser: MRCP 1
Home
Subscriptions Question Browser Exam Builder Saved Exams
» MRCP 1
• Question Browser
• Timed Test A 51-year-old woman presents to her GP with polyuria, tiredness and a finger prick
• Mock Exam glucose level of 13.0 mmol/l. According to the ADA criteria what should happen
next?
• Past Papers
• Random Questions She should be reassured that the result is normal
• My Performance She may have diabetes mellitus and requires a fasting blood test Your answer
• Media Bank the following day to confirm the diagnosis

• New Multimedia She probably has impaired glucose tolerance and should undergo
a 2-h glucose tolerance test
Online Extras
A 2-h glucose tolerance test plasma glucose level of 10.5 mmol/l
Library would confirm diabetes mellitus
Community A fasting plasma glucose of 7.2 mmol/l the following day would
Help suggest impaired fasting glucose
PasTest Store

My Account
The American Diabetes Association (ADA) criteria were developed in 1997, with a
Profile suggestion that a glucose tolerance test should be avoided, if possible, to make
diagnosis simpler and quicker. Three methods of classification of diabetes mellitus
Newsletters
were developed: symptoms and a random plasma glucose of greater than or equal
My Career and Exams to 11.1 mmol/l; fasting plasma glucose of greater than or equal to 7.0 mmol/l;
Order History and 75-g oral glucose tolerance test 2-h glucose of greater than or equal to 11.1
mmol/l. These diagnostic criteria still stand the test of time today. Ideally a
Learning Goals random raised fasting plasma glucose should be confirmed with a fasting sample.
Question Filters ‘Impaired fasting glucose’ (analogous to ‘impaired glucose tolerance’), is defined as
a plasma glucose level of greater than or equal to 5.6 mmol/l and less than 7.0
mmol/l.
Security

Change Password 1409


Sign Out

file:///E|/Shakhawan/Endocrinology/54a.htm[3/13/2012 7:03:58 PM]


MyPasTest

Main Navigation
Question Browser: MRCP 1
Home
Subscriptions Question Browser Exam Builder Saved Exams
» MRCP 1 Session Progress
• Question Browser
Questions Correct 4
• Timed Test A 26-year-old woman has been recently diagnosed with
type-1 diabetes. She has read a great deal about the Questions Incorrect 0
• Mock Exam
prognosis of renal disease in type-1 diabetes and has a Questions Total 4
• Past Papers number of questions to ask.
Questions Percentage 100 %
• Random Questions Which of the following statements best describes the
renal disease in patients with type-1 diabetes? More
• My Performance
• Media Bank Microalbuminuria usually occurs within 2 years of the
• New Multimedia diagnosis of type-1 diabetes
Online Extras Peak incidence of frank albuminuria is 17 years after Reference: Normal Values
the diagnosis of type-1 diabetes
Library
After the serum creatinine level reaches 200 µmol/l, Click to
Community a fall in GFR of 0.5 ml/min per month might be open/closeHaematology
Help expected Click to
End-stage renal failure usually occurs within 5 years open/closeImmunoglobulins
PasTest Store
of the onset of albuminuria Click to
open/closeBiochemistry
Nephropaths have a 10 times higher mortality rate Click to open/closeDiabetes
My Account
than age- and sex-matched controls Click to
Profile open/closeEndocrinology
1410 Click to open/closeBlood
Newsletters gases Click to
My Career and Exams open/closeCSF
Order History
Learning Goals
Question Filters

Security

Change Password
Sign Out

© 2011 PasTest Ltd | About Us | Contact Us | Help

file:///E|/Shakhawan/Endocrinology/55.htm[3/13/2012 7:04:00 PM]


MyPasTest

Main Navigation
Question Browser: MRCP 1
Home
Subscriptions Question Browser Exam Builder Saved Exams
» MRCP 1 Session Progress
• Question Browser
Questions Correct 4
• Timed Test A 26-year-old woman has been recently diagnosed with
type-1 diabetes. She has read a great deal about the Questions Incorrect 1
• Mock Exam
prognosis of renal disease in type-1 diabetes and has a Questions Total 5
• Past Papers number of questions to ask.
Questions Percentage 80 %
• Random Questions Which of the following statements best describes the
renal disease in patients with type-1 diabetes? More
• My Performance
• Media Bank Microalbuminuria usually occurs
• New Multimedia within 2 years of the diagnosis of
type-1 diabetes
Online Extras
Peak incidence of frank albuminuria Correct answer
Library is 17 years after the diagnosis of
Community type-1 diabetes
Help After the serum creatinine level Your answer
reaches 200 µmol/l, a fall in GFR of
PasTest Store
0.5 ml/min per month might be
expected
My Account End-stage renal failure usually
occurs within 5 years of the onset of
Profile albuminuria
Newsletters Nephropaths have a 10 times higher
My Career and Exams mortality rate than age- and sex-
matched controls
Order History Reference: Normal Values
Learning Goals
Click to open/closeHaematology
Question Filters Click to open/closeImmunoglobulins
Click to open/closeBiochemistry
In type-1 diabetes microalbuminuria usually occurs 5– Click to open/closeDiabetes Click to
Security 15 years after the diagnosis of diabetes mellitus. The open/closeEndocrinology Click to
peak incidence of frank albuminuria is around 17 years open/closeBlood gases Click to
Change Password after diagnosis. It is reasonable to expect a fall in the open/closeCSF
Sign Out glomerular filtration rate (GFR) of 1 ml/min per month
once the serum creatinine level has reached around 200
µmol/l.Traditionally, end-stage renal failure is said to
occur some 7–10 years after a diagnosis of albuminuria,
but it is hoped that improved management of
hyperglycaemia and hypertension may stretch this
period a little. Unfortunately, nephropathy carries a 20–
100 times higher mortality than that for age- and sex-
matched controls: 40% of deaths of patients with
nephropathy is due to cardiovascular disease.

1410

© 2011 PasTest Ltd | About Us | Contact Us | Help

file:///E|/Shakhawan/Endocrinology/55a.htm[3/13/2012 7:04:01 PM]


MyPasTest

Main Navigation
Question Browser: MRCP 1
Home
Subscriptions Question Browser Exam Builder Saved Exams
» MRCP 1
• Question Browser
• Timed Test A 70-year-old man is brought unconscious to the emergency department, his blood
• Mock Exam sugar level is 70 mmol/l. There is no evidence of ketoacidosis. His chest X-ray reveals
evidence of left-sided consolidation.
• Past Papers
• Random Questions What is the most likely diagnosis?
• My Performance
Simple pneumonia
• Media Bank
A complication of type-1 diabetes
• New Multimedia
A complication of type-2 diabetes
Online Extras
Stroke
Library
Glucagonoma
Community
Help 1411

PasTest Store

My Account

Profile
Newsletters
My Career and Exams
Order History
Learning Goals
Question Filters

Security

Change Password
Sign Out

file:///E|/Shakhawan/Endocrinology/56.htm[3/13/2012 7:04:02 PM]


MyPasTest

Main Navigation
Question Browser: MRCP 1
Home
Subscriptions Question Browser Exam Builder Saved Exams
» MRCP 1 Session Progress
• Question Browser
Questions Correct 5
• Timed Test A 70-year-old man is brought unconscious to the
emergency department, his blood sugar level is 70 mmol/l. Questions Incorrect 1
• Mock Exam
There is no evidence of ketoacidosis. His chest X-ray Questions Total 6
• Past Papers reveals evidence of left-sided consolidation.
Questions Percentage 83 %
• Random Questions
What is the most likely diagnosis? More
• My Performance
• Media Bank Simple pneumonia
• New Multimedia A complication of type-1 diabetes
Online Extras A complication of type-2 diabetes Your answer
Library Stroke
Community Glucagonoma
Help
PasTest Store

My Account He is most likely to be suffering from hyperosmolar


non-ketotic coma, which is associated with type-2
Profile diabetes and with coexistent infection. The common
presentation is often with blood glucose levels of over
Newsletters 50 mmol/l, a markedly raised serum osmolality and a
My Career and Exams decreased conscious level, without ketosis. Management
is with iv insulin and isotonic saline infusion, although Reference: Normal Values
Order History
the blood glucose level often drops quickly after insulin
Learning Goals is initiated. Hyperosmolar coma may be the initial Click to open/closeHaematology
Question Filters presentation of diabetes mellitus, and two-thirds of Click to open/closeImmunoglobulins
sufferers present with type-2 diabetes mellitus for the Click to open/closeBiochemistry
first time. Mortality is high, particularly in elderly Click to open/closeDiabetes Click to
Security sufferers. Glucagonoma is vanishingly rare, with an open/closeEndocrinology Click to
annual incidence of 1/20 million in the UK. open/closeBlood gases Click to
Change Password open/closeCSF
Sign Out
1411

© 2011 PasTest Ltd | About Us | Contact Us | Help

file:///E|/Shakhawan/Endocrinology/56a.htm[3/13/2012 7:04:04 PM]


MyPasTest

Main Navigation
Question Browser: MRCP 1
Home
Subscriptions Question Browser Exam Builder Saved Exams
» MRCP 1
• Question Browser
• Timed Test A 65-year-old obese woman who has had type-2 diabetes for 4 years presents to the
• Mock Exam clinic for her annual review. Her current medication is metformin 2 g/day. You decide
to add in gliclazide 80 mg po bd because of poor control as her Hb A 1c is 8.5%.
• Past Papers
• Random Questions Which of the following best describes sulphonylureas?
• My Performance
In the UKPDS study they demonstrated no effects on microvascular outcome
• Media Bank
In the UKPDS study they demonstrated no effect on macrovascular outcome
• New Multimedia
In the UKPDS study they demonstrated positive effects on macrovascular
Online Extras outcome
Library Mild hypoglycaemia in response to sulphonylurea therapy is rare
Community Gliclazide has positive effects on insulin resistance
Help
1412
PasTest Store

My Account

Profile
Newsletters
My Career and Exams
Order History
Learning Goals
Question Filters

Security

Change Password
Sign Out

file:///E|/Shakhawan/Endocrinology/57.htm[3/13/2012 7:04:05 PM]


MyPasTest

Main Navigation
Question Browser: MRCP 1
Home
Subscriptions Question Browser Exam Builder Saved Exams
» MRCP 1
• Question Browser
• Timed Test A 65-year-old obese woman who has had type-2 diabetes for 4 years presents to the
• Mock Exam clinic for her annual review. Her current medication is metformin 2 g/day. You decide
to add in gliclazide 80 mg po bd because of poor control as her Hb A 1c is 8.5%.
• Past Papers
• Random Questions Which of the following best describes sulphonylureas?
• My Performance
In the UKPDS study they demonstrated no effects on
• Media Bank microvascular outcome
• New Multimedia In the UKPDS study they demonstrated no effect on Correct answer
Online Extras macrovascular outcome
Library In the UKPDS study they demonstrated positive effects on Your answer
macrovascular outcome
Community
Mild hypoglycaemia in response to sulphonylurea therapy is
Help rare
PasTest Store Gliclazide has positive effects on insulin resistance

My Account

Profile
Newsletters Although sulphonylurea therapy was proven by the UKPDS study to have
microvascular benefits, no benefit on macrovascular outcome (cardiovascular
My Career and Exams disease) was demonstrated: this was, of course, in contrast to metformin.
Order History Sulphonylureas work by stimulating insulin release from pancreatic beta cells and
are associated with mild hypoglycaemia in up to 16% of users, although
Learning Goals hypoglycaemia may be less frequent when short-acting compounds are used.
Question Filters Certain groups of people (the elderly and those with renal impairment) may be at
a higher risk of hypoglycaemia. Sulphonylureas have no effect on insulin
resistance.
Security

Change Password 1412


Sign Out

file:///E|/Shakhawan/Endocrinology/57a.htm[3/13/2012 7:04:06 PM]


MyPasTest

Main Navigation
Question Browser: MRCP 1
Home
Subscriptions Question Browser Exam Builder Saved Exams
» MRCP 1 Session Progress
• Question Browser
Questions Correct 5
• Timed Test A 56-year-old, highly insulin resistant, type-2 diabetes
sufferer has been taking 200 units total daily dose of sc Questions Incorrect 2
• Mock Exam
insulin per day. His weight is increasing and his control Questions Total 7
• Past Papers worsening, with an Hb A 1C of 9.1%. You add in pioglitazone
Questions Percentage 71 %
• Random Questions 30 mg to his insulin. Some 4 weeks later he presents to
• My Performance the emergency department in heart failure. More
• Media Bank Which of the following statements best describes
• New Multimedia pioglitazone therapy?
Online Extras Pioglitazone causes heart failure by exerting a Reference: Normal Values
Library directly toxic effect on the myocardium
Click to
Community Pioglitazone has no more peripheral insulin- open/closeHaematology
sensitising effects than metformin Click to
Help
Troglitazone and pioglitazone have similar profiles of open/closeImmunoglobulins
PasTest Store
hepatotoxicity Click to
open/closeBiochemistry
Heart failure may be precipitated in some patients
My Account Click to open/closeDiabetes
taking glitazones due to fluid retention
Click to
Profile Glitazones act primarily at the site of the PPAR- α open/closeEndocrinology
receptor Click to open/closeBlood
Newsletters gases Click to
My Career and Exams 1413 open/closeCSF
Order History
Learning Goals
Question Filters

Security

Change Password
Sign Out

© 2011 PasTest Ltd | About Us | Contact Us | Help

file:///E|/Shakhawan/Endocrinology/58.htm[3/13/2012 7:04:08 PM]


MyPasTest

Main Navigation
Question Browser: MRCP 1
Home
Subscriptions Question Browser Exam Builder Saved Exams
» MRCP 1 Session Progress
• Question Browser
Questions Correct 6
• Timed Test A 56-year-old, highly insulin resistant, type-2 diabetes
sufferer has been taking 200 units total daily dose of sc Questions Incorrect 2
• Mock Exam
insulin per day. His weight is increasing and his control Questions Total 8
• Past Papers worsening, with an Hb A 1C of 9.1%. You add in pioglitazone
Questions Percentage 75 %
• Random Questions 30 mg to his insulin. Some 4 weeks later he presents to
• My Performance the emergency department in heart failure. More
• Media Bank Which of the following statements best describes
• New Multimedia pioglitazone therapy?
Online Extras Pioglitazone causes heart failure by
Library exerting a directly toxic effect on the
myocardium
Community
Pioglitazone has no more peripheral
Help
insulin-sensitising effects than
PasTest Store metformin
Troglitazone and pioglitazone have
My Account similar profiles of hepatotoxicity
Heart failure may be precipitated in Your answer
Profile some patients taking glitazones due to
Newsletters fluid retention
My Career and Exams Glitazones act primarily at the site of
the PPAR- α receptor Reference: Normal Values
Order History
Learning Goals
Click to open/closeHaematology
Question Filters Click to open/closeImmunoglobulins
Click to open/closeBiochemistry
There is currently one glitazone (insulin sensitisers) Click to open/closeDiabetes Click to
Security available across most of the world, pioglitazone. It acts open/closeEndocrinology Click to
at the site of the PPAR- γ receptor, promoting the open/closeBlood gases Click to
Change Password open/closeCSF
transcription of genes predominantly linked to fatty acid
Sign Out metabolism. Fibrates, and some newer dual agonists
currently in development, have activity at the PPAR- α
receptor site. Fluid retention may be precipitated by
these drugs, which may (in less than 1% of users)
precipitate heart failure. Experimental evidence has
demonstrated that these drugs actually improve
myocardial blood flow and glucose utilisation. Metformin
acts primarily by affecting hepatic insulin resistance,
whereas pioglitazone acts by improving insulin
sensitivity and peripheral glucose uptake in skeletal
muscle and fat. Troglitazone has been withdrawn due to
liver side-effects.

1413

© 2011 PasTest Ltd | About Us | Contact Us | Help

file:///E|/Shakhawan/Endocrinology/58a.htm[3/13/2012 7:04:09 PM]


MyPasTest

Main Navigation
Question Browser: MRCP 1
Home
Subscriptions Question Browser Exam Builder Saved Exams
» MRCP 1
• Question Browser
• Timed Test A 17-year-old young woman presents to the emergency department with a blood
• Mock Exam glucose of 29 mmol/l. She is known to have type-1 diabetes. Her pH is 7.12 with a
serum bicarbonate of 11 mmol/l. There is ketonuria.
• Past Papers
• Random Questions Which of the following statements best fits the predisposing factors involved
in DKA?
• My Performance
• Media Bank Myocardial infarction may be the precipitating factor in up to 5% of cases of
• New Multimedia DKA

Online Extras Infection may be the precipitating cause in 60% of cases of DKA

Library The patient is not previously known to have diabetes in 30% of DKA cases

Community Non-compliance with treatment is the cause in 25% of DKA cases

Help Inappropriate alterations to insulin are the cause in 20% of DKA cases

PasTest Store 1414

My Account

Profile
Newsletters
My Career and Exams
Order History
Learning Goals
Question Filters

Security

Change Password
Sign Out

file:///E|/Shakhawan/Endocrinology/59.htm[3/13/2012 7:04:10 PM]


MyPasTest

Main Navigation
Question Browser: MRCP 1
Home
Subscriptions Question Browser Exam Builder Saved Exams
» MRCP 1 Session Progress
• Question Browser
Questions Correct 6
• Timed Test A 17-year-old young woman presents to the emergency
department with a blood glucose of 29 mmol/l. She is Questions Incorrect 3
• Mock Exam
known to have type-1 diabetes. Her pH is 7.12 with a Questions Total 9
• Past Papers serum bicarbonate of 11 mmol/l. There is ketonuria.
Questions Percentage 66 %
• Random Questions
Which of the following statements best fits the More
• My Performance predisposing factors involved in DKA?
• Media Bank
• New Multimedia Myocardial infarction may be the
precipitating factor in up to 5% of
Online Extras cases of DKA
Library Infection may be the precipitating Your answer
Community cause in 60% of cases of DKA
Help The patient is not previously known
to have diabetes in 30% of DKA
PasTest Store cases
Non-compliance with treatment is Correct answer
My Account the cause in 25% of DKA cases
Inappropriate alterations to insulin
Profile
are the cause in 20% of DKA cases
Newsletters
My Career and Exams
Order History Reference: Normal Values
Learning Goals The commonest precipitant of diabetic ketoacidosis
Click to open/closeHaematology
Question Filters (DKA) is infection (30–40%). This is closely followed by
Click to open/closeImmunoglobulins
non-compliance with treatment (25%), alterations to
Click to open/closeBiochemistry
insulin dose (13%), newly diagnosed diabetes (10–
Click to open/closeDiabetes Click to
Security 20%) and myocardial infarction (< 1%). The condition
open/closeEndocrinology Click to
is characterised by hyperglycaemia, acidosis and
open/closeBlood gases Click to
Change Password ketonuria. DKA is common, with 9.1% of patients in the
open/closeCSF
EURODIAB study reporting hospitalisation over the
Sign Out
period of a year. DKA is caused by insulin deficiency and
counter-regulatory hormone excess. The mortality rate
is 2–5%, but up to 50% in more elderly patients.

1414

© 2011 PasTest Ltd | About Us | Contact Us | Help

file:///E|/Shakhawan/Endocrinology/59a.htm[3/13/2012 7:04:12 PM]


MyPasTest

Main Navigation
Question Browser: MRCP 1
Home
Subscriptions Question Browser Exam Builder Saved Exams
» MRCP 1 Session Progress
• Question Browser
Questions Correct 6
• Timed Test A 36-year-old woman who is 28 weeks pregnant attends
the midwife clinic for a pregnancy check. She is noted to Questions Incorrect 3
• Mock Exam
have glycosuria and has a BMI of 30 You understand her Questions Total 9
• Past Papers mother has Type-2 diabetes.
Questions Percentage 66 %
• Random Questions What is her correct management plan?
• My Performance More
Observe and follow up in 4 weeks time with repeat
• Media Bank urine glucose testing
• New Multimedia Carry out a fasting blood glucose; if it is less than 7.0
Online Extras mmol/l then repeat the sample in 4 weeks’ time Reference: Normal Values
Library Carry out a fasting blood glucose; if it is over 5.5
mmol/l then proceed to administer a 75-g oral glucose Click to
Community tolerance test open/closeHaematology
Help Carry out a fasting blood glucose; it is over 7.0 Click to
mmol/l then proceed to administer a 75-g oral glucose open/closeImmunoglobulins
PasTest Store
tolerance test Click to
open/closeBiochemistry
Proceed directly to administer an oral glucose Click to open/closeDiabetes
My Account
tolerance test Click to
Profile open/closeEndocrinology
1415 Click to open/closeBlood
Newsletters gases Click to
My Career and Exams open/closeCSF
Order History
Learning Goals
Question Filters

Security

Change Password
Sign Out

© 2011 PasTest Ltd | About Us | Contact Us | Help

file:///E|/Shakhawan/Endocrinology/60.htm[3/13/2012 7:04:13 PM]


MyPasTest

Main Navigation
Question Browser: MRCP 1
Home
Subscriptions Question Browser Exam Builder Saved Exams
» MRCP 1 Session Progress
• Question Browser
Questions Correct 7
• Timed Test A 36-year-old woman who is 28 weeks pregnant attends
the midwife clinic for a pregnancy check. She is noted to Questions Incorrect 3
• Mock Exam
have glycosuria and has a BMI of 30 You understand her Questions Total 10
• Past Papers mother has Type-2 diabetes.
Questions Percentage 70 %
• Random Questions What is her correct management plan?
• My Performance More
Observe and follow up in 4 weeks time
• Media Bank with repeat urine glucose testing
• New Multimedia Carry out a fasting blood glucose; if it
Online Extras is less than 7.0 mmol/l then repeat the
sample in 4 weeks’ time
Library
Carry out a fasting blood glucose; if it Your answer
Community is over 5.5 mmol/l then proceed to
Help administer a 75-g oral glucose tolerance
test
PasTest Store
Carry out a fasting blood glucose; it is
over 7.0 mmol/l then proceed to
My Account administer a 75-g oral glucose tolerance
test
Profile
Proceed directly to administer an oral
Newsletters glucose tolerance test
My Career and Exams
Order History Reference: Normal Values
Learning Goals
Click to open/closeHaematology
Question Filters A glucose tolerance test is required if the patient’s Click to open/closeImmunoglobulins
fasting blood glucose level is over 5.5 mmol/l. Diabetes Click to open/closeBiochemistry
is defined as a fasting level of > 7.0 mmol/l or > 11.0 Click to open/closeDiabetes Click to
Security mmol/l 2 h after a glucose tolerance test. Gestational open/closeEndocrinology Click to
impaired glucose tolerance (IGT) is defined as a fasting open/closeBlood gases Click to
Change Password blood glucose level of 6.0–7.8 mmol/l and/or a 2-h level open/closeCSF
Sign Out of 9–11 mmol/l. Although initial treatment for
gestational diabetes is with dietary advice, insulin is
also required in 10–30% of cases. Insulin should be
considered for those with fasting levels above 6.0
mmol/l after dietary intervention, or postprandial sugar
levels above 8.0 mmol/l. Untreated gestational diabetes
is said to carry a perinatal mortality rate of 4.4–6.4%,
compared to 0.5–1.5% in a similarly matched
normoglycaemic population: hence, good glycaemic
control is essential for this reason.

1415

© 2011 PasTest Ltd | About Us | Contact Us | Help

file:///E|/Shakhawan/Endocrinology/60a.htm[3/13/2012 7:04:14 PM]


MyPasTest

Main Navigation
Question Browser: MRCP 1
Home
Subscriptions Question Browser Exam Builder Saved Exams
» MRCP 1 Session Progress
• Question Browser
Questions Correct 1
• Timed Test A 28-year-old mature student nurse is admitted from the
emergency department, where she is on placement, having Questions Incorrect 0
• Mock Exam
suffered a collapse. Her blood sugar was noted to be 1.5 Questions Total 1
• Past Papers mmol/l and she responded to 50 ml of 50% iv dextrose.
Questions Percentage 100 %
• Random Questions After a second collapse, her insulin level was raised without
a similar rise in c-peptide, her glucose level was 1.9 More
• My Performance
mmol/l.
• Media Bank What is the most appropriate next investigation?
• New Multimedia
A 3-day fast in an attempt to precipitate a further
Online Extras attack Reference: Normal Values
Library Audit of insulin stocks that she has had access to
Click to
Community Alcohol history open/closeHaematology
Help Chest X-ray to exclude malignancy Click to
open/closeImmunoglobulins
PasTest Store Discharge the patient Click to
open/closeBiochemistry
1416 Click to open/closeDiabetes
My Account
Click to
Profile open/closeEndocrinology
Click to open/closeBlood
Newsletters gases Click to
My Career and Exams open/closeCSF
Order History
Learning Goals
Question Filters

Security

Change Password
Sign Out

© 2011 PasTest Ltd | About Us | Contact Us | Help

file:///E|/Shakhawan/Endocrinology/61.htm[3/13/2012 7:04:16 PM]


MyPasTest

Main Navigation
Question Browser: MRCP 1
Home
Subscriptions Question Browser Exam Builder Saved Exams
» MRCP 1 Session Progress
• Question Browser
Questions Correct 2
• Timed Test A 28-year-old mature student nurse is admitted from the
emergency department, where she is on placement, having Questions Incorrect 0
• Mock Exam
suffered a collapse. Her blood sugar was noted to be 1.5 Questions Total 2
• Past Papers mmol/l and she responded to 50 ml of 50% iv dextrose.
Questions Percentage 100 %
• Random Questions After a second collapse, her insulin level was raised without
a similar rise in c-peptide, her glucose level was 1.9 More
• My Performance
mmol/l.
• Media Bank What is the most appropriate next investigation?
• New Multimedia
A 3-day fast in an attempt to
Online Extras precipitate a further attack
Library Audit of insulin stocks that she has had Your answer
Community access to
Help Alcohol history
PasTest Store Chest X-ray to exclude malignancy
Discharge the patient
My Account

Profile
Newsletters
The clue is the fact that her insulin level was raised
My Career and Exams without a corresponding rise in c-peptide. This
Order History essentially rules out insulinoma, and suggests that the Reference: Normal Values
patient is either injecting excess insulin herself or is
Learning Goals taking sulphonylureas. In fact, on further questioning Click to open/closeHaematology
Question Filters her aunt has type-2 diabetes, she has been struggling Click to open/closeImmunoglobulins
with her studies and taking sulphonylureas to Click to open/closeBiochemistry
precipitate the attacks as a ‘cry for help’. Standard Click to open/closeDiabetes Click to
Security work-up for hypoglycaemia should include laboratory open/closeEndocrinology Click to
blood glucose (not test strips) measurement, liver open/closeBlood gases Click to
Change Password function tests to rule out significant liver dysfunction, open/closeCSF
Sign Out blood alcohol and alcohol history, cortisol ± Synacthen
testing, insulin and c-peptide levels taken during an
attack as well as a chest X-ray to exclude occult
malignancy.

1416

© 2011 PasTest Ltd | About Us | Contact Us | Help

file:///E|/Shakhawan/Endocrinology/61a.htm[3/13/2012 7:04:17 PM]


MyPasTest

Main Navigation
Question Browser: MRCP 1
Home
Subscriptions Question Browser Exam Builder Saved Exams
» MRCP 1 Session Progress
• Question Browser
Questions Correct 2
• Timed Test A 68-year-old man attends his GP on the insistence of his
wife because his left forefoot has become increasingly Questions Incorrect 0
• Mock Exam
unstable and abnormally shaped with bony swelling. He has Questions Total 2
• Past Papers a long history of diabetes mellitus and his control has been
Questions Percentage 100 %
• Random Questions erratic, with Hb A 1c in the range of 9–10%.
• My Performance More
Which of the following pieces of information best fits
• Media Bank the pathology or management of Charcot’s foot?
• New Multimedia
Tight glucose control is unlikely to be of benefit in
Online Extras this condition Reference: Normal Values
Library Reduction of oedema is unimportant with respect to Click to
Community preventing deterioration open/closeHaematology
Help Orthotic shoes are unnecessary in the management Click to
of this condition open/closeImmunoglobulins
PasTest Store
Click to
Plain radiography is always abnormal, even early in
open/closeBiochemistry
this condition
My Account Click to open/closeDiabetes
The pathology of this condition is thought to be due Click to
Profile to sympathetic dysfunction, excessive blood flow to open/closeEndocrinology
the joint and osteoclast activity Click to open/closeBlood
Newsletters gases Click to
My Career and Exams 1417 open/closeCSF
Order History
Learning Goals
Question Filters

Security

Change Password
Sign Out

© 2011 PasTest Ltd | About Us | Contact Us | Help

file:///E|/Shakhawan/Endocrinology/62.htm[3/13/2012 7:04:19 PM]


MyPasTest

Main Navigation
Question Browser: MRCP 1
Home
Subscriptions Question Browser Exam Builder Saved Exams
» MRCP 1 Session Progress
• Question Browser
Questions Correct 3
• Timed Test A 68-year-old man attends his GP on the insistence of his
wife because his left forefoot has become increasingly Questions Incorrect 0
• Mock Exam
unstable and abnormally shaped with bony swelling. He has Questions Total 3
• Past Papers a long history of diabetes mellitus and his control has been
Questions Percentage 100 %
• Random Questions erratic, with Hb A 1c in the range of 9–10%.
• My Performance More
Which of the following pieces of information best fits
• Media Bank the pathology or management of Charcot’s foot?
• New Multimedia
Tight glucose control is unlikely to be
Online Extras of benefit in this condition
Library Reduction of oedema is unimportant
Community with respect to preventing deterioration
Help Orthotic shoes are unnecessary in the
management of this condition
PasTest Store
Plain radiography is always abnormal,
even early in this condition
My Account
The pathology of this condition is Your answer
Profile thought to be due to sympathetic
dysfunction, excessive blood flow to the
Newsletters joint and osteoclast activity
My Career and Exams
Order History Reference: Normal Values
Learning Goals
Click to open/closeHaematology
Question Filters This man has a Charcot left foot. This is a rare condition Click to open/closeImmunoglobulins
now, with the average general hospital clinic seeing a Click to open/closeBiochemistry
maximum of 10 Charcot’s patients. Chronic untreated Click to open/closeDiabetes Click to
Security Charcot’s foot results in either ‘rocker-bottom’ foot due open/closeEndocrinology Click to
to downward displacement and subluxation of the open/closeBlood gases Click to
Change Password tarsus, or medial convexity due to talonavicular joint open/closeCSF
Sign Out displacement or dislocation of the tarsometatarsal
joints. Management involves tight glycaemic control,
reduction of oedema and orthotic and chiropody
involvement. Intravenous bisphosphonate treatment
may be of benefit in some patients. Although plain
radiographs may be normal in the early stages of the
disease, they later show joint destruction, osteolysis,
joint reorganisation and subluxation.

1417

© 2011 PasTest Ltd | About Us | Contact Us | Help

file:///E|/Shakhawan/Endocrinology/62a.htm[3/13/2012 7:04:20 PM]


MyPasTest

Main Navigation
Question Browser: MRCP 1
Home
Subscriptions Question Browser Exam Builder Saved Exams
» MRCP 1 Session Progress
• Question Browser
Questions Correct 3
• Timed Test A 56-year-old patient on gliclazide for his type-2 diabetes
presents with an acute, central, crushing chest pain. He is Questions Incorrect 0
• Mock Exam
diagnosed with myocardial infarction. Questions Total 3
• Past Papers
Which of the following best fits the outcome or Questions Percentage 100 %
• Random Questions
management of myocardial infarction associated with More
• My Performance type-2 diabetes?
• Media Bank
• New Multimedia The mortality rate from myocardial infarction in
patients with type-2 diabetes is the same as that for
Online Extras non-diabetics Reference: Normal Values
Library Intravenous insulin followed by sc insulin after MI
reduces mortality by 11% at 3.5 years compared to Click to
Community open/closeHaematology
controls
Help Click to
Use of ACE inhibition after MI improves the 6-week open/closeImmunoglobulins
PasTest Store mortality rate by 50% Click to
Statins should always be started unless they are open/closeBiochemistry
My Account contra-indicated Click to open/closeDiabetes
Click to
Blood pressure target should be 150/80 mmHg open/closeEndocrinology
Profile
Click to open/closeBlood
Newsletters 1418 gases Click to
My Career and Exams open/closeCSF
Order History
Learning Goals
Question Filters

Security

Change Password
Sign Out

© 2011 PasTest Ltd | About Us | Contact Us | Help

file:///E|/Shakhawan/Endocrinology/63.htm[3/13/2012 7:04:22 PM]


MyPasTest

Main Navigation
Question Browser: MRCP 1
Home
Subscriptions Question Browser Exam Builder Saved Exams
» MRCP 1 Session Progress
• Question Browser
Questions Correct 4
• Timed Test A 56-year-old patient on gliclazide for his type-2 diabetes
presents with an acute, central, crushing chest pain. He is Questions Incorrect 0
• Mock Exam
diagnosed with myocardial infarction. Questions Total 4
• Past Papers
Which of the following best fits the outcome or Questions Percentage 100 %
• Random Questions
management of myocardial infarction associated with More
• My Performance type-2 diabetes?
• Media Bank
• New Multimedia The mortality rate from myocardial
infarction in patients with type-2
Online Extras diabetes is the same as that for non-
Library diabetics
Community Intravenous insulin followed by sc
insulin after MI reduces mortality by
Help 11% at 3.5 years compared to controls
PasTest Store Use of ACE inhibition after MI improves
the 6-week mortality rate by 50%
My Account Statins should always be started unless Your answer
they are contra-indicated
Profile
Blood pressure target should be
Newsletters 150/80 mmHg
My Career and Exams
Order History Reference: Normal Values
Learning Goals
Click to open/closeHaematology
Question Filters Diabetics may be up to twice as likely to die from
Click to open/closeImmunoglobulins
myocardial infarction as non-diabetics, and are more
Click to open/closeBiochemistry
likely to suffer an MI in the first place. The DIGAMI
Click to open/closeDiabetes Click to
Security study used iv insulin for 24 h followed by sc insulin for
open/closeEndocrinology Click to
patients who had suffered an MI. Risk reductions
open/closeBlood gases Click to
Change Password equalled 7.5% at 1 year and 11% at 3.5 years
open/closeCSF
compared to controls. There is now significant doubt
Sign Out
about this result as the study was repeated in the
DIGAMI-2 study. This showed no advantage in terms of
cardiac outcomes for patients treated for a prolonged
period with insulin. Use of ACE inhibitors (GISSI-3
Diabetic Subgroup Study) is associated with a 30%
relative-risk reduction in the 6-week mortality rate
(8.7% vs 12.4%). Blood pressure reduction should be
aggressively targeted, aiming for 130/80 mmHg, and all
patients with diabetes who are over 40 years of age
should be started on statin treatment unless there is a
contraindication, (Joint British Societies Guidelines).

1418

© 2011 PasTest Ltd | About Us | Contact Us | Help

file:///E|/Shakhawan/Endocrinology/63a.htm[3/13/2012 7:04:23 PM]


MyPasTest

Main Navigation
Question Browser: MRCP 1
Home
Subscriptions Question Browser Exam Builder Saved Exams
» MRCP 1 Session Progress
• Question Browser
Questions Correct 4
• Timed Test A 51-year-old patient on metformin and gliclazide for his
type-2 diabetes presents for review. He has a BMI of 31 Questions Incorrect 0
• Mock Exam
and his blood pressure is 158/92 mmHg. Questions Total 4
• Past Papers
When considering hypertension in type-2 diabetes, Questions Percentage 100 %
• Random Questions
which of the following statements best fits the More
• My Performance condition?
• Media Bank
• New Multimedia Development of hypertension is strongly linked to a
genetically inherited variant of red cell sodium–lithium
Online Extras counter-transport activity Reference: Normal Values
Library Good hypertensive control has a weaker effect on
macrovascular outcomes than good tight glucose Click to
Community open/closeHaematology
control
Help Click to
Hypertension in type-2 diabetes is primarily open/closeImmunoglobulins
PasTest Store associated with hyperinsulinaemia and insulin Click to
resistance open/closeBiochemistry
My Account Hypertension affects up to 20% of patients with Click to open/closeDiabetes
type-2 diabetes Click to
Profile open/closeEndocrinology
Thiazide diuretics are the treatment of choice in Click to open/closeBlood
Newsletters patients with hypertension and microalbuminuria gases Click to
My Career and Exams open/closeCSF
1419
Order History
Learning Goals
Question Filters

Security

Change Password
Sign Out

© 2011 PasTest Ltd | About Us | Contact Us | Help

file:///E|/Shakhawan/Endocrinology/64.htm[3/13/2012 7:04:25 PM]


MyPasTest

Main Navigation
Question Browser: MRCP 1
Home
Subscriptions Question Browser Exam Builder Saved Exams
» MRCP 1 Session Progress
• Question Browser
Questions Correct 4
• Timed Test A 51-year-old patient on metformin and gliclazide for his
Questions Incorrect 1
• Mock Exam type-2 diabetes presents for review. He has a BMI of 31
and his blood pressure is 158/92 mmHg. Questions Total 5
• Past Papers
Questions Percentage 80 %
• Random Questions When considering hypertension in type-2 diabetes,
which of the following statements best fits the More
• My Performance condition?
• Media Bank
• New Multimedia Development of hypertension is Your answer
strongly linked to a genetically
Online Extras inherited variant of red cell sodium–
Library lithium counter-transport activity
Community Good hypertensive control has a
weaker effect on macrovascular
Help outcomes than good tight glucose
PasTest Store control
Hypertension in type-2 diabetes is Correct answer
primarily associated with
My Account hyperinsulinaemia and insulin
Profile resistance

Newsletters Hypertension affects up to 20% of


patients with type-2 diabetes
My Career and Exams
Thiazide diuretics are the treatment
Order History of choice in patients with Reference: Normal Values
Learning Goals hypertension and microalbuminuria
Click to open/closeHaematology
Question Filters Click to open/closeImmunoglobulins
Click to open/closeBiochemistry
Click to open/closeDiabetes Click to
Security open/closeEndocrinology Click to
Hyperinsulinaemia directly stimulates sympathetic
open/closeBlood gases Click to
Change Password nervous system activity, increasing renal sodium
open/closeCSF
reabsorption and promoting vascular smooth muscle
Sign Out
proliferation. Insulin resistance is also associated with
the reduced activity of nitric oxide synthase, leading to
reduced vasodilatation in response to vascular stress.
Red cell sodium–lithium counter-transport activity
abnormalities are associated with the early development
of hypertension in type-1 diabetes. Between 30 and
50% of Caucasians with type-2 diabetes have a systolic
blood pressure above 160 mmHg. ACE inhibitors are the
treatment of choice for diabetic hypertension,
particularly where microalbuminuria coexists. Thiazide
diuretics and β-blockers may worsen glycaemic control.

1419

© 2011 PasTest Ltd | About Us | Contact Us | Help

file:///E|/Shakhawan/Endocrinology/64a.htm[3/13/2012 7:04:26 PM]


MyPasTest

Main Navigation
Question Browser: MRCP 1
Home
Subscriptions Question Browser Exam Builder Saved Exams
» MRCP 1 Session Progress
• Question Browser
Questions Correct 0
• Timed Test A 52-year-old man is sent by his GP for an urgent review.
He has been maintained on metformin and gliclazide for his Questions Incorrect 0
• Mock Exam
type-2 diabetes. For the past week or so he has had Questions Total 0
• Past Papers feelings of severe aching pain and paraesthesias in his
Questions Percentage 0%
• Random Questions upper legs. He has felt ‘off his food’ during the past week
and has begun losing weight. There is also proximal muscle More
• My Performance
weakness.
• Media Bank
Which of the following best fits diabetic amyotrophy?
• New Multimedia
Online Extras 75% of patients recover fully from this condition Reference: Normal Values
Library Transference to insulin therapy is the mainstay of
treatment Click to
Community open/closeHaematology
Help Recovery from this condition usually takes over 1 Click to
year open/closeImmunoglobulins
PasTest Store
The condition is much more common in type-1 Click to
diabetes open/closeBiochemistry
My Account Click to open/closeDiabetes
Weight loss is unlikely to be related to the diagnosis Click to
Profile open/closeEndocrinology
1420 Click to open/closeBlood
Newsletters gases Click to
My Career and Exams open/closeCSF
Order History
Learning Goals
Question Filters

Security

Change Password
Sign Out

© 2011 PasTest Ltd | About Us | Contact Us | Help

file:///E|/Shakhawan/Endocrinology/65.htm[3/13/2012 7:04:27 PM]


MyPasTest

Main Navigation
Question Browser: MRCP 1
Home
Subscriptions Question Browser Exam Builder Saved Exams
» MRCP 1 Session Progress
• Question Browser
Questions Correct 0
• Timed Test A 52-year-old man is sent by his GP for an urgent review.
He has been maintained on metformin and gliclazide for his Questions Incorrect 1
• Mock Exam
type-2 diabetes. For the past week or so he has had Questions Total 1
• Past Papers feelings of severe aching pain and paraesthesias in his
Questions Percentage 0%
• Random Questions upper legs. He has felt ‘off his food’ during the past week
and has begun losing weight. There is also proximal muscle More
• My Performance
weakness.
• Media Bank
Which of the following best fits diabetic amyotrophy?
• New Multimedia
Online Extras 75% of patients recover fully from
Library this condition
Community Transference to insulin therapy is Correct answer
the mainstay of treatment
Help
Recovery from this condition usually Your answer
PasTest Store takes over 1 year
The condition is much more
My Account common in type-1 diabetes
Weight loss is unlikely to be related
Profile
to the diagnosis
Newsletters
My Career and Exams
Order History Reference: Normal Values
Learning Goals Diabetic amyotrophy is said to occur most commonly in Click to open/closeHaematology
Question Filters men in their fifties with type-2 diabetes treated with Click to open/closeImmunoglobulins
oral hypoglycaemic agents. It is a mixed motor and Click to open/closeBiochemistry
sensory proximal neuropathy said to cause severe pain, Click to open/closeDiabetes Click to
Security which is responsible for anorexia and weight loss. Some open/closeEndocrinology Click to
50% of patients recover fully from this condition, open/closeBlood gases Click to
Change Password usually within 3–4 months. The mainstay of treatment open/closeCSF
is supportive care and transference to insulin therapy.
Sign Out

1420

© 2011 PasTest Ltd | About Us | Contact Us | Help

file:///E|/Shakhawan/Endocrinology/65a.htm[3/13/2012 7:04:29 PM]


MyPasTest

Main Navigation
Question Browser: MRCP 1
Home
Subscriptions Question Browser Exam Builder Saved Exams
» MRCP 1 Session Progress
• Question Browser
Questions Correct 0
• Timed Test A 63-year-old man presents to the diabetes clinic for
review. He attends with his wife who has accused him of a Questions Incorrect 1
• Mock Exam
lack of interest in her as he is no longer able to sustain an Questions Total 1
• Past Papers erection.
Questions Percentage 0%
• Random Questions
Which of the following statements is most strongly More
• My Performance associated with impotence in type-2 diabetes?
• Media Bank
• New Multimedia Libido is often also affected in addition to physical
pathology
Online Extras Reference: Normal Values
Estimates say that 20% of men with diabetes
Library mellitus of more than 6 years’ duration are affected
Click to
Community Other autonomic neuropathy is unlikely to be present open/closeHaematology
Help Penile pain is a common association Click to
open/closeImmunoglobulins
PasTest Store b-Blockers and thiazide diuretics may exacerbate the Click to
problem open/closeBiochemistry
My Account Click to open/closeDiabetes
1421 Click to
Profile open/closeEndocrinology
Click to open/closeBlood
Newsletters gases Click to
My Career and Exams open/closeCSF
Order History
Learning Goals
Question Filters

Security

Change Password
Sign Out

© 2011 PasTest Ltd | About Us | Contact Us | Help

file:///E|/Shakhawan/Endocrinology/66.htm[3/13/2012 7:04:30 PM]


MyPasTest

Main Navigation
Question Browser: MRCP 1
Home
Subscriptions Question Browser Exam Builder Saved Exams
» MRCP 1 Session Progress
• Question Browser
Questions Correct 1
• Timed Test A 63-year-old man presents to the diabetes clinic for
review. He attends with his wife who has accused him of a Questions Incorrect 1
• Mock Exam
lack of interest in her as he is no longer able to sustain an Questions Total 2
• Past Papers erection.
Questions Percentage 50 %
• Random Questions
Which of the following statements is most strongly More
• My Performance associated with impotence in type-2 diabetes?
• Media Bank
• New Multimedia Libido is often also affected in addition
to physical pathology
Online Extras
Estimates say that 20% of men with
Library diabetes mellitus of more than 6 years’
Community duration are affected
Help Other autonomic neuropathy is unlikely
to be present
PasTest Store
Penile pain is a common association
b-Blockers and thiazide diuretics may Your answer
My Account exacerbate the problem
Profile
Newsletters
My Career and Exams
Libido is rarely affected, which makes the condition all Reference: Normal Values
Order History
the more frustrating for affected men. Between 30 and
Learning Goals 50% of men with diabetes of more than 6 years’ Click to open/closeHaematology
Question Filters duration are affected. There is an association with other Click to open/closeImmunoglobulins
autonomic neuropathies such as postural hypotension Click to open/closeBiochemistry
and gastroparesis. The condition is not usually Click to open/closeDiabetes Click to
Security associated with pain, and may be exacerbated by both open/closeEndocrinology Click to
β -blockers and thiazide diuretics. PDE-5 inhibitors are open/closeBlood gases Click to
Change Password now available as oral therapies for impotence, but these open/closeCSF
Sign Out are often required at high dose. Alprostadil and vacuum
devices are alternative methods. Estimates put
successful treatment at only around 70%.

1421

© 2011 PasTest Ltd | About Us | Contact Us | Help

file:///E|/Shakhawan/Endocrinology/66a.htm[3/13/2012 7:04:31 PM]


MyPasTest

Main Navigation
Question Browser: MRCP 1
Home
Subscriptions Question Browser Exam Builder Saved Exams
» MRCP 1 Session Progress
• Question Browser
Questions Correct 1
• Timed Test A 54-year-old woman presents to the diabetes clinic for
review. She has suffered from diabetes mellitus for many Questions Incorrect 1
• Mock Exam
years and has now progressed from sulphonylurea Questions Total 2
• Past Papers treatment to insulin therapy. A past history of gallstones is
Questions Percentage 50 %
• Random Questions noted and she failed to tolerate metformin due to chronic
diarrhoea. She has been slowly losing weight over the past More
• My Performance
few months and has mild anaemia. Ultrasound scan of her
• Media Bank liver reveals a suggestion of a number of small metastases
• New Multimedia within the liver.

Online Extras When considering that this woman has a primary Reference: Normal Values
Library pancreatic tumour, which of the following statements
best fits the condition? Click to
Community open/closeHaematology
Help Somatostatin, ACTH and calcitonin may all be raised Click to
Contrast spiral-CT scanning is effective in open/closeImmunoglobulins
PasTest Store
demonstrating the primary tumour in 90% of cases Click to
open/closeBiochemistry
Curative surgery is possible in the majority of cases Click to open/closeDiabetes
My Account
There is almost always an association with MEN-1 Click to
Profile open/closeEndocrinology
Most tumours are detected because of the clinical Click to open/closeBlood
Newsletters syndrome gases Click to
My Career and Exams open/closeCSF
1422
Order History
Learning Goals
Question Filters

Security

Change Password
Sign Out

© 2011 PasTest Ltd | About Us | Contact Us | Help

file:///E|/Shakhawan/Endocrinology/67.htm[3/13/2012 7:04:33 PM]


MyPasTest

Main Navigation
Question Browser: MRCP 1
Home
Subscriptions Question Browser Exam Builder Saved Exams
» MRCP 1 Session Progress
• Question Browser
Questions Correct 2
• Timed Test A 54-year-old woman presents to the diabetes clinic for
review. She has suffered from diabetes mellitus for many Questions Incorrect 1
• Mock Exam
years and has now progressed from sulphonylurea Questions Total 3
• Past Papers treatment to insulin therapy. A past history of gallstones is
Questions Percentage 66 %
• Random Questions noted and she failed to tolerate metformin due to chronic
diarrhoea. She has been slowly losing weight over the past More
• My Performance
few months and has mild anaemia. Ultrasound scan of her
• Media Bank liver reveals a suggestion of a number of small metastases
• New Multimedia within the liver.

Online Extras When considering that this woman has a primary


Library pancreatic tumour, which of the following statements
best fits the condition?
Community
Help Somatostatin, ACTH and calcitonin may Your answer
all be raised
PasTest Store
Contrast spiral-CT scanning is effective
in demonstrating the primary tumour in
My Account 90% of cases

Profile Curative surgery is possible in the


majority of cases
Newsletters
There is almost always an association
My Career and Exams with MEN-1
Order History Most tumours are detected because of Reference: Normal Values
Learning Goals the clinical syndrome
Click to open/closeHaematology
Question Filters Click to open/closeImmunoglobulins
Click to open/closeBiochemistry
Click to open/closeDiabetes Click to
Security open/closeEndocrinology Click to
The history is suggestive of a somatostatinoma, the open/closeBlood gases Click to
Change Password annual incidence of which is estimated at 1 in 40 open/closeCSF
million. There is a 95% association with impaired
Sign Out
glucose tolerance (IGT) or diabetes mellitus, 68% with
gallstones, weight loss 25% and anaemia 14%, there is
also an association with diarrhoea. The tumours are
often multisecretory and ACTH and calcitonin levels may
be raised in addition to that of somatostatin. There is an
association with MEN-1 in some (7%). Many tumours
may occur undetected, and not causing the
somatostatinoma syndrome. Contrast spiral-CT
scanning is effective in detecting the primary tumour in
only 50% of cases; radiolabelled octreotide or
endoscopic ultrasound scanning may often be required.
Although surgical cure is rarely possible due to the
presence of metastases, hepatic embolisation may be
helpful for symptom control.

1422

© 2011 PasTest Ltd | About Us | Contact Us | Help

file:///E|/Shakhawan/Endocrinology/67a.htm[3/13/2012 7:04:34 PM]


MyPasTest

Main Navigation
Question Browser: MRCP 1
Home
Subscriptions Question Browser Exam Builder Saved Exams
» MRCP 1 Session Progress
• Question Browser
Questions Correct 2
• Timed Test A 52-year-old woman, diagnosed with type-2 diabetes
mellitus and losing weight, is referred for an opinion; her Questions Incorrect 1
• Mock Exam
GP is thinking about insulin therapy. A normochromic, Questions Total 3
• Past Papers normocytic anaemia is noted. On examination she has
Questions Percentage 66 %
• Random Questions angular stomatitis and a well-demarcated erythematous
rash in her groin, which extends to her lower limbs, More
• My Performance
buttocks and perineum.
• Media Bank
What is the next step in her management?
• New Multimedia
Online Extras Refer to nursing colleagues for conversion to insulin Reference: Normal Values
Library Try high-dose sulphonylurea therapy
Click to
Community Observe and see again in 6 months open/closeHaematology
Help Measure plasma glucagon levels Click to
open/closeImmunoglobulins
PasTest Store Measure plasma somatostatin levels Click to
open/closeBiochemistry
1423 Click to open/closeDiabetes
My Account
Click to
Profile open/closeEndocrinology
Click to open/closeBlood
Newsletters gases Click to
My Career and Exams open/closeCSF
Order History
Learning Goals
Question Filters

Security

Change Password
Sign Out

© 2011 PasTest Ltd | About Us | Contact Us | Help

file:///E|/Shakhawan/Endocrinology/68.htm[3/13/2012 7:04:36 PM]


MyPasTest

Main Navigation
Question Browser: MRCP 1
Home
Subscriptions Question Browser Exam Builder Saved Exams
» MRCP 1 Session Progress
• Question Browser
Questions Correct 3
• Timed Test A 52-year-old woman, diagnosed with type-2 diabetes
mellitus and losing weight, is referred for an opinion; her Questions Incorrect 1
• Mock Exam
GP is thinking about insulin therapy. A normochromic, Questions Total 4
• Past Papers normocytic anaemia is noted. On examination she has
Questions Percentage 75 %
• Random Questions angular stomatitis and a well-demarcated erythematous
rash in her groin, which extends to her lower limbs, More
• My Performance
buttocks and perineum.
• Media Bank
What is the next step in her management?
• New Multimedia
Online Extras Refer to nursing colleagues for
Library conversion to insulin
Community Try high-dose sulphonylurea therapy
Help Observe and see again in 6 months
PasTest Store Measure plasma glucagon levels Your answer
Measure plasma somatostatin levels

My Account

Profile
Newsletters There is a suggestion that this patient has a
My Career and Exams glucagonoma, 90% of which are associated with the
characteristic skin rash – necrolytic migratory Reference: Normal Values
Order History
erythema. The annual incidence of glucagonoma is
Learning Goals estimated at 1 in 20 million. Over 70% of glucagonomas Click to open/closeHaematology
Question Filters are malignant, but, because of their indolent Click to open/closeImmunoglobulins
presentation, the diagnosis is often overlooked for many Click to open/closeBiochemistry
years. Due to insulin antagonism over 90% are Click to open/closeDiabetes Click to
Security associated with impaired glucose tolerance, and open/closeEndocrinology Click to
eventually weight loss due to protein catabolism. open/closeBlood gases Click to
Change Password Surgical cure rate is as low as 5%, due to metastases open/closeCSF
many years prior to presentation. Octreotide improves
Sign Out
the skin rash but may have a detrimental effect on
glucose control; combination chemotherapy with
streptazocin and 5-fluoruracil (5-FU) gives good
palliative results, as does hepatic embolisation.

1423

© 2011 PasTest Ltd | About Us | Contact Us | Help

file:///E|/Shakhawan/Endocrinology/68a.htm[3/13/2012 7:04:37 PM]


MyPasTest

Main Navigation
Question Browser: MRCP 1
Home
Subscriptions Question Browser Exam Builder Saved Exams
» MRCP 1 Session Progress
• Question Browser
Questions Correct 3
• Timed Test A 52-year-old woman is referred for opinion, she has been
diagnosed with type-2 diabetes mellitus and is losing Questions Incorrect 1
• Mock Exam
weight, her GP is thinking about insulin therapy. A Questions Total 4
• Past Papers normochromic, normocytic anaemia is noted. On
Questions Percentage 75 %
• Random Questions examination she has angular stomatitis and a well-
demarcated erythematous rash in her groin, extending to More
• My Performance
her lower limbs, buttocks and perineum.
• Media Bank
What is the underlying diagnosis?
• New Multimedia
Online Extras Poorly controlled type-2 diabetes Reference: Normal Values
Library Latent type-1 diabetes
Click to
Community Somatostatinoma open/closeHaematology
Help Cushing’s disease Click to
open/closeImmunoglobulins
PasTest Store Glucagonoma Click to
open/closeBiochemistry
1424 Click to open/closeDiabetes
My Account
Click to
Profile open/closeEndocrinology
Click to open/closeBlood
Newsletters gases Click to
My Career and Exams open/closeCSF
Order History
Learning Goals
Question Filters

Security

Change Password
Sign Out

© 2011 PasTest Ltd | About Us | Contact Us | Help

file:///E|/Shakhawan/Endocrinology/69.htm[3/13/2012 7:04:40 PM]


MyPasTest

Main Navigation
Question Browser: MRCP 1
Home
Subscriptions Question Browser Exam Builder Saved Exams
» MRCP 1
• Question Browser
• Timed Test A 52-year-old woman is referred for opinion, she has been diagnosed with type-2
• Mock Exam diabetes mellitus and is losing weight, her GP is thinking about insulin therapy. A
normochromic, normocytic anaemia is noted. On examination she has angular
• Past Papers stomatitis and a well-demarcated erythematous rash in her groin, extending to her
• Random Questions lower limbs, buttocks and perineum.
• My Performance What is the underlying diagnosis?
• Media Bank
• New Multimedia Poorly controlled type-2 diabetes

Online Extras Latent type-1 diabetes

Library Somatostatinoma

Community Cushing’s disease

Help Glucagonoma Your answer

PasTest Store

My Account
The skin rash, necrolytic migratory erythema, glucose intolerance, a
Profile normochromic, normocytic anaemia, weight loss and angular stomatitis are all
highly suggestive of glucagonoma. The annual incidence of glucagonoma is
Newsletters estimated at 1/20 million. Over 70% of glucagonomas are malignant, but because
My Career and Exams of their indolent presentation, the diagnosis is often overlooked for many years.
Due to insulin antagonism over 90% are associated with impaired glucose
Order History
tolerance, and eventually weight loss due to protein catabolism. Surgical cure rate
Learning Goals is as low as 5%, as metastases occur many years prior to presentation. Octreotide
Question Filters improves the skin rash but may have a detrimental effect on glucose control;
combination chemotherapy with streptazocin and 5-fluoruracil (5-FU) gives good
palliative results, as does hepatic embolisation.
Security

Change Password 1424


Sign Out

file:///E|/Shakhawan/Endocrinology/69a.htm[3/13/2012 7:04:41 PM]


MyPasTest

Main Navigation
Question Browser: MRCP 1
Home
Subscriptions Question Browser Exam Builder Saved Exams
» MRCP 1 Session Progress
• Question Browser
Questions Correct 4
• Timed Test A 36-year-old man is referred after his first myocardial
infarction for your opinion. He has a total cholesterol level Questions Incorrect 1
• Mock Exam
of 9.5 mmol/l, with normal triglycerides. You note the Questions Total 5
• Past Papers presence of corneal arcus and tendon xanthomas.
Questions Percentage 80 %
• Random Questions
Which of the following genetic factors is not likely to More
• My Performance be a cause of the hypercholesterolaemia?
• Media Bank
• New Multimedia An excess of LDL receptors

Online Extras Not producing any LDL receptors


Reference: Normal Values
Library Failure of the LDL receptors to move to the cell
surface Click to
Community open/closeHaematology
Abnormal receptor-binding to LDL
Help Click to
Inability to internalise LDL for metabolism open/closeImmunoglobulins
PasTest Store
Click to
1425 open/closeBiochemistry
My Account Click to open/closeDiabetes
Click to
Profile open/closeEndocrinology
Click to open/closeBlood
Newsletters gases Click to
My Career and Exams open/closeCSF
Order History
Learning Goals
Question Filters

Security

Change Password
Sign Out

© 2011 PasTest Ltd | About Us | Contact Us | Help

file:///E|/Shakhawan/Endocrinology/70.htm[3/13/2012 7:04:43 PM]


MyPasTest

Main Navigation
Question Browser: MRCP 1
Home
Subscriptions Question Browser Exam Builder Saved Exams
» MRCP 1 Session Progress
• Question Browser
Questions Correct 5
• Timed Test A 36-year-old man is referred after his first myocardial
infarction for your opinion. He has a total cholesterol level Questions Incorrect 1
• Mock Exam
of 9.5 mmol/l, with normal triglycerides. You note the Questions Total 6
• Past Papers presence of corneal arcus and tendon xanthomas.
Questions Percentage 83 %
• Random Questions
Which of the following genetic factors is not likely to More
• My Performance be a cause of the hypercholesterolaemia?
• Media Bank
• New Multimedia An excess of LDL receptors Your answer

Online Extras Not producing any LDL receptors

Library Failure of the LDL receptors to move to


the cell surface
Community
Abnormal receptor-binding to LDL
Help
Inability to internalise LDL for
PasTest Store metabolism

My Account

Profile
Familial hypercholesterolaemia is associated with an
Newsletters autosomal-dominant mutation on the short arm of
My Career and Exams chromosome 19, thereby reducing the number of high-
affinity, low-density lipoprotein (LDL) receptors by up to Reference: Normal Values
Order History
50%. Over 300 mutations have been described so far
Learning Goals and they may be a combination of any of options B–E. Click to open/closeHaematology
Question Filters Patients present with premature coronary heart disease Click to open/closeImmunoglobulins
(CHD) and have a standardised mortality ratio of up to Click to open/closeBiochemistry
nine times that of age- and sex-matched controls. Click to open/closeDiabetes Click to
Security Homozygotes present with CHD in childhood, open/closeEndocrinology Click to
heterozygotes present with CHD at the age of 30+ open/closeBlood gases Click to
Change Password years. open/closeCSF
Sign Out
1425

© 2011 PasTest Ltd | About Us | Contact Us | Help

file:///E|/Shakhawan/Endocrinology/70a.htm[3/13/2012 7:04:44 PM]


MyPasTest

Main Navigation
Question Browser: MRCP 1
Home
Subscriptions Question Browser Exam Builder Saved Exams
» MRCP 1 Session Progress
• Question Browser
Questions Correct 1
• Timed Test A 31-year-old man presents with acute pancreatitis. This is
the third occasion he has presented in the last 5 years. You Questions Incorrect 0
• Mock Exam
follow him up in clinic, and find he has a markedly elevated Questions Total 1
• Past Papers triglyceride fraction and hypercholesterolaemia, but with
Questions Percentage 100 %
• Random Questions HDL and LDL cholesterol within the normal range.
• My Performance More
Which of the following statements best fits his
• Media Bank abnormal lipid picture?
• New Multimedia
Usually improved by thiazide diuretics
Online Extras Reference: Normal Values
Usually improved by glucocorticoids
Library
Usually improved by alcohol Click to
Community open/closeHaematology
Has an autosomal-recessive mode of inheritance
Help Click to
Affects up to 1 in 300 people open/closeImmunoglobulins
PasTest Store
Click to
1426 open/closeBiochemistry
My Account Click to open/closeDiabetes
Click to
Profile open/closeEndocrinology
Click to open/closeBlood
Newsletters gases Click to
My Career and Exams open/closeCSF
Order History
Learning Goals
Question Filters

Security

Change Password
Sign Out

© 2011 PasTest Ltd | About Us | Contact Us | Help

file:///E|/Shakhawan/Endocrinology/71.htm[3/13/2012 7:04:45 PM]


MyPasTest

Main Navigation
Question Browser: MRCP 1
Home
Subscriptions Question Browser Exam Builder Saved Exams
» MRCP 1 Session Progress
• Question Browser
Questions Correct 2
• Timed Test A 31-year-old man presents with acute pancreatitis. This is
the third occasion he has presented in the last 5 years. You Questions Incorrect 0
• Mock Exam
follow him up in clinic, and find he has a markedly elevated Questions Total 2
• Past Papers triglyceride fraction and hypercholesterolaemia, but with
Questions Percentage 100 %
• Random Questions HDL and LDL cholesterol within the normal range.
• My Performance More
Which of the following statements best fits his
• Media Bank abnormal lipid picture?
• New Multimedia
Usually improved by thiazide diuretics
Online Extras
Usually improved by glucocorticoids
Library
Usually improved by alcohol
Community
Has an autosomal-recessive mode of
Help inheritance
PasTest Store Affects up to 1 in 300 people Your answer

My Account

Profile
This is familial hypertriglyceridaemia, an autosomal-
Newsletters dominant condition affecting 1 in 300 people, associated
My Career and Exams with raised very-low-density lipoproteins (VLDL) and
triglyceride levels. It may be exacerbated by alcohol, Reference: Normal Values
Order History
glucocorticoids and thiazide diuretics. Avoidance of
Learning Goals alcohol is advised. Clinical features include eruptive Click to open/closeHaematology
Question Filters xanthomas and lipaemia retinalis. Click to open/closeImmunoglobulins
Click to open/closeBiochemistry
Click to open/closeDiabetes Click to
Security 1426 open/closeEndocrinology Click to
open/closeBlood gases Click to
Change Password open/closeCSF
Sign Out

© 2011 PasTest Ltd | About Us | Contact Us | Help

file:///E|/Shakhawan/Endocrinology/71a.htm[3/13/2012 7:04:47 PM]


MyPasTest

Main Navigation
Question Browser: MRCP 1
Home
Subscriptions Question Browser Exam Builder Saved Exams
» MRCP 1 Session Progress
• Question Browser
Questions Correct 2
• Timed Test A 61-year-old, non-smoking woman with no previous
cardiac history presents from her GP at the cardiac risk- Questions Incorrect 0
• Mock Exam
factor clinic. Her total cholesterol is 9.0 mmol/l. She is Questions Total 2
• Past Papers overweight and has sleep apnoea. On examination you
Questions Percentage 100 %
• Random Questions notice her skin is particularly dry and there appears to be
some evidence of hair loss. More
• My Performance
• Media Bank What is the next step in her management?
• New Multimedia
Screening of family members for
Online Extras hypercholesterolaemia Reference: Normal Values
Library Lifestyle advice and reassurance
Click to
Community Start high-dose statin therapy open/closeHaematology
Help Check her TSH Click to
open/closeImmunoglobulins
PasTest Store Check her fasting blood glucose Click to
open/closeBiochemistry
1427 Click to open/closeDiabetes
My Account
Click to
Profile open/closeEndocrinology
Click to open/closeBlood
Newsletters gases Click to
My Career and Exams open/closeCSF
Order History
Learning Goals
Question Filters

Security

Change Password
Sign Out

© 2011 PasTest Ltd | About Us | Contact Us | Help

file:///E|/Shakhawan/Endocrinology/72.htm[3/13/2012 7:04:48 PM]


MyPasTest

Main Navigation
Question Browser: MRCP 1
Home
Subscriptions Question Browser Exam Builder Saved Exams
» MRCP 1 Session Progress
• Question Browser
Questions Correct 3
• Timed Test A 61-year-old, non-smoking woman with no previous
cardiac history presents from her GP at the cardiac risk- Questions Incorrect 0
• Mock Exam
factor clinic. Her total cholesterol is 9.0 mmol/l. She is Questions Total 3
• Past Papers overweight and has sleep apnoea. On examination you
Questions Percentage 100 %
• Random Questions notice her skin is particularly dry and there appears to be
some evidence of hair loss. More
• My Performance
• Media Bank What is the next step in her management?
• New Multimedia
Screening of family members for
Online Extras hypercholesterolaemia
Library Lifestyle advice and reassurance
Community Start high-dose statin therapy
Help Check her TSH Your answer
PasTest Store Check her fasting blood glucose

My Account

Profile
The suspicion with the history of dry skin, hair loss,
Newsletters obesity and sleep apnoea is that she has
My Career and Exams hypothyroidism. This would be the cause of her
secondary hypercholesterolaemia. Frank hypothyroidism Reference: Normal Values
Order History
is said to occur in 4% of patients with dyslipidaemias,
Learning Goals and a further 10% of patients with dyslipidaemia have Click to open/closeHaematology
Question Filters a raised TSH level, but with a free T 4 still within the Click to open/closeImmunoglobulins
normal range. Total cholesterol often improves Click to open/closeBiochemistry
somewhat with thyroxine therapy, but statins may be Click to open/closeDiabetes Click to
Security required in addition. open/closeEndocrinology Click to
open/closeBlood gases Click to
Change Password open/closeCSF
Sign Out 1427

© 2011 PasTest Ltd | About Us | Contact Us | Help

file:///E|/Shakhawan/Endocrinology/72a.htm[3/13/2012 7:04:49 PM]


MyPasTest

Main Navigation
Question Browser: MRCP 1
Home
Subscriptions Question Browser Exam Builder Saved Exams
» MRCP 1 Session Progress
• Question Browser
Questions Correct 3
• Timed Test You review a 67-year-old man in the lipid clinic. He is
taking pravastatin 40 mg/day. Which of the following Questions Incorrect 0
• Mock Exam
drugs should be used with caution? Questions Total 3
• Past Papers
Questions Percentage 100 %
• Random Questions Warfarin
• My Performance Erythromycin More
• Media Bank Digoxin
• New Multimedia Antacids
Online Extras Antifungals Reference: Normal Values
Library
1428 Click to
Community open/closeHaematology
Help Click to
open/closeImmunoglobulins
PasTest Store
Click to
open/closeBiochemistry
My Account Click to open/closeDiabetes
Click to
Profile open/closeEndocrinology
Click to open/closeBlood
Newsletters gases Click to
My Career and Exams open/closeCSF
Order History
Learning Goals
Question Filters

Security

Change Password
Sign Out

© 2011 PasTest Ltd | About Us | Contact Us | Help

file:///E|/Shakhawan/Endocrinology/73.htm[3/13/2012 7:04:51 PM]


MyPasTest

Main Navigation
Question Browser: MRCP 1
Home
Subscriptions Question Browser Exam Builder Saved Exams
» MRCP 1 Session Progress
• Question Browser
Questions Correct 3
• Timed Test You review a 67-year-old man in the lipid clinic. He is
taking pravastatin 40 mg/day. Which of the following Questions Incorrect 1
• Mock Exam
drugs should be used with caution? Questions Total 4
• Past Papers
Questions Percentage 75 %
• Random Questions Warfarin
• My Performance Erythromycin Correct answer More
• Media Bank Digoxin
• New Multimedia Antacids
Online Extras Antifungals Your answer
Library
Community
Help
PasTest Store Warfarin and digoxin have well-recognised interactions
with atorvastatin and simvastatin. Antacid and
antifungal agents are known to interact with
My Account atorvastatin. Erythromycin has known interactions with
simvastatin, fluvastatin, atorvastatin and pravastatin.
Profile All statins interact with ciclosporin and nicotinic acid
and should be used with caution in patients on fibrate
Newsletters
therapy as the risk of rhabdomyolysis may be increased.
My Career and Exams
Order History Reference: Normal Values
1428
Learning Goals
Click to open/closeHaematology
Question Filters Click to open/closeImmunoglobulins
Click to open/closeBiochemistry
Click to open/closeDiabetes Click to
Security open/closeEndocrinology Click to
open/closeBlood gases Click to
Change Password open/closeCSF
Sign Out

© 2011 PasTest Ltd | About Us | Contact Us | Help

file:///E|/Shakhawan/Endocrinology/73a.htm[3/13/2012 7:04:52 PM]


MyPasTest

Main Navigation
Question Browser: MRCP 1
Home
Subscriptions Question Browser Exam Builder Saved Exams
» MRCP 1 Session Progress
• Question Browser
Questions Correct 3
• Timed Test The presence of galactorrhoea is MOST suggestive of
which one of the following conditions? Questions Incorrect 1
• Mock Exam
Questions Total 4
• Past Papers
Turner’s syndrome Questions Percentage 75 %
• Random Questions
Polycystic ovary disease
• My Performance More
Hypothyroidism
• Media Bank
Sheehan’s syndrome
• New Multimedia
Bromocriptine therapy
Online Extras Reference: Normal Values
Library 1605
Click to
Community open/closeHaematology
Help Click to
open/closeImmunoglobulins
PasTest Store
Click to
open/closeBiochemistry
My Account Click to open/closeDiabetes
Click to
Profile open/closeEndocrinology
Click to open/closeBlood
Newsletters gases Click to
My Career and Exams open/closeCSF
Order History
Learning Goals
Question Filters

Security

Change Password
Sign Out

© 2011 PasTest Ltd | About Us | Contact Us | Help

file:///E|/Shakhawan/Endocrinology/74.htm[3/13/2012 7:04:54 PM]


MyPasTest

Main Navigation
Question Browser: MRCP 1
Home
Subscriptions Question Browser Exam Builder Saved Exams
» MRCP 1 Session Progress
• Question Browser
Questions Correct 4
• Timed Test The presence of galactorrhoea is MOST suggestive of
which one of the following conditions? Questions Incorrect 1
• Mock Exam
Questions Total 5
• Past Papers
Turner’s syndrome Questions Percentage 80 %
• Random Questions
Polycystic ovary disease
• My Performance More
Hypothyroidism Your answer
• Media Bank
Sheehan’s syndrome
• New Multimedia
Bromocriptine therapy
Online Extras
Library
Community
Galactorrhoea is nonpuerperial expression of milk.
Help Hyperprolactinaemia causes galactorrhoea and
PasTest Store amenorrhoea, which may result from a prolactin
secreting pituitary tumour or hyperplasia.
Hypothyroidism is a cause of raised prolactin levels. In
My Account acromegaly one-third of patients have mild elevation of
prolactin levels resulting in galactorrhoea, amenorrhoea
Profile and decreased libido. Similar elevation of prolactin
Newsletters levels occurs in a small percentage of patients with
primary hypothyroidism. Sheehan’s syndrome is a
My Career and Exams primary hypopituitarism due to ischaemic necrosis of
Order History the pituitary gland caused by postpartum haemorrhage; Reference: Normal Values
it is characterised by failure of postpartum lactation and
Learning Goals failure to resume normal cyclic menstruation. Turner’s Click to open/closeHaematology
Question Filters syndrome (ovarian dysgenesis) causes primary Click to open/closeImmunoglobulins
amenorrhoea with poor development of the breast. Click to open/closeBiochemistry
Bromocriptine is a dopaminergic agent that has an Click to open/closeDiabetes Click to
Security inhibitory effect on prolactin and is frequently used to open/closeEndocrinology Click to
treat hyperprolactinaemia. Galactorrhoea and raised open/closeBlood gases Click to
Change Password prolactin levels may also be seen in PCOS. open/closeCSF
Sign Out
1605

© 2011 PasTest Ltd | About Us | Contact Us | Help

file:///E|/Shakhawan/Endocrinology/74a.htm[3/13/2012 7:04:55 PM]


MyPasTest

Main Navigation
Question Browser: MRCP 1
Home
Subscriptions Question Browser Exam Builder Saved Exams
» MRCP 1 Session Progress
• Question Browser
Questions Correct 4
• Timed Test Which one of the following proteins is most likely to
be associated with very high levels of plasma Questions Incorrect 1
• Mock Exam
chylomicrons? Questions Total 5
• Past Papers
Questions Percentage 80 %
• Random Questions Apoprotein E
• My Performance Apoprotein CII More
• Media Bank Apoprotein AII
• New Multimedia Lipoprotein B
Online Extras LDL receptor Reference: Normal Values
Library
1606 Click to
Community open/closeHaematology
Help Click to
open/closeImmunoglobulins
PasTest Store
Click to
open/closeBiochemistry
My Account Click to open/closeDiabetes
Click to
Profile open/closeEndocrinology
Click to open/closeBlood
Newsletters gases Click to
My Career and Exams open/closeCSF
Order History
Learning Goals
Question Filters

Security

Change Password
Sign Out

© 2011 PasTest Ltd | About Us | Contact Us | Help

file:///E|/Shakhawan/Endocrinology/75.htm[3/13/2012 7:04:57 PM]


MyPasTest

Main Navigation
Question Browser: MRCP 1
Home
Subscriptions Question Browser Exam Builder Saved Exams
» MRCP 1 Session Progress
• Question Browser
Questions Correct 5
• Timed Test Which one of the following proteins is most likely to
be associated with very high levels of plasma Questions Incorrect 1
• Mock Exam
chylomicrons? Questions Total 6
• Past Papers
Questions Percentage 83 %
• Random Questions Apoprotein E
• My Performance Apoprotein CII Your answer More
• Media Bank Apoprotein AII
• New Multimedia Lipoprotein B
Online Extras LDL receptor
Library
Community
Help
PasTest Store Dietary triglycerides in cholesterol are packaged by
gastrointestinal epithelial cells into large lipoprotein
particles called chylomicrons. After secretion into the
My Account intestinal lymph and passage into the general
circulation, chylomicrons bind to the enzyme lipoprotein
Profile lipase, which is located on endothelial surfaces. This
enzyme is activated by a protein contained in the
Newsletters
chylomicron, apoprotein CII, liberating free fatty acids
My Career and Exams and monoglycerides, which then pass through the
Order History endothelial cells and enter adipocyte or muscle cells. Reference: Normal Values
Therefore, complete inactivation of either lipoprotein
Learning Goals lipase or apoprotein CII as a result of the inheritance of Click to open/closeHaematology
Question Filters two defective copies of the relevant gene results in an Click to open/closeImmunoglobulins
accumulation of chylomicrons (type I lipoprotein Click to open/closeBiochemistry
elevation) owing to failure of conversion to the Click to open/closeDiabetes Click to
Security chylomicron remnant particle. Patients with familial open/closeEndocrinology Click to
lipoprotein lipase deficiency usually present in infancy open/closeBlood gases Click to
Change Password with recurrent attacks of abdominal pain caused by open/closeCSF
Sign Out pancreatitis. They also have eruptive xanthomas
resulting from triglyceride deposition. Treatment should
consist of a low-fat diet that may be supplemented by
medium-chain triglycerides, which are not incorporated
into chylomicrons. The absence of functional apoprotein
CII, with consequent failure to activate lipoprotein
lipase, presents with a similar phenotype, although the
affected patients are typically detected at a somewhat
later age than are patients with familial lipoprotein
lipase deficiency.

1606

© 2011 PasTest Ltd | About Us | Contact Us | Help

file:///E|/Shakhawan/Endocrinology/75a.htm[3/13/2012 7:04:58 PM]


MyPasTest

Main Navigation
Question Browser: MRCP 1
Home
Subscriptions Question Browser Exam Builder Saved Exams
» MRCP 1 Session Progress
• Question Browser
Questions Correct 5
• Timed Test A 65-year-old woman known to have chronic low back pain
notices severe sharp pain in the left groin after a minor fall Questions Incorrect 1
• Mock Exam
and is unable to walk. Left neck of femur fracture is Questions Total 6
• Past Papers identified on radiological examination. Routine laboratory
Questions Percentage 83 %
• Random Questions evaluation discloses a serum calcium concentration of 1.9
mmol/l, a serum phosphorus concentration of 0.68 mmol/l More
• My Performance
and increased serum alkaline phosphatase activity. The
• Media Bank serum parathyroid hormone level was subsequently found
• New Multimedia to be elevated. The most likely diagnosis is?

Online Extras Primary hyperparathyroidism Reference: Normal Values


Library Hypervitaminosis D
Click to
Community Paget’s disease of bone open/closeHaematology
Help Osteoporosis Click to
open/closeImmunoglobulins
PasTest Store Vitamin D deficiency Click to
open/closeBiochemistry
1607 Click to open/closeDiabetes
My Account
Click to
Profile open/closeEndocrinology
Click to open/closeBlood
Newsletters gases Click to
My Career and Exams open/closeCSF
Order History
Learning Goals
Question Filters

Security

Change Password
Sign Out

© 2011 PasTest Ltd | About Us | Contact Us | Help

file:///E|/Shakhawan/Endocrinology/76.htm[3/13/2012 7:04:59 PM]


MyPasTest

Main Navigation
Question Browser: MRCP 1
Home
Subscriptions Question Browser Exam Builder Saved Exams
» MRCP 1 Session Progress
• Question Browser
Questions Correct 6
• Timed Test A 65-year-old woman known to have chronic low back pain
notices severe sharp pain in the left groin after a minor fall Questions Incorrect 1
• Mock Exam
and is unable to walk. Left neck of femur fracture is Questions Total 7
• Past Papers identified on radiological examination. Routine laboratory
Questions Percentage 85 %
• Random Questions evaluation discloses a serum calcium concentration of 1.9
mmol/l, a serum phosphorus concentration of 0.68 mmol/l More
• My Performance
and increased serum alkaline phosphatase activity. The
• Media Bank serum parathyroid hormone level was subsequently found
• New Multimedia to be elevated. The most likely diagnosis is?

Online Extras Primary hyperparathyroidism


Library Hypervitaminosis D
Community Paget’s disease of bone
Help Osteoporosis
PasTest Store Vitamin D deficiency Your answer

My Account

Profile
The combination of hypocalcaemia and
Newsletters
hypophosphataemia points to the diagnosis of
My Career and Exams osteomalacia and vitamin D deficiency. Dietary
Order History deficiency and malabsorption are common causes of Reference: Normal Values
vitamin D deficiency. Primary hyperparathyroidism and
Learning Goals hypervitaminosis D are associated with hypercalcaemia Click to open/closeHaematology
Question Filters rather than hypocalcaemia. Paget’s disease is Click to open/closeImmunoglobulins
associated with increased risk of fracture and increased Click to open/closeBiochemistry
serum alkaline phosphatase activity, but the serum Click to open/closeDiabetes Click to
Security calcium is usually within normal limits. Osteoporosis is open/closeEndocrinology Click to
the most common cause of fracture of neck of femur open/closeBlood gases Click to
Change Password and is not associated with any specific abnormality in open/closeCSF
Sign Out the standard bone biochemistry profile.

1607

© 2011 PasTest Ltd | About Us | Contact Us | Help

file:///E|/Shakhawan/Endocrinology/76a.htm[3/13/2012 7:05:01 PM]


MyPasTest

Main Navigation
Question Browser: MRCP 1
Home
Subscriptions Question Browser Exam Builder Saved Exams
» MRCP 1 Session Progress
• Question Browser
Questions Correct 6
• Timed Test In glucagonoma the MOST likely associated skin
lesion is? Questions Incorrect 1
• Mock Exam
Questions Total 7
• Past Papers
Erythema chronicum migrans Questions Percentage 85 %
• Random Questions
Acanthosis nigricans
• My Performance More
Panniculitis
• Media Bank
Ichthyosis
• New Multimedia
Necrolytic migratory erythema
Online Extras Reference: Normal Values
Library 1608
Click to
Community open/closeHaematology
Help Click to
open/closeImmunoglobulins
PasTest Store
Click to
open/closeBiochemistry
My Account Click to open/closeDiabetes
Click to
Profile open/closeEndocrinology
Click to open/closeBlood
Newsletters gases Click to
My Career and Exams open/closeCSF
Order History
Learning Goals
Question Filters

Security

Change Password
Sign Out

© 2011 PasTest Ltd | About Us | Contact Us | Help

file:///E|/Shakhawan/Endocrinology/77.htm[3/13/2012 7:05:02 PM]


MyPasTest

Main Navigation
Question Browser: MRCP 1
Home
Subscriptions Question Browser Exam Builder Saved Exams
» MRCP 1 Session Progress
• Question Browser
Questions Correct 7
• Timed Test In glucagonoma the MOST likely associated skin
lesion is? Questions Incorrect 1
• Mock Exam
Questions Total 8
• Past Papers
Erythema chronicum migrans Questions Percentage 87 %
• Random Questions
Acanthosis nigricans
• My Performance More
Panniculitis
• Media Bank
Ichthyosis
• New Multimedia
Necrolytic migratory erythema Your answer
Online Extras
Library
Community
Help Glucagonoma syndrome (diabetes mellitus, weight loss
PasTest Store and anaemia) is associated with a characteristic skin
rash (necrolytic migratory erythema) in 75% of cases.
The lesion starts as an indurated erythema at the
My Account perineum, face and nose. Within a few days blisters will
cover the surface of the skin which then crust and heal
Profile leaving hyperpigmented skin. This process takes 7–14
days with lesions developing in one area while others
Newsletters
are resolving.
My Career and Exams
Order History Reference: Normal Values
1608
Learning Goals
Click to open/closeHaematology
Question Filters Click to open/closeImmunoglobulins
Click to open/closeBiochemistry
Click to open/closeDiabetes Click to
Security open/closeEndocrinology Click to
open/closeBlood gases Click to
Change Password open/closeCSF
Sign Out

© 2011 PasTest Ltd | About Us | Contact Us | Help

file:///E|/Shakhawan/Endocrinology/77a.htm[3/13/2012 7:05:04 PM]


MyPasTest

Main Navigation
Question Browser: MRCP 1
Home
Subscriptions Question Browser Exam Builder Saved Exams
» MRCP 1 Session Progress
• Question Browser
Questions Correct 7
• Timed Test A 23-year-old woman presents to her GP after the birth of
her second child. She complains of extreme tiredness and a Questions Incorrect 1
• Mock Exam
persistent hoarse voice that she is having problems shaking Questions Total 8
• Past Papers off. Despite breast-feeding her child she is failing to lose
Questions Percentage 87 %
• Random Questions her pregnancy weight. Thyroid autoantibodies are negative.
Her TSH is 12 mU/l, with a free T 4 of 5 pmol/l. There is no More
• My Performance
thyroid tenderness on examination. Her GP notes that her
• Media Bank pulse is only 52 beats per minute.
• New Multimedia
What diagnosis fits best with this clinical picture?
Online Extras Reference: Normal Values
Library Hashimoto’s thyroiditis
Click to
Community Postpartum thyroiditis open/closeHaematology
Help Atrophic hypothyroidism Click to
open/closeImmunoglobulins
PasTest Store Iodine deficiency
Click to
Hyperthyroidism open/closeBiochemistry
My Account Click to open/closeDiabetes
1910 Click to
Profile open/closeEndocrinology
Click to open/closeBlood
Newsletters gases Click to
My Career and Exams open/closeCSF
Order History
Learning Goals
Question Filters

Security

Change Password
Sign Out

© 2011 PasTest Ltd | About Us | Contact Us | Help

file:///E|/Shakhawan/Endocrinology/78.htm[3/13/2012 7:05:05 PM]


MyPasTest

Main Navigation
Question Browser: MRCP 1
Home
Subscriptions Question Browser Exam Builder Saved Exams
» MRCP 1 Session Progress
• Question Browser
Questions Correct 8
• Timed Test A 23-year-old woman presents to her GP after the birth of
her second child. She complains of extreme tiredness and a Questions Incorrect 1
• Mock Exam
persistent hoarse voice that she is having problems shaking Questions Total 9
• Past Papers off. Despite breast-feeding her child she is failing to lose
Questions Percentage 88 %
• Random Questions her pregnancy weight. Thyroid autoantibodies are negative.
Her TSH is 12 mU/l, with a free T 4 of 5 pmol/l. There is no More
• My Performance
thyroid tenderness on examination. Her GP notes that her
• Media Bank pulse is only 52 beats per minute.
• New Multimedia
What diagnosis fits best with this clinical picture?
Online Extras
Library Hashimoto’s thyroiditis
Community Postpartum thyroiditis Your answer
Help Atrophic hypothyroidism
PasTest Store Iodine deficiency
Hyperthyroidism
My Account

Profile
Newsletters
This woman is hypothyroid after the birth of her second
My Career and Exams child. Her thyroid autoantibodies are negative and she
Order History has no signs of autoimmune disease, which might Reference: Normal Values
suggest atrophic hypothyroidism were they present (eg
Learning Goals vitiligo). Iodine deficiency is now rare and tends to Click to open/closeHaematology
Question Filters occur in isolated mountain areas. Postpartum thyroiditis Click to open/closeImmunoglobulins
is usually transient, and may involve hyperthyroidism, Click to open/closeBiochemistry
hypothyroidism or the two in sequence. It is thought to Click to open/closeDiabetes Click to
Security be due to changes in the immune system after open/closeEndocrinology Click to
pregnancy and is histologically a lymphocytic thyroiditis. open/closeBlood gases Click to
Change Password It is usually self-limiting. open/closeCSF
Sign Out

1910

© 2011 PasTest Ltd | About Us | Contact Us | Help

file:///E|/Shakhawan/Endocrinology/78a.htm[3/13/2012 7:05:07 PM]


MyPasTest

Main Navigation
Question Browser: MRCP 1
Home
Subscriptions Question Browser Exam Builder Saved Exams
» MRCP 1 Session Progress
• Question Browser
Questions Correct 8
• Timed Test A 57-year-old woman presents with a feeling of shortness
of breath and choking on lying down. Some 4 months Questions Incorrect 1
• Mock Exam
earlier she had been diagnosed with atrial fibrillation and Questions Total 9
• Past Papers was started on aspirin and digoxin by her GP. On
Questions Percentage 88 %
• Random Questions examination her GP could feel a goitre. Plain radiography
confirmed retrosternal extension, which was presumed to More
• My Performance
be contributing to her shortness of breath. Her TSH level
• Media Bank was less than 0.05 mU/l. Thyroid autoantibodies were
• New Multimedia negative.

Online Extras What diagnosis best fits with this clinical picture? Reference: Normal Values
Library
Hashimoto’s thyroiditis Click to
Community open/closeHaematology
Large, toxic, multinodular goitre
Help Click to
Thyroglossal cyst open/closeImmunoglobulins
PasTest Store
Thyroid carcinoma Click to
open/closeBiochemistry
Grave’s disease Click to open/closeDiabetes
My Account
Click to
Profile 1911 open/closeEndocrinology
Click to open/closeBlood
Newsletters gases Click to
My Career and Exams open/closeCSF
Order History
Learning Goals
Question Filters

Security

Change Password
Sign Out

© 2011 PasTest Ltd | About Us | Contact Us | Help

file:///E|/Shakhawan/Endocrinology/79.htm[3/13/2012 7:05:08 PM]


MyPasTest

Main Navigation
Question Browser: MRCP 1
Home
Subscriptions Question Browser Exam Builder Saved Exams
» MRCP 1 Session Progress
• Question Browser
Questions Correct 9
• Timed Test A 57-year-old woman presents with a feeling of shortness
of breath and choking on lying down. Some 4 months Questions Incorrect 1
• Mock Exam
earlier she had been diagnosed with atrial fibrillation and Questions Total 10
• Past Papers was started on aspirin and digoxin by her GP. On
Questions Percentage 90 %
• Random Questions examination her GP could feel a goitre. Plain radiography
confirmed retrosternal extension, which was presumed to More
• My Performance
be contributing to her shortness of breath. Her TSH level
• Media Bank was less than 0.05 mU/l. Thyroid autoantibodies were
• New Multimedia negative.

Online Extras What diagnosis best fits with this clinical picture?
Library
Hashimoto’s thyroiditis
Community
Large, toxic, multinodular goitre Your answer
Help
Thyroglossal cyst
PasTest Store
Thyroid carcinoma
Grave’s disease
My Account

Profile
Newsletters
My Career and Exams Toxic multinodular goitre usually occurs in women over
55 years of age and is more common than Grave’s Reference: Normal Values
Order History
disease in the elderly. Her atrial fibrillation may well be
Learning Goals related to this. Her goitre is obstructing, with significant Click to open/closeHaematology
Question Filters retrosternal extension, and surgery is the treatment of Click to open/closeImmunoglobulins
choice. Initially, however, she should be rendered Click to open/closeBiochemistry
euthyroid with antithyroid drugs such as carbimazole. Click to open/closeDiabetes Click to
Security open/closeEndocrinology Click to
open/closeBlood gases Click to
Change Password 1911 open/closeCSF
Sign Out

© 2011 PasTest Ltd | About Us | Contact Us | Help

file:///E|/Shakhawan/Endocrinology/79a.htm[3/13/2012 7:05:10 PM]


MyPasTest

Main Navigation
Question Browser: MRCP 1
Home
Subscriptions Question Browser Exam Builder Saved Exams
» MRCP 1 Session Progress
• Question Browser
Questions Correct 9
• Timed Test A 62-year-old man presents for review some 3 months
after first being diagnosed with type-2 diabetes. His BMI is Questions Incorrect 1
• Mock Exam
30. Despite having lost about 7 kg in weight, his morning Questions Total 10
• Past Papers blood sugars are still around 9 mmol/l; an Hb A 1C check
Questions Percentage 90 %
• Random Questions was 8.9%. He is hypertensive and taking ramipril, his
• My Performance triglycerides are raised and his HDL cholesterol is low. More
• Media Bank Which therapy for his diabetes would be the best
• New Multimedia initial choice for his hyperglycaemia?
Online Extras Glibenclamide Reference: Normal Values
Library Gliclazide Click to
Community Pioglitazone open/closeHaematology
Help Click to
Metformin
open/closeImmunoglobulins
PasTest Store Acarbose Click to
open/closeBiochemistry
1912 Click to open/closeDiabetes
My Account
Click to
Profile open/closeEndocrinology
Click to open/closeBlood
Newsletters gases Click to
My Career and Exams open/closeCSF
Order History
Learning Goals
Question Filters

Security

Change Password
Sign Out

© 2011 PasTest Ltd | About Us | Contact Us | Help

file:///E|/Shakhawan/Endocrinology/80.htm[3/13/2012 7:05:11 PM]


MyPasTest

Main Navigation
Question Browser: MRCP 1
Home
Subscriptions Question Browser Exam Builder Saved Exams
» MRCP 1 Session Progress
• Question Browser
Questions Correct 10
• Timed Test A 62-year-old man presents for review some 3 months
after first being diagnosed with type-2 diabetes. His BMI is Questions Incorrect 1
• Mock Exam
30. Despite having lost about 7 kg in weight, his morning Questions Total 11
• Past Papers blood sugars are still around 9 mmol/l; an Hb A 1C check
Questions Percentage 90 %
• Random Questions was 8.9%. He is hypertensive and taking ramipril, his
• My Performance triglycerides are raised and his HDL cholesterol is low. More
• Media Bank Which therapy for his diabetes would be the best
• New Multimedia initial choice for his hyperglycaemia?
Online Extras Glibenclamide
Library Gliclazide
Community Pioglitazone
Help Metformin Your answer
PasTest Store Acarbose

My Account

Profile
Newsletters The best initial therapy for this man, who clearly has
the metabolic syndrome, is metformin. This drug
My Career and Exams
should be introduced at a dose of 500 mg per day,
Order History with a gradual increase in the dose over a few weeks Reference: Normal Values
Learning Goals to around 1.5–2 g total daily dose (divided into
morning and evening doses). Click to open/closeHaematology
Question Filters Click to open/closeImmunoglobulins
The UKPDS (United Kingdom Prospective Diabetes Click to open/closeBiochemistry
Study) showed that for macrovascular risk, metformin Click to open/closeDiabetes Click to
Security was superior to sulphonylureas or insulin (a open/closeEndocrinology Click to
statistically significant risk reduction for myocardial open/closeBlood gases Click to
Change Password infarction compared to conventional therapy, which open/closeCSF
was not found in the sulphonylurea or insulin group).
Sign Out
Metformin is a partial insulin sensitiser that works to
reduce hepatic glucose output and also has some
anti-inflammatory action, showing positive effects on
plasminogen-activator inhibitor-1 (PAI-1) in
particular. PAI-1 is associated with an increased
tendency to blood clotting and may be associated with
an increased vascular risk in sufferers of the
metabolic syndrome.

1912

© 2011 PasTest Ltd | About Us | Contact Us | Help

file:///E|/Shakhawan/Endocrinology/80a.htm[3/13/2012 7:05:13 PM]


MyPasTest

Main Navigation
Question Browser: MRCP 1
Home
Subscriptions Question Browser Exam Builder Saved Exams
» MRCP 1 Session Progress
• Question Browser
Questions Correct 10
• Timed Test A 57-year-old man, with a BMI of 30 and a history of
hypertension, dyslipidaemia and type-2 diabetes presents Questions Incorrect 1
• Mock Exam
for review. He has tolerated 2 g/day of metformin well, but Questions Total 11
• Past Papers 2 years after the initial diagnosis of diabetes his blood
Questions Percentage 90 %
• Random Questions sugars are still too high in the morning on occasions, and a
recent HB A 1c was 7.5%. His job entails occasional shift More
• My Performance
work, during which he is unable to eat for long periods.
• Media Bank
• New Multimedia Which would be the most appropriate add-in therapy
to his metformin treatment?
Online Extras Reference: Normal Values
Library Acarbose
Click to
Community Glargine open/closeHaematology
Help Pioglitazone Click to
open/closeImmunoglobulins
PasTest Store Chlorpropamide
Click to
Glibenclamide open/closeBiochemistry
My Account Click to open/closeDiabetes
1913 Click to
Profile open/closeEndocrinology
Click to open/closeBlood
Newsletters gases Click to
My Career and Exams open/closeCSF
Order History
Learning Goals
Question Filters

Security

Change Password
Sign Out

© 2011 PasTest Ltd | About Us | Contact Us | Help

file:///E|/Shakhawan/Endocrinology/81.htm[3/13/2012 7:05:14 PM]


MyPasTest

Main Navigation
Question Browser: MRCP 1
Home
Subscriptions Question Browser Exam Builder Saved Exams
» MRCP 1 Session Progress
• Question Browser
Questions Correct 10
• Timed Test A 57-year-old man, with a BMI of 30 and a history of
hypertension, dyslipidaemia and type-2 diabetes presents Questions Incorrect 2
• Mock Exam
for review. He has tolerated 2 g/day of metformin well, but Questions Total 12
• Past Papers 2 years after the initial diagnosis of diabetes his blood
Questions Percentage 83 %
• Random Questions sugars are still too high in the morning on occasions, and a
recent HB A 1c was 7.5%. His job entails occasional shift More
• My Performance
work, during which he is unable to eat for long periods.
• Media Bank
• New Multimedia Which would be the most appropriate add-in therapy
to his metformin treatment?
Online Extras
Library Acarbose
Community Glargine Your answer
Help Pioglitazone Correct answer
PasTest Store Chlorpropamide
Glibenclamide
My Account

Profile
Newsletters
The choice is between the addition of sulphonylurea
My Career and Exams
therapy, insulin or a glitazone. Sulphonylurea therapy
Order History or insulin may not be appropriate due to his shift Reference: Normal Values
Learning Goals work and risk of hypoglycaemia. In addition, he has
features of the insulin resistance (metabolic) Click to open/closeHaematology
Question Filters syndrome, and so a glitazone may be the more logical Click to open/closeImmunoglobulins
addition. Click to open/closeBiochemistry
Click to open/closeDiabetes Click to
Security Pioglitazone works by increasing glucose uptake into open/closeEndocrinology Click to
skeletal muscle and fat. Pioglitazone binds to the open/closeBlood gases Click to
Change Password PPAR-gamma nuclear receptor (PPAR, peroxisome open/closeCSF
proliferator-activated receptor), which promotes the
Sign Out
transcription of a number of enzymes concerned with
glucose and lipid metabolism. Clinical trials indicate
that by targeting insulin resistance they appear to
have positive effects on other features of the
metabolic syndrome such as blood pressure and HDL
cholesterol.

1913

© 2011 PasTest Ltd | About Us | Contact Us | Help

file:///E|/Shakhawan/Endocrinology/81a.htm[3/13/2012 7:05:16 PM]


MyPasTest

Main Navigation
Question Browser: MRCP 1
Home
Subscriptions Question Browser Exam Builder Saved Exams
» MRCP 1 Session Progress
• Question Browser
Questions Correct 10
• Timed Test A 56-year-old man with type-2 diabetes presents with
background diabetic retinopathy. His HB A 1c has been Questions Incorrect 2
• Mock Exam
consistently above 9% for the past 5 years. Questions Total 12
• Past Papers
Questions Percentage 83 %
• Random Questions Which of the following factors would most worsen
• My Performance prognosis for his retinopathy? More
• Media Bank Rapid improvement in blood glucose levels
• New Multimedia Total cholesterol 5.2 mmol/l
Online Extras Long-term improvements in blood glucose control Reference: Normal Values
Library Triglyceride levels of 2.1 mmol/l
Click to
Community Stopping smoking open/closeHaematology
Help Click to
1914 open/closeImmunoglobulins
PasTest Store
Click to
open/closeBiochemistry
My Account Click to open/closeDiabetes
Click to
Profile open/closeEndocrinology
Click to open/closeBlood
Newsletters gases Click to
My Career and Exams open/closeCSF
Order History
Learning Goals
Question Filters

Security

Change Password
Sign Out

© 2011 PasTest Ltd | About Us | Contact Us | Help

file:///E|/Shakhawan/Endocrinology/82.htm[3/13/2012 7:05:17 PM]


MyPasTest

Main Navigation
Question Browser: MRCP 1
Home
Subscriptions Question Browser Exam Builder Saved Exams
» MRCP 1 Session Progress
• Question Browser
Questions Correct 11
• Timed Test A 56-year-old man with type-2 diabetes presents with
background diabetic retinopathy. His HB A 1c has been Questions Incorrect 2
• Mock Exam
consistently above 9% for the past 5 years. Questions Total 13
• Past Papers
Questions Percentage 84 %
• Random Questions Which of the following factors would most worsen
• My Performance prognosis for his retinopathy? More
• Media Bank Rapid improvement in blood glucose Your answer
• New Multimedia levels
Online Extras Total cholesterol 5.2 mmol/l
Library Long-term improvements in blood
glucose control
Community
Triglyceride levels of 2.1 mmol/l
Help
Stopping smoking
PasTest Store

My Account

Profile Rapid improvement in blood glucose levels may be


Newsletters associated with worsening of diabetic eye disease.
Both the Diabetes Control and Complications Trial
My Career and Exams
(DCCT) in type-1 diabetes and the UKPDS (United
Order History Kingdom Prospective Diabetes Study) in type-2 Reference: Normal Values
Learning Goals diabetes have demonstrated the long-term benefits of
reducing blood sugar levels in controlling future Click to open/closeHaematology
Question Filters microvascular complications. All diabetic patients Click to open/closeImmunoglobulins
should be offered yearly eye screening, preferably Click to open/closeBiochemistry
with a digital retinal camera. Rapidly deteriorating Click to open/closeDiabetes Click to
Security visual acuity, hard exudates encroaching on the open/closeEndocrinology Click to
macula, preproliferative changes or new-vessel open/closeBlood gases Click to
Change Password open/closeCSF
formation are all reasons for early referral to an
Sign Out ophthalmologist.
Diabetic retinopathy has been known for many years
to be the leading cause of blindness in the UK in
people of working age, but progression to blindness is
now slowing and tailing off due to improved
ophthalmic care. Smoking has no effect on the
prognosis of retinopathy.

1914

© 2011 PasTest Ltd | About Us | Contact Us | Help

file:///E|/Shakhawan/Endocrinology/82a.htm[3/13/2012 7:05:18 PM]


MyPasTest

Main Navigation
Question Browser: MRCP 1
Home
Subscriptions Question Browser Exam Builder Saved Exams
» MRCP 1 Session Progress
• Question Browser
Questions Correct 11
• Timed Test A 45-year-old man is to undergo trans-sphenoidal surgery
for resection of a growth hormone-secreting pituitary Questions Incorrect 2
• Mock Exam
adenoma. He asks about the prospect of a cure and you Questions Total 13
• Past Papers explain that he will require biochemical monitoring over the
Questions Percentage 84 %
• Random Questions next few months and years to assess this.
• My Performance More
Which of the following biochemical tests is the best
• Media Bank way to monitor for recurrence?
• New Multimedia
Blood glucose
Online Extras
Serum cortisol
Library
IGF-1 or growth hormone level Correct answer
Community
Prolactin
Help
Thyroid function testing
PasTest Store

My Account

Profile The blood glucose level may be a marker of


Newsletters recurrence via the development of diabetes mellitus,
but in itself is not the best way to monitor. The aim of
My Career and Exams therapy is to keep the growth hormone level below 5
Order History mU/l or insulin-like growth factor-1 (IGF-1) levels Reference: Normal Values
within normal limits. Growth hormone levels above 5
Learning Goals mU/l are associated with a worse prognosis. Quoted Click to open/closeHaematology
Question Filters failure rates for surgery are around 13.3% for Click to open/closeImmunoglobulins
tumours of < 1 cm or 11.1% for tumours > 1 cm Click to open/closeBiochemistry
confined to the sella. Click to open/closeDiabetes Click to
Security open/closeEndocrinology Click to
Treatment failure may be managed with somatostatin
analogues or external-beam radiotherapy. Chronic open/closeBlood gases Click to
Change Password open/closeCSF
somatostatin analogue use may be associated with
Sign Out increased gallstones, and the response to external-
beam radiotherapy is often slow, so neither treatment
is perfect.

1915

© 2011 PasTest Ltd | About Us | Contact Us | Help

file:///E|/Shakhawan/Endocrinology/83.htm[3/13/2012 7:05:20 PM]


MyPasTest

Main Navigation
Question Browser: MRCP 1
Home
Subscriptions Question Browser Exam Builder Saved Exams
» MRCP 1 Session Progress
• Question Browser
Questions Correct 11
• Timed Test A 42-year-old man presents to his GP complaining of
decreased libido. He has also noticed having to shave less Questions Incorrect 2
• Mock Exam
frequently. On the few occasions he has tried to have sex Questions Total 13
• Past Papers he has failed to maintain his erection to penetration. His
Questions Percentage 84 %
• Random Questions visual field testing is normal and he has no medication
history. Serum prolactin levels are 890 mg/l, growh More
• My Performance
hormone and thyroid function levels are normal.
• Media Bank
What diagnosis fits best with this clinical picture?
• New Multimedia
Online Extras Microprolactinoma Reference: Normal Values
Library Macroprolactinoma
Click to
Community Psychogenic impotence open/closeHaematology
Help Hypothyroidism Click to
open/closeImmunoglobulins
PasTest Store Acromegaly Click to
open/closeBiochemistry
1916 Click to open/closeDiabetes
My Account
Click to
Profile open/closeEndocrinology
Click to open/closeBlood
Newsletters gases Click to
My Career and Exams open/closeCSF
Order History
Learning Goals
Question Filters

Security

Change Password
Sign Out

© 2011 PasTest Ltd | About Us | Contact Us | Help

file:///E|/Shakhawan/Endocrinology/84.htm[3/13/2012 7:05:21 PM]


MyPasTest

Main Navigation
Question Browser: MRCP 1
Home
Subscriptions Question Browser Exam Builder Saved Exams
» MRCP 1 Session Progress
• Question Browser
Questions Correct 11
• Timed Test A 42-year-old man presents to his GP complaining of
decreased libido. He has also noticed having to shave less Questions Incorrect 3
• Mock Exam
frequently. On the few occasions he has tried to have sex Questions Total 14
• Past Papers he has failed to maintain his erection to penetration. His
Questions Percentage 78 %
• Random Questions visual field testing is normal and he has no medication
history. Serum prolactin levels are 890 mg/l, growh More
• My Performance
hormone and thyroid function levels are normal.
• Media Bank
What diagnosis fits best with this clinical picture?
• New Multimedia
Online Extras Microprolactinoma Correct answer
Library Macroprolactinoma
Community Psychogenic impotence
Help Hypothyroidism
PasTest Store Acromegaly Your answer

My Account

Profile
Men with prolactinoma tend to present substantially
Newsletters later than women, the latter presenting with cessation
My Career and Exams of periods and galactorrhoea. Macroprolactinoma is
unlikely given his normal visual fields. Prolactinoma is Reference: Normal Values
Order History
the most common pituitary tumour; microadenomas are
Learning Goals more common in women, whereas macroadenomas Click to open/closeHaematology
Question Filters appear more commonly in men. Medical therapy is Click to open/closeImmunoglobulins
usually attempted first with a dopamine agonist, such Click to open/closeBiochemistry
as cabergoline or bromocriptine, with surgery reserved Click to open/closeDiabetes Click to
Security for non-responders. open/closeEndocrinology Click to
open/closeBlood gases Click to
Change Password open/closeCSF
Sign Out 1916

© 2011 PasTest Ltd | About Us | Contact Us | Help

file:///E|/Shakhawan/Endocrinology/84a.htm[3/13/2012 7:05:23 PM]


MyPasTest

Main Navigation
Question Browser: MRCP 1
Home
Subscriptions Question Browser Exam Builder Saved Exams
» MRCP 1 Session Progress
• Question Browser
Questions Correct 11
• Timed Test A 19-year-old student is brought to A&E by his flatmates.
He had been playing squash that afternoon, and while Questions Incorrect 3
• Mock Exam
resting (after having had a pasta meal), complained of Questions Total 14
• Past Papers generalised weakness. He was unable to stand and had to
Questions Percentage 78 %
• Random Questions be carried in by his friends. His potassium level was noted
to be 2.6 mmol/l. Urine screen for diabetic and laxative More
• My Performance
abuse was normal. Apparently, he has had similar attacks
• Media Bank since his early teenage years. Symptoms were aborted by
• New Multimedia potassium chloride.

Online Extras What type of mutation best fits the underlying Reference: Normal Values
Library pathology of this autosomal-dominant condition
involving intermittent paralysis? Click to
Community open/closeHaematology
Help Mutation of a muscle voltage-gated sodium channel Click to
Mutation in a muscle voltage-gated potassium open/closeImmunoglobulins
PasTest Store
channel Click to
open/closeBiochemistry
Mutation in a renal potassium channel Click to open/closeDiabetes
My Account
Mutation in a renal sodium channel Click to
Profile open/closeEndocrinology
Mutation in a muscle voltage-gated calcium channel Click to open/closeBlood
Newsletters gases Click to
My Career and Exams 1917 open/closeCSF
Order History
Learning Goals
Question Filters

Security

Change Password
Sign Out

© 2011 PasTest Ltd | About Us | Contact Us | Help

file:///E|/Shakhawan/Endocrinology/85.htm[3/13/2012 7:05:24 PM]


MyPasTest

Main Navigation
Question Browser: MRCP 1
Home
Subscriptions Question Browser Exam Builder Saved Exams
» MRCP 1 Session Progress
• Question Browser
Questions Correct 12
• Timed Test A 19-year-old student is brought to A&E by his flatmates.
He had been playing squash that afternoon, and while Questions Incorrect 3
• Mock Exam
resting (after having had a pasta meal), complained of Questions Total 15
• Past Papers generalised weakness. He was unable to stand and had to
Questions Percentage 80 %
• Random Questions be carried in by his friends. His potassium level was noted
to be 2.6 mmol/l. Urine screen for diabetic and laxative More
• My Performance
abuse was normal. Apparently, he has had similar attacks
• Media Bank since his early teenage years. Symptoms were aborted by
• New Multimedia potassium chloride.

Online Extras What type of mutation best fits the underlying


Library pathology of this autosomal-dominant condition
involving intermittent paralysis?
Community
Help Mutation of a muscle voltage-gated
sodium channel
PasTest Store
Mutation in a muscle voltage-gated
potassium channel
My Account
Mutation in a renal potassium channel
Profile Mutation in a renal sodium channel
Newsletters Mutation in a muscle voltage-gated Your answer
My Career and Exams calcium channel

Order History Reference: Normal Values


Learning Goals
Click to open/closeHaematology
Question Filters Click to open/closeImmunoglobulins
Hypokalaemic periodic paralysis is related to a muscle
Click to open/closeBiochemistry
calcium-channel mutation (CAClN1A3). It is an
Click to open/closeDiabetes Click to
Security autosomal-dominant condition; attacks usually begin in
open/closeEndocrinology Click to
the teenage years and may remit after around 35 years
open/closeBlood gases Click to
Change Password of age. Potassium is normally below 3 mmol/l during an
open/closeCSF
attack and symptoms resolve with the administration of
Sign Out
potassium chloride. Attacks appear to be precipitated
either by a high carbohydrate meal or by a period of
rest after extreme exercise. Loss of function of muscles
concerning speech, bulbar or generalised weakness may
occur and attacks can last for several hours.

1917

© 2011 PasTest Ltd | About Us | Contact Us | Help

file:///E|/Shakhawan/Endocrinology/85a.htm[3/13/2012 7:05:26 PM]


MyPasTest

Main Navigation
Question Browser: MRCP 1
Home
Subscriptions Question Browser Exam Builder Saved Exams
» MRCP 1 Session Progress
• Question Browser
Questions Correct 12
• Timed Test A 23-year-old woman with type-1 diabetes presents with
an unusual lesion on her shin. It began as a patch of Questions Incorrect 3
• Mock Exam
spreading erythema, but now looks yellow and has begun Questions Total 15
• Past Papers to ulcerate.
Questions Percentage 80 %
• Random Questions
What diagnosis best fits this lesion? More
• My Performance
• Media Bank Acanthosis nigricans
• New Multimedia Dermatitis herpetiformis
Online Extras Granuloma annulare Reference: Normal Values
Library Necrobiosis lipoidica
Click to
Community Xanthoma open/closeHaematology
Help Click to
1918 open/closeImmunoglobulins
PasTest Store
Click to
open/closeBiochemistry
My Account Click to open/closeDiabetes
Click to
Profile open/closeEndocrinology
Click to open/closeBlood
Newsletters gases Click to
My Career and Exams open/closeCSF
Order History
Learning Goals
Question Filters

Security

Change Password
Sign Out

© 2011 PasTest Ltd | About Us | Contact Us | Help

file:///E|/Shakhawan/Endocrinology/86.htm[3/13/2012 7:05:27 PM]


MyPasTest

Main Navigation
Question Browser: MRCP 1
Home
Subscriptions Question Browser Exam Builder Saved Exams
» MRCP 1 Session Progress
• Question Browser
Questions Correct 13
• Timed Test A 23-year-old woman with type-1 diabetes presents with
an unusual lesion on her shin. It began as a patch of Questions Incorrect 3
• Mock Exam
spreading erythema, but now looks yellow and has begun Questions Total 16
• Past Papers to ulcerate.
Questions Percentage 81 %
• Random Questions
What diagnosis best fits this lesion? More
• My Performance
• Media Bank Acanthosis nigricans
• New Multimedia Dermatitis herpetiformis
Online Extras Granuloma annulare
Library Necrobiosis lipoidica Your answer
Community Xanthoma
Help
PasTest Store

My Account Necrobiosis lipoidica occurs in patients with type-1


diabetes, beginning as a patch of erythema that spreads
Profile across the shin, begins to yellow and then may
ulcerate. Acanthosis nigricans is associated with insulin
Newsletters resistance and may occur in those with type-2 diabetes
My Career and Exams and is usually found in the axillas. Granuloma annulare
is normally diffuse in diabetes and occurs as a ring of Reference: Normal Values
Order History
papules. Dermatitis herpetiformis is, of course,
Learning Goals associated with coeliac disease (gluten enteropathy). Click to open/closeHaematology
Question Filters Click to open/closeImmunoglobulins
Click to open/closeBiochemistry
1918 Click to open/closeDiabetes Click to
Security open/closeEndocrinology Click to
open/closeBlood gases Click to
Change Password open/closeCSF
Sign Out

© 2011 PasTest Ltd | About Us | Contact Us | Help

file:///E|/Shakhawan/Endocrinology/86a.htm[3/13/2012 7:05:29 PM]


MyPasTest

Main Navigation
Question Browser: MRCP 1
Home
Subscriptions Question Browser Exam Builder Saved Exams
» MRCP 1 Session Progress
• Question Browser
Questions Correct 13
• Timed Test A 71-year-old man with established sarcoidosis presents for
review. He suffers from a degree of pulmonary fibrosis due Questions Incorrect 3
• Mock Exam
to previous pulmonary infiltration and has been taking Questions Total 16
• Past Papers corticosteroids intermittently. You are asked for a
Questions Percentage 81 %
• Random Questions consultation as his serum calcium concentration is 3.1
mmol/l. His renal function is normal and parathyroid More
• My Performance
hormone is just below the lower end of the normal range.
• Media Bank
What is the most likely cause of his hypercalcaemia?
• New Multimedia
Online Extras Hyperparathyroidism Reference: Normal Values
Library Hypoparathyroidism
Click to
Community Increased hydroxylation of Vitamin D open/closeHaematology
Help Chronic renal failure Click to
open/closeImmunoglobulins
PasTest Store Milk-alkali syndrome Click to
open/closeBiochemistry
1919 Click to open/closeDiabetes
My Account
Click to
Profile open/closeEndocrinology
Click to open/closeBlood
Newsletters gases Click to
My Career and Exams open/closeCSF
Order History
Learning Goals
Question Filters

Security

Change Password
Sign Out

© 2011 PasTest Ltd | About Us | Contact Us | Help

file:///E|/Shakhawan/Endocrinology/87.htm[3/13/2012 7:05:30 PM]


MyPasTest

Main Navigation
Question Browser: MRCP 1
Home
Subscriptions Question Browser Exam Builder Saved Exams
» MRCP 1 Session Progress
• Question Browser
Questions Correct 14
• Timed Test A 71-year-old man with established sarcoidosis presents for
review. He suffers from a degree of pulmonary fibrosis due Questions Incorrect 3
• Mock Exam
to previous pulmonary infiltration and has been taking Questions Total 17
• Past Papers corticosteroids intermittently. You are asked for a
Questions Percentage 82 %
• Random Questions consultation as his serum calcium concentration is 3.1
mmol/l. His renal function is normal and parathyroid More
• My Performance
hormone is just below the lower end of the normal range.
• Media Bank
What is the most likely cause of his hypercalcaemia?
• New Multimedia
Online Extras Hyperparathyroidism
Library Hypoparathyroidism
Community Increased hydroxylation of Vitamin D Your answer
Help Chronic renal failure
PasTest Store Milk-alkali syndrome

My Account

Profile
In this case, parathyroid hormone is virtually in the
Newsletters normal range and his renal function is normal, so there
My Career and Exams is nothing to suggest a diagnosis of the milk–alkali
syndrome. Hypercalcaemia is found in 10% of Reference: Normal Values
Order History
established cases of sarcoid, and may eventually lead to
Learning Goals nephrocalcinosis. The cause is increased 1 a- Click to open/closeHaematology
Question Filters hydroxylation of vitamin D by sarcoid macrophages, in Click to open/closeImmunoglobulins
addition to that taking place in the kidney. Click to open/closeBiochemistry
Click to open/closeDiabetes Click to
Security open/closeEndocrinology Click to
1919 open/closeBlood gases Click to
Change Password open/closeCSF
Sign Out

© 2011 PasTest Ltd | About Us | Contact Us | Help

file:///E|/Shakhawan/Endocrinology/87a.htm[3/13/2012 7:05:31 PM]


MyPasTest

Main Navigation
Question Browser: MRCP 1
Home
Subscriptions Question Browser Exam Builder Saved Exams
» MRCP 1 Session Progress
• Question Browser
Questions Correct 14
• Timed Test You are asked to review a 54-year-old psychiatric patient
by his GP. This patient has been diagnosed with impaired Questions Incorrect 3
• Mock Exam
glucose tolerance. He also has a history of hypertension, Questions Total 17
• Past Papers for which he takes ramipril.
Questions Percentage 82 %
• Random Questions Which of the following drug classes is most well
known as a cause of impaired glucose tolerance? More
• My Performance
• Media Bank Thiazolidinediones
• New Multimedia ACE inhibitors
Online Extras Atypical antipsychotics Reference: Normal Values
Library Biguanides
Click to
Community Sulphonylureas open/closeHaematology
Help Click to
1920 open/closeImmunoglobulins
PasTest Store
Click to
open/closeBiochemistry
My Account Click to open/closeDiabetes
Click to
Profile open/closeEndocrinology
Click to open/closeBlood
Newsletters gases Click to
My Career and Exams open/closeCSF
Order History
Learning Goals
Question Filters

Security

Change Password
Sign Out

© 2011 PasTest Ltd | About Us | Contact Us | Help

file:///E|/Shakhawan/Endocrinology/88.htm[3/13/2012 7:05:33 PM]


MyPasTest

Main Navigation
Question Browser: MRCP 1
Home
Subscriptions Question Browser Exam Builder Saved Exams
» MRCP 1 Session Progress
• Question Browser
Questions Correct 15
• Timed Test You are asked to review a 54-year-old psychiatric patient
by his GP. This patient has been diagnosed with impaired Questions Incorrect 3
• Mock Exam
glucose tolerance. He also has a history of hypertension, Questions Total 18
• Past Papers for which he takes ramipril.
Questions Percentage 83 %
• Random Questions Which of the following drug classes is most well
known as a cause of impaired glucose tolerance? More
• My Performance
• Media Bank Thiazolidinediones
• New Multimedia ACE inhibitors
Online Extras Atypical antipsychotics Your answer
Library Biguanides
Community Sulphonylureas
Help
PasTest Store

My Account Older atypical antipsychotic agents have been


implicated as a cause of impaired glucose tolerance,
Profile and are thought to put those taking them at increased
Newsletters risk of type-2 diabetes.
My Career and Exams Thiazides and β -blockers may also impact negatively
Order History on the incidence of new diabetes, and this Reference: Normal Values
combination of antihypertensives is no longer a first
Learning Goals choice for patients with a high type-2 diabetes risk. Click to open/closeHaematology
Question Filters Ramipril was shown in the HOPE (Healthcare Options Click to open/closeImmunoglobulins
Plan Entitlement) study to reduce progression to Click to open/closeBiochemistry
type-2 diabetes; this is thought to be mediated by a Click to open/closeDiabetes Click to
Security reduction in insulin resistance. The LIFE (Longitudinal open/closeEndocrinology Click to
Interval Follow-up Evaluation) study using losartan open/closeBlood gases Click to
Change Password open/closeCSF
has demonstrated that angiotensin-receptor blockers
Sign Out also probably reduce the incidence of new cases of
type-2 diabetes.
Thiazolidinediones and biguanides work by primarily
reducing insulin resistance, sulphonylureas stimulate
insulin release; all three are used as treatments for
type-2 diabetes.

1920

© 2011 PasTest Ltd | About Us | Contact Us | Help

file:///E|/Shakhawan/Endocrinology/88a.htm[3/13/2012 7:05:34 PM]


MyPasTest

Main Navigation
Question Browser: MRCP 1
Home
Subscriptions Question Browser Exam Builder Saved Exams
» MRCP 1 Session Progress
• Question Browser
Questions Correct 15
• Timed Test You are asked by a GP to review a 16-year-old girl who
presents with primary amenorrhoea. She appears on Questions Incorrect 3
• Mock Exam
examination to have minimal body hair but normal breast Questions Total 18
• Past Papers development. Examination also reveals a blind-ended
Questions Percentage 83 %
• Random Questions vagina. Biochemistry reveals increased LH, normal FSH,
raised estradiol and raised testosterone levels. More
• My Performance
• Media Bank Which diagnosis fits best with this history and
examination?
• New Multimedia
Online Extras Polycystic ovarian syndrome Reference: Normal Values
Library Turner’s syndrome
Click to
Community Asherman’s syndrome open/closeHaematology
Help Testicular feminisation Click to
open/closeImmunoglobulins
PasTest Store Pregnancy Click to
open/closeBiochemistry
1921 Click to open/closeDiabetes
My Account
Click to
Profile open/closeEndocrinology
Click to open/closeBlood
Newsletters gases Click to
My Career and Exams open/closeCSF
Order History
Learning Goals
Question Filters

Security

Change Password
Sign Out

© 2011 PasTest Ltd | About Us | Contact Us | Help

file:///E|/Shakhawan/Endocrinology/89.htm[3/13/2012 7:05:36 PM]


MyPasTest

Main Navigation
Question Browser: MRCP 1
Home
Subscriptions Question Browser Exam Builder Saved Exams
» MRCP 1 Session Progress
• Question Browser
Questions Correct 16
• Timed Test You are asked by a GP to review a 16-year-old girl who
presents with primary amenorrhoea. She appears on Questions Incorrect 3
• Mock Exam
examination to have minimal body hair but normal breast Questions Total 19
• Past Papers development. Examination also reveals a blind-ended
Questions Percentage 84 %
• Random Questions vagina. Biochemistry reveals increased LH, normal FSH,
raised estradiol and raised testosterone levels. More
• My Performance
• Media Bank Which diagnosis fits best with this history and
examination?
• New Multimedia
Online Extras Polycystic ovarian syndrome
Library Turner’s syndrome
Community Asherman’s syndrome
Help Testicular feminisation Your answer
PasTest Store Pregnancy

My Account

Profile
Newsletters This picture fits a diagnosis of testicular feminisation,
in which androgen receptors are defective. This girl,
My Career and Exams although having the external phenotype of a female,
Order History will have the XY karyotype. These individuals are, of Reference: Normal Values
course, infertile. Patients with testicular feminisation
Learning Goals do possess testes, and orchidectomy is advised since Click to open/closeHaematology
Question Filters there is a high risk of testicular malignancy. Click to open/closeImmunoglobulins
Girls with Turner’s syndrome usually have the typical Click to open/closeBiochemistry
Turner’s phenotype of short stature and webbed neck. Click to open/closeDiabetes Click to
Security open/closeEndocrinology Click to
Asherman’s syndrome is amenorrhoea due to uterine
synechiae after infection; there is no history to open/closeBlood gases Click to
Change Password open/closeCSF
suggest this and it can be ruled out on the
Sign Out examination findings.

1921

© 2011 PasTest Ltd | About Us | Contact Us | Help

file:///E|/Shakhawan/Endocrinology/89a.htm[3/13/2012 7:05:37 PM]


MyPasTest

Main Navigation
Question Browser: MRCP 1
Home
Subscriptions Question Browser Exam Builder Saved Exams
» MRCP 1 Session Progress
• Question Browser
Questions Correct 16
• Timed Test A 27-year-old man presents via his GP for review. He
Questions Incorrect 3
• Mock Exam wants to start a relationship but is concerned about his
small phallus. He also has difficulty becoming aroused. On Questions Total 19
• Past Papers examination he is slim and there is gynaecomastia. There is Questions Percentage 84 %
• Random Questions a general paucity of body hair, his penis is small and he has
small testes. Which diagnosis fits best with this history More
• My Performance
and examination?
• Media Bank
• New Multimedia Testicular feminisation
Online Extras Klinefelter’s syndrome Reference: Normal Values
Library Congenital adrenal hyperplasia
True hermaphroditism Click to
Community open/closeHaematology
Help 5 α -reductase deficiency Click to
open/closeImmunoglobulins
PasTest Store
1922 Click to
open/closeBiochemistry
My Account Click to open/closeDiabetes
Click to
Profile open/closeEndocrinology
Click to open/closeBlood
Newsletters gases Click to
My Career and Exams open/closeCSF
Order History
Learning Goals
Question Filters

Security

Change Password
Sign Out

© 2011 PasTest Ltd | About Us | Contact Us | Help

file:///E|/Shakhawan/Endocrinology/90.htm[3/13/2012 7:05:39 PM]


MyPasTest

Main Navigation
Question Browser: MRCP 1
Home
Subscriptions Question Browser Exam Builder Saved Exams
» MRCP 1 Session Progress
• Question Browser
Questions Correct 17
• Timed Test A 27-year-old man presents via his GP for review. He
wants to start a relationship but is concerned about his Questions Incorrect 3
• Mock Exam
small phallus. He also has difficulty becoming aroused. On Questions Total 20
• Past Papers examination he is slim and there is gynaecomastia. There is
Questions Percentage 85 %
• Random Questions a general paucity of body hair, his penis is small and he has
small testes. Which diagnosis fits best with this history More
• My Performance
and examination?
• Media Bank
• New Multimedia Testicular feminisation
Online Extras Klinefelter’s syndrome Your answer
Library Congenital adrenal hyperplasia
Community True hermaphroditism
Help 5 α -reductase deficiency
PasTest Store

My Account
This man is likely to have karyotype XXY and has the
Profile
features of Klinefelter’s syndrome. Individuals with
Newsletters testicular feminisation are phenotypic females.
My Career and Exams Karyotypic females with congenital adrenal
hyperplasia may have clitoromegaly and labial fusion,
Order History as well as ambiguous external genitalia that may Reference: Normal Values
Learning Goals appear male; they do not, however, have testes.
Click to open/closeHaematology
Question Filters True hermaphrodites are vanishingly rare and may be Click to open/closeImmunoglobulins
XX/XY or mosaic in karyotype; they possess both Click to open/closeBiochemistry
testes and ovaries and usually have male or Click to open/closeDiabetes Click to
Security ambiguous external genitalia. open/closeEndocrinology Click to
open/closeBlood gases Click to
Change Password open/closeCSF
Sign Out 1922

© 2011 PasTest Ltd | About Us | Contact Us | Help

file:///E|/Shakhawan/Endocrinology/90a.htm[3/13/2012 7:05:40 PM]


MyPasTest

Main Navigation
Question Browser: MRCP 1
Home
Subscriptions Question Browser Exam Builder Saved Exams
» MRCP 1 Session Progress
• Question Browser
Questions Correct 17
• Timed Test A 54-year-old man presents to the diabetes clinic for
review. He has had type-1 diabetes for 30 years. Recently Questions Incorrect 3
• Mock Exam
he has suffered a number of falls, which he describes as Questions Total 20
• Past Papers attacks where he feels ‘faint’ and loses his footing. He has
Questions Percentage 85 %
• Random Questions suffered from impotence for a number of years and takes
antireflux medication. On examination he has a postural More
• My Performance
drop of 35 mmHg in his blood pressure. Which diagnosis
• Media Bank fits best with this history and examination?
• New Multimedia
Diabetic autonomic neuropathy
Online Extras Reference: Normal Values
Transient ischaemic attacks
Library
Arrhythmia Click to
Community open/closeHaematology
Simple fainting
Help Click to
Somatisation disorder open/closeImmunoglobulins
PasTest Store
Click to
1923 open/closeBiochemistry
My Account Click to open/closeDiabetes
Click to
Profile open/closeEndocrinology
Click to open/closeBlood
Newsletters gases Click to
My Career and Exams open/closeCSF
Order History
Learning Goals
Question Filters

Security

Change Password
Sign Out

© 2011 PasTest Ltd | About Us | Contact Us | Help

file:///E|/Shakhawan/Endocrinology/91.htm[3/13/2012 7:05:42 PM]


MyPasTest

Main Navigation
Question Browser: MRCP 1
Home
Subscriptions Question Browser Exam Builder Saved Exams
» MRCP 1 Session Progress
• Question Browser
Questions Correct 18
• Timed Test A 54-year-old man presents to the diabetes clinic for
review. He has had type-1 diabetes for 30 years. Recently Questions Incorrect 3
• Mock Exam
he has suffered a number of falls, which he describes as Questions Total 21
• Past Papers attacks where he feels ‘faint’ and loses his footing. He has
Questions Percentage 85 %
• Random Questions suffered from impotence for a number of years and takes
antireflux medication. On examination he has a postural More
• My Performance
drop of 35 mmHg in his blood pressure. Which diagnosis
• Media Bank fits best with this history and examination?
• New Multimedia
Diabetic autonomic neuropathy Your answer
Online Extras
Transient ischaemic attacks
Library
Arrhythmia
Community
Simple fainting
Help
Somatisation disorder
PasTest Store

My Account

Profile This man has a marked postural drop, features of GI


tract neuropathy and impotence. Further assessment of
Newsletters
his cardiovascular system is likely to reveal tachycardia
My Career and Exams and impaired cardiovascular response to the Valsalva
Order History manoeuvre. Other symptoms of GI tract involvement Reference: Normal Values
may include diarrhoea and intractable vomiting. Bladder
Learning Goals involvement may result in atonic bladder and problems Click to open/closeHaematology
Question Filters with painless urinary retention and recurrent urinary Click to open/closeImmunoglobulins
tract infections. Click to open/closeBiochemistry
Click to open/closeDiabetes Click to
Security open/closeEndocrinology Click to
1923 open/closeBlood gases Click to
Change Password open/closeCSF
Sign Out

© 2011 PasTest Ltd | About Us | Contact Us | Help

file:///E|/Shakhawan/Endocrinology/91a.htm[3/13/2012 7:05:43 PM]


MyPasTest

Main Navigation
Question Browser: MRCP 1
Home
Subscriptions Question Browser Exam Builder Saved Exams
» MRCP 1 Session Progress
• Question Browser
Questions Correct 18
• Timed Test A 42-year-old woman presents with difficult-to-treat
hypertension. She is taking ramipril, atenolol and Questions Incorrect 3
• Mock Exam
bendrofluazide yet her blood pressure as measured in the Questions Total 21
• Past Papers clinic is still 150/100 mmHg. Her serum potassium
Questions Percentage 85 %
• Random Questions concentration as measured in clinic was 2.9 mmol/l. She
has been weaned off her antihypertensives for a period of More
• My Performance
4–6 weeks; at this time her renin level was noted to be
• Media Bank suppressed and her aldosterone level was above normal.
• New Multimedia Abdominal ultrasound suggests enlargement of the right
adrenal.
Online Extras Reference: Normal Values
Library Which of the following diagnoses is most likely to fit
with this clinical picture? Click to
Community open/closeHaematology
Help Idiopathic hyperaldosteronism Click to
Conn’s adenoma open/closeImmunoglobulins
PasTest Store
Click to
Glucocorticoid suppressible hyperaldosteronism open/closeBiochemistry
My Account Aldosterone-producing carcinoma Click to open/closeDiabetes
Click to
Carcinoid syndrome open/closeEndocrinology
Profile
Click to open/closeBlood
Newsletters 1924 gases Click to
My Career and Exams open/closeCSF
Order History
Learning Goals
Question Filters

Security

Change Password
Sign Out

© 2011 PasTest Ltd | About Us | Contact Us | Help

file:///E|/Shakhawan/Endocrinology/92.htm[3/13/2012 7:05:45 PM]


MyPasTest

Main Navigation
Question Browser: MRCP 1
Home
Subscriptions Question Browser Exam Builder Saved Exams
» MRCP 1 Session Progress
• Question Browser
Questions Correct 19
• Timed Test A 42-year-old woman presents with difficult-to-treat
hypertension. She is taking ramipril, atenolol and Questions Incorrect 3
• Mock Exam
bendrofluazide yet her blood pressure as measured in the Questions Total 22
• Past Papers clinic is still 150/100 mmHg. Her serum potassium
Questions Percentage 86 %
• Random Questions concentration as measured in clinic was 2.9 mmol/l. She
has been weaned off her antihypertensives for a period of More
• My Performance
4–6 weeks; at this time her renin level was noted to be
• Media Bank suppressed and her aldosterone level was above normal.
• New Multimedia Abdominal ultrasound suggests enlargement of the right
adrenal.
Online Extras
Library Which of the following diagnoses is most likely to fit
with this clinical picture?
Community
Help Idiopathic hyperaldosteronism
PasTest Store Conn’s adenoma Your answer
Glucocorticoid suppressible
hyperaldosteronism
My Account
Aldosterone-producing carcinoma
Profile
Carcinoid syndrome
Newsletters
My Career and Exams
Order History Reference: Normal Values
Learning Goals There is controversy about the commonest cause of Click to open/closeHaematology
Question Filters hyporeninaemic hyperaldosteronism, previously 60- Click to open/closeImmunoglobulins
80% were said to be related to an adenoma, but the Click to open/closeBiochemistry
prevailing view now is that 75% may be related to Click to open/closeDiabetes Click to
Security adrenal hyperplasia. Glucocorticoid suppressible open/closeEndocrinology Click to
hyperaldosteronism and aldosterone-producing open/closeBlood gases Click to
Change Password carcinoma make up less than 1% of cases each. open/closeCSF
Sign Out Primary hyperaldosteronism is estimated to be
responsible for 1–2% of cases of hypertension and is
generally asymptomatic. Rarely, there may be muscle
cramps associated with hypokalaemia.
To estimate renin and aldosterone levels accurately it
may be necessary to wean the patient off
antihypertensive drugs for a period of a few weeks,
since most classes of antihypertensives have effects
on the renin–angiotensin axis. CT or MRI scanning
may be used to confirm the adenoma.
After removal of the adenoma, blood pressure is
normal in 70% of patients at 1 year. At 5 years’
postsurgery, 50% of patients remain normotensive.

1924

© 2011 PasTest Ltd | About Us | Contact Us | Help

file:///E|/Shakhawan/Endocrinology/92a.htm[3/13/2012 7:05:46 PM]


MyPasTest

Main Navigation
Question Browser: MRCP 1
Home
Subscriptions Question Browser Exam Builder Saved Exams
» MRCP 1 Session Progress
• Question Browser
Questions Correct 19
• Timed Test An 18-year-old girl is referred by her GP who is concerned
that she may have an underlying endocrine problem. The Questions Incorrect 3
• Mock Exam
girl had been taken to the GP by her mother. She is a good Questions Total 22
• Past Papers student and has just won a place at university. She weighs
Questions Percentage 86 %
• Random Questions only 38 kg (6 stone) and is 1.78 m (5ft 10 inches) tall. She
is emaciated, her skin is dry and she has excessive growth More
• My Performance
of lanugo hair. She has been amenorrhoeic for 9 months.
• Media Bank Her cortisol level is elevated, her free T 4 is normal. She has
• New Multimedia an anaemia and associated reduced white cell and platelet
count. Which of the following diagnoses is most likely
Online Extras Reference: Normal Values
to fit with this clinical picture?
Library
Addison’s disease Click to
Community open/closeHaematology
Help HIV Click to
Occult carcinoma open/closeImmunoglobulins
PasTest Store
Click to
Hypothyroidism open/closeBiochemistry
My Account Anorexia nervosa Click to open/closeDiabetes
Click to
Profile 1925 open/closeEndocrinology
Click to open/closeBlood
Newsletters gases Click to
My Career and Exams open/closeCSF
Order History
Learning Goals
Question Filters

Security

Change Password
Sign Out

© 2011 PasTest Ltd | About Us | Contact Us | Help

file:///E|/Shakhawan/Endocrinology/93.htm[3/13/2012 7:05:48 PM]


MyPasTest

Main Navigation
Question Browser: MRCP 1
Home
Subscriptions Question Browser Exam Builder Saved Exams
» MRCP 1 Session Progress
• Question Browser
Questions Correct 20
• Timed Test An 18-year-old girl is referred by her GP who is concerned
that she may have an underlying endocrine problem. The Questions Incorrect 3
• Mock Exam
girl had been taken to the GP by her mother. She is a good Questions Total 23
• Past Papers student and has just won a place at university. She weighs
Questions Percentage 86 %
• Random Questions only 38 kg (6 stone) and is 1.78 m (5ft 10 inches) tall. She
is emaciated, her skin is dry and she has excessive growth More
• My Performance
of lanugo hair. She has been amenorrhoeic for 9 months.
• Media Bank Her cortisol level is elevated, her free T 4 is normal. She has
• New Multimedia an anaemia and associated reduced white cell and platelet
count. Which of the following diagnoses is most likely
Online Extras
to fit with this clinical picture?
Library
Community Addison’s disease

Help HIV

PasTest Store Occult carcinoma


Hypothyroidism

My Account Anorexia nervosa Your answer

Profile
Newsletters
My Career and Exams
Anorexia nervosa has a female to male
Order History preponderance of 9:1. It is estimated that around Reference: Normal Values
Learning Goals 0.5–1% of American women between 15 and 30
years of age have anorexia, and the numbers are Click to open/closeHaematology
Question Filters likely to be similar in the UK. The aetiology of Click to open/closeImmunoglobulins
anorexia is unknown, but it is likely to be an Click to open/closeBiochemistry
interaction between environmental and genetic Click to open/closeDiabetes Click to
Security factors, American studies report rates of sexual abuse open/closeEndocrinology Click to
as high as 50% in anorexic females. Despite an open/closeBlood gases Click to
Change Password open/closeCSF
association with depressive and other psychiatric
Sign Out illness, demographic studies show no impairment of
cognitive level nor educational achievement in the
group of patients with anorexia nervosa as compared
to the general population.
Indeed such patients may fit the profile of being "high
achievers", which is what is being hinted at in this
question.
Typical laboratory tests include decreased FSH, LH,
oestrogens and 17-OH steroids. Free T 4 and TSH
levels are usually normal. Anaemia with decreased
white and platelet cell count may also occur. There
may be metabolic alkalosis, hypocalcaemia,
hypokalaemia and hypomagnesaemia.
Treatment may involve complex psychotherapy for a
number of years, and referral to a specialist in the
field is recommended.

1925

© 2011 PasTest Ltd | About Us | Contact Us | Help

file:///E|/Shakhawan/Endocrinology/93a.htm[3/13/2012 7:05:49 PM]


MyPasTest

Main Navigation
Question Browser: MRCP 1
Home
Subscriptions Question Browser Exam Builder Saved Exams
» MRCP 1 Session Progress
• Question Browser
Questions Correct 20
• Timed Test A 60-year-old woman is sent by A&E for endocrine review.
During the past 18 months she has suffered two Colle’s Questions Incorrect 3
• Mock Exam
fractures and a fractured neck of her left femur. Results of Questions Total 23
• Past Papers thyroid function testing, serum protein electrophoresis and
Questions Percentage 86 %
• Random Questions serum parathyroid hormone estimation are all normal. Bone
densitometry of the lumbar spine and femoral neck on the More
• My Performance
non-replaced side reveal a bone density within the
• Media Bank osteoporotic range. Which of the following
• New Multimedia interventions would be most appropriate for her?

Online Extras Observe and repeat the densitometry in 12 months Reference: Normal Values
Library Initiate bisphosphonate therapy
Click to
Community Initiate calcium and vitamin D therapy open/closeHaematology
Help Initiate HRT Click to
open/closeImmunoglobulins
PasTest Store Initiate calcium supplementation Click to
open/closeBiochemistry
1926 Click to open/closeDiabetes
My Account
Click to
Profile open/closeEndocrinology
Click to open/closeBlood
Newsletters gases Click to
My Career and Exams open/closeCSF
Order History
Learning Goals
Question Filters

Security

Change Password
Sign Out

© 2011 PasTest Ltd | About Us | Contact Us | Help

file:///E|/Shakhawan/Endocrinology/94.htm[3/13/2012 7:05:51 PM]


MyPasTest

Main Navigation
Question Browser: MRCP 1
Home
Subscriptions Question Browser Exam Builder Saved Exams
» MRCP 1 Session Progress
• Question Browser
Questions Correct 21
• Timed Test A 60-year-old woman is sent by A&E for endocrine review.
During the past 18 months she has suffered two Colle’s Questions Incorrect 3
• Mock Exam
fractures and a fractured neck of her left femur. Results of Questions Total 24
• Past Papers thyroid function testing, serum protein electrophoresis and
Questions Percentage 87 %
• Random Questions serum parathyroid hormone estimation are all normal. Bone
densitometry of the lumbar spine and femoral neck on the More
• My Performance
non-replaced side reveal a bone density within the
• Media Bank osteoporotic range. Which of the following
• New Multimedia interventions would be most appropriate for her?

Online Extras Observe and repeat the densitometry


Library in 12 months
Community Initiate bisphosphonate therapy Your answer
Help Initiate calcium and vitamin D therapy
PasTest Store Initiate HRT
Initiate calcium supplementation
My Account

Profile
Newsletters
This woman has idiopathic osteoporosis, with normal
My Career and Exams thyroid function and parathyroid hormone assay ruling
Order History out hyperthyroidism and hyperparathyroidism. Her Reference: Normal Values
osteoporosis is clearly of clinical significance in that she
Learning Goals has suffered two fractures of the distal radius and a Click to open/closeHaematology
Question Filters fractured neck of a femur during the past 19 months. Click to open/closeImmunoglobulins
Hormone replacement therapy (HRT) would not be Click to open/closeBiochemistry
advised in this age group due to the increased Click to open/closeDiabetes Click to
Security cardiovascular and breast cancer risk. Bisphosphonate open/closeEndocrinology Click to
therapy, which inhibits osteoclast activity, would be the open/closeBlood gases Click to
Change Password best choice in this patient. open/closeCSF
Sign Out

1926

© 2011 PasTest Ltd | About Us | Contact Us | Help

file:///E|/Shakhawan/Endocrinology/94a.htm[3/13/2012 7:05:52 PM]


MyPasTest

Main Navigation
Question Browser: MRCP 1
Home
Subscriptions Question Browser Exam Builder Saved Exams
» MRCP 1 Session Progress
• Question Browser
Questions Correct 21
• Timed Test A 58-year-old woman is taking alendronate for
osteoporosis. She visits the clinic and is keen to discuss the Questions Incorrect 3
• Mock Exam
mechanism of action of this class of drugs as she has been Questions Total 24
• Past Papers studying them on the Internet. Which of the following
Questions Percentage 87 %
• Random Questions options best describes the mode of action of the
bisphosphonate class of agents? More
• My Performance
• Media Bank Inhibits osteoblast activity
• New Multimedia Stimulates osteoblast activity
Online Extras Stimulates osteoclast activity Reference: Normal Values
Library Inhibits osteoclast activity
Click to
Community Increases the bioavailability of vitamin D open/closeHaematology
Help Click to
1927 open/closeImmunoglobulins
PasTest Store
Click to
open/closeBiochemistry
My Account Click to open/closeDiabetes
Click to
Profile open/closeEndocrinology
Click to open/closeBlood
Newsletters gases Click to
My Career and Exams open/closeCSF
Order History
Learning Goals
Question Filters

Security

Change Password
Sign Out

© 2011 PasTest Ltd | About Us | Contact Us | Help

file:///E|/Shakhawan/Endocrinology/95.htm[3/13/2012 7:05:54 PM]


MyPasTest

Main Navigation
Question Browser: MRCP 1
Home
Subscriptions Question Browser Exam Builder Saved Exams
» MRCP 1 Session Progress
• Question Browser
Questions Correct 22
• Timed Test A 58-year-old woman is taking alendronate for
osteoporosis. She visits the clinic and is keen to discuss the Questions Incorrect 3
• Mock Exam
mechanism of action of this class of drugs as she has been Questions Total 25
• Past Papers studying them on the Internet. Which of the following
Questions Percentage 88 %
• Random Questions options best describes the mode of action of the
bisphosphonate class of agents? More
• My Performance
• Media Bank Inhibits osteoblast activity
• New Multimedia Stimulates osteoblast activity
Online Extras Stimulates osteoclast activity
Library Inhibits osteoclast activity Your answer
Community Increases the bioavailability of vitamin
Help D
PasTest Store

My Account
The primary mode of action of the bisphosphonates is
Profile inhibition of osteoclast activity. With the results from
large-scale HRT studies suggesting an increased risk of
Newsletters
carcinoma or cardiovascular disease, they have become
My Career and Exams the first-choice therapy for osteoporosis. They may be
Order History given orally, although some clinicians prefer intermittent Reference: Normal Values
intravenous use. This class of drugs may be associated
Learning Goals with an increased risk of oesophageal ulceration. When Click to open/closeHaematology
Question Filters the class was developed there was initial concern over Click to open/closeImmunoglobulins
prolonged inhibition of bone turnover over a number of Click to open/closeBiochemistry
years, but this appears to have been unfounded. Click to open/closeDiabetes Click to
Security open/closeEndocrinology Click to
open/closeBlood gases Click to
Change Password 1927 open/closeCSF
Sign Out

© 2011 PasTest Ltd | About Us | Contact Us | Help

file:///E|/Shakhawan/Endocrinology/95a.htm[3/13/2012 7:05:55 PM]


MyPasTest

Main Navigation
Question Browser: MRCP 1
Home
Subscriptions Question Browser Exam Builder Saved Exams
» MRCP 1 Session Progress
• Question Browser
Questions Correct 22
• Timed Test A 61-year-old farmer who has long-standing type-1
diabetes is brought to the clinic by his wife. He has been Questions Incorrect 3
• Mock Exam
limping for a while and his wife noticed that his ankle was Questions Total 25
• Past Papers rather abnormally shaped after he stepped out of the
Questions Percentage 88 %
• Random Questions shower. Examination of his right ankle reveals a painless
joint that is warm and swollen. There is crepitus and what More
• My Performance
appears to be palpable bone debris. X-ray reveals gross
• Media Bank joint destruction and apparent dislocation. His CRP and
• New Multimedia white count are normal, the joint aspiration fluid shows no
microbes and historical review of HB A 1c reveals that it has
Online Extras Reference: Normal Values
rarely been below 9%. What is the most likely
Library diagnosis in this case?
Click to
Community open/closeHaematology
Charcot’s ankle
Help Click to
Osteomyelitis open/closeImmunoglobulins
PasTest Store
Old healed fracture Click to
open/closeBiochemistry
Rheumatoid arthritis
My Account Click to open/closeDiabetes
Osteoarthritis Click to
Profile open/closeEndocrinology
1928 Click to open/closeBlood
Newsletters gases Click to
My Career and Exams open/closeCSF
Order History
Learning Goals
Question Filters

Security

Change Password
Sign Out

© 2011 PasTest Ltd | About Us | Contact Us | Help

file:///E|/Shakhawan/Endocrinology/96.htm[3/13/2012 7:05:57 PM]


MyPasTest

Main Navigation
Question Browser: MRCP 1
Home
Subscriptions Question Browser Exam Builder Saved Exams
» MRCP 1 Session Progress
• Question Browser
Questions Correct 23
• Timed Test A 61-year-old farmer who has long-standing type-1
diabetes is brought to the clinic by his wife. He has been Questions Incorrect 3
• Mock Exam
limping for a while and his wife noticed that his ankle was Questions Total 26
• Past Papers rather abnormally shaped after he stepped out of the
Questions Percentage 88 %
• Random Questions shower. Examination of his right ankle reveals a painless
joint that is warm and swollen. There is crepitus and what More
• My Performance
appears to be palpable bone debris. X-ray reveals gross
• Media Bank joint destruction and apparent dislocation. His CRP and
• New Multimedia white count are normal, the joint aspiration fluid shows no
microbes and historical review of HB A 1c reveals that it has
Online Extras
rarely been below 9%. What is the most likely
Library diagnosis in this case?
Community
Charcot’s ankle Your answer
Help
Osteomyelitis
PasTest Store
Old healed fracture
Rheumatoid arthritis
My Account
Osteoarthritis
Profile
Newsletters
My Career and Exams
Order History Charcot’s joint is said to occur in 1 in every 750 Reference: Normal Values
patients with diabetes mellitus, although this is
Learning Goals
increased to 5 in every 100 in those with proven Click to open/closeHaematology
Question Filters neuropathy. However, a Charcot’s joint does not just Click to open/closeImmunoglobulins
occur in those with diabetes but may occur in 20–40% Click to open/closeBiochemistry
of patients with syringomyelia and 5–10% of those with Click to open/closeDiabetes Click to
Security long-standing syphilis (although this condition is now open/closeEndocrinology Click to
very rare). Management in diabetes includes tight open/closeBlood gases Click to
Change Password control of blood glucose, appropriate orthotic open/closeCSF
Sign Out intervention, and there is evidence that
bisphosphonates may slow the pace of joint destruction.

1928

© 2011 PasTest Ltd | About Us | Contact Us | Help

file:///E|/Shakhawan/Endocrinology/96a.htm[3/13/2012 7:05:58 PM]


MyPasTest

Main Navigation
Question Browser: MRCP 1
Home
Subscriptions Question Browser Exam Builder Saved Exams
» MRCP 1 Session Progress
• Question Browser
Questions Correct 23
• Timed Test A 32-year-old merchant banker presents for endocrine
review. Apart from a past history of reflux symptoms, her Questions Incorrect 3
• Mock Exam
previous medical history is unremarkable. She gives a Questions Total 26
• Past Papers history of increasing fatigue, being sometimes unable to
Questions Percentage 88 %
• Random Questions leave her bedroom; she says that she is only able to walk a
few steps without feeling exhausted. She has lost a little More
• My Performance
weight and gone on long-term sick leave from her job. She
• Media Bank feels unable to concentrate, has headaches, intermittent
• New Multimedia sore throats and feels that when she is able to sleep, she
awakes unrefreshed. Full blood count, viscosity, urea and
Online Extras electrolytes, liver function testing, thyroid function testing Reference: Normal Values
Library and a Synacthen test arranged by her GP have all been
normal. On examination in the clinic there are no abnormal Click to
Community open/closeHaematology
physical findings in this normal weight, normal height
Help young woman. What diagnosis fits best with this Click to
clinical picture? open/closeImmunoglobulins
PasTest Store
Click to
Addison’s disease open/closeBiochemistry
My Account Click to open/closeDiabetes
Hypothyroidism
Click to
Profile Chronic fatigue syndrome open/closeEndocrinology
Myasthenia gravis Click to open/closeBlood
Newsletters gases Click to
My Career and Exams Occult malignant disease open/closeCSF
Order History
1929
Learning Goals
Question Filters

Security

Change Password
Sign Out

© 2011 PasTest Ltd | About Us | Contact Us | Help

file:///E|/Shakhawan/Endocrinology/97.htm[3/13/2012 7:05:59 PM]


MyPasTest

Main Navigation
Question Browser: MRCP 1
Home
Subscriptions Question Browser Exam Builder Saved Exams
» MRCP 1 Session Progress
• Question Browser
Questions Correct 24
• Timed Test A 32-year-old merchant banker presents for endocrine
review. Apart from a past history of reflux symptoms, her Questions Incorrect 3
• Mock Exam
previous medical history is unremarkable. She gives a Questions Total 27
• Past Papers history of increasing fatigue, being sometimes unable to
Questions Percentage 88 %
• Random Questions leave her bedroom; she says that she is only able to walk a
few steps without feeling exhausted. She has lost a little More
• My Performance
weight and gone on long-term sick leave from her job. She
• Media Bank feels unable to concentrate, has headaches, intermittent
• New Multimedia sore throats and feels that when she is able to sleep, she
awakes unrefreshed. Full blood count, viscosity, urea and
Online Extras electrolytes, liver function testing, thyroid function testing
Library and a Synacthen test arranged by her GP have all been
normal. On examination in the clinic there are no abnormal
Community
physical findings in this normal weight, normal height
Help young woman. What diagnosis fits best with this
PasTest Store clinical picture?

Addison’s disease
My Account Hypothyroidism
Profile Chronic fatigue syndrome Your answer
Newsletters Myasthenia gravis
My Career and Exams Occult malignant disease
Order History Reference: Normal Values
Learning Goals
Click to open/closeHaematology
Question Filters Click to open/closeImmunoglobulins
The prevalence of chronic fatigue syndrome is estimated Click to open/closeBiochemistry
at 1–3 per 1000 individuals. The predominant age at Click to open/closeDiabetes Click to
Security presentation is young adulthood to middle age, and open/closeEndocrinology Click to
there is a slight female preponderance. Sufferers often open/closeBlood gases Click to
Change Password report a short flu-like illness at the point of onset and open/closeCSF
Sign Out some scientists believe that an initial viral trigger may
be responsible. Conventional investigation and physical
examination proves normal. Initial hopes centred on the
Epstein–Barr virus infection being that trigger, but
concrete proof has not yet been demonstrated.
Education, counselling and goal setting have proved
useful in stimulating recovery, and trials have
demonstrated improvements with amitriptyline use, but
many patients take years to return to normal life.

1929

© 2011 PasTest Ltd | About Us | Contact Us | Help

file:///E|/Shakhawan/Endocrinology/97a.htm[3/13/2012 7:06:01 PM]


MyPasTest

Main Navigation
Question Browser: MRCP 1
Home
Subscriptions Question Browser Exam Builder Saved Exams
» MRCP 1 Session Progress
• Question Browser
Questions Correct 24
• Timed Test A 62-year-old man presents with evidence of
gynaecomastia. He has been taking long-term digitalis and Questions Incorrect 3
• Mock Exam
warfarin therapy for persistent atrial fibrillation. In addition, Questions Total 27
• Past Papers he takes another tablet for heartburn. Results of tests for
Questions Percentage 88 %
• Random Questions androgens, HCG, liver function and thyroid function are all
normal. More
• My Performance
• Media Bank What is the most likely cause of his gynaecomastia?
• New Multimedia
Warfarin therapy
Online Extras Reference: Normal Values
Digoxin therapy
Library
Furosemide Click to
Community open/closeHaematology
Ranitidine
Help Click to
Sodium bicarbonate open/closeImmunoglobulins
PasTest Store
Click to
2214 open/closeBiochemistry
My Account Click to open/closeDiabetes
Click to
Profile open/closeEndocrinology
Click to open/closeBlood
Newsletters gases Click to
My Career and Exams open/closeCSF
Order History
Learning Goals
Question Filters

Security

Change Password
Sign Out

© 2011 PasTest Ltd | About Us | Contact Us | Help

file:///E|/Shakhawan/Endocrinology/98.htm[3/13/2012 7:06:02 PM]


MyPasTest

Main Navigation
Question Browser: MRCP 1
Home
Subscriptions Question Browser Exam Builder Saved Exams
» MRCP 1 Session Progress
• Question Browser
Questions Correct 25
• Timed Test A 62-year-old man presents with evidence of
gynaecomastia. He has been taking long-term digitalis and Questions Incorrect 3
• Mock Exam
warfarin therapy for persistent atrial fibrillation. In addition, Questions Total 28
• Past Papers he takes another tablet for heartburn. Results of tests for
Questions Percentage 89 %
• Random Questions androgens, HCG, liver function and thyroid function are all
normal. More
• My Performance
• Media Bank What is the most likely cause of his gynaecomastia?
• New Multimedia
Warfarin therapy
Online Extras
Digoxin therapy Your answer
Library
Furosemide
Community
Ranitidine
Help
Sodium bicarbonate
PasTest Store

My Account

Profile Digitalis is one of a number of drugs that may be


associated with gynaecomastia. Other causes include
Newsletters oestrogens, cannabis, diamorphine, spironolactone,
My Career and Exams cimetidine, cyproterone, gonadotrophins and some
cytotoxics. Whilst gynaecomastia is occasionally seen Reference: Normal Values
Order History
with ranitidine, this is much less common versus digoxin
Learning Goals or cimetidine. Thyrotoxicosis, liver disease, oestrogen- Click to open/closeHaematology
Question Filters producing tumours, HCG-producing tumours, Click to open/closeImmunoglobulins
starvation/refeeding and carcinoma of the breast are Click to open/closeBiochemistry
also associated with gynaecomastia. Surgical removal is Click to open/closeDiabetes Click to
Security occasionally carried out in younger men. open/closeEndocrinology Click to
open/closeBlood gases Click to
Change Password open/closeCSF
Sign Out 2214

© 2011 PasTest Ltd | About Us | Contact Us | Help

file:///E|/Shakhawan/Endocrinology/98a.htm[3/13/2012 7:06:03 PM]


MyPasTest

Main Navigation
Question Browser: MRCP 1
Home
Subscriptions Question Browser Exam Builder Saved Exams
» MRCP 1 Session Progress
• Question Browser
Questions Correct 25
• Timed Test A 21-year-old university student presents for review. She is
distressed by the fact that she is overweight and is having Questions Incorrect 3
• Mock Exam
to shave or pluck excessive facial hair. She also notices Questions Total 28
• Past Papers that she appears to have more generalised body hair than
Questions Percentage 89 %
• Random Questions other women. On further questioning you elicit a history
that she can sometimes miss a couple of menses, but More
• My Performance
pregnancy testing is consistently negative. Her mother
• Media Bank apparently had similar problems in her youth, took a while
• New Multimedia to conceive and now has type-2 diabetes. Testosterone is
just outside the upper limit of normal range, her LH:FSH
Online Extras ratio is increased and prolactin is normal. Which Reference: Normal Values
Library diagnosis fits best with this woman’s clinical picture?
Click to
Community open/closeHaematology
Polycystic ovarian syndrome (PCOS)
Help Click to
Hyperprolactinaemia
open/closeImmunoglobulins
PasTest Store
Androgen-secreting tumour Click to
Cushing’s disease open/closeBiochemistry
My Account Click to open/closeDiabetes
Type-2 diabetes Click to
Profile open/closeEndocrinology
2216 Click to open/closeBlood
Newsletters gases Click to
My Career and Exams open/closeCSF
Order History
Learning Goals
Question Filters

Security

Change Password
Sign Out

© 2011 PasTest Ltd | About Us | Contact Us | Help

file:///E|/Shakhawan/Endocrinology/99.htm[3/13/2012 7:06:05 PM]


MyPasTest

Main Navigation
Question Browser: MRCP 1
Home
Subscriptions Question Browser Exam Builder Saved Exams
» MRCP 1 Session Progress
• Question Browser
Questions Correct 26
• Timed Test A 21-year-old university student presents for review. She is
distressed by the fact that she is overweight and is having Questions Incorrect 3
• Mock Exam
to shave or pluck excessive facial hair. She also notices Questions Total 29
• Past Papers that she appears to have more generalised body hair than
Questions Percentage 89 %
• Random Questions other women. On further questioning you elicit a history
that she can sometimes miss a couple of menses, but More
• My Performance
pregnancy testing is consistently negative. Her mother
• Media Bank apparently had similar problems in her youth, took a while
• New Multimedia to conceive and now has type-2 diabetes. Testosterone is
just outside the upper limit of normal range, her LH:FSH
Online Extras ratio is increased and prolactin is normal. Which
Library diagnosis fits best with this woman’s clinical picture?
Community
Polycystic ovarian syndrome (PCOS) Your answer
Help
Hyperprolactinaemia
PasTest Store
Androgen-secreting tumour
Cushing’s disease
My Account Type-2 diabetes
Profile
Newsletters
My Career and Exams
Order History PCOS is said to occur in 3% of adult women. Symptoms Reference: Normal Values
usually begin around the time of menarche and the
Learning Goals diagnosis is often made in adolescence. Obesity is Click to open/closeHaematology
Question Filters present in around 40% of sufferers and there is an Click to open/closeImmunoglobulins
increased association with type-2 diabetes due to Click to open/closeBiochemistry
insulin resistance. Biochemical abnormalities Click to open/closeDiabetes Click to
Security characteristically include a mildly raised testosterone open/closeEndocrinology Click to
level and an increased LH:FSH ratio. Treatment includes open/closeBlood gases Click to
Change Password weight loss or metformin therapy. Surgical intervention open/closeCSF
Sign Out with wedge ovarian resection may reduce androgen
secretion and symptoms. Many women just require
reassurance and effective local treatment for hirsutism.
Those who wish to conceive do so with increased
frequency after a trial of metformin therapy, clomifene,
or in some cases gonadotrophins.

2216

© 2011 PasTest Ltd | About Us | Contact Us | Help

file:///E|/Shakhawan/Endocrinology/99a.htm[3/13/2012 7:06:06 PM]


MyPasTest

Main Navigation
Question Browser: MRCP 1
Home
Subscriptions Question Browser Exam Builder Saved Exams
» MRCP 1 Session Progress
• Question Browser
Questions Correct 26
• Timed Test A 21-year-old woman presents for review. She is concerned
on this occasion because she has not had a period for 5 Questions Incorrect 3
• Mock Exam
months. She is 1.76 m in height and weighs only 43.7 kg Questions Total 29
• Past Papers (7 stone). A pregnancy test is negative and thyroid function
Questions Percentage 89 %
• Random Questions testing is normal. Which diagnosis fits best with this
woman’s clinical picture? More
• My Performance
• Media Bank Gonadotrophin deficiency
• New Multimedia Weight-related amenorrhoea
Online Extras Hyperprolactinaemia Reference: Normal Values
Library Primary ovarian failure
Click to
Community Polycystic ovarian syndrome open/closeHaematology
Help Click to
2218 open/closeImmunoglobulins
PasTest Store
Click to
open/closeBiochemistry
My Account Click to open/closeDiabetes
Click to
Profile open/closeEndocrinology
Click to open/closeBlood
Newsletters gases Click to
My Career and Exams open/closeCSF
Order History
Learning Goals
Question Filters

Security

Change Password
Sign Out

© 2011 PasTest Ltd | About Us | Contact Us | Help

file:///E|/Shakhawan/Endocrinology/100.htm[3/13/2012 7:06:08 PM]


MyPasTest

Main Navigation
Question Browser: MRCP 1
Home
Subscriptions Question Browser Exam Builder Saved Exams
» MRCP 1 Session Progress
• Question Browser
Questions Correct 27
• Timed Test A 21-year-old woman presents for review. She is concerned
on this occasion because she has not had a period for 5 Questions Incorrect 3
• Mock Exam
months. She is 1.76 m in height and weighs only 43.7 kg Questions Total 30
• Past Papers (7 stone). A pregnancy test is negative and thyroid function
Questions Percentage 90 %
• Random Questions testing is normal. Which diagnosis fits best with this
woman’s clinical picture? More
• My Performance
• Media Bank Gonadotrophin deficiency
• New Multimedia Weight-related amenorrhoea Your answer
Online Extras Hyperprolactinaemia
Library Primary ovarian failure
Community Polycystic ovarian syndrome
Help
PasTest Store

My Account A woman needs to maintain a minimum body weight for


menstruation, and amenorrhoea may even be seen at
Profile weights considered to be at the lower end of the normal
range. The biochemical picture may be indistinguishable
Newsletters
from gonadotrophin deficiency. Gaining body weight to
My Career and Exams above the 50th centile for height normally results in the
Order History restoration of menstruation, but if this cannot be Reference: Normal Values
achieved then oestrogen replacement may be
Learning Goals considered. Amenorrhoea is often seen in ballet dancers Click to open/closeHaematology
Question Filters who maintain a low weight and carry out periods of Click to open/closeImmunoglobulins
extreme physical exercise. Click to open/closeBiochemistry
Click to open/closeDiabetes Click to
Security open/closeEndocrinology Click to
2218 open/closeBlood gases Click to
Change Password open/closeCSF
Sign Out

© 2011 PasTest Ltd | About Us | Contact Us | Help

file:///E|/Shakhawan/Endocrinology/100a.htm[3/13/2012 7:06:09 PM]


MyPasTest

Main Navigation
Question Browser: MRCP 1
Home
Subscriptions Question Browser Exam Builder Saved Exams
» MRCP 1 Session Progress
• Question Browser
Questions Correct 27
• Timed Test A 75-year-old woman is admitted in an unconscious state.
Her daughter found her on the floor. On examination in Questions Incorrect 3
• Mock Exam
casualty she is found to have a core temperature of 33 °C Questions Total 30
• Past Papers and also to be in left ventricular failure. Her blood glucose
Questions Percentage 90 %
• Random Questions level is 5.7 mmol/l, random cortisol is elevated. By chance
you also catch the twice weekly run of thyroid function More
• My Performance
testing and her free T 4 is 4.4 pmol/l. A CT scan of her brain
• Media Bank reveals no focal lesion and a cursory assessment reveals no
• New Multimedia gross focal neurology. Which diagnosis fits best with
this woman’s clinical picture?
Online Extras Reference: Normal Values
Library Hypoglycaemia
Click to
Community Addison’s disease open/closeHaematology
Help Profound hypothyroidism Click to
open/closeImmunoglobulins
PasTest Store Massive stroke Click to
Alcohol excess open/closeBiochemistry
My Account Click to open/closeDiabetes
2219 Click to
Profile open/closeEndocrinology
Click to open/closeBlood
Newsletters gases Click to
My Career and Exams open/closeCSF
Order History
Learning Goals
Question Filters

Security

Change Password
Sign Out

© 2011 PasTest Ltd | About Us | Contact Us | Help

file:///E|/Shakhawan/Endocrinology/101.htm[3/13/2012 7:06:10 PM]


MyPasTest

Main Navigation
Question Browser: MRCP 1
Home
Subscriptions Question Browser Exam Builder Saved Exams
» MRCP 1 Session Progress
• Question Browser
Questions Correct 28
• Timed Test A 75-year-old woman is admitted in an unconscious state.
Her daughter found her on the floor. On examination in Questions Incorrect 3
• Mock Exam
casualty she is found to have a core temperature of 33 °C Questions Total 31
• Past Papers and also to be in left ventricular failure. Her blood glucose
Questions Percentage 90 %
• Random Questions level is 5.7 mmol/l, random cortisol is elevated. By chance
you also catch the twice weekly run of thyroid function More
• My Performance
testing and her free T 4 is 4.4 pmol/l. A CT scan of her brain
• Media Bank reveals no focal lesion and a cursory assessment reveals no
• New Multimedia gross focal neurology. Which diagnosis fits best with
this woman’s clinical picture?
Online Extras
Library Hypoglycaemia
Community Addison’s disease
Help Profound hypothyroidism Your answer
PasTest Store Massive stroke
Alcohol excess
My Account

Profile
Newsletters
This woman has a greatly reduced free T 4
My Career and Exams
concentration, is hypothermic, unconscious and has
Order History evidence of associated heart failure. Mortality Reference: Normal Values
Learning Goals associated with this condition used to be as high as
50%, but with modern intensive care management, Click to open/closeHaematology
Question Filters survival has improved. T 3 is usually given via a Click to open/closeImmunoglobulins
nasogastric (NG) tube or iv injection at the rate of 2.5– Click to open/closeBiochemistry
Click to open/closeDiabetes Click to
Security 5 μ g every 8 h, with conversion to T 4 after the patient
open/closeEndocrinology Click to
regains consciousness. There is a risk of precipitating open/closeBlood gases Click to
Change Password heart failure if larger doses are given in the initial open/closeCSF
Sign Out period. Other supportive measures include oxygen
therapy, hydrocortisone iv therapy and glucose infusion,
but none of these have been effectively proven in a
randomised, controlled trial setting.

2219

© 2011 PasTest Ltd | About Us | Contact Us | Help

file:///E|/Shakhawan/Endocrinology/101a.htm[3/13/2012 7:06:12 PM]


MyPasTest

Main Navigation
Question Browser: MRCP 1
Home
Subscriptions Question Browser Exam Builder Saved Exams
» MRCP 1 Session Progress
• Question Browser
Questions Correct 28
• Timed Test A 62-year-old woman undergoes subtotal thyroidectomy
for goitre. Some 12-h postsurgery she calls the duty doctor Questions Incorrect 3
• Mock Exam
and is found to be suffering from pins and needles in her Questions Total 31
• Past Papers hands and carpopedal spasm. An urgent corrected calcium
Questions Percentage 90 %
• Random Questions level is found to be 2.1 mmol/l. What is the most likely
cause of her hypocalcaemia and what is the long- More
• My Performance
term prognosis?
• Media Bank
• New Multimedia She is likely to have permanent hypoparathyroidism
due to surgery
Online Extras Reference: Normal Values
Her hypocalcaemia is likely to be transient due to
Library local trauma at the time of surgery
Click to
Community Her hypocalcaemia is likely to be related to an acute open/closeHaematology
Help fall in thyroid hormone concentration after thyroid Click to
gland removal and she will recover open/closeImmunoglobulins
PasTest Store
Her hypocalcaemia is probably due to a fall in Click to
calcitonin after thyroidectomy open/closeBiochemistry
My Account Click to open/closeDiabetes
Her hypocalcaemia is probably due to coexistent Click to
Profile vitamin D deficiency and is likely to respond to open/closeEndocrinology
treatment with vitamin D Click to open/closeBlood
Newsletters gases Click to
My Career and Exams 2220 open/closeCSF
Order History
Learning Goals
Question Filters

Security

Change Password
Sign Out

© 2011 PasTest Ltd | About Us | Contact Us | Help

file:///E|/Shakhawan/Endocrinology/102.htm[3/13/2012 7:06:13 PM]


MyPasTest

Main Navigation
Question Browser: MRCP 1
Home
Subscriptions Question Browser Exam Builder Saved Exams
» MRCP 1 Session Progress
• Question Browser
Questions Correct 29
• Timed Test A 62-year-old woman undergoes subtotal thyroidectomy
for goitre. Some 12-h postsurgery she calls the duty doctor Questions Incorrect 3
• Mock Exam
and is found to be suffering from pins and needles in her Questions Total 32
• Past Papers hands and carpopedal spasm. An urgent corrected calcium
Questions Percentage 90 %
• Random Questions level is found to be 2.1 mmol/l. What is the most likely
cause of her hypocalcaemia and what is the long- More
• My Performance
term prognosis?
• Media Bank
• New Multimedia She is likely to have permanent
hypoparathyroidism due to surgery
Online Extras
Her hypocalcaemia is likely to be Your answer
Library transient due to local trauma at the time
Community of surgery
Help Her hypocalcaemia is likely to be
related to an acute fall in thyroid
PasTest Store
hormone concentration after thyroid
gland removal and she will recover
My Account Her hypocalcaemia is probably due to
a fall in calcitonin after thyroidectomy
Profile
Her hypocalcaemia is probably due to
Newsletters coexistent vitamin D deficiency and is
My Career and Exams likely to respond to treatment with
vitamin D
Order History Reference: Normal Values
Learning Goals
Click to open/closeHaematology
Question Filters Click to open/closeImmunoglobulins
Click to open/closeBiochemistry
Up to 10% of patients who undergo subtotal Click to open/closeDiabetes Click to
Security thyroidectomy suffer transient hypocalcaemia. open/closeEndocrinology Click to
Thankfully, it becomes permanent hypoparathyroidism open/closeBlood gases Click to
Change Password in less than 1% of patients. Management includes open/closeCSF
Sign Out calcium chloride infusion (slowly) under ECG monitoring,
which should lead to the rapid resolution of symptoms.
Expert thyroid surgeons have now minimised the risk of
permanent hypoparathyroidism.

2220

© 2011 PasTest Ltd | About Us | Contact Us | Help

file:///E|/Shakhawan/Endocrinology/102a.htm[3/13/2012 7:06:15 PM]


MyPasTest

Main Navigation
Question Browser: MRCP 1
Home
Subscriptions Question Browser Exam Builder Saved Exams
» MRCP 1 Session Progress
• Question Browser
Questions Correct 29
• Timed Test You review a 32-year-old woman with relapsed Grave’s
disease. TSH is less than 0.05 mU/l, with a free T 4 of 32.5 Questions Incorrect 3
• Mock Exam
pmol/l. She has severe bilateral thyroid eye disease with Questions Total 32
• Past Papers
marked orbital oedema and proptosis. You are considering Questions Percentage 90 %
• Random Questions radiotherapy as she has failed drug treatment. Which of
• My Performance the following statements best fits the management of More
her thyroid eye disease?
• Media Bank
• New Multimedia Systemic steroids are of no value in managing the
eye disease
Online Extras Reference: Normal Values
Orbital irradiation is commonly used to treat thyroid
Library
eye disease Click to
Community open/closeHaematology
Corrective eye muscle surgery should now be
Help considered Click to
open/closeImmunoglobulins
PasTest Store She should not be given methylcellulose drops as Click to
these may worsen oedema open/closeBiochemistry
My Account Her thyroid eye disease may be worsened by Click to open/closeDiabetes
radioiodine treatment Click to
Profile open/closeEndocrinology
2221 Click to open/closeBlood
Newsletters gases Click to
My Career and Exams open/closeCSF
Order History
Learning Goals
Question Filters

Security

Change Password
Sign Out

© 2011 PasTest Ltd | About Us | Contact Us | Help

file:///E|/Shakhawan/Endocrinology/103.htm[3/13/2012 7:06:16 PM]


MyPasTest

Main Navigation
Question Browser: MRCP 1
Home
Subscriptions Question Browser Exam Builder Saved Exams
» MRCP 1 Session Progress
• Question Browser
Questions Correct 30
• Timed Test You review a 32-year-old woman with relapsed Grave’s
disease. TSH is less than 0.05 mU/l, with a free T 4 of 32.5 Questions Incorrect 3
• Mock Exam
pmol/l. She has severe bilateral thyroid eye disease with Questions Total 33
• Past Papers
marked orbital oedema and proptosis. You are considering Questions Percentage 90 %
• Random Questions radiotherapy as she has failed drug treatment. Which of
• My Performance the following statements best fits the management of More
her thyroid eye disease?
• Media Bank
• New Multimedia Systemic steroids are of no value in
managing the eye disease
Online Extras
Orbital irradiation is commonly used to
Library
treat thyroid eye disease
Community
Corrective eye muscle surgery should
Help now be considered
PasTest Store She should not be given
methylcellulose drops as these may
worsen oedema
My Account
Her thyroid eye disease may be Your answer
Profile worsened by radioiodine treatment
Newsletters
My Career and Exams
Order History Reference: Normal Values
Exacerbation of eye disease is more common after
Learning Goals
radioiodine treatment (15% vs 3% on drug therapy Click to open/closeHaematology
Question Filters alone). Only 5–10% of these worsening cases threaten Click to open/closeImmunoglobulins
sight, but discomfort and deteriorating cosmetic Click to open/closeBiochemistry
appearance may cause much patient anxiety. Systemic Click to open/closeDiabetes Click to
Security steroids and oral non-steroidal anti-inflammatory drugs open/closeEndocrinology Click to
(NSAIDs) may ease the discomfort and decrease open/closeBlood gases Click to
Change Password inflammation in severe symptoms. Orbital irradiation open/closeCSF
Sign Out should be reserved for all but the most severe of cases.
Corrective eye muscle surgery should be delayed until
thyroid eye disease has been stable for at least 6
months, but may be of value in improving diplopia.

2221

© 2011 PasTest Ltd | About Us | Contact Us | Help

file:///E|/Shakhawan/Endocrinology/103a.htm[3/13/2012 7:06:18 PM]


MyPasTest

Main Navigation
Question Browser: MRCP 1
Home
Subscriptions Question Browser Exam Builder Saved Exams
» MRCP 1 Session Progress
• Question Browser
Questions Correct 30
• Timed Test A 34-year-old woman with a long history of type-1
diabetes, microalbuminuria and diabetic eye disease Questions Incorrect 3
• Mock Exam
presents for review. She has been amenorrhoeic for 12 Questions Total 33
• Past Papers months and recent pregnancy tests have been negative.
Questions Percentage 90 %
• Random Questions She has a healthy 1-year-old child but suffered a
postpartum haemorrhage just after his birth. Over the past More
• My Performance
few months she has also been increasingly tired and has
• Media Bank noticed thinning of her pubic and axillary hair. On
• New Multimedia examination she is slim and pale, with small breasts and
thin or largely absent pubic and axillary hair. ACTH, FSH,
Online Extras LH are all low, with TSH just below the normal range. MRI Reference: Normal Values
Library of the pituitary gland reveals an empty sella. What
diagnosis fits best with this clinical picture? Click to
Community open/closeHaematology
Help Prolactinoma Click to
open/closeImmunoglobulins
PasTest Store Weight-related pituitary failure
Click to
Polyglandular syndrome open/closeBiochemistry
My Account Sheehan’s syndrome Click to open/closeDiabetes
Click to
Profile Metastatic carcinoma open/closeEndocrinology
Click to open/closeBlood
Newsletters 2222 gases Click to
My Career and Exams open/closeCSF
Order History
Learning Goals
Question Filters

Security

Change Password
Sign Out

© 2011 PasTest Ltd | About Us | Contact Us | Help

file:///E|/Shakhawan/Endocrinology/104.htm[3/13/2012 7:06:19 PM]


MyPasTest

Main Navigation
Question Browser: MRCP 1
Home
Subscriptions Question Browser Exam Builder Saved Exams
» MRCP 1 Session Progress
• Question Browser
Questions Correct 31
• Timed Test A 34-year-old woman with a long history of type-1
diabetes, microalbuminuria and diabetic eye disease Questions Incorrect 3
• Mock Exam
presents for review. She has been amenorrhoeic for 12 Questions Total 34
• Past Papers months and recent pregnancy tests have been negative.
Questions Percentage 91 %
• Random Questions She has a healthy 1-year-old child but suffered a
postpartum haemorrhage just after his birth. Over the past More
• My Performance
few months she has also been increasingly tired and has
• Media Bank noticed thinning of her pubic and axillary hair. On
• New Multimedia examination she is slim and pale, with small breasts and
thin or largely absent pubic and axillary hair. ACTH, FSH,
Online Extras LH are all low, with TSH just below the normal range. MRI
Library of the pituitary gland reveals an empty sella. What
diagnosis fits best with this clinical picture?
Community
Help Prolactinoma
PasTest Store Weight-related pituitary failure
Polyglandular syndrome
My Account Sheehan’s syndrome Your answer

Profile Metastatic carcinoma

Newsletters
My Career and Exams
Order History Reference: Normal Values
Sheehan’s syndrome is well known to occur in women
Learning Goals due to postpartum haemorrhage and hypovolaemic Click to open/closeHaematology
Question Filters shock, but the risk of it occurring is increased in women Click to open/closeImmunoglobulins
with type-1 diabetes who have microvascular disease, Click to open/closeBiochemistry
and in patients with sickle-cell anaemia. It is said to Click to open/closeDiabetes Click to
Security occur in 1 in 10,000 deliveries. Initial management open/closeEndocrinology Click to
includes immediate steroid therapy with later full open/closeBlood gases Click to
Change Password endocrine assessment and replacement of pituitary- open/closeCSF
Sign Out dependent hormones (eg thyroxine) as required.
Further conception may be difficult and require pulsed
gonadotrophin therapy to restart ovulation. Diabetes
insipidus is not usually associated with the syndrome.

2222

© 2011 PasTest Ltd | About Us | Contact Us | Help

file:///E|/Shakhawan/Endocrinology/104a.htm[3/13/2012 7:06:20 PM]


MyPasTest

Main Navigation
Question Browser: MRCP 1
Home
Subscriptions Question Browser Exam Builder Saved Exams
» MRCP 1
• Question Browser
• Timed Test A 23-year-old woman is admitted to the Casualty department from her office, for the
• Mock Exam third time in the space of 2 months, after having suffered a syncopal attack. On
questioning she admits to feeling very tired over the past few months and being dizzy
• Past Papers on a number of occasions. On examination she looked slim and tanned, her blood
• Random Questions pressure was 110/70 mmHg lying, but dropped to 85/65 mmHg on standing. ACTH is
markedly raised, free thyroxine is below the lower limit of normal and cortisol is low.
• My Performance
Which diagnosis fits best with the clinical picture?
• Media Bank
• New Multimedia Hypothyroidism
Online Extras Primary hypoadrenalism
Library Psychiatric symptoms
Community Hypovolaemia
Help HIV
PasTest Store 2223

My Account

Profile
Newsletters
My Career and Exams
Order History
Learning Goals
Question Filters

Security

Change Password
Sign Out

file:///E|/Shakhawan/Endocrinology/105.htm[3/13/2012 7:06:22 PM]


MyPasTest

Main Navigation
Question Browser: MRCP 1
Home
Subscriptions Question Browser Exam Builder Saved Exams
» MRCP 1 Session Progress
• Question Browser
Questions Correct 32
• Timed Test A 23-year-old woman is admitted to the Casualty
department from her office, for the third time in the space Questions Incorrect 3
• Mock Exam
of 2 months, after having suffered a syncopal attack. On Questions Total 35
• Past Papers questioning she admits to feeling very tired over the past
Questions Percentage 91 %
• Random Questions few months and being dizzy on a number of occasions. On
examination she looked slim and tanned, her blood More
• My Performance
pressure was 110/70 mmHg lying, but dropped to 85/65
• Media Bank mmHg on standing. ACTH is markedly raised, free thyroxine
• New Multimedia is below the lower limit of normal and cortisol is low.
Which diagnosis fits best with the clinical picture?
Online Extras
Library Hypothyroidism
Community Primary hypoadrenalism Your answer
Help Psychiatric symptoms
PasTest Store Hypovolaemia
HIV
My Account

Profile
Newsletters This woman has a marked postural drop, increased
pigmentation due to her high ACTH and a low cortisol,
My Career and Exams making primary hypoadrenalism the most likely
Order History diagnosis. Free thyroxine may also be low at time of Reference: Normal Values
diagnosis, but thyroid hormone replacement should not
Learning Goals be started as this may worsen the adrenal crisis. Click to open/closeHaematology
Question Filters Instead, steroid replacement therapy should be started, Click to open/closeImmunoglobulins
accompanied by fluid replacement, and thyroid function Click to open/closeBiochemistry
reassessed at a later stage. Autoimmune destruction of Click to open/closeDiabetes Click to
Security the adrenal glands is responsible for 80% of cases. open/closeEndocrinology Click to
Tuberculosis accounts for a further 15% of cases. There open/closeBlood gases Click to
Change Password is a female:male predominance of 2:1, and a open/closeCSF
Sign Out prevalence of around 5 per 100,000.

2223

© 2011 PasTest Ltd | About Us | Contact Us | Help

file:///E|/Shakhawan/Endocrinology/105a.htm[3/13/2012 7:06:23 PM]


MyPasTest

Main Navigation
Question Browser: MRCP 1
Home
Subscriptions Question Browser Exam Builder Saved Exams
» MRCP 1 Session Progress
• Question Browser
Questions Correct 32
• Timed Test A 54-year-old publican is referred by his GP for endocrine
assessment. He is obese with a BMI of 32 and has Questions Incorrect 3
• Mock Exam
hypertension, which is poorly controlled on atenolol, Questions Total 35
• Past Papers ramipril and bendrofluazide. A recent fasting blood glucose
Questions Percentage 91 %
• Random Questions test has revealed type-2 diabetes. On examination he looks
cushingoid with a blood pressure of 150/95 mmHg. You More
• My Performance
order a 24-h urinary free cortisol estimation, which turns
• Media Bank out to be just above the normal range. An overnight
• New Multimedia dexamethasone suppression test is also unremarkable.
Which diagnosis fits best with this clinical picture?
Online Extras Reference: Normal Values
Library Cushing’s disease
Click to
Community Pseudo-Cushing’s
open/closeHaematology
Help Simple obesity Click to
Essential hypertension open/closeImmunoglobulins
PasTest Store
Click to
Primary aldosteronism open/closeBiochemistry
My Account Click to open/closeDiabetes
2224 Click to
Profile open/closeEndocrinology
Click to open/closeBlood
Newsletters gases Click to
My Career and Exams open/closeCSF
Order History
Learning Goals
Question Filters

Security

Change Password
Sign Out

© 2011 PasTest Ltd | About Us | Contact Us | Help

file:///E|/Shakhawan/Endocrinology/106.htm[3/13/2012 7:06:25 PM]


MyPasTest

Main Navigation
Question Browser: MRCP 1
Home
Subscriptions Question Browser Exam Builder Saved Exams
» MRCP 1 Session Progress
• Question Browser
Questions Correct 33
• Timed Test A 54-year-old publican is referred by his GP for endocrine
assessment. He is obese with a BMI of 32 and has Questions Incorrect 3
• Mock Exam
hypertension, which is poorly controlled on atenolol, Questions Total 36
• Past Papers ramipril and bendrofluazide. A recent fasting blood glucose
Questions Percentage 91 %
• Random Questions test has revealed type-2 diabetes. On examination he looks
cushingoid with a blood pressure of 150/95 mmHg. You More
• My Performance
order a 24-h urinary free cortisol estimation, which turns
• Media Bank out to be just above the normal range. An overnight
• New Multimedia dexamethasone suppression test is also unremarkable.
Which diagnosis fits best with this clinical picture?
Online Extras
Library Cushing’s disease
Community Pseudo-Cushing’s Your answer
Help Simple obesity
PasTest Store Essential hypertension
Primary aldosteronism
My Account

Profile
Newsletters Obese patients who consume alcohol to chronic
excess may acquire a cushingoid appearance. In this
My Career and Exams
case, this man’s occupation as a publican suggests
Order History that he may have easy access to alcohol. His two Reference: Normal Values
Learning Goals screening tests for Cushing’s disease – the
dexamethasone suppression test and 24-h urinary Click to open/closeHaematology
Question Filters free cortisol – are normal, which effectively rules out Click to open/closeImmunoglobulins
option A. His type-2 diabetes is likely to be related to Click to open/closeBiochemistry
obesity and his sedentary lifestyle, although diabetes Click to open/closeDiabetes Click to
Security mellitus could, of course, also be due to alcohol- open/closeEndocrinology Click to
induced chronic pancreatitis. open/closeBlood gases Click to
Change Password open/closeCSF
Sign Out Management involves lifestyle measures to promote
weight loss, and strict control of his alcohol intake.
Metformin would be the ideal treatment for his
diabetes, although this would be contraindicated in
the presence of continued alcohol excess.

2224

© 2011 PasTest Ltd | About Us | Contact Us | Help

file:///E|/Shakhawan/Endocrinology/106a.htm[3/13/2012 7:06:26 PM]


MyPasTest

Main Navigation
Question Browser: MRCP 1
Home
Subscriptions Question Browser Exam Builder Saved Exams
» MRCP 1 Session Progress
• Question Browser
Questions Correct 33
• Timed Test You are asked to review a 36-year-old man who has
suffered a myocardial infarction. He is a non-smoker with Questions Incorrect 3
• Mock Exam
no past history of note and is not diabetic. On admission Questions Total 36
• Past Papers his total cholesterol was 10.2 mmol/l, with triglycerides just
Questions Percentage 91 %
• Random Questions above the normal range, normal HDL and markedly raised
LDL cholesterol. His father died of a myocardial infarction at More
• My Performance
the age of 43. What is the most likely cause of his
• Media Bank raised cholesterol?
• New Multimedia
Heterozygous familial hypercholesterolaemia
Online Extras Reference: Normal Values
Familial combined hyperlipidaemia
Library
Familial hypertriglyceridaemia Click to
Community open/closeHaematology
Secondary hyperlipidaemia
Help Click to
Remnant hyperlipidaemia open/closeImmunoglobulins
PasTest Store
Click to
2225 open/closeBiochemistry
My Account Click to open/closeDiabetes
Click to
Profile open/closeEndocrinology
Click to open/closeBlood
Newsletters gases Click to
My Career and Exams open/closeCSF
Order History
Learning Goals
Question Filters

Security

Change Password
Sign Out

© 2011 PasTest Ltd | About Us | Contact Us | Help

file:///E|/Shakhawan/Endocrinology/107.htm[3/13/2012 7:06:28 PM]


MyPasTest

Main Navigation
Question Browser: MRCP 1
Home
Subscriptions Question Browser Exam Builder Saved Exams
» MRCP 1 Session Progress
• Question Browser
Questions Correct 34
• Timed Test You are asked to review a 36-year-old man who has
suffered a myocardial infarction. He is a non-smoker with Questions Incorrect 3
• Mock Exam
no past history of note and is not diabetic. On admission Questions Total 37
• Past Papers his total cholesterol was 10.2 mmol/l, with triglycerides just
Questions Percentage 91 %
• Random Questions above the normal range, normal HDL and markedly raised
LDL cholesterol. His father died of a myocardial infarction at More
• My Performance
the age of 43. What is the most likely cause of his
• Media Bank raised cholesterol?
• New Multimedia
Heterozygous familial Your answer
Online Extras hypercholesterolaemia
Library Familial combined hyperlipidaemia
Community Familial hypertriglyceridaemia
Help Secondary hyperlipidaemia
PasTest Store Remnant hyperlipidaemia

My Account

Profile
Heterozygous familial hypercholesterolaemia is an
Newsletters
autosomal-dominant monogenic disorder present in 1 in
My Career and Exams 500 of the population; the prevalence is increased in
Order History French-Canadians, Finns and South Africans. Reference: Normal Values
Homozygous FH occurs in 1 in 10 6 patients and is
Learning Goals
associated with early cardiovascular death in childhood. Click to open/closeHaematology
Question Filters Clinical features may include tendon xanthomas and Click to open/closeImmunoglobulins
xanthelasma. There is a genetic abnormality of the liver Click to open/closeBiochemistry
LDL receptor. It is said that 50% of men with this Click to open/closeDiabetes Click to
Security disorder will die by the age of 60 from cardiovascular open/closeEndocrinology Click to
disease if untreated. High-dose statin therapy is the open/closeBlood gases Click to
Change Password standard therapy, and specialist lipidologist advice is open/closeCSF
Sign Out recommended for these patients.

2225

© 2011 PasTest Ltd | About Us | Contact Us | Help

file:///E|/Shakhawan/Endocrinology/107a.htm[3/13/2012 7:06:29 PM]


MyPasTest

Main Navigation
Question Browser: MRCP 1
Home
Subscriptions Question Browser Exam Builder Saved Exams
» MRCP 1 Session Progress
• Question Browser
Questions Correct 34
• Timed Test A 28-year-old man presents with acute pancreatitis. He
admits to occasionally drinking wine, but not to excess, and Questions Incorrect 3
• Mock Exam
there have been no symptoms to suggest gall-bladder Questions Total 37
• Past Papers disease. He suffered a left retinal vein thrombosis 2 years
Questions Percentage 91 %
• Random Questions ago. Triglyceride concentration was estimated at 10
mmol/l, with normal HDL and LDL levels. What is the More
• My Performance
most likely cause of his clinical presentation?
• Media Bank
• New Multimedia Secondary hyperlipidaemia
Online Extras Familial hypercholesterolaemia Reference: Normal Values
Library Familial hypertriglyceridaemia
Hypolipidaemia Click to
Community open/closeHaematology
Help Abetalipoproteinaemia Click to
open/closeImmunoglobulins
PasTest Store 2226 Click to
open/closeBiochemistry
My Account Click to open/closeDiabetes
Click to
Profile open/closeEndocrinology
Click to open/closeBlood
Newsletters gases Click to
My Career and Exams open/closeCSF
Order History
Learning Goals
Question Filters

Security

Change Password
Sign Out

© 2011 PasTest Ltd | About Us | Contact Us | Help

file:///E|/Shakhawan/Endocrinology/108.htm[3/13/2012 7:06:31 PM]


MyPasTest

Main Navigation
Question Browser: MRCP 1
Home
Subscriptions Question Browser Exam Builder Saved Exams
» MRCP 1 Session Progress
• Question Browser
Questions Correct 35
• Timed Test A 28-year-old man presents with acute pancreatitis. He
admits to occasionally drinking wine, but not to excess, and Questions Incorrect 3
• Mock Exam
there have been no symptoms to suggest gall-bladder Questions Total 38
• Past Papers disease. He suffered a left retinal vein thrombosis 2 years
Questions Percentage 92 %
• Random Questions ago. Triglyceride concentration was estimated at 10
mmol/l, with normal HDL and LDL levels. What is the More
• My Performance
most likely cause of his clinical presentation?
• Media Bank
• New Multimedia Secondary hyperlipidaemia
Online Extras Familial hypercholesterolaemia
Library Familial hypertriglyceridaemia Your answer
Community Hypolipidaemia
Help Abetalipoproteinaemia
PasTest Store

My Account Triglyceride concentrations above 6 mmol/l carry a


significant risk of complications, and this unfortunate
Profile man has suffered both a retinal vein thrombosis and
Newsletters acute pancreatitis. Inherited lipoprotein lipase or
apoprotein C-II deficiency may be responsible, but
My Career and Exams these defects are rare and normally present in childhood
Order History with eruptive xanthomas, lipaemia retinalis, pancreatitis Reference: Normal Values
and retinal vein thrombosis. So-called ‘familial
Learning Goals hypertriglyceridaemia’ presenting in this way, with Click to open/closeHaematology
Question Filters complications in adulthood and no specific identified Click to open/closeImmunoglobulins
genetic defect, is more common. Treatment in this case Click to open/closeBiochemistry
would involve the avoidance of alcohol and use of a Click to open/closeDiabetes Click to
Security fibrate-type agent. open/closeEndocrinology Click to
open/closeBlood gases Click to
Change Password open/closeCSF
2226
Sign Out

© 2011 PasTest Ltd | About Us | Contact Us | Help

file:///E|/Shakhawan/Endocrinology/108a.htm[3/13/2012 7:06:32 PM]


MyPasTest

Main Navigation
Question Browser: MRCP 1
Home
Subscriptions Question Browser Exam Builder Saved Exams
» MRCP 1 Session Progress
• Question Browser
Questions Correct 35
• Timed Test A 24-year-old man with learning difficulties presents for
review. He complains of a sudden deterioration of vision in Questions Incorrect 3
• Mock Exam
his left eye. His past history of note includes a deep vein Questions Total 38
• Past Papers thrombosis. On examination he appears tall and slim and
Questions Percentage 92 %
• Random Questions almost marfanoid in appearance. He has a markedly
elevated urinary homocysteine. Which enzyme defect is More
• My Performance
most likely to be responsible for this clinical picture?
• Media Bank
• New Multimedia Methylene tetrahydrofolate reductase
Online Extras Histidase Reference: Normal Values
Library Homogentisic acid oxidase
Branch-chain ketoacid dehydrogenase Click to
Community open/closeHaematology
Help Cystathionine synthetase Click to
open/closeImmunoglobulins
PasTest Store 2227 Click to
open/closeBiochemistry
My Account Click to open/closeDiabetes
Click to
Profile open/closeEndocrinology
Click to open/closeBlood
Newsletters gases Click to
My Career and Exams open/closeCSF
Order History
Learning Goals
Question Filters

Security

Change Password
Sign Out

© 2011 PasTest Ltd | About Us | Contact Us | Help

file:///E|/Shakhawan/Endocrinology/109.htm[3/13/2012 7:06:34 PM]


MyPasTest

Main Navigation
Question Browser: MRCP 1
Home
Subscriptions Question Browser Exam Builder Saved Exams
» MRCP 1 Session Progress
• Question Browser
Questions Correct 35
• Timed Test A 24-year-old man with learning difficulties presents for
review. He complains of a sudden deterioration of vision in Questions Incorrect 4
• Mock Exam
his left eye. His past history of note includes a deep vein Questions Total 39
• Past Papers thrombosis. On examination he appears tall and slim and
Questions Percentage 89 %
• Random Questions almost marfanoid in appearance. He has a markedly
elevated urinary homocysteine. Which enzyme defect is More
• My Performance
most likely to be responsible for this clinical picture?
• Media Bank
• New Multimedia Methylene tetrahydrofolate
reductase
Online Extras
Histidase
Library
Homogentisic acid oxidase Your answer
Community
Branch-chain ketoacid
Help dehydrogenase
PasTest Store Cystathionine synthetase Correct answer

My Account

Profile
This is the presentation of type-1 homocystinuria,
Newsletters
where a defect in cystathionine synthetase is
My Career and Exams responsible. Patients present with mild to moderate
Order History mental handicap, a marfan-like syndrome and Reference: Normal Values
thrombotic episodes. The sudden visual deterioration
Learning Goals may be due to a thrombotic episode or to the lens Click to open/closeHaematology
Question Filters dislocation associated with this condition. Click to open/closeImmunoglobulins
Homocystinuria may also associated with defects in Click to open/closeBiochemistry
methylene tetrahydrofolate reductase (type-2 disease), Click to open/closeDiabetes Click to
Security but survivors often have more severe mental open/closeEndocrinology Click to
retardation and rarely survive the neonatal period. open/closeBlood gases Click to
Change Password open/closeCSF
Sign Out
2227

© 2011 PasTest Ltd | About Us | Contact Us | Help

file:///E|/Shakhawan/Endocrinology/109a.htm[3/13/2012 7:06:35 PM]


MyPasTest

Main Navigation
Question Browser: MRCP 1
Home
Subscriptions Question Browser Exam Builder Saved Exams
» MRCP 1 Session Progress
• Question Browser
Questions Correct 35
• Timed Test A 54-year-old man with long-standing type-2 diabetes
presents for review. He has a history of hypertension, a 20 Questions Incorrect 4
• Mock Exam
cigarettes per day smoking habit; recent Hb A 1c results Questions Total 39
• Past Papers
have averaged 8.9%. He wore deck shoes without socks on Questions Percentage 89 %
• Random Questions a recent holiday in Spain, and his wife noticed a large ulcer
• My Performance over the big toe on his left foot. On examination there is More
obvious loss of sensation, probing of the depth of the ulcer
• Media Bank elicits no pain. His foot appears warm with a bounding
• New Multimedia dorsalis pedis pulse, and there is some toe clawing. What
is the likely cause of his ulcer?
Online Extras Reference: Normal Values
Library Simple trauma with no underlying pathology
Click to
Community Peripheral vascular disease open/closeHaematology
Help Local trauma combined with diabetic neuropathy Click to
open/closeImmunoglobulins
PasTest Store Vasculitis Click to
Self-neglect open/closeBiochemistry
My Account Click to open/closeDiabetes
2228 Click to
Profile open/closeEndocrinology
Click to open/closeBlood
Newsletters gases Click to
My Career and Exams open/closeCSF
Order History
Learning Goals
Question Filters

Security

Change Password
Sign Out

© 2011 PasTest Ltd | About Us | Contact Us | Help

file:///E|/Shakhawan/Endocrinology/110.htm[3/13/2012 7:06:37 PM]


MyPasTest

Main Navigation
Question Browser: MRCP 1
Home
Subscriptions Question Browser Exam Builder Saved Exams
» MRCP 1 Session Progress
• Question Browser
Questions Correct 36
• Timed Test A 54-year-old man with long-standing type-2 diabetes
presents for review. He has a history of hypertension, a 20 Questions Incorrect 4
• Mock Exam
cigarettes per day smoking habit; recent Hb A 1c results Questions Total 40
• Past Papers
have averaged 8.9%. He wore deck shoes without socks on Questions Percentage 90 %
• Random Questions a recent holiday in Spain, and his wife noticed a large ulcer
• My Performance over the big toe on his left foot. On examination there is More
obvious loss of sensation, probing of the depth of the ulcer
• Media Bank elicits no pain. His foot appears warm with a bounding
• New Multimedia dorsalis pedis pulse, and there is some toe clawing. What
is the likely cause of his ulcer?
Online Extras
Library Simple trauma with no underlying
Community pathology

Help Peripheral vascular disease

PasTest Store Local trauma combined with diabetic Your answer


neuropathy
Vasculitis
My Account
Self-neglect
Profile
Newsletters
My Career and Exams
Order History Some 10–15% of patients with diabetes suffer foot Reference: Normal Values
Learning Goals ulceration at some stage during their lives; while 50%
of all lower limb amputations are performed on Click to open/closeHaematology
Question Filters patients with diabetes, effective supervision may Click to open/closeImmunoglobulins
prevent a number of these from occurring. Click to open/closeBiochemistry
Click to open/closeDiabetes Click to
Security This man has the features of neuropathy, although he open/closeEndocrinology Click to
certainly has risk factors for peripheral vascular open/closeBlood gases Click to
Change Password disease. The neuropathic foot is said to be painless, open/closeCSF
Sign Out or to have abnormal neuropathic pain, to be high
arched with toe clawing. It is often warm with
bounding pulses and ulceration tends to occur on the
plantar surface. Peripheral ischaemia is said to be
associated with rest pain in a cold and nearly
pulseless foot. There is often gravity-dependent
reddening of the foot, which vanishes if the foot is
elevated, ulceration tends to be painful and often
presents in the heal area. Of course, it is perfectly
possible for the two conditions to coexist, and for a
mixed ischaemic/neuropathic pathology to be the
underlying cause.

2228

© 2011 PasTest Ltd | About Us | Contact Us | Help

file:///E|/Shakhawan/Endocrinology/110a.htm[3/13/2012 7:06:38 PM]


MyPasTest

Main Navigation
Question Browser: MRCP 1
Home
Subscriptions Question Browser Exam Builder Saved Exams
» MRCP 1 Session Progress
• Question Browser
Questions Correct 0
• Timed Test A 69-year-old woman presented via her GP with episodes
of facial flushing and diarrhoea. An ultrasound scan Questions Incorrect 0
• Mock Exam
revealed multiple hepatic lesions, and a 24-h urine Questions Total 0
• Past Papers collection revealed an elevated 5-HIAA. Unfortunately she
Questions Percentage 0%
• Random Questions did not consent to follow-up and next presented 2 years
later. Her son noticed a gradual deterioration in her More
• My Performance
condition so that she appeared unable to cope at home.
• Media Bank There was apathy, depression and the onset of mild
• New Multimedia confusion. The diarrhoea is still present at review, and now
she appears to have photosensitive dermatitis, glossitis and
Online Extras angular stomatitis. What diagnosis fits best with her Reference: Normal Values
Library clinical picture?
Click to
Community open/closeHaematology
Pellagra
Help Click to
Alzheimer’s disease
open/closeImmunoglobulins
PasTest Store
Coeliac disease Click to
Riboflavin deficiency open/closeBiochemistry
My Account Click to open/closeDiabetes
Thiamine deficiency Click to
Profile open/closeEndocrinology
2229 Click to open/closeBlood
Newsletters gases Click to
My Career and Exams open/closeCSF
Order History
Learning Goals
Question Filters

Security

Change Password
Sign Out

© 2011 PasTest Ltd | About Us | Contact Us | Help

file:///E|/Shakhawan/Endocrinology/111.htm[3/13/2012 7:06:39 PM]


MyPasTest

Main Navigation
Question Browser: MRCP 1
Home
Subscriptions Question Browser Exam Builder Saved Exams
» MRCP 1 Session Progress
• Question Browser
Questions Correct 1
• Timed Test A 69-year-old woman presented via her GP with episodes
of facial flushing and diarrhoea. An ultrasound scan Questions Incorrect 0
• Mock Exam
revealed multiple hepatic lesions, and a 24-h urine Questions Total 1
• Past Papers collection revealed an elevated 5-HIAA. Unfortunately she
Questions Percentage 100 %
• Random Questions did not consent to follow-up and next presented 2 years
later. Her son noticed a gradual deterioration in her More
• My Performance
condition so that she appeared unable to cope at home.
• Media Bank There was apathy, depression and the onset of mild
• New Multimedia confusion. The diarrhoea is still present at review, and now
she appears to have photosensitive dermatitis, glossitis and
Online Extras angular stomatitis. What diagnosis fits best with her
Library clinical picture?
Community
Pellagra Your answer
Help
Alzheimer’s disease
PasTest Store
Coeliac disease
Riboflavin deficiency
My Account Thiamine deficiency
Profile
Newsletters
My Career and Exams
In the presence of hepatic metastases, diarrhoea,
Order History facial flushing and a raised urinary 5-HIAA (5- Reference: Normal Values
Learning Goals hydroxyindoleacetic acid), it is highly likely that this
woman has the carcinoid syndrome. Unfortunately Click to open/closeHaematology
Question Filters during the 2 years that she has been lost to follow- Click to open/closeImmunoglobulins
up, it appears that she has developed pellagra. Click to open/closeBiochemistry
Click to open/closeDiabetes Click to
Security Pellagra is caused by a deficiency of niacin. Niacin is open/closeEndocrinology Click to
manufactured via an enzymatic pathway that involves open/closeBlood gases Click to
Change Password the metabolism of tryptophan. Tryptophan is also the open/closeCSF
Sign Out substrate amino acid used by carcinoid tumours to
produce 5-hydroxytryptamine. As the carcinoid
tumour mass increases, more and more of the
available tryptophan is consumed, and less is
available for niacin production. Eventually patients
may become niacin deficient, suffering the triad of
dermatitis, diarrhoea and dementia, the features of
pellagra.

2229

© 2011 PasTest Ltd | About Us | Contact Us | Help

file:///E|/Shakhawan/Endocrinology/111a.htm[3/13/2012 7:06:41 PM]


MyPasTest

Main Navigation
Question Browser: MRCP 1
Home
Subscriptions Question Browser Exam Builder Saved Exams
» MRCP 1 Session Progress
• Question Browser
Questions Correct 1
• Timed Test A 45-year-old woman is referred to the endocrine clinic by
her GP for review. She has a body mass index of 35, Questions Incorrect 0
• Mock Exam
hypertension and impaired glucose tolerance. By the time Questions Total 1
• Past Papers she visits you in clinic she has succeeded in losing 3 kg in
Questions Percentage 100 %
• Random Questions weight. You decide to give her a trial of orlistat and
behavioural therapy. Which of the following best More
• My Performance
describes the mode of action of orlistat?
• Media Bank
• New Multimedia Orlistat is a centrally acting appetite suppressant
Online Extras Orlistat is a pancreatic and gastric lipase inhibitor Reference: Normal Values
Library Orlistat reduces hepatic glucose production
Orlistat reduces insulin resistance Click to
Community open/closeHaematology
Help Orlistat is a b 3 -agonist Click to
open/closeImmunoglobulins
PasTest Store
2230 Click to
open/closeBiochemistry
My Account Click to open/closeDiabetes
Click to
Profile open/closeEndocrinology
Click to open/closeBlood
Newsletters gases Click to
My Career and Exams open/closeCSF
Order History
Learning Goals
Question Filters

Security

Change Password
Sign Out

© 2011 PasTest Ltd | About Us | Contact Us | Help

file:///E|/Shakhawan/Endocrinology/112.htm[3/13/2012 7:06:42 PM]


MyPasTest

Main Navigation
Question Browser: MRCP 1
Home
Subscriptions Question Browser Exam Builder Saved Exams
» MRCP 1 Session Progress
• Question Browser
Questions Correct 2
• Timed Test A 45-year-old woman is referred to the endocrine clinic by
her GP for review. She has a body mass index of 35, Questions Incorrect 0
• Mock Exam
hypertension and impaired glucose tolerance. By the time Questions Total 2
• Past Papers she visits you in clinic she has succeeded in losing 3 kg in
Questions Percentage 100 %
• Random Questions weight. You decide to give her a trial of orlistat and
behavioural therapy. Which of the following best More
• My Performance
describes the mode of action of orlistat?
• Media Bank
• New Multimedia Orlistat is a centrally acting appetite
suppressant
Online Extras
Orlistat is a pancreatic and gastric Your answer
Library lipase inhibitor
Community Orlistat reduces hepatic glucose
Help production
PasTest Store Orlistat reduces insulin resistance
Orlistat is a b 3 -agonist
My Account

Profile
Newsletters Orlistat, a pancreatic lipase inhibitor, blocks the
breakdown and hence absorption of dietary fat. In
My Career and Exams
essence, this means that ingested fat continues its
Order History passage through the gut. If patients taking orlistat do Reference: Normal Values
Learning Goals not maintain a low-fat diet then they may suffer
distressing oily diarrhoea. This is why it is essential to Click to open/closeHaematology
Question Filters combine orlistat therapy with an effective patient Click to open/closeImmunoglobulins
support programme. Sibutramine is a centrally acting Click to open/closeBiochemistry
drug that acts on serotoninergic and noradrenergic Click to open/closeDiabetes Click to
Security pathways to encourage earlier satiety and the ingestion open/closeEndocrinology Click to
of smaller meal portions. open/closeBlood gases Click to
Change Password open/closeCSF
Sign Out 2230

© 2011 PasTest Ltd | About Us | Contact Us | Help

file:///E|/Shakhawan/Endocrinology/112a.htm[3/13/2012 7:06:44 PM]


MyPasTest

Main Navigation
Question Browser: MRCP 1
Home
Subscriptions Question Browser Exam Builder Saved Exams
» MRCP 1 Session Progress
• Question Browser
Questions Correct 2
• Timed Test A 42-year-old man with long-standing HIV infection
presents for review. He has been taking antiretroviral Questions Incorrect 0
• Mock Exam
therapy for 5 years and has been relatively free of Questions Total 2
• Past Papers associated disease. You notice on examination that he
Questions Percentage 100 %
• Random Questions appears to have lost subcutaneous fat on his arms, legs
and face, and has increased deposition of fat around his More
• My Performance
abdomen. His lipids are also abnormal, with a raised
• Media Bank triglyceride level and low HDL cholesterol. What is the
• New Multimedia most likely cause of this clinical picture?

Online Extras Antiretroviral-related lipodystrophy Reference: Normal Values


Library HIV wasting
Click to
Community HIV-associated malignancy open/closeHaematology
Help Likely gastrointestinal pathology Click to
open/closeImmunoglobulins
PasTest Store An inherited insulin-resistance syndrome Click to
open/closeBiochemistry
2231 Click to open/closeDiabetes
My Account
Click to
Profile open/closeEndocrinology
Click to open/closeBlood
Newsletters gases Click to
My Career and Exams open/closeCSF
Order History
Learning Goals
Question Filters

Security

Change Password
Sign Out

© 2011 PasTest Ltd | About Us | Contact Us | Help

file:///E|/Shakhawan/Endocrinology/113.htm[3/13/2012 7:06:45 PM]


MyPasTest

Main Navigation
Question Browser: MRCP 1
Home
Subscriptions Question Browser Exam Builder Saved Exams
» MRCP 1 Session Progress
• Question Browser
Questions Correct 3
• Timed Test A 42-year-old man with long-standing HIV infection
presents for review. He has been taking antiretroviral Questions Incorrect 0
• Mock Exam
therapy for 5 years and has been relatively free of Questions Total 3
• Past Papers associated disease. You notice on examination that he
Questions Percentage 100 %
• Random Questions appears to have lost subcutaneous fat on his arms, legs
and face, and has increased deposition of fat around his More
• My Performance
abdomen. His lipids are also abnormal, with a raised
• Media Bank triglyceride level and low HDL cholesterol. What is the
• New Multimedia most likely cause of this clinical picture?

Online Extras Antiretroviral-related lipodystrophy Your answer


Library HIV wasting
Community HIV-associated malignancy
Help Likely gastrointestinal pathology
PasTest Store An inherited insulin-resistance
syndrome
My Account

Profile
Newsletters
As the survival of HIV-positive patients improves, more
My Career and Exams complications of antiretroviral therapy (ARVs) are
Order History becoming apparent. These include a lipodystrophy-type Reference: Normal Values
syndrome characterised by the loss of peripheral and
Learning Goals facial subcutaneous fat, but increased abdominal and Click to open/closeHaematology
Question Filters visceral fat deposition. There is also an increase in the Click to open/closeImmunoglobulins
size of the dorsocervical fat pad ‘buffalo-hump’. This fat Click to open/closeBiochemistry
redistribution is associated with a picture of Click to open/closeDiabetes Click to
Security abnormalities usually associated with insulin resistance, open/closeEndocrinology Click to
such as impaired glucose tolerance, low HDL cholesterol open/closeBlood gases Click to
Change Password and high triglycerides. Glitazones may be of value in open/closeCSF
Sign Out treating the condition, although some small trials have
proved equivocal.

2231

© 2011 PasTest Ltd | About Us | Contact Us | Help

file:///E|/Shakhawan/Endocrinology/113a.htm[3/13/2012 7:06:47 PM]


MyPasTest

Main Navigation
Question Browser: MRCP 1
Home
Subscriptions Question Browser Exam Builder Saved Exams
» MRCP 1 Session Progress
• Question Browser
Questions Correct 3
• Timed Test You are asked by the psychiatrists to review a 42-year-old
woman who has long-standing bipolar disorder for which Questions Incorrect 0
• Mock Exam
she takes lithium. During a recent inpatient stay she Questions Total 3
• Past Papers appeared to be drinking vast amounts of water and getting
Questions Percentage 100 %
• Random Questions up many times in the night to urinate. Urea and electrolyte
testing reveals elevated sodium and urea concentrations, More
• My Performance
suggesting possible mild dehydration. Water deprivation
• Media Bank testing reveals a progressively rising serum osmolality to
• New Multimedia above 300 mOsm/kg, without increased urine osmolality.
Blood glucose is normal. What is the most likely cause
Online Extras of this clinical picture? Reference: Normal Values
Library
Cranial diabetes insipidus Click to
Community open/closeHaematology
Nephrogenic diabetes insipidus
Help Click to
Psychogenic polydipsia open/closeImmunoglobulins
PasTest Store
Diabetes mellitus Click to
open/closeBiochemistry
Syndrome of inappropriate ADH secretion Click to open/closeDiabetes
My Account
Click to
Profile 2232 open/closeEndocrinology
Click to open/closeBlood
Newsletters gases Click to
My Career and Exams open/closeCSF
Order History
Learning Goals
Question Filters

Security

Change Password
Sign Out

© 2011 PasTest Ltd | About Us | Contact Us | Help

file:///E|/Shakhawan/Endocrinology/114.htm[3/13/2012 7:06:48 PM]


MyPasTest

Main Navigation
Question Browser: MRCP 1
Home
Subscriptions Question Browser Exam Builder Saved Exams
» MRCP 1 Session Progress
• Question Browser
Questions Correct 4
• Timed Test You are asked by the psychiatrists to review a 42-year-old
woman who has long-standing bipolar disorder for which Questions Incorrect 0
• Mock Exam
she takes lithium. During a recent inpatient stay she Questions Total 4
• Past Papers appeared to be drinking vast amounts of water and getting
Questions Percentage 100 %
• Random Questions up many times in the night to urinate. Urea and electrolyte
testing reveals elevated sodium and urea concentrations, More
• My Performance
suggesting possible mild dehydration. Water deprivation
• Media Bank testing reveals a progressively rising serum osmolality to
• New Multimedia above 300 mOsm/kg, without increased urine osmolality.
Blood glucose is normal. What is the most likely cause
Online Extras of this clinical picture?
Library
Cranial diabetes insipidus
Community
Nephrogenic diabetes insipidus Your answer
Help
Psychogenic polydipsia
PasTest Store
Diabetes mellitus
Syndrome of inappropriate ADH
My Account secretion
Profile
Newsletters
My Career and Exams
Order History This woman has nephrogenic diabetes insipidus (DI) Reference: Normal Values
secondary to chronic lithium therapy. There is usually
Learning Goals some recovery after withdrawing lithium therapy but Click to open/closeHaematology
Question Filters many patients suffer some degree of permanent Click to open/closeImmunoglobulins
nephrogenic DI. Lithium therapy should only be Click to open/closeBiochemistry
discontinued with psychiatric advice, but at least now Click to open/closeDiabetes Click to
Security there are possible substitute medications including open/closeEndocrinology Click to
valproate and lamotrigine. Other drugs that may cause open/closeBlood gases Click to
Change Password DI include demeclocycline and glibenclamide. open/closeCSF
Sign Out

2232

© 2011 PasTest Ltd | About Us | Contact Us | Help

file:///E|/Shakhawan/Endocrinology/114a.htm[3/13/2012 7:06:50 PM]


MyPasTest

Main Navigation
Question Browser: MRCP 1
Home
Subscriptions Question Browser Exam Builder Saved Exams
» MRCP 1 Session Progress
• Question Browser
Questions Correct 4
• Timed Test A 35-year-old woman is referred by her GP because of
Questions Incorrect 0
• Mock Exam recurrent headaches. These tend to come on at times of
stress or exercise and appear almost ‘in a flash’. She also Questions Total 4
• Past Papers complains of intermittent palpitations and problems with Questions Percentage 100 %
• Random Questions sweating. He has tried her on a course of antidepressants,
which only seemed to make her symptoms worse. Her 24-h More
• My Performance
urinary catecholamines are markedly raised. An MRI scan
• Media Bank reveals a mass in the right adrenal medulla. Her blood
• New Multimedia pressure in clinic is 145/95 mmHg.
What is the best management plan?
Online Extras Reference: Normal Values
Library Urgent β-blockade
Click to
Community Urgent surgery open/closeHaematology
Help Urgent a-blockade, then β-blockade if required, and Click to
surgery open/closeImmunoglobulins
PasTest Store
Click to
Observation open/closeBiochemistry
My Account β-Blockade followed by urgent surgery Click to open/closeDiabetes
Click to
Profile open/closeEndocrinology
2233
Click to open/closeBlood
Newsletters gases Click to
My Career and Exams open/closeCSF
Order History
Learning Goals
Question Filters

Security

Change Password
Sign Out

© 2011 PasTest Ltd | About Us | Contact Us | Help

file:///E|/Shakhawan/Endocrinology/115.htm[3/13/2012 7:06:51 PM]


MyPasTest

Main Navigation
Question Browser: MRCP 1
Home
Subscriptions Question Browser Exam Builder Saved Exams
» MRCP 1
• Question Browser
• Timed Test A 35-year-old woman is referred by her GP because of recurrent headaches. These
• Mock Exam tend to come on at times of stress or exercise and appear almost ‘in a flash’. She also
complains of intermittent palpitations and problems with sweating. He has tried her on
• Past Papers a course of antidepressants, which only seemed to make her symptoms worse. Her
• Random Questions 24-h urinary catecholamines are markedly raised. An MRI scan reveals a mass in the
right adrenal medulla. Her blood pressure in clinic is 145/95 mmHg.
• My Performance
What is the best management plan?
• Media Bank
• New Multimedia Urgent β-blockade
Online Extras Urgent surgery
Library Urgent a-blockade, then β-blockade if required, and surgery Your answer
Community Observation
Help β-Blockade followed by urgent surgery
PasTest Store

My Account

Profile This woman has a phaeochromocytoma of the right adrenal medulla. The
commonest presenting features are headache (80%), palpitations (70%),
Newsletters hyperhydrosis (60%) and hypertension (sustained in 55% and paroxysmal in 45%
My Career and Exams of patients). Drugs that inhibit catecholamine reuptake, such as tricyclic
antidepressants and cocaine, can exacerbate or unmask the symptoms of this
Order History condition. MRI or MIBG (metaiodobenzyl guanidine) scanning are the definitive
Learning Goals methods of localisation. Management involves a-blockade prior to surgery to avoid
a hypertensive crisis. The 5-year survival rate is 95% for patients with benign
Question Filters
phaeochromocytoma, but this falls to 40% in those with malignant disease.
Phaeochromocytomas are three times more likely to be malignant in women.
Security

Change Password 2233


Sign Out

file:///E|/Shakhawan/Endocrinology/115a.htm[3/13/2012 7:06:53 PM]


MyPasTest

Main Navigation
Question Browser: MRCP 1
Home
Subscriptions Question Browser Exam Builder Saved Exams
» MRCP 1
• Question Browser
• Timed Test A 25-year-old woman presents with a lump on the left-hand side of her neck in the
• Mock Exam thyroid region. Thyroid function is normal, and uptake scanning reveals it to be a cold
nodule. Fine-needle aspiration biopsy reveals architecture suspicious of follicular
• Past Papers carcinoma of the thyroid, this is confirmed on thyroid lobectomy.
• Random Questions
Which of the following is the most appropriate management plan?
• My Performance
• Media Bank Oral thyroxine therapy
• New Multimedia No further surgery
Online Extras Total thyroidectomy
Library Total thyroidectomy, radioiodine therapy with oral thyroxine replacement
Community therapy

Help Radioiodine therapy

PasTest Store 2234

My Account

Profile
Newsletters
My Career and Exams
Order History
Learning Goals
Question Filters

Security

Change Password
Sign Out

file:///E|/Shakhawan/Endocrinology/116.htm[3/13/2012 7:06:55 PM]


MyPasTest

Main Navigation
Question Browser: MRCP 1
Home
Subscriptions Question Browser Exam Builder Saved Exams
» MRCP 1 Session Progress
• Question Browser
Questions Correct 6
• Timed Test A 25-year-old woman presents with a lump on the left-
hand side of her neck in the thyroid region. Thyroid Questions Incorrect 0
• Mock Exam
function is normal, and uptake scanning reveals it to be a Questions Total 6
• Past Papers cold nodule. Fine-needle aspiration biopsy reveals
Questions Percentage 100 %
• Random Questions architecture suspicious of follicular carcinoma of the thyroid,
this is confirmed on thyroid lobectomy. More
• My Performance
• Media Bank Which of the following is the most appropriate
management plan?
• New Multimedia
Online Extras Oral thyroxine therapy
Library No further surgery
Community Total thyroidectomy
Help Total thyroidectomy, radioiodine Your answer
PasTest Store therapy with oral thyroxine replacement
therapy
Radioiodine therapy
My Account

Profile
Newsletters
My Career and Exams The definitive management for follicular carcinoma of
the thyroid without metastases is total thyroidectomy, Reference: Normal Values
Order History
followed by radioiodine therapy, with thyroxine
Learning Goals replacement to a TSH-suppressive dose. If metastases Click to open/closeHaematology
Question Filters are present they usually respond to radioiodine therapy, Click to open/closeImmunoglobulins
and this is added. Metastases are subject to Click to open/closeBiochemistry
haematological spread, with bone being the commonest Click to open/closeDiabetes Click to
Security metastatic site. The 5-year survival rate approaches open/closeEndocrinology Click to
80% for follicular carcinoma. This is much higher than open/closeBlood gases Click to
Change Password for anaplastic carcinoma of the thyroid, which usually open/closeCSF
occurs in the elderly and only carries a 5-year survival
Sign Out
rate of 5%.

2234

© 2011 PasTest Ltd | About Us | Contact Us | Help

file:///E|/Shakhawan/Endocrinology/116a.htm[3/13/2012 7:06:57 PM]


MyPasTest

Main Navigation
Question Browser: MRCP 1
Home
Subscriptions Question Browser Exam Builder Saved Exams
» MRCP 1 Session Progress
• Question Browser
Questions Correct 6
• Timed Test A 58-year-old woman presents with fever, marked anxiety
and agitation, palpitations, marked muscle weakness and Questions Incorrect 0
• Mock Exam
diarrhoea. On examination she has a goitre, and is in fast Questions Total 6
• Past Papers atrial fibrillation with a ventricular rate of 135 beats per
Questions Percentage 100 %
• Random Questions minute. Urine testing reveals evidence of protein, white
cells and red blood cells. Thyroid function testing reveals a More
• My Performance
TSH of < 0.05 mU/l. Which of the following diagnoses
• Media Bank best fits this clinical picture?
• New Multimedia
Urinary tract infection
Online Extras Reference: Normal Values
Paroxysmal atrial fibrillation
Library
Hypothyroidism Click to
Community open/closeHaematology
Phaeochromocytoma
Help Click to
Thyroid storm open/closeImmunoglobulins
PasTest Store
Click to
2235 open/closeBiochemistry
My Account Click to open/closeDiabetes
Click to
Profile open/closeEndocrinology
Click to open/closeBlood
Newsletters gases Click to
My Career and Exams open/closeCSF
Order History
Learning Goals
Question Filters

Security

Change Password
Sign Out

© 2011 PasTest Ltd | About Us | Contact Us | Help

file:///E|/Shakhawan/Endocrinology/117.htm[3/13/2012 7:06:58 PM]


MyPasTest

Main Navigation
Question Browser: MRCP 1
Home
Subscriptions Question Browser Exam Builder Saved Exams
» MRCP 1 Session Progress
• Question Browser
Questions Correct 7
• Timed Test A 58-year-old woman presents with fever, marked anxiety
and agitation, palpitations, marked muscle weakness and Questions Incorrect 0
• Mock Exam
diarrhoea. On examination she has a goitre, and is in fast Questions Total 7
• Past Papers atrial fibrillation with a ventricular rate of 135 beats per
Questions Percentage 100 %
• Random Questions minute. Urine testing reveals evidence of protein, white
cells and red blood cells. Thyroid function testing reveals a More
• My Performance
TSH of < 0.05 mU/l. Which of the following diagnoses
• Media Bank best fits this clinical picture?
• New Multimedia
Urinary tract infection
Online Extras
Paroxysmal atrial fibrillation
Library
Hypothyroidism
Community
Phaeochromocytoma
Help
Thyroid storm Your answer
PasTest Store

My Account
Thyroid storm is an unusual presentation of
Profile thyrotoxicosis, precipitated by acute stress in a
Newsletters previously undiagnosed patient. Causes include
infection (as in this case), myocardial infarction or
My Career and Exams
diabetic ketoacidosis. Inadequate adherence to
Order History therapy in an already diagnosed patient may also be Reference: Normal Values
Learning Goals responsible. Symptoms include fever, anxiety and
agitation, sweating, marked proximal muscle Click to open/closeHaematology
Question Filters weakness, tachycardias, diarrhoea and sometimes Click to open/closeImmunoglobulins
heart failure, particularly in the elderly. Click to open/closeBiochemistry
Click to open/closeDiabetes Click to
Security Acute general management includes carbimazole open/closeEndocrinology Click to
therapy, b -blockade, digitalisation of atrial fibrillation open/closeBlood gases Click to
Change Password and occasionally corticosteroids, which may inhibit open/closeCSF
Sign Out thyroid hormone release. Lugol’s iodine, used in
conjunction with carbimazole therapy may bring a
more rapid resolution.

2235

© 2011 PasTest Ltd | About Us | Contact Us | Help

file:///E|/Shakhawan/Endocrinology/117a.htm[3/13/2012 7:07:00 PM]


MyPasTest

Main Navigation
Question Browser: MRCP 1
Home
Subscriptions Question Browser Exam Builder Saved Exams
» MRCP 1 Session Progress
• Question Browser
Questions Correct 7
• Timed Test A 54-year-old thyrotoxic man has been treated with
radioiodine. What is the best advice for the patient Questions Incorrect 0
• Mock Exam
during the post-radioiodine period? Questions Total 7
• Past Papers
Questions Percentage 100 %
• Random Questions Antithyroid drugs should never be taken after
• My Performance radioiodine treatment More
• Media Bank He should not have close contact with children under
the age of 11 years for about 2 weeks after treatment
• New Multimedia
There is no need to monitor his TSH level
Online Extras Reference: Normal Values
He will never need further doses of radioiodine
Library
He may have close contact with children within 3–4 Click to
Community days open/closeHaematology
Help Click to
2250 open/closeImmunoglobulins
PasTest Store
Click to
open/closeBiochemistry
My Account Click to open/closeDiabetes
Click to
Profile open/closeEndocrinology
Click to open/closeBlood
Newsletters gases Click to
My Career and Exams open/closeCSF
Order History
Learning Goals
Question Filters

Security

Change Password
Sign Out

© 2011 PasTest Ltd | About Us | Contact Us | Help

file:///E|/Shakhawan/Endocrinology/118.htm[3/13/2012 7:07:01 PM]


MyPasTest

Main Navigation
Question Browser: MRCP 1
Home
Subscriptions Question Browser Exam Builder Saved Exams
» MRCP 1 Session Progress
• Question Browser
Questions Correct 8
• Timed Test A 54-year-old thyrotoxic man has been treated with
radioiodine. What is the best advice for the patient Questions Incorrect 0
• Mock Exam
during the post-radioiodine period? Questions Total 8
• Past Papers
Questions Percentage 100 %
• Random Questions Antithyroid drugs should never be
• My Performance taken after radioiodine treatment More
• Media Bank He should not have close contact with Your answer
children under the age of 11 years for
• New Multimedia about 2 weeks after treatment
Online Extras There is no need to monitor his TSH
Library level
Community He will never need further doses of
radioiodine
Help
He may have close contact with
PasTest Store children within 3–4 days

My Account

Profile Antithyroid drugs may be recommenced after


Newsletters radioiodine administration, but treatment should be
withdrawn gradually and guided by 6–8 weekly TFTs.
My Career and Exams Early post-radioiodine hypothyroidism may be transient.
Order History The patient’s TSH level should be monitored every 6 Reference: Normal Values
months after radioiodine therapy to determine late
Learning Goals hypothyroidism. Patients should not have close contact Click to open/closeHaematology
Question Filters with children under the age of 11 years for about 2 Click to open/closeImmunoglobulins
weeks after treatment, this is to prevent exposing Click to open/closeBiochemistry
children to radioactivity. However, there is no overall Click to open/closeDiabetes Click to
Security excess risk of cancer. open/closeEndocrinology Click to
open/closeBlood gases Click to
Change Password open/closeCSF
2250
Sign Out

© 2011 PasTest Ltd | About Us | Contact Us | Help

file:///E|/Shakhawan/Endocrinology/118a.htm[3/13/2012 7:07:03 PM]


MyPasTest

Main Navigation
Question Browser: MRCP 1
Home
Subscriptions Question Browser Exam Builder Saved Exams
» MRCP 1 Session Progress
• Question Browser
Questions Correct 8
• Timed Test A 61-year-old patient with a history of recent
thyrotoxicosis underwent major surgery a week ago. He Questions Incorrect 0
• Mock Exam
now presents with altered mental status, tachycardia, high- Questions Total 8
• Past Papers grade fever, vomiting and cardiac failure. A diagnosis of
Questions Percentage 100 %
• Random Questions thyroid storm (crisis) is made. What is the most
important next step in management? More
• My Performance
• Media Bank Transfer the patient to ITU
• New Multimedia Avoid chlorpromazine in the treatment of agitation
Online Extras Avoid carbimazole treatment Reference: Normal Values
Library Potassium iodide should be given immediately before
propylthiouracil Click to
Community open/closeHaematology
Help Peritoneal dialysis should be started immediately Click to
open/closeImmunoglobulins
PasTest Store 2251 Click to
open/closeBiochemistry
My Account Click to open/closeDiabetes
Click to
Profile open/closeEndocrinology
Click to open/closeBlood
Newsletters gases Click to
My Career and Exams open/closeCSF
Order History
Learning Goals
Question Filters

Security

Change Password
Sign Out

© 2011 PasTest Ltd | About Us | Contact Us | Help

file:///E|/Shakhawan/Endocrinology/119.htm[3/13/2012 7:07:05 PM]


MyPasTest

Main Navigation
Question Browser: MRCP 1
Home
Subscriptions Question Browser Exam Builder Saved Exams
» MRCP 1 Session Progress
• Question Browser
Questions Correct 9
• Timed Test A 61-year-old patient with a history of recent
thyrotoxicosis underwent major surgery a week ago. He Questions Incorrect 0
• Mock Exam
now presents with altered mental status, tachycardia, high- Questions Total 9
• Past Papers grade fever, vomiting and cardiac failure. A diagnosis of
Questions Percentage 100 %
• Random Questions thyroid storm (crisis) is made. What is the most
important next step in management? More
• My Performance
• Media Bank Transfer the patient to ITU Your answer
• New Multimedia Avoid chlorpromazine in the treatment
Online Extras of agitation
Library Avoid carbimazole treatment
Community Potassium iodide should be given
immediately before propylthiouracil
Help
Peritoneal dialysis should be started
PasTest Store immediately

My Account

Profile Any acute stressful precipitating factor such as


Newsletters surgery can result in a thyroid storm or crisis. This
can also occur in patients with acute infections,
My Career and Exams
postpartum and when antithyroid drugs are being
Order History withdrawn. Thyroid crisis is associated with a Reference: Normal Values
Learning Goals significant mortality rate (30–50%) and is best
managed in an intensive care unit where close Click to open/closeHaematology
Question Filters attention can be paid to cardiorespiratory status, fluid Click to open/closeImmunoglobulins
balance and cooling. Click to open/closeBiochemistry
Click to open/closeDiabetes Click to
Security Chlorpromazine can be used to treat agitation and, open/closeEndocrinology Click to
because of its effect in inhibiting central open/closeBlood gases Click to
Change Password thermoregulation, it may be useful in treating the open/closeCSF
Sign Out hyperpyrexia. Specific treatment includes
propylthiouracil (PTU), potassium iodide (after starting
PTU), β-blockers and glucocorticoids. There is no
clinical data comparing PTU and carbimazole in this
situation. Plasmapheresis and peritoneal dialysis may
be effective in cases resistant to pharmacological
measures.

2251

© 2011 PasTest Ltd | About Us | Contact Us | Help

file:///E|/Shakhawan/Endocrinology/119a.htm[3/13/2012 7:07:06 PM]


MyPasTest

Main Navigation
Question Browser: MRCP 1
Home
Subscriptions Question Browser Exam Builder Saved Exams
» MRCP 1 Session Progress
• Question Browser
Questions Correct 9
• Timed Test A 25-year-old woman develops hyperthyroidism 6 weeks
after delivery. On examination she has painless, firm Questions Incorrect 0
• Mock Exam
enlargement of the thyroid gland. Questions Total 9
• Past Papers
Which of the following statements is most correct Questions Percentage 100 %
• Random Questions
with regards to the probable diagnosis? More
• My Performance
• Media Bank The condition is more likely in those in whom thyroid
• New Multimedia peroxidase (TPO) antibodies were positive prior to
delivery
Online Extras Reference: Normal Values
When followed up, most patients have lifelong
Library hypothyroidism
Click to
Community It is less common in patients with a history of Type-1 open/closeHaematology
Help diabetes Click to
Around 20% of women have some degree of thyroid open/closeImmunoglobulins
PasTest Store
dysfunction post partum Click to
open/closeBiochemistry
If it is associated with intense fibrosis of the thyroid Click to open/closeDiabetes
My Account gland Click to
Profile open/closeEndocrinology
2252 Click to open/closeBlood
Newsletters gases Click to
My Career and Exams open/closeCSF
Order History
Learning Goals
Question Filters

Security

Change Password
Sign Out

© 2011 PasTest Ltd | About Us | Contact Us | Help

file:///E|/Shakhawan/Endocrinology/120.htm[3/13/2012 7:07:08 PM]


MyPasTest

Main Navigation
Question Browser: MRCP 1
Home
Subscriptions Question Browser Exam Builder Saved Exams
» MRCP 1 Session Progress
• Question Browser
Questions Correct 10
• Timed Test A 25-year-old woman develops hyperthyroidism 6 weeks
after delivery. On examination she has painless, firm Questions Incorrect 0
• Mock Exam
enlargement of the thyroid gland. Questions Total 10
• Past Papers
Which of the following statements is most correct Questions Percentage 100 %
• Random Questions
with regards to the probable diagnosis? More
• My Performance
• Media Bank The condition is more likely in those in Your answer
• New Multimedia whom thyroid peroxidase (TPO)
antibodies were positive prior to delivery
Online Extras
When followed up, most patients have
Library lifelong hypothyroidism
Community It is less common in patients with a
Help history of Type-1 diabetes
PasTest Store Around 20% of women have some
degree of thyroid dysfunction post
partum
My Account If it is associated with intense fibrosis
of the thyroid gland
Profile
Newsletters
My Career and Exams
Order History Reference: Normal Values
Postpartum thyroiditis is thyroid dysfunction occurring
Learning Goals within the first 6 months’ postpartum. Prevalence Click to open/closeHaematology
Question Filters ranges from 5 to 7%. It develops in 30–52% of women Click to open/closeImmunoglobulins
who have positive TPO antibodies. Most patients have a Click to open/closeBiochemistry
complete remission but some may progress to Click to open/closeDiabetes Click to
Security permanent hypothyroidism. It is twice as common in open/closeEndocrinology Click to
patients with type-1 DM. open/closeBlood gases Click to
Change Password open/closeCSF
Sign Out
2252

© 2011 PasTest Ltd | About Us | Contact Us | Help

file:///E|/Shakhawan/Endocrinology/120a.htm[3/13/2012 7:07:09 PM]


MyPasTest

Main Navigation
Question Browser: MRCP 1
Home
Subscriptions Question Browser Exam Builder Saved Exams
» MRCP 1 Session Progress
• Question Browser
Questions Correct 10
• Timed Test A 27-year-old woman on bromocriptine for
microprolactinoma becomes pregnant. What is the most Questions Incorrect 0
• Mock Exam
appropriate management advice? Questions Total 10
• Past Papers
Questions Percentage 100 %
• Random Questions Continue bromocriptine at the same dose
• My Performance Visual field testing to be done every fortnight More
• Media Bank Continuing bromocriptine would significantly increase
• New Multimedia congenital abnormalities

Online Extras Stop bromocriptine as soon as pregnancy is


Reference: Normal Values
confirmed
Library
Needs cerebral magnetic resonance imaging (MRI) at Click to
Community least twice during pregnancy open/closeHaematology
Help Click to
2254 open/closeImmunoglobulins
PasTest Store
Click to
open/closeBiochemistry
My Account Click to open/closeDiabetes
Click to
Profile open/closeEndocrinology
Click to open/closeBlood
Newsletters gases Click to
My Career and Exams open/closeCSF
Order History
Learning Goals
Question Filters

Security

Change Password
Sign Out

© 2011 PasTest Ltd | About Us | Contact Us | Help

file:///E|/Shakhawan/Endocrinology/121.htm[3/13/2012 7:07:11 PM]


MyPasTest

Main Navigation
Question Browser: MRCP 1
Home
Subscriptions Question Browser Exam Builder Saved Exams
» MRCP 1 Session Progress
• Question Browser
Questions Correct 10
• Timed Test A 27-year-old woman on bromocriptine for
microprolactinoma becomes pregnant. What is the most Questions Incorrect 1
• Mock Exam
appropriate management advice? Questions Total 11
• Past Papers
Questions Percentage 90 %
• Random Questions Continue bromocriptine at the same Your answer
• My Performance dose More
• Media Bank Visual field testing to be done every
fortnight
• New Multimedia
Continuing bromocriptine would
Online Extras significantly increase congenital
Library abnormalities
Community Stop bromocriptine as soon as Correct answer
pregnancy is confirmed
Help
Needs cerebral magnetic resonance
PasTest Store imaging (MRI) at least twice during
pregnancy
My Account

Profile
Newsletters
Whilst bromocriptine is an effective treatment for
My Career and Exams microprolactinoma in terms of suppression of prolactin
Order History secretion, it may interfere with the natural rise in Reference: Normal Values
prolactin seen in pregnancy. Bromocriptine should
Learning Goals therefore be stopped as soon as pregnancy is Click to open/closeHaematology
Question Filters confirmed. Visual field testing and MRI are indicated Click to open/closeImmunoglobulins
only in the occasional patient who becomes Click to open/closeBiochemistry
symptomatic. Data on children whose mothers received Click to open/closeDiabetes Click to
Security bromocriptine throughout pregnancy indicate that the open/closeEndocrinology Click to
incidence of congenital abnormalities is negligible, hence open/closeBlood gases Click to
Change Password the actual risk of possible harm is low. open/closeCSF
Sign Out

2254

© 2011 PasTest Ltd | About Us | Contact Us | Help

file:///E|/Shakhawan/Endocrinology/121a.htm[3/13/2012 7:07:12 PM]


MyPasTest

Main Navigation
Question Browser: MRCP 1
Home
Subscriptions Question Browser Exam Builder Saved Exams
» MRCP 1 Session Progress
• Question Browser
Questions Correct 10
• Timed Test A 41-year-old heavy smoker presents with a serum sodium
level of 112 mmol/l. A diagnosis of SIADH is confirmed. Questions Incorrect 1
• Mock Exam
What is the most appropriate initial management of Questions Total 11
• Past Papers his fluid balance?
Questions Percentage 90 %
• Random Questions
• My Performance Immediate normal saline infusion More
• Media Bank Fluid restriction

• New Multimedia Desmopressin

Online Extras Glucocorticoids


Reference: Normal Values
Library Perform fluid deprivation test
Click to
Community 2255 open/closeHaematology
Help Click to
open/closeImmunoglobulins
PasTest Store
Click to
open/closeBiochemistry
My Account Click to open/closeDiabetes
Click to
Profile open/closeEndocrinology
Click to open/closeBlood
Newsletters gases Click to
My Career and Exams open/closeCSF
Order History
Learning Goals
Question Filters

Security

Change Password
Sign Out

© 2011 PasTest Ltd | About Us | Contact Us | Help

file:///E|/Shakhawan/Endocrinology/122.htm[3/13/2012 7:07:13 PM]


MyPasTest

Main Navigation
Question Browser: MRCP 1
Home
Subscriptions Question Browser Exam Builder Saved Exams
» MRCP 1 Session Progress
• Question Browser
Questions Correct 10
• Timed Test A 41-year-old heavy smoker presents with a serum sodium
level of 112 mmol/l. A diagnosis of SIADH is confirmed. Questions Incorrect 2
• Mock Exam
What is the most appropriate initial management of Questions Total 12
• Past Papers his fluid balance?
Questions Percentage 83 %
• Random Questions
• My Performance Immediate normal saline infusion More
• Media Bank Fluid restriction Correct answer

• New Multimedia Desmopressin

Online Extras Glucocorticoids

Library Perform fluid deprivation test Your answer

Community
Help
PasTest Store In patients with the syndrome of inappropriate ADH
(SIADH) secretion, it is important to restrict fluids to
500–1000 ml/24 hours. The underlying cause must also
My Account be detected and treated. If the problem is not
temporary and long-term fluid restriction is likely to be
Profile difficult, then demeclocycline may be effective by
Newsletters inducing partial nephrogenic diabetes insipidus. In an
emergency, saline infusion may be required; however,
My Career and Exams great care is required as rapid overcorrection of
Order History hyponatraemia may cause central pontine myelinolysis. Reference: Normal Values
Learning Goals
2255 Click to open/closeHaematology
Question Filters Click to open/closeImmunoglobulins
Click to open/closeBiochemistry
Click to open/closeDiabetes Click to
Security open/closeEndocrinology Click to
open/closeBlood gases Click to
Change Password open/closeCSF
Sign Out

© 2011 PasTest Ltd | About Us | Contact Us | Help

file:///E|/Shakhawan/Endocrinology/122a.htm[3/13/2012 7:07:15 PM]


MyPasTest

Main Navigation
Question Browser: MRCP 1
Home
Subscriptions Question Browser Exam Builder Saved Exams
» MRCP 1 Session Progress
• Question Browser
Questions Correct 10
• Timed Test You are called to the psychiatric unit to see an 18-year-old
woman with anorexia nervosa. What would you expect Questions Incorrect 2
• Mock Exam
to see in the results of her biochemical Questions Total 12
• Past Papers investigations?
Questions Percentage 83 %
• Random Questions
• My Performance Raised LH, FSH More
• Media Bank Elevated circulating cortisol

• New Multimedia Low resting growth hormone levels

Online Extras Increased GnRH


Reference: Normal Values
Library Normal oestrogen levels
Click to
Community 2256 open/closeHaematology
Help Click to
open/closeImmunoglobulins
PasTest Store
Click to
open/closeBiochemistry
My Account Click to open/closeDiabetes
Click to
Profile open/closeEndocrinology
Click to open/closeBlood
Newsletters gases Click to
My Career and Exams open/closeCSF
Order History
Learning Goals
Question Filters

Security

Change Password
Sign Out

© 2011 PasTest Ltd | About Us | Contact Us | Help

file:///E|/Shakhawan/Endocrinology/123.htm[3/13/2012 7:07:16 PM]


MyPasTest

Main Navigation
Question Browser: MRCP 1
Home
Subscriptions Question Browser Exam Builder Saved Exams
» MRCP 1 Session Progress
• Question Browser
Questions Correct 11
• Timed Test You are called to the psychiatric unit to see an 18-year-old
woman with anorexia nervosa. What would you expect Questions Incorrect 2
• Mock Exam
to see in the results of her biochemical Questions Total 13
• Past Papers investigations?
Questions Percentage 84 %
• Random Questions
• My Performance Raised LH, FSH More
• Media Bank Elevated circulating cortisol Your answer

• New Multimedia Low resting growth hormone levels

Online Extras Increased GnRH

Library Normal oestrogen levels

Community
Help
PasTest Store The typical patient with anorexia nervosa is a woman
aged < 25 years with weight loss, amenorrhea and
behavioural changes. There is a long-term risk of severe
My Account osteoporosis. Endocrine abnormalities include GnRH
deficiency, low LH and FSH, low oestrogen in women,
Profile raised circulating cortisol, low to normal thyroxine,
Newsletters reduced T 3 , normal TSH and increased resting GH
levels.
My Career and Exams
Order History Reference: Normal Values
2256
Learning Goals
Click to open/closeHaematology
Question Filters Click to open/closeImmunoglobulins
Click to open/closeBiochemistry
Click to open/closeDiabetes Click to
Security open/closeEndocrinology Click to
open/closeBlood gases Click to
Change Password open/closeCSF
Sign Out

© 2011 PasTest Ltd | About Us | Contact Us | Help

file:///E|/Shakhawan/Endocrinology/123a.htm[3/13/2012 7:07:18 PM]


MyPasTest

Main Navigation
Question Browser: MRCP 1
Home
Subscriptions Question Browser Exam Builder Saved Exams
» MRCP 1 Session Progress
• Question Browser
Questions Correct 11
• Timed Test A 72-year-old woman recently diagnosed as having
hyperparathyroidism has a serum calcium concentration of Questions Incorrect 2
• Mock Exam
3.2 mmol/l. What is the most appropriate treatment? Questions Total 13
• Past Papers
Questions Percentage 84 %
• Random Questions Surgery
• My Performance Long-term observation with regular blood tests More
• Media Bank Long-term bisphosphonates
• New Multimedia Hormone replacement therapy
Online Extras Do further tests to see if there is end-organ damage Reference: Normal Values
Library
2257 Click to
Community open/closeHaematology
Help Click to
open/closeImmunoglobulins
PasTest Store
Click to
open/closeBiochemistry
My Account Click to open/closeDiabetes
Click to
Profile open/closeEndocrinology
Click to open/closeBlood
Newsletters gases Click to
My Career and Exams open/closeCSF
Order History
Learning Goals
Question Filters

Security

Change Password
Sign Out

© 2011 PasTest Ltd | About Us | Contact Us | Help

file:///E|/Shakhawan/Endocrinology/124.htm[3/13/2012 7:07:19 PM]


MyPasTest

Main Navigation
Question Browser: MRCP 1
Home
Subscriptions Question Browser Exam Builder Saved Exams
» MRCP 1 Session Progress
• Question Browser
Questions Correct 12
• Timed Test A 72-year-old woman recently diagnosed as having
hyperparathyroidism has a serum calcium concentration of Questions Incorrect 2
• Mock Exam
3.2 mmol/l. What is the most appropriate treatment? Questions Total 14
• Past Papers
Questions Percentage 85 %
• Random Questions Surgery Your answer
• My Performance Long-term observation with regular More
• Media Bank blood tests

• New Multimedia Long-term bisphosphonates

Online Extras Hormone replacement therapy

Library Do further tests to see if there is end-


organ damage
Community
Help
PasTest Store
In primary hyperparathyroidism, a patient with a
My Account markedly elevated serum calcium > 3 mmol/l should be
referred for surgery, unless this is contraindicated for
Profile other reasons. Other indications for surgery include
impaired renal function, renal stones, nephrocalcinosis,
Newsletters
reduced bone mineral density (BMD), substantially
My Career and Exams elevated urinary calcium excretion of >10 mmol/24
Order History hours. Medical management is only indicated if the Reference: Normal Values
patient is unsuitable for surgery.
Learning Goals
Click to open/closeHaematology
Question Filters Click to open/closeImmunoglobulins
2257 Click to open/closeBiochemistry
Click to open/closeDiabetes Click to
Security open/closeEndocrinology Click to
open/closeBlood gases Click to
Change Password open/closeCSF
Sign Out

© 2011 PasTest Ltd | About Us | Contact Us | Help

file:///E|/Shakhawan/Endocrinology/124a.htm[3/13/2012 7:07:21 PM]


MyPasTest

Main Navigation
Question Browser: MRCP 1
Home
Subscriptions Question Browser Exam Builder Saved Exams
» MRCP 1 Session Progress
• Question Browser
Questions Correct 12
• Timed Test What is the commonest cause of death in patients
with von Hippel–Lindau disease? Questions Incorrect 2
• Mock Exam
Questions Total 14
• Past Papers
Cerebellar haemangioblastoma Questions Percentage 85 %
• Random Questions
Renal carcinoma
• My Performance More
Retinal tumours
• Media Bank
Myocardial infarction
• New Multimedia
Phaeochromocytoma
Online Extras Reference: Normal Values
Library 2259
Click to
Community open/closeHaematology
Help Click to
open/closeImmunoglobulins
PasTest Store
Click to
open/closeBiochemistry
My Account Click to open/closeDiabetes
Click to
Profile open/closeEndocrinology
Click to open/closeBlood
Newsletters gases Click to
My Career and Exams open/closeCSF
Order History
Learning Goals
Question Filters

Security

Change Password
Sign Out

© 2011 PasTest Ltd | About Us | Contact Us | Help

file:///E|/Shakhawan/Endocrinology/125.htm[3/13/2012 7:07:22 PM]


MyPasTest

Main Navigation
Question Browser: MRCP 1
Home
Subscriptions Question Browser Exam Builder Saved Exams
» MRCP 1 Session Progress
• Question Browser
Questions Correct 12
• Timed Test What is the commonest cause of death in patients
with von Hippel–Lindau disease? Questions Incorrect 3
• Mock Exam
Questions Total 15
• Past Papers
Cerebellar haemangioblastoma Questions Percentage 80 %
• Random Questions
Renal carcinoma Correct answer
• My Performance More
Retinal tumours
• Media Bank
Myocardial infarction Your answer
• New Multimedia
Phaeochromocytoma
Online Extras
Library
Community
VHL disease is an autosomal-dominant condition, with
Help the VHL gene being located on chromosome 3.
PasTest Store Estimated prevalence is 1/39,000. Retinal angiomatosis
is the initial presentation in 40% patients. Cerebellar
haemangioblastoma is a common initial presentation.
My Account Renal carcinoma is the commonest cause of death, with
phaeochromocytoma occurring in 40% of patients with
Profile VHL.
Newsletters
2259
My Career and Exams
Order History Reference: Normal Values
Learning Goals
Click to open/closeHaematology
Question Filters Click to open/closeImmunoglobulins
Click to open/closeBiochemistry
Click to open/closeDiabetes Click to
Security open/closeEndocrinology Click to
open/closeBlood gases Click to
Change Password open/closeCSF
Sign Out

© 2011 PasTest Ltd | About Us | Contact Us | Help

file:///E|/Shakhawan/Endocrinology/125a.htm[3/13/2012 7:07:24 PM]


MyPasTest

Main Navigation
Question Browser: MRCP 1
Home
Subscriptions Question Browser Exam Builder Saved Exams
» MRCP 1 Session Progress
• Question Browser
Questions Correct 12
• Timed Test A 30-year-old man arrives in A&E in an unconscious state.
His initial blood glucose reading by monitor is 2.1 mmol/l. Questions Incorrect 3
• Mock Exam
What is the most important immediate management Questions Total 15
• Past Papers of this patient?
Questions Percentage 80 %
• Random Questions
• My Performance Oral glucose More
• Media Bank Mannitol

• New Multimedia 50% glucose into a large vein after first taking a
blood sample
Online Extras Reference: Normal Values
10 mg glucagon im
Library
Check his serum ethanol concentration Click to
Community open/closeHaematology
Help 2260 Click to
open/closeImmunoglobulins
PasTest Store
Click to
open/closeBiochemistry
My Account Click to open/closeDiabetes
Click to
Profile open/closeEndocrinology
Click to open/closeBlood
Newsletters gases Click to
My Career and Exams open/closeCSF
Order History
Learning Goals
Question Filters

Security

Change Password
Sign Out

© 2011 PasTest Ltd | About Us | Contact Us | Help

file:///E|/Shakhawan/Endocrinology/126.htm[3/13/2012 7:07:25 PM]


MyPasTest

Main Navigation
Question Browser: MRCP 1
Home
Subscriptions Question Browser Exam Builder Saved Exams
» MRCP 1 Session Progress
• Question Browser
Questions Correct 13
• Timed Test A 30-year-old man arrives in A&E in an unconscious state.
His initial blood glucose reading by monitor is 2.1 mmol/l. Questions Incorrect 3
• Mock Exam
What is the most important immediate management Questions Total 16
• Past Papers of this patient?
Questions Percentage 81 %
• Random Questions
• My Performance Oral glucose More
• Media Bank Mannitol

• New Multimedia 50% glucose into a large vein after Your answer
first taking a blood sample
Online Extras
10 mg glucagon im
Library
Check his serum ethanol concentration
Community
Help
PasTest Store
Hypoglycaemia is defined as a plasma glucose
concentration < 2.5 mmol/l and associated with
My Account symptoms of neuroglycopenia. Large-bore venous
access should be achieved and blood samples for serum
Profile glucose, liver function, ethanol, cortisol, insulin, C-
Newsletters peptide, proinsulin and sulphonylurea levels should be
sent for testing. If the patient’s Glasgow Coma Score
My Career and Exams (GCS) is < 13, then 25–50 ml 50% glucose should be
Order History given intravenously. Glucagon (1 mg im) may be Reference: Normal Values
administered if no intravenous access can be obtained.
Learning Goals
Click to open/closeHaematology
Question Filters 2260 Click to open/closeImmunoglobulins
Click to open/closeBiochemistry
Click to open/closeDiabetes Click to
Security open/closeEndocrinology Click to
open/closeBlood gases Click to
Change Password open/closeCSF
Sign Out

© 2011 PasTest Ltd | About Us | Contact Us | Help

file:///E|/Shakhawan/Endocrinology/126a.htm[3/13/2012 7:07:26 PM]


MyPasTest

Main Navigation
Question Browser: MRCP 1
Home
Subscriptions Question Browser Exam Builder Saved Exams
» MRCP 1 Session Progress
• Question Browser
Questions Correct 13
• Timed Test A 28-year-old man presents with erectile dysfunction.
What is the commonest cause in this age group? Questions Incorrect 3
• Mock Exam
Questions Total 16
• Past Papers
Alcohol Questions Percentage 81 %
• Random Questions
b-Blockers
• My Performance More
Diabetes mellitus
• Media Bank
Psychological factors
• New Multimedia
Testicular tumour
Online Extras Reference: Normal Values
Library 2261
Click to
Community open/closeHaematology
Help Click to
open/closeImmunoglobulins
PasTest Store
Click to
open/closeBiochemistry
My Account Click to open/closeDiabetes
Click to
Profile open/closeEndocrinology
Click to open/closeBlood
Newsletters gases Click to
My Career and Exams open/closeCSF
Order History
Learning Goals
Question Filters

Security

Change Password
Sign Out

© 2011 PasTest Ltd | About Us | Contact Us | Help

file:///E|/Shakhawan/Endocrinology/127.htm[3/13/2012 7:07:28 PM]


MyPasTest

Main Navigation
Question Browser: MRCP 1
Home
Subscriptions Question Browser Exam Builder Saved Exams
» MRCP 1 Session Progress
• Question Browser
Questions Correct 14
• Timed Test A 28-year-old man presents with erectile dysfunction.
What is the commonest cause in this age group? Questions Incorrect 3
• Mock Exam
Questions Total 17
• Past Papers
Alcohol Questions Percentage 82 %
• Random Questions
b-Blockers
• My Performance More
Diabetes mellitus
• Media Bank
Psychological factors Your answer
• New Multimedia
Testicular tumour
Online Extras
Library
Community
Help Erectile dysfunction is present in approximately 10% of
PasTest Store all men and >50% of men over the age of 70 years.
Common causes are psychological factors (20%), drugs
(25%) and endocrine causes. Baseline investigations
My Account include serum testosterone, prolactin, fasting glucose,
LH and FSH, thyroid function tests, liver and renal
Profile function, lipids and ferritin.
Newsletters
My Career and Exams 2261
Order History Reference: Normal Values
Learning Goals
Click to open/closeHaematology
Question Filters Click to open/closeImmunoglobulins
Click to open/closeBiochemistry
Click to open/closeDiabetes Click to
Security open/closeEndocrinology Click to
open/closeBlood gases Click to
Change Password open/closeCSF
Sign Out

© 2011 PasTest Ltd | About Us | Contact Us | Help

file:///E|/Shakhawan/Endocrinology/127a.htm[3/13/2012 7:07:29 PM]


MyPasTest

Main Navigation
Question Browser: MRCP 1
Home
Subscriptions Question Browser Exam Builder Saved Exams
» MRCP 1 Session Progress
• Question Browser
Questions Correct 14
• Timed Test Which of the following is the most likely long-term
consequence of the menopause? Questions Incorrect 3
• Mock Exam
Questions Total 17
• Past Papers
Decreased thrombotic tendency Questions Percentage 82 %
• Random Questions
Less likelihood of developing ischaemic heart disease
• My Performance (IHD) More
• Media Bank Increased possibility of developing Alzheimer’s
• New Multimedia dementia

Online Extras Increased bone mineral density (BMD)


Reference: Normal Values
Library Increased insulin sensitivity
Click to
Community 2262 open/closeHaematology
Help Click to
open/closeImmunoglobulins
PasTest Store
Click to
open/closeBiochemistry
My Account Click to open/closeDiabetes
Click to
Profile open/closeEndocrinology
Click to open/closeBlood
Newsletters gases Click to
My Career and Exams open/closeCSF
Order History
Learning Goals
Question Filters

Security

Change Password
Sign Out

© 2011 PasTest Ltd | About Us | Contact Us | Help

file:///E|/Shakhawan/Endocrinology/128.htm[3/13/2012 7:07:31 PM]


MyPasTest

Main Navigation
Question Browser: MRCP 1
Home
Subscriptions Question Browser Exam Builder Saved Exams
» MRCP 1 Session Progress
• Question Browser
Questions Correct 15
• Timed Test Which of the following is the most likely long-term
consequence of the menopause? Questions Incorrect 3
• Mock Exam
Questions Total 18
• Past Papers
Decreased thrombotic tendency Questions Percentage 83 %
• Random Questions
Less likelihood of developing ischaemic
• My Performance heart disease (IHD) More
• Media Bank Increased possibility of developing Your answer
• New Multimedia Alzheimer’s dementia

Online Extras Increased bone mineral density (BMD)

Library Increased insulin sensitivity

Community
Help
PasTest Store During the perimenopausal period there is an
accelerated loss of BMD, rendering postmenopausal
women more susceptible to osteoporotic fractures. They
My Account are also more likely to develop IHD, decreased insulin
sensitivity and increased thrombotic tendency. Women
Profile are 2–3 times more likely to develop Alzheimer’s
Newsletters disease than men. It is suggested that oestrogen
deficiency may play a role in the development of
My Career and Exams dementia.
Order History Reference: Normal Values
Learning Goals 2262
Click to open/closeHaematology
Question Filters Click to open/closeImmunoglobulins
Click to open/closeBiochemistry
Click to open/closeDiabetes Click to
Security open/closeEndocrinology Click to
open/closeBlood gases Click to
Change Password open/closeCSF
Sign Out

© 2011 PasTest Ltd | About Us | Contact Us | Help

file:///E|/Shakhawan/Endocrinology/128a.htm[3/13/2012 7:07:32 PM]


MyPasTest

Main Navigation
Question Browser: MRCP 1
Home
Subscriptions Question Browser Exam Builder Saved Exams
» MRCP 1 Session Progress
• Question Browser
Questions Correct 15
• Timed Test Which of the following in a 64-year-old man with
diabetes mellitus warrants urgent ophthalmology Questions Incorrect 3
• Mock Exam
referral? Questions Total 18
• Past Papers
Questions Percentage 83 %
• Random Questions Background diabetic retinopathy
• My Performance Vitreous haemorrhage More
• Media Bank Cataracts
• New Multimedia Drusen
Online Extras Non-proliferative changes in the periphery Reference: Normal Values
Library
2263 Click to
Community open/closeHaematology
Help Click to
open/closeImmunoglobulins
PasTest Store
Click to
open/closeBiochemistry
My Account Click to open/closeDiabetes
Click to
Profile open/closeEndocrinology
Click to open/closeBlood
Newsletters gases Click to
My Career and Exams open/closeCSF
Order History
Learning Goals
Question Filters

Security

Change Password
Sign Out

© 2011 PasTest Ltd | About Us | Contact Us | Help

file:///E|/Shakhawan/Endocrinology/129.htm[3/13/2012 7:07:34 PM]


MyPasTest

Main Navigation
Question Browser: MRCP 1
Home
Subscriptions Question Browser Exam Builder Saved Exams
» MRCP 1 Session Progress
• Question Browser
Questions Correct 16
• Timed Test Which of the following in a 64-year-old man with
diabetes mellitus warrants urgent ophthalmology Questions Incorrect 3
• Mock Exam
referral? Questions Total 19
• Past Papers
Questions Percentage 84 %
• Random Questions Background diabetic retinopathy
• My Performance Vitreous haemorrhage Your answer More
• Media Bank Cataracts
• New Multimedia Drusen
Online Extras Non-proliferative changes in the
Library periphery

Community
Help
PasTest Store Immediate referral is needed for patients with
proliferative retinopathy (as it carries a 40% risk of
blindness if untreated and laser treatment reduces this),
My Account rubeosis iridis, vitreous haemorrhage, advanced
retinopathy with fibrous tissue, retinal detachment or
Profile maculopathy.
Newsletters
2263
My Career and Exams
Order History Reference: Normal Values
Learning Goals
Click to open/closeHaematology
Question Filters Click to open/closeImmunoglobulins
Click to open/closeBiochemistry
Click to open/closeDiabetes Click to
Security open/closeEndocrinology Click to
open/closeBlood gases Click to
Change Password open/closeCSF
Sign Out

© 2011 PasTest Ltd | About Us | Contact Us | Help

file:///E|/Shakhawan/Endocrinology/129a.htm[3/13/2012 7:07:35 PM]


MyPasTest

Main Navigation
Question Browser: MRCP 1
Home
Subscriptions Question Browser Exam Builder Saved Exams
» MRCP 1 Session Progress
• Question Browser
Questions Correct 16
• Timed Test In which of the following coexisting medical
conditions would prescription of the oral Questions Incorrect 3
• Mock Exam
contraceptive pill (OCP) be acceptable? Questions Total 19
• Past Papers
Questions Percentage 84 %
• Random Questions Severe migraine
• My Performance History of venous thrombosis More
• Media Bank Liver disease
• New Multimedia Pulmonary hypertension
Online Extras Hypothyroidism Reference: Normal Values
Library
2264 Click to
Community open/closeHaematology
Help Click to
open/closeImmunoglobulins
PasTest Store
Click to
open/closeBiochemistry
My Account Click to open/closeDiabetes
Click to
Profile open/closeEndocrinology
Click to open/closeBlood
Newsletters gases Click to
My Career and Exams open/closeCSF
Order History
Learning Goals
Question Filters

Security

Change Password
Sign Out

© 2011 PasTest Ltd | About Us | Contact Us | Help

file:///E|/Shakhawan/Endocrinology/130.htm[3/13/2012 7:07:36 PM]


MyPasTest

Main Navigation
Question Browser: MRCP 1
Home
Subscriptions Question Browser Exam Builder Saved Exams
» MRCP 1 Session Progress
• Question Browser
Questions Correct 16
• Timed Test In which of the following coexisting medical
conditions would prescription of the oral Questions Incorrect 4
• Mock Exam
contraceptive pill (OCP) be acceptable? Questions Total 20
• Past Papers
Questions Percentage 80 %
• Random Questions Severe migraine
• My Performance History of venous thrombosis More
• Media Bank Liver disease Your answer
• New Multimedia Pulmonary hypertension
Online Extras Hypothyroidism Correct answer
Library
Community
Help Absolute contraindications for OCP includes a history of
PasTest Store heart disease, pulmonary hypertension, history of
arterial or venous thrombosis, history of
cerebrovascular disease, liver disease, severe migraine,
My Account breast or genital tract cancer and age over 35 years.
Hypothyroidism is not a contraindication to taking the
Profile OCP.
Newsletters
2264
My Career and Exams
Order History Reference: Normal Values
Learning Goals
Click to open/closeHaematology
Question Filters Click to open/closeImmunoglobulins
Click to open/closeBiochemistry
Click to open/closeDiabetes Click to
Security open/closeEndocrinology Click to
open/closeBlood gases Click to
Change Password open/closeCSF
Sign Out

© 2011 PasTest Ltd | About Us | Contact Us | Help

file:///E|/Shakhawan/Endocrinology/130a.htm[3/13/2012 7:07:38 PM]


MyPasTest

Main Navigation
Question Browser: MRCP 1
Home
Subscriptions Question Browser Exam Builder Saved Exams
» MRCP 1 Session Progress
• Question Browser
Questions Correct 16
• Timed Test A 28-year-old pregnant woman is detected to have positive
thyroid antibodies and is euthyroid. Which of the Questions Incorrect 4
• Mock Exam
following potential consequences should the Questions Total 20
• Past Papers obstetrician warn of during early pregnancy?
Questions Percentage 80 %
• Random Questions
• My Performance Higher risk of spontaneous abortions More
• Media Bank Increased risk of neonatal hypothyroidism

• New Multimedia Increased risk of pregnancy-induced hypertension


(PIH)
Online Extras Reference: Normal Values
Increased fetal size
Library
Increased risk of thyroid cancer in the mother Click to
Community open/closeHaematology
Help 2265 Click to
open/closeImmunoglobulins
PasTest Store
Click to
open/closeBiochemistry
My Account Click to open/closeDiabetes
Click to
Profile open/closeEndocrinology
Click to open/closeBlood
Newsletters gases Click to
My Career and Exams open/closeCSF
Order History
Learning Goals
Question Filters

Security

Change Password
Sign Out

© 2011 PasTest Ltd | About Us | Contact Us | Help

file:///E|/Shakhawan/Endocrinology/131.htm[3/13/2012 7:07:39 PM]


MyPasTest

Main Navigation
Question Browser: MRCP 1
Home
Subscriptions Question Browser Exam Builder Saved Exams
» MRCP 1 Session Progress
• Question Browser
Questions Correct 16
• Timed Test A 28-year-old pregnant woman is detected to have positive
thyroid antibodies and is euthyroid. Which of the Questions Incorrect 5
• Mock Exam
following potential consequences should the Questions Total 21
• Past Papers obstetrician warn of during early pregnancy?
Questions Percentage 76 %
• Random Questions
• My Performance Higher risk of spontaneous abortions Correct answer More
• Media Bank Increased risk of neonatal Your answer
hypothyroidism
• New Multimedia
Increased risk of pregnancy-induced
Online Extras hypertension (PIH)
Library Increased fetal size
Community Increased risk of thyroid cancer in
Help the mother
PasTest Store

My Account
A pregnant mother with positive thyroid antibodies, but
Profile who is euthyroid, has a higher risk of spontaneous
abortions. There is no risk of neonatal hypothyroidism
Newsletters
and the risk of PIH is not increased. The occasional
My Career and Exams mother will develop hypothyroidism at the end of
Order History pregnancy, so the TSH level should be checked between Reference: Normal Values
28 and 32 weeks’ gestation, then at 3 months’
Learning Goals postpartum. Click to open/closeHaematology
Question Filters Click to open/closeImmunoglobulins
Click to open/closeBiochemistry
2265 Click to open/closeDiabetes Click to
Security open/closeEndocrinology Click to
open/closeBlood gases Click to
Change Password open/closeCSF
Sign Out

© 2011 PasTest Ltd | About Us | Contact Us | Help

file:///E|/Shakhawan/Endocrinology/131a.htm[3/13/2012 7:07:40 PM]


MyPasTest

Main Navigation
Question Browser: MRCP 1
Home
Subscriptions Question Browser Exam Builder Saved Exams
» MRCP 1 Session Progress
• Question Browser
Questions Correct 16
• Timed Test A 26-year-old woman presents with symptoms of flushing,
diarrhoea and wheezing. After examination a clinical Questions Incorrect 5
• Mock Exam
diagnosis of carcinoid syndrome is suspected. Which of Questions Total 21
• Past Papers the following tests would be the most sensitive
Questions Percentage 76 %
• Random Questions marker for carcinoid syndrome?
• My Performance More
Urinary 5-HIAA
• Media Bank
Alkaline phosphatase
• New Multimedia
Echocardiography
Online Extras Reference: Normal Values
Plasma chromogranin A
Library
Plasma gut-hormone profile Click to
Community open/closeHaematology
Help 2266 Click to
open/closeImmunoglobulins
PasTest Store
Click to
open/closeBiochemistry
My Account Click to open/closeDiabetes
Click to
Profile open/closeEndocrinology
Click to open/closeBlood
Newsletters gases Click to
My Career and Exams open/closeCSF
Order History
Learning Goals
Question Filters

Security

Change Password
Sign Out

© 2011 PasTest Ltd | About Us | Contact Us | Help

file:///E|/Shakhawan/Endocrinology/132.htm[3/13/2012 7:07:42 PM]


MyPasTest

Main Navigation
Question Browser: MRCP 1
Home
Subscriptions Question Browser Exam Builder Saved Exams
» MRCP 1 Session Progress
• Question Browser
Questions Correct 17
• Timed Test A 26-year-old woman presents with symptoms of flushing,
diarrhoea and wheezing. After examination a clinical Questions Incorrect 5
• Mock Exam
diagnosis of carcinoid syndrome is suspected. Which of Questions Total 22
• Past Papers the following tests would be the most sensitive
Questions Percentage 77 %
• Random Questions marker for carcinoid syndrome?
• My Performance More
Urinary 5-HIAA
• Media Bank
Alkaline phosphatase
• New Multimedia
Echocardiography
Online Extras
Plasma chromogranin A Your answer
Library
Plasma gut-hormone profile
Community
Help
PasTest Store
The most sensitive marker for carcinoid syndrome is
My Account plasma chromogranin A, which has been found in 100%
of patients. Although the specificity is lower than for 5-
Profile HIAA (5-hydroxyindoleacetic acid), most tumours with
neuroendocrine differentiation are associated with
Newsletters
increased levels of plasma chromogranin A. This test is
My Career and Exams not widely available for clinical use. Urinary 5-HIAA has
Order History a sensitivity of about 70% and a specificity of 100% in Reference: Normal Values
patients with carcinoid syndrome. Alkaline phosphatase
Learning Goals may remain normal despite liver involvement and a Click to open/closeHaematology
Question Filters plasma gut-hormone profile may be useful. Click to open/closeImmunoglobulins
Click to open/closeBiochemistry
Click to open/closeDiabetes Click to
Security 2266 open/closeEndocrinology Click to
open/closeBlood gases Click to
Change Password open/closeCSF
Sign Out

© 2011 PasTest Ltd | About Us | Contact Us | Help

file:///E|/Shakhawan/Endocrinology/132a.htm[3/13/2012 7:07:43 PM]


MyPasTest

Main Navigation
Question Browser: MRCP 1
Home
Subscriptions Question Browser Exam Builder Saved Exams
» MRCP 1 Session Progress
• Question Browser
Questions Correct 0
• Timed Test What is the most appropriate investigation to confirm
a diagnosis of acromegaly biochemically after initial Questions Incorrect 0
• Mock Exam
screening? Questions Total 0
• Past Papers
Questions Percentage 0%
• Random Questions Increased IGF-1
• My Performance Random growth hormone (GH) assay More
• Media Bank Insulin tolerance test
• New Multimedia Thyroid function test
Online Extras Oral glucose tolerance test (OGGT) with growth Reference: Normal Values
Library hormone assay
Click to
Community 2267 open/closeHaematology
Help Click to
open/closeImmunoglobulins
PasTest Store
Click to
open/closeBiochemistry
My Account Click to open/closeDiabetes
Click to
Profile open/closeEndocrinology
Click to open/closeBlood
Newsletters gases Click to
My Career and Exams open/closeCSF
Order History
Learning Goals
Question Filters

Security

Change Password
Sign Out

© 2011 PasTest Ltd | About Us | Contact Us | Help

file:///E|/Shakhawan/Endocrinology/133.htm[3/13/2012 7:07:45 PM]


MyPasTest

Main Navigation
Question Browser: MRCP 1
Home
Subscriptions Question Browser Exam Builder Saved Exams
» MRCP 1 Session Progress
• Question Browser
Questions Correct 1
• Timed Test What is the most appropriate investigation to confirm
a diagnosis of acromegaly biochemically after initial Questions Incorrect 0
• Mock Exam
screening? Questions Total 1
• Past Papers
Questions Percentage 100 %
• Random Questions Increased IGF-1
• My Performance Random growth hormone (GH) assay More
• Media Bank Insulin tolerance test
• New Multimedia Thyroid function test
Online Extras Oral glucose tolerance test (OGGT) Your answer
Library with growth hormone assay

Community
Help
PasTest Store
In acromegaly, there is a failure to suppress GH to < 2
mU/l in response to a 75-g oral glucose load. In
My Account contrast, the normal response is GH suppression to
undetectable levels. Random GH is not useful in the
Profile diagnosis of acromegaly because, although normal
subjects have undetectable growth hormone levels
Newsletters
throughout the day, there are pulses of GH which are
My Career and Exams impossible to differentiate from the levels seen in
Order History acromegaly. Following a TRH (thyrotrophin-releasing Reference: Normal Values
hormone) test, 80% of patients with acromegaly show
Learning Goals increased levels of GH. IGF-1 is useful for inital Click to open/closeHaematology
Question Filters screening for acromegaly, as it provides an integrated Click to open/closeImmunoglobulins
measure of growth hormone levels over time. Click to open/closeBiochemistry
Click to open/closeDiabetes Click to
Security open/closeEndocrinology Click to
2267 open/closeBlood gases Click to
Change Password open/closeCSF
Sign Out

© 2011 PasTest Ltd | About Us | Contact Us | Help

file:///E|/Shakhawan/Endocrinology/133a.htm[3/13/2012 7:07:46 PM]


MyPasTest

Main Navigation
Question Browser: MRCP 1
Home
Subscriptions Question Browser Exam Builder Saved Exams
» MRCP 1 Session Progress
• Question Browser
Questions Correct 1
• Timed Test An 18-week pregnant woman presents to hospital with
failure to gain weight, hyperemesis and persistent Questions Incorrect 0
• Mock Exam
tachycardia. She is found to be thyrotoxic with a Questions Total 1
• Past Papers suppressed TSH of 0.05.
Questions Percentage 100 %
• Random Questions
Which of the following is the best management step? More
• My Performance
• Media Bank Radioiodine
• New Multimedia Immediate surgical referral
Online Extras Block-and-replace regime Reference: Normal Values
Library Propylthiouracil
Click to
Community Observe and wait for normalisation open/closeHaematology
Help Click to
2268 open/closeImmunoglobulins
PasTest Store
Click to
open/closeBiochemistry
My Account Click to open/closeDiabetes
Click to
Profile open/closeEndocrinology
Click to open/closeBlood
Newsletters gases Click to
My Career and Exams open/closeCSF
Order History
Learning Goals
Question Filters

Security

Change Password
Sign Out

© 2011 PasTest Ltd | About Us | Contact Us | Help

file:///E|/Shakhawan/Endocrinology/134.htm[3/13/2012 7:07:48 PM]


MyPasTest

Main Navigation
Question Browser: MRCP 1
Home
Subscriptions Question Browser Exam Builder Saved Exams
» MRCP 1 Session Progress
• Question Browser
Questions Correct 2
• Timed Test An 18-week pregnant woman presents to hospital with
failure to gain weight, hyperemesis and persistent Questions Incorrect 0
• Mock Exam
tachycardia. She is found to be thyrotoxic with a Questions Total 2
• Past Papers suppressed TSH of 0.05.
Questions Percentage 100 %
• Random Questions
Which of the following is the best management step? More
• My Performance
• Media Bank Radioiodine
• New Multimedia Immediate surgical referral
Online Extras Block-and-replace regime
Library Propylthiouracil Your answer
Community Observe and wait for normalisation
Help
PasTest Store
The aim of treatment of thyrotoxicosis in pregnancy is
My Account the alleviation of symptoms and normalisation of tests
in the shortest time. Both propylthiouracil and
Profile carbimazole are effective in controlling disease in
pregnancy. The use of radioiodine is contraindicated in
Newsletters
pregnancy. Surgery is rarely performed during
My Career and Exams pregnancy and is reserved for patients who do not
Order History respond to antithyroid drugs. A block-and-replace Reference: Normal Values
regime should not be used as this will result in fetal
Learning Goals hypothyroidism. This is a different scenario to that of Click to open/closeHaematology
Question Filters hyperemesis and suppressed TSH, which is seen in the Click to open/closeImmunoglobulins
first trimester and usually resolves. Click to open/closeBiochemistry
Click to open/closeDiabetes Click to
Security 2268 open/closeEndocrinology Click to
open/closeBlood gases Click to
Change Password open/closeCSF
Sign Out

© 2011 PasTest Ltd | About Us | Contact Us | Help

file:///E|/Shakhawan/Endocrinology/134a.htm[3/13/2012 7:07:49 PM]


MyPasTest

Main Navigation
Question Browser: MRCP 1
Home
Subscriptions Question Browser Exam Builder Saved Exams
» MRCP 1 Session Progress
• Question Browser
Questions Correct 2
• Timed Test A 34-year-old woman presents with 4-month history of
anorexia and weight loss, excess pigmentation and Questions Incorrect 0
• Mock Exam
dizziness on standing. Initial investigations reveal Questions Total 2
• Past Papers hyponatraemia and hyperkalaemia. Which of the
Questions Percentage 100 %
• Random Questions following tests will be most useful to confirm the
diagnosis? More
• My Performance
• Media Bank ESR
• New Multimedia Thyroid function tests
Online Extras Serum urea Reference: Normal Values
Library Short Synacthen test
Click to
Community Serum calcium open/closeHaematology
Help Click to
2269 open/closeImmunoglobulins
PasTest Store
Click to
open/closeBiochemistry
My Account Click to open/closeDiabetes
Click to
Profile open/closeEndocrinology
Click to open/closeBlood
Newsletters gases Click to
My Career and Exams open/closeCSF
Order History
Learning Goals
Question Filters

Security

Change Password
Sign Out

© 2011 PasTest Ltd | About Us | Contact Us | Help

file:///E|/Shakhawan/Endocrinology/135.htm[3/13/2012 7:07:50 PM]


MyPasTest

Main Navigation
Question Browser: MRCP 1
Home
Subscriptions Question Browser Exam Builder Saved Exams
» MRCP 1 Session Progress
• Question Browser
Questions Correct 3
• Timed Test A 34-year-old woman presents with 4-month history of
anorexia and weight loss, excess pigmentation and Questions Incorrect 0
• Mock Exam
dizziness on standing. Initial investigations reveal Questions Total 3
• Past Papers hyponatraemia and hyperkalaemia. Which of the
Questions Percentage 100 %
• Random Questions following tests will be most useful to confirm the
diagnosis? More
• My Performance
• Media Bank ESR
• New Multimedia Thyroid function tests
Online Extras Serum urea
Library Short Synacthen test Your answer
Community Serum calcium
Help
PasTest Store

My Account Hyponatraemia is present in 90% and hyperkalemia in


65% of patients with primary adrenal insufficiency.
Profile Although increased urea, increased ESR and raised TSH
levels and mild hypercalcaemia are seen in Addison’s
Newsletters
disease, these are not particularly useful in confirming
My Career and Exams the diagnosis. However, failure to respond following a
Order History short Synacthen test suggests adrenal failure. Reference: Normal Values
Learning Goals
Click to open/closeHaematology
Question Filters 2269 Click to open/closeImmunoglobulins
Click to open/closeBiochemistry
Click to open/closeDiabetes Click to
Security open/closeEndocrinology Click to
open/closeBlood gases Click to
Change Password open/closeCSF
Sign Out

© 2011 PasTest Ltd | About Us | Contact Us | Help

file:///E|/Shakhawan/Endocrinology/135a.htm[3/13/2012 7:07:52 PM]


MyPasTest

Main Navigation
Question Browser: MRCP 1
Home
Subscriptions Question Browser Exam Builder Saved Exams
» MRCP 1 Session Progress
• Question Browser
Questions Correct 3
• Timed Test A 72-year-old man with a long history of COPD was
admitted with pneumonia. Prior to admission he had Questions Incorrect 0
• Mock Exam
become increasingly confused. On examination he was Questions Total 3
• Past Papers drowsy, had a BP of 142/75 mmHg and was clinically
Questions Percentage 100 %
• Random Questions euvolaemic. Chest auscultation was consistent with
pneumonia. Blood biochemistry investigations revealed Na More
• My Performance
121 mmol/l, K 3.9 mmol/l, urea 2.4 mmol/l, creatinine 64
• Media Bank µmol/l and glucose 4.2 mmol/l. Plasma osmolarity was 261
• New Multimedia mOsmol/kg, and thyroid function testing was normal. Urine
testing revealed an osmolality of 560 mOsmol/kg and a
Online Extras sodium concentration of 55 mmol/l. Reference: Normal Values
Library
What was the most likely cause of his Click to
Community hyponatraemia? open/closeHaematology
Help Click to
Addison’s disease open/closeImmunoglobulins
PasTest Store
Renal failure Click to
open/closeBiochemistry
Cardiac failure Click to open/closeDiabetes
My Account
Cirrhosis Click to
Profile open/closeEndocrinology
Syndrome of inappropriate ADH secretion (SIADH) Click to open/closeBlood
Newsletters gases Click to
My Career and Exams 2396 open/closeCSF
Order History
Learning Goals
Question Filters

Security

Change Password
Sign Out

© 2011 PasTest Ltd | About Us | Contact Us | Help

file:///E|/Shakhawan/Endocrinology/136.htm[3/13/2012 7:07:53 PM]


MyPasTest

Main Navigation
Question Browser: MRCP 1
Home
Subscriptions Question Browser Exam Builder Saved Exams
» MRCP 1 Session Progress
• Question Browser
Questions Correct 4
• Timed Test A 72-year-old man with a long history of COPD was
admitted with pneumonia. Prior to admission he had Questions Incorrect 0
• Mock Exam
become increasingly confused. On examination he was Questions Total 4
• Past Papers drowsy, had a BP of 142/75 mmHg and was clinically
Questions Percentage 100 %
• Random Questions euvolaemic. Chest auscultation was consistent with
pneumonia. Blood biochemistry investigations revealed Na More
• My Performance
121 mmol/l, K 3.9 mmol/l, urea 2.4 mmol/l, creatinine 64
• Media Bank µmol/l and glucose 4.2 mmol/l. Plasma osmolarity was 261
• New Multimedia mOsmol/kg, and thyroid function testing was normal. Urine
testing revealed an osmolality of 560 mOsmol/kg and a
Online Extras sodium concentration of 55 mmol/l.
Library
What was the most likely cause of his
Community hyponatraemia?
Help
Addison’s disease
PasTest Store
Renal failure
Cardiac failure
My Account
Cirrhosis
Profile
Syndrome of inappropriate ADH Your answer
Newsletters secretion (SIADH)
My Career and Exams
Order History Reference: Normal Values
Learning Goals This is a not uncommon consequence of pneumonia, Click to open/closeHaematology
Question Filters particularly in patients of this age group. Small-cell Click to open/closeImmunoglobulins
bronchial carcinoma is another cause of SIADH. Click to open/closeBiochemistry
Diagnosis is made on an inappropriately high urine Click to open/closeDiabetes Click to
Security osmolality compared to plasma osmolality, urinary open/closeEndocrinology Click to
sodium concentration above 30 mmol/l, euvolaemia and open/closeBlood gases Click to
Change Password normal thyroid and adrenal function. The normal open/closeCSF
Sign Out potassium level here makes Addison’s disease unlikely.
A number of drugs can cause of SIADH, including
chlorpropamide, phenothiazines and carbamazepine.
Both fluid restriction and demeclocycline precipitate
renal diabetes insipidus, which may be of assistance in
the treatment of SIADH.

2396

© 2011 PasTest Ltd | About Us | Contact Us | Help

file:///E|/Shakhawan/Endocrinology/136a.htm[3/13/2012 7:07:55 PM]


MyPasTest

Main Navigation
Question Browser: MRCP 1
Home
Subscriptions Question Browser Exam Builder Saved Exams
» MRCP 1 Session Progress
• Question Browser
Questions Correct 4
• Timed Test A 45-year-old woman is due to undergo a hysterectomy.
She has a previous history of hypertension, headaches and Questions Incorrect 0
• Mock Exam
panic attacks over the past few years. She currently takes Questions Total 4
• Past Papers ramipril for her blood pressure. The preoperative
Questions Percentage 100 %
• Random Questions examination revealed a blood pressure of 150/85 mmHg,
normal renal function and calcium concentration, an ECG More
• My Performance
showed mild LVH. Unfortunately during the operation, as
• Media Bank the surgeon attempts to mobilise her uterus, her blood
• New Multimedia pressure rises to 210/110 mmHg, her pulse to 130 bpm
and she suffers an acute myocardial infarction. What is the
Online Extras most likely cause of her intraoperative hypertension Reference: Normal Values
Library and myocardial infarction?
Click to
Community open/closeHaematology
Essential hypertension
Help Click to
Occult coronary artery disease
open/closeImmunoglobulins
PasTest Store
Undiagnosed phaeochromocytoma Click to
MEN-1 open/closeBiochemistry
My Account Click to open/closeDiabetes
Renal artery stenosis Click to
Profile open/closeEndocrinology
2397 Click to open/closeBlood
Newsletters gases Click to
My Career and Exams open/closeCSF
Order History
Learning Goals
Question Filters

Security

Change Password
Sign Out

© 2011 PasTest Ltd | About Us | Contact Us | Help

file:///E|/Shakhawan/Endocrinology/137.htm[3/13/2012 7:07:56 PM]


MyPasTest

Main Navigation
Question Browser: MRCP 1
Home
Subscriptions Question Browser Exam Builder Saved Exams
» MRCP 1 Session Progress
• Question Browser
Questions Correct 5
• Timed Test A 45-year-old woman is due to undergo a hysterectomy.
She has a previous history of hypertension, headaches and Questions Incorrect 0
• Mock Exam
panic attacks over the past few years. She currently takes Questions Total 5
• Past Papers ramipril for her blood pressure. The preoperative
Questions Percentage 100 %
• Random Questions examination revealed a blood pressure of 150/85 mmHg,
normal renal function and calcium concentration, an ECG More
• My Performance
showed mild LVH. Unfortunately during the operation, as
• Media Bank the surgeon attempts to mobilise her uterus, her blood
• New Multimedia pressure rises to 210/110 mmHg, her pulse to 130 bpm
and she suffers an acute myocardial infarction. What is the
Online Extras most likely cause of her intraoperative hypertension
Library and myocardial infarction?
Community
Essential hypertension
Help
Occult coronary artery disease
PasTest Store
Undiagnosed phaeochromocytoma Your answer
MEN-1
My Account Renal artery stenosis
Profile
Newsletters
My Career and Exams Given her history, she is likely to have been suffering
Order History paroxysms of catecholamine release. Mobilisation of the Reference: Normal Values
uterus is likely to have precipitated an acute release of
Learning Goals large amounts of catecholamines, causing her Click to open/closeHaematology
Question Filters catastrophic rise in blood pressure and intraoperative Click to open/closeImmunoglobulins
myocardial infarction. Standard therapy for Click to open/closeBiochemistry
phaeochromocytoma is aggressive α -blockade prior to Click to open/closeDiabetes Click to
Security surgery, then surgical excision. The 5-year prognosis open/closeEndocrinology Click to
for benign tumours approaches 95%, but this falls to open/closeBlood gases Click to
Change Password open/closeCSF
around 40% in patients with malignant disease.
Sign Out
2397

© 2011 PasTest Ltd | About Us | Contact Us | Help

file:///E|/Shakhawan/Endocrinology/137a.htm[3/13/2012 7:07:57 PM]


MyPasTest

Main Navigation
Question Browser: MRCP 1
Home
Subscriptions Question Browser Exam Builder Saved Exams
» MRCP 1 Session Progress
• Question Browser
Questions Correct 5
• Timed Test A marfanoid-looking, 21-year-old man is referred by his GP
for review. He has been suffering from acute headaches Questions Incorrect 0
• Mock Exam
and panic attacks and is unable to continue his course work Questions Total 5
• Past Papers at university. On examination his blood pressure is 148/98
Questions Percentage 100 %
• Random Questions mmHg and you notice a number of neuromas around his
lips. His 24-h urinary catecholamine levels are raised. More
• My Performance
• Media Bank What diagnosis fits best with this clinical picture?
• New Multimedia
Multiple endocrine neoplasia (MEN)-1
Online Extras Reference: Normal Values
Multiple endocrine neoplasia (MEN)-2a
Library
Neurofibromatosis Click to
Community open/closeHaematology
Multiple endocrine neoplasia (MEN)-2b
Help Click to
Carcinoid syndrome open/closeImmunoglobulins
PasTest Store
Click to
2398 open/closeBiochemistry
My Account Click to open/closeDiabetes
Click to
Profile open/closeEndocrinology
Click to open/closeBlood
Newsletters gases Click to
My Career and Exams open/closeCSF
Order History
Learning Goals
Question Filters

Security

Change Password
Sign Out

© 2011 PasTest Ltd | About Us | Contact Us | Help

file:///E|/Shakhawan/Endocrinology/138.htm[3/13/2012 7:07:59 PM]


MyPasTest

Main Navigation
Question Browser: MRCP 1
Home
Subscriptions Question Browser Exam Builder Saved Exams
» MRCP 1
• Question Browser
• Timed Test A marfanoid-looking, 21-year-old man is referred by his GP for review. He has been
• Mock Exam suffering from acute headaches and panic attacks and is unable to continue his course
work at university. On examination his blood pressure is 148/98 mmHg and you notice
• Past Papers a number of neuromas around his lips. His 24-h urinary catecholamine levels are
• Random Questions raised.
• My Performance What diagnosis fits best with this clinical picture?
• Media Bank
• New Multimedia Multiple endocrine neoplasia (MEN)-1

Online Extras Multiple endocrine neoplasia (MEN)-2a

Library Neurofibromatosis

Community Multiple endocrine neoplasia (MEN)-2b Your answer

Help Carcinoid syndrome

PasTest Store

My Account MEN-2b is characterised by a marfanoid habitus, visceral and intestinal


ganglioneuromas (which may occur around the lips and tongue), adrenal tumours,
Profile and medullary thyroid carcinoma, parathyroid hyperplasia occurs much more rarely
than in MEN-2a. Adrenal tumours leading to Cushing’s syndrome or
Newsletters
phaeochromocytomas (70% bilateral) may occur. Screening is recommended for
My Career and Exams MEN-2: total thyroidectomy in childhood is often recommended for those with a
Order History known gene defect. Screening for medullary thyroid carcinoma is via the
pentagastrin test. The best screen for phaeochromocytoma is a 24-h urinary
Learning Goals catecholamine estimation.
Question Filters
2398

Security

Change Password
Sign Out

file:///E|/Shakhawan/Endocrinology/138a.htm[3/13/2012 7:08:00 PM]


MyPasTest

Main Navigation
Question Browser: MRCP 1
Home
Subscriptions Question Browser Exam Builder Saved Exams
» MRCP 1 Session Progress
• Question Browser
Questions Correct 6
• Timed Test A 45-year-old man presents for review at the type-2
diabetes clinic. He is on maximal metformin and his Hb A 1c Questions Incorrect 0
• Mock Exam
is still 7.8%. You elect to add in a peroxisome proliferator- Questions Total 6
• Past Papers
activated receptor (PPAR)-gamma agonist, pioglitazone. Questions Percentage 100 %
• Random Questions
• My Performance Which of the following best describes the mode of More
action of PPAR-γ agonists?
• Media Bank
• New Multimedia They act at the PPAR- γ receptor site, promoting
Online Extras binding as a heterodimer with the retinoid X-receptor Reference: Normal Values
to DNA
Library
They bind to a promoter region of DNA as a Click to
Community heterodimer with the retinoid A-receptor open/closeHaematology
Help Click to
They act by reducing peripheral insulin sensitivity
open/closeImmunoglobulins
PasTest Store They act by stimulating insulin output Click to
They act via receptors at the cell surface open/closeBiochemistry
My Account Click to open/closeDiabetes
2399 Click to
Profile open/closeEndocrinology
Click to open/closeBlood
Newsletters gases Click to
My Career and Exams open/closeCSF
Order History
Learning Goals
Question Filters

Security

Change Password
Sign Out

© 2011 PasTest Ltd | About Us | Contact Us | Help

file:///E|/Shakhawan/Endocrinology/139.htm[3/13/2012 7:08:02 PM]


MyPasTest

Main Navigation
Question Browser: MRCP 1
Home
Subscriptions Question Browser Exam Builder Saved Exams
» MRCP 1 Session Progress
• Question Browser
Questions Correct 7
• Timed Test A 45-year-old man presents for review at the type-2
Questions Incorrect 0
• Mock Exam diabetes clinic. He is on maximal metformin and his Hb A 1c
is still 7.8%. You elect to add in a peroxisome proliferator- Questions Total 7
• Past Papers
activated receptor (PPAR)-gamma agonist, pioglitazone. Questions Percentage 100 %
• Random Questions
• My Performance Which of the following best describes the mode of More
action of PPAR-γ agonists?
• Media Bank
• New Multimedia They act at the PPAR- γ receptor site, Your answer
Online Extras promoting binding as a heterodimer with
the retinoid X-receptor to DNA
Library
They bind to a promoter region of DNA
Community as a heterodimer with the retinoid A-
Help receptor
PasTest Store They act by reducing peripheral insulin
sensitivity
They act by stimulating insulin output
My Account
They act via receptors at the cell
Profile surface
Newsletters
My Career and Exams
Order History Reference: Normal Values
PPAR- γ agonists act by binding to the PPAR-gamma
Learning Goals receptor, which binds to a promoter region of DNA in
Click to open/closeHaematology
Question Filters tandem with the retinoid X-receptor. This then leads to
Click to open/closeImmunoglobulins
the upregulation of a number of enzymes concerned
Click to open/closeBiochemistry
with lipid metabolism, bringing about a fall in free fatty
Click to open/closeDiabetes Click to
Security acids. The fall in free fatty acids promotes a reduction in
open/closeEndocrinology Click to
hepatic insulin resistance, a rise in adiponectin and
open/closeBlood gases Click to
Change Password improved peripheral insulin sensitivity. Retinoid X-
open/closeCSF
receptors are also now under investigation for their
Sign Out
possible action in improving insulin sensitivity.
Currently, there is one PPAR- γ agonist on the market,
pioglitazone.

2399

© 2011 PasTest Ltd | About Us | Contact Us | Help

file:///E|/Shakhawan/Endocrinology/139a.htm[3/13/2012 7:08:03 PM]


MyPasTest

Main Navigation
Question Browser: MRCP 1
Home
Subscriptions Question Browser Exam Builder Saved Exams
» MRCP 1 Session Progress
• Question Browser
Questions Correct 7
• Timed Test A 52-year-old woman is referred by the A&E department
after attending with a Colles’ fracture, the second in the Questions Incorrect 0
• Mock Exam
last 3 years. She underwent a total hysterectomy at the Questions Total 7
• Past Papers age of 38 for carcinoma. Bone densitometry confirms
Questions Percentage 100 %
• Random Questions osteoporosis. The calcium and parathyroid hormone assays
are normal. Which of the following is the most likely More
• My Performance
concerning her underlying pathophysiology?
• Media Bank
• New Multimedia She has suffered predominantly cortical bone loss
Online Extras She has probably suffered an equal mix of cortical Reference: Normal Values
and trabecular bone loss
Library
She has probably suffered predominantly trabecular Click to
Community bone loss open/closeHaematology
Help Underlying hyperparathyroidism will have contributed Click to
open/closeImmunoglobulins
PasTest Store Underlying hypoparathyroidism will have contributed Click to
open/closeBiochemistry
2400 Click to open/closeDiabetes
My Account
Click to
Profile open/closeEndocrinology
Click to open/closeBlood
Newsletters gases Click to
My Career and Exams open/closeCSF
Order History
Learning Goals
Question Filters

Security

Change Password
Sign Out

© 2011 PasTest Ltd | About Us | Contact Us | Help

file:///E|/Shakhawan/Endocrinology/140.htm[3/13/2012 7:08:05 PM]


MyPasTest

Main Navigation
Question Browser: MRCP 1
Home
Subscriptions Question Browser Exam Builder Saved Exams
» MRCP 1 Session Progress
• Question Browser
Questions Correct 7
• Timed Test A 52-year-old woman is referred by the A&E department
after attending with a Colles’ fracture, the second in the Questions Incorrect 1
• Mock Exam
last 3 years. She underwent a total hysterectomy at the Questions Total 8
• Past Papers age of 38 for carcinoma. Bone densitometry confirms
Questions Percentage 87 %
• Random Questions osteoporosis. The calcium and parathyroid hormone assays
are normal. Which of the following is the most likely More
• My Performance
concerning her underlying pathophysiology?
• Media Bank
• New Multimedia She has suffered predominantly
cortical bone loss
Online Extras
She has probably suffered an equal Your answer
Library mix of cortical and trabecular bone
Community loss
Help She has probably suffered Correct answer
predominantly trabecular bone loss
PasTest Store
Underlying hyperparathyroidism will
have contributed
My Account Underlying hypoparathyroidism will
Profile have contributed

Newsletters
My Career and Exams
Order History Reference: Normal Values
Hypo-oestrogenisation, as in this case, is usually
Learning Goals characterised by predominantly trabecular bone loss. Click to open/closeHaematology
Question Filters Involutional bone loss of old age is characterised by a Click to open/closeImmunoglobulins
mixed picture of trabecular and cortical bone loss. In Click to open/closeBiochemistry
the presence of normal calcium and parathyroid Click to open/closeDiabetes Click to
Security hormone levels, parathyroid disease is somewhat open/closeEndocrinology Click to
unlikely. Hormone replacement therapy was formerly open/closeBlood gases Click to
Change Password the mainstay of treatment, but this has fallen out of open/closeCSF
Sign Out favour due to increased thromboembolic disease and
the risk of breast carcinoma. Bisphosphonates are now
the favoured treatment modality in this group of
patients.

2400

© 2011 PasTest Ltd | About Us | Contact Us | Help

file:///E|/Shakhawan/Endocrinology/140a.htm[3/13/2012 7:08:06 PM]


MyPasTest

Main Navigation
Question Browser: MRCP 1
Home
Subscriptions Question Browser Exam Builder Saved Exams
» MRCP 1 Session Progress
• Question Browser
Questions Correct 7
• Timed Test A 32-year-old aromatherapist is referred to the endocrine
clinic for review. She has been suffering intermittent Questions Incorrect 1
• Mock Exam
tachycardias and panic attacks. There is no significant past Questions Total 8
• Past Papers medical history. She takes a number of vitamin and mineral
Questions Percentage 87 %
• Random Questions supplements, including kelp. On examination there are no
eye signs and no goitre. TSH is < 0.05 mU/l and thyroid More
• My Performance
antibodies are negative. What is the most likely cause
• Media Bank of her thyrotoxicosis?
• New Multimedia
Grave’s disease
Online Extras Reference: Normal Values
Toxic multinodular goitre
Library
Solitary toxic nodule Click to
Community open/closeHaematology
Excess iodine ingestion
Help Click to
Excess TSH secretion open/closeImmunoglobulins
PasTest Store
Click to
2401 open/closeBiochemistry
My Account Click to open/closeDiabetes
Click to
Profile open/closeEndocrinology
Click to open/closeBlood
Newsletters gases Click to
My Career and Exams open/closeCSF
Order History
Learning Goals
Question Filters

Security

Change Password
Sign Out

© 2011 PasTest Ltd | About Us | Contact Us | Help

file:///E|/Shakhawan/Endocrinology/141.htm[3/13/2012 7:08:08 PM]


MyPasTest

Main Navigation
Question Browser: MRCP 1
Home
Subscriptions Question Browser Exam Builder Saved Exams
» MRCP 1 Session Progress
• Question Browser
Questions Correct 7
• Timed Test A 32-year-old aromatherapist is referred to the endocrine
clinic for review. She has been suffering intermittent Questions Incorrect 2
• Mock Exam
tachycardias and panic attacks. There is no significant past Questions Total 9
• Past Papers medical history. She takes a number of vitamin and mineral
Questions Percentage 77 %
• Random Questions supplements, including kelp. On examination there are no
eye signs and no goitre. TSH is < 0.05 mU/l and thyroid More
• My Performance
antibodies are negative. What is the most likely cause
• Media Bank of her thyrotoxicosis?
• New Multimedia
Grave’s disease Your answer
Online Extras
Toxic multinodular goitre
Library
Solitary toxic nodule
Community
Excess iodine ingestion Correct answer
Help
Excess TSH secretion
PasTest Store

My Account

Profile Kelp is a very rich source of iodine. In patients with


pre-existing thyroid hyperplasia or adenoma, the
Newsletters
ingestion of large amounts of iodine may precipitate
My Career and Exams thyrotoxicosis. Treatment is withdrawal of the kelp with
Order History monitoring of thyroid function. Thyrotoxicosis affects Reference: Normal Values
2% of women and 0.2% of men in their lifetimes.
Learning Goals Grave’s disease is responsible for around 80% of cases Click to open/closeHaematology
Question Filters of thyrotoxicosis, toxic multinodular goitre is the next Click to open/closeImmunoglobulins
commonest cause. Patients with Grave’s disease may be Click to open/closeBiochemistry
offered radioiodine, if suitable, or a period of medical Click to open/closeDiabetes Click to
Security therapy with antithyroid drugs and thyroxine open/closeEndocrinology Click to
replacement. Patients who then fail on medical therapy open/closeBlood gases Click to
Change Password are offered radiotherapy or surgery. open/closeCSF
Sign Out

2401

© 2011 PasTest Ltd | About Us | Contact Us | Help

file:///E|/Shakhawan/Endocrinology/141a.htm[3/13/2012 7:08:09 PM]


MyPasTest

Main Navigation
Question Browser: MRCP 1
Home
Subscriptions Question Browser Exam Builder Saved Exams
» MRCP 1 Session Progress
• Question Browser
Questions Correct 7
• Timed Test A 17-year-old student presents with intermittent weakness
and feelings of tiredness. Her GP requested some blood Questions Incorrect 2
• Mock Exam
tests and found her to have a potassium level of 2.8 Questions Total 9
• Past Papers mmol/l and a bicarbonate of 32 mmol/l. She is
Questions Percentage 77 %
• Random Questions normotensive. You arrange a renin and aldosterone level,
and both are elevated. Urinary calcium excretion is More
• My Performance
elevated. Urinary diuretic screen is negative.
• Media Bank
Which diagnosis fits best with this clinical picture?
• New Multimedia
Online Extras Diuretic abuse Reference: Normal Values
Library Gitelman’s syndrome
Click to
Community Liddle’s syndrome open/closeHaematology
Help Bartter’s syndrome Click to
open/closeImmunoglobulins
PasTest Store Conn’s syndrome Click to
open/closeBiochemistry
2402 Click to open/closeDiabetes
My Account
Click to
Profile open/closeEndocrinology
Click to open/closeBlood
Newsletters gases Click to
My Career and Exams open/closeCSF
Order History
Learning Goals
Question Filters

Security

Change Password
Sign Out

© 2011 PasTest Ltd | About Us | Contact Us | Help

file:///E|/Shakhawan/Endocrinology/142.htm[3/13/2012 7:08:11 PM]


MyPasTest

Main Navigation
Question Browser: MRCP 1
Home
Subscriptions Question Browser Exam Builder Saved Exams
» MRCP 1 Session Progress
• Question Browser
Questions Correct 8
• Timed Test A 17-year-old student presents with intermittent weakness
and feelings of tiredness. Her GP requested some blood Questions Incorrect 2
• Mock Exam
tests and found her to have a potassium level of 2.8 Questions Total 10
• Past Papers mmol/l and a bicarbonate of 32 mmol/l. She is
Questions Percentage 80 %
• Random Questions normotensive. You arrange a renin and aldosterone level,
and both are elevated. Urinary calcium excretion is More
• My Performance
elevated. Urinary diuretic screen is negative.
• Media Bank
Which diagnosis fits best with this clinical picture?
• New Multimedia
Online Extras Diuretic abuse
Library Gitelman’s syndrome
Community Liddle’s syndrome
Help Bartter’s syndrome Your answer
PasTest Store Conn’s syndrome

My Account

Profile
This syndrome is characterised by increased urinary
Newsletters calcium excretion, hypokalaemia, metabolic alkalosis
My Career and Exams and raised renin and angiotensin levels. Age at onset
may vary depending on the severity of the causative Reference: Normal Values
Order History
mutation, whilst most patients present in childhood,
Learning Goals some do present later. This is not Conn’s syndrome Click to open/closeHaematology
Question Filters because the patient is normotensive and both her renin Click to open/closeImmunoglobulins
and angiotensin levels are raised. Bartter’s syndrome is Click to open/closeBiochemistry
due to a mutation in the Na + –K + –2Cl – co-transporter, Click to open/closeDiabetes Click to
Security the ATP-regulated potassium channel, or the kidney- open/closeEndocrinology Click to
specific basolateral chloride channel. This impairs open/closeBlood gases Click to
Change Password sodium and chloride reabsorption, and causes an open/closeCSF
Sign Out increase in the production of renal prostaglandin E 2 .
Hyperplasia of the juxtaglomerular apparatus is seen on
renal biopsy. Gitelman’s syndrome is due to a different
mutation, and produces a clinical picture similar to that
seen with chronic thiazide administration. Liddle’s
syndrome is characterised by potassium wasting and
hypertension.

2402

© 2011 PasTest Ltd | About Us | Contact Us | Help

file:///E|/Shakhawan/Endocrinology/142a.htm[3/13/2012 7:08:12 PM]


MyPasTest

Main Navigation
Question Browser: MRCP 1
Home
Subscriptions Question Browser Exam Builder Saved Exams
» MRCP 1 Session Progress
• Question Browser
Questions Correct 8
• Timed Test A 28-year-old man presents with hypertension that his GP
is finding difficult to manage. There are a number of Questions Incorrect 2
• Mock Exam
metabolic abnormalities and he is concerned about the Questions Total 10
• Past Papers possibility of Conn’s syndrome. The blood picture is one of
Questions Percentage 80 %
• Random Questions metabolic acidosis, hyperkalaemia and low renin and
aldosterone levels. What diagnosis fits with this clinical More
• My Performance
picture?
• Media Bank
• New Multimedia Gordon’s syndrome
Online Extras Bartter’s syndrome Reference: Normal Values
Library Addison’s disease
Conn’s syndrome Click to
Community open/closeHaematology
Help Gitelman’s syndrome Click to
open/closeImmunoglobulins
PasTest Store 2403 Click to
open/closeBiochemistry
My Account Click to open/closeDiabetes
Click to
Profile open/closeEndocrinology
Click to open/closeBlood
Newsletters gases Click to
My Career and Exams open/closeCSF
Order History
Learning Goals
Question Filters

Security

Change Password
Sign Out

© 2011 PasTest Ltd | About Us | Contact Us | Help

file:///E|/Shakhawan/Endocrinology/143.htm[3/13/2012 7:08:13 PM]


MyPasTest

Main Navigation
Question Browser: MRCP 1
Home
Subscriptions Question Browser Exam Builder Saved Exams
» MRCP 1 Session Progress
• Question Browser
Questions Correct 9
• Timed Test A 28-year-old man presents with hypertension that his GP
is finding difficult to manage. There are a number of Questions Incorrect 2
• Mock Exam
metabolic abnormalities and he is concerned about the Questions Total 11
• Past Papers possibility of Conn’s syndrome. The blood picture is one of
Questions Percentage 81 %
• Random Questions metabolic acidosis, hyperkalaemia and low renin and
aldosterone levels. What diagnosis fits with this clinical More
• My Performance
picture?
• Media Bank
• New Multimedia Gordon’s syndrome Your answer
Online Extras Bartter’s syndrome
Library Addison’s disease
Community Conn’s syndrome
Help Gitelman’s syndrome
PasTest Store

My Account
Gordon’s syndrome presents as almost the mirror image
Profile of the metabolic abnormalities seen in Bartter’s
syndrome, and is caused primarily by renal sodium
Newsletters
retention and volume expansion. Addison’s disease is,
My Career and Exams of course, associated with hypotension and
Order History hyperkalaemia. Conn’s syndrome is primary Reference: Normal Values
hyperaldosteronism and is associated with
Learning Goals hypokalaemia. In general, the emergency treatment of Click to open/closeHaematology
Question Filters hyperkalaemia is well documented, with acute Click to open/closeImmunoglobulins
administration of calcium to reduce the risk of Click to open/closeBiochemistry
arrhythmias, insulin to drive potassium into cells and Click to open/closeDiabetes Click to
Security the use of ion-exchange resins to increase the excretion open/closeEndocrinology Click to
of potassium via the gastrointestinal tract. open/closeBlood gases Click to
Change Password open/closeCSF
Sign Out
2403

© 2011 PasTest Ltd | About Us | Contact Us | Help

file:///E|/Shakhawan/Endocrinology/143a.htm[3/13/2012 7:08:15 PM]


MyPasTest

Main Navigation
Question Browser: MRCP 1
Home
Subscriptions Question Browser Exam Builder Saved Exams
» MRCP 1 Session Progress
• Question Browser
Questions Correct 9
• Timed Test A 35-year-old woman, with a strong family history of
breast cancer, visits you because she is keen to start on Questions Incorrect 2
• Mock Exam
tamoxifen for breast cancer prophylaxis. Which of the Questions Total 11
• Past Papers following statements best describes the mode of
Questions Percentage 81 %
• Random Questions action of tamoxifen?
• My Performance More
It is a progesterone-receptor agonist
• Media Bank
It is a progesterone-receptor antagonist
• New Multimedia
It is an oestrogen-receptor agonist
Online Extras Reference: Normal Values
It is an oestrogen-receptor antagonist
Library
It is a mixed oestrogen-receptor antagonist and Click to
Community partial agonist open/closeHaematology
Help Click to
2404 open/closeImmunoglobulins
PasTest Store
Click to
open/closeBiochemistry
My Account Click to open/closeDiabetes
Click to
Profile open/closeEndocrinology
Click to open/closeBlood
Newsletters gases Click to
My Career and Exams open/closeCSF
Order History
Learning Goals
Question Filters

Security

Change Password
Sign Out

© 2011 PasTest Ltd | About Us | Contact Us | Help

file:///E|/Shakhawan/Endocrinology/144.htm[3/13/2012 7:08:16 PM]


MyPasTest

Main Navigation
Question Browser: MRCP 1
Home
Subscriptions Question Browser Exam Builder Saved Exams
» MRCP 1 Session Progress
• Question Browser
Questions Correct 10
• Timed Test A 35-year-old woman, with a strong family history of
breast cancer, visits you because she is keen to start on Questions Incorrect 2
• Mock Exam
tamoxifen for breast cancer prophylaxis. Which of the Questions Total 12
• Past Papers following statements best describes the mode of
Questions Percentage 83 %
• Random Questions action of tamoxifen?
• My Performance More
It is a progesterone-receptor agonist
• Media Bank
It is a progesterone-receptor
• New Multimedia antagonist
Online Extras It is an oestrogen-receptor agonist
Library It is an oestrogen-receptor antagonist
Community It is a mixed oestrogen-receptor Your answer
Help antagonist and partial agonist
PasTest Store

My Account The use of tamoxifen after breast cancer is associated


with a 25% risk reduction in metastatic disease, and
Profile is also associated with a reduction in the risk of
Newsletters primary breast cancer in high-risk women. It is a
mixed antagonist and partial agonist at the oestrogen
My Career and Exams
receptor. Its partial agonist action is probably
Order History reflected in the increased risk of endometrial Reference: Normal Values
Learning Goals carcinoma associated with chronic tamoxifen use.
Tamoxifen has been considered the treatment of Click to open/closeHaematology
Question Filters choice, although positive evidence for other hormone Click to open/closeImmunoglobulins
modulators is accumulating. Click to open/closeBiochemistry
Click to open/closeDiabetes Click to
Security Additional modern selective oestrogen-receptor open/closeEndocrinology Click to
modifiers now exist, one example being raloxifene open/closeBlood gases Click to
Change Password which is currently used for the treatment of open/closeCSF
Sign Out osteoporosis.

2404

© 2011 PasTest Ltd | About Us | Contact Us | Help

file:///E|/Shakhawan/Endocrinology/144a.htm[3/13/2012 7:08:18 PM]


MyPasTest

Main Navigation
Question Browser: MRCP 1
Home
Subscriptions Question Browser Exam Builder Saved Exams
» MRCP 1 Session Progress
• Question Browser
Questions Correct 0
• Timed Test You are asked to urgently review a 58-year-old woman
who presents with a slowly enlarging hard mass in the Questions Incorrect 0
• Mock Exam
anterior neck. Thyroid ultrasound reveals infiltration, and Questions Total 0
• Past Papers biopsy does reveal dense infiltration of the gland. Free T 4 is
Questions Percentage 0%
• Random Questions low and her TSH is markedly raised, consistent with
• My Performance hypothyroidism. Thyroid autoantibodies are negative. More
Which of the following is the most likely diagnosis
• Media Bank given this clinical picture?
• New Multimedia
Riedel’s thyroiditis
Online Extras Reference: Normal Values
Thyroid carcinoma
Library
Hashimoto’s thyroiditis Click to
Community open/closeHaematology
Grave’s disease Click to
Help
Toxic multinodular goitre open/closeImmunoglobulins
PasTest Store
Click to
2405 open/closeBiochemistry
My Account Click to open/closeDiabetes
Click to
Profile open/closeEndocrinology
Click to open/closeBlood
Newsletters gases Click to
My Career and Exams open/closeCSF
Order History
Learning Goals
Question Filters

Security

Change Password
Sign Out

© 2011 PasTest Ltd | About Us | Contact Us | Help

file:///E|/Shakhawan/Endocrinology/145.htm[3/13/2012 7:08:19 PM]


MyPasTest

Main Navigation
Question Browser: MRCP 1
Home
Subscriptions Question Browser Exam Builder Saved Exams
» MRCP 1 Session Progress
• Question Browser
Questions Correct 1
• Timed Test You are asked to urgently review a 58-year-old woman
who presents with a slowly enlarging hard mass in the Questions Incorrect 0
• Mock Exam
anterior neck. Thyroid ultrasound reveals infiltration, and Questions Total 1
• Past Papers biopsy does reveal dense infiltration of the gland. Free T 4 is
Questions Percentage 100 %
• Random Questions low and her TSH is markedly raised, consistent with
• My Performance hypothyroidism. Thyroid autoantibodies are negative. More
Which of the following is the most likely diagnosis
• Media Bank given this clinical picture?
• New Multimedia
Riedel’s thyroiditis Your answer
Online Extras
Thyroid carcinoma
Library
Hashimoto’s thyroiditis
Community
Grave’s disease
Help
Toxic multinodular goitre
PasTest Store

My Account
By the nature of its presentation, Riedel’s thyroiditis is
Profile often confused with thyroid carcinoma. It is
Newsletters characterised by marked fibrous infiltration of the
thyroid gland, the aetiology of this being as yet
My Career and Exams
unidentified. Treatment is with thyroxine replacement.
Order History Hashimoto’s thyroiditis is autoimmune in aetiology and Reference: Normal Values
Learning Goals is characterised by lymphocytic infiltration and the
presence of antimicrosomal antibodies. Grave’s disease Click to open/closeHaematology
Question Filters is associated with thyroid-stimulating autoantibodies Click to open/closeImmunoglobulins
and hyperthyroidism.Thyroid ultrasound reveals Click to open/closeBiochemistry
infiltration but no discrete mass is identified, and biopsy Click to open/closeDiabetes Click to
Security does reveal dense infiltration of the gland. open/closeEndocrinology Click to
open/closeBlood gases Click to
Change Password open/closeCSF
2405
Sign Out

© 2011 PasTest Ltd | About Us | Contact Us | Help

file:///E|/Shakhawan/Endocrinology/145a.htm[3/13/2012 7:08:21 PM]


MyPasTest

Main Navigation
Question Browser: MRCP 1
Home
Subscriptions Question Browser Exam Builder Saved Exams
» MRCP 1 Session Progress
• Question Browser
Questions Correct 1
• Timed Test You are referred a 15-year-old girl from a family of
travellers who has never previously attended medical care. Questions Incorrect 0
• Mock Exam
Her mother is concerned that she has short stature and is Questions Total 1
• Past Papers still to commence her periods. On examination she is
Questions Percentage 100 %
• Random Questions clearly below the 50th centile for height. She appears to
have excess skin around her neck, and has poor More
• My Performance
development of secondary sexual characteristics. There is a
• Media Bank murmur suggestive of aortic stenosis. Chromosome analysis
• New Multimedia reveals a 45 X:0 picture. What diagnosis fits best with
this clinical picture?
Online Extras Reference: Normal Values
Library Noonan’s syndrome
Click to
Community Turner’s syndrome
open/closeHaematology
Help Testicular feminisation Click to
Congenital adrenal hyperplasia open/closeImmunoglobulins
PasTest Store
Click to
Congenital lymphoedema open/closeBiochemistry
My Account Click to open/closeDiabetes
2406 Click to
Profile open/closeEndocrinology
Click to open/closeBlood
Newsletters gases Click to
My Career and Exams open/closeCSF
Order History
Learning Goals
Question Filters

Security

Change Password
Sign Out

© 2011 PasTest Ltd | About Us | Contact Us | Help

file:///E|/Shakhawan/Endocrinology/146.htm[3/13/2012 7:08:22 PM]


MyPasTest

Main Navigation
Question Browser: MRCP 1
Home
Subscriptions Question Browser Exam Builder Saved Exams
» MRCP 1 Session Progress
• Question Browser
Questions Correct 2
• Timed Test You are referred a 15-year-old girl from a family of
travellers who has never previously attended medical care. Questions Incorrect 0
• Mock Exam
Her mother is concerned that she has short stature and is Questions Total 2
• Past Papers still to commence her periods. On examination she is
Questions Percentage 100 %
• Random Questions clearly below the 50th centile for height. She appears to
have excess skin around her neck, and has poor More
• My Performance
development of secondary sexual characteristics. There is a
• Media Bank murmur suggestive of aortic stenosis. Chromosome analysis
• New Multimedia reveals a 45 X:0 picture. What diagnosis fits best with
this clinical picture?
Online Extras
Library Noonan’s syndrome
Community Turner’s syndrome Your answer
Help Testicular feminisation
PasTest Store Congenital adrenal hyperplasia
Congenital lymphoedema
My Account

Profile
Newsletters Turner’s syndrome occurs in between 1 in 2500 and 1
in 5000 live births. Rarely, girls with Turner’s
My Career and Exams
syndrome may still present late if they have limited
Order History contact with medical services, as in this case. In older Reference: Normal Values
Learning Goals children, symptoms include slow linear growth, short
stature, delayed development of secondary sexual Click to open/closeHaematology
Question Filters characteristics and absence of menses. There may Click to open/closeImmunoglobulins
also be dyspraxia, poor spatial awareness or mild Click to open/closeBiochemistry
intellectual impairment. Associated cardiovascular Click to open/closeDiabetes Click to
Security defects may include aortic stenosis, bicuspid aortic open/closeEndocrinology Click to
valve or coarctation of the aorta. Patients with open/closeBlood gases Click to
Change Password open/closeCSF
Turner’s syndrome also have an increased risk of
Sign Out hypothyroidism.
Noonan’s syndrome presents with a similar phenotypic
picture to Turner’s syndrome but without the absent X
chromosome.

2406

© 2011 PasTest Ltd | About Us | Contact Us | Help

file:///E|/Shakhawan/Endocrinology/146a.htm[3/13/2012 7:08:24 PM]


MyPasTest

Main Navigation
Question Browser: MRCP 1
Home
Subscriptions Question Browser Exam Builder Saved Exams
» MRCP 1 Session Progress
• Question Browser
Questions Correct 2
• Timed Test A 74-year-old man who is maintained on metformin for
type-2 diabetes presents to the emergency department Questions Incorrect 0
• Mock Exam
acutely unwell. He is shocked, drowsy and confused. Blood Questions Total 2
• Past Papers testing reveals a metabolic acidosis with an anion gap of 24
Questions Percentage 100 %
• Random Questions mmol/l. Ketones are not significantly elevated and random
blood glucose was 8.7 mmol/l. More
• My Performance
• Media Bank What is the mainstay of treatment for this condition?
• New Multimedia
Intravenous insulin therapy
Online Extras Reference: Normal Values
8.2% sodium bicarbonate
Library
1.26% sodium bicarbonate iv Click to
Community open/closeHaematology
4.1% sodium bicarbonate iv and rehydration
Help Click to
Rehydration open/closeImmunoglobulins
PasTest Store
Click to
2407 open/closeBiochemistry
My Account Click to open/closeDiabetes
Click to
Profile open/closeEndocrinology
Click to open/closeBlood
Newsletters gases Click to
My Career and Exams open/closeCSF
Order History
Learning Goals
Question Filters

Security

Change Password
Sign Out

© 2011 PasTest Ltd | About Us | Contact Us | Help

file:///E|/Shakhawan/Endocrinology/147.htm[3/13/2012 7:08:25 PM]


MyPasTest

Main Navigation
Question Browser: MRCP 1
Home
Subscriptions Question Browser Exam Builder Saved Exams
» MRCP 1 Session Progress
• Question Browser
Questions Correct 2
• Timed Test A 74-year-old man who is maintained on metformin for
type-2 diabetes presents to the emergency department Questions Incorrect 1
• Mock Exam
acutely unwell. He is shocked, drowsy and confused. Blood Questions Total 3
• Past Papers testing reveals a metabolic acidosis with an anion gap of 24
Questions Percentage 66 %
• Random Questions mmol/l. Ketones are not significantly elevated and random
blood glucose was 8.7 mmol/l. More
• My Performance
• Media Bank What is the mainstay of treatment for this condition?
• New Multimedia
Intravenous insulin therapy
Online Extras
8.2% sodium bicarbonate
Library
1.26% sodium bicarbonate iv
Community
4.1% sodium bicarbonate iv and Your answer
Help rehydration
PasTest Store Rehydration Correct answer

My Account

Profile The patient has lactic acidosis and requires close


monitoring and should be admitted to an HDU or ITU
Newsletters
ward. The lactate is the cause of the raised anion gap.
My Career and Exams The mainstay of treatment is rehydration. Previous
Order History commentaries recommended sodium bicarbonate, Reference: Normal Values
although no randomised clinical trials exist to support its
Learning Goals use and it may be deleterious. Despite modern fluid Click to open/closeHaematology
Question Filters management and aggressive treatment of these Click to open/closeImmunoglobulins
patients, mortality still approaches 50%. It is important Click to open/closeBiochemistry
to monitor creatinine in patients on metformin as a Click to open/closeDiabetes Click to
Security raised serum creatinine level significantly increases the open/closeEndocrinology Click to
risk of lactic acidosis. Many physicians reduce or open/closeBlood gases Click to
Change Password discontinue metformin when the serum creatinine level open/closeCSF
Sign Out is above 130 µmol/l in women or 150 mmol/lin men,
although the serum creatinine level should always be
considered in the context of overall muscle mass.

2407

© 2011 PasTest Ltd | About Us | Contact Us | Help

file:///E|/Shakhawan/Endocrinology/147a.htm[3/13/2012 7:08:26 PM]


MyPasTest

Main Navigation
Question Browser: MRCP 1
Home
Subscriptions Question Browser Exam Builder Saved Exams
» MRCP 1 Session Progress
• Question Browser
Questions Correct 2
• Timed Test A 54-year-old type-2 diabetic man presents for review.
Which of the following laboratory test results would Questions Incorrect 1
• Mock Exam
be most significantly associated with an increased Questions Total 3
• Past Papers incidence of cardiovascular disease in his case?
Questions Percentage 66 %
• Random Questions
• My Performance Raised proinsulin levels More
• Media Bank Decreased proinsulin levels

• New Multimedia Normal or decreased LDL cholesterol

Online Extras Increased HDL cholesterol levels


Reference: Normal Values
Library Increased triglyceride level to 10% above the normal
range Click to
Community open/closeHaematology
Help 2408 Click to
open/closeImmunoglobulins
PasTest Store
Click to
open/closeBiochemistry
My Account Click to open/closeDiabetes
Click to
Profile open/closeEndocrinology
Click to open/closeBlood
Newsletters gases Click to
My Career and Exams open/closeCSF
Order History
Learning Goals
Question Filters

Security

Change Password
Sign Out

© 2011 PasTest Ltd | About Us | Contact Us | Help

file:///E|/Shakhawan/Endocrinology/148.htm[3/13/2012 7:08:28 PM]


MyPasTest

Main Navigation
Question Browser: MRCP 1
Home
Subscriptions Question Browser Exam Builder Saved Exams
» MRCP 1 Session Progress
• Question Browser
Questions Correct 3
• Timed Test A 54-year-old type-2 diabetic man presents for review.
Which of the following laboratory test results would Questions Incorrect 1
• Mock Exam
be most significantly associated with an increased Questions Total 4
• Past Papers incidence of cardiovascular disease in his case?
Questions Percentage 75 %
• Random Questions
• My Performance Raised proinsulin levels Your answer More
• Media Bank Decreased proinsulin levels

• New Multimedia Normal or decreased LDL cholesterol

Online Extras Increased HDL cholesterol levels

Library Increased triglyceride level to 10%


above the normal range
Community
Help
PasTest Store
Raised levels of proinsulin were shown in the
Caerphilly Cohort Study to be independently
My Account associated with an increased incidence of
cardiovascular disease.
Profile Intervention trials with proinsulin were also
Newsletters discontinued after a possible CV event signal was
seen. The reason behind this association has not yet
My Career and Exams
however been clearly elucidated.
Order History Reference: Normal Values
Raised LDL levels are an independent cardiovascular
Learning Goals risk factor and treatment with statins has been Click to open/closeHaematology
Question Filters proven to reduce cardiovascular risk. Click to open/closeImmunoglobulins
Increased HDL cholesterol levels are associated with Click to open/closeBiochemistry
reverse cholesterol transport and increased Click to open/closeDiabetes Click to
Security processing of cholesterol by the liver, and are open/closeEndocrinology Click to
therefore protective. open/closeBlood gases Click to
Change Password open/closeCSF
Sign Out Increased triglyceride levels are associated with
increased cardiovascular risk, but their importance as
a cardiovascular risk factor has yet to be proven
categorically via an intervention study.
Insulin resistance, a root cause of type-2 diabetes, is
also independently associated with an increased
cardiovascular risk. Whether it is a risk marker or a
risk factor in its own right is open to debate. Results
of glitazone intervention to reduce insulin resistance in
Type 2 diabetes have been mixed at best, with the
Proactive study demonstrating perhaps a small benefit
in reducing events, and the RECORD study
demonstrating at best a neutral effect.

2408

© 2011 PasTest Ltd | About Us | Contact Us | Help

file:///E|/Shakhawan/Endocrinology/148a.htm[3/13/2012 7:08:29 PM]


MyPasTest

Main Navigation
Question Browser: MRCP 1
Home
Subscriptions Question Browser Exam Builder Saved Exams
» MRCP 1 Session Progress
• Question Browser
Questions Correct 3
• Timed Test You are reviewing a 52-year-old woman who has a history
of type-1 diabetes, autoimmune thyroid disease and coeliac Questions Incorrect 1
• Mock Exam
disease. Her daughter has been researching autoimmune Questions Total 4
• Past Papers disease on the Internet and has been learning about the
Questions Percentage 75 %
• Random Questions association between certain HLA types and disease. Which
of the following HLA subtypes is most strongly More
• My Performance
associated with autoimmune thyroid disease or type-
• Media Bank 1 diabetes?
• New Multimedia
HLA-B47
Online Extras Reference: Normal Values
HLA-B27
Library
HLA-DR3 Click to
Community open/closeHaematology
HLA-A28
Help Click to
HLA-DR7 open/closeImmunoglobulins
PasTest Store
Click to
2409 open/closeBiochemistry
My Account Click to open/closeDiabetes
Click to
Profile open/closeEndocrinology
Click to open/closeBlood
Newsletters gases Click to
My Career and Exams open/closeCSF
Order History
Learning Goals
Question Filters

Security

Change Password
Sign Out

© 2011 PasTest Ltd | About Us | Contact Us | Help

file:///E|/Shakhawan/Endocrinology/149.htm[3/13/2012 7:08:31 PM]


MyPasTest

Main Navigation
Question Browser: MRCP 1
Home
Subscriptions Question Browser Exam Builder Saved Exams
» MRCP 1 Session Progress
• Question Browser
Questions Correct 4
• Timed Test You are reviewing a 52-year-old woman who has a history
of type-1 diabetes, autoimmune thyroid disease and coeliac Questions Incorrect 1
• Mock Exam
disease. Her daughter has been researching autoimmune Questions Total 5
• Past Papers disease on the Internet and has been learning about the
Questions Percentage 80 %
• Random Questions association between certain HLA types and disease. Which
of the following HLA subtypes is most strongly More
• My Performance
associated with autoimmune thyroid disease or type-
• Media Bank 1 diabetes?
• New Multimedia
HLA-B47
Online Extras
HLA-B27
Library
HLA-DR3 Your answer
Community
HLA-A28
Help
HLA-DR7
PasTest Store

My Account
HLA-DR3 is associated with diabetes mellitus,
Profile autoimmune hepatitis, dermatitis herpetiformis, Grave’s
Newsletters disease, membranous glomerulonephritis, myasthenia
gravis, Addison’s disease, Sjögren’s syndrome and
My Career and Exams systemic lupus erythematosus (SLE). HLA-B47 is
Order History associated with congenital adrenal hyperplasia, -B27 Reference: Normal Values
with seronegative arthropathies, -A28 with
Learning Goals schizophrenia and -DR7 with minimal-change disease Click to open/closeHaematology
Question Filters (nephrotic syndrome). Click to open/closeImmunoglobulins
Click to open/closeBiochemistry
2409 Click to open/closeDiabetes Click to
Security open/closeEndocrinology Click to
open/closeBlood gases Click to
Change Password open/closeCSF
Sign Out

© 2011 PasTest Ltd | About Us | Contact Us | Help

file:///E|/Shakhawan/Endocrinology/149a.htm[3/13/2012 7:08:32 PM]


MyPasTest

Main Navigation
Question Browser: MRCP 1
Home
Subscriptions Question Browser Exam Builder Saved Exams
» MRCP 1 Session Progress
• Question Browser
Questions Correct 4
• Timed Test An obese 48-year-old woman visits the endocrine clinic.
Screening for endocrine disease, including diabetes mellitus, Questions Incorrect 1
• Mock Exam
hypothyroidism and Cushing’s disease, is negative. She Questions Total 5
• Past Papers asks for dietary advice, particularly about her fat intake.
Questions Percentage 80 %
• Random Questions Which of the following is the best advice to give her
concerning her fat intake? More
• My Performance
• Media Bank Total fat intake should be restricted to less than 40%
• New Multimedia of total dietary energy
Online Extras Saturated fats should provide no more than 5% of Reference: Normal Values
dietary energy
Library
Monounsaturated fats should provide around 6% of Click to
Community dietary energy open/closeHaematology
Help Polyunsaturated fats should provide around 3% of Click to
dietary energy open/closeImmunoglobulins
PasTest Store
Click to
Total fat intake should be restricted to less than 30% open/closeBiochemistry
of total dietary energy Click to open/closeDiabetes
My Account
Click to
Profile 2410 open/closeEndocrinology
Click to open/closeBlood
Newsletters gases Click to
My Career and Exams open/closeCSF
Order History
Learning Goals
Question Filters

Security

Change Password
Sign Out

© 2011 PasTest Ltd | About Us | Contact Us | Help

file:///E|/Shakhawan/Endocrinology/150.htm[3/13/2012 7:08:34 PM]


MyPasTest

Main Navigation
Question Browser: MRCP 1
Home
Subscriptions Question Browser Exam Builder Saved Exams
» MRCP 1 Session Progress
• Question Browser
Questions Correct 4
• Timed Test An obese 48-year-old woman visits the endocrine clinic.
Screening for endocrine disease, including diabetes mellitus, Questions Incorrect 2
• Mock Exam
hypothyroidism and Cushing’s disease, is negative. She Questions Total 6
• Past Papers asks for dietary advice, particularly about her fat intake.
Questions Percentage 66 %
• Random Questions Which of the following is the best advice to give her
concerning her fat intake? More
• My Performance
• Media Bank Total fat intake should be restricted Your answer
• New Multimedia to less than 40% of total dietary
energy
Online Extras
Saturated fats should provide no
Library more than 5% of dietary energy
Community Monounsaturated fats should
Help provide around 6% of dietary energy
PasTest Store Polyunsaturated fats should provide
around 3% of dietary energy
Total fat intake should be restricted Correct answer
My Account to less than 30% of total dietary
Profile energy

Newsletters
My Career and Exams
Order History The current recommendations in the UK for fat intake Reference: Normal Values
are that total fat intake should be restricted to less than
Learning Goals 30% of dietary energy: that monounsaturated fats Click to open/closeHaematology
Question Filters should provide around 12%, polyunsaturated fats Click to open/closeImmunoglobulins
around 6% and that saturated fats should provide no Click to open/closeBiochemistry
more than 10% of dietary energy. Increased dietary fat Click to open/closeDiabetes Click to
Security intake has an epidemiological association with open/closeEndocrinology Click to
cardiovascular disease, cancers, obesity and type-2 open/closeBlood gases Click to
Change Password diabetes. Essential fatty acid deficiency may open/closeCSF
Sign Out occasionally occur in patients on long-term parenteral
nutrition who are given a mix of protein and glucose.
Alopecia, thrombocytopenia, anaemia and dermatitis
may occur.

2410

© 2011 PasTest Ltd | About Us | Contact Us | Help

file:///E|/Shakhawan/Endocrinology/150a.htm[3/13/2012 7:08:35 PM]


MyPasTest

Main Navigation
Question Browser: MRCP 1
Home
Subscriptions Question Browser Exam Builder Saved Exams
» MRCP 1 Session Progress
• Question Browser
Questions Correct 4
• Timed Test You are asked by the respiratory physicians to see a 74-
year-old man who has been admitted from A&E with an Questions Incorrect 2
• Mock Exam
abnormal chest X-ray revealing a right hilar mass. There is Questions Total 6
• Past Papers a history of 40 years of cigarette smoking. He is noted to
Questions Percentage 66 %
• Random Questions have a markedly raised corrected calcium level of 3.25
mmol/l. Apart from a mildly raised urea level due to More
• My Performance
dehydration, his other renal function testing is normal. He
• Media Bank also has a normochromic normocytic anaemia with a
• New Multimedia haemoglobin of 10.0 mg/dl.

Online Extras What is likely to be the underlying cause of his Reference: Normal Values
Library hypercalcaemia?
Click to
Community Primary hyperparathyroidism open/closeHaematology
Help Secondary hyperparathyroidism Click to
open/closeImmunoglobulins
PasTest Store Tertiary hyperparathyroidism Click to
Pseudo-hyperparathyroidism open/closeBiochemistry
My Account Click to open/closeDiabetes
Raised parathyroid hormone-related protein Click to
Profile open/closeEndocrinology
2412 Click to open/closeBlood
Newsletters gases Click to
My Career and Exams open/closeCSF
Order History
Learning Goals
Question Filters

Security

Change Password
Sign Out

© 2011 PasTest Ltd | About Us | Contact Us | Help

file:///E|/Shakhawan/Endocrinology/151.htm[3/13/2012 7:08:37 PM]


MyPasTest

Main Navigation
Question Browser: MRCP 1
Home
Subscriptions Question Browser Exam Builder Saved Exams
» MRCP 1 Session Progress
• Question Browser
Questions Correct 5
• Timed Test You are asked by the respiratory physicians to see a 74-
year-old man who has been admitted from A&E with an Questions Incorrect 2
• Mock Exam
abnormal chest X-ray revealing a right hilar mass. There is Questions Total 7
• Past Papers a history of 40 years of cigarette smoking. He is noted to
Questions Percentage 71 %
• Random Questions have a markedly raised corrected calcium level of 3.25
mmol/l. Apart from a mildly raised urea level due to More
• My Performance
dehydration, his other renal function testing is normal. He
• Media Bank also has a normochromic normocytic anaemia with a
• New Multimedia haemoglobin of 10.0 mg/dl.

Online Extras What is likely to be the underlying cause of his


Library hypercalcaemia?

Community Primary hyperparathyroidism


Help Secondary hyperparathyroidism
PasTest Store Tertiary hyperparathyroidism
Pseudo-hyperparathyroidism
My Account Raised parathyroid hormone-related Your answer
protein
Profile
Newsletters
My Career and Exams
Order History Reference: Normal Values
Parathyroid hormone-related protein (PTH-rp) is a 144
Learning Goals amino acid polypeptide, the initial sequence of which Click to open/closeHaematology
Question Filters shows some homology with the biologically active part Click to open/closeImmunoglobulins
of PTH. This is being produced by his bronchial Click to open/closeBiochemistry
carcinoma. Common primary tumours which may be Click to open/closeDiabetes Click to
Security associated with hypercalcaemia include bronchial, open/closeEndocrinology Click to
breast, renal, prostate carcinomas, myeloma and open/closeBlood gases Click to
Change Password lymphoma. Treatment involves rehydration and open/closeCSF
intravenous bisphosphonate therapy. Patients are often
Sign Out
trialled on prednisolone, but this is rarely successful.

2412

© 2011 PasTest Ltd | About Us | Contact Us | Help

file:///E|/Shakhawan/Endocrinology/151a.htm[3/13/2012 7:08:38 PM]


MyPasTest

Main Navigation
Question Browser: MRCP 1
Home
Subscriptions Question Browser Exam Builder Saved Exams
» MRCP 1 Session Progress
• Question Browser
Questions Correct 5
• Timed Test A 19-year-old student nurse was admitted after her third
collapse in recent months. She was noted to have a blood Questions Incorrect 2
• Mock Exam
sugar of 0.9 mmol/l on finger -prick testing and responded Questions Total 7
• Past Papers well to intravenous glucose therapy. Venous blood taken at
Questions Percentage 71 %
• Random Questions the same time as obtaining venous access showed a
markedly raised insulin level, but her C-peptide levels were More
• My Performance
normal. What diagnosis fits best with this clinical
• Media Bank picture?
• New Multimedia
Insulinoma
Online Extras Reference: Normal Values
Glucagonoma
Library
Occult administration of sulphonylureas Click to
Community open/closeHaematology
Self-administration of a short-acting insulin
Help Click to
Type-1 diabetes open/closeImmunoglobulins
PasTest Store
Click to
2413 open/closeBiochemistry
My Account Click to open/closeDiabetes
Click to
Profile open/closeEndocrinology
Click to open/closeBlood
Newsletters gases Click to
My Career and Exams open/closeCSF
Order History
Learning Goals
Question Filters

Security

Change Password
Sign Out

© 2011 PasTest Ltd | About Us | Contact Us | Help

file:///E|/Shakhawan/Endocrinology/152.htm[3/13/2012 7:08:39 PM]


MyPasTest

Main Navigation
Question Browser: MRCP 1
Home
Subscriptions Question Browser Exam Builder Saved Exams
» MRCP 1 Session Progress
• Question Browser
Questions Correct 6
• Timed Test A 19-year-old student nurse was admitted after her third
collapse in recent months. She was noted to have a blood Questions Incorrect 2
• Mock Exam
sugar of 0.9 mmol/l on finger -prick testing and responded Questions Total 8
• Past Papers well to intravenous glucose therapy. Venous blood taken at
Questions Percentage 75 %
• Random Questions the same time as obtaining venous access showed a
markedly raised insulin level, but her C-peptide levels were More
• My Performance
normal. What diagnosis fits best with this clinical
• Media Bank picture?
• New Multimedia
Insulinoma
Online Extras
Glucagonoma
Library
Occult administration of sulphonylureas
Community
Self-administration of a short-acting Your answer
Help insulin
PasTest Store Type-1 diabetes

My Account

Profile This woman has hyperinsulinaemia and hypoglycaemia,


Newsletters but her C-peptide levels are normal. This pattern is
strongly suggestive of the fact that she is self-
My Career and Exams administering insulin. On further questioning in this case
Order History it was determined that her father has type-1 diabetes Reference: Normal Values
and she had been using his Actrapid to induce
Learning Goals hypoglycaemia during times of stress at work. If both Click to open/closeHaematology
Question Filters the insulin and C-peptide levels were raised, then it Click to open/closeImmunoglobulins
would be worth screening her urine for a sulphonylurea Click to open/closeBiochemistry
assay. Only after this had been excluded would it be Click to open/closeDiabetes Click to
Security worth pursuing investigation for insulinoma. open/closeEndocrinology Click to
open/closeBlood gases Click to
Change Password open/closeCSF
2413
Sign Out

© 2011 PasTest Ltd | About Us | Contact Us | Help

file:///E|/Shakhawan/Endocrinology/152a.htm[3/13/2012 7:08:41 PM]


MyPasTest

Main Navigation
Question Browser: MRCP 1
Home
Subscriptions Question Browser Exam Builder Saved Exams
» MRCP 1 Session Progress
• Question Browser
Questions Correct 6
• Timed Test A 35-year-old woman visits you in the paediatric diabetes
clinic with her 2-year-old son who has recently developed Questions Incorrect 2
• Mock Exam
type-1 diabetes. He has an identical twin brother and she is Questions Total 8
• Past Papers concerned about his risk of developing diabetes. What
Questions Percentage 75 %
• Random Questions advice would you give regarding his future risk?
• My Performance More
He has a 100% future risk of developing type-1
• Media Bank diabetes
• New Multimedia A trial of low-dose insulin in the unaffected child will
Online Extras reduce his future risk of diabetes Reference: Normal Values
Library GAD antibodies in the unaffected child are not
predictive of the risk of diabetes Click to
Community open/closeHaematology
He has a 30–50% future risk of developing type-1
Help diabetes Click to
open/closeImmunoglobulins
PasTest Store IA-2 antibodies in the unaffected child are not Click to
predictive of the risk of diabetes open/closeBiochemistry
My Account Click to open/closeDiabetes
2414 Click to
Profile open/closeEndocrinology
Click to open/closeBlood
Newsletters gases Click to
My Career and Exams open/closeCSF
Order History
Learning Goals
Question Filters

Security

Change Password
Sign Out

© 2011 PasTest Ltd | About Us | Contact Us | Help

file:///E|/Shakhawan/Endocrinology/153.htm[3/13/2012 7:08:42 PM]


MyPasTest

Main Navigation
Question Browser: MRCP 1
Home
Subscriptions Question Browser Exam Builder Saved Exams
» MRCP 1 Session Progress
• Question Browser
Questions Correct 7
• Timed Test A 35-year-old woman visits you in the paediatric diabetes
clinic with her 2-year-old son who has recently developed Questions Incorrect 2
• Mock Exam
type-1 diabetes. He has an identical twin brother and she is Questions Total 9
• Past Papers concerned about his risk of developing diabetes. What
Questions Percentage 77 %
• Random Questions advice would you give regarding his future risk?
• My Performance More
He has a 100% future risk of
• Media Bank developing type-1 diabetes
• New Multimedia A trial of low-dose insulin in the
Online Extras unaffected child will reduce his future
risk of diabetes
Library
GAD antibodies in the unaffected child
Community are not predictive of the risk of diabetes
Help He has a 30–50% future risk of Your answer
PasTest Store developing type-1 diabetes
IA-2 antibodies in the unaffected child
are not predictive of the risk of diabetes
My Account

Profile
Newsletters
My Career and Exams Twin studies have suggested that the future risk of
Order History diabetes in the unaffected child is 30–50%. The Reference: Normal Values
presence of glutamic acid decarboxylase (GAD), islet-
Learning Goals cell or IA-2 antibodies in the unaffected child increases Click to open/closeHaematology
Question Filters the likelihood that the child will go on to develop type-1 Click to open/closeImmunoglobulins
diabetes. Children of type-1 diabetic patients have a Click to open/closeBiochemistry
slightly increased risk of developing the disease (3–6% Click to open/closeDiabetes Click to
Security risk where the father is diabetic, 2–3% where the open/closeEndocrinology Click to
mother is diabetic). Twin studies in type-2 diabetes open/closeBlood gases Click to
Change Password have suggested that if one identical twin develops the open/closeCSF
Sign Out disease, then the risk in the unaffected twin rises to
50% or greater. Maturity-onset diabetes of the young
(MODY) is a rare variant of type-2 diabetes with a very
strong familial pattern of inheritance.

2414

© 2011 PasTest Ltd | About Us | Contact Us | Help

file:///E|/Shakhawan/Endocrinology/153a.htm[3/13/2012 7:08:44 PM]


MyPasTest

Main Navigation
Question Browser: MRCP 1
Home
Subscriptions Question Browser Exam Builder Saved Exams
» MRCP 1 Session Progress
• Question Browser
Questions Correct 7
• Timed Test You are reviewing a 45-year-old man who has type-2
diabetes. He works a varying shift pattern as a taxi driver Questions Incorrect 2
• Mock Exam
and has not tolerated metformin therapy due to Questions Total 9
• Past Papers gastrointestinal side-effects. You decide that the
Questions Percentage 77 %
• Random Questions postprandial glucose regulator nateglinide is the most
appropriate therapy choice. Which of the following best More
• My Performance
describes the mode of action of nateglinide?
• Media Bank
• New Multimedia It acts by reducing hepatic glucose output
Online Extras It acts by reducing peripheral insulin resistance Reference: Normal Values
Library It acts by closure of β -cell calcium channels
Click to
Community It acts by closure of the β-cell K–ATP channel open/closeHaematology
Help It prevents the gastrointestinal absorption of sugars Click to
open/closeImmunoglobulins
PasTest Store
2415 Click to
open/closeBiochemistry
My Account Click to open/closeDiabetes
Click to
Profile open/closeEndocrinology
Click to open/closeBlood
Newsletters gases Click to
My Career and Exams open/closeCSF
Order History
Learning Goals
Question Filters

Security

Change Password
Sign Out

© 2011 PasTest Ltd | About Us | Contact Us | Help

file:///E|/Shakhawan/Endocrinology/154.htm[3/13/2012 7:08:45 PM]


MyPasTest

Main Navigation
Question Browser: MRCP 1
Home
Subscriptions Question Browser Exam Builder Saved Exams
» MRCP 1 Session Progress
• Question Browser
Questions Correct 8
• Timed Test You are reviewing a 45-year-old man who has type-2
Questions Incorrect 2
• Mock Exam diabetes. He works a varying shift pattern as a taxi driver
and has not tolerated metformin therapy due to Questions Total 10
• Past Papers gastrointestinal side-effects. You decide that the Questions Percentage 80 %
• Random Questions postprandial glucose regulator nateglinide is the most
appropriate therapy choice. Which of the following best More
• My Performance
describes the mode of action of nateglinide?
• Media Bank
• New Multimedia It acts by reducing hepatic glucose
output
Online Extras
It acts by reducing peripheral insulin
Library resistance
Community
It acts by closure of β -cell calcium
Help channels
PasTest Store It acts by closure of the β-cell K–ATP Your answer
channel
My Account It prevents the gastrointestinal
absorption of sugars
Profile
Newsletters
My Career and Exams
Order History It acts by closure of the β-cell K-ATP channel promoting Reference: Normal Values
Learning Goals release of insulin. It is short acting and taken to
coincide with meals. Nateglinide is more expensive than Click to open/closeHaematology
Question Filters Click to open/closeImmunoglobulins
sulphonylureas but offers advantages for shift workers
and patients who may fast for a period of time as doses Click to open/closeBiochemistry
can be skipped when meals are missed. In these patient Click to open/closeDiabetes Click to
Security open/closeEndocrinology Click to
groups there may be less incidence of hyperglycaemia.
It appears that the shorter action of duration appears to open/closeBlood gases Click to
Change Password open/closeCSF
result in less weight gain compared to traditional
Sign Out sulphonylureas, and in theory these agents should
result in “less strain” on the β-cell.

2415

© 2011 PasTest Ltd | About Us | Contact Us | Help

file:///E|/Shakhawan/Endocrinology/154a.htm[3/13/2012 7:08:46 PM]


MyPasTest

Main Navigation
Question Browser: MRCP 1
Home
Subscriptions Question Browser Exam Builder Saved Exams
» MRCP 1 Session Progress
• Question Browser
Questions Correct 8
• Timed Test A 74-year-old man is admitted to A&E in an acute
confusional state. His serum sodium concentration is 105 Questions Incorrect 2
• Mock Exam
mmol/l and because he has heavily nicotine-stained Questions Total 10
• Past Papers fingers, SIAD is suspected.
Questions Percentage 80 %
• Random Questions
Which of the following biochemical findings best More
• My Performance supports this diagnosis?
• Media Bank
• New Multimedia Random plasma cortisol concentration of 548 nmol/l

Online Extras Random urine osmolality of 380 mmol/kg


Reference: Normal Values
Library Random urine sodium concentration of 72 mmol/l
Serum osmolality of 230 mmol/kg Click to
Community open/closeHaematology
Help Serum urea concentration of 2.1 mmol/l Click to
open/closeImmunoglobulins
PasTest Store 2573 Click to
open/closeBiochemistry
My Account Click to open/closeDiabetes
Click to
Profile open/closeEndocrinology
Click to open/closeBlood
Newsletters gases Click to
My Career and Exams open/closeCSF
Order History
Learning Goals
Question Filters

Security

Change Password
Sign Out

© 2011 PasTest Ltd | About Us | Contact Us | Help

file:///E|/Shakhawan/Endocrinology/155.htm[3/13/2012 7:08:48 PM]


MyPasTest

Main Navigation
Question Browser: MRCP 1
Home
Subscriptions Question Browser Exam Builder Saved Exams
» MRCP 1 Session Progress
• Question Browser
Questions Correct 8
• Timed Test A 74-year-old man is admitted to A&E in an acute
confusional state. His serum sodium concentration is 105 Questions Incorrect 3
• Mock Exam
mmol/l and because he has heavily nicotine-stained Questions Total 11
• Past Papers fingers, SIAD is suspected.
Questions Percentage 72 %
• Random Questions
Which of the following biochemical findings best More
• My Performance supports this diagnosis?
• Media Bank
• New Multimedia Random plasma cortisol
concentration of 548 nmol/l
Online Extras
Random urine osmolality of 380 Correct answer
Library mmol/kg
Community Random urine sodium concentration Your answer
Help of 72 mmol/l
PasTest Store Serum osmolality of 230 mmol/kg
Serum urea concentration of 2.1
mmol/l
My Account

Profile
Newsletters
A diagnosis of SIAD (syndrome of inappropriate
My Career and Exams antidiuresis, also known as the syndrome of
Order History inappropriate secretion of antidiuretic hormone) is Reference: Normal Values
often made without all the criteria being satisfied.
Learning Goals
Clinically, there should be no evidence of fluid Click to open/closeHaematology
Question Filters overload (oedema) or dehydration; normal adrenal Click to open/closeImmunoglobulins
and renal function must be demonstrable; serum Click to open/closeBiochemistry
osmolality must be decreased and, critically, the urine Click to open/closeDiabetes Click to
Security osmolality should be greater than that of the serum open/closeEndocrinology Click to
(establishing that there is impaired water excretion). open/closeBlood gases Click to
Change Password The cortisol concentration in this case, given that the open/closeCSF
Sign Out patient is sick, effectively excludes adrenal failure and
satisfies one criterion. Urea concentration is low: this
is likely to be because of water overload.
Patients with SIAD are usually in sodium balance and
sodium excretion is appropriate for intake. A very low
urine sodium would suggest that hyponatraemia was
due to sodium depletion.

2573

© 2011 PasTest Ltd | About Us | Contact Us | Help

file:///E|/Shakhawan/Endocrinology/155a.htm[3/13/2012 7:08:49 PM]


MyPasTest

Main Navigation
Question Browser: MRCP 1
Home
Subscriptions Question Browser Exam Builder Saved Exams
» MRCP 1 Session Progress
• Question Browser
Questions Correct 8
• Timed Test Following a head injury, a 24-year-old patient develops
polyuria and polydipsia and is suspected of having cranial Questions Incorrect 3
• Mock Exam
diabetes insipidus. He undergoes a water deprivation test. Questions Total 11
• Past Papers Which one of the following responses would most
Questions Percentage 72 %
• Random Questions indicate a positive (abnormal) result?
• My Performance More
A rise in plasma osmolality to 302 mmol/kg during
• Media Bank water deprivation
• New Multimedia Failure to concentrate the urine during water
Online Extras deprivation, but achievement of urine osmolality of Reference: Normal Values
720 mmol/kg following the administration of
Library desmopressin
Click to
Community Failure to concentrate the urine either with water open/closeHaematology
Help deprivation or following the administration of Click to
desmopressin open/closeImmunoglobulins
PasTest Store
Failure to concentrate the urine with a plasma Click to
osmolality of 280 mmol/kg at the end of the period of open/closeBiochemistry
My Account water deprivation Click to open/closeDiabetes
Click to
Profile Weight loss of 5% during the investigation open/closeEndocrinology
Click to open/closeBlood
Newsletters 2574 gases Click to
My Career and Exams open/closeCSF
Order History
Learning Goals
Question Filters

Security

Change Password
Sign Out

© 2011 PasTest Ltd | About Us | Contact Us | Help

file:///E|/Shakhawan/Endocrinology/156.htm[3/13/2012 7:08:51 PM]


MyPasTest

Main Navigation
Question Browser: MRCP 1
Home
Subscriptions Question Browser Exam Builder Saved Exams
» MRCP 1 Session Progress
• Question Browser
Questions Correct 9
• Timed Test Following a head injury, a 24-year-old patient develops
polyuria and polydipsia and is suspected of having cranial Questions Incorrect 3
• Mock Exam
diabetes insipidus. He undergoes a water deprivation test. Questions Total 12
• Past Papers Which one of the following responses would most
Questions Percentage 75 %
• Random Questions indicate a positive (abnormal) result?
• My Performance More
A rise in plasma osmolality to 302
• Media Bank mmol/kg during water deprivation
• New Multimedia Failure to concentrate the urine during Your answer
Online Extras water deprivation, but achievement of
urine osmolality of 720 mmol/kg
Library following the administration of
Community desmopressin
Help Failure to concentrate the urine either
with water deprivation or following the
PasTest Store
administration of desmopressin
Failure to concentrate the urine with a
My Account plasma osmolality of 280 mmol/kg at
the end of the period of water
Profile deprivation
Newsletters Weight loss of 5% during the
My Career and Exams investigation
Order History Reference: Normal Values
Learning Goals
Click to open/closeHaematology
Question Filters Significant weight loss, an increase in plasma osmolality Click to open/closeImmunoglobulins
and a failure of urinary concentration during water Click to open/closeBiochemistry
deprivation are all diagnostic of diabetes insipidus (DI) Click to open/closeDiabetes Click to
Security but do not differentiate between this being cranial or open/closeEndocrinology Click to
nephrogenic in origin. In nephrogenic DI, however, open/closeBlood gases Click to
Change Password there is typically no response to desmopressin, whereas open/closeCSF
Sign Out this causes the urine to become concentrated in cranial
DI. A low plasma osmolality at the end of the
investigation, with a persistently dilute urine, suggests
prior water loading or continued water intake. Note that
cranial DI can be transient following head injury.

2574

© 2011 PasTest Ltd | About Us | Contact Us | Help

file:///E|/Shakhawan/Endocrinology/156a.htm[3/13/2012 7:08:52 PM]


MyPasTest

Main Navigation
Question Browser: MRCP 1
Home
Subscriptions Question Browser Exam Builder Saved Exams
» MRCP 1 Session Progress
• Question Browser
Questions Correct 9
• Timed Test A 49-year-old woman presents complaining of fatigue and
weight loss. She was successfully treated medically for a Questions Incorrect 3
• Mock Exam
prolactinoma 3 years ago, and it is suspected that she Questions Total 12
• Past Papers might now have MEN-1. Which of the following
Questions Percentage 75 %
• Random Questions underlying conditions is she most likely to have?
• My Performance More
Medullar carcinoma of thyroid
• Media Bank
Parathyroid hyperplasia
• New Multimedia
Pancreatic islet-cell tumour
Online Extras Reference: Normal Values
Pituitary adenoma
Library
Phaeochromocytoma Click to
Community open/closeHaematology
Help 2575 Click to
open/closeImmunoglobulins
PasTest Store
Click to
open/closeBiochemistry
My Account Click to open/closeDiabetes
Click to
Profile open/closeEndocrinology
Click to open/closeBlood
Newsletters gases Click to
My Career and Exams open/closeCSF
Order History
Learning Goals
Question Filters

Security

Change Password
Sign Out

© 2011 PasTest Ltd | About Us | Contact Us | Help

file:///E|/Shakhawan/Endocrinology/157.htm[3/13/2012 7:08:54 PM]


MyPasTest

Main Navigation
Question Browser: MRCP 1
Home
Subscriptions Question Browser Exam Builder Saved Exams
» MRCP 1 Session Progress
• Question Browser
Questions Correct 10
• Timed Test A 49-year-old woman presents complaining of fatigue and
weight loss. She was successfully treated medically for a Questions Incorrect 3
• Mock Exam
prolactinoma 3 years ago, and it is suspected that she Questions Total 13
• Past Papers might now have MEN-1. Which of the following
Questions Percentage 76 %
• Random Questions underlying conditions is she most likely to have?
• My Performance More
Medullar carcinoma of thyroid
• Media Bank
Parathyroid hyperplasia Your answer
• New Multimedia
Pancreatic islet-cell tumour
Online Extras
Pituitary adenoma
Library
Phaeochromocytoma
Community
Help
PasTest Store
Parathyroid hyperplasia is seen in the vast majority of
patients with multiple endocrine neoplasia type 1 (MEN-
My Account 1), and the hypercalcaemia that they cause is the most
common presenting feature of the condition.
Profile Hypercalcaemia may be discovered incidentally or
Newsletters because it is symptomatic, but the symptoms are often
non-specific. Islet-cell tumours (most frequently
My Career and Exams gastrinomas) are present in about 40% of cases.
Order History Pituitary tumours occur in about 30% of patients: over Reference: Normal Values
a half are prolactinomas. Medullary-cell carcinomas of
Learning Goals the thyroid and phaeochromocytomas occur in MEN-2 Click to open/closeHaematology
Question Filters but not in MEN-1. Click to open/closeImmunoglobulins
Click to open/closeBiochemistry
2575 Click to open/closeDiabetes Click to
Security open/closeEndocrinology Click to
open/closeBlood gases Click to
Change Password open/closeCSF
Sign Out

© 2011 PasTest Ltd | About Us | Contact Us | Help

file:///E|/Shakhawan/Endocrinology/157a.htm[3/13/2012 7:08:56 PM]


MyPasTest

Main Navigation
Question Browser: MRCP 1
Home
Subscriptions Question Browser Exam Builder Saved Exams
» MRCP 1 Session Progress
• Question Browser
Questions Correct 10
• Timed Test A 51-year-old man presents with weight loss and is found
on examination to have hepatomegaly. He has facial Questions Incorrect 3
• Mock Exam
telangiectasia. Urinary 5-HIAA levels are found to be Questions Total 13
• Past Papers elevated. Given the diagnosis of carcinoid syndrome,
Questions Percentage 76 %
• Random Questions which additional clinical feature is the most likely to
be present? More
• My Performance
• Media Bank Abdominal pain
• New Multimedia Diarrhoea
Online Extras Hypertension Reference: Normal Values
Library Skin rash
Click to
Community Wheezing open/closeHaematology
Help Click to
2576 open/closeImmunoglobulins
PasTest Store
Click to
open/closeBiochemistry
My Account Click to open/closeDiabetes
Click to
Profile open/closeEndocrinology
Click to open/closeBlood
Newsletters gases Click to
My Career and Exams open/closeCSF
Order History
Learning Goals
Question Filters

Security

Change Password
Sign Out

© 2011 PasTest Ltd | About Us | Contact Us | Help

file:///E|/Shakhawan/Endocrinology/158.htm[3/13/2012 7:08:58 PM]


MyPasTest

Main Navigation
Question Browser: MRCP 1
Home
Subscriptions Question Browser Exam Builder Saved Exams
» MRCP 1 Session Progress
• Question Browser
Questions Correct 11
• Timed Test A 51-year-old man presents with weight loss and is found
on examination to have hepatomegaly. He has facial Questions Incorrect 3
• Mock Exam
telangiectasia. Urinary 5-HIAA levels are found to be Questions Total 14
• Past Papers elevated. Given the diagnosis of carcinoid syndrome,
Questions Percentage 78 %
• Random Questions which additional clinical feature is the most likely to
be present? More
• My Performance
• Media Bank Abdominal pain
• New Multimedia Diarrhoea Your answer
Online Extras Hypertension
Library Skin rash
Community Wheezing
Help
PasTest Store

Diarrhoea and flushing, occurring separately or


My Account together, are the most frequent presenting features of
carcinoid syndrome (approximately half of all patients).
Profile Abdominal pain occurs in about 10%. Wheezing is a
Newsletters recognised feature of the condition but is uncommon,
and a skin rash (pellagra) due to niacin deficiency
My Career and Exams (niacin synthesis may be decreased in carcinoid
Order History syndrome) is rare. Hypertension is not a feature of the Reference: Normal Values
condition.
Learning Goals
Click to open/closeHaematology
Question Filters 2576 Click to open/closeImmunoglobulins
Click to open/closeBiochemistry
Click to open/closeDiabetes Click to
Security open/closeEndocrinology Click to
open/closeBlood gases Click to
Change Password open/closeCSF
Sign Out

© 2011 PasTest Ltd | About Us | Contact Us | Help

file:///E|/Shakhawan/Endocrinology/158a.htm[3/13/2012 7:08:59 PM]


MyPasTest

Main Navigation
Question Browser: MRCP 1
Home
Subscriptions Question Browser Exam Builder Saved Exams
» MRCP 1 Session Progress
• Question Browser
Questions Correct 11
• Timed Test A 73-year-old woman presents with weight loss and is
found to have a serum calcium concentration of 3.22 Questions Incorrect 3
• Mock Exam
mmol/l. A skeletal survey is normal. Non-metastatic Questions Total 14
• Past Papers hypercalcaemia is suspected. Endoscopy reveals a gastric
Questions Percentage 78 %
• Random Questions carcinoma.
• My Performance More
Secretion of which of the following substances by the
• Media Bank tumour is most likely to be responsible?
• New Multimedia
Calcitonin
Online Extras Reference: Normal Values
Calcitriol (1,25-dihydroxycholecalciferol)
Library
Osteoclast-activating cytokines Click to
Community open/closeHaematology
Parathyroid hormone
Help Click to
Parathyroid hormone-related peptide open/closeImmunoglobulins
PasTest Store
Click to
2577 open/closeBiochemistry
My Account Click to open/closeDiabetes
Click to
Profile open/closeEndocrinology
Click to open/closeBlood
Newsletters gases Click to
My Career and Exams open/closeCSF
Order History
Learning Goals
Question Filters

Security

Change Password
Sign Out

© 2011 PasTest Ltd | About Us | Contact Us | Help

file:///E|/Shakhawan/Endocrinology/159.htm[3/13/2012 7:09:01 PM]


MyPasTest

Main Navigation
Question Browser: MRCP 1
Home
Subscriptions Question Browser Exam Builder Saved Exams
» MRCP 1 Session Progress
• Question Browser
Questions Correct 12
• Timed Test A 73-year-old woman presents with weight loss and is
found to have a serum calcium concentration of 3.22 Questions Incorrect 3
• Mock Exam
mmol/l. A skeletal survey is normal. Non-metastatic Questions Total 15
• Past Papers hypercalcaemia is suspected. Endoscopy reveals a gastric
Questions Percentage 80 %
• Random Questions carcinoma.
• My Performance More
Secretion of which of the following substances by the
• Media Bank tumour is most likely to be responsible?
• New Multimedia
Calcitonin
Online Extras
Calcitriol (1,25-
Library dihydroxycholecalciferol)
Community Osteoclast-activating cytokines
Help Parathyroid hormone
PasTest Store Parathyroid hormone-related peptide Your answer

My Account

Profile In patients with solid tumours, non-metastatic


hypercalcaemia is most frequently the result of
Newsletters
secretion of parathyroid hormone-related peptide
My Career and Exams (PTHrP) by the tumour, and is reported in association
Order History with gastric cancer. Non-parathyroid secretion of PTH Reference: Normal Values
itself is very rare. Unregulated production of calcitriol
Learning Goals has been described in patients with lymphoma but is Click to open/closeHaematology
Question Filters rare. Secretion of calcitonin, either ectopically or by Click to open/closeImmunoglobulins
medullary-cell carcinoma of the thyroid, is clinically Click to open/closeBiochemistry
silent: it does not perturb calcium homeostasis. Click to open/closeDiabetes Click to
Security Secretion of osteoclast-activating cytokines is an open/closeEndocrinology Click to
important cause of hypercalcaemia in patients with open/closeBlood gases Click to
Change Password myeloma, but this is a relatively uncommon malignancy. open/closeCSF
Sign Out
2577

© 2011 PasTest Ltd | About Us | Contact Us | Help

file:///E|/Shakhawan/Endocrinology/159a.htm[3/13/2012 7:09:02 PM]


MyPasTest

Main Navigation
Question Browser: MRCP 1
Home
Subscriptions Question Browser Exam Builder Saved Exams
» MRCP 1 Session Progress
• Question Browser
Questions Correct 12
• Timed Test A 55-year-old man is found incidentally to have
hypercalcaemia during a routine health screen. Questions Incorrect 3
• Mock Exam
Which one of the following biochemical findings Questions Total 15
• Past Papers would be most suggestive of this being caused by
Questions Percentage 80 %
• Random Questions primary hyperparathyroidism rather than any other
cause of hypercalcaemia? More
• My Performance
• Media Bank Elevated 24-h urinary calcium excretion
• New Multimedia Elevated serum alkaline phosphatase activity
Online Extras Low serum concentration of calcitriol (1,25- Reference: Normal Values
Library dihydroxycholecalciferol)
Normal serum phosphate concentration Click to
Community open/closeHaematology
Help Serum PTH concentration within the normal range Click to
open/closeImmunoglobulins
PasTest Store 2578 Click to
open/closeBiochemistry
My Account Click to open/closeDiabetes
Click to
Profile open/closeEndocrinology
Click to open/closeBlood
Newsletters gases Click to
My Career and Exams open/closeCSF
Order History
Learning Goals
Question Filters

Security

Change Password
Sign Out

© 2011 PasTest Ltd | About Us | Contact Us | Help

file:///E|/Shakhawan/Endocrinology/160.htm[3/13/2012 7:09:03 PM]


MyPasTest

Main Navigation
Question Browser: MRCP 1
Home
Subscriptions Question Browser Exam Builder Saved Exams
» MRCP 1 Session Progress
• Question Browser
Questions Correct 13
• Timed Test A 55-year-old man is found incidentally to have
hypercalcaemia during a routine health screen. Questions Incorrect 3
• Mock Exam
Which one of the following biochemical findings Questions Total 16
• Past Papers would be most suggestive of this being caused by
Questions Percentage 81 %
• Random Questions primary hyperparathyroidism rather than any other
cause of hypercalcaemia? More
• My Performance
• Media Bank Elevated 24-h urinary calcium
• New Multimedia excretion
Online Extras Elevated serum alkaline phosphatase
activity
Library
Low serum concentration of calcitriol
Community (1,25-dihydroxycholecalciferol)
Help Normal serum phosphate concentration
PasTest Store Serum PTH concentration within the Your answer
normal range
My Account

Profile
Newsletters Although parathyroid hormone (PTH) concentrations are
often increased in patients with hyperparathyroidism,
My Career and Exams they are not always so. PTH secretion should be
Order History suppressed by hypercalcaemia from any other cause, so Reference: Normal Values
that a PTH value in the normal range is consistent with
Learning Goals the diagnosis. Urinary calcium excretion is increased in Click to open/closeHaematology
Question Filters many causes of hypercalcaemia (except familial Click to open/closeImmunoglobulins
hypocalciuric hypercalcaemia), including Click to open/closeBiochemistry
hyperparathyroidism. Serum alkaline phosphatase Click to open/closeDiabetes Click to
Security activity can also be elevated with hypercalcaemia, open/closeEndocrinology Click to
regardless of the cause (with the exception of open/closeBlood gases Click to
Change Password myeloma). Serum phosphate concentrations tend to be open/closeCSF
Sign Out reduced in hyperparathyroidism (PTH is phosphaturic)
and calcitriol concentrations to be increased (PTH
stimulates the formation of this hormone).

2578

© 2011 PasTest Ltd | About Us | Contact Us | Help

file:///E|/Shakhawan/Endocrinology/160a.htm[3/13/2012 7:09:05 PM]


MyPasTest

Main Navigation
Question Browser: MRCP 1
Home
Subscriptions Question Browser Exam Builder Saved Exams
» MRCP 1 Session Progress
• Question Browser
Questions Correct 13
• Timed Test A 57-year-old woman is brought to A&E after sustaining a
head injury. A skull X-ray shows there is no fracture, but Questions Incorrect 3
• Mock Exam
that there is erosion of the anterior clinoid processes of the Questions Total 16
• Past Papers pituitary fossa, suggesting a pituitary tumour. Which of
Questions Percentage 81 %
• Random Questions the following hormones is most likely to be being
secreted in excess? More
• My Performance
• Media Bank Adrenocorticotrophin (ACTH)
• New Multimedia Follicle-stimulating hormone (FSH)
Online Extras Growth hormone Reference: Normal Values
Library Prolactin
Click to
Community Thyroid-stimulating hormone (TSH) open/closeHaematology
Help Click to
2579 open/closeImmunoglobulins
PasTest Store
Click to
open/closeBiochemistry
My Account Click to open/closeDiabetes
Click to
Profile open/closeEndocrinology
Click to open/closeBlood
Newsletters gases Click to
My Career and Exams open/closeCSF
Order History
Learning Goals
Question Filters

Security

Change Password
Sign Out

© 2011 PasTest Ltd | About Us | Contact Us | Help

file:///E|/Shakhawan/Endocrinology/161.htm[3/13/2012 7:09:07 PM]


MyPasTest

Main Navigation
Question Browser: MRCP 1
Home
Subscriptions Question Browser Exam Builder Saved Exams
» MRCP 1 Session Progress
• Question Browser
Questions Correct 14
• Timed Test A 57-year-old woman is brought to A&E after sustaining a
head injury. A skull X-ray shows there is no fracture, but Questions Incorrect 3
• Mock Exam
that there is erosion of the anterior clinoid processes of the Questions Total 17
• Past Papers pituitary fossa, suggesting a pituitary tumour. Which of
Questions Percentage 82 %
• Random Questions the following hormones is most likely to be being
secreted in excess? More
• My Performance
• Media Bank Adrenocorticotrophin (ACTH)
• New Multimedia Follicle-stimulating hormone (FSH)
Online Extras Growth hormone
Library Prolactin Your answer
Community Thyroid-stimulating hormone (TSH)
Help
PasTest Store

Prolactinomas are the most frequently occurring


My Account functional pituitary tumours; however, prolactin
secretion may be increased in patients with other
Profile pituitary tumours because of the interruption of the
Newsletters inhibitory effect of dopamine secreted by the
hypothalamus. ACTH-secreting and growth hormone-
My Career and Exams secreting tumours both occur less frequently and TSH-
Order History and FSH-secreting tumours are uncommon. Non- Reference: Normal Values
functional tumours can occur at any age but are more
Learning Goals frequent in the elderly. Click to open/closeHaematology
Question Filters Click to open/closeImmunoglobulins
2579 Click to open/closeBiochemistry
Click to open/closeDiabetes Click to
Security open/closeEndocrinology Click to
open/closeBlood gases Click to
Change Password open/closeCSF
Sign Out

© 2011 PasTest Ltd | About Us | Contact Us | Help

file:///E|/Shakhawan/Endocrinology/161a.htm[3/13/2012 7:09:08 PM]


MyPasTest

Main Navigation
Question Browser: MRCP 1
Home
Subscriptions Question Browser Exam Builder Saved Exams
» MRCP 1 Session Progress
• Question Browser
Questions Correct 14
• Timed Test A 42-year-old patient complains of severe fatigue following
surgery for a pituitary tumour and is put on growth Questions Incorrect 3
• Mock Exam
hormone replacement. Which of the following is a well- Questions Total 17
• Past Papers recognised effect of this treatment?
Questions Percentage 82 %
• Random Questions
• My Performance Decrease in serum lipoprotein(a) concentration More
• Media Bank Increase in fasting serum triglyceride concentration

• New Multimedia Increase in fat mass

Online Extras Increase in lean body mass


Reference: Normal Values
Library Increase in serum total cholesterol concentration
Click to
Community 2580 open/closeHaematology
Help Click to
open/closeImmunoglobulins
PasTest Store
Click to
open/closeBiochemistry
My Account Click to open/closeDiabetes
Click to
Profile open/closeEndocrinology
Click to open/closeBlood
Newsletters gases Click to
My Career and Exams open/closeCSF
Order History
Learning Goals
Question Filters

Security

Change Password
Sign Out

© 2011 PasTest Ltd | About Us | Contact Us | Help

file:///E|/Shakhawan/Endocrinology/162.htm[3/13/2012 7:09:09 PM]


MyPasTest

Main Navigation
Question Browser: MRCP 1
Home
Subscriptions Question Browser Exam Builder Saved Exams
» MRCP 1 Session Progress
• Question Browser
Questions Correct 15
• Timed Test A 42-year-old patient complains of severe fatigue following
surgery for a pituitary tumour and is put on growth Questions Incorrect 3
• Mock Exam
hormone replacement. Which of the following is a well- Questions Total 18
• Past Papers recognised effect of this treatment?
Questions Percentage 83 %
• Random Questions
• My Performance Decrease in serum lipoprotein(a) More
concentration
• Media Bank
Increase in fasting serum triglyceride
• New Multimedia concentration
Online Extras Increase in fat mass
Library Increase in lean body mass Your answer
Community Increase in serum total cholesterol
Help concentration
PasTest Store

My Account Growth hormone replacement in adults with deficiency


of the hormone has numerous effects, including an
Profile increase in vitality and overall quality of life. Lean body
Newsletters mass tends to increase but body fat decreases, often to
a greater extent. Serum total- and LDL-cholesterol
My Career and Exams concentrations and triglycerides tend to decrease but
Order History lipoprotein(a) concentration may increase. Reference: Normal Values
Learning Goals
2580 Click to open/closeHaematology
Question Filters Click to open/closeImmunoglobulins
Click to open/closeBiochemistry
Click to open/closeDiabetes Click to
Security open/closeEndocrinology Click to
open/closeBlood gases Click to
Change Password open/closeCSF
Sign Out

© 2011 PasTest Ltd | About Us | Contact Us | Help

file:///E|/Shakhawan/Endocrinology/162a.htm[3/13/2012 7:09:11 PM]


MyPasTest

Main Navigation
Question Browser: MRCP 1
Home
Subscriptions Question Browser Exam Builder Saved Exams
» MRCP 1
• Question Browser
• Timed Test A 37-year-old man with a diagnosis of hypogonadotrophic hypogonadism is being
• Mock Exam followed in the endocrine clinic. He does not desire fertility at present. Which would
be the most appropriate treatment at this stage?
• Past Papers
• Random Questions Oral testosterone replacement
• My Performance Pulsatile subcutaneous administration of gonadotrophin-releasing hormone
• Media Bank (GnRH)

• New Multimedia Regular injections of human chorionic gonadotrophin

Online Extras Regular injections of human menopausal gonadotrophin (HMG)

Library Regular testosterone injections

Community 2581
Help
PasTest Store

My Account

Profile
Newsletters
My Career and Exams
Order History
Learning Goals
Question Filters

Security

Change Password
Sign Out

file:///E|/Shakhawan/Endocrinology/163.htm[3/13/2012 7:09:12 PM]


MyPasTest

Main Navigation
Question Browser: MRCP 1
Home
Subscriptions Question Browser Exam Builder Saved Exams
» MRCP 1 Session Progress
• Question Browser
Questions Correct 16
• Timed Test A 37-year-old man with a diagnosis of hypogonadotrophic
hypogonadism is being followed in the endocrine clinic. He Questions Incorrect 3
• Mock Exam
does not desire fertility at present. Which would be the Questions Total 19
• Past Papers most appropriate treatment at this stage?
Questions Percentage 84 %
• Random Questions
• My Performance Oral testosterone replacement More
• Media Bank Pulsatile subcutaneous administration
of gonadotrophin-releasing hormone
• New Multimedia (GnRH)
Online Extras Regular injections of human chorionic
Library gonadotrophin
Community Regular injections of human
menopausal gonadotrophin (HMG)
Help
Regular testosterone injections Your answer
PasTest Store

My Account
If fertility is not required, there is no need to stimulate
Profile spermatogenesis with GnRH or gonadotrophins: only
Newsletters testosterone replacement is required. Testosterone
undecanoate is available for oral use, but frequent
My Career and Exams dosage is required; and, because of poor absorption,
Order History the plasma concentrations that can be achieved are Reference: Normal Values
often subnormal. Testosterone injections, implants or
Learning Goals patches are more reliable. Click to open/closeHaematology
Question Filters Click to open/closeImmunoglobulins
2581 Click to open/closeBiochemistry
Click to open/closeDiabetes Click to
Security open/closeEndocrinology Click to
open/closeBlood gases Click to
Change Password open/closeCSF
Sign Out

© 2011 PasTest Ltd | About Us | Contact Us | Help

file:///E|/Shakhawan/Endocrinology/163a.htm[3/13/2012 7:09:13 PM]


MyPasTest

Main Navigation
Question Browser: MRCP 1
Home
Subscriptions Question Browser Exam Builder Saved Exams
» MRCP 1 Session Progress
• Question Browser
Questions Correct 16
• Timed Test A 44-year-old man is surprised to find that he cannot
easily get his feet into a pair of shoes that he last wore 5 Questions Incorrect 3
• Mock Exam
years ago. He goes to buy a new pair and is told that his Questions Total 19
• Past Papers size has increased. He trawls the Internet for an
Questions Percentage 84 %
• Random Questions explanation and, deciding that he may have acromegaly,
consults his GP. The GP has not seen him for several years More
• My Performance
and thinks his appearance has changed, so refers him to
• Media Bank the endocrine clinic.
• New Multimedia
Which of the following would be the most useful
Online Extras first-line test for investigating him? Reference: Normal Values
Library
Glucose tolerance test with measurement of growth Click to
Community hormone open/closeHaematology
Help Insulin hypoglycaemia test (insulin tolerance test, Click to
insulin stress test) open/closeImmunoglobulins
PasTest Store
Click to
Measurement of serum growth hormone during sleep open/closeBiochemistry
My Account Measurement of serum growth hormone following Click to open/closeDiabetes
exercise Click to
Profile open/closeEndocrinology
Measurement of serum insulin-like growth factor-1 Click to open/closeBlood
Newsletters (IGF-1) concentration gases Click to
My Career and Exams open/closeCSF
2582
Order History
Learning Goals
Question Filters

Security

Change Password
Sign Out

© 2011 PasTest Ltd | About Us | Contact Us | Help

file:///E|/Shakhawan/Endocrinology/164.htm[3/13/2012 7:09:15 PM]


MyPasTest

Main Navigation
Question Browser: MRCP 1
Home
Subscriptions Question Browser Exam Builder Saved Exams
» MRCP 1 Session Progress
• Question Browser
Questions Correct 17
• Timed Test A 44-year-old man is surprised to find that he cannot
easily get his feet into a pair of shoes that he last wore 5 Questions Incorrect 3
• Mock Exam
years ago. He goes to buy a new pair and is told that his Questions Total 20
• Past Papers size has increased. He trawls the Internet for an
Questions Percentage 85 %
• Random Questions explanation and, deciding that he may have acromegaly,
consults his GP. The GP has not seen him for several years More
• My Performance
and thinks his appearance has changed, so refers him to
• Media Bank the endocrine clinic.
• New Multimedia
Which of the following would be the most useful
Online Extras first-line test for investigating him?
Library
Glucose tolerance test with
Community measurement of growth hormone
Help Insulin hypoglycaemia test (insulin
PasTest Store tolerance test, insulin stress test)
Measurement of serum growth
hormone during sleep
My Account
Measurement of serum growth
Profile hormone following exercise
Newsletters Measurement of serum insulin-like Your answer
My Career and Exams growth factor-1 (IGF-1) concentration

Order History Reference: Normal Values


Learning Goals
Click to open/closeHaematology
Question Filters In healthy individuals, growth hormone secretion is Click to open/closeImmunoglobulins
suppressed following the administration of glucose. Click to open/closeBiochemistry
Failure of suppression is diagnostic of excessive growth Click to open/closeDiabetes Click to
Security hormone secretion. As an initial screening test however, open/closeEndocrinology Click to
it may be more practical to measure serum IGF-1 and open/closeBlood gases Click to
Change Password to follow this with an OGTT if the results are abnormal. open/closeCSF
Sign Out An advantage of IGF-1 is that levels are relatively
stable over time. The insulin hypoglycaemia test,
measurement of growth hormone during sleep and
measurement following exercise, may be used in the
investigation of suspected growth hormone
insufficiency.

2582

© 2011 PasTest Ltd | About Us | Contact Us | Help

file:///E|/Shakhawan/Endocrinology/164a.htm[3/13/2012 7:09:16 PM]


MyPasTest

Main Navigation
Question Browser: MRCP 1
Home
Subscriptions Question Browser Exam Builder Saved Exams
» MRCP 1 Session Progress
• Question Browser
Questions Correct 17
• Timed Test MR imaging indicates a microadenoma in a 36-year-old
man presenting with features of acromegaly and proven to Questions Incorrect 3
• Mock Exam
have excessive growth hormone secretion. Which of the Questions Total 20
• Past Papers following would usually be the treatment of choice?
Questions Percentage 85 %
• Random Questions
• My Performance Medical treatment with a growth hormone-receptor More
antagonist
• Media Bank
Medical treatment with a somatostatin analogue
• New Multimedia
Radiotherapy
Online Extras Reference: Normal Values
Transfrontal surgery
Library
Trans-sphenoidal surgery Click to
Community open/closeHaematology
Help 2583 Click to
open/closeImmunoglobulins
PasTest Store
Click to
open/closeBiochemistry
My Account Click to open/closeDiabetes
Click to
Profile open/closeEndocrinology
Click to open/closeBlood
Newsletters gases Click to
My Career and Exams open/closeCSF
Order History
Learning Goals
Question Filters

Security

Change Password
Sign Out

© 2011 PasTest Ltd | About Us | Contact Us | Help

file:///E|/Shakhawan/Endocrinology/165.htm[3/13/2012 7:09:18 PM]


MyPasTest

Main Navigation
Question Browser: MRCP 1
Home
Subscriptions Question Browser Exam Builder Saved Exams
» MRCP 1 Session Progress
• Question Browser
Questions Correct 17
• Timed Test MR imaging indicates a microadenoma in a 36-year-old
man presenting with features of acromegaly and proven to Questions Incorrect 4
• Mock Exam
have excessive growth hormone secretion. Which of the Questions Total 21
• Past Papers following would usually be the treatment of choice?
Questions Percentage 80 %
• Random Questions
• My Performance Medical treatment with a growth More
hormone-receptor antagonist
• Media Bank
Medical treatment with a
• New Multimedia somatostatin analogue
Online Extras Radiotherapy
Library Transfrontal surgery Your answer
Community Trans-sphenoidal surgery Correct answer
Help
PasTest Store

My Account Surgery is generally regarded as the first-line treatment


for patients with acromegaly. The trans-sphenoidal
Profile route is preferred except with large tumours, which may
require a transfrontal approach. Radiotherapy is
Newsletters
sometimes used if surgery does not reduce growth
My Career and Exams hormone concentrations to acceptable levels. Medical
Order History treatment is used principally as an adjunct to surgery. Reference: Normal Values
Dopamine agonists (eg cabergoline) have been
Learning Goals superseded by somatostatin agonists (eg octreotide) Click to open/closeHaematology
Question Filters and growth hormone-receptor antagonists (eg Click to open/closeImmunoglobulins
pegvisomant) are now becoming available. Click to open/closeBiochemistry
Click to open/closeDiabetes Click to
Security open/closeEndocrinology Click to
2583 open/closeBlood gases Click to
Change Password open/closeCSF
Sign Out

© 2011 PasTest Ltd | About Us | Contact Us | Help

file:///E|/Shakhawan/Endocrinology/165a.htm[3/13/2012 7:09:19 PM]


MyPasTest

Main Navigation
Question Browser: MRCP 1
Home
Subscriptions Question Browser Exam Builder Saved Exams
» MRCP 1 Session Progress
• Question Browser
Questions Correct 17
• Timed Test A 49-year-old woman is investigated to determine the
cause of Cushing’s syndrome following the demonstration of Questions Incorrect 4
• Mock Exam
hypercortisolaemia by a high 24-hour urinary cortisol Questions Total 21
• Past Papers excretion. Her midnight plasma cortisol concentration is
Questions Percentage 80 %
• Random Questions elevated. Her 0900-h plasma cortisol concentration falls by
60% following a high-dose dexamethasone suppression More
• My Performance
test; plasma cortisol concentration increases by 25%
• Media Bank following intravenous corticotrophin-releasing hormone
• New Multimedia (CRH). Which is the most likely diagnosis?

Online Extras Adrenal adenoma Reference: Normal Values


Library Adrenal carcinoma
Click to
Community Bilateral adrenal hyperplasia open/closeHaematology
Help Cushing’s disease (pituitary-dependent Cushing’s Click to
syndrome) open/closeImmunoglobulins
PasTest Store
Click to
Ectopic secretion of ACTH open/closeBiochemistry
My Account Click to open/closeDiabetes
2585 Click to
Profile open/closeEndocrinology
Click to open/closeBlood
Newsletters gases Click to
My Career and Exams open/closeCSF
Order History
Learning Goals
Question Filters

Security

Change Password
Sign Out

© 2011 PasTest Ltd | About Us | Contact Us | Help

file:///E|/Shakhawan/Endocrinology/166.htm[3/13/2012 7:09:20 PM]


MyPasTest

Main Navigation
Question Browser: MRCP 1
Home
Subscriptions Question Browser Exam Builder Saved Exams
» MRCP 1 Session Progress
• Question Browser
Questions Correct 18
• Timed Test A 49-year-old woman is investigated to determine the
cause of Cushing’s syndrome following the demonstration of Questions Incorrect 4
• Mock Exam
hypercortisolaemia by a high 24-hour urinary cortisol Questions Total 22
• Past Papers excretion. Her midnight plasma cortisol concentration is
Questions Percentage 81 %
• Random Questions elevated. Her 0900-h plasma cortisol concentration falls by
60% following a high-dose dexamethasone suppression More
• My Performance
test; plasma cortisol concentration increases by 25%
• Media Bank following intravenous corticotrophin-releasing hormone
• New Multimedia (CRH). Which is the most likely diagnosis?

Online Extras Adrenal adenoma


Library Adrenal carcinoma
Community Bilateral adrenal hyperplasia
Help Cushing’s disease (pituitary-dependent Your answer
PasTest Store Cushing’s syndrome)
Ectopic secretion of ACTH
My Account

Profile
Newsletters Incomplete suppression of cortisol following high-dose
dexamethasone (2 mg 6-hourly for 48 h) is typical of
My Career and Exams Cushing’s disease: there is typically no suppression in
Order History patients with other causes of Cushing’s syndrome. An Reference: Normal Values
increase in cortisol following the administration of CRH
Learning Goals is also suggestive of Cushing’s disease, but with other Click to open/closeHaematology
Question Filters causes there is usually no response. Occasionally, Click to open/closeImmunoglobulins
carcinoid tumours secrete ACTH and cause similar Click to open/closeBiochemistry
responses in these tests to those seen in Cushing’s Click to open/closeDiabetes Click to
Security disease. This, however, is a rare cause of Cushing’s open/closeEndocrinology Click to
syndrome. open/closeBlood gases Click to
Change Password open/closeCSF
Sign Out 2585

© 2011 PasTest Ltd | About Us | Contact Us | Help

file:///E|/Shakhawan/Endocrinology/166a.htm[3/13/2012 7:09:22 PM]


MyPasTest

Main Navigation
Question Browser: MRCP 1
Home
Subscriptions Question Browser Exam Builder Saved Exams
» MRCP 1 Session Progress
• Question Browser
Questions Correct 18
• Timed Test A 50-year-old woman with hypertension that has been
difficult to control with drugs is found to have Questions Incorrect 4
• Mock Exam
hypercortisolaemia. Which one of the following clinical Questions Total 22
• Past Papers findings would most suggest that ectopic secretion of
Questions Percentage 81 %
• Random Questions ACTH is the cause of the condition?
• My Performance More
Glycosuria
• Media Bank
Worsening hypertension
• New Multimedia
Hypokalaemia
Online Extras Reference: Normal Values
Muscle wasting
Library
Weight loss Click to
Community open/closeHaematology
Help 2586 Click to
open/closeImmunoglobulins
PasTest Store
Click to
open/closeBiochemistry
My Account Click to open/closeDiabetes
Click to
Profile open/closeEndocrinology
Click to open/closeBlood
Newsletters gases Click to
My Career and Exams open/closeCSF
Order History
Learning Goals
Question Filters

Security

Change Password
Sign Out

© 2011 PasTest Ltd | About Us | Contact Us | Help

file:///E|/Shakhawan/Endocrinology/167.htm[3/13/2012 7:09:23 PM]


MyPasTest

Main Navigation
Question Browser: MRCP 1
Home
Subscriptions Question Browser Exam Builder Saved Exams
» MRCP 1 Session Progress
• Question Browser
Questions Correct 18
• Timed Test A 50-year-old woman with hypertension that has been
difficult to control with drugs is found to have Questions Incorrect 5
• Mock Exam
hypercortisolaemia. Which one of the following clinical Questions Total 23
• Past Papers findings would most suggest that ectopic secretion of
Questions Percentage 78 %
• Random Questions ACTH is the cause of the condition?
• My Performance More
Glycosuria
• Media Bank
Worsening hypertension
• New Multimedia
Hypokalaemia Your answer
Online Extras
Muscle wasting
Library
Weight loss Correct answer
Community
Help
PasTest Store
Glycosuria, hypertension, hypokalaemia and muscle
wasting can occur with Cushing’s syndrome from any
My Account cause. The most common manifestation is centripetal
fat deposition, often with weight gain, but weight loss
Profile suggests there is an underlying malignancy.
Newsletters
2586
My Career and Exams
Order History Reference: Normal Values
Learning Goals
Click to open/closeHaematology
Question Filters Click to open/closeImmunoglobulins
Click to open/closeBiochemistry
Click to open/closeDiabetes Click to
Security open/closeEndocrinology Click to
open/closeBlood gases Click to
Change Password open/closeCSF
Sign Out

© 2011 PasTest Ltd | About Us | Contact Us | Help

file:///E|/Shakhawan/Endocrinology/167a.htm[3/13/2012 7:09:25 PM]


MyPasTest

Main Navigation
Question Browser: MRCP 1
Home
Subscriptions Question Browser Exam Builder Saved Exams
» MRCP 1 Session Progress
• Question Browser
Questions Correct 18
• Timed Test An 18-year-old woman complains of a 2-month history of
vague ill health and nausea. She has had several episodes Questions Incorrect 5
• Mock Exam
of dizziness and is found to have postural hypotension. Questions Total 23
• Past Papers Which of the following investigations is required to
Questions Percentage 78 %
• Random Questions best demonstrate that these features are the result
of adrenal failure? More
• My Performance
• Media Bank Measurement of early morning and midnight plasma
• New Multimedia cortisol concentrations
Online Extras Measurement of early morning plasma ACTH Reference: Normal Values
(corticotrophin) concentration
Library
Measurement of 24-h urinary cortisol excretion Click to
Community open/closeHaematology
Overnight dexamethasone suppression test
Help Click to
Short ACTH (Synacthen) stimulation test open/closeImmunoglobulins
PasTest Store
Click to
2587 open/closeBiochemistry
My Account Click to open/closeDiabetes
Click to
Profile open/closeEndocrinology
Click to open/closeBlood
Newsletters gases Click to
My Career and Exams open/closeCSF
Order History
Learning Goals
Question Filters

Security

Change Password
Sign Out

© 2011 PasTest Ltd | About Us | Contact Us | Help

file:///E|/Shakhawan/Endocrinology/168.htm[3/13/2012 7:09:26 PM]


MyPasTest

Main Navigation
Question Browser: MRCP 1
Home
Subscriptions Question Browser Exam Builder Saved Exams
» MRCP 1 Session Progress
• Question Browser
Questions Correct 19
• Timed Test An 18-year-old woman complains of a 2-month history of
vague ill health and nausea. She has had several episodes Questions Incorrect 5
• Mock Exam
of dizziness and is found to have postural hypotension. Questions Total 24
• Past Papers Which of the following investigations is required to
Questions Percentage 79 %
• Random Questions best demonstrate that these features are the result
of adrenal failure? More
• My Performance
• Media Bank Measurement of early morning and
• New Multimedia midnight plasma cortisol concentrations
Online Extras Measurement of early morning plasma
ACTH (corticotrophin) concentration
Library
Measurement of 24-h urinary cortisol
Community excretion
Help Overnight dexamethasone suppression
PasTest Store test
Short ACTH (Synacthen) stimulation Your answer
test
My Account

Profile
Newsletters
My Career and Exams A diagnosis of adrenal failure is demonstrated by a
Order History failure of the plasma cortisol concentration to increase Reference: Normal Values
in response to ACTH. Demonstration of a high plasma
Learning Goals ACTH concentration indicates primary rather than Click to open/closeHaematology
Question Filters secondary (to pituitary insufficiency) adrenal failure. Click to open/closeImmunoglobulins
Random measurements of cortisol are only of value in Click to open/closeBiochemistry
sick patients, in whom cortisol concentrations should Click to open/closeDiabetes Click to
Security normally be high. Measurement of the diurnal variation open/closeEndocrinology Click to
in secretion (which is lost), the cortisol response to open/closeBlood gases Click to
Change Password dexamethasone (decreased) or urinary cortisol open/closeCSF
Sign Out excretion (increased) are used in the diagnosis of
Cushing’s syndrome but have no place in the diagnosis
of adrenal failure.

2587

© 2011 PasTest Ltd | About Us | Contact Us | Help

file:///E|/Shakhawan/Endocrinology/168a.htm[3/13/2012 7:09:28 PM]


MyPasTest

Main Navigation
Question Browser: MRCP 1
Home
Subscriptions Question Browser Exam Builder Saved Exams
» MRCP 1 Session Progress
• Question Browser
Questions Correct 19
• Timed Test A 17-year-old young woman is admitted to A&E having
collapsed at a rave. She is in a shocked state and unable to Questions Incorrect 5
• Mock Exam
give a coherent history, but is found to have a card in her Questions Total 24
• Past Papers purse that indicates that she is on steroids for adrenal
Questions Percentage 79 %
• Random Questions failure. A clinical diagnosis of an addisonian crisis is made
and a blood sample is taken for cortisol measurement. More
• My Performance
Finger prick glucose testing reveals a BM of 3.4 mmol/l.
• Media Bank Which of the following should be given the most
• New Multimedia priority in her management?

Online Extras Intravenous glucose infusion Reference: Normal Values


Library Parenteral administration of hydrocortisone
Click to
Community Replacement of mineralocorticoid open/closeHaematology
Help Resuscitation with intravenous physiological saline Click to
and hydrocortisone open/closeImmunoglobulins
PasTest Store
Click to
Treatment of any precipitating factor open/closeBiochemistry
My Account Click to open/closeDiabetes
2588 Click to
Profile open/closeEndocrinology
Click to open/closeBlood
Newsletters gases Click to
My Career and Exams open/closeCSF
Order History
Learning Goals
Question Filters

Security

Change Password
Sign Out

© 2011 PasTest Ltd | About Us | Contact Us | Help

file:///E|/Shakhawan/Endocrinology/169.htm[3/13/2012 7:09:29 PM]


MyPasTest

Main Navigation
Question Browser: MRCP 1
Home
Subscriptions Question Browser Exam Builder Saved Exams
» MRCP 1 Session Progress
• Question Browser
Questions Correct 19
• Timed Test A 17-year-old young woman is admitted to A&E having
collapsed at a rave. She is in a shocked state and unable to Questions Incorrect 6
• Mock Exam
give a coherent history, but is found to have a card in her Questions Total 25
• Past Papers purse that indicates that she is on steroids for adrenal
Questions Percentage 76 %
• Random Questions failure. A clinical diagnosis of an addisonian crisis is made
and a blood sample is taken for cortisol measurement. More
• My Performance
Finger prick glucose testing reveals a BM of 3.4 mmol/l.
• Media Bank Which of the following should be given the most
• New Multimedia priority in her management?

Online Extras Intravenous glucose infusion


Library Parenteral administration of Your answer
Community hydrocortisone
Help Replacement of mineralocorticoid
PasTest Store Resuscitation with intravenous Correct answer
physiological saline and
hydrocortisone
My Account Treatment of any precipitating factor
Profile
Newsletters
My Career and Exams
Order History All these measures (with the exception of Reference: Normal Values
mineralocorticoid replacement) are required in an
Learning Goals addisonian crisis, glucocorticoid and fluid replacements Click to open/closeHaematology
Question Filters the priority, often given simultaneously in clinical Click to open/closeImmunoglobulins
practice, but the immediate priority is fluid Click to open/closeBiochemistry
resuscitation. Mineralocorticoid replacement may be Click to open/closeDiabetes Click to
Security required long term, but is unnecessary in the acute open/closeEndocrinology Click to
setting because the large doses of hydrocortisone used open/closeBlood gases Click to
Change Password supply mineralocorticoid activity. open/closeCSF
Sign Out

2588

© 2011 PasTest Ltd | About Us | Contact Us | Help

file:///E|/Shakhawan/Endocrinology/169a.htm[3/13/2012 7:09:30 PM]


MyPasTest

Main Navigation
Question Browser: MRCP 1
Home
Subscriptions Question Browser Exam Builder Saved Exams
» MRCP 1 Session Progress
• Question Browser
Questions Correct 19
• Timed Test A 24-year-old man is found to have hypertension during an
examination for life assurance purposes. Over the next few Questions Incorrect 6
• Mock Exam
months, this is demonstrated to fluctuate considerably in Questions Total 25
• Past Papers severity and proves difficult to control. Which of the
Questions Percentage 76 %
• Random Questions following additional features would most suggest
that a phaeochromocytoma is causing his More
• My Performance
hypertension?
• Media Bank
• New Multimedia Diarrhoea
Online Extras Flushing Reference: Normal Values
Library Headache
Muscle weakness Click to
Community open/closeHaematology
Help Tremor Click to
open/closeImmunoglobulins
PasTest Store 2589 Click to
open/closeBiochemistry
My Account Click to open/closeDiabetes
Click to
Profile open/closeEndocrinology
Click to open/closeBlood
Newsletters gases Click to
My Career and Exams open/closeCSF
Order History
Learning Goals
Question Filters

Security

Change Password
Sign Out

© 2011 PasTest Ltd | About Us | Contact Us | Help

file:///E|/Shakhawan/Endocrinology/170.htm[3/13/2012 7:09:32 PM]


MyPasTest

Main Navigation
Question Browser: MRCP 1
Home
Subscriptions Question Browser Exam Builder Saved Exams
» MRCP 1 Session Progress
• Question Browser
Questions Correct 20
• Timed Test A 24-year-old man is found to have hypertension during an
examination for life assurance purposes. Over the next few Questions Incorrect 6
• Mock Exam
months, this is demonstrated to fluctuate considerably in Questions Total 26
• Past Papers severity and proves difficult to control. Which of the
Questions Percentage 76 %
• Random Questions following additional features would most suggest
that a phaeochromocytoma is causing his More
• My Performance
hypertension?
• Media Bank
• New Multimedia Diarrhoea
Online Extras Flushing
Library Headache Your answer
Community Muscle weakness
Help Tremor
PasTest Store

My Account Approximately 80% of patients with


phaeochromocytoma complain of headaches, which are
Profile often paroxysmal. Tremor is much less common. Pallor,
Newsletters not flushing, is seen in those with a
phaeochromocytoma and patients may complain of
My Career and Exams constipation. Flushing and diarrhoea are features of the
Order History carcinoid syndrome. Approximately one-quarter of Reference: Normal Values
patients experience muscle weakness, which can also
Learning Goals be present in primary aldosteronism, another endocrine Click to open/closeHaematology
Question Filters cause of hypertension. Click to open/closeImmunoglobulins
Click to open/closeBiochemistry
2589 Click to open/closeDiabetes Click to
Security open/closeEndocrinology Click to
open/closeBlood gases Click to
Change Password open/closeCSF
Sign Out

© 2011 PasTest Ltd | About Us | Contact Us | Help

file:///E|/Shakhawan/Endocrinology/170a.htm[3/13/2012 7:09:33 PM]


MyPasTest

Main Navigation
Question Browser: MRCP 1
Home
Subscriptions Question Browser Exam Builder Saved Exams
» MRCP 1 Session Progress
• Question Browser
Questions Correct 20
• Timed Test A 39-year-old man with untreated hypertension has a
Questions Incorrect 6
• Mock Exam plasma potassium concentration of 2.8 mmol/l. When
measured at 0900 h with the patient supine, his plasma Questions Total 26
• Past Papers aldosterone concentration is elevated and renin activity is Questions Percentage 76 %
• Random Questions low. When measured at 1200 h with the patient upright,
the plasma aldosterone concentration increases by 20%. More
• My Performance
Plasma cortisol concentration was also measured at both
• Media Bank times and found to be decreased by 50%.
• New Multimedia
What is the most likely diagnosis?
Online Extras Reference: Normal Values
Library Adrenal adenoma secreting aldosterone
Bilateral adrenal hyperplasia Click to
Community open/closeHaematology
Help Liddle’s syndrome Click to
Secondary aldosteronism open/closeImmunoglobulins
PasTest Store
Click to
Steroid 11 β-hydroxylase deficiency open/closeBiochemistry
My Account Click to open/closeDiabetes
2590 Click to
Profile open/closeEndocrinology
Click to open/closeBlood
Newsletters gases Click to
My Career and Exams open/closeCSF
Order History
Learning Goals
Question Filters

Security

Change Password
Sign Out

© 2011 PasTest Ltd | About Us | Contact Us | Help

file:///E|/Shakhawan/Endocrinology/171.htm[3/13/2012 7:09:35 PM]


MyPasTest

Main Navigation
Question Browser: MRCP 1
Home
Subscriptions Question Browser Exam Builder Saved Exams
» MRCP 1 Session Progress
• Question Browser
Questions Correct 20
• Timed Test A 39-year-old man with untreated hypertension has a
plasma potassium concentration of 2.8 mmol/l. When Questions Incorrect 7
• Mock Exam
measured at 0900 h with the patient supine, his plasma Questions Total 27
• Past Papers aldosterone concentration is elevated and renin activity is
Questions Percentage 74 %
• Random Questions low. When measured at 1200 h with the patient upright,
the plasma aldosterone concentration increases by 20%. More
• My Performance
Plasma cortisol concentration was also measured at both
• Media Bank times and found to be decreased by 50%.
• New Multimedia
What is the most likely diagnosis?
Online Extras
Library Adrenal adenoma secreting Your answer
aldosterone
Community
Bilateral adrenal hyperplasia Correct answer
Help
Liddle’s syndrome
PasTest Store
Secondary aldosteronism
Steroid 11 β-hydroxylase deficiency
My Account

Profile
Newsletters
Adrenal adenomas that secrete aldosterone (now
My Career and Exams thought to be the less likely cause of hyporeninaemic
Order History hyperaldosteronism) are usually sensitive to ACTH, and Reference: Normal Values
aldosterone secretion falls during the day as ACTH
Learning Goals
secretion (and that of cortisol) declines. With bilateral Click to open/closeHaematology
Question Filters adrenal hyperplasia, however, the aldosterone Click to open/closeImmunoglobulins
concentration is usually higher when the patient is erect Click to open/closeBiochemistry
than when supine. Hypertension is also a feature of Click to open/closeDiabetes Click to
Security Liddle’s syndrome and steroid 11 β -hydroxylase open/closeEndocrinology Click to
deficiency, but aldosterone concentrations are low. In open/closeBlood gases Click to
Change Password open/closeCSF
secondary aldosteronism, aldosterone secretion is
Sign Out increased secondary to an increase in renin secretion,
and plasma renin activity is normal or increased.

2590

© 2011 PasTest Ltd | About Us | Contact Us | Help

file:///E|/Shakhawan/Endocrinology/171a.htm[3/13/2012 7:09:36 PM]


MyPasTest

Main Navigation
Question Browser: MRCP 1
Home
Subscriptions Question Browser Exam Builder Saved Exams
» MRCP 1 Session Progress
• Question Browser
Questions Correct 0
• Timed Test A 52-year-old woman with primary hypothyroidism is being
treated with thyroxine replacement, the dose of which is Questions Incorrect 0
• Mock Exam
being titrated against the results of biochemical thyroid Questions Total 0
• Past Papers function tests. Two weeks after the last increase in dose,
Questions Percentage 0%
• Random Questions the results of thyroid function tests are: free thyroxine 28
pmol/l (normal 9–26); TSH 14 mU/l (normal 0.2–5.0). More
• My Performance
Which (if any) would be the most appropriate next
• Media Bank step in the management of this patient?
• New Multimedia
A decrease in the dose of thyroxine
Online Extras Reference: Normal Values
A further increase in the dose of thyroxine
Library
No change in dose at this time Click to
Community open/closeHaematology
Question patient’s compliance with medication
Help Click to
Replace thyroxine with triiodothyronine open/closeImmunoglobulins
PasTest Store
Click to
2591 open/closeBiochemistry
My Account Click to open/closeDiabetes
Click to
Profile open/closeEndocrinology
Click to open/closeBlood
Newsletters gases Click to
My Career and Exams open/closeCSF
Order History
Learning Goals
Question Filters

Security

Change Password
Sign Out

© 2011 PasTest Ltd | About Us | Contact Us | Help

file:///E|/Shakhawan/Endocrinology/172.htm[3/13/2012 7:09:38 PM]


MyPasTest

Main Navigation
Question Browser: MRCP 1
Home
Subscriptions Question Browser Exam Builder Saved Exams
» MRCP 1 Session Progress
• Question Browser
Questions Correct 0
• Timed Test A 52-year-old woman with primary hypothyroidism is being
treated with thyroxine replacement, the dose of which is Questions Incorrect 1
• Mock Exam
being titrated against the results of biochemical thyroid Questions Total 1
• Past Papers function tests. Two weeks after the last increase in dose,
Questions Percentage 0%
• Random Questions the results of thyroid function tests are: free thyroxine 28
pmol/l (normal 9–26); TSH 14 mU/l (normal 0.2–5.0). More
• My Performance
Which (if any) would be the most appropriate next
• Media Bank step in the management of this patient?
• New Multimedia
A decrease in the dose of thyroxine
Online Extras
A further increase in the dose of
Library thyroxine
Community No change in dose at this time Correct answer
Help Question patient’s compliance with Your answer
PasTest Store medication
Replace thyroxine with
triiodothyronine
My Account

Profile
Newsletters
It is too soon after the last change in dose for a new
My Career and Exams
steady state to have been achieved: it is usual to wait
Order History for a month before reviewing the response. When Reference: Normal Values
Learning Goals thyroxine replacement is started, the TSH
concentration often falls to normal more slowly than Click to open/closeHaematology
Question Filters free thyroxine increases. Free thyroxine Click to open/closeImmunoglobulins
concentrations in clinically euthyroid patients on Click to open/closeBiochemistry
thyroxine replacement are often high normal or even Click to open/closeDiabetes Click to
Security slightly elevated, reflecting the lack of production of open/closeEndocrinology Click to
triiodothyronine (T3 ) by the thyroid: peripheral open/closeBlood gases Click to
Change Password open/closeCSF
metabolism of thyroxine is the only source of T 3 .
Sign Out
Irregular medication could explain these results but
should not be considered until sufficient time has
elapsed for the data reliably to reflect the effects of
replacement.
Triiodothyronine is usually only used in very severe
hypothyroidism, when a rapid response to treatment
is required.

2591

© 2011 PasTest Ltd | About Us | Contact Us | Help

file:///E|/Shakhawan/Endocrinology/172a.htm[3/13/2012 7:09:39 PM]


MyPasTest

Main Navigation
Question Browser: MRCP 1
Home
Subscriptions Question Browser Exam Builder Saved Exams
» MRCP 1 Session Progress
• Question Browser
Questions Correct 0
• Timed Test A 20-year-old woman presents with anxiety and weight
loss with increased appetite. Thyrotoxicosis is suspected Questions Incorrect 0
• Mock Exam
and various investigations are performed. Which of the Questions Total 0
• Past Papers following findings would most suggest that she has
Questions Percentage 0%
• Random Questions Graves’ disease?
• My Performance More
High ESR
• Media Bank
High serum triiodothyronine (T3 ) concentration but
• New Multimedia normal thyroxine (T4 ) concentration
Online Extras Reference: Normal Values
High titre of thyroid peroxidase autoantibodies
Library
Low thyroid uptake of technecium-99m Click to
Community open/closeHaematology
Normal serum TSH concentration
Help Click to
2592 open/closeImmunoglobulins
PasTest Store
Click to
open/closeBiochemistry
My Account Click to open/closeDiabetes
Click to
Profile open/closeEndocrinology
Click to open/closeBlood
Newsletters gases Click to
My Career and Exams open/closeCSF
Order History
Learning Goals
Question Filters

Security

Change Password
Sign Out

© 2011 PasTest Ltd | About Us | Contact Us | Help

file:///E|/Shakhawan/Endocrinology/173.htm[3/13/2012 7:09:41 PM]


MyPasTest

Main Navigation
Question Browser: MRCP 1
Home
Subscriptions Question Browser Exam Builder Saved Exams
» MRCP 1 Session Progress
• Question Browser
Questions Correct 1
• Timed Test A 20-year-old woman presents with anxiety and weight
loss with increased appetite. Thyrotoxicosis is suspected Questions Incorrect 0
• Mock Exam
and various investigations are performed. Which of the Questions Total 1
• Past Papers following findings would most suggest that she has
Questions Percentage 100 %
• Random Questions Graves’ disease?
• My Performance More
High ESR
• Media Bank
High serum triiodothyronine (T3 )
• New Multimedia concentration but normal thyroxine (T4 )
Online Extras concentration
Library High titre of thyroid peroxidase Your answer
Community autoantibodies
Help Low thyroid uptake of technecium-99m
PasTest Store Normal serum TSH concentration

My Account
Antibodies to thyroid peroxidase and thyroglobulin are
Profile
found in the serum of the majority of patients with
Newsletters Graves’ disease, in which radioisotope uptake is
My Career and Exams typically increased. A low uptake occurs in subacute (de
Quervain’s) thyroiditis (in which the erythrocyte
Order History sedimentation rate (ESR) is typically elevated). Elevated Reference: Normal Values
Learning Goals T 3 with a normal T 4 (‘T3 toxicosis’) can occur early in
Click to open/closeHaematology
Question Filters the course of thyrotoxicosis from any cause. The high
Click to open/closeImmunoglobulins
concentrations of thyroid hormones suppress TSH
Click to open/closeBiochemistry
secretion to very low levels in thyrotoxicosis due to
Click to open/closeDiabetes Click to
Security thyroid disease: detectable (not always elevated) TSH in
open/closeEndocrinology Click to
thyrotoxicosis suggests that this is caused by a pituitary
open/closeBlood gases Click to
Change Password adenoma secreting TSH.
open/closeCSF
Sign Out
2592

© 2011 PasTest Ltd | About Us | Contact Us | Help

file:///E|/Shakhawan/Endocrinology/173a.htm[3/13/2012 7:09:42 PM]


MyPasTest

Main Navigation
Question Browser: MRCP 1
Home
Subscriptions Question Browser Exam Builder Saved Exams
» MRCP 1 Session Progress
• Question Browser
Questions Correct 1
• Timed Test A 35-year-old woman had a febrile infection associated
with a painful swelling in her neck a week ago. Her thyroid Questions Incorrect 0
• Mock Exam
function tests show evidence of thyrotoxicosis and her ESR Questions Total 1
• Past Papers is raised. What is the most likely diagnosis?
Questions Percentage 100 %
• Random Questions
• My Performance Sporadic goitre More
• Media Bank Hashimoto thyroiditis

• New Multimedia Fibromatosis

Online Extras De Quervain’s thyroiditis


Reference: Normal Values
Library Follicular thyroid carcinoma
Click to
Community 2603 open/closeHaematology
Help Click to
open/closeImmunoglobulins
PasTest Store
Click to
open/closeBiochemistry
My Account Click to open/closeDiabetes
Click to
Profile open/closeEndocrinology
Click to open/closeBlood
Newsletters gases Click to
My Career and Exams open/closeCSF
Order History
Learning Goals
Question Filters

Security

Change Password
Sign Out

© 2011 PasTest Ltd | About Us | Contact Us | Help

file:///E|/Shakhawan/Endocrinology/174.htm[3/13/2012 7:09:43 PM]


MyPasTest

Main Navigation
Question Browser: MRCP 1
Home
Subscriptions Question Browser Exam Builder Saved Exams
» MRCP 1
• Question Browser
• Timed Test A 35-year-old woman had a febrile infection associated with a painful swelling in her
• Mock Exam neck a week ago. Her thyroid function tests show evidence of thyrotoxicosis and her
ESR is raised. What is the most likely diagnosis?
• Past Papers
• Random Questions Sporadic goitre
• My Performance Hashimoto thyroiditis
• Media Bank Fibromatosis
• New Multimedia De Quervain’s thyroiditis Your answer
Online Extras Follicular thyroid carcinoma
Library
Community
Help
PasTest Store Subacute (or de Quervain’s) thyroiditis is due to thyroid infection by any of a
number of viruses, especially paramyxoviruses (mumps), coxsackieviruses,
influenza viruses, adenoviruses and echoviruses.
My Account
The most prominent symptom is pain in the thyroid, often radiating to the ears.
Profile A small, tender goitre can be palpated that is usually diffuse, but there can be
asymmetrical involvement. Systemic upset with fever is variable but sometimes
Newsletters profound, and symptoms of a prodromal viral infection several weeks earlier may
My Career and Exams be recalled. There is a granulomatous thyroid inflammation with follicular
destruction, and the release of thyroid hormones often results in a transient
Order History
thyrotoxicosis, lasting for 1–4 weeks. Continuing thyroid destruction then leads
Learning Goals to a phase of hypothyroidism once stored hormone is depleted. This lasts 4–12
Question Filters weeks before euthyroidism is restored, but relapses occur in 10–20% of cases.
Sometimes only one phase of thyroid disturbance is seen.
Confirmation of the clinical diagnosis is made by finding an elevated erythrocyte
Security sedimentation rate (ESR) and low or absent radioiodine uptake by the thyroid.
Thyroid function requires continuous monitoring as the disease evolves. Mild
Change Password cases may resolve spontaneously with aspirin as symptomatic treatment, but
Sign Out most patients benefit from prednisolone 40–60 mg daily, which rapidly alleviates
the pain. The dose is tapered over 6–8 weeks, depending largely on symptoms.
Propranolol may be useful for thyrotoxic symptoms, and temporary thyroxine
replacement is sometimes needed during the hypothyroid phase.

2603

file:///E|/Shakhawan/Endocrinology/174a.htm[3/13/2012 7:09:45 PM]


MyPasTest

Main Navigation
Question Browser: MRCP 1
Home
Subscriptions Question Browser Exam Builder Saved Exams
» MRCP 1 Session Progress
• Question Browser
Questions Correct 2
• Timed Test A 60-year-old woman presented complaining of a 6.3 kg (1
stone) weight loss, polyuria and depressive mood for 8 Questions Incorrect 0
• Mock Exam
weeks. The following laboratory results are obtained: Questions Total 2
• Past Papers calcium 3.4 mmol/l, phosphate 1.1 mmol/l, parathyroid
Questions Percentage 100 %
• Random Questions hormone 5 ng/l (reference < 60 ng/l). What is the most
likely cause for her complaints? More
• My Performance
• Media Bank Primary hyperparathyroidism
• New Multimedia Secondary hyperparathyroidism
Online Extras Hypercalcaemia due to cancer Reference: Normal Values
Library Vitamin D deficiency
Click to
Community Hyperthyroidism open/closeHaematology
Help Click to
2604 open/closeImmunoglobulins
PasTest Store
Click to
open/closeBiochemistry
My Account Click to open/closeDiabetes
Click to
Profile open/closeEndocrinology
Click to open/closeBlood
Newsletters gases Click to
My Career and Exams open/closeCSF
Order History
Learning Goals
Question Filters

Security

Change Password
Sign Out

© 2011 PasTest Ltd | About Us | Contact Us | Help

file:///E|/Shakhawan/Endocrinology/175.htm[3/13/2012 7:09:46 PM]


MyPasTest

Main Navigation
Question Browser: MRCP 1
Home
Subscriptions Question Browser Exam Builder Saved Exams
» MRCP 1 Session Progress
• Question Browser
Questions Correct 3
• Timed Test A 60-year-old woman presented complaining of a 6.3 kg (1
stone) weight loss, polyuria and depressive mood for 8 Questions Incorrect 0
• Mock Exam
weeks. The following laboratory results are obtained: Questions Total 3
• Past Papers calcium 3.4 mmol/l, phosphate 1.1 mmol/l, parathyroid
Questions Percentage 100 %
• Random Questions hormone 5 ng/l (reference < 60 ng/l). What is the most
likely cause for her complaints? More
• My Performance
• Media Bank Primary hyperparathyroidism
• New Multimedia Secondary hyperparathyroidism
Online Extras Hypercalcaemia due to cancer Your answer
Library Vitamin D deficiency
Community Hyperthyroidism
Help
PasTest Store

My Account The causes of hypercalcaemia can be classified


according to whether serum parathyroid hormone
Profile concentrations are elevated (ie primary
Newsletters hyperparathyroidism) or low (ie not due to a
parathyroid tumour). Primary hyperparathyroidism
My Career and Exams and malignancy are the most common causes and
Order History account for more than 90% of patients with Reference: Normal Values
hypercalcaemia.
Learning Goals
Click to open/closeHaematology
Question Filters A detailed clinical history and examination will usually
Click to open/closeImmunoglobulins
help to differentiate between these two diagnoses. In
Click to open/closeBiochemistry
primary hyperparathyroidism, the hypercalcaemia is
Click to open/closeDiabetes Click to
Security often less than 3 mmol/l, asymptomatic and may
open/closeEndocrinology Click to
have been present for months or years. However, in
open/closeBlood gases Click to
Change Password malignancy, the patients are usually acutely ill, often
open/closeCSF
with neurological symptoms, the hypercalcaemia is
Sign Out
more than 3 mmol/l and the cancer (eg lung, breast
or myeloma) is often readily apparent.

2604

© 2011 PasTest Ltd | About Us | Contact Us | Help

file:///E|/Shakhawan/Endocrinology/175a.htm[3/13/2012 7:09:48 PM]


MyPasTest

Main Navigation
Question Browser: MRCP 1
Home
Subscriptions Question Browser Exam Builder Saved Exams
» MRCP 1 Session Progress
• Question Browser
Questions Correct 3
• Timed Test A 17-year-old young woman has been referred by her
gynaecologist. She has been complaining of amenorrhoea Questions Incorrect 0
• Mock Exam
for 5 months, although no gynaecological abnormality has Questions Total 3
• Past Papers been found. She feels well and is very active but her weight
Questions Percentage 100 %
• Random Questions has decreased from 61 kg to 43 kg in the last 6 months.
Her height is 168 cm. On examination her BP is 90/60 More
• My Performance
mmHg, heart rate 64 bpm. What is the most likely
• Media Bank diagnosis?
• New Multimedia
Conn’s syndrome
Online Extras Reference: Normal Values
Crohn’s disease
Library
Anorexia nervosa Click to
Community open/closeHaematology
Hyperthyroidism
Help Click to
Diabetes mellitus open/closeImmunoglobulins
PasTest Store
Click to
2605 open/closeBiochemistry
My Account Click to open/closeDiabetes
Click to
Profile open/closeEndocrinology
Click to open/closeBlood
Newsletters gases Click to
My Career and Exams open/closeCSF
Order History
Learning Goals
Question Filters

Security

Change Password
Sign Out

© 2011 PasTest Ltd | About Us | Contact Us | Help

file:///E|/Shakhawan/Endocrinology/176.htm[3/13/2012 7:09:49 PM]


MyPasTest

Main Navigation
Question Browser: MRCP 1
Home
Subscriptions Question Browser Exam Builder Saved Exams
» MRCP 1
• Question Browser
• Timed Test A 17-year-old young woman has been referred by her gynaecologist. She has been
• Mock Exam complaining of amenorrhoea for 5 months, although no gynaecological abnormality
has been found. She feels well and is very active but her weight has decreased from
• Past Papers 61 kg to 43 kg in the last 6 months. Her height is 168 cm. On examination her BP is
• Random Questions 90/60 mmHg, heart rate 64 bpm. What is the most likely diagnosis?
• My Performance
Conn’s syndrome
• Media Bank
Crohn’s disease
• New Multimedia
Anorexia nervosa Your answer
Online Extras
Hyperthyroidism
Library
Diabetes mellitus
Community
Help
PasTest Store
Patients with anorexia nervosa actively maintain an unduly low body weight. For
My Account diagnostic purposes, an ‘unduly low body weight’ may be defined as a weight at
least 15% below that expected for the person’s age, height and sex, or as a body
Profile mass index below 17.5. Amenorrhoea (in postmenarchal women who are not
taking an oral contraceptive) is almost always present in these patients.
Newsletters
My Career and Exams
Order History 2605
Learning Goals
Question Filters

Security

Change Password
Sign Out

file:///E|/Shakhawan/Endocrinology/176a.htm[3/13/2012 7:09:50 PM]


MyPasTest

Main Navigation
Question Browser: MRCP 1
Home
Subscriptions Question Browser Exam Builder Saved Exams
» MRCP 1 Session Progress
• Question Browser
Questions Correct 4
• Timed Test A patient has been referred by her GP because she has
been complaining of frequent episodes of sweating and Questions Incorrect 0
• Mock Exam
palpitations associated with a low blood glucose level. The Questions Total 4
• Past Papers family history reveals a brother with type-1 diabetes. A
Questions Percentage 100 %
• Random Questions blood test shows the following results: glucose 1.1 mmol/l,
insulin > 500 pmol/l (reference 15–100 pmol/l) and C- More
• My Performance
peptide of < 0.2 nmol/l (reference 0.2–1.4 nmol/l). What
• Media Bank is the most likely diagnosis?
• New Multimedia
Type-1 diabetes mellitus
Online Extras Reference: Normal Values
Insulinoma
Library
Pancreatic carcinoma Click to
Community open/closeHaematology
Factitious insulin-induced hypoglycaemia
Help Click to
Maturity-onset diabetes of the young (MODY) open/closeImmunoglobulins
PasTest Store
Click to
2606 open/closeBiochemistry
My Account Click to open/closeDiabetes
Click to
Profile open/closeEndocrinology
Click to open/closeBlood
Newsletters gases Click to
My Career and Exams open/closeCSF
Order History
Learning Goals
Question Filters

Security

Change Password
Sign Out

© 2011 PasTest Ltd | About Us | Contact Us | Help

file:///E|/Shakhawan/Endocrinology/177.htm[3/13/2012 7:09:52 PM]


MyPasTest

Main Navigation
Question Browser: MRCP 1
Home
Subscriptions Question Browser Exam Builder Saved Exams
» MRCP 1 Session Progress
• Question Browser
Questions Correct 5
• Timed Test A patient has been referred by her GP because she has
been complaining of frequent episodes of sweating and Questions Incorrect 0
• Mock Exam
palpitations associated with a low blood glucose level. The Questions Total 5
• Past Papers family history reveals a brother with type-1 diabetes. A
Questions Percentage 100 %
• Random Questions blood test shows the following results: glucose 1.1 mmol/l,
insulin > 500 pmol/l (reference 15–100 pmol/l) and C- More
• My Performance
peptide of < 0.2 nmol/l (reference 0.2–1.4 nmol/l). What
• Media Bank is the most likely diagnosis?
• New Multimedia
Type-1 diabetes mellitus
Online Extras
Insulinoma
Library
Pancreatic carcinoma
Community
Factitious insulin-induced Your answer
Help hypoglycaemia
PasTest Store Maturity-onset diabetes of the young
(MODY)
My Account

Profile
Newsletters
Factitious insulin-induced hypoglycaemia is as common
My Career and Exams in previously healthy subjects as in insulin-dependent
Order History diabetics and is due to the deliberate, but concealed, Reference: Normal Values
injection of insulin. The history suggests insulinoma, but
Learning Goals this is eliminated by the laboratory results that reveal Click to open/closeHaematology
Question Filters high plasma insulin and low C-peptide (and proinsulin) Click to open/closeImmunoglobulins
concentrations during hypoglycaemia. In longstanding Click to open/closeBiochemistry
factitious hypoglycaemia, and in insulin-treated Click to open/closeDiabetes Click to
Security diabetics, insulin antibodies may be present in the open/closeEndocrinology Click to
plasma. Although once considered a strong pointer to open/closeBlood gases Click to
Change Password factitious hypoglycaemia, the presence of insulin open/closeCSF
Sign Out antibodies should nowadays suggest autoimmune insulin
syndrome.

2606

© 2011 PasTest Ltd | About Us | Contact Us | Help

file:///E|/Shakhawan/Endocrinology/177a.htm[3/13/2012 7:09:53 PM]


MyPasTest

Main Navigation
Question Browser: MRCP 1
Home
Subscriptions Question Browser Exam Builder Saved Exams
» MRCP 1 Session Progress
• Question Browser
Questions Correct 5
• Timed Test A 55-year-old obese patient with hypercholesterolaemia
(LDL 5.2 mmol/l, HDL 1 mmol/l), well-controlled type-2 Questions Incorrect 0
• Mock Exam
diabetes and hypertension has been on a low cholesterol Questions Total 5
• Past Papers diet for the last 6 months. His latest LDL level is 4.8
Questions Percentage 100 %
• Random Questions mmol/l, and triglycerides are within the normal range. In
terms of primary prevention, what is the next More
• My Performance
therapeutic step in his management?
• Media Bank
• New Multimedia Add pioglitazone
Online Extras Add gemfibrozil Reference: Normal Values
Library Add clopidogrel
Add statins Click to
Community open/closeHaematology
Help Add ezetimibe Click to
open/closeImmunoglobulins
PasTest Store 2607 Click to
open/closeBiochemistry
My Account Click to open/closeDiabetes
Click to
Profile open/closeEndocrinology
Click to open/closeBlood
Newsletters gases Click to
My Career and Exams open/closeCSF
Order History
Learning Goals
Question Filters

Security

Change Password
Sign Out

© 2011 PasTest Ltd | About Us | Contact Us | Help

file:///E|/Shakhawan/Endocrinology/178.htm[3/13/2012 7:09:55 PM]


MyPasTest

Main Navigation
Question Browser: MRCP 1
Home
Subscriptions Question Browser Exam Builder Saved Exams
» MRCP 1 Session Progress
• Question Browser
Questions Correct 6
• Timed Test A 55-year-old obese patient with hypercholesterolaemia
(LDL 5.2 mmol/l, HDL 1 mmol/l), well-controlled type-2 Questions Incorrect 0
• Mock Exam
diabetes and hypertension has been on a low cholesterol Questions Total 6
• Past Papers diet for the last 6 months. His latest LDL level is 4.8
Questions Percentage 100 %
• Random Questions mmol/l, and triglycerides are within the normal range. In
terms of primary prevention, what is the next More
• My Performance
therapeutic step in his management?
• Media Bank
• New Multimedia Add pioglitazone
Online Extras Add gemfibrozil
Library Add clopidogrel
Community Add statins Your answer
Help Add ezetimibe
PasTest Store

My Account
The latest European guidelines suggest that LDL
Profile cholesterol should be < 4.5 mmol/l rather than < 5
mmol/l in secondary prevention, while evidence from
Newsletters
primary prevention suggests that similar targets should
My Career and Exams also apply to primary prevention. Statins are the
Order History medication of choice. As targets are reduced and further Reference: Normal Values
evidence from the low–dose–high-dose statin
Learning Goals comparator trials are announced, it is likely that targets Click to open/closeHaematology
Question Filters will fall further and that combination therapy for Click to open/closeImmunoglobulins
hyperlipidaemia will become routine as is the case in Click to open/closeBiochemistry
hypertension. The challenge will still remain with Click to open/closeDiabetes Click to
Security implementation, but the benefits of satisfactory risk- open/closeEndocrinology Click to
factor control are that both morbidity and mortality open/closeBlood gases Click to
Change Password from cardiovascular disease will be decreased. open/closeCSF
Sign Out

2607

© 2011 PasTest Ltd | About Us | Contact Us | Help

file:///E|/Shakhawan/Endocrinology/178a.htm[3/13/2012 7:09:56 PM]


MyPasTest

Main Navigation
Question Browser: MRCP 1
Home
Subscriptions Question Browser Exam Builder Saved Exams
» MRCP 1 Session Progress
• Question Browser
Questions Correct 6
• Timed Test 50-year-old woman has been diagnosed with a pericardial
effusion. Her BMI is 31, and her hair is thinning on Questions Incorrect 0
• Mock Exam
examination. Questions Total 6
• Past Papers
Which endocrine disease is most likely to be Questions Percentage 100 %
• Random Questions
associated with this finding? More
• My Performance
• Media Bank Hyperthyroidism
• New Multimedia Hypothyroidism
Online Extras Phaeochromocytoma Reference: Normal Values
Library Chronic renal insufficiency
Click to
Community Hypogonadism open/closeHaematology
Help Click to
2608 open/closeImmunoglobulins
PasTest Store
Click to
open/closeBiochemistry
My Account Click to open/closeDiabetes
Click to
Profile open/closeEndocrinology
Click to open/closeBlood
Newsletters gases Click to
My Career and Exams open/closeCSF
Order History
Learning Goals
Question Filters

Security

Change Password
Sign Out

© 2011 PasTest Ltd | About Us | Contact Us | Help

file:///E|/Shakhawan/Endocrinology/179.htm[3/13/2012 7:09:58 PM]


MyPasTest

Main Navigation
Question Browser: MRCP 1
Home
Subscriptions Question Browser Exam Builder Saved Exams
» MRCP 1 Session Progress
• Question Browser
Questions Correct 7
• Timed Test 50-year-old woman has been diagnosed with a pericardial
effusion. Her BMI is 31, and her hair is thinning on Questions Incorrect 0
• Mock Exam
examination. Questions Total 7
• Past Papers
Which endocrine disease is most likely to be Questions Percentage 100 %
• Random Questions
associated with this finding? More
• My Performance
• Media Bank Hyperthyroidism
• New Multimedia Hypothyroidism Your answer
Online Extras Phaeochromocytoma
Library Chronic renal insufficiency
Community Hypogonadism
Help
PasTest Store

My Account Clinically silent pericardial effusion is common in


untreated hypothyroidism, it may be seen in
Profile hyperthyroidism but incidence is less. The effusion itself
has a high cholesterol content, which may produce an
Newsletters unusual secondary pericarditis with cholesterol deposits
My Career and Exams of ‘gold paint’ appearance. The pericardial effusion very
rarely needs to be treated in its own right, and subsides Reference: Normal Values
Order History
when thyroid replacement therapy is given.
Learning Goals
Click to open/closeHaematology
Question Filters Click to open/closeImmunoglobulins
2608 Click to open/closeBiochemistry
Click to open/closeDiabetes Click to
Security open/closeEndocrinology Click to
open/closeBlood gases Click to
Change Password open/closeCSF
Sign Out

© 2011 PasTest Ltd | About Us | Contact Us | Help

file:///E|/Shakhawan/Endocrinology/179a.htm[3/13/2012 7:09:59 PM]


MyPasTest

Main Navigation
Question Browser: MRCP 1
Home
Subscriptions Question Browser Exam Builder Saved Exams
» MRCP 1 Session Progress
• Question Browser
Questions Correct 7
• Timed Test What is the most reliable investigation for
determining the volume of the thyroid gland? Questions Incorrect 0
• Mock Exam
Questions Total 7
• Past Papers
Thyroid isotope uptake technetium-99m Questions Percentage 100 %
• Random Questions
Ultrasound
• My Performance More
X-ray of the neck
• Media Bank
Thyroid isotope uptake iodine-125
• New Multimedia
PET scan
Online Extras Reference: Normal Values
Library 2609
Click to
Community open/closeHaematology
Help Click to
open/closeImmunoglobulins
PasTest Store
Click to
open/closeBiochemistry
My Account Click to open/closeDiabetes
Click to
Profile open/closeEndocrinology
Click to open/closeBlood
Newsletters gases Click to
My Career and Exams open/closeCSF
Order History
Learning Goals
Question Filters

Security

Change Password
Sign Out

© 2011 PasTest Ltd | About Us | Contact Us | Help

file:///E|/Shakhawan/Endocrinology/180.htm[3/13/2012 7:10:00 PM]


MyPasTest

Main Navigation
Question Browser: MRCP 1
Home
Subscriptions Question Browser Exam Builder Saved Exams
» MRCP 1 Session Progress
• Question Browser
Questions Correct 8
• Timed Test What is the most reliable investigation for
determining the volume of the thyroid gland? Questions Incorrect 0
• Mock Exam
Questions Total 8
• Past Papers
Thyroid isotope uptake technetium- Questions Percentage 100 %
• Random Questions 99m
• My Performance Ultrasound Your answer More
• Media Bank X-ray of the neck
• New Multimedia Thyroid isotope uptake iodine-125
Online Extras PET scan
Library
Community
Help
PasTest Store Ultrasound is useful in determining thyroid size
accurately. Thyroidal uptake of radioisotopes (especially
technetium-99m) is indicated if destructive thyroiditis is
My Account suspected as a cause of goitre. Otherwise, the major
role for imaging is to ensure there is no tracheal
Profile compression or intrathoracic/retrosternal component in
a patient with suggestive symptoms, and a CT scan is
Newsletters
then the preferred investigation.
My Career and Exams
Order History Reference: Normal Values
2609
Learning Goals
Click to open/closeHaematology
Question Filters Click to open/closeImmunoglobulins
Click to open/closeBiochemistry
Click to open/closeDiabetes Click to
Security open/closeEndocrinology Click to
open/closeBlood gases Click to
Change Password open/closeCSF
Sign Out

© 2011 PasTest Ltd | About Us | Contact Us | Help

file:///E|/Shakhawan/Endocrinology/180a.htm[3/13/2012 7:10:02 PM]


MyPasTest

Main Navigation
Question Browser: MRCP 1
Home
Subscriptions Question Browser Exam Builder Saved Exams
» MRCP 1 Session Progress
• Question Browser
Questions Correct 8
• Timed Test A 30-year-old man presents with a 3-month history of
deteriorating physical performance at work, associated with Questions Incorrect 0
• Mock Exam
dysarthria and clumsiness. On examination he looks Questions Total 8
• Past Papers anaemic, has hepatomegaly and Kayser–Fleischer rings in
Questions Percentage 100 %
• Random Questions the cornea. What would be the most important
investigation to support the suspected clinical More
• My Performance
diagnosis?
• Media Bank
• New Multimedia CT scan of the brain
Online Extras MRI scan of the brain Reference: Normal Values
Library Liver biopsy
Serum ceruloplasmin level Click to
Community open/closeHaematology
Help CRP level Click to
open/closeImmunoglobulins
PasTest Store 2610 Click to
open/closeBiochemistry
My Account Click to open/closeDiabetes
Click to
Profile open/closeEndocrinology
Click to open/closeBlood
Newsletters gases Click to
My Career and Exams open/closeCSF
Order History
Learning Goals
Question Filters

Security

Change Password
Sign Out

© 2011 PasTest Ltd | About Us | Contact Us | Help

file:///E|/Shakhawan/Endocrinology/181.htm[3/13/2012 7:10:03 PM]


MyPasTest

Main Navigation
Question Browser: MRCP 1
Home
Subscriptions Question Browser Exam Builder Saved Exams
» MRCP 1 Session Progress
• Question Browser
Questions Correct 8
• Timed Test A 30-year-old man presents with a 3-month history of
deteriorating physical performance at work, associated with Questions Incorrect 1
• Mock Exam
dysarthria and clumsiness. On examination he looks Questions Total 9
• Past Papers anaemic, has hepatomegaly and Kayser–Fleischer rings in
Questions Percentage 88 %
• Random Questions the cornea. What would be the most important
investigation to support the suspected clinical More
• My Performance
diagnosis?
• Media Bank
• New Multimedia CT scan of the brain
Online Extras MRI scan of the brain
Library Liver biopsy Your answer
Community Serum ceruloplasmin level Correct answer
Help CRP level
PasTest Store

My Account
Serum ceruloplasmin can be measured enzymatically
Profile (copper oxidase), by radial immunodiffusion or by
reverse passive haemagglutination. The activity and
Newsletters
concentration of this glycoprotein is reduced or absent
My Career and Exams (less than 200 mg/l) in 95% of patients with Wilson’s
Order History disease. In the absence of Kayser-Fleischer rings or Reference: Normal Values
neurologic abnormalities, a liver biopsy for quantitative
Learning Goals copper determination would be essential to establish the Click to open/closeHaematology
Question Filters diagnosis of Wilson's disease, but this would likely not Click to open/closeImmunoglobulins
be necessary here given the typical clinical picture. Click to open/closeBiochemistry
Click to open/closeDiabetes Click to
Security open/closeEndocrinology Click to
2610 open/closeBlood gases Click to
Change Password open/closeCSF
Sign Out

© 2011 PasTest Ltd | About Us | Contact Us | Help

file:///E|/Shakhawan/Endocrinology/181a.htm[3/13/2012 7:10:05 PM]


MyPasTest

Main Navigation
Question Browser: MRCP 1
Home
Subscriptions Question Browser Exam Builder Saved Exams
» MRCP 1 Session Progress
• Question Browser
Questions Correct 8
• Timed Test A 30-year-old man presents with a 3-month history of
deteriorating physical performance at work, associated with Questions Incorrect 1
• Mock Exam
dysarthria and clumsiness. On examination he looks Questions Total 9
• Past Papers anaemic, has hepatomegaly and Kayser-Fleischer rings in
Questions Percentage 88 %
• Random Questions the cornea. What is the most likely diagnosis?
• My Performance More
Hepatitis C infection
• Media Bank
Wilson’s disease
• New Multimedia
Alcohol abuse
Online Extras Reference: Normal Values
Motor neurone disease
Library
Subacute sclerosing panencephalitis Click to
Community open/closeHaematology
Help 2611 Click to
open/closeImmunoglobulins
PasTest Store
Click to
open/closeBiochemistry
My Account Click to open/closeDiabetes
Click to
Profile open/closeEndocrinology
Click to open/closeBlood
Newsletters gases Click to
My Career and Exams open/closeCSF
Order History
Learning Goals
Question Filters

Security

Change Password
Sign Out

© 2011 PasTest Ltd | About Us | Contact Us | Help

file:///E|/Shakhawan/Endocrinology/182.htm[3/13/2012 7:10:06 PM]


MyPasTest

Main Navigation
Question Browser: MRCP 1
Home
Subscriptions Question Browser Exam Builder Saved Exams
» MRCP 1 Session Progress
• Question Browser
Questions Correct 9
• Timed Test A 30-year-old man presents with a 3-month history of
deteriorating physical performance at work, associated with Questions Incorrect 1
• Mock Exam
dysarthria and clumsiness. On examination he looks Questions Total 10
• Past Papers anaemic, has hepatomegaly and Kayser-Fleischer rings in
Questions Percentage 90 %
• Random Questions the cornea. What is the most likely diagnosis?
• My Performance More
Hepatitis C infection
• Media Bank
Wilson’s disease Your answer
• New Multimedia
Alcohol abuse
Online Extras
Motor neurone disease
Library
Subacute sclerosing panencephalitis
Community
Help
PasTest Store
Wilson’s disease may present in childhood, adolescence
My Account or early adulthood. Symptoms and signs may be
clinically undetectable under 5 years of age, and few
Profile present after the age of 35 years, although diagnosis
over 55 years has been reported. In 90% of patients,
Newsletters
the disease presents with juvenile hepatic disease or
My Career and Exams with neurological/psychiatric manifestations. In large
Order History studies of patients with Wilson’s disease, initial Reference: Normal Values
manifestations were: hepatic (40%); neurological
Learning Goals (30%); psychiatric (10%); haematological (12%); and Click to open/closeHaematology
Question Filters renal (1%). Some 25% of patients have two or more Click to open/closeImmunoglobulins
organs involved (usually liver and brain) at the initial Click to open/closeBiochemistry
assessment. Kayser–Fleischer rings are almost Click to open/closeDiabetes Click to
Security pathognomonic of Wilson’s disease. open/closeEndocrinology Click to
open/closeBlood gases Click to
Change Password open/closeCSF
Sign Out 2611

© 2011 PasTest Ltd | About Us | Contact Us | Help

file:///E|/Shakhawan/Endocrinology/182a.htm[3/13/2012 7:10:08 PM]


MyPasTest

Main Navigation
Question Browser: MRCP 1
Home
Subscriptions Question Browser Exam Builder Saved Exams
» MRCP 1 Session Progress
• Question Browser
Questions Correct 9
• Timed Test A 30-year-old man presents with a 3-month history of
deteriorating physical performance at work, associated with Questions Incorrect 1
• Mock Exam
dysarthria and clumsiness. On examination he looks Questions Total 10
• Past Papers anaemic, has hepatomegaly and Kayser–Fleischer rings in
Questions Percentage 90 %
• Random Questions the cornea. Given the likely diagnosis, what would be
the most important treatment option? More
• My Performance
• Media Bank Blood transfusion
• New Multimedia d-Penicillamine
Online Extras Ciclosporin Reference: Normal Values
Library Azathioprine
Click to
Community Interferon-alpha open/closeHaematology
Help Click to
2612 open/closeImmunoglobulins
PasTest Store
Click to
open/closeBiochemistry
My Account Click to open/closeDiabetes
Click to
Profile open/closeEndocrinology
Click to open/closeBlood
Newsletters gases Click to
My Career and Exams open/closeCSF
Order History
Learning Goals
Question Filters

Security

Change Password
Sign Out

© 2011 PasTest Ltd | About Us | Contact Us | Help

file:///E|/Shakhawan/Endocrinology/183.htm[3/13/2012 7:10:10 PM]


MyPasTest

Main Navigation
Question Browser: MRCP 1
Home
Subscriptions Question Browser Exam Builder Saved Exams
» MRCP 1 Session Progress
• Question Browser
Questions Correct 10
• Timed Test A 30-year-old man presents with a 3-month history of
deteriorating physical performance at work, associated with Questions Incorrect 1
• Mock Exam
dysarthria and clumsiness. On examination he looks Questions Total 11
• Past Papers anaemic, has hepatomegaly and Kayser–Fleischer rings in
Questions Percentage 90 %
• Random Questions the cornea. Given the likely diagnosis, what would be
the most important treatment option? More
• My Performance
• Media Bank Blood transfusion
• New Multimedia d-Penicillamine Your answer
Online Extras Ciclosporin
Library Azathioprine
Community Interferon-alpha
Help
PasTest Store

My Account d-Penicillamine is effective in removing copper from


patients with Wilson’s disease. The optimum time for
Profile treatment is in the early stages, and all patients with
Newsletters Wilson’s disease should be treated, even if
asymptomatic. Treatment is lifelong, unless the
My Career and Exams patient undergoes liver transplantation. The aim of
Order History treating a patient with Wilson’s disease is to reduce Reference: Normal Values
toxic copper levels in the body tissues. This can be
Learning Goals
achieved by increasing the urinary excretion of Click to open/closeHaematology
Question Filters copper. A negative copper balance should be Click to open/closeImmunoglobulins
monitored carefully since, with an increase in urine Click to open/closeBiochemistry
and faecal copper excretion that exceeds copper Click to open/closeDiabetes Click to
Security intake, increased urinary copper may reflect increased open/closeEndocrinology Click to
plasma non-ceruloplasmin copper (copper in its most open/closeBlood gases Click to
Change Password damaging form). open/closeCSF
Sign Out
The effect of any therapy should be regularly
monitored by clinical and radiological assessment, and
by biochemical monitoring of abnormal liver enzymes
and liver function.

2612

© 2011 PasTest Ltd | About Us | Contact Us | Help

file:///E|/Shakhawan/Endocrinology/183a.htm[3/13/2012 7:10:11 PM]


MyPasTest

Main Navigation
Question Browser: MRCP 1
Home
Subscriptions Question Browser Exam Builder Saved Exams
» MRCP 1 Session Progress
• Question Browser
Questions Correct 10
• Timed Test A patient was referred by his GP because of a borderline
fasting glucose. You arrange for a glucose tolerance test to Questions Incorrect 1
• Mock Exam
clarify the diagnosis. For this test how much glucose is Questions Total 11
• Past Papers dissolved in 250 ml of water?
Questions Percentage 90 %
• Random Questions
• My Performance 10 g More
• Media Bank 50 g

• New Multimedia 75 g

Online Extras 100 g


Reference: Normal Values
Library 150 g
Click to
Community 2613 open/closeHaematology
Help Click to
open/closeImmunoglobulins
PasTest Store
Click to
open/closeBiochemistry
My Account Click to open/closeDiabetes
Click to
Profile open/closeEndocrinology
Click to open/closeBlood
Newsletters gases Click to
My Career and Exams open/closeCSF
Order History
Learning Goals
Question Filters

Security

Change Password
Sign Out

© 2011 PasTest Ltd | About Us | Contact Us | Help

file:///E|/Shakhawan/Endocrinology/184.htm[3/13/2012 7:10:13 PM]


MyPasTest

Main Navigation
Question Browser: MRCP 1
Home
Subscriptions Question Browser Exam Builder Saved Exams
» MRCP 1 Session Progress
• Question Browser
Questions Correct 11
• Timed Test A patient was referred by his GP because of a borderline
fasting glucose. You arrange for a glucose tolerance test to Questions Incorrect 1
• Mock Exam
clarify the diagnosis. For this test how much glucose is Questions Total 12
• Past Papers dissolved in 250 ml of water?
Questions Percentage 91 %
• Random Questions
• My Performance 10 g More
• Media Bank 50 g

• New Multimedia 75 g Your answer

Online Extras 100 g

Library 150 g

Community
Help
PasTest Store
After an overnight fast, the subject drinks 75 g of
anhydrous glucose dissolved in 250 ml water, venous
My Account blood is sampled at baseline and 2 hours later. Food
intake should be normal during the preceding few days:
Profile poor nutrition can cause delayed hyperglycaemia with a
raised 2-hour value (the ‘lag’ curve).
Newsletters
My Career and Exams
Order History 2613 Reference: Normal Values
Learning Goals
Click to open/closeHaematology
Question Filters Click to open/closeImmunoglobulins
Click to open/closeBiochemistry
Click to open/closeDiabetes Click to
Security open/closeEndocrinology Click to
open/closeBlood gases Click to
Change Password open/closeCSF
Sign Out

© 2011 PasTest Ltd | About Us | Contact Us | Help

file:///E|/Shakhawan/Endocrinology/184a.htm[3/13/2012 7:10:14 PM]


MyPasTest

Main Navigation
Question Browser: MRCP 1
Home
Subscriptions Question Browser Exam Builder Saved Exams
» MRCP 1 Session Progress
• Question Browser
Questions Correct 11
• Timed Test A patient with longstanding type-2 diabetes was found to
have a urinary albumin excretion rate of 400 mg/l. His Questions Incorrect 1
• Mock Exam
diabetes is well controlled and he is normotensive. What Questions Total 12
• Past Papers additional drug should he be prescribed?
Questions Percentage 91 %
• Random Questions
• My Performance Aspirin More
• Media Bank Atenolol

• New Multimedia Lisinopril

Online Extras Clopidogrel


Reference: Normal Values
Library Pioglitazone
Click to
Community 2614 open/closeHaematology
Help Click to
open/closeImmunoglobulins
PasTest Store
Click to
open/closeBiochemistry
My Account Click to open/closeDiabetes
Click to
Profile open/closeEndocrinology
Click to open/closeBlood
Newsletters gases Click to
My Career and Exams open/closeCSF
Order History
Learning Goals
Question Filters

Security

Change Password
Sign Out

© 2011 PasTest Ltd | About Us | Contact Us | Help

file:///E|/Shakhawan/Endocrinology/185.htm[3/13/2012 7:10:15 PM]


MyPasTest

Main Navigation
Question Browser: MRCP 1
Home
Subscriptions Question Browser Exam Builder Saved Exams
» MRCP 1 Session Progress
• Question Browser
Questions Correct 12
• Timed Test A patient with longstanding type-2 diabetes was found to
have a urinary albumin excretion rate of 400 mg/l. His Questions Incorrect 1
• Mock Exam
diabetes is well controlled and he is normotensive. What Questions Total 13
• Past Papers additional drug should he be prescribed?
Questions Percentage 92 %
• Random Questions
• My Performance Aspirin More
• Media Bank Atenolol

• New Multimedia Lisinopril Your answer

Online Extras Clopidogrel

Library Pioglitazone

Community
Help
PasTest Store
In patients with type-1 diabetes with a mean entry
blood pressure of 122/77 mmHg, a combined analysis
My Account of one European and one American study (total n =
225) showed an adjusted risk reduction of 63% (95%
Profile CI, 16–84%; p = 0.017) for the development of clinical
nephropathy comparing the ACE inhibitor captopril 100
Newsletters
mg/day with placebo. Three smaller studies in
My Career and Exams normotensive type-2 patients have reported a similar
Order History reduction in the rate of development of clinical Reference: Normal Values
nephropathy. Thus blockade of the renin–angiotensin
Learning Goals system by any means appears to confer benefit. Click to open/closeHaematology
Question Filters Click to open/closeImmunoglobulins
Click to open/closeBiochemistry
2614 Click to open/closeDiabetes Click to
Security open/closeEndocrinology Click to
open/closeBlood gases Click to
Change Password open/closeCSF
Sign Out

© 2011 PasTest Ltd | About Us | Contact Us | Help

file:///E|/Shakhawan/Endocrinology/185a.htm[3/13/2012 7:10:17 PM]


MyPasTest

Main Navigation
Question Browser: MRCP 1
Home
Subscriptions Question Browser Exam Builder Saved Exams
» MRCP 1 Session Progress
• Question Browser
Questions Correct 0
• Timed Test A patient presents with truncal obesity, insulin resistance
and dyslipidaemia. What additional clinical feature Questions Incorrect 0
• Mock Exam
might you expect to be present? Questions Total 0
• Past Papers
Questions Percentage 0%
• Random Questions Asthma
• My Performance Renal failure More
• Media Bank Hypertension
• New Multimedia Cancer
Online Extras Ophthalmoplegia Reference: Normal Values
Library
2615 Click to
Community open/closeHaematology
Help Click to
open/closeImmunoglobulins
PasTest Store
Click to
open/closeBiochemistry
My Account Click to open/closeDiabetes
Click to
Profile open/closeEndocrinology
Click to open/closeBlood
Newsletters gases Click to
My Career and Exams open/closeCSF
Order History
Learning Goals
Question Filters

Security

Change Password
Sign Out

© 2011 PasTest Ltd | About Us | Contact Us | Help

file:///E|/Shakhawan/Endocrinology/186.htm[3/13/2012 7:10:18 PM]


MyPasTest

Main Navigation
Question Browser: MRCP 1
Home
Subscriptions Question Browser Exam Builder Saved Exams
» MRCP 1 Session Progress
• Question Browser
Questions Correct 1
• Timed Test A patient presents with truncal obesity, insulin resistance
and dyslipidaemia. What additional clinical feature Questions Incorrect 0
• Mock Exam
might you expect to be present? Questions Total 1
• Past Papers
Questions Percentage 100 %
• Random Questions Asthma
• My Performance Renal failure More
• Media Bank Hypertension Your answer
• New Multimedia Cancer
Online Extras Ophthalmoplegia
Library
Community
Help
PasTest Store Metabolic syndrome X is the term given to the co-
occurrence of insulin resistance and glucose intolerance
(ranging from mild to overt type-2 diabetes), with
My Account truncal obesity, dyslipidaemia (raised triglycerides and a
high LDL:HDL ratio) and hypertension.
Profile
Newsletters
2615
My Career and Exams
Order History Reference: Normal Values
Learning Goals
Click to open/closeHaematology
Question Filters Click to open/closeImmunoglobulins
Click to open/closeBiochemistry
Click to open/closeDiabetes Click to
Security open/closeEndocrinology Click to
open/closeBlood gases Click to
Change Password open/closeCSF
Sign Out

© 2011 PasTest Ltd | About Us | Contact Us | Help

file:///E|/Shakhawan/Endocrinology/186a.htm[3/13/2012 7:10:19 PM]


MyPasTest

Main Navigation
Question Browser: MRCP 1
Home
Subscriptions Question Browser Exam Builder Saved Exams
» MRCP 1 Session Progress
• Question Browser
Questions Correct 1
• Timed Test A patient has been complaining of a 2-month history of
intermittent flush associated with tachycardia and Questions Incorrect 0
• Mock Exam
wheezing. There have also been episodes of profuse watery Questions Total 1
• Past Papers diarrhoea. Given the suspected diagnosis, what would
Questions Percentage 100 %
• Random Questions be the most appropriate investigation?
• My Performance More
Dexamethasone suppression test
• Media Bank
Urinary catecholamine collection
• New Multimedia
Urinary 5-hydroxyindoleacetic acid collection
Online Extras Reference: Normal Values
Abdominal ultrasound
Library
24-hour ambulatory blood pressure recording Click to
Community open/closeHaematology
Help 2616 Click to
open/closeImmunoglobulins
PasTest Store
Click to
open/closeBiochemistry
My Account Click to open/closeDiabetes
Click to
Profile open/closeEndocrinology
Click to open/closeBlood
Newsletters gases Click to
My Career and Exams open/closeCSF
Order History
Learning Goals
Question Filters

Security

Change Password
Sign Out

© 2011 PasTest Ltd | About Us | Contact Us | Help

file:///E|/Shakhawan/Endocrinology/187.htm[3/13/2012 7:10:21 PM]


MyPasTest

Main Navigation
Question Browser: MRCP 1
Home
Subscriptions Question Browser Exam Builder Saved Exams
» MRCP 1 Session Progress
• Question Browser
Questions Correct 2
• Timed Test A patient has been complaining of a 2-month history of
intermittent flush associated with tachycardia and Questions Incorrect 0
• Mock Exam
wheezing. There have also been episodes of profuse watery Questions Total 2
• Past Papers diarrhoea. Given the suspected diagnosis, what would
Questions Percentage 100 %
• Random Questions be the most appropriate investigation?
• My Performance More
Dexamethasone suppression test
• Media Bank
Urinary catecholamine collection
• New Multimedia
Urinary 5-hydroxyindoleacetic acid Your answer
Online Extras collection
Library Abdominal ultrasound
Community 24-hour ambulatory blood pressure
Help recording
PasTest Store

My Account
The biologically active metabolite characteristically
Profile produced by metastatic carcinoid tumours is 5-
hydroxytryptamine (5-HT, serotonin), synthesised from
Newsletters
the amino acid tryptophan. 5-HT plays a part in the
My Career and Exams pathogenesis of some of the symptoms of the carcinoid
Order History syndrome, particularly the diarrhoea and Reference: Normal Values
bronchoconstriction. It is metabolised to 5-
Learning Goals hydroxyindoleacetic acid (5-HIAA), which accounts for Click to open/closeHaematology
Question Filters 95% of the urinary excretion of 5-HT. Click to open/closeImmunoglobulins
Click to open/closeBiochemistry
Click to open/closeDiabetes Click to
Security 2616 open/closeEndocrinology Click to
open/closeBlood gases Click to
Change Password open/closeCSF
Sign Out

© 2011 PasTest Ltd | About Us | Contact Us | Help

file:///E|/Shakhawan/Endocrinology/187a.htm[3/13/2012 7:10:22 PM]


MyPasTest

Main Navigation
Question Browser: MRCP 1
Home
Subscriptions Question Browser Exam Builder Saved Exams
» MRCP 1 Session Progress
• Question Browser
Questions Correct 2
• Timed Test A 55-year-old woman has been complaining of a 6-month
history of weight gain. She is otherwise well and takes no Questions Incorrect 0
• Mock Exam
medication. On examination her BMI is 28, BP 170/100 Questions Total 2
• Past Papers mmHg, she has a round red face and a slight atrophy of
Questions Percentage 100 %
• Random Questions her arm muscles. Renal function test and urinalysis are
normal. What is the next step in obtaining the More
• My Performance
diagnosis?
• Media Bank
• New Multimedia Dexamethasone suppression test
Online Extras Urinary catecholamine collection Reference: Normal Values
Library Urinary 5-hydroxyindoleacetic acid collection
Abdominal ultrasound Click to
Community open/closeHaematology
Help CT scan abdomen Click to
open/closeImmunoglobulins
PasTest Store 2617 Click to
open/closeBiochemistry
My Account Click to open/closeDiabetes
Click to
Profile open/closeEndocrinology
Click to open/closeBlood
Newsletters gases Click to
My Career and Exams open/closeCSF
Order History
Learning Goals
Question Filters

Security

Change Password
Sign Out

© 2011 PasTest Ltd | About Us | Contact Us | Help

file:///E|/Shakhawan/Endocrinology/188.htm[3/13/2012 7:10:24 PM]


MyPasTest

Main Navigation
Question Browser: MRCP 1
Home
Subscriptions Question Browser Exam Builder Saved Exams
» MRCP 1 Session Progress
• Question Browser
Questions Correct 3
• Timed Test A 55-year-old woman has been complaining of a 6-month
history of weight gain. She is otherwise well and takes no Questions Incorrect 0
• Mock Exam
medication. On examination her BMI is 28, BP 170/100 Questions Total 3
• Past Papers mmHg, she has a round red face and a slight atrophy of
Questions Percentage 100 %
• Random Questions her arm muscles. Renal function test and urinalysis are
normal. What is the next step in obtaining the More
• My Performance
diagnosis?
• Media Bank
• New Multimedia Dexamethasone suppression test Your answer
Online Extras Urinary catecholamine collection
Library Urinary 5-hydroxyindoleacetic acid
collection
Community
Abdominal ultrasound
Help
CT scan abdomen
PasTest Store

My Account

Profile Low-dose dexamethasone suppression tests the diurnal


circadian rhythm.
Newsletters
In normal subjects, the cortisol concentration begins to
My Career and Exams rise at 0200 h, peaking at 0530–0930 h. Concentrations
Order History drop during the morning and reach a nadir at around Reference: Normal Values
2400 h (cortisol < 50 nmol/l). Blood is taken for ACTH
Learning Goals and cortisol at 0900 h and 2400 h. The finding of a Click to open/closeHaematology
Question Filters normal circadian rhythm virtually rules out the presence Click to open/closeImmunoglobulins
of active Cushing's syndrome. Click to open/closeBiochemistry
Click to open/closeDiabetes Click to
Security open/closeEndocrinology Click to
2617 open/closeBlood gases Click to
Change Password open/closeCSF
Sign Out

© 2011 PasTest Ltd | About Us | Contact Us | Help

file:///E|/Shakhawan/Endocrinology/188a.htm[3/13/2012 7:10:25 PM]


MyPasTest

Main Navigation
Question Browser: MRCP 1
Home
Subscriptions Question Browser Exam Builder Saved Exams
» MRCP 1 Session Progress
• Question Browser
Questions Correct 3
• Timed Test A patient with poorly diet-controlled type-2 diabetes
mellitus needs to be started on medication. Which Questions Incorrect 0
• Mock Exam
concomitant condition would be a contraindication Questions Total 3
• Past Papers for starting metformin?
Questions Percentage 100 %
• Random Questions
• My Performance Hypertension More
• Media Bank Respiratory insufficiency

• New Multimedia Hyperlipidaemia

Online Extras Hyperthyroidism


Reference: Normal Values
Library Adipositas
Click to
Community 2618 open/closeHaematology
Help Click to
open/closeImmunoglobulins
PasTest Store
Click to
open/closeBiochemistry
My Account Click to open/closeDiabetes
Click to
Profile open/closeEndocrinology
Click to open/closeBlood
Newsletters gases Click to
My Career and Exams open/closeCSF
Order History
Learning Goals
Question Filters

Security

Change Password
Sign Out

© 2011 PasTest Ltd | About Us | Contact Us | Help

file:///E|/Shakhawan/Endocrinology/189.htm[3/13/2012 7:10:27 PM]


MyPasTest

Main Navigation
Question Browser: MRCP 1
Home
Subscriptions Question Browser Exam Builder Saved Exams
» MRCP 1 Session Progress
• Question Browser
Questions Correct 4
• Timed Test A patient with poorly diet-controlled type-2 diabetes
mellitus needs to be started on medication. Which Questions Incorrect 0
• Mock Exam
concomitant condition would be a contraindication Questions Total 4
• Past Papers for starting metformin?
Questions Percentage 100 %
• Random Questions
• My Performance Hypertension More
• Media Bank Respiratory insufficiency Your answer

• New Multimedia Hyperlipidaemia

Online Extras Hyperthyroidism

Library Adipositas

Community
Help
PasTest Store
Lactic acidosis is best known in diabetic patients as a
rare, but often fatal, complication of the biguanide
My Account metformin, which acts mainly by inhibiting hepatic
gluconeogenesis. The risk is much increased in patients
Profile with respiratory insufficiency.
Newsletters
My Career and Exams 2618
Order History Reference: Normal Values
Learning Goals
Click to open/closeHaematology
Question Filters Click to open/closeImmunoglobulins
Click to open/closeBiochemistry
Click to open/closeDiabetes Click to
Security open/closeEndocrinology Click to
open/closeBlood gases Click to
Change Password open/closeCSF
Sign Out

© 2011 PasTest Ltd | About Us | Contact Us | Help

file:///E|/Shakhawan/Endocrinology/189a.htm[3/13/2012 7:10:28 PM]


MyPasTest

Main Navigation
Question Browser: MRCP 1
Home
Subscriptions Question Browser Exam Builder Saved Exams
» MRCP 1 Session Progress
• Question Browser
Questions Correct 4
• Timed Test A patient with small-cell carcinoma of the lung has been
complaining of weakness in his leg muscles, weight gain Questions Incorrect 0
• Mock Exam
and darkening of his skin. What is the most likely Questions Total 4
• Past Papers diagnosis?
Questions Percentage 100 %
• Random Questions
• My Performance Adrenocortical tumour More
• Media Bank Radiation side-effects

• New Multimedia Ectopic ACTH secretion

Online Extras Diabetes mellitus


Reference: Normal Values
Library Side-effects from steroids
Click to
Community 2619 open/closeHaematology
Help Click to
open/closeImmunoglobulins
PasTest Store
Click to
open/closeBiochemistry
My Account Click to open/closeDiabetes
Click to
Profile open/closeEndocrinology
Click to open/closeBlood
Newsletters gases Click to
My Career and Exams open/closeCSF
Order History
Learning Goals
Question Filters

Security

Change Password
Sign Out

© 2011 PasTest Ltd | About Us | Contact Us | Help

file:///E|/Shakhawan/Endocrinology/190.htm[3/13/2012 7:10:30 PM]


MyPasTest

Main Navigation
Question Browser: MRCP 1
Home
Subscriptions Question Browser Exam Builder Saved Exams
» MRCP 1 Session Progress
• Question Browser
Questions Correct 5
• Timed Test A patient with small-cell carcinoma of the lung has been
complaining of weakness in his leg muscles, weight gain Questions Incorrect 0
• Mock Exam
and darkening of his skin. What is the most likely Questions Total 5
• Past Papers diagnosis?
Questions Percentage 100 %
• Random Questions
• My Performance Adrenocortical tumour More
• Media Bank Radiation side-effects

• New Multimedia Ectopic ACTH secretion Your answer

Online Extras Diabetes mellitus

Library Side-effects from steroids

Community
Help
PasTest Store
Ectopic ACTH syndrome, which accounts for about 15–
20% of patients with Cushing's syndrome, occurs two to
My Account three times more frequently in male patients. Small-cell
lung carcinoma accounts for about 50–75% of all cases
Profile of ectopic ACTH secretion.
Newsletters
My Career and Exams 2619
Order History Reference: Normal Values
Learning Goals
Click to open/closeHaematology
Question Filters Click to open/closeImmunoglobulins
Click to open/closeBiochemistry
Click to open/closeDiabetes Click to
Security open/closeEndocrinology Click to
open/closeBlood gases Click to
Change Password open/closeCSF
Sign Out

© 2011 PasTest Ltd | About Us | Contact Us | Help

file:///E|/Shakhawan/Endocrinology/190a.htm[3/13/2012 7:10:31 PM]


MyPasTest

Main Navigation
Question Browser: MRCP 1
Home
Subscriptions Question Browser Exam Builder Saved Exams
» MRCP 1 Session Progress
• Question Browser
Questions Correct 5
• Timed Test An overweight patient with type-2 diet-controlled diabetes
is seen in the outpatient clinic. Despite referral and Questions Incorrect 0
• Mock Exam
appropriate diet and increased exercise his blood glucose is Questions Total 5
• Past Papers not well controlled. What would be the first-line
Questions Percentage 100 %
• Random Questions treatment?
• My Performance More
Pioglitazone
• Media Bank
Short-acting insulin
• New Multimedia
Long-acting insulin
Online Extras Reference: Normal Values
Metformin
Library
Sulphonylurea Click to
Community open/closeHaematology
Help 2620 Click to
open/closeImmunoglobulins
PasTest Store
Click to
open/closeBiochemistry
My Account Click to open/closeDiabetes
Click to
Profile open/closeEndocrinology
Click to open/closeBlood
Newsletters gases Click to
My Career and Exams open/closeCSF
Order History
Learning Goals
Question Filters

Security

Change Password
Sign Out

© 2011 PasTest Ltd | About Us | Contact Us | Help

file:///E|/Shakhawan/Endocrinology/191.htm[3/13/2012 7:10:33 PM]


MyPasTest

Main Navigation
Question Browser: MRCP 1
Home
Subscriptions Question Browser Exam Builder Saved Exams
» MRCP 1 Session Progress
• Question Browser
Questions Correct 6
• Timed Test An overweight patient with type-2 diet-controlled diabetes
is seen in the outpatient clinic. Despite referral and Questions Incorrect 0
• Mock Exam
appropriate diet and increased exercise his blood glucose is Questions Total 6
• Past Papers not well controlled. What would be the first-line
Questions Percentage 100 %
• Random Questions treatment?
• My Performance More
Pioglitazone
• Media Bank
Short-acting insulin
• New Multimedia
Long-acting insulin
Online Extras
Metformin Your answer
Library
Sulphonylurea
Community
Help
PasTest Store
Metformin is used as a first-line treatment of type-2
My Account diabetes, particularly in overweight people, when diet
and exercise have failed to control blood sugar levels. It
Profile can also be used in combination with other antidiabetic
medicines to provide better control of blood sugar.
Newsletters
My Career and Exams
Order History 2620 Reference: Normal Values
Learning Goals
Click to open/closeHaematology
Question Filters Click to open/closeImmunoglobulins
Click to open/closeBiochemistry
Click to open/closeDiabetes Click to
Security open/closeEndocrinology Click to
open/closeBlood gases Click to
Change Password open/closeCSF
Sign Out

© 2011 PasTest Ltd | About Us | Contact Us | Help

file:///E|/Shakhawan/Endocrinology/191a.htm[3/13/2012 7:10:34 PM]


MyPasTest

Main Navigation
Question Browser: MRCP 1
Home
Subscriptions Question Browser Exam Builder Saved Exams
» MRCP 1
• Question Browser
• Timed Test A 20-year-old patient presents with large-volume watery diarrhoea which has
• Mock Exam gradually increased over the last 2 weeks. He has no other complaints and there has
been no recent travel abroad. On examination he looks dehydrated. His blood results
• Past Papers show a hypokalaemic acidosis. Which of the following is the most likely cause for
• Random Questions his symptoms?
• My Performance
AIDS
• Media Bank
VIPoma
• New Multimedia
Diabetes mellitus
Online Extras
Phaeochromocytoma
Library
Giardia lamblia infection
Community
Help 2621
PasTest Store

My Account

Profile
Newsletters
My Career and Exams
Order History
Learning Goals
Question Filters

Security

Change Password
Sign Out

file:///E|/Shakhawan/Endocrinology/192.htm[3/13/2012 7:10:35 PM]


MyPasTest

Main Navigation
Question Browser: MRCP 1
Home
Subscriptions Question Browser Exam Builder Saved Exams
» MRCP 1 Session Progress
• Question Browser
Questions Correct 7
• Timed Test A 20-year-old patient presents with large-volume watery
diarrhoea which has gradually increased over the last 2 Questions Incorrect 0
• Mock Exam
weeks. He has no other complaints and there has been no Questions Total 7
• Past Papers recent travel abroad. On examination he looks dehydrated.
Questions Percentage 100 %
• Random Questions His blood results show a hypokalaemic acidosis. Which of
the following is the most likely cause for his More
• My Performance
symptoms?
• Media Bank
• New Multimedia AIDS
Online Extras VIPoma Your answer
Library Diabetes mellitus
Community Phaeochromocytoma
Help Giardia lamblia infection
PasTest Store

My Account
VIPomas arise in the pancreas in 90% of cases. The
Profile remaining tumours are mainly gangliomas or
Newsletters ganglioneuroblastomas originating in the sympathetic
chain or adrenal medulla, and these tumours are
My Career and Exams especially common in children. Most extrapancreatic
Order History tumours are benign, but 50% of pancreatic VIPomas Reference: Normal Values
have metastasised at the time of diagnosis, usually to
Learning Goals
local lymph nodes and the liver. Click to open/closeHaematology
Question Filters Click to open/closeImmunoglobulins
The features of the VIPoma (aka Verner–Morrison,
Click to open/closeBiochemistry
pancreatic cholera, WHDA (watery diarrhoea,
Click to open/closeDiabetes Click to
Security hypokalaemia, and achlorhydria)) syndrome reflect
open/closeEndocrinology Click to
the known biological actions of vasoactive intestinal
open/closeBlood gases Click to
Change Password peptide (VIP). Large-volume diarrhoea without
open/closeCSF
steatorrhoea is the main symptom; most patients
Sign Out
excrete more than 3 litres daily, with volumes of over
20 litres having been described. It is often
intermittent at first, but in severe crises the volume
loss coupled with the vasodilatory effects of VIP and
the associated hypokalaemia may precipitate
cardiovascular collapse. Hypokalaemia results from
loss in stools and activation of the renin–angiotensin
system, and may be profound. The loss of bicarbonate
in the stool leads to acidosis, which may mask the
true potassium deficit.

2621

© 2011 PasTest Ltd | About Us | Contact Us | Help

file:///E|/Shakhawan/Endocrinology/192a.htm[3/13/2012 7:10:37 PM]


MyPasTest

Main Navigation
Question Browser: MRCP 1
Home
Subscriptions Question Browser Exam Builder Saved Exams
» MRCP 1 Session Progress
• Question Browser
Questions Correct 7
• Timed Test A patient has been diagnosed with hyperparathyroidism.
Her raised prolactin levels were also recently diagnosed as Questions Incorrect 0
• Mock Exam
being due to a prolactinoma. Endocrine tumours in Questions Total 7
• Past Papers which organ would you additionally expect?
Questions Percentage 100 %
• Random Questions
• My Performance Lungs More
• Media Bank Muscle

• New Multimedia Spinal cord

Online Extras Pancreas


Reference: Normal Values
Library Liver
Click to
Community 2622 open/closeHaematology
Help Click to
open/closeImmunoglobulins
PasTest Store
Click to
open/closeBiochemistry
My Account Click to open/closeDiabetes
Click to
Profile open/closeEndocrinology
Click to open/closeBlood
Newsletters gases Click to
My Career and Exams open/closeCSF
Order History
Learning Goals
Question Filters

Security

Change Password
Sign Out

© 2011 PasTest Ltd | About Us | Contact Us | Help

file:///E|/Shakhawan/Endocrinology/193.htm[3/13/2012 7:10:38 PM]


MyPasTest

Main Navigation
Question Browser: MRCP 1
Home
Subscriptions Question Browser Exam Builder Saved Exams
» MRCP 1 Session Progress
• Question Browser
Questions Correct 8
• Timed Test A patient has been diagnosed with hyperparathyroidism.
Her raised prolactin levels were also recently diagnosed as Questions Incorrect 0
• Mock Exam
being due to a prolactinoma. Endocrine tumours in Questions Total 8
• Past Papers which organ would you additionally expect?
Questions Percentage 100 %
• Random Questions
• My Performance Lungs More
• Media Bank Muscle

• New Multimedia Spinal cord

Online Extras Pancreas Your answer

Library Liver

Community
Help
PasTest Store
Multiple endocrine neoplasia-1 (MEN-1) is characterised
by the association of parathyroid hyperplasia, pancreatic
My Account endocrine tumours and pituitary adenomas. It has an
autosomal-dominant mode of inheritance. The
Profile prevalence of the condition has been estimated at about
1 in 10,000. The affected gene has been termed ‘menin’
Newsletters
and is found on the 11q13 region of the long arm of
My Career and Exams chromosome 11.
Order History Reference: Normal Values
Learning Goals 2622 Click to open/closeHaematology
Question Filters Click to open/closeImmunoglobulins
Click to open/closeBiochemistry
Click to open/closeDiabetes Click to
Security open/closeEndocrinology Click to
open/closeBlood gases Click to
Change Password open/closeCSF
Sign Out

© 2011 PasTest Ltd | About Us | Contact Us | Help

file:///E|/Shakhawan/Endocrinology/193a.htm[3/13/2012 7:10:40 PM]


MyPasTest

Main Navigation
Question Browser: MRCP 1
Home
Subscriptions Question Browser Exam Builder Saved Exams
» MRCP 1 Session Progress
• Question Browser
Questions Correct 8
• Timed Test A 46-year-old magazine columnist is referred by her
general practitioner (GP) complaining of tiredness and Questions Incorrect 0
• Mock Exam
lethargy. There has been no weight loss. The GP found her Questions Total 8
• Past Papers serum sodium to be 128 mmol/l (137–144) and 130 mmol/l
Questions Percentage 100 %
• Random Questions on two occasions (with a normal creatinine and normal
glucose concentrations), and ordered a chest X-ray. This More
• My Performance
was reported as showing a small area of increased
• Media Bank radiodensity in the right lung apex. Paired plasma and urine
• New Multimedia osmolalities are 255 mosmol/kg (285–295) and 148
mosmol/kg (50–1200) respectively. Which is the most
Online Extras likely diagnosis? Reference: Normal Values
Library
A tumour of the posterior pituitary Click to
Community open/closeHaematology
Diabetes insipidus
Help Click to
Ectopic antidiuretic hormone (ADH) hypersecretion open/closeImmunoglobulins
PasTest Store
Hysterical polydipsia Click to
open/closeBiochemistry
Pseudohyponatraemia Click to open/closeDiabetes
My Account
Click to
Profile 3191 open/closeEndocrinology
Click to open/closeBlood
Newsletters gases Click to
My Career and Exams open/closeCSF
Order History
Learning Goals
Question Filters

Security

Change Password
Sign Out

© 2011 PasTest Ltd | About Us | Contact Us | Help

file:///E|/Shakhawan/Endocrinology/194.htm[3/13/2012 7:10:41 PM]


MyPasTest

Main Navigation
Question Browser: MRCP 1
Home
Subscriptions Question Browser Exam Builder Saved Exams
» MRCP 1 Session Progress
• Question Browser
Questions Correct 1
• Timed Test A 46-year-old magazine columnist is referred by her
general practitioner (GP) complaining of tiredness and Questions Incorrect 0
• Mock Exam
lethargy. There has been no weight loss. The GP found her Questions Total 1
• Past Papers serum sodium to be 128 mmol/l (137–144) and 130 mmol/l
Questions Percentage 100 %
• Random Questions on two occasions (with a normal creatinine and normal
glucose concentrations), and ordered a chest X-ray. This More
• My Performance
was reported as showing a small area of increased
• Media Bank radiodensity in the right lung apex. Paired plasma and urine
• New Multimedia osmolalities are 255 mosmol/kg (285–295) and 148
mosmol/kg (50–1200) respectively. Which is the most
Online Extras likely diagnosis?
Library
A tumour of the posterior pituitary
Community
Diabetes insipidus
Help
Ectopic antidiuretic hormone (ADH)
PasTest Store
hypersecretion
Hysterical polydipsia Your answer
My Account Pseudohyponatraemia
Profile
Newsletters
My Career and Exams
Order History Hyperlipidaemia can cause pseudohyponatraemia when Reference: Normal Values
sodium is measured by flame photometry (since the
Learning Goals lipid occupies a significant volume, but the sodium is Click to open/closeHaematology
Question Filters only present in the water phase). Plasma osmolality Click to open/closeImmunoglobulins
(normally 285–295 mosmol/kg) and vasopressin Click to open/closeBiochemistry
secretion are both unaltered in pseudohyponatraemia. Click to open/closeDiabetes Click to
Security In inappropriate ADH syndrome the urine osmolality is open/closeEndocrinology Click to
inappropriately raised in the face of low plasma open/closeBlood gases Click to
Change Password osmolality. In diabetes insipidus the plasma osmolality open/closeCSF
Sign Out is raised and the urine osmolality is inappropriately low.
A tumour of the posterior pituitary is exceedingly rare.
Hypothalamic tumours can either cause ADH excess or
diabetes insipidus with the features mentioned above.
The diagnosis that is most compatible with the
biochemical results is hysterical polydipsia. This is
associated with low plasma and urine osmolalities due
to a very large throughput of water. The chest x-ray
changes are an incidental finding.

3191

© 2011 PasTest Ltd | About Us | Contact Us | Help

file:///E|/Shakhawan/Endocrinology/194a.htm[3/13/2012 7:10:43 PM]


MyPasTest

Main Navigation
Question Browser: MRCP 1
Home
Subscriptions Question Browser Exam Builder Saved Exams
» MRCP 1 Session Progress
• Question Browser
Questions Correct 1
• Timed Test A 28-year-old single machinist, who describes herself as
always being anxious, presents concerned about weight loss Questions Incorrect 0
• Mock Exam
(59 to 50 kg), tiredness and amenorrhoea over the past Questions Total 1
• Past Papers year. Examination reveals: pallor, pulse 96/min regular
Questions Percentage 100 %
• Random Questions lying, blood pressure 110/78 mmHg lying and 92/66mmHg
standing, tendon reflexes slightly slow relaxing. There is a More
• My Performance
normochromic, normocytic anaemia (haemoglobin (Hb) 9.9
• Media Bank g/dl). Urea and electrolytes, liver function tests, plasma
• New Multimedia glucose and serum calcium are normal. Which of the
following is the most likely diagnosis?
Online Extras Reference: Normal Values
Library Microprolactinoma
Click to
Community Hypothyroidism
open/closeHaematology
Help Anorexia nervosa Click to
Hypopituitarism open/closeImmunoglobulins
PasTest Store
Click to
Thyrotoxicosis open/closeBiochemistry
My Account Click to open/closeDiabetes
3192 Click to
Profile open/closeEndocrinology
Click to open/closeBlood
Newsletters gases Click to
My Career and Exams open/closeCSF
Order History
Learning Goals
Question Filters

Security

Change Password
Sign Out

© 2011 PasTest Ltd | About Us | Contact Us | Help

file:///E|/Shakhawan/Endocrinology/195.htm[3/13/2012 7:10:44 PM]


MyPasTest

Main Navigation
Question Browser: MRCP 1
Home
Subscriptions Question Browser Exam Builder Saved Exams
» MRCP 1 Session Progress
• Question Browser
Questions Correct 1
• Timed Test A 28-year-old single machinist, who describes herself as
always being anxious, presents concerned about weight loss Questions Incorrect 0
• Mock Exam
(59 to 50 kg), tiredness and amenorrhoea over the past Questions Total 1
• Past Papers year. Examination reveals: pallor, pulse 96/min regular
Questions Percentage 100 %
• Random Questions lying, blood pressure 110/78 mmHg lying and 92/66mmHg
standing, tendon reflexes slightly slow relaxing. There is a More
• My Performance
normochromic, normocytic anaemia (haemoglobin (Hb) 9.9
• Media Bank g/dl). Urea and electrolytes, liver function tests, plasma
• New Multimedia glucose and serum calcium are normal. Which of the
following is the most likely diagnosis?
Online Extras
Library Microprolactinoma
Community Hypothyroidism
Help Anorexia nervosa
PasTest Store Hypopituitarism Your answer
Thyrotoxicosis
My Account

Profile
Newsletters
Hypopituitarism produces pallor (due to loss of the
My Career and Exams melanocyte-stimulating properties of
Order History adrenocorticotrophic hormone (ACTH)), and a moderate Reference: Normal Values
normochromic, normocytic anaemia which responds to
Learning Goals thyroid and steroid replacement. Postural hypotension Click to open/closeHaematology
Question Filters occurs due to adrenocortical underactivity, tiredness Click to open/closeImmunoglobulins
and slow reacting reflexes due to hypothyroidism and Click to open/closeBiochemistry
amenorrhoea due to loss of gonadotrophins. Most Click to open/closeDiabetes Click to
Security patients with microprolactinoma are well, apart from open/closeEndocrinology Click to
amenorrhoea and galactorrhea. Although weight loss, open/closeBlood gases Click to
Change Password and a normochromic anaemia can occasionally be found open/closeCSF
Sign Out in hypothyroidism, weight gain, menorrhagia and
macrocytic anaemia are more common (unless
menorrhagia is very severe, in which case an iron-
deficient microcytic picture might be seen). The
presence of slow reacting reflexes and amenorrhoea,
and menorrhagia and a macrocytic anaemia are more
common. The presence of slow relaxing reflexes and
amenorrhoea (rather than oligomenorrhea) is
uncommon in thyrotoxicosis. It is unusual for anorexia
nervosia patients to present concerned about their
health. Others often initiate the call for help. There are
too few features of anorexia, and too many
hypopituitary features to make anorexia the most likely
diagnosis.

3192

© 2011 PasTest Ltd | About Us | Contact Us | Help

file:///E|/Shakhawan/Endocrinology/195a.htm[3/13/2012 7:10:46 PM]


MyPasTest

Main Navigation
Question Browser: MRCP 1
Home
Subscriptions Question Browser Exam Builder Saved Exams
» MRCP 1 Session Progress
• Question Browser
Questions Correct 1
• Timed Test A 28-year-old machinist presents to her general
practitioner (GP) with weight loss despite a good appetite, Questions Incorrect 0
• Mock Exam
palpitations and excess sweating. The GP suspects Questions Total 1
• Past Papers thyrotoxicosis and confirms this by finding a free tetra-
Questions Percentage 100 %
• Random Questions iodothyronine (T4 ) of 58 pmol/l (10–22) with an
• My Performance undetectable thyroid-stimulating hormone (TSH). On More
examination she has a normal sized thyroid gland with a
• Media Bank suggestion of bulkiness on the right hand side. A
• New Multimedia technetium uptake scan reveals a single ‘hot’ nodule in the
right lobe of the thyroid with suppression of activity
Online Extras Reference: Normal Values
elsewhere. Which of the following clinical features
Library would she be most likely to have?
Click to
Community open/closeHaematology
Diplopia
Help Click to
Lid retraction and lid lag open/closeImmunoglobulins
PasTest Store
Periorbital oedema Click to
open/closeBiochemistry
Presence of thyroid-stimulating immunoglobulins Click to open/closeDiabetes
My Account (TSI) Click to
Profile Unilateral exophthalmos open/closeEndocrinology
Click to open/closeBlood
Newsletters gases Click to
3193
My Career and Exams open/closeCSF
Order History
Learning Goals
Question Filters

Security

Change Password
Sign Out

© 2011 PasTest Ltd | About Us | Contact Us | Help

file:///E|/Shakhawan/Endocrinology/196.htm[3/13/2012 7:10:47 PM]


MyPasTest

Main Navigation
Question Browser: MRCP 1
Home
Subscriptions Question Browser Exam Builder Saved Exams
» MRCP 1 Session Progress
• Question Browser
Questions Correct 2
• Timed Test A 28-year-old machinist presents to her general
practitioner (GP) with weight loss despite a good appetite, Questions Incorrect 0
• Mock Exam
palpitations and excess sweating. The GP suspects Questions Total 2
• Past Papers thyrotoxicosis and confirms this by finding a free tetra-
Questions Percentage 100 %
• Random Questions iodothyronine (T4 ) of 58 pmol/l (10–22) with an
• My Performance undetectable thyroid-stimulating hormone (TSH). On More
examination she has a normal sized thyroid gland with a
• Media Bank suggestion of bulkiness on the right hand side. A
• New Multimedia technetium uptake scan reveals a single ‘hot’ nodule in the
right lobe of the thyroid with suppression of activity
Online Extras
elsewhere. Which of the following clinical features
Library would she be most likely to have?
Community
Diplopia
Help
Lid retraction and lid lag Your answer
PasTest Store
Periorbital oedema
Presence of thyroid-stimulating
My Account immunoglobulins (TSI)
Profile Unilateral exophthalmos
Newsletters
My Career and Exams
Order History Reference: Normal Values
Lid retraction and lag are signs of sympathetic
Learning Goals
overactivity, and occur in any thyrotoxic state Click to open/closeHaematology
Question Filters (thyroxine potentiates the action of catecholamines). Click to open/closeImmunoglobulins
The other symptoms are features of thyroid eye disease Click to open/closeBiochemistry
(which may be unilateral). They are characteristic of, Click to open/closeDiabetes Click to
Security and restricted to, Graves' disease, which is due to open/closeEndocrinology Click to
stimulation of TSH receptors in the thyroid by specific open/closeBlood gases Click to
Change Password immunoglobulin C (IgC) autoantibodies (TSI). open/closeCSF
Sign Out

3193

© 2011 PasTest Ltd | About Us | Contact Us | Help

file:///E|/Shakhawan/Endocrinology/196a.htm[3/13/2012 7:10:49 PM]


MyPasTest

Main Navigation
Question Browser: MRCP 1
Home
Subscriptions Question Browser Exam Builder Saved Exams
» MRCP 1 Session Progress
• Question Browser
Questions Correct 2
• Timed Test An apparently healthy, 20-year-old man has blood taken
for lipid analysis in connection with an application for life Questions Incorrect 0
• Mock Exam
insurance. His body mass index (BMI) is 22 kg/m2 . The Questions Total 2
• Past Papers
fasting triglyceride concentration is 4.0 mmol/l. Which of Questions Percentage 100 %
• Random Questions the following is the most likely cause of the
• My Performance hypertriglyceridaemia? More
• Media Bank
Undiagnosed diabetes mellitus
• New Multimedia
World Health Organization (WHO) type 1
Online Extras hyperlipidaemia (chylomicronaemia syndrome) Reference: Normal Values
Library A high alcohol intake
Click to
Community A high saturated fat intake open/closeHaematology
Help Non-alcoholic steatohepatitis (NASH) Click to
open/closeImmunoglobulins
PasTest Store
3224 Click to
open/closeBiochemistry
My Account Click to open/closeDiabetes
Click to
Profile open/closeEndocrinology
Click to open/closeBlood
Newsletters gases Click to
My Career and Exams open/closeCSF
Order History
Learning Goals
Question Filters

Security

Change Password
Sign Out

© 2011 PasTest Ltd | About Us | Contact Us | Help

file:///E|/Shakhawan/Endocrinology/197.htm[3/13/2012 7:10:50 PM]


MyPasTest

Main Navigation
Question Browser: MRCP 1
Home
Subscriptions Question Browser Exam Builder Saved Exams
» MRCP 1 Session Progress
• Question Browser
Questions Correct 2
• Timed Test An apparently healthy, 20-year-old man has blood taken
for lipid analysis in connection with an application for life Questions Incorrect 1
• Mock Exam
insurance. His body mass index (BMI) is 22 kg/m2 . The Questions Total 3
• Past Papers
fasting triglyceride concentration is 4.0 mmol/l. Which of Questions Percentage 66 %
• Random Questions the following is the most likely cause of the
• My Performance hypertriglyceridaemia? More
• Media Bank
Undiagnosed diabetes mellitus
• New Multimedia
World Health Organization (WHO)
Online Extras type 1 hyperlipidaemia
Library (chylomicronaemia syndrome)

Community A high alcohol intake Correct answer

Help A high saturated fat intake

PasTest Store Non-alcoholic steatohepatitis Your answer


(NASH)

My Account

Profile
Newsletters Alcohol consumption is a common cause of
hypertriglyceridaemia. Diabetes at this age would be
My Career and Exams most likely to be type 1 (especially given his normal
Order History body mass index) and to present acutely. Reference: Normal Values
Chylomicronaemia syndrome is rare, usually presents in
Learning Goals
childhood, and is associated with higher triglyceride Click to open/closeHaematology
Question Filters concentrations. A high saturated fat intake is more Click to open/closeImmunoglobulins
likely to elevate plasma cholesterol than triglyceride Click to open/closeBiochemistry
concentrations. Hypertriglyceridaemia is often present in Click to open/closeDiabetes Click to
Security NASH, but this condition is typically (though not always) open/closeEndocrinology Click to
associated with overweight. open/closeBlood gases Click to
Change Password open/closeCSF
Sign Out
3224

© 2011 PasTest Ltd | About Us | Contact Us | Help

file:///E|/Shakhawan/Endocrinology/197a.htm[3/13/2012 7:10:51 PM]


MyPasTest

Main Navigation
Question Browser: MRCP 1
Home
Subscriptions Question Browser Exam Builder Saved Exams
» MRCP 1 Session Progress
• Question Browser
Questions Correct 2
• Timed Test An 82-year-old Caucasian woman has short history of bone
pain. Serum biochemistry reveals plasma calcium Questions Incorrect 1
• Mock Exam
concentration 2.14 mmol/l, phosphate 0.70 mmol/l, Questions Total 3
• Past Papers alkaline phosphatase activity twice upper limit of normal.
Questions Percentage 66 %
• Random Questions The concentration of parathyroid hormone is elevated. The
most likely diagnosis is? More
• My Performance
• Media Bank Osteomalacia
• New Multimedia Osteolytic metastasis
Online Extras Osteoporosis Reference: Normal Values
Library Renal osteodystrophy
Click to
Community Primary hyperparathyroidism open/closeHaematology
Help Click to
3225 open/closeImmunoglobulins
PasTest Store
Click to
open/closeBiochemistry
My Account Click to open/closeDiabetes
Click to
Profile open/closeEndocrinology
Click to open/closeBlood
Newsletters gases Click to
My Career and Exams open/closeCSF
Order History
Learning Goals
Question Filters

Security

Change Password
Sign Out

© 2011 PasTest Ltd | About Us | Contact Us | Help

file:///E|/Shakhawan/Endocrinology/198.htm[3/13/2012 7:10:53 PM]


MyPasTest

Main Navigation
Question Browser: MRCP 1
Home
Subscriptions Question Browser Exam Builder Saved Exams
» MRCP 1 Session Progress
• Question Browser
Questions Correct 3
• Timed Test An 82-year-old Caucasian woman has short history of bone
pain. Serum biochemistry reveals plasma calcium Questions Incorrect 1
• Mock Exam
concentration 2.14 mmol/l, phosphate 0.70 mmol/l, Questions Total 4
• Past Papers alkaline phosphatase activity twice upper limit of normal.
Questions Percentage 75 %
• Random Questions The concentration of parathyroid hormone is elevated. The
most likely diagnosis is? More
• My Performance
• Media Bank Osteomalacia Your answer
• New Multimedia Osteolytic metastasis
Online Extras Osteoporosis
Library Renal osteodystrophy
Community Primary hyperparathyroidism
Help
PasTest Store

My Account Plasma alkaline phosphatase activity may be increased


with osteolytic secondaries but calcium concentration is
Profile usually normal or may be elevated due to secretion of
parathyroid hormone-related peptide. Calcium,
Newsletters
phospate and alkaline phosphatase are all are normal in
My Career and Exams uncomplicated osteoporosis. Plasma phosphate
Order History concentration is increased in renal osteodystrophy, Reference: Normal Values
because of the underlying renal failure. Although
Learning Goals hypophosphataemia is characteristic of primary Click to open/closeHaematology
Question Filters hyperparathyroidism, plasma calcium concentration is Click to open/closeImmunoglobulins
usually elevated (an exception is when there is Click to open/closeBiochemistry
concomitant vitamin D deficiency). Osteomalacia is most Click to open/closeDiabetes Click to
Security frequently related to lack or impaired metabolism of open/closeEndocrinology Click to
vitamin D: hypocalcaemia may not be striking, because open/closeBlood gases Click to
Change Password of increased parathyroid hormone secretion (secondary open/closeCSF
Sign Out hyperparathyroidism), which also increases renal
phosphate excretion; the elevated alkaline phosphatase
reflects increased osteoblastic activity.

3225

© 2011 PasTest Ltd | About Us | Contact Us | Help

file:///E|/Shakhawan/Endocrinology/198a.htm[3/13/2012 7:10:55 PM]


MyPasTest

Main Navigation
Question Browser: MRCP 1
Home
Subscriptions Question Browser Exam Builder Saved Exams
» MRCP 1 Session Progress
• Question Browser
Questions Correct 3
• Timed Test Which ONE of the following findings in an elderly
man with pain in the back and known prostatic Questions Incorrect 1
• Mock Exam
carcinoma suggests a diagnosis of Paget’s disease Questions Total 4
• Past Papers rather than metastatic disease?
Questions Percentage 75 %
• Random Questions
• My Performance Mixed sclerotic and lytic lesions on X-ray More
• Media Bank Serum alkaline phosphatase activity five times the
upper limit of normal
• New Multimedia
Hypercalcaemia
Online Extras Reference: Normal Values
Serum prostate-specific antigen (PSA) 8 ng/l
Library
Increased serum concentration of parathyroid Click to
Community hormone open/closeHaematology
Help Click to
3226 open/closeImmunoglobulins
PasTest Store
Click to
open/closeBiochemistry
My Account Click to open/closeDiabetes
Click to
Profile open/closeEndocrinology
Click to open/closeBlood
Newsletters gases Click to
My Career and Exams open/closeCSF
Order History
Learning Goals
Question Filters

Security

Change Password
Sign Out

© 2011 PasTest Ltd | About Us | Contact Us | Help

file:///E|/Shakhawan/Endocrinology/199.htm[3/13/2012 7:10:56 PM]


MyPasTest

Main Navigation
Question Browser: MRCP 1
Home
Subscriptions Question Browser Exam Builder Saved Exams
» MRCP 1 Session Progress
• Question Browser
Questions Correct 3
• Timed Test Which ONE of the following findings in an elderly
man with pain in the back and known prostatic Questions Incorrect 2
• Mock Exam
carcinoma suggests a diagnosis of Paget’s disease Questions Total 5
• Past Papers rather than metastatic disease?
Questions Percentage 60 %
• Random Questions
• My Performance Mixed sclerotic and lytic lesions on More
X-ray
• Media Bank
Serum alkaline phosphatase activity Correct answer
• New Multimedia five times the upper limit of normal
Online Extras Hypercalcaemia
Library Serum prostate-specific antigen Your answer
Community (PSA) 8 ng/l
Help Increased serum concentration of
parathyroid hormone
PasTest Store

My Account

Profile The radiographic appearances of metastatic prostate


cancer and Paget’s disease can be similar.
Newsletters
Hypercalcaemia can occur in both conditions (in Paget’s
My Career and Exams usually only with immobilisation) but is uncommon.
Order History Serum parathyroid hormone concentration is not Reference: Normal Values
directly affected in either condition. This slightly
Learning Goals elevated PSA would be unusual for a patient with Click to open/closeHaematology
Question Filters metastases (though it does not exclude the diagnosis). Click to open/closeImmunoglobulins
Such a high alkaline phosphatase would be more typical Click to open/closeBiochemistry
of Paget’s than cancer. Click to open/closeDiabetes Click to
Security open/closeEndocrinology Click to
open/closeBlood gases Click to
Change Password 3226 open/closeCSF
Sign Out

© 2011 PasTest Ltd | About Us | Contact Us | Help

file:///E|/Shakhawan/Endocrinology/199a.htm[3/13/2012 7:10:57 PM]


MyPasTest

Main Navigation
Question Browser: MRCP 1
Home
Subscriptions Question Browser Exam Builder Saved Exams
» MRCP 1 Session Progress
• Question Browser
Questions Correct 3
• Timed Test A 45-year-old man presents with episodes of tremor,
sweating and inco-ordination particularly when he first Questions Incorrect 2
• Mock Exam
wakes in the morning. What would be the most useful Questions Total 5
• Past Papers investigation(s) to establish a diagnosis of an
Questions Percentage 60 %
• Random Questions insulinoma?
• My Performance More
An insulin hypoglycaemia test (insulin stress test)
• Media Bank
An oral glucose tolerance test
• New Multimedia
Measurement of glycated haemoglobin (HbA 1c )
Online Extras Reference: Normal Values
Measurement of serum 3-hydroxybutyrate
Library
Simultaneous measurement of fasting blood glucose, Click to
Community and serum insulin and C-peptide concentrations open/closeHaematology
Help Click to
3229 open/closeImmunoglobulins
PasTest Store
Click to
open/closeBiochemistry
My Account Click to open/closeDiabetes
Click to
Profile open/closeEndocrinology
Click to open/closeBlood
Newsletters gases Click to
My Career and Exams open/closeCSF
Order History
Learning Goals
Question Filters

Security

Change Password
Sign Out

© 2011 PasTest Ltd | About Us | Contact Us | Help

file:///E|/Shakhawan/Endocrinology/200.htm[3/13/2012 7:10:59 PM]


MyPasTest

Main Navigation
Question Browser: MRCP 1
Home
Subscriptions Question Browser Exam Builder Saved Exams
» MRCP 1 Session Progress
• Question Browser
Questions Correct 4
• Timed Test A 45-year-old man presents with episodes of tremor,
sweating and inco-ordination particularly when he first Questions Incorrect 2
• Mock Exam
wakes in the morning. What would be the most useful Questions Total 6
• Past Papers investigation(s) to establish a diagnosis of an
Questions Percentage 66 %
• Random Questions insulinoma?
• My Performance More
An insulin hypoglycaemia test (insulin
• Media Bank stress test)
• New Multimedia An oral glucose tolerance test
Online Extras Measurement of glycated haemoglobin
Library (HbA 1c )
Community Measurement of serum 3-
hydroxybutyrate
Help
Simultaneous measurement of fasting Your answer
PasTest Store
blood glucose, and serum insulin and C-
peptide concentrations
My Account

Profile
Newsletters
In health, serum insulin concentration falls to low values
My Career and Exams during fasting: the combination of fasting
Order History hypoglycaemia and hyperinsulinaemia suggests that Reference: Normal Values
insulin is causing the hypoglycaemia. Measurement of
Learning Goals
C-peptide is important to exclude a diagnosis of self- Click to open/closeHaematology
Question Filters administration of insulin: insulin for therapeutic use Click to open/closeImmunoglobulins
does not contain C-peptide, which is released from Click to open/closeBiochemistry
pancreatic islet cells during insulin secretion. Transient Click to open/closeDiabetes Click to
Security hypoglycaemia sometimes occurs during an oral glucose open/closeEndocrinology Click to
tolerance test in normal individuals but the test is not open/closeBlood gases Click to
Change Password useful for the investigation of hypoglycaemia. The open/closeCSF
Sign Out insulin hypoglycaemia test is used for testing
hypothalamo-pituitary function. Measurement of
glycated haemoglobin is used to monitor glycaemic
control in diabetes but is of no value in the diagnosis of
hypoglycaemia. Hyperinsulinism suppresses lipolysis,
therefore the induced hypoglycaemia is not associated
with a ketosis, as is usual. Measurement of 3-
hydroxybutyrate can be helpful to exclude
hyperinsulinism but is not diagnostic.

3229

© 2011 PasTest Ltd | About Us | Contact Us | Help

file:///E|/Shakhawan/Endocrinology/200a.htm[3/13/2012 7:11:00 PM]


MyPasTest

Main Navigation
Question Browser: MRCP 1
Home
Subscriptions Question Browser Exam Builder Saved Exams
» MRCP 1 Session Progress
• Question Browser
Questions Correct 4
• Timed Test A patient with type-1 diabetes mellitus has a deficiency of
insulin. Which cells secrete insulin? Questions Incorrect 2
• Mock Exam
Questions Total 6
• Past Papers
A cells of the islets of Langerhans Questions Percentage 66 %
• Random Questions
B cells of the pancreatic islets
• My Performance More
Hepatocytes
• Media Bank
Fat cells
• New Multimedia
Melanocytes
Online Extras Reference: Normal Values
Library 3281
Click to
Community open/closeHaematology
Help Click to
open/closeImmunoglobulins
PasTest Store
Click to
open/closeBiochemistry
My Account Click to open/closeDiabetes
Click to
Profile open/closeEndocrinology
Click to open/closeBlood
Newsletters gases Click to
My Career and Exams open/closeCSF
Order History
Learning Goals
Question Filters

Security

Change Password
Sign Out

© 2011 PasTest Ltd | About Us | Contact Us | Help

file:///E|/Shakhawan/Endocrinology/201.htm[3/13/2012 7:11:02 PM]


MyPasTest

Main Navigation
Question Browser: MRCP 1
Home
Subscriptions Question Browser Exam Builder Saved Exams
» MRCP 1 Session Progress
• Question Browser
Questions Correct 5
• Timed Test A patient with type-1 diabetes mellitus has a deficiency of
insulin. Which cells secrete insulin? Questions Incorrect 2
• Mock Exam
Questions Total 7
• Past Papers
A cells of the islets of Langerhans Questions Percentage 71 %
• Random Questions
B cells of the pancreatic islets Your answer
• My Performance More
Hepatocytes
• Media Bank
Fat cells
• New Multimedia
Melanocytes
Online Extras
Library
Community
Help Insulin is synthesised and stored by the B cells of the
PasTest Store pancreatic islets. The B cells make up 60–70% of the
volume of the islets. Glucagon is synthesised and stored
in the A cells (10–20% of the islets). Somatostatin is
My Account synthesised and stored in the D cells (5–10% of the
islets).
Profile
Newsletters
3281
My Career and Exams
Order History Reference: Normal Values
Learning Goals
Click to open/closeHaematology
Question Filters Click to open/closeImmunoglobulins
Click to open/closeBiochemistry
Click to open/closeDiabetes Click to
Security open/closeEndocrinology Click to
open/closeBlood gases Click to
Change Password open/closeCSF
Sign Out

© 2011 PasTest Ltd | About Us | Contact Us | Help

file:///E|/Shakhawan/Endocrinology/201a.htm[3/13/2012 7:11:03 PM]


MyPasTest

Main Navigation
Question Browser: MRCP 1
Home
Subscriptions Question Browser Exam Builder Saved Exams
» MRCP 1 Session Progress
• Question Browser
Questions Correct 5
• Timed Test A patient with impaired glucose tolerance has an increased
risk of developing frank diabetes. What is the five-year Questions Incorrect 2
• Mock Exam
progression risk? Questions Total 7
• Past Papers
Questions Percentage 71 %
• Random Questions 1–2%
• My Performance 5–10% More
• Media Bank 10–20%
• New Multimedia 20–30%
Online Extras 100% Reference: Normal Values
Library
3282 Click to
Community open/closeHaematology
Help Click to
open/closeImmunoglobulins
PasTest Store
Click to
open/closeBiochemistry
My Account Click to open/closeDiabetes
Click to
Profile open/closeEndocrinology
Click to open/closeBlood
Newsletters gases Click to
My Career and Exams open/closeCSF
Order History
Learning Goals
Question Filters

Security

Change Password
Sign Out

© 2011 PasTest Ltd | About Us | Contact Us | Help

file:///E|/Shakhawan/Endocrinology/202.htm[3/13/2012 7:11:05 PM]


MyPasTest

Main Navigation
Question Browser: MRCP 1
Home
Subscriptions Question Browser Exam Builder Saved Exams
» MRCP 1 Session Progress
• Question Browser
Questions Correct 6
• Timed Test A patient with impaired glucose tolerance has an increased
risk of developing frank diabetes. What is the five-year Questions Incorrect 2
• Mock Exam
progression risk? Questions Total 8
• Past Papers
Questions Percentage 75 %
• Random Questions 1–2%
• My Performance 5–10% More
• Media Bank 10–20%
• New Multimedia 20–30% Your answer
Online Extras 100%
Library
Community
Help
PasTest Store Impaired glucose tolerance (IGT) is defined by the
World Health Organization (WHO) criteria as being
present in individuals with a fasting plasma glucose of
My Account less than 7.0 mmol/l, and a plasma glucose of between
7.8 and 11.1 mmol/l when measured two hours after an
Profile oral 75 g glucose load. The risk of progression to frank
diabetes is 20–30% within five years. Half of patients
Newsletters
with IGT will have normal glucose tolerance on later
My Career and Exams follow-up testing.
Order History Reference: Normal Values
Learning Goals 3282 Click to open/closeHaematology
Question Filters Click to open/closeImmunoglobulins
Click to open/closeBiochemistry
Click to open/closeDiabetes Click to
Security open/closeEndocrinology Click to
open/closeBlood gases Click to
Change Password open/closeCSF
Sign Out

© 2011 PasTest Ltd | About Us | Contact Us | Help

file:///E|/Shakhawan/Endocrinology/202a.htm[3/13/2012 7:11:06 PM]


MyPasTest

Main Navigation
Question Browser: MRCP 1
Home
Subscriptions Question Browser Exam Builder Saved Exams
» MRCP 1 Session Progress
• Question Browser
Questions Correct 6
• Timed Test A 52-year-old business man undergoes a routine medical
examination as part of his company's health programme. Questions Incorrect 2
• Mock Exam
His blood tests, performed on a fasting venous sample, Questions Total 8
• Past Papers show amongst other results a serum glucose concentration
Questions Percentage 75 %
• Random Questions of 7.1 mmol/l. This is repeated by his GP two days later
and again is 7.1 mmol/l. More
• My Performance
• Media Bank How would you interpret this result?
• New Multimedia
Normal results
Online Extras Reference: Normal Values
Impaired glucose tolerance
Library
Diabetes mellitus Click to
Community open/closeHaematology
Syndrome X
Help Click to
Insulin resistance open/closeImmunoglobulins
PasTest Store
Click to
3283 open/closeBiochemistry
My Account Click to open/closeDiabetes
Click to
Profile open/closeEndocrinology
Click to open/closeBlood
Newsletters gases Click to
My Career and Exams open/closeCSF
Order History
Learning Goals
Question Filters

Security

Change Password
Sign Out

© 2011 PasTest Ltd | About Us | Contact Us | Help

file:///E|/Shakhawan/Endocrinology/203.htm[3/13/2012 7:11:07 PM]


MyPasTest

Main Navigation
Question Browser: MRCP 1
Home
Subscriptions Question Browser Exam Builder Saved Exams
» MRCP 1 Session Progress
• Question Browser
Questions Correct 7
• Timed Test A 52-year-old business man undergoes a routine medical
examination as part of his company's health programme. Questions Incorrect 2
• Mock Exam
His blood tests, performed on a fasting venous sample, Questions Total 9
• Past Papers show amongst other results a serum glucose concentration
Questions Percentage 77 %
• Random Questions of 7.1 mmol/l. This is repeated by his GP two days later
and again is 7.1 mmol/l. More
• My Performance
• Media Bank How would you interpret this result?
• New Multimedia
Normal results
Online Extras
Impaired glucose tolerance
Library
Diabetes mellitus Your answer
Community
Syndrome X
Help
Insulin resistance
PasTest Store

My Account

Profile Diabetes mellitus: fasting glucose more than 7.0 mmol/l


and/or a value exceeding 11.1 mmol/l, either at 2 h
Newsletters during an oral glucose tolerance test or in a random
My Career and Exams sample. The corresponding levels in non-SI units are
126 and 200 mg/dl respectively. The diagnostic fasting Reference: Normal Values
Order History
glucose level was lowered from the previous value of
Learning Goals 7.8 mmol/l to reflect more accurately the risk of Click to open/closeHaematology
Question Filters developing diabetic retinopathy. Click to open/closeImmunoglobulins
Click to open/closeBiochemistry
Impaired glucose tolerance: fasting glucose less than
Click to open/closeDiabetes Click to
Security 7.0 mmol/l and 2 h oral glucose tolerance test value
open/closeEndocrinology Click to
between 7.8 and 11.1 mmol/l.
open/closeBlood gases Click to
Change Password open/closeCSF
Impaired fasting glucose: fasting glucose 6.1 to 6.9
Sign Out mmol/l (110–124 mg/dl).

3283

© 2011 PasTest Ltd | About Us | Contact Us | Help

file:///E|/Shakhawan/Endocrinology/203a.htm[3/13/2012 7:11:09 PM]


MyPasTest

Main Navigation
Question Browser: MRCP 1
Home
Subscriptions Question Browser Exam Builder Saved Exams
» MRCP 1 Session Progress
• Question Browser
Questions Correct 7
• Timed Test You undertake a 12-month attachment to the renal unit of
a large district general hospital, and have a weekly clinic Questions Incorrect 2
• Mock Exam
session in their diabetic nephropathy clinic. Which is the Questions Total 9
• Past Papers most appropriate oral hypoglycaemic drug in patients
Questions Percentage 77 %
• Random Questions with impaired renal function?
• My Performance More
Chlorpropamide
• Media Bank
Glibenclamide
• New Multimedia
Glimepiride
Online Extras Reference: Normal Values
Metformin
Library
Tolbutamide Click to
Community open/closeHaematology
Help 3284 Click to
open/closeImmunoglobulins
PasTest Store
Click to
open/closeBiochemistry
My Account Click to open/closeDiabetes
Click to
Profile open/closeEndocrinology
Click to open/closeBlood
Newsletters gases Click to
My Career and Exams open/closeCSF
Order History
Learning Goals
Question Filters

Security

Change Password
Sign Out

© 2011 PasTest Ltd | About Us | Contact Us | Help

file:///E|/Shakhawan/Endocrinology/204.htm[3/13/2012 7:11:11 PM]


MyPasTest

Main Navigation
Question Browser: MRCP 1
Home
Subscriptions Question Browser Exam Builder Saved Exams
» MRCP 1 Session Progress
• Question Browser
Questions Correct 7
• Timed Test You undertake a 12-month attachment to the renal unit of
a large district general hospital, and have a weekly clinic Questions Incorrect 3
• Mock Exam
session in their diabetic nephropathy clinic. Which is the Questions Total 10
• Past Papers most appropriate oral hypoglycaemic drug in patients
Questions Percentage 70 %
• Random Questions with impaired renal function?
• My Performance More
Chlorpropamide
• Media Bank
Glibenclamide Your answer
• New Multimedia
Glimepiride
Online Extras
Metformin
Library
Tolbutamide Correct answer
Community
Help
PasTest Store
Chlorpropamide has a long duration of action and is
My Account excreted largely unchanged by the kidneys.
Glibenclamide has a long biological duration of action
Profile (active metabolites) and is largely excreted by the
kidneys. Both should be avoided in renal failure and in
Newsletters
the elderly to avoid accumulation and subsequent
My Career and Exams hypoglycaemia. Tolbutamide is largely eliminated by
Order History hepatic metabolism and has a short duration of action. Reference: Normal Values
It is useful in renal failure. Metformin is largely excreted
Learning Goals by the kidneys and thus accumulates as they fail. Click to open/closeHaematology
Question Filters Glimepiride is a sulphonylurea which is renally excreted Click to open/closeImmunoglobulins
and thus is contraindicated in renal impairment. Click to open/closeBiochemistry
Click to open/closeDiabetes Click to
Security open/closeEndocrinology Click to
3284 open/closeBlood gases Click to
Change Password open/closeCSF
Sign Out

© 2011 PasTest Ltd | About Us | Contact Us | Help

file:///E|/Shakhawan/Endocrinology/204a.htm[3/13/2012 7:11:12 PM]


MyPasTest

Main Navigation
Question Browser: MRCP 1
Home
Subscriptions Question Browser Exam Builder Saved Exams
» MRCP 1 Session Progress
• Question Browser
Questions Correct 7
• Timed Test An obese patient with type-2 diabetes mellitus that is not
controlled by appropriate diet needs hypoglycaemic Questions Incorrect 3
• Mock Exam
medication. Which would be the most appropriate Questions Total 10
• Past Papers drug?
Questions Percentage 70 %
• Random Questions
• My Performance Sulphonylureas More
• Media Bank Metformin

• New Multimedia Sitagliptin

Online Extras Insulin


Reference: Normal Values
Library Pioglitazone
Click to
Community 3285 open/closeHaematology
Help Click to
open/closeImmunoglobulins
PasTest Store
Click to
open/closeBiochemistry
My Account Click to open/closeDiabetes
Click to
Profile open/closeEndocrinology
Click to open/closeBlood
Newsletters gases Click to
My Career and Exams open/closeCSF
Order History
Learning Goals
Question Filters

Security

Change Password
Sign Out

© 2011 PasTest Ltd | About Us | Contact Us | Help

file:///E|/Shakhawan/Endocrinology/205.htm[3/13/2012 7:11:14 PM]


MyPasTest

Main Navigation
Question Browser: MRCP 1
Home
Subscriptions Question Browser Exam Builder Saved Exams
» MRCP 1 Session Progress
• Question Browser
Questions Correct 8
• Timed Test An obese patient with type-2 diabetes mellitus that is not
controlled by appropriate diet needs hypoglycaemic Questions Incorrect 3
• Mock Exam
medication. Which would be the most appropriate Questions Total 11
• Past Papers drug?
Questions Percentage 72 %
• Random Questions
• My Performance Sulphonylureas More
• Media Bank Metformin Your answer

• New Multimedia Sitagliptin

Online Extras Insulin

Library Pioglitazone

Community
Help
PasTest Store
Metformin is preferable to sulphonylureas in treating
obese non-insulin diabetes mellitus (NIDDM) patients
My Account since it promotes weight loss and sulphonylureas tend
to cause weight gain. Metformin accumulates in the
Profile intestinal mucosa after ingestion and frequently causes
gastrointestinal side-effects. These tend to reduce in
Newsletters
severity with continued treatment. Glimepiride is a
My Career and Exams sulphonylurea which is renally excreted and thus is
Order History contraindicated in renal impairment. Reference: Normal Values
Learning Goals
Click to open/closeHaematology
Question Filters 3285 Click to open/closeImmunoglobulins
Click to open/closeBiochemistry
Click to open/closeDiabetes Click to
Security open/closeEndocrinology Click to
open/closeBlood gases Click to
Change Password open/closeCSF
Sign Out

© 2011 PasTest Ltd | About Us | Contact Us | Help

file:///E|/Shakhawan/Endocrinology/205a.htm[3/13/2012 7:11:15 PM]


MyPasTest

Main Navigation
Question Browser: MRCP 1
Home
Subscriptions Question Browser Exam Builder Saved Exams
» MRCP 1 Session Progress
• Question Browser
Questions Correct 8
• Timed Test A patient with longstanding type-2 diabetes is found to
have a urinary albumin excretion rate of 300mg/l. His Questions Incorrect 3
• Mock Exam
diabetes is well controlled on metformin and he is Questions Total 11
• Past Papers normotensive. What is the most appropriate additional
Questions Percentage 72 %
• Random Questions medication from which he will benefit?
• My Performance More
Sulfonylureas
• Media Bank
Insulin
• New Multimedia
Lisinopril
Online Extras Reference: Normal Values
Furosemide
Library
Pioglitazone Click to
Community open/closeHaematology
Help 3286 Click to
open/closeImmunoglobulins
PasTest Store
Click to
open/closeBiochemistry
My Account Click to open/closeDiabetes
Click to
Profile open/closeEndocrinology
Click to open/closeBlood
Newsletters gases Click to
My Career and Exams open/closeCSF
Order History
Learning Goals
Question Filters

Security

Change Password
Sign Out

© 2011 PasTest Ltd | About Us | Contact Us | Help

file:///E|/Shakhawan/Endocrinology/206.htm[3/13/2012 7:11:17 PM]


MyPasTest

Main Navigation
Question Browser: MRCP 1
Home
Subscriptions Question Browser Exam Builder Saved Exams
» MRCP 1 Session Progress
• Question Browser
Questions Correct 9
• Timed Test A patient with longstanding type-2 diabetes is found to
have a urinary albumin excretion rate of 300mg/l. His Questions Incorrect 3
• Mock Exam
diabetes is well controlled on metformin and he is Questions Total 12
• Past Papers normotensive. What is the most appropriate additional
Questions Percentage 75 %
• Random Questions medication from which he will benefit?
• My Performance More
Sulfonylureas
• Media Bank
Insulin
• New Multimedia
Lisinopril Your answer
Online Extras
Furosemide
Library
Pioglitazone
Community
Help
PasTest Store
In patients with type-1 diabetes with a mean entry
My Account blood pressure of 122/77 mmHg, a combined analysis
of one European and one American study showed an
Profile adjusted risk reduction of 63% for the development of
clinical nephropathy comparing the Angiotensin-
Newsletters
converting enzyme (ACE) inhibitor captopril 100 mg/day
My Career and Exams with placebo. Three smaller studies in normotensive
Order History patients with type-2 diabetes have reported a similar Reference: Normal Values
reduction in the rate of development of clinical
Learning Goals nephropathy. Thus blockade of the renin–angiotensin Click to open/closeHaematology
Question Filters system by any means appears to confer benefit. Click to open/closeImmunoglobulins
Click to open/closeBiochemistry
Click to open/closeDiabetes Click to
Security 3286 open/closeEndocrinology Click to
open/closeBlood gases Click to
Change Password open/closeCSF
Sign Out

© 2011 PasTest Ltd | About Us | Contact Us | Help

file:///E|/Shakhawan/Endocrinology/206a.htm[3/13/2012 7:11:18 PM]


MyPasTest

Main Navigation
Question Browser: MRCP 1
Home
Subscriptions Question Browser Exam Builder Saved Exams
» MRCP 1 Session Progress
• Question Browser
Questions Correct 0
• Timed Test A 34-year-old accountant is referred to you with a history
of diabetes mellitus since the age of sixteen years. He is a Questions Incorrect 0
• Mock Exam
methodical man and appears always to have striven to Questions Total 0
• Past Papers maintain reasonable metabolic control of his condition,
Questions Percentage 0%
• Random Questions while managing a normal lifestyle.
• My Performance More
What is the most likely finding on examination and
• Media Bank routine testing?
• New Multimedia
A raised albumin to creatinine ratio in the urine
Online Extras (microalbuminuria) Reference: Normal Values
Library Absent ankle jerks
Click to
Community Anti-thyroid antibodies open/closeHaematology
Help Background retinopathy Click to
open/closeImmunoglobulins
PasTest Store Macroproteinuria dipstick urine testing Click to
open/closeBiochemistry
3287 Click to open/closeDiabetes
My Account
Click to
Profile open/closeEndocrinology
Click to open/closeBlood
Newsletters gases Click to
My Career and Exams open/closeCSF
Order History
Learning Goals
Question Filters

Security

Change Password
Sign Out

© 2011 PasTest Ltd | About Us | Contact Us | Help

file:///E|/Shakhawan/Endocrinology/207.htm[3/13/2012 7:11:19 PM]


MyPasTest

Main Navigation
Question Browser: MRCP 1
Home
Subscriptions Question Browser Exam Builder Saved Exams
» MRCP 1 Session Progress
• Question Browser
Questions Correct 0
• Timed Test A 34-year-old accountant is referred to you with a history
of diabetes mellitus since the age of sixteen years. He is a Questions Incorrect 1
• Mock Exam
methodical man and appears always to have striven to Questions Total 1
• Past Papers maintain reasonable metabolic control of his condition,
Questions Percentage 0%
• Random Questions while managing a normal lifestyle.
• My Performance More
What is the most likely finding on examination and
• Media Bank routine testing?
• New Multimedia
A raised albumin to creatinine ratio Your answer
Online Extras in the urine (microalbuminuria)
Library Absent ankle jerks
Community Anti-thyroid antibodies
Help Background retinopathy Correct answer
PasTest Store Macroproteinuria dipstick urine
testing

My Account

Profile
Newsletters The risk of developing retinopathy progressively
My Career and Exams increases with increasing duration of diabetes.
Standardised methods of assessing retinopathy, Reference: Normal Values
Order History
including retinal photography, have revealed that 97%
Learning Goals of insulin-dependent diabetics have some evidence of Click to open/closeHaematology
Question Filters retinopathy (ie background changes at the least) 15 Click to open/closeImmunoglobulins
years after diagnosis. Half have proliferative changes 20 Click to open/closeBiochemistry
years after diagnosis. Click to open/closeDiabetes Click to
Security open/closeEndocrinology Click to
Diabetic nephropathy affects 25–30% of people with
open/closeBlood gases Click to
Change Password insulin dependent diabetes and most of the cases that
open/closeCSF
are ever going to occur will have occurred by 20 years
Sign Out of diabetes in this group of patients. These patients will
have macroproteinuria. Insulin dependent diabetes is an
autoimmune disease, and there is an increased
prevalence of other autoimmune diseases – despite this,
only a minority have positive anti-thyroid antibodies.

3287

© 2011 PasTest Ltd | About Us | Contact Us | Help

file:///E|/Shakhawan/Endocrinology/207a.htm[3/13/2012 7:11:21 PM]


MyPasTest

Main Navigation
Question Browser: MRCP 1
Home
Subscriptions Question Browser Exam Builder Saved Exams
» MRCP 1 Session Progress
• Question Browser
Questions Correct 0
• Timed Test A patient presents with diabetic ketoacidosis. They have a
GCS of 12 and their airway is not compromised but have a Questions Incorrect 1
• Mock Exam
respiratory rate of 32, you commence oxygen therapy. Questions Total 1
• Past Papers
What is the most important next step? Questions Percentage 0%
• Random Questions
• My Performance More
Subcutaneous insulin
• Media Bank
Fluid and electrolytes
• New Multimedia
Intravenous insulin
Online Extras Reference: Normal Values
Intubation
Library
Oxygen Click to
Community open/closeHaematology
Help 3288 Click to
open/closeImmunoglobulins
PasTest Store
Click to
open/closeBiochemistry
My Account Click to open/closeDiabetes
Click to
Profile open/closeEndocrinology
Click to open/closeBlood
Newsletters gases Click to
My Career and Exams open/closeCSF
Order History
Learning Goals
Question Filters

Security

Change Password
Sign Out

© 2011 PasTest Ltd | About Us | Contact Us | Help

file:///E|/Shakhawan/Endocrinology/208.htm[3/13/2012 7:11:22 PM]


MyPasTest

Main Navigation
Question Browser: MRCP 1
Home
Subscriptions Question Browser Exam Builder Saved Exams
» MRCP 1 Session Progress
• Question Browser
Questions Correct 1
• Timed Test A patient presents with diabetic ketoacidosis. They have a
GCS of 12 and their airway is not compromised but have a Questions Incorrect 1
• Mock Exam
respiratory rate of 32, you commence oxygen therapy. Questions Total 2
• Past Papers
What is the most important next step? Questions Percentage 50 %
• Random Questions
• My Performance More
Subcutaneous insulin
• Media Bank
Fluid and electrolytes Your answer
• New Multimedia
Intravenous insulin
Online Extras
Intubation
Library
Oxygen
Community
Help
PasTest Store
Patients with ketoacidosis tend to die of circulatory
My Account problems due to hypovolaemia, and to electrolyte
disturbances, rather than due to insulin deficiency per
Profile se. Early fluid and electrolyte replacement takes
precedence over insulin treatment. All patients with
Newsletters ketoacidosis are depleted of potassium. Insulin will push
My Career and Exams potassium into cells with a risk of hypokalaemia, even in
oliguric patients. Potassium should be given with fluid Reference: Normal Values
Order History
from the beginning and only withdrawn later if oliguria
Learning Goals is confirmed and the plasma potassium is not below Click to open/closeHaematology
Question Filters normal. Click to open/closeImmunoglobulins
Click to open/closeBiochemistry
Intravenous insulin infusion (or hourly intramuscular
Click to open/closeDiabetes Click to
Security insulin injections) is mandatory, as blood flow to the
open/closeEndocrinology Click to
subcutaneous tissues is poor in ketoacidosis. A
open/closeBlood gases Click to
Change Password nasogastric tube should be used for patients with
open/closeCSF
impaired consciousness as fluid accumulates in the
Sign Out stomach due to gastroparesis and gastric dilation.
Vomiting and aspiration are resultant hazards.
Gluconeogenesis is increased in ketoacidosis and
contributes to the hyperglycaemia.

3288

© 2011 PasTest Ltd | About Us | Contact Us | Help

file:///E|/Shakhawan/Endocrinology/208a.htm[3/13/2012 7:11:24 PM]


MyPasTest

Main Navigation
Question Browser: MRCP 1
Home
Subscriptions Question Browser Exam Builder Saved Exams
» MRCP 1 Session Progress
• Question Browser
Questions Correct 1
• Timed Test You are working in a city with a hard-pressed
ophthalmology service. You perform an annual review of a Questions Incorrect 1
• Mock Exam
patient with longstanding diabetes mellitus. Her visual Questions Total 2
• Past Papers acuity is 6/9 in both eyes. Which of the following
Questions Percentage 50 %
• Random Questions fundoscopic findings would be likely to warrant you
making an urgent referral to an ophthalmologist? More
• My Performance
• Media Bank Hard exudates five disk widths medial to the optic
• New Multimedia disk
Online Extras More than five blot haemorrhages anywhere in the Reference: Normal Values
retina
Library
More than five microaneurysms in the macular area Click to
Community open/closeHaematology
Neovascularisation near the optic disk
Help Click to
Scattered cotton wool spots open/closeImmunoglobulins
PasTest Store
Click to
3289 open/closeBiochemistry
My Account Click to open/closeDiabetes
Click to
Profile open/closeEndocrinology
Click to open/closeBlood
Newsletters gases Click to
My Career and Exams open/closeCSF
Order History
Learning Goals
Question Filters

Security

Change Password
Sign Out

© 2011 PasTest Ltd | About Us | Contact Us | Help

file:///E|/Shakhawan/Endocrinology/209.htm[3/13/2012 7:11:25 PM]


MyPasTest

Main Navigation
Question Browser: MRCP 1
Home
Subscriptions Question Browser Exam Builder Saved Exams
» MRCP 1 Session Progress
• Question Browser
Questions Correct 2
• Timed Test You are working in a city with a hard-pressed
ophthalmology service. You perform an annual review of a Questions Incorrect 1
• Mock Exam
patient with longstanding diabetes mellitus. Her visual Questions Total 3
• Past Papers acuity is 6/9 in both eyes. Which of the following
Questions Percentage 66 %
• Random Questions fundoscopic findings would be likely to warrant you
making an urgent referral to an ophthalmologist? More
• My Performance
• Media Bank Hard exudates five disk widths medial
• New Multimedia to the optic disk
Online Extras More than five blot haemorrhages
anywhere in the retina
Library
More than five microaneurysms in the
Community macular area
Help Neovascularisation near the optic disk Your answer
PasTest Store Scattered cotton wool spots

My Account

Profile
Microaneurysms, blot haemorrhages and hard exudates
Newsletters
are features of background retinopathy. These do not in
My Career and Exams themselves require ophthalmologic referral or
Order History treatment. The exception is hard exudates within one Reference: Normal Values
disk width of the macula. These may be associated with
Learning Goals macular oedema (invisible with the ophthalmoscope), Click to open/closeHaematology
Question Filters which may permanently distort macular vision. Such Click to open/closeImmunoglobulins
hard exudates near the macula do warrant referral for Click to open/closeBiochemistry
possible macular grid laser photocoagulation. Click to open/closeDiabetes Click to
Security open/closeEndocrinology Click to
Cotton wool spots are due to retinal infarcts. These are open/closeBlood gases Click to
Change Password pre-proliferative, not background, changes and warrant open/closeCSF
non-urgent referral. New vessels near the disk are likely
Sign Out
to rupture and cause vitreous haemorrhage. They
warrant urgent, not routine, referral to an
ophthalmologist with a view to likely laser
photocoagulation treatment.

3289

© 2011 PasTest Ltd | About Us | Contact Us | Help

file:///E|/Shakhawan/Endocrinology/209a.htm[3/13/2012 7:11:27 PM]


MyPasTest

Main Navigation
Question Browser: MRCP 1
Home
Subscriptions Question Browser Exam Builder Saved Exams
» MRCP 1 Session Progress
• Question Browser
Questions Correct 2
• Timed Test A 29-year-old man is brought to Accident and Emergency.
He has been found unconscious by the ambulance service, Questions Incorrect 1
• Mock Exam
having collapsed at a darts match. The family history Questions Total 3
• Past Papers reveals a brother with type-1 diabetes. A blood test shows
Questions Percentage 66 %
• Random Questions the following results: Glucose 1.1 mmol/l, Insulin > 500
pmol/l (reference 15–100pmol/l) and C peptide of More
• My Performance
<0.2pmol/l (reference 0.2–1.4pmol/l). What is the most
• Media Bank likely diagnosis?
• New Multimedia
Insulinoma
Online Extras Reference: Normal Values
Type 1 diabetes mellitus
Library
Type 2 diabetes mellitus Click to
Community open/closeHaematology
Insulin resistance
Help Click to
Factitious hypoglycaemia open/closeImmunoglobulins
PasTest Store
Click to
3290 open/closeBiochemistry
My Account Click to open/closeDiabetes
Click to
Profile open/closeEndocrinology
Click to open/closeBlood
Newsletters gases Click to
My Career and Exams open/closeCSF
Order History
Learning Goals
Question Filters

Security

Change Password
Sign Out

© 2011 PasTest Ltd | About Us | Contact Us | Help

file:///E|/Shakhawan/Endocrinology/210.htm[3/13/2012 7:11:28 PM]


MyPasTest

Main Navigation
Question Browser: MRCP 1
Home
Subscriptions Question Browser Exam Builder Saved Exams
» MRCP 1 Session Progress
• Question Browser
Questions Correct 3
• Timed Test A 29-year-old man is brought to Accident and Emergency.
He has been found unconscious by the ambulance service, Questions Incorrect 1
• Mock Exam
having collapsed at a darts match. The family history Questions Total 4
• Past Papers reveals a brother with type-1 diabetes. A blood test shows
Questions Percentage 75 %
• Random Questions the following results: Glucose 1.1 mmol/l, Insulin > 500
pmol/l (reference 15–100pmol/l) and C peptide of More
• My Performance
<0.2pmol/l (reference 0.2–1.4pmol/l). What is the most
• Media Bank likely diagnosis?
• New Multimedia
Insulinoma
Online Extras
Type 1 diabetes mellitus
Library
Type 2 diabetes mellitus
Community
Insulin resistance
Help
Factitious hypoglycaemia Your answer
PasTest Store

My Account

Profile Factitious insulin-induced hypoglycaemia is as common


in previously healthy subjects as in insulin-dependent
Newsletters
diabetics and is due to deliberate, but concealed
My Career and Exams injection of insulin. The history suggests insulinoma, but
Order History is eliminated by the laboratory results, which reveal Reference: Normal Values
high plasma insulin and low C-peptide (and proinsulin)
Learning Goals concentrations during hypoglycaemia. Click to open/closeHaematology
Question Filters Click to open/closeImmunoglobulins
In long-standing factitious hypoglycaemia, and in Click to open/closeBiochemistry
insulin-treated diabetics, insulin antibodies may be Click to open/closeDiabetes Click to
Security present in the plasma. Although once considered a open/closeEndocrinology Click to
strong pointer to factitious hypoglycaemia, the presence open/closeBlood gases Click to
Change Password of insulin antibodies should nowadays suggest open/closeCSF
autoimmune insulin syndrome.
Sign Out

3290

© 2011 PasTest Ltd | About Us | Contact Us | Help

file:///E|/Shakhawan/Endocrinology/210a.htm[3/13/2012 7:11:30 PM]


MyPasTest

Main Navigation
Question Browser: MRCP 1
Home
Subscriptions Question Browser Exam Builder Saved Exams
» MRCP 1 Session Progress
• Question Browser
Questions Correct 3
• Timed Test A 29-year-old woman is undergoing investigations for
subfertility. You wish to determine whether or not she is Questions Incorrect 1
• Mock Exam
ovulating normally. Which of the following hormone Questions Total 4
• Past Papers levels would be most likely to indicate the occurrence
Questions Percentage 75 %
• Random Questions of ovulation?
• My Performance More
Estradiol
• Media Bank
Testosterone
• New Multimedia
Follicle-stimulating hormone
Online Extras Reference: Normal Values
Luteinising hormone
Library
Prolactin Click to
Community open/closeHaematology
Help 3409 Click to
open/closeImmunoglobulins
PasTest Store
Click to
open/closeBiochemistry
My Account Click to open/closeDiabetes
Click to
Profile open/closeEndocrinology
Click to open/closeBlood
Newsletters gases Click to
My Career and Exams open/closeCSF
Order History
Learning Goals
Question Filters

Security

Change Password
Sign Out

© 2011 PasTest Ltd | About Us | Contact Us | Help

file:///E|/Shakhawan/Endocrinology/211.htm[3/13/2012 7:11:31 PM]


MyPasTest

Main Navigation
Question Browser: MRCP 1
Home
Subscriptions Question Browser Exam Builder Saved Exams
» MRCP 1 Session Progress
• Question Browser
Questions Correct 4
• Timed Test A 29-year-old woman is undergoing investigations for
subfertility. You wish to determine whether or not she is Questions Incorrect 1
• Mock Exam
ovulating normally. Which of the following hormone Questions Total 5
• Past Papers levels would be most likely to indicate the occurrence
Questions Percentage 80 %
• Random Questions of ovulation?
• My Performance More
Estradiol
• Media Bank
Testosterone
• New Multimedia
Follicle-stimulating hormone
Online Extras
Luteinising hormone Your answer
Library
Prolactin
Community
Help
PasTest Store
In a normal menstrual cycle, there is a gradual increase
My Account in estradiol levels during the follicular phase. Just before
ovulation, this estradiol rise, coupled with a small rise in
Profile progesterone, causes a sudden increase in the pituitary
production of luteinising hormone (LH) and follicle-
Newsletters
stimulating hormone (FSH). The surge in LH is of
My Career and Exams particular significance as it is responsible for the
Order History induction of ovulation. Prolactin levels are unaffected in Reference: Normal Values
a normal menstrual cycle.
Learning Goals
Click to open/closeHaematology
Question Filters Click to open/closeImmunoglobulins
3409 Click to open/closeBiochemistry
Click to open/closeDiabetes Click to
Security open/closeEndocrinology Click to
open/closeBlood gases Click to
Change Password open/closeCSF
Sign Out

© 2011 PasTest Ltd | About Us | Contact Us | Help

file:///E|/Shakhawan/Endocrinology/211a.htm[3/13/2012 7:11:33 PM]


MyPasTest

Main Navigation
Question Browser: MRCP 1
Home
Subscriptions Question Browser Exam Builder Saved Exams
» MRCP 1 Session Progress
• Question Browser
Questions Correct 4
• Timed Test A 25-year-old insulin-dependent diabetic man is on long-
acting insulin preparations. Which of the following Questions Incorrect 1
• Mock Exam
enzymes is most likely to be inhibited by insulin? Questions Total 5
• Past Papers
Questions Percentage 80 %
• Random Questions Pyruvate carboxylase
• My Performance Pyruvate kinase More
• Media Bank Glycogen synthetase
• New Multimedia Glucose-6-phosphate dehydrogenase
Online Extras Pyruvate dehydrogenase Reference: Normal Values
Library
3410 Click to
Community open/closeHaematology
Help Click to
open/closeImmunoglobulins
PasTest Store
Click to
open/closeBiochemistry
My Account Click to open/closeDiabetes
Click to
Profile open/closeEndocrinology
Click to open/closeBlood
Newsletters gases Click to
My Career and Exams open/closeCSF
Order History
Learning Goals
Question Filters

Security

Change Password
Sign Out

© 2011 PasTest Ltd | About Us | Contact Us | Help

file:///E|/Shakhawan/Endocrinology/212.htm[3/13/2012 7:11:34 PM]


MyPasTest

Main Navigation
Question Browser: MRCP 1
Home
Subscriptions Question Browser Exam Builder Saved Exams
» MRCP 1 Session Progress
• Question Browser
Questions Correct 4
• Timed Test A 25-year-old insulin-dependent diabetic man is on long-
acting insulin preparations. Which of the following Questions Incorrect 2
• Mock Exam
enzymes is most likely to be inhibited by insulin? Questions Total 6
• Past Papers
Questions Percentage 66 %
• Random Questions Pyruvate carboxylase Correct answer
• My Performance Pyruvate kinase Your answer More
• Media Bank Glycogen synthetase
• New Multimedia Glucose-6-phosphate
Online Extras dehydrogenase

Library Pyruvate dehydrogenase

Community
Help
PasTest Store
Insulin inhibits gluconeogenesis by inhibiting pyruvate
carboxylase. It increases glycolysis by inducing the
My Account synthesis of glucose-6-phosphate dehydrogenase. Both
pyruvate dehydrogenase and glycogen synthetase are
Profile activated by insulin.
Newsletters
My Career and Exams 3410
Order History Reference: Normal Values
Learning Goals
Click to open/closeHaematology
Question Filters Click to open/closeImmunoglobulins
Click to open/closeBiochemistry
Click to open/closeDiabetes Click to
Security open/closeEndocrinology Click to
open/closeBlood gases Click to
Change Password open/closeCSF
Sign Out

© 2011 PasTest Ltd | About Us | Contact Us | Help

file:///E|/Shakhawan/Endocrinology/212a.htm[3/13/2012 7:11:35 PM]


MyPasTest

Main Navigation
Question Browser: MRCP 1
Home
Subscriptions Question Browser Exam Builder Saved Exams
» MRCP 1 Session Progress
• Question Browser
Questions Correct 4
• Timed Test A 32-year-old woman undergoing infertility treatment is
found to have proliferative endometrium on routine Questions Incorrect 2
• Mock Exam
endometrial sampling. What is the most probable Questions Total 6
• Past Papers diagnosis in this case?
Questions Percentage 66 %
• Random Questions
• My Performance Pregnancy More
• Media Bank Anovulation

• New Multimedia Ovulation

Online Extras Neoplasia


Reference: Normal Values
Library Luteal-phase defect
Click to
Community 3411 open/closeHaematology
Help Click to
open/closeImmunoglobulins
PasTest Store
Click to
open/closeBiochemistry
My Account Click to open/closeDiabetes
Click to
Profile open/closeEndocrinology
Click to open/closeBlood
Newsletters gases Click to
My Career and Exams open/closeCSF
Order History
Learning Goals
Question Filters

Security

Change Password
Sign Out

© 2011 PasTest Ltd | About Us | Contact Us | Help

file:///E|/Shakhawan/Endocrinology/213.htm[3/13/2012 7:11:37 PM]


MyPasTest

Main Navigation
Question Browser: MRCP 1
Home
Subscriptions Question Browser Exam Builder Saved Exams
» MRCP 1 Session Progress
• Question Browser
Questions Correct 5
• Timed Test A 32-year-old woman undergoing infertility treatment is
found to have proliferative endometrium on routine Questions Incorrect 2
• Mock Exam
endometrial sampling. What is the most probable Questions Total 7
• Past Papers diagnosis in this case?
Questions Percentage 71 %
• Random Questions
• My Performance Pregnancy More
• Media Bank Anovulation Your answer

• New Multimedia Ovulation

Online Extras Neoplasia

Library Luteal-phase defect

Community
Help
PasTest Store
The presence of proliferative endometrium indicates
that ovulation has not occurred. Endometrial biopsy is
My Account still performed in many clinics world-wide as an
indicator of ovulation and adequate progesterone
Profile secretion in the luteal phase. Proliferative endometrium
is seen in the follicular phase before ovulation.
Newsletters
Secretory endometrium is the hallmark of ovulation: the
My Career and Exams endometrium thickens and contains more tortuous
Order History glands that secrete mucus. Aplastic cells would be seen Reference: Normal Values
in neoplasia of the endometrium. Luteal-phase defect is
Learning Goals the result of insufficient progesterone production by the Click to open/closeHaematology
Question Filters corpus luteum. The endometrium would be secretory in Click to open/closeImmunoglobulins
this case. In pregnancy, the secretory endometrium Click to open/closeBiochemistry
would be much thicker and is known as ‘decidua’. Click to open/closeDiabetes Click to
Security open/closeEndocrinology Click to
open/closeBlood gases Click to
Change Password 3411 open/closeCSF
Sign Out

© 2011 PasTest Ltd | About Us | Contact Us | Help

file:///E|/Shakhawan/Endocrinology/213a.htm[3/13/2012 7:11:38 PM]


MyPasTest

Main Navigation
Question Browser: MRCP 1
Home
Subscriptions Question Browser Exam Builder Saved Exams
» MRCP 1 Session Progress
• Question Browser
Questions Correct 5
• Timed Test A 16-year-old youth is seen in the clinic because of poor
development of secondary sex characteristics. On Questions Incorrect 2
• Mock Exam
examination pubic hair growth is minimal. Facial hair Questions Total 7
• Past Papers growth is absent. The external genitalia are small. Both
Questions Percentage 71 %
• Random Questions testes are normal in size and located in the scrotum.
Which of the following hormones is most likely to be More
• My Performance
malfunctional in this case?
• Media Bank
• New Multimedia Testosterone
Online Extras Androstenedione Reference: Normal Values
Library Dihydrotestosterone
Dehydroepiandrosterone Click to
Community open/closeHaematology
Help Androsterone Click to
open/closeImmunoglobulins
PasTest Store 3412 Click to
open/closeBiochemistry
My Account Click to open/closeDiabetes
Click to
Profile open/closeEndocrinology
Click to open/closeBlood
Newsletters gases Click to
My Career and Exams open/closeCSF
Order History
Learning Goals
Question Filters

Security

Change Password
Sign Out

© 2011 PasTest Ltd | About Us | Contact Us | Help

file:///E|/Shakhawan/Endocrinology/214.htm[3/13/2012 7:11:40 PM]


MyPasTest

Main Navigation
Question Browser: MRCP 1
Home
Subscriptions Question Browser Exam Builder Saved Exams
» MRCP 1 Session Progress
• Question Browser
Questions Correct 5
• Timed Test A 16-year-old youth is seen in the clinic because of poor
development of secondary sex characteristics. On Questions Incorrect 3
• Mock Exam
examination pubic hair growth is minimal. Facial hair Questions Total 8
• Past Papers growth is absent. The external genitalia are small. Both
Questions Percentage 62 %
• Random Questions testes are normal in size and located in the scrotum.
Which of the following hormones is most likely to be More
• My Performance
malfunctional in this case?
• Media Bank
• New Multimedia Testosterone Your answer
Online Extras Androstenedione
Library Dihydrotestosterone Correct answer
Community Dehydroepiandrosterone
Help Androsterone
PasTest Store

My Account
Testosterone is responsible for the development of
Profile internal genitalia and spermatogenesis. It is converted
to dihydrotestosterone (DHT) in the body by the
Newsletters
enzyme 5 α -reductase. DHT is a more active compound
My Career and Exams than testosterone and is involved in the expression of
Order History male secondary sex characteristics. The absence of 5 α- Reference: Normal Values
Learning Goals reductase or the absence of DHT receptors leads to
testicular feminisation. Click to open/closeHaematology
Question Filters Click to open/closeImmunoglobulins
Click to open/closeBiochemistry
3412 Click to open/closeDiabetes Click to
Security open/closeEndocrinology Click to
open/closeBlood gases Click to
Change Password open/closeCSF
Sign Out

© 2011 PasTest Ltd | About Us | Contact Us | Help

file:///E|/Shakhawan/Endocrinology/214a.htm[3/13/2012 7:11:41 PM]


MyPasTest

Main Navigation
Question Browser: MRCP 1
Home
Subscriptions Question Browser Exam Builder Saved Exams
» MRCP 1 Session Progress
• Question Browser
Questions Correct 5
• Timed Test A 25-year-old woman has been prescribed emergency
contraception (levonorgestrel 1.5 mg) following unprotected Questions Incorrect 3
• Mock Exam
sexual intercourse. Her periods have been regular to date, Questions Total 8
• Past Papers and her last period was 2 weeks ago. What is the most
Questions Percentage 62 %
• Random Questions probable mechanism of action of emergency
contraception in preventing conception in this case? More
• My Performance
• Media Bank Thickening the cervical mucus
• New Multimedia Decreasing tubal motility and ciliary activity
Online Extras Inhibiting ovulation Reference: Normal Values
Library Rendering the endometrium unfavourable for
implantation Click to
Community open/closeHaematology
Help Blocking the secretion of progesterone by the corpus Click to
luteum open/closeImmunoglobulins
PasTest Store
Click to
3413 open/closeBiochemistry
My Account Click to open/closeDiabetes
Click to
Profile open/closeEndocrinology
Click to open/closeBlood
Newsletters gases Click to
My Career and Exams open/closeCSF
Order History
Learning Goals
Question Filters

Security

Change Password
Sign Out

© 2011 PasTest Ltd | About Us | Contact Us | Help

file:///E|/Shakhawan/Endocrinology/215.htm[3/13/2012 7:11:43 PM]


MyPasTest

Main Navigation
Question Browser: MRCP 1
Home
Subscriptions Question Browser Exam Builder Saved Exams
» MRCP 1 Session Progress
• Question Browser
Questions Correct 6
• Timed Test A 25-year-old woman has been prescribed emergency
contraception (levonorgestrel 1.5 mg) following unprotected Questions Incorrect 3
• Mock Exam
sexual intercourse. Her periods have been regular to date, Questions Total 9
• Past Papers and her last period was 2 weeks ago. What is the most
Questions Percentage 66 %
• Random Questions probable mechanism of action of emergency
contraception in preventing conception in this case? More
• My Performance
• Media Bank Thickening the cervical mucus
• New Multimedia Decreasing tubal motility and ciliary Your answer
Online Extras activity
Library Inhibiting ovulation
Community Rendering the endometrium
unfavourable for implantation
Help
Blocking the secretion of progesterone
PasTest Store by the corpus luteum

My Account

Profile
This woman would most probably have ovulated on day
Newsletters
12 or 13, ie a day or two prior to unprotected sexual
My Career and Exams intercourse. Intake of emergency contraception at this
Order History stage would decrease tubal peristalsis and ciliary Reference: Normal Values
activity in the tubal mucosa, thereby preventing sperm
Learning Goals from reaching the oocyte in the ampulla of the tube. Click to open/closeHaematology
Question Filters Thickening of the cervical mucus may not be effective, Click to open/closeImmunoglobulins
as sperm may already have entered the uterine cavity. Click to open/closeBiochemistry
Emergency contraception with levonorgestrel does not Click to open/closeDiabetes Click to
Security render the endometrium unfavourable for implantation, open/closeEndocrinology Click to
nor does it block progesterone secretion by the corpus open/closeBlood gases Click to
Change Password luteum. open/closeCSF
Sign Out

3413

© 2011 PasTest Ltd | About Us | Contact Us | Help

file:///E|/Shakhawan/Endocrinology/215a.htm[3/13/2012 7:11:44 PM]


MyPasTest

Main Navigation
Question Browser: MRCP 1
Home
Subscriptions Question Browser Exam Builder Saved Exams
» MRCP 1 Session Progress
• Question Browser
Questions Correct 6
• Timed Test A 30-year-old schoolteacher is on an oral contraceptive pill
Questions Incorrect 3
• Mock Exam containing 20 μg of ethinylestradiol. She asks about its
possible side-effects. Which of the following side- Questions Total 9
• Past Papers
effects is most likely to occur with this dose of Questions Percentage 66 %
• Random Questions oestrogen?
• My Performance More
Deep vein thrombosis
• Media Bank
Nausea and vomiting
• New Multimedia
Increased pregnancy rate
Online Extras Reference: Normal Values
Breakthrough bleeding
Library
Migraine Click to
Community open/closeHaematology
Help 3414 Click to
open/closeImmunoglobulins
PasTest Store
Click to
open/closeBiochemistry
My Account Click to open/closeDiabetes
Click to
Profile open/closeEndocrinology
Click to open/closeBlood
Newsletters gases Click to
My Career and Exams open/closeCSF
Order History
Learning Goals
Question Filters

Security

Change Password
Sign Out

© 2011 PasTest Ltd | About Us | Contact Us | Help

file:///E|/Shakhawan/Endocrinology/216.htm[3/13/2012 7:11:46 PM]


MyPasTest

Main Navigation
Question Browser: MRCP 1
Home
Subscriptions Question Browser Exam Builder Saved Exams
» MRCP 1 Session Progress
• Question Browser
Questions Correct 0
• Timed Test A 30-year-old schoolteacher is on an oral contraceptive pill
Questions Incorrect 1
• Mock Exam containing 20 μg of ethinylestradiol. She asks about its
possible side-effects. Which of the following side- Questions Total 1
• Past Papers
effects is most likely to occur with this dose of Questions Percentage 0%
• Random Questions oestrogen?
• My Performance More
Deep vein thrombosis
• Media Bank
Nausea and vomiting
• New Multimedia
Increased pregnancy rate Your answer
Online Extras
Breakthrough bleeding Correct answer
Library
Migraine
Community
Help
PasTest Store

Breakthrough bleeding is most commonly associated


My Account with low-dose oral contraceptive pills, especially those
containing 20 μg ethinylestradiol. Nausea and vomiting
Profile are less with this dose. Migraine may occur in
Newsletters susceptible women irrespective of the dosage. There is
no appreciable increase in clotting risk or pregnancy
My Career and Exams
rate with these pills.
Order History Reference: Normal Values
Learning Goals
3414 Click to open/closeHaematology
Question Filters Click to open/closeImmunoglobulins
Click to open/closeBiochemistry
Click to open/closeDiabetes Click to
Security open/closeEndocrinology Click to
open/closeBlood gases Click to
Change Password open/closeCSF
Sign Out

© 2011 PasTest Ltd | About Us | Contact Us | Help

file:///E|/Shakhawan/Endocrinology/216a.htm[3/13/2012 7:11:47 PM]


MyPasTest

Main Navigation
Question Browser: MRCP 1
Home
Subscriptions Question Browser Exam Builder Saved Exams
» MRCP 1 Session Progress
• Question Browser
Questions Correct 0
• Timed Test An infant with hypoglycaemia is administered glucagon.
Which of the following actions of glucagon would be Questions Incorrect 1
• Mock Exam
most likely to be effective in treating this condition? Questions Total 1
• Past Papers
Questions Percentage 0%
• Random Questions Increases glycogenolysis in muscle
• My Performance Induces the synthesis of phosphofructokinase More
• Media Bank Stimulates acetyl-CoA carboxylase
• New Multimedia Promotes the formation of cyclic AMP from ATP
Online Extras Activates adenylate cyclase Reference: Normal Values
Library
3415 Click to
Community open/closeHaematology
Help Click to
open/closeImmunoglobulins
PasTest Store
Click to
open/closeBiochemistry
My Account Click to open/closeDiabetes
Click to
Profile open/closeEndocrinology
Click to open/closeBlood
Newsletters gases Click to
My Career and Exams open/closeCSF
Order History
Learning Goals
Question Filters

Security

Change Password
Sign Out

© 2011 PasTest Ltd | About Us | Contact Us | Help

file:///E|/Shakhawan/Endocrinology/217.htm[3/13/2012 7:11:48 PM]


MyPasTest

Main Navigation
Question Browser: MRCP 1
Home
Subscriptions Question Browser Exam Builder Saved Exams
» MRCP 1 Session Progress
• Question Browser
Questions Correct 0
• Timed Test An infant with hypoglycaemia is administered glucagon.
Which of the following actions of glucagon would be Questions Incorrect 2
• Mock Exam
most likely to be effective in treating this condition? Questions Total 2
• Past Papers
Questions Percentage 0%
• Random Questions Increases glycogenolysis in muscle
• My Performance Induces the synthesis of Your answer More
• Media Bank phosphofructokinase

• New Multimedia Stimulates acetyl-CoA carboxylase

Online Extras Promotes the formation of cyclic


AMP from ATP
Library
Activates adenylate cyclase Correct answer
Community
Help
PasTest Store
Glucagon acts mainly on the liver and has no action on
My Account muscle. It increases glycogenolysis and gluconeogenesis
and also stimulates lipolysis in adipose tissue. Adenylate
Profile cyclase is a key enzyme involved in the breakdown of
glycogen polymers to increase availability of glucose.
Newsletters
Activation of this and other enzymes is responsible for
My Career and Exams restoring blood glucose levels in this case. Insulin
Order History induces the synthesis of phosphofructokinase, which Reference: Normal Values
increases glycolysis, and stimulates acetyl-CoA
Learning Goals carboxylase to increase the synthesis of fatty acids. Click to open/closeHaematology
Question Filters Glucagon acts via cyclic AMP to stimulate lipolysis, Click to open/closeImmunoglobulins
producing free fatty acids that can act as a major Click to open/closeBiochemistry
source of energy. Click to open/closeDiabetes Click to
Security open/closeEndocrinology Click to
open/closeBlood gases Click to
Change Password 3415 open/closeCSF
Sign Out

© 2011 PasTest Ltd | About Us | Contact Us | Help

file:///E|/Shakhawan/Endocrinology/217a.htm[3/13/2012 7:11:50 PM]


MyPasTest

Main Navigation
Question Browser: MRCP 1
Home
Subscriptions Question Browser Exam Builder Saved Exams
» MRCP 1 Session Progress
• Question Browser
Questions Correct 0
• Timed Test A 47-year-old woman is undergoing investigations for
secondary amenorrhoea and joint pains. Blood tests show Questions Incorrect 2
• Mock Exam
her follicle-stimulating hormone level to be 40 IU/l (> 30 in Questions Total 2
• Past Papers postmenopausal women). What would be the best
Questions Percentage 0%
• Random Questions method for diagnosing osteoporosis in this patient?
• My Performance More
Dual-energy X-ray absorptiometry
• Media Bank
X-ray of long bones
• New Multimedia
Bone scintigraphy
Online Extras Reference: Normal Values
Quantitative computed tomography
Library
Ultrasonography Click to
Community open/closeHaematology
Help 3416 Click to
open/closeImmunoglobulins
PasTest Store
Click to
open/closeBiochemistry
My Account Click to open/closeDiabetes
Click to
Profile open/closeEndocrinology
Click to open/closeBlood
Newsletters gases Click to
My Career and Exams open/closeCSF
Order History
Learning Goals
Question Filters

Security

Change Password
Sign Out

© 2011 PasTest Ltd | About Us | Contact Us | Help

file:///E|/Shakhawan/Endocrinology/218.htm[3/13/2012 7:11:51 PM]


MyPasTest

Main Navigation
Question Browser: MRCP 1
Home
Subscriptions Question Browser Exam Builder Saved Exams
» MRCP 1 Session Progress
• Question Browser
Questions Correct 1
• Timed Test A 47-year-old woman is undergoing investigations for
secondary amenorrhoea and joint pains. Blood tests show Questions Incorrect 2
• Mock Exam
her follicle-stimulating hormone level to be 40 IU/l (> 30 in Questions Total 3
• Past Papers postmenopausal women). What would be the best
Questions Percentage 33 %
• Random Questions method for diagnosing osteoporosis in this patient?
• My Performance More
Dual-energy X-ray absorptiometry Your answer
• Media Bank
X-ray of long bones
• New Multimedia
Bone scintigraphy
Online Extras
Quantitative computed tomography
Library
Ultrasonography
Community
Help
PasTest Store
This patient is most probably menopausal judging by
My Account her FSH level. All the options are non-invasive
techniques for estimating skeletal mass or bone mineral
Profile density. Dual-energy X-ray absorptiometry (DXA) is the
‘gold standard’ for diagnosing osteoporosis. A DXA scan
Newsletters
provides a T-score reflecting fracture risk, which may
My Career and Exams influence treatment decisions. Fractures are readily
Order History detected by bone scintigraphy even if plain films are Reference: Normal Values
normal.
Learning Goals
Click to open/closeHaematology
Question Filters Click to open/closeImmunoglobulins
3416 Click to open/closeBiochemistry
Click to open/closeDiabetes Click to
Security open/closeEndocrinology Click to
open/closeBlood gases Click to
Change Password open/closeCSF
Sign Out

© 2011 PasTest Ltd | About Us | Contact Us | Help

file:///E|/Shakhawan/Endocrinology/218a.htm[3/13/2012 7:11:53 PM]


MyPasTest

Main Navigation
Question Browser: MRCP 1
Home
Subscriptions Question Browser Exam Builder Saved Exams
» MRCP 1 Session Progress
• Question Browser
Questions Correct 1
• Timed Test A 70-year-old man is admitted to hospital in a comatose
condition. His wife gives a history of past episodes of Questions Incorrect 2
• Mock Exam
sweating and palpitations, confusion, fits and occasional Questions Total 3
• Past Papers abnormal behaviour. On examination his pulse is 92 bpm
Questions Percentage 33 %
• Random Questions and his blood pressure is 140/90 mmHg. His plasma
catecholamine level is normal and blood glucose is 1.9 More
• My Performance
mmol/l (3.0–6.0). What is the most probable
• Media Bank diagnosis?
• New Multimedia
Multiple endocrine neoplasia (MEN)-type II syndrome
Online Extras Reference: Normal Values
Phaeochromocytoma
Library
Insulinoma Click to
Community open/closeHaematology
Pseudodementia
Help Click to
Epilepsy open/closeImmunoglobulins
PasTest Store
Click to
3417 open/closeBiochemistry
My Account Click to open/closeDiabetes
Click to
Profile open/closeEndocrinology
Click to open/closeBlood
Newsletters gases Click to
My Career and Exams open/closeCSF
Order History
Learning Goals
Question Filters

Security

Change Password
Sign Out

© 2011 PasTest Ltd | About Us | Contact Us | Help

file:///E|/Shakhawan/Endocrinology/219.htm[3/13/2012 7:11:54 PM]


MyPasTest

Main Navigation
Question Browser: MRCP 1
Home
Subscriptions Question Browser Exam Builder Saved Exams
» MRCP 1 Session Progress
• Question Browser
Questions Correct 2
• Timed Test A 70-year-old man is admitted to hospital in a comatose
condition. His wife gives a history of past episodes of Questions Incorrect 2
• Mock Exam
sweating and palpitations, confusion, fits and occasional Questions Total 4
• Past Papers abnormal behaviour. On examination his pulse is 92 bpm
Questions Percentage 50 %
• Random Questions and his blood pressure is 140/90 mmHg. His plasma
catecholamine level is normal and blood glucose is 1.9 More
• My Performance
mmol/l (3.0–6.0). What is the most probable
• Media Bank diagnosis?
• New Multimedia
Multiple endocrine neoplasia (MEN)-
Online Extras type II syndrome
Library Phaeochromocytoma
Community Insulinoma Your answer
Help Pseudodementia
PasTest Store Epilepsy

My Account

Profile
This patient most probably has an insulinoma.
Newsletters
Phaeochromocytoma, either isolated or as part of the
My Career and Exams MEN-II syndrome, is unlikely as his blood pressure and
Order History catecholamine levels are normal. Pseudodementia is Reference: Normal Values
associated with severe depressive illness. Memory
Learning Goals impairment, slowed thinking and lack of spontaneity are Click to open/closeHaematology
Question Filters characteristic features. Hypoglycaemia is not associated Click to open/closeImmunoglobulins
with this disorder. The history and tests are not Click to open/closeBiochemistry
suggestive of epilepsy. Click to open/closeDiabetes Click to
Security open/closeEndocrinology Click to
open/closeBlood gases Click to
Change Password 3417 open/closeCSF
Sign Out

© 2011 PasTest Ltd | About Us | Contact Us | Help

file:///E|/Shakhawan/Endocrinology/219a.htm[3/13/2012 7:11:56 PM]


MyPasTest

Main Navigation
Question Browser: MRCP 1
Home
Subscriptions Question Browser Exam Builder Saved Exams
» MRCP 1 Session Progress
• Question Browser
Questions Correct 2
• Timed Test A 40-year-old man is receiving lithium for a bipolar
disorder. He now complains of increased urinary frequency Questions Incorrect 2
• Mock Exam
and nocturia. What is the most likely cause for his Questions Total 4
• Past Papers symptoms?
Questions Percentage 50 %
• Random Questions
• My Performance Deficiency of vasopressin More
• Media Bank Reduced creatinine clearance

• New Multimedia Onset of glomerulonephritis

Online Extras Development of resistance to vasopressin


Reference: Normal Values
Library Excessive water intake
Click to
Community 3418 open/closeHaematology
Help Click to
open/closeImmunoglobulins
PasTest Store
Click to
open/closeBiochemistry
My Account Click to open/closeDiabetes
Click to
Profile open/closeEndocrinology
Click to open/closeBlood
Newsletters gases Click to
My Career and Exams open/closeCSF
Order History
Learning Goals
Question Filters

Security

Change Password
Sign Out

© 2011 PasTest Ltd | About Us | Contact Us | Help

file:///E|/Shakhawan/Endocrinology/220.htm[3/13/2012 7:11:57 PM]


MyPasTest

Main Navigation
Question Browser: MRCP 1
Home
Subscriptions Question Browser Exam Builder Saved Exams
» MRCP 1 Session Progress
• Question Browser
Questions Correct 3
• Timed Test A 40-year-old man is receiving lithium for a bipolar
disorder. He now complains of increased urinary frequency Questions Incorrect 2
• Mock Exam
and nocturia. What is the most likely cause for his Questions Total 5
• Past Papers symptoms?
Questions Percentage 60 %
• Random Questions
• My Performance Deficiency of vasopressin More
• Media Bank Reduced creatinine clearance

• New Multimedia Onset of glomerulonephritis

Online Extras Development of resistance to Your answer


vasopressin
Library
Excessive water intake
Community
Help
PasTest Store
Long-term treatment with lithium causes nephrogenic
My Account diabetes insipidus. This is due to the development of
resistance to vasopressin in the renal tubules.
Profile Symptoms include polyuria, polydipsia, nocturia and
enuresis.
Newsletters
My Career and Exams
Order History 3418 Reference: Normal Values
Learning Goals
Click to open/closeHaematology
Question Filters Click to open/closeImmunoglobulins
Click to open/closeBiochemistry
Click to open/closeDiabetes Click to
Security open/closeEndocrinology Click to
open/closeBlood gases Click to
Change Password open/closeCSF
Sign Out

© 2011 PasTest Ltd | About Us | Contact Us | Help

file:///E|/Shakhawan/Endocrinology/220a.htm[3/13/2012 7:11:59 PM]


MyPasTest

Main Navigation
Question Browser: MRCP 1
Home
Subscriptions Question Browser Exam Builder Saved Exams
» MRCP 1 Session Progress
• Question Browser
Questions Correct 3
• Timed Test Exposure to darkness is found to increase melatonin
secretion. What is the most common mechanism by Questions Incorrect 2
• Mock Exam
which this is achieved? Questions Total 5
• Past Papers
Questions Percentage 60 %
• Random Questions Decreased activity of suprachiasmatic nuclei
• My Performance Increased serotonin N-acetyltransferase More
• Media Bank Decreased hydroxyindole-o-methyltransferase
• New Multimedia activity

Online Extras Blockade of noradrenaline release from sympathetic


Reference: Normal Values
nerve terminals
Library
Increased intracellular cAMP in the hypothalamus Click to
Community open/closeHaematology
Help 3419 Click to
open/closeImmunoglobulins
PasTest Store
Click to
open/closeBiochemistry
My Account Click to open/closeDiabetes
Click to
Profile open/closeEndocrinology
Click to open/closeBlood
Newsletters gases Click to
My Career and Exams open/closeCSF
Order History
Learning Goals
Question Filters

Security

Change Password
Sign Out

© 2011 PasTest Ltd | About Us | Contact Us | Help

file:///E|/Shakhawan/Endocrinology/221.htm[3/13/2012 7:12:00 PM]


MyPasTest

Main Navigation
Question Browser: MRCP 1
Home
Subscriptions Question Browser Exam Builder Saved Exams
» MRCP 1
• Question Browser
• Timed Test Exposure to darkness is found to increase melatonin secretion. What is the most
• Mock Exam common mechanism by which this is achieved?
• Past Papers
Decreased activity of suprachiasmatic nuclei
• Random Questions
Increased serotonin N-acetyltransferase Correct answer
• My Performance
Decreased hydroxyindole-o-methyltransferase activity
• Media Bank
Blockade of noradrenaline release from sympathetic nerve
• New Multimedia terminals
Online Extras Increased intracellular cAMP in the hypothalamus Your answer
Library
Community
Help
PasTest Store Melatonin is synthesised and secreted by the pineal gland from serotonin: N-
acetyltransferase + acetyl-CoA converts serotonin to N-acetyl serotonin, which is
then catalysed by hydroxyindole-o-methyltransferase to melatonin.
My Account
Exposure to darkness causes activation of the hypothalamus by the
Profile retinohypothalamic nerves. This results in increased noradrenaline secretion by the
postganglionic sympathetic nerves (nervi conarii) innervating the pineal gland.
Newsletters
Noradrenaline acts via β-adrenergic receptors in the pineal gland to increase the
My Career and Exams intracellular levels of cAMP, this then causes an increase in N-acetyltransferase
Order History activity.
Learning Goals
Question Filters 3419

Security

Change Password
Sign Out

file:///E|/Shakhawan/Endocrinology/221a.htm[3/13/2012 7:12:01 PM]


MyPasTest

Main Navigation
Question Browser: MRCP 1
Home
Subscriptions Question Browser Exam Builder Saved Exams
» MRCP 1 Session Progress
• Question Browser
Questions Correct 3
• Timed Test A 27-year-old woman presenting with polyuria is found to
have diabetes insipidus. Which part of the nephron is Questions Incorrect 3
• Mock Exam
most affected in this condition? Questions Total 6
• Past Papers
Questions Percentage 50 %
• Random Questions Proximal tubule
• My Performance Cortical and medullary collecting tubules More
• Media Bank Distal convoluted tubule
• New Multimedia Thin ascending limb of Henle
Online Extras Glomerular efferent arterioles Reference: Normal Values
Library
3420 Click to
Community open/closeHaematology
Help Click to
open/closeImmunoglobulins
PasTest Store
Click to
open/closeBiochemistry
My Account Click to open/closeDiabetes
Click to
Profile open/closeEndocrinology
Click to open/closeBlood
Newsletters gases Click to
My Career and Exams open/closeCSF
Order History
Learning Goals
Question Filters

Security

Change Password
Sign Out

© 2011 PasTest Ltd | About Us | Contact Us | Help

file:///E|/Shakhawan/Endocrinology/222.htm[3/13/2012 7:12:03 PM]


MyPasTest

Main Navigation
Question Browser: MRCP 1
Home
Subscriptions Question Browser Exam Builder Saved Exams
» MRCP 1 Session Progress
• Question Browser
Questions Correct 4
• Timed Test A 27-year-old woman presenting with polyuria is found to
have diabetes insipidus. Which part of the nephron is Questions Incorrect 3
• Mock Exam
most affected in this condition? Questions Total 7
• Past Papers
Questions Percentage 57 %
• Random Questions Proximal tubule
• My Performance Cortical and medullary collecting Your answer More
• Media Bank tubules

• New Multimedia Distal convoluted tubule

Online Extras Thin ascending limb of Henle

Library Glomerular efferent arterioles

Community
Help
PasTest Store
Fine-tuning of salt and water balance is achieved in the
distal and collecting tubules under the influence of
My Account aldosterone and antidiuretic hormone (ADH). ADH,
however, plays a central role in urinary concentration
Profile by increasing the water permeability of the normally
impermeable cortical and medullary collecting tubules.
Newsletters
In the absence of ADH, little water is reabsorbed in the
My Career and Exams collecting tubules and dilute urine is excreted.
Order History Reference: Normal Values
Learning Goals 3420 Click to open/closeHaematology
Question Filters Click to open/closeImmunoglobulins
Click to open/closeBiochemistry
Click to open/closeDiabetes Click to
Security open/closeEndocrinology Click to
open/closeBlood gases Click to
Change Password open/closeCSF
Sign Out

© 2011 PasTest Ltd | About Us | Contact Us | Help

file:///E|/Shakhawan/Endocrinology/222a.htm[3/13/2012 7:12:04 PM]


MyPasTest

Main Navigation
Question Browser: MRCP 1
Home
Subscriptions Question Browser Exam Builder Saved Exams
» MRCP 1 Session Progress
• Question Browser
Questions Correct 0
• Timed Test A 45-year-old diabetic man on oral hypoglycaemic drugs is
found to have hyponatraemia. Which of the following Questions Incorrect 0
• Mock Exam
drugs is most likely to cause this complication? Questions Total 0
• Past Papers
Questions Percentage 0%
• Random Questions Chlorpropamide
• My Performance Tolbutamide More
• Media Bank Glibenclamide
• New Multimedia Gliclazide
Online Extras Glimepiride Reference: Normal Values
Library
3421 Click to
Community open/closeHaematology
Help Click to
open/closeImmunoglobulins
PasTest Store
Click to
open/closeBiochemistry
My Account Click to open/closeDiabetes
Click to
Profile open/closeEndocrinology
Click to open/closeBlood
Newsletters gases Click to
My Career and Exams open/closeCSF
Order History
Learning Goals
Question Filters

Security

Change Password
Sign Out

© 2011 PasTest Ltd | About Us | Contact Us | Help

file:///E|/Shakhawan/Endocrinology/223.htm[3/13/2012 7:12:06 PM]


MyPasTest

Main Navigation
Question Browser: MRCP 1
Home
Subscriptions Question Browser Exam Builder Saved Exams
» MRCP 1 Session Progress
• Question Browser
Questions Correct 1
• Timed Test A 45-year-old diabetic man on oral hypoglycaemic drugs is
found to have hyponatraemia. Which of the following Questions Incorrect 0
• Mock Exam
drugs is most likely to cause this complication? Questions Total 1
• Past Papers
Questions Percentage 100 %
• Random Questions Chlorpropamide Your answer
• My Performance Tolbutamide More
• Media Bank Glibenclamide
• New Multimedia Gliclazide
Online Extras Glimepiride
Library
Community
Help
PasTest Store Up to 5% of patients taking chlorpropamide may
develop inappropriate ADH-like syndrome resulting in
dilutional hyponatraemia. This side-effect may also
My Account occur with glibenclamide, glimepiride and glipizide.

Profile
Newsletters 3421

My Career and Exams


Order History Reference: Normal Values
Learning Goals
Click to open/closeHaematology
Question Filters Click to open/closeImmunoglobulins
Click to open/closeBiochemistry
Click to open/closeDiabetes Click to
Security open/closeEndocrinology Click to
open/closeBlood gases Click to
Change Password open/closeCSF
Sign Out

© 2011 PasTest Ltd | About Us | Contact Us | Help

file:///E|/Shakhawan/Endocrinology/223a.htm[3/13/2012 7:12:07 PM]


MyPasTest

Main Navigation
Question Browser: MRCP 1
Home
Subscriptions Question Browser Exam Builder Saved Exams
» MRCP 1
• Question Browser
• Timed Test A 35-year-old woman has been prescribed a sulphonylurea for type-2 maturity-onset
• Mock Exam diabetes mellitus. She complains of severe facial flushing and sweating when she
drinks alcohol. Which of the following is most probably responsible for this
• Past Papers reaction?
• Random Questions
• My Performance Tolbutamide

• Media Bank Gliclazide

• New Multimedia Chlorpropamide

Online Extras Glipizide

Library Glibenclamide

Community 3422
Help
PasTest Store

My Account

Profile
Newsletters
My Career and Exams
Order History
Learning Goals
Question Filters

Security

Change Password
Sign Out

file:///E|/Shakhawan/Endocrinology/224.htm[3/13/2012 7:12:08 PM]


MyPasTest

Main Navigation
Question Browser: MRCP 1
Home
Subscriptions Question Browser Exam Builder Saved Exams
» MRCP 1 Session Progress
• Question Browser
Questions Correct 2
• Timed Test A 35-year-old woman has been prescribed a sulphonylurea
for type-2 maturity-onset diabetes mellitus. She complains Questions Incorrect 0
• Mock Exam
of severe facial flushing and sweating when she drinks Questions Total 2
• Past Papers alcohol. Which of the following is most probably
Questions Percentage 100 %
• Random Questions responsible for this reaction?
• My Performance More
Tolbutamide
• Media Bank
Gliclazide
• New Multimedia
Chlorpropamide Your answer
Online Extras
Glipizide
Library
Glibenclamide
Community
Help
PasTest Store
Of the sulphonylureas, chlorpropamide is the main one
My Account associated with facial flushing following alcohol intake,
although a lesser reaction may be seen with other SUs.
Profile This disulfiram-like reaction is also seen in those who
are on treatment with metronidazole.
Newsletters
My Career and Exams
Order History 3422 Reference: Normal Values
Learning Goals
Click to open/closeHaematology
Question Filters Click to open/closeImmunoglobulins
Click to open/closeBiochemistry
Click to open/closeDiabetes Click to
Security open/closeEndocrinology Click to
open/closeBlood gases Click to
Change Password open/closeCSF
Sign Out

© 2011 PasTest Ltd | About Us | Contact Us | Help

file:///E|/Shakhawan/Endocrinology/224a.htm[3/13/2012 7:12:10 PM]


MyPasTest

Main Navigation
Question Browser: MRCP 1
Home
Subscriptions Question Browser Exam Builder Saved Exams
» MRCP 1 Session Progress
• Question Browser
Questions Correct 2
• Timed Test A 29-year-old woman with thyrotoxicosis presents with a
6-week history of amenorrhoea. Laboratory investigations Questions Incorrect 0
• Mock Exam
confirm an early pregnancy. Which of the following Questions Total 2
• Past Papers drugs would be most appropriate in this condition?
Questions Percentage 100 %
• Random Questions
• My Performance Carbimazole More
• Media Bank Radioactive iodine

• New Multimedia Propranolol

Online Extras Propylthiouracil


Reference: Normal Values
Library Methimazole
Click to
Community 3423 open/closeHaematology
Help Click to
open/closeImmunoglobulins
PasTest Store
Click to
open/closeBiochemistry
My Account Click to open/closeDiabetes
Click to
Profile open/closeEndocrinology
Click to open/closeBlood
Newsletters gases Click to
My Career and Exams open/closeCSF
Order History
Learning Goals
Question Filters

Security

Change Password
Sign Out

© 2011 PasTest Ltd | About Us | Contact Us | Help

file:///E|/Shakhawan/Endocrinology/225.htm[3/13/2012 7:12:11 PM]


MyPasTest

Main Navigation
Question Browser: MRCP 1
Home
Subscriptions Question Browser Exam Builder Saved Exams
» MRCP 1 Session Progress
• Question Browser
Questions Correct 3
• Timed Test A 29-year-old woman with thyrotoxicosis presents with a
6-week history of amenorrhoea. Laboratory investigations Questions Incorrect 0
• Mock Exam
confirm an early pregnancy. Which of the following Questions Total 3
• Past Papers drugs would be most appropriate in this condition?
Questions Percentage 100 %
• Random Questions
• My Performance Carbimazole More
• Media Bank Radioactive iodine

• New Multimedia Propranolol

Online Extras Propylthiouracil Your answer

Library Methimazole

Community
Help
PasTest Store
Propylthiouracil is the drug of choice in pregnancy as its
greater protein binding reduces the risk of
My Account transplacental passage. Methimazole is the active
metabolite of carbimazole and is used in the USA.
Profile Radioactive iodine is contraindicated in pregnancy as it
can destroy the fetal thyroid gland.
Newsletters
My Career and Exams
Order History 3423 Reference: Normal Values
Learning Goals
Click to open/closeHaematology
Question Filters Click to open/closeImmunoglobulins
Click to open/closeBiochemistry
Click to open/closeDiabetes Click to
Security open/closeEndocrinology Click to
open/closeBlood gases Click to
Change Password open/closeCSF
Sign Out

© 2011 PasTest Ltd | About Us | Contact Us | Help

file:///E|/Shakhawan/Endocrinology/225a.htm[3/13/2012 7:12:13 PM]


MyPasTest

Main Navigation
Question Browser: MRCP 1
Home
Subscriptions Question Browser Exam Builder Saved Exams
» MRCP 1 Session Progress
• Question Browser
Questions Correct 3
• Timed Test A 40-year-old man is suffering from type-2 diabetes
mellitus and hypertension. Questions Incorrect 0
• Mock Exam
Questions Total 3
• Past Papers Which of the following antihypertensive drugs is
most likely to cause hyperglycaemia if prescribed for Questions Percentage 100 %
• Random Questions
this patient? More
• My Performance
• Media Bank Losartan
• New Multimedia Lisinopril
Online Extras Hydrochlorothiazide Reference: Normal Values
Library Diltiazem
Click to
Community Amlodipine open/closeHaematology
Help Click to
3424 open/closeImmunoglobulins
PasTest Store
Click to
open/closeBiochemistry
My Account Click to open/closeDiabetes
Click to
Profile open/closeEndocrinology
Click to open/closeBlood
Newsletters gases Click to
My Career and Exams open/closeCSF
Order History
Learning Goals
Question Filters

Security

Change Password
Sign Out

© 2011 PasTest Ltd | About Us | Contact Us | Help

file:///E|/Shakhawan/Endocrinology/226.htm[3/13/2012 7:12:14 PM]


MyPasTest

Main Navigation
Question Browser: MRCP 1
Home
Subscriptions Question Browser Exam Builder Saved Exams
» MRCP 1
• Question Browser
• Timed Test A 40-year-old man is suffering from type-2 diabetes mellitus and hypertension.
• Mock Exam
Which of the following antihypertensive drugs is most likely to cause
• Past Papers hyperglycaemia if prescribed for this patient?
• Random Questions
Losartan
• My Performance
Lisinopril
• Media Bank
Hydrochlorothiazide Your answer
• New Multimedia
Diltiazem
Online Extras
Amlodipine
Library
Community
Help
PasTest Store
Thiazide diuretics are associated with impaired glucose tolerance. They are usually
not recommended for the treatment of hypertension in patients with diabetes
My Account mellitus.

Profile
3424
Newsletters
My Career and Exams
Order History
Learning Goals
Question Filters

Security

Change Password
Sign Out

file:///E|/Shakhawan/Endocrinology/226a.htm[3/13/2012 7:12:16 PM]


MyPasTest

Main Navigation
Question Browser: MRCP 1
Home
Subscriptions Question Browser Exam Builder Saved Exams
» MRCP 1 Session Progress
• Question Browser
Questions Correct 4
• Timed Test A 39-year-old man with Grave’s disease is being
considered for subtotal thyroidectomy. What is the most Questions Incorrect 0
• Mock Exam
common postoperative complication following this Questions Total 4
• Past Papers operation?
Questions Percentage 100 %
• Random Questions
• My Performance Laryngeal nerve palsy More
• Media Bank Hypoparathyroidism

• New Multimedia Recurrent hyperthyroidism

Online Extras Tracheal compression


Reference: Normal Values
Library Hypothyroidism
Click to
Community 3425 open/closeHaematology
Help Click to
open/closeImmunoglobulins
PasTest Store
Click to
open/closeBiochemistry
My Account Click to open/closeDiabetes
Click to
Profile open/closeEndocrinology
Click to open/closeBlood
Newsletters gases Click to
My Career and Exams open/closeCSF
Order History
Learning Goals
Question Filters

Security

Change Password
Sign Out

© 2011 PasTest Ltd | About Us | Contact Us | Help

file:///E|/Shakhawan/Endocrinology/227.htm[3/13/2012 7:12:17 PM]


MyPasTest

Main Navigation
Question Browser: MRCP 1
Home
Subscriptions Question Browser Exam Builder Saved Exams
» MRCP 1 Session Progress
• Question Browser
Questions Correct 5
• Timed Test A 39-year-old man with Grave’s disease is being
considered for subtotal thyroidectomy. What is the most Questions Incorrect 0
• Mock Exam
common postoperative complication following this Questions Total 5
• Past Papers operation?
Questions Percentage 100 %
• Random Questions
• My Performance Laryngeal nerve palsy More
• Media Bank Hypoparathyroidism

• New Multimedia Recurrent hyperthyroidism

Online Extras Tracheal compression

Library Hypothyroidism Your answer

Community
Help
PasTest Store
Hypothyroidism occurs in about 10% of such patients
within 1 year of surgery. This percentage increases with
My Account time and a high proportion of patients become
hypothyroid in the long term. Laryngeal nerve palsy
Profile occurs in 1% of cases. Transient hypocalcaemia occurs
in up to 10% of patients, but permanent
Newsletters
hypoparathyroidism is seen in less than 1% of patients.
My Career and Exams Recurrent hyperthyroidism occurs in 1–3% of cases
Order History within 1 year and 1% per year thereafter. Early Reference: Normal Values
postoperative bleeding may cause tracheal compression.
Learning Goals This is a rare complication. Click to open/closeHaematology
Question Filters Click to open/closeImmunoglobulins
Click to open/closeBiochemistry
3425 Click to open/closeDiabetes Click to
Security open/closeEndocrinology Click to
open/closeBlood gases Click to
Change Password open/closeCSF
Sign Out

© 2011 PasTest Ltd | About Us | Contact Us | Help

file:///E|/Shakhawan/Endocrinology/227a.htm[3/13/2012 7:12:18 PM]


MyPasTest

Main Navigation
Question Browser: MRCP 1
Home
Subscriptions Question Browser Exam Builder Saved Exams
» MRCP 1 Session Progress
• Question Browser
Questions Correct 5
• Timed Test A 14-year-old boy presents with poor development of
secondary sex characteristics, colour blindness and a Questions Incorrect 0
• Mock Exam
decreased sense of smell. On examination, his testes are Questions Total 5
• Past Papers located in the scrotum and are small and soft. What is the
Questions Percentage 100 %
• Random Questions most probable diagnosis?
• My Performance More
Klinefelter’s syndrome
• Media Bank
Kallmann’s syndrome
• New Multimedia
Mumps orchitis
Online Extras Reference: Normal Values
Hyperprolactinaemia
Library
Cryptorchidism Click to
Community open/closeHaematology
Help 3426 Click to
open/closeImmunoglobulins
PasTest Store
Click to
open/closeBiochemistry
My Account Click to open/closeDiabetes
Click to
Profile open/closeEndocrinology
Click to open/closeBlood
Newsletters gases Click to
My Career and Exams open/closeCSF
Order History
Learning Goals
Question Filters

Security

Change Password
Sign Out

© 2011 PasTest Ltd | About Us | Contact Us | Help

file:///E|/Shakhawan/Endocrinology/228.htm[3/13/2012 7:12:20 PM]


MyPasTest

Main Navigation
Question Browser: MRCP 1
Home
Subscriptions Question Browser Exam Builder Saved Exams
» MRCP 1 Session Progress
• Question Browser
Questions Correct 6
• Timed Test A 14-year-old boy presents with poor development of
secondary sex characteristics, colour blindness and a Questions Incorrect 0
• Mock Exam
decreased sense of smell. On examination, his testes are Questions Total 6
• Past Papers located in the scrotum and are small and soft. What is the
Questions Percentage 100 %
• Random Questions most probable diagnosis?
• My Performance More
Klinefelter’s syndrome
• Media Bank
Kallmann’s syndrome Your answer
• New Multimedia
Mumps orchitis
Online Extras
Hyperprolactinaemia
Library
Cryptorchidism
Community
Help
PasTest Store
Kallmann’s syndrome is due to isolated gonadotrophin-
My Account releasing hormone (GnRH) deficiency. It is often familial
and X-linked. Klinefelter’s syndrome is also associated
Profile with hypogonadism, but the other clinical features are
not seen. Cryptorchidism is ruled out by the presence of
Newsletters
testes in the scrotum. The features are not suggestive
My Career and Exams of mumps orchitis or hyperprolactinaemia.
Order History Reference: Normal Values
Learning Goals 3426 Click to open/closeHaematology
Question Filters Click to open/closeImmunoglobulins
Click to open/closeBiochemistry
Click to open/closeDiabetes Click to
Security open/closeEndocrinology Click to
open/closeBlood gases Click to
Change Password open/closeCSF
Sign Out

© 2011 PasTest Ltd | About Us | Contact Us | Help

file:///E|/Shakhawan/Endocrinology/228a.htm[3/13/2012 7:12:21 PM]


MyPasTest

Main Navigation
Question Browser: MRCP 1
Home
Subscriptions Question Browser Exam Builder Saved Exams
» MRCP 1 Session Progress
• Question Browser
Questions Correct 6
• Timed Test A 12-year-old girl presents with short stature, webbed
neck and primary amenorrhoea. Given the likely clinical Questions Incorrect 0
• Mock Exam
diagnosis, which hormone replacement is most Questions Total 6
• Past Papers crucial over the longer term?
Questions Percentage 100 %
• Random Questions
• My Performance Growth hormone More
• Media Bank Progesterone

• New Multimedia Oestrogen

Online Extras Pulsatile GnRH


Reference: Normal Values
Library Luteinising hormone
Click to
Community 3427 open/closeHaematology
Help Click to
open/closeImmunoglobulins
PasTest Store
Click to
open/closeBiochemistry
My Account Click to open/closeDiabetes
Click to
Profile open/closeEndocrinology
Click to open/closeBlood
Newsletters gases Click to
My Career and Exams open/closeCSF
Order History
Learning Goals
Question Filters

Security

Change Password
Sign Out

© 2011 PasTest Ltd | About Us | Contact Us | Help

file:///E|/Shakhawan/Endocrinology/229.htm[3/13/2012 7:12:22 PM]


MyPasTest

Main Navigation
Question Browser: MRCP 1
Home
Subscriptions Question Browser Exam Builder Saved Exams
» MRCP 1 Session Progress
• Question Browser
Questions Correct 7
• Timed Test A 12-year-old girl presents with short stature, webbed
neck and primary amenorrhoea. Given the likely clinical Questions Incorrect 0
• Mock Exam
diagnosis, which hormone replacement is most Questions Total 7
• Past Papers crucial over the longer term?
Questions Percentage 100 %
• Random Questions
• My Performance Growth hormone More
• Media Bank Progesterone

• New Multimedia Oestrogen Your answer

Online Extras Pulsatile GnRH

Library Luteinising hormone

Community
Help
PasTest Store
This girl most probably has Turner’s syndrome, which is
most commonly associated with absence of one X
My Account chromosome (XO pattern). Characteristic features in
this condition include streak ovaries and absent internal
Profile genitalia. Oestrogen administration will develop female
secondary sex characters and life-long therapy will be
Newsletters
required. Growth hormone may be useful for a limited
My Career and Exams period to increase her height. As the ovaries are absent,
Order History pulsatile GnRH and luteinising hormone would be of no Reference: Normal Values
use. Progesterone has no role to play in this condition.
Learning Goals
Click to open/closeHaematology
Question Filters Click to open/closeImmunoglobulins
3427 Click to open/closeBiochemistry
Click to open/closeDiabetes Click to
Security open/closeEndocrinology Click to
open/closeBlood gases Click to
Change Password open/closeCSF
Sign Out

© 2011 PasTest Ltd | About Us | Contact Us | Help

file:///E|/Shakhawan/Endocrinology/229a.htm[3/13/2012 7:12:24 PM]


MyPasTest

Main Navigation
Question Browser: MRCP 1
Home
Subscriptions Question Browser Exam Builder Saved Exams
» MRCP 1 Session Progress
• Question Browser
Questions Correct 7
• Timed Test A 15-year-old girl presents with primary amenorrhoea. She
represents her school in athletics and is now training for the Questions Incorrect 0
• Mock Exam
national championships. Which of the following would Questions Total 7
• Past Papers be the most appropriate treatment for her?
Questions Percentage 100 %
• Random Questions
• My Performance Oestrogen More
• Media Bank Gonadotrophin-releasing hormone

• New Multimedia Gonadotrophins

Online Extras Adequate diet and observation


Reference: Normal Values
Library Oestrogen and progesterone
Click to
Community 3428 open/closeHaematology
Help Click to
open/closeImmunoglobulins
PasTest Store
Click to
open/closeBiochemistry
My Account Click to open/closeDiabetes
Click to
Profile open/closeEndocrinology
Click to open/closeBlood
Newsletters gases Click to
My Career and Exams open/closeCSF
Order History
Learning Goals
Question Filters

Security

Change Password
Sign Out

© 2011 PasTest Ltd | About Us | Contact Us | Help

file:///E|/Shakhawan/Endocrinology/230.htm[3/13/2012 7:12:25 PM]


MyPasTest

Main Navigation
Question Browser: MRCP 1
Home
Subscriptions Question Browser Exam Builder Saved Exams
» MRCP 1 Session Progress
• Question Browser
Questions Correct 8
• Timed Test A 15-year-old girl presents with primary amenorrhoea. She
represents her school in athletics and is now training for the Questions Incorrect 0
• Mock Exam
national championships. Which of the following would Questions Total 8
• Past Papers be the most appropriate treatment for her?
Questions Percentage 100 %
• Random Questions
• My Performance Oestrogen More
• Media Bank Gonadotrophin-releasing hormone

• New Multimedia Gonadotrophins

Online Extras Adequate diet and observation Your answer

Library Oestrogen and progesterone

Community
Help
PasTest Store
Menarche may be delayed in girls who are involved in
strenuous physical activity. Usually no treatment is
My Account required, as the majority would menstruate by the age
of 16. Provision of an adequate diet and the relief of
Profile stress would be the most appropriate treatment in this
case.
Newsletters
My Career and Exams
Order History 3428 Reference: Normal Values
Learning Goals
Click to open/closeHaematology
Question Filters Click to open/closeImmunoglobulins
Click to open/closeBiochemistry
Click to open/closeDiabetes Click to
Security open/closeEndocrinology Click to
open/closeBlood gases Click to
Change Password open/closeCSF
Sign Out

© 2011 PasTest Ltd | About Us | Contact Us | Help

file:///E|/Shakhawan/Endocrinology/230a.htm[3/13/2012 7:12:27 PM]


MyPasTest

Main Navigation
Question Browser: MRCP 1
Home
Subscriptions Question Browser Exam Builder Saved Exams
» MRCP 1 Session Progress
• Question Browser
Questions Correct 0
• Timed Test A 45-year-old woman presents with heavy periods. A fall
in which of the following hormones is most Questions Incorrect 0
• Mock Exam
commonly associated with shedding of the Questions Total 0
• Past Papers endometrium during menstruation?
Questions Percentage 0%
• Random Questions
• My Performance Luteinising hormone More
• Media Bank Follicle-stimulating hormone

• New Multimedia Gonadotrophin-releasing hormone

Online Extras Progesterone


Reference: Normal Values
Library Estrone
Click to
Community 3429 open/closeHaematology
Help Click to
open/closeImmunoglobulins
PasTest Store
Click to
open/closeBiochemistry
My Account Click to open/closeDiabetes
Click to
Profile open/closeEndocrinology
Click to open/closeBlood
Newsletters gases Click to
My Career and Exams open/closeCSF
Order History
Learning Goals
Question Filters

Security

Change Password
Sign Out

© 2011 PasTest Ltd | About Us | Contact Us | Help

file:///E|/Shakhawan/Endocrinology/231.htm[3/13/2012 7:12:28 PM]


MyPasTest

Main Navigation
Question Browser: MRCP 1
Home
Subscriptions Question Browser Exam Builder Saved Exams
» MRCP 1 Session Progress
• Question Browser
Questions Correct 1
• Timed Test A 45-year-old woman presents with heavy periods. A fall
in which of the following hormones is most Questions Incorrect 0
• Mock Exam
commonly associated with shedding of the Questions Total 1
• Past Papers endometrium during menstruation?
Questions Percentage 100 %
• Random Questions
• My Performance Luteinising hormone More
• Media Bank Follicle-stimulating hormone

• New Multimedia Gonadotrophin-releasing hormone

Online Extras Progesterone Your answer

Library Estrone

Community
Help
PasTest Store
Involution of the corpus luteum results in a decrease in
progesterone and estradiol secretion. This occurs
My Account around day 24 of a 28-day menstrual cycle. The loss of
hormonal support causes endometrial shedding. LH,
Profile FSH, GnRH and estrone are not involved in this process.
Newsletters
My Career and Exams 3429
Order History Reference: Normal Values
Learning Goals
Click to open/closeHaematology
Question Filters Click to open/closeImmunoglobulins
Click to open/closeBiochemistry
Click to open/closeDiabetes
Security

Change Password
Sign Out

file:///E|/Shakhawan/Endocrinology/231a.htm[3/13/2012 7:12:30 PM]


MyPasTest

Main Navigation
Question Browser: MRCP 1
Home
Subscriptions Question Browser Exam Builder Saved Exams
» MRCP 1 Session Progress
• Question Browser
Questions Correct 1
• Timed Test A 30-year-old man presents complaining of headaches and
visual field defects. On examination he has clinical features Questions Incorrect 0
• Mock Exam
of hypopituitarism, eg malaise, cold intolerance, mild Questions Total 1
• Past Papers hypotension and hyponatraemia. Blood tests reveal a
Questions Percentage 100 %
• Random Questions testosterone level of 8 nmol/l (9–35). An MRI scan shows a
12-mm pituitary tumour. More
• My Performance
What is the most probable diagnosis?
• Media Bank
• New Multimedia Somatotroph adenoma
Online Extras Prolactinoma Reference: Normal
Library Basophil adenoma Values
Community Non-functioning pituitary tumour
Click to
Help Craniopharyngioma open/closeHaematology
PasTest Store 3430

My Account

Profile
Newsletters
My Career and Exams
Order History
Learning Goals
Question Filters

Security

Change Password
Sign Out

file:///E|/Shakhawan/Endocrinology/232.htm[3/13/2012 7:12:31 PM]


MyPasTest

Main Navigation
Question Browser: MRCP 1
Home
Subscriptions Question Browser Exam Builder Saved Exams
» MRCP 1
• Question Browser
• Timed Test A 30-year-old man presents complaining of headaches and visual field defects. On
• Mock Exam examination he has clinical features of hypopituitarism, eg malaise, cold intolerance,
mild hypotension and hyponatraemia. Blood tests reveal a testosterone level of 8
• Past Papers nmol/l (9–35). An MRI scan shows a 12-mm pituitary tumour.
• Random Questions What is the most probable diagnosis?
• My Performance
Somatotroph adenoma
• Media Bank
Prolactinoma Correct answer
• New Multimedia
Basophil adenoma
Online Extras
Non-functioning pituitary tumour Your answer
Library
Craniopharyngioma
Community
Help
PasTest Store
Prolactinomas cause hyperprolactinaemia. Large tumours (macroprolactinomas)
My Account are over 10 mm in size and may cause headaches, visual field defects and
hypopituitarism. Hypogonadism occurs as prolactin inhibits gonadotrophin-
Profile releasing hormone and hence gonadotrophin secretion. Somatotroph adenomas
result in growth hormone excess and acromegaly, while basophil adenomas cause
Newsletters
Cushing’s disease. Craniopharyngiomas usually occur in those below 20 years of
My Career and Exams age and are often very large and cystic. Non-functioning tumours are also large (>
Order History 10 mm). Hypogonadism, however, is not a feature of this condition.

Learning Goals
Question Filters 3430

Security

Change Password
Sign Out

file:///E|/Shakhawan/Endocrinology/232a.htm[3/13/2012 7:12:33 PM]


MyPasTest

Main Navigation
Question Browser: MRCP 1
Home
Subscriptions Question Browser Exam Builder Saved Exams
» MRCP 1 Session Progress
• Question Browser
Questions Correct 1
• Timed Test A 50-year-old diabetic man presents with malaise and loss
of libido. On examination there is evidence of Questions Incorrect 1
• Mock Exam
hepatomegaly. Blood tests show Hb 14.1 g/dl (13–18), Questions Total 2
• Past Papers fasting glucose 8 mmol/l (3–6), serum iron 45 µmol/l (14–
Questions Percentage 50 %
• Random Questions 29) and serum ferritin 250,000 µmol/l (15–200).
• My Performance More
What would be the most effective treatment for his
• Media Bank loss of libido?
• New Multimedia
Testosterone replacement
Online Extras Reference: Normal Values
Desferrioxamine
Library
Venesection Click to
Community open/closeHaematology
Insulin therapy
Help Click to
Liver transplant open/closeImmunoglobulins
PasTest Store
Click to
3432 open/closeBiochemistry
My Account Click to open/closeDiabetes
Click to
Profile open/closeEndocrinology
Click to open/closeBlood
Newsletters gases Click to
My Career and Exams open/closeCSF
Order History
Learning Goals
Question Filters

Security

Change Password
Sign Out

© 2011 PasTest Ltd | About Us | Contact Us | Help

file:///E|/Shakhawan/Endocrinology/233.htm[3/13/2012 7:12:34 PM]


MyPasTest

Main Navigation
Question Browser: MRCP 1
Home
Subscriptions Question Browser Exam Builder Saved Exams
» MRCP 1
• Question Browser
• Timed Test A 50-year-old diabetic man presents with malaise and loss of libido. On examination
• Mock Exam there is evidence of hepatomegaly. Blood tests show Hb 14.1 g/dl (13–18), fasting
glucose 8 mmol/l (3–6), serum iron 45 µmol/l (14–29) and serum ferritin 250,000
• Past Papers µmol/l (15–200).
• Random Questions
What would be the most effective treatment for his loss of libido?
• My Performance
• Media Bank Testosterone replacement Correct answer
• New Multimedia Desferrioxamine
Online Extras Venesection Your answer
Library Insulin therapy
Community Liver transplant
Help
PasTest Store

My Account This patient most probably has hereditary haemochromatosis. Although


venesection is the treatment of choice, this will not correct testicular atrophy or
Profile hypopituitarism, either or both of which may contribute to a loss of libido.
Testosterone replacement is often helpful in these patients. Desferrioxamine is
Newsletters indicated in rare cases who cannot tolerate venesection (because of severe cardiac
My Career and Exams disease or anaemia). Both insulin treatment and liver transplantation would not be
helpful in restoring libido.
Order History
Learning Goals
Question Filters 3432

Security

Change Password
Sign Out

file:///E|/Shakhawan/Endocrinology/233a.htm[3/13/2012 7:12:36 PM]


MyPasTest

Main Navigation
Question Browser: MRCP 1
Home
Subscriptions Question Browser Exam Builder Saved Exams
» MRCP 1 Session Progress
• Question Browser
Questions Correct 1
• Timed Test A 13-year-old boy is brought to your clinic with a complaint
of delayed pubertal changes. While examining the Questions Incorrect 2
• Mock Exam
patient which of the following features is most likely Questions Total 3
• Past Papers to indicate that pubertal change may have
Questions Percentage 33 %
• Random Questions commenced?
• My Performance More
Increase in testicular volume
• Media Bank
Growth of external genitalia
• New Multimedia
Growth of pubic hair
Online Extras Reference: Normal Values
Change in voice
Library
Appearance of axillary hair Click to
Community open/closeHaematology
Help 3433 Click to
open/closeImmunoglobulins
PasTest Store
Click to
open/closeBiochemistry
My Account Click to open/closeDiabetes
Click to
Profile open/closeEndocrinology
Click to open/closeBlood
Newsletters gases Click to
My Career and Exams open/closeCSF
Order History
Learning Goals
Question Filters

Security

Change Password
Sign Out

© 2011 PasTest Ltd | About Us | Contact Us | Help

file:///E|/Shakhawan/Endocrinology/234.htm[3/13/2012 7:12:37 PM]


MyPasTest

Main Navigation
Question Browser: MRCP 1
Home
Subscriptions Question Browser Exam Builder Saved Exams
» MRCP 1 Session Progress
• Question Browser
Questions Correct 2
• Timed Test A 13-year-old boy is brought to your clinic with a complaint
of delayed pubertal changes. While examining the Questions Incorrect 2
• Mock Exam
patient which of the following features is most likely Questions Total 4
• Past Papers to indicate that pubertal change may have
Questions Percentage 50 %
• Random Questions commenced?
• My Performance More
Increase in testicular volume Your answer
• Media Bank
Growth of external genitalia
• New Multimedia
Growth of pubic hair
Online Extras
Change in voice
Library
Appearance of axillary hair
Community
Help
PasTest Store
The first clinical sign of the onset of puberty in boys is
My Account an increase in testicular volume (> 5 ml). This is
followed by growth of external genitalia, deepening of
Profile the voice and pubic and axillary hair growth.
Newsletters
My Career and Exams 3433
Order History Reference: Normal Values
Learning Goals
Click to open/closeHaematology
Question Filters Click to open/closeImmunoglobulins
Click to open/closeBiochemistry
Click to open/closeDiabetes Click to
Security open/closeEndocrinology Click to
open/closeBlood gases Click to
Change Password open/closeCSF
Sign Out

© 2011 PasTest Ltd | About Us | Contact Us | Help

file:///E|/Shakhawan/Endocrinology/234a.htm[3/13/2012 7:12:39 PM]


MyPasTest

Main Navigation
Question Browser: MRCP 1
Home
Subscriptions Question Browser Exam Builder Saved Exams
» MRCP 1 Session Progress
• Question Browser
Questions Correct 2
• Timed Test A 49-year-old overweight police constable presents with
amenorrhoea of 6 months’ duration. There is no family Questions Incorrect 2
• Mock Exam
history of osteoporosis or fracture. She does not smoke and Questions Total 4
• Past Papers is a teetotaler. Blood tests: FSH 35 IU/l (postmenopausal
Questions Percentage 50 %
• Random Questions level > 30), LH 57 IU/l (postmenopausal level > 30). Bone
density measurements with a DXA scan are within normal More
• My Performance
limits. What is the chief reason that this woman has
• Media Bank not developed osteoporosis?
• New Multimedia
Continued physical activity
Online Extras Reference: Normal Values
Absence of positive family history
Library
Non-smoker Click to
Community open/closeHaematology
Overweight condition
Help Click to
Non-consumption of alcohol open/closeImmunoglobulins
PasTest Store
Click to
3434 open/closeBiochemistry
My Account Click to open/closeDiabetes
Click to
Profile open/closeEndocrinology
Click to open/closeBlood
Newsletters gases Click to
My Career and Exams open/closeCSF
Order History
Learning Goals
Question Filters

Security

Change Password
Sign Out

© 2011 PasTest Ltd | About Us | Contact Us | Help

file:///E|/Shakhawan/Endocrinology/235.htm[3/13/2012 7:12:40 PM]


MyPasTest

Main Navigation
Question Browser: MRCP 1
Home
Subscriptions Question Browser Exam Builder Saved Exams
» MRCP 1 Session Progress
• Question Browser
Questions Correct 3
• Timed Test A 49-year-old overweight police constable presents with
amenorrhoea of 6 months’ duration. There is no family Questions Incorrect 2
• Mock Exam
history of osteoporosis or fracture. She does not smoke and Questions Total 5
• Past Papers is a teetotaler. Blood tests: FSH 35 IU/l (postmenopausal
Questions Percentage 60 %
• Random Questions level > 30), LH 57 IU/l (postmenopausal level > 30). Bone
density measurements with a DXA scan are within normal More
• My Performance
limits. What is the chief reason that this woman has
• Media Bank not developed osteoporosis?
• New Multimedia
Continued physical activity
Online Extras
Absence of positive family history
Library
Non-smoker
Community
Overweight condition Your answer
Help
Non-consumption of alcohol
PasTest Store

My Account

Profile All the options contribute to a decreased incidence of


osteoporosis. However, the fact that she is overweight
Newsletters
suggests that her estrone levels are high. Estrone is the
My Career and Exams main sex hormone produced after the menopause.
Order History Ovarian thecal androgens are converted to estrone in Reference: Normal Values
adipose tissue by the enzyme aromatase. High levels of
Learning Goals estrone in this woman may thus be the main factor Click to open/closeHaematology
Question Filters preventing the occurrence of bone resorption and Click to open/closeImmunoglobulins
osteoporosis. Click to open/closeBiochemistry
Click to open/closeDiabetes Click to
Security open/closeEndocrinology Click to
3434 open/closeBlood gases Click to
Change Password open/closeCSF
Sign Out

© 2011 PasTest Ltd | About Us | Contact Us | Help

file:///E|/Shakhawan/Endocrinology/235a.htm[3/13/2012 7:12:42 PM]


MyPasTest

Main Navigation
Question Browser: MRCP 1
Home
Subscriptions Question Browser Exam Builder Saved Exams
» MRCP 1 Session Progress
• Question Browser
Questions Correct 3
• Timed Test A 40-year-old man presents with headaches, excessive
sweating, galactorrhoea, polyuria and polydipsia. Visual Questions Incorrect 2
• Mock Exam
field examination reveals bitemporal hemianopia. His blood Questions Total 5
• Past Papers pressure is 160/110 mmHg. Blood tests: fasting glucose 9
Questions Percentage 60 %
• Random Questions mmol/l (3-6), prolactin 1200 mU/l (<360).
What is the most probable diagnosis? More
• My Performance
• Media Bank Prolactinoma
• New Multimedia Diabetes mellitus
Online Extras Acromegaly Reference: Normal Values
Library Cushing’s disease
Click to
Community Craniopharyngioma open/closeHaematology
Help Click to
3435 open/closeImmunoglobulins
PasTest Store
Click to
open/closeBiochemistry
My Account Click to open/closeDiabetes
Click to
Profile open/closeEndocrinology
Click to open/closeBlood
Newsletters gases Click to
My Career and Exams open/closeCSF
Order History
Learning Goals
Question Filters

Security

Change Password
Sign Out

© 2011 PasTest Ltd | About Us | Contact Us | Help

file:///E|/Shakhawan/Endocrinology/236.htm[3/13/2012 7:12:43 PM]


MyPasTest

Main Navigation
Question Browser: MRCP 1
Home
Subscriptions Question Browser Exam Builder Saved Exams
» MRCP 1 Session Progress
• Question Browser
Questions Correct 4
• Timed Test A 40-year-old man presents with headaches, excessive
sweating, galactorrhoea, polyuria and polydipsia. Visual Questions Incorrect 2
• Mock Exam
field examination reveals bitemporal hemianopia. His blood Questions Total 6
• Past Papers pressure is 160/110 mmHg. Blood tests: fasting glucose 9
Questions Percentage 66 %
• Random Questions mmol/l (3-6), prolactin 1200 mU/l (<360).
What is the most probable diagnosis? More
• My Performance
• Media Bank Prolactinoma
• New Multimedia Diabetes mellitus
Online Extras Acromegaly Your answer
Library Cushing’s disease
Community Craniopharyngioma
Help
PasTest Store

My Account This patient most probably has acromegaly. Although


the characteristic changes in appearance may not be
Profile apparent in nearly half the cases, associating the
different clinical features that occur in this condition
Newsletters
usually results in the diagnosis. A pituitary tumour is
My Career and Exams the most common cause. In 30% of patients, mild to
Order History moderate hyperprolactinaemia occurs. In some, the Reference: Normal Values
adenoma secretes both growth hormone and prolactin.
Learning Goals Craniopharyngiomas usually occur in patients under 20 Click to open/closeHaematology
Question Filters years of age. Headaches, visual field defects and growth Click to open/closeImmunoglobulins
failure are usually seen, but galactorrhoea, hypertension Click to open/closeBiochemistry
and diabetes mellitus do not occur. A pituitary adenoma Click to open/closeDiabetes Click to
Security causing Cushing’s disease is often small and is unlikely open/closeEndocrinology Click to
to cause headaches and visual field defects. open/closeBlood gases Click to
Change Password open/closeCSF
Sign Out
3435

© 2011 PasTest Ltd | About Us | Contact Us | Help

file:///E|/Shakhawan/Endocrinology/236a.htm[3/13/2012 7:12:45 PM]


MyPasTest

Main Navigation
Question Browser: MRCP 1
Home
Subscriptions Question Browser Exam Builder Saved Exams
» MRCP 1 Session Progress
• Question Browser
Questions Correct 4
• Timed Test A 15-year-old boy who is 155 cm tall is worried that he
may have stopped growing. Which hormone is chiefly Questions Incorrect 2
• Mock Exam
responsible for epiphyseal fusion and cessation of Questions Total 6
• Past Papers growth?
Questions Percentage 66 %
• Random Questions
• My Performance Growth hormone More
• Media Bank Testosterone

• New Multimedia Somatostatin

Online Extras Oestrogen


Reference: Normal Values
Library Thyroxine
Click to
Community 3436 open/closeHaematology
Help Click to
open/closeImmunoglobulins
PasTest Store
Click to
open/closeBiochemistry
My Account Click to open/closeDiabetes
Click to
Profile open/closeEndocrinology
Click to open/closeBlood
Newsletters gases Click to
My Career and Exams open/closeCSF
Order History
Learning Goals
Question Filters

Security

Change Password
Sign Out

© 2011 PasTest Ltd | About Us | Contact Us | Help

file:///E|/Shakhawan/Endocrinology/237.htm[3/13/2012 7:12:46 PM]


MyPasTest

Main Navigation
Question Browser: MRCP 1
Home
Subscriptions Question Browser Exam Builder Saved Exams
» MRCP 1 Session Progress
• Question Browser
Questions Correct 5
• Timed Test A 15-year-old boy who is 155 cm tall is worried that he
may have stopped growing. Which hormone is chiefly Questions Incorrect 2
• Mock Exam
responsible for epiphyseal fusion and cessation of Questions Total 7
• Past Papers growth?
Questions Percentage 71 %
• Random Questions
• My Performance Growth hormone More
• Media Bank Testosterone

• New Multimedia Somatostatin

Online Extras Oestrogen Your answer

Library Thyroxine

Community
Help
PasTest Store
The growth spurt at puberty is brought about by the
secretion of androgens in the male and oestrogens in
My Account the female. However, it is oestrogens that ultimately
terminate growth by causing the epiphyses in the long
Profile bones to fuse. Thus oestrogens rather than androgens
are responsible for skeletal maturation, epiphyseal
Newsletters
fusion and cessation of growth in males and females. All
My Career and Exams the other hormones are involved in growth alone.
Order History Reference: Normal Values
Learning Goals 3436 Click to open/closeHaematology
Question Filters Click to open/closeImmunoglobulins
Click to open/closeBiochemistry
Click to open/closeDiabetes Click to
Security open/closeEndocrinology Click to
open/closeBlood gases Click to
Change Password open/closeCSF
Sign Out

© 2011 PasTest Ltd | About Us | Contact Us | Help

file:///E|/Shakhawan/Endocrinology/237a.htm[3/13/2012 7:12:47 PM]


MyPasTest

Main Navigation
Question Browser: MRCP 1
Home
Subscriptions Question Browser Exam Builder Saved Exams
» MRCP 1 Session Progress
• Question Browser
Questions Correct 5
• Timed Test A 27-year-old man is found to have oligospermia. Blood
tests: testosterone 30 nmol/l (9–35), LH 10 IU/l (1–10) Questions Incorrect 2
• Mock Exam
and FSH 0.5 IU/l (1–7). Which of the following is most Questions Total 7
• Past Papers likely to be the cause for oligospermia?
Questions Percentage 71 %
• Random Questions
• My Performance Increased inhibin More
• Media Bank Decreased androgen-binding proteins

• New Multimedia Decreased dihydrotestosterone

Online Extras Deficiency of androgen receptors


Reference: Normal Values
Library Hypothalamic disorder
Click to
Community 3437 open/closeHaematology
Help Click to
open/closeImmunoglobulins
PasTest Store
Click to
open/closeBiochemistry
My Account Click to open/closeDiabetes
Click to
Profile open/closeEndocrinology
Click to open/closeBlood
Newsletters gases Click to
My Career and Exams open/closeCSF
Order History
Learning Goals
Question Filters

Security

Change Password
Sign Out

© 2011 PasTest Ltd | About Us | Contact Us | Help

file:///E|/Shakhawan/Endocrinology/238.htm[3/13/2012 7:12:49 PM]


MyPasTest

Main Navigation
Question Browser: MRCP 1
Home
Subscriptions Question Browser Exam Builder Saved Exams
» MRCP 1 Session Progress
• Question Browser
Questions Correct 5
• Timed Test A 27-year-old man is found to have oligospermia. Blood
tests: testosterone 30 nmol/l (9–35), LH 10 IU/l (1–10) Questions Incorrect 3
• Mock Exam
and FSH 0.5 IU/l (1–7). Which of the following is most Questions Total 8
• Past Papers likely to be the cause for oligospermia?
Questions Percentage 62 %
• Random Questions
• My Performance Increased inhibin More
• Media Bank Decreased androgen-binding Correct answer
proteins
• New Multimedia
Decreased dihydrotestosterone Your answer
Online Extras
Deficiency of androgen receptors
Library
Hypothalamic disorder
Community
Help
PasTest Store
This patient has isolated follicle-stimulating hormone
My Account (FSH) deficiency. Normal levels of luteinising hormone
(LH) suggest that there is no hypothalamic disorder.
Profile FSH stimulates the Sertoli cells in the testes to produce
androgen-binding proteins (ABP) and inhibin. ABP is
Newsletters
responsible for binding testosterone and making it
My Career and Exams available for spermatogenesis in the seminiferous
Order History tubules. In the absence of FSH, ABP is not produced Reference: Normal Values
and this causes oligospermia. Testosterone levels are
Learning Goals normal; hence there is no reason to suspect Click to open/closeHaematology
Question Filters dihydrotestosterone (DHT) deficiency. Deficiency of DHT Click to open/closeImmunoglobulins
or androgen receptors would cause testicular Click to open/closeBiochemistry
feminisation. FSH levels would be normal in these Click to open/closeDiabetes Click to
Security cases. open/closeEndocrinology Click to
open/closeBlood gases Click to
Change Password open/closeCSF
Sign Out 3437

© 2011 PasTest Ltd | About Us | Contact Us | Help

file:///E|/Shakhawan/Endocrinology/238a.htm[3/13/2012 7:12:50 PM]


MyPasTest

Main Navigation
Question Browser: MRCP 1
Home
Subscriptions Question Browser Exam Builder Saved Exams
» MRCP 1 Session Progress
• Question Browser
Questions Correct 5
• Timed Test A 16-year-old girl presents with primary amenorrhoea.
General physical examination reveals no abnormality. Questions Incorrect 3
• Mock Exam
Secondary sex characteristics appear well developed. On Questions Total 8
• Past Papers karyotyping, she is found to have XY genotype. What is
Questions Percentage 62 %
• Random Questions the most probable cause for her condition?
• My Performance More
Lack of testosterone formation
• Media Bank
Decreased 5 α-reductase
• New Multimedia
Increased aromatase synthesis
Online Extras Reference: Normal Values
Lack of dihydrotestosterone receptors
Library
Total absence of testes Click to
Community open/closeHaematology
Help 3438 Click to
open/closeImmunoglobulins
PasTest Store
Click to
open/closeBiochemistry
My Account Click to open/closeDiabetes
Click to
Profile open/closeEndocrinology
Click to open/closeBlood
Newsletters gases Click to
My Career and Exams open/closeCSF
Order History
Learning Goals
Question Filters

Security

Change Password
Sign Out

© 2011 PasTest Ltd | About Us | Contact Us | Help

file:///E|/Shakhawan/Endocrinology/239.htm[3/13/2012 7:12:52 PM]


MyPasTest

Main Navigation
Question Browser: MRCP 1
Home
Subscriptions Question Browser Exam Builder Saved Exams
» MRCP 1 Session Progress
• Question Browser
Questions Correct 6
• Timed Test A 16-year-old girl presents with primary amenorrhoea.
General physical examination reveals no abnormality. Questions Incorrect 3
• Mock Exam
Secondary sex characteristics appear well developed. On Questions Total 9
• Past Papers karyotyping, she is found to have XY genotype. What is
Questions Percentage 66 %
• Random Questions the most probable cause for her condition?
• My Performance More
Lack of testosterone formation
• Media Bank
Decreased 5 α-reductase
• New Multimedia
Increased aromatase synthesis
Online Extras
Lack of dihydrotestosterone receptors Your answer
Library
Total absence of testes
Community
Help
PasTest Store

This girl has testicular feminisation, wherein


My Account testosterone is being produced in adequate amounts by
the testes located in the abdomen. The majority of
Profile cases occur due to a lack of androgen receptors to
Newsletters dihydrotestosterone (DHT). DHT is formed from
testosterone by 5 α-reductase and is responsible for the
My Career and Exams
development of male secondary sex characteristics. In
Order History the absence of androgen receptors, the cells are more Reference: Normal Values
Learning Goals receptive to estradiol, which is also produced from
testosterone by the enzyme aromatase. Thus, although Click to open/closeHaematology
Question Filters the genotype is XY, these girls will have well-developed Click to open/closeImmunoglobulins
feminine secondary sex characteristics. Primary Click to open/closeBiochemistry
amenorrhoea is often the only symptom that brings Click to open/closeDiabetes Click to
Security these girls to medical attention. open/closeEndocrinology Click to
open/closeBlood gases Click to
Change Password open/closeCSF
Sign Out 3438

© 2011 PasTest Ltd | About Us | Contact Us | Help

file:///E|/Shakhawan/Endocrinology/239a.htm[3/13/2012 7:12:53 PM]


MyPasTest

Main Navigation
Question Browser: MRCP 1
Home
Subscriptions Question Browser Exam Builder Saved Exams
» MRCP 1 Session Progress
• Question Browser
Questions Correct 6
• Timed Test A 25-year-old woman presents with recurrent episodes of
headaches and sweating. Her mother had renal calculi and Questions Incorrect 3
• Mock Exam
died of a tumour in her neck. On examination a nodule is Questions Total 9
• Past Papers felt in the patient’s neck in the region of the thyroid gland.
Questions Percentage 66 %
• Random Questions A surgeon advises complete thyroidectomy. What is the
most important investigation the surgeon must More
• My Performance
undertake prior to surgery?
• Media Bank
• New Multimedia Serum thyroxine
Online Extras Serum calcium Reference: Normal Values
Library 24-h urine test for 5-hydroxyindoleacetic acid
24-hour urinary catecholamines Click to
Community open/closeHaematology
Help Serum calcitonin Click to
open/closeImmunoglobulins
PasTest Store 3439 Click to
open/closeBiochemistry
My Account Click to open/closeDiabetes
Click to
Profile open/closeEndocrinology
Click to open/closeBlood
Newsletters gases Click to
My Career and Exams open/closeCSF
Order History
Learning Goals
Question Filters

Security

Change Password
Sign Out

© 2011 PasTest Ltd | About Us | Contact Us | Help

file:///E|/Shakhawan/Endocrinology/240.htm[3/13/2012 7:12:55 PM]


MyPasTest

Main Navigation
Question Browser: MRCP 1
Home
Subscriptions Question Browser Exam Builder Saved Exams
» MRCP 1 Session Progress
• Question Browser
Questions Correct 7
• Timed Test A 25-year-old woman presents with recurrent episodes of
headaches and sweating. Her mother had renal calculi and Questions Incorrect 3
• Mock Exam
died of a tumour in her neck. On examination a nodule is Questions Total 10
• Past Papers felt in the patient’s neck in the region of the thyroid gland.
Questions Percentage 70 %
• Random Questions A surgeon advises complete thyroidectomy. What is the
most important investigation the surgeon must More
• My Performance
undertake prior to surgery?
• Media Bank
• New Multimedia Serum thyroxine
Online Extras Serum calcium
Library 24-h urine test for 5-
hydroxyindoleacetic acid
Community
24-hour urinary catecholamines Your answer
Help
Serum calcitonin
PasTest Store

My Account

Profile This patient probably has medullary carcinoma of the


thyroid, which is usually associated with
Newsletters
phaeochromocytoma and hyperparathyroidism as part
My Career and Exams of the MEN-2a syndrome. Here, the serum thyroxine
Order History level is unchanged, while A 24-h urine test for 5– Reference: Normal Values
hydroxyindoleacetic acid is performed for the diagnosis
Learning Goals of carcinoid syndrome. Serum calcium levels will be Click to open/closeHaematology
Question Filters raised, but this is not an essential investigation prior to Click to open/closeImmunoglobulins
surgery. Phaeochromocytoma must be excluded before Click to open/closeBiochemistry
surgery. If present, the effect of catecholamines must Click to open/closeDiabetes Click to
Security be blocked prior to surgery. open/closeEndocrinology Click to
open/closeBlood gases Click to
Change Password open/closeCSF
Sign Out 3439

© 2011 PasTest Ltd | About Us | Contact Us | Help

file:///E|/Shakhawan/Endocrinology/240a.htm[3/13/2012 7:12:56 PM]


MyPasTest

Main Navigation
Question Browser: MRCP 1
Home
Subscriptions Question Browser Exam Builder Saved Exams
» MRCP 1 Session Progress
• Question Browser
Questions Correct 7
• Timed Test A 33-year-old woman presents with polydipsia and
polyuria. Her symptoms started soon after a road traffic Questions Incorrect 3
• Mock Exam
accident 6 months ago. Her blood pressure is 120/80 Questions Total 10
• Past Papers mmHg. The daily urinary output is 6–8 litres. Blood tests:
Questions Percentage 70 %
• Random Questions sodium 130 mmol/l (137–144), potassium 3.5 mmol/l (3.5–
4.9), urea 6 mmol/l (2.5–7.5) and glucose 4 mmol/l (3–6). More
• My Performance
Plasma osmolality is 268 mOsmol/l (278–305) and urine
• Media Bank osmolality is 45 mOsmol/l (350–1000).
• New Multimedia
What is the most likely diagnosis?
Online Extras Reference: Normal Values
Library Cranial diabetes insipidus
Diuretic phase of acute renal failure Click to
Community open/closeHaematology
Help Nephrogenic diabetes insipidus Click to
Psychogenic polydipsia open/closeImmunoglobulins
PasTest Store
Click to
Syndrome of inappropriate antidiuretic hormone open/closeBiochemistry
secretion Click to open/closeDiabetes
My Account
Click to
Profile 3440 open/closeEndocrinology
Click to open/closeBlood
Newsletters gases Click to
My Career and Exams open/closeCSF
Order History
Learning Goals
Question Filters

Security

Change Password
Sign Out

© 2011 PasTest Ltd | About Us | Contact Us | Help

file:///E|/Shakhawan/Endocrinology/241.htm[3/13/2012 7:12:58 PM]


MyPasTest

Main Navigation
Question Browser: MRCP 1
Home
Subscriptions Question Browser Exam Builder Saved Exams
» MRCP 1 Session Progress
• Question Browser
Questions Correct 8
• Timed Test A 33-year-old woman presents with polydipsia and
polyuria. Her symptoms started soon after a road traffic Questions Incorrect 3
• Mock Exam
accident 6 months ago. Her blood pressure is 120/80 Questions Total 11
• Past Papers mmHg. The daily urinary output is 6–8 litres. Blood tests:
Questions Percentage 72 %
• Random Questions sodium 130 mmol/l (137–144), potassium 3.5 mmol/l (3.5–
4.9), urea 6 mmol/l (2.5–7.5) and glucose 4 mmol/l (3–6). More
• My Performance
Plasma osmolality is 268 mOsmol/l (278–305) and urine
• Media Bank osmolality is 45 mOsmol/l (350–1000).
• New Multimedia
What is the most likely diagnosis?
Online Extras
Library Cranial diabetes insipidus
Community Diuretic phase of acute renal failure
Help Nephrogenic diabetes insipidus
PasTest Store Psychogenic polydipsia Your answer
Syndrome of inappropriate antidiuretic
hormone secretion
My Account

Profile
Newsletters
My Career and Exams In psychogenic polydipsia, there is excessive water
intake and hence serum sodium, plasma osmolality and Reference: Normal Values
Order History
urine osmolality will be decreased. In diabetes insipidus,
Learning Goals whether cranial or nephrogenic, there will be an Click to open/closeHaematology
Question Filters increase in plasma osmolality and serum sodium levels Click to open/closeImmunoglobulins
as water is lost from the body. In the diuretic phase of Click to open/closeBiochemistry
acute renal failure, the serum sodium level will be Click to open/closeDiabetes Click to
Security increased. The syndrome of inappropriate secretion of open/closeEndocrinology Click to
ADH (SIADH) leads to retention of water and open/closeBlood gases Click to
Change Password hyponatraemia. Plasma osmolality will be decreased as open/closeCSF
well. However, urine osmolality will be raised and may
Sign Out
be even higher than plasma osmolality.

3440

© 2011 PasTest Ltd | About Us | Contact Us | Help

file:///E|/Shakhawan/Endocrinology/241a.htm[3/13/2012 7:12:59 PM]


MyPasTest

Main Navigation
Question Browser: MRCP 1
Home
Subscriptions Question Browser Exam Builder Saved Exams
» MRCP 1 Session Progress
• Question Browser
Questions Correct 8
• Timed Test A 22-year-old man presents with tremor, sweating,
flushing and weight loss. On examination his pulse is 135 Questions Incorrect 3
• Mock Exam
bpm and his blood pressure is 160/110 mmHg. He has a Questions Total 11
• Past Papers large port-wine naevus on one side of his face. What is
Questions Percentage 72 %
• Random Questions the most probable diagnosis?
• My Performance More
von Recklinghausen’s disease
• Media Bank
von Hippel–Lindau syndrome
• New Multimedia
Sturge–Weber syndrome
Online Extras Reference: Normal Values
Ataxia telangiectasia
Library
Hereditary haemorrhagic telangiectasia Click to
Community open/closeHaematology
Help 3441 Click to
open/closeImmunoglobulins
PasTest Store
Click to
open/closeBiochemistry
My Account Click to open/closeDiabetes
Click to
Profile open/closeEndocrinology
Click to open/closeBlood
Newsletters gases Click to
My Career and Exams open/closeCSF
Order History
Learning Goals
Question Filters

Security

Change Password
Sign Out

© 2011 PasTest Ltd | About Us | Contact Us | Help

file:///E|/Shakhawan/Endocrinology/242.htm[3/13/2012 7:13:01 PM]


MyPasTest

Main Navigation
Question Browser: MRCP 1
Home
Subscriptions Question Browser Exam Builder Saved Exams
» MRCP 1 Session Progress
• Question Browser
Questions Correct 8
• Timed Test A 40-year-old woman who has been prescribed
metoclopramide complains of galactorrhoea and Questions Incorrect 4
• Mock Exam
amenorrhoea. What is the main mechanism of action Questions Total 12
• Past Papers of metoclopramide in this case?
Questions Percentage 66 %
• Random Questions
• My Performance Stimulates dopamine release by the hypothalamus More
• Media Bank Direct stimulatory effect on pituitary lactotrophs

• New Multimedia Binds to D 2 -receptors on pituitary lactotrophs

Online Extras Promotes pituitary adenoma growth Reference: Normal Values


Library Inhibits luteinising hormone secretion by the anterior
pituitary Click to
Community open/closeHaematology
Help 3442 Click to
open/closeImmunoglobulins
PasTest Store
Click to
open/closeBiochemistry
My Account Click to open/closeDiabetes
Click to
Profile open/closeEndocrinology
Click to open/closeBlood
Newsletters gases Click to
My Career and Exams open/closeCSF
Order History
Learning Goals
Question Filters

Security

Change Password
Sign Out

© 2011 PasTest Ltd | About Us | Contact Us | Help

file:///E|/Shakhawan/Endocrinology/243.htm[3/13/2012 7:13:02 PM]


MyPasTest

Main Navigation
Question Browser: MRCP 1
Home
Subscriptions Question Browser Exam Builder Saved Exams
» MRCP 1 Session Progress
• Question Browser
Questions Correct 9
• Timed Test A 40-year-old woman who has been prescribed
metoclopramide complains of galactorrhoea and Questions Incorrect 4
• Mock Exam
amenorrhoea. What is the main mechanism of action Questions Total 13
• Past Papers of metoclopramide in this case?
Questions Percentage 69 %
• Random Questions
• My Performance Stimulates dopamine release by the More
hypothalamus
• Media Bank
Direct stimulatory effect on pituitary
• New Multimedia lactotrophs
Online Extras Binds to D 2 -receptors on pituitary Your answer
Library lactotrophs
Community Promotes pituitary adenoma growth
Help Inhibits luteinising hormone secretion
by the anterior pituitary
PasTest Store

My Account

Profile This patient has developed hyperprolactinaemia due to


Newsletters metoclopramide therapy. Metoclopramide is a dopamine
antagonist that binds to D 2 -receptors on pituitary
My Career and Exams
lactotrophs. This stimulates the release of prolactin by
Order History the anterior pituitary thereby resulting in galactorrhoea. Reference: Normal Values
Learning Goals Prolactin inhibits gonadotrophin-releasing hormone and
gonadotrophin secretion per se, and hence Click to open/closeHaematology
Question Filters oligomenorrhoea or amenorrhoea may occur. Click to open/closeImmunoglobulins
Click to open/closeBiochemistry
Click to open/closeDiabetes Click to
Security open/closeEndocrinology Click to
3442
open/closeBlood gases Click to
Change Password open/closeCSF
Sign Out

© 2011 PasTest Ltd | About Us | Contact Us | Help

file:///E|/Shakhawan/Endocrinology/243a.htm[3/13/2012 7:13:03 PM]


MyPasTest

Main Navigation
Question Browser: MRCP 1
Home
Subscriptions Question Browser Exam Builder Saved Exams
» MRCP 1 Session Progress
• Question Browser
Questions Correct 9
• Timed Test A 35-year-old man presents with weight loss, irritability,
tremor, palpitations and heat intolerance. On examination Questions Incorrect 4
• Mock Exam
his pulse is 110 bpm, BP 150/100 mmHg. Lid lag is Questions Total 13
• Past Papers noticeable and a diffuse swelling can be felt in his neck.
Questions Percentage 69 %
• Random Questions
Which of the following blood tests is most sensitive More
• My Performance in establishing whether there is excess thyroid
• Media Bank activity?
• New Multimedia
Free T 4 levels
Online Extras Reference: Normal Values
T 3 level
Library
TSH-receptor antibodies Click to
Community open/closeHaematology
Help TPO (thyroid peroxidase) antibodies Click to
TSH level open/closeImmunoglobulins
PasTest Store
Click to
3443 open/closeBiochemistry
My Account Click to open/closeDiabetes
Click to
Profile open/closeEndocrinology
Click to open/closeBlood
Newsletters gases Click to
My Career and Exams open/closeCSF
Order History
Learning Goals
Question Filters

Security

Change Password
Sign Out

© 2011 PasTest Ltd | About Us | Contact Us | Help

file:///E|/Shakhawan/Endocrinology/244.htm[3/13/2012 7:13:05 PM]


MyPasTest

Main Navigation
Question Browser: MRCP 1
Home
Subscriptions Question Browser Exam Builder Saved Exams
» MRCP 1 Session Progress
• Question Browser
Questions Correct 9
• Timed Test A 35-year-old man presents with weight loss, irritability,
tremor, palpitations and heat intolerance. On examination Questions Incorrect 5
• Mock Exam
his pulse is 110 bpm, BP 150/100 mmHg. Lid lag is Questions Total 14
• Past Papers noticeable and a diffuse swelling can be felt in his neck.
Questions Percentage 64 %
• Random Questions
Which of the following blood tests is most sensitive More
• My Performance in establishing whether there is excess thyroid
• Media Bank activity?
• New Multimedia
Free T 4 levels
Online Extras
T 3 level Correct answer
Library
Community TSH-receptor antibodies

Help TPO (thyroid peroxidase) antibodies

PasTest Store TSH level Your answer

My Account

Profile Both thyroxine (T4 ) and triiodothyronine (T3 ) levels are


Newsletters raised in hyperthyroidism; however, T 3 is more
My Career and Exams sensitive, as occasional cases of isolated T 3 toxicosis
Order History can occur. The serum thyroid stimulating hormone Reference: Normal Values
(TSH) level is suppressed in hyperthyroidism, but there
Learning Goals
are rare instances of TSH hypersecretion. TPO and TSH Click to open/closeHaematology
Question Filters receptor antibodies are present in most cases of Graves’ Click to open/closeImmunoglobulins
disease and also in other conditions, including Click to open/closeBiochemistry
autoimmune thyroiditis. Click to open/closeDiabetes Click to
Security open/closeEndocrinology Click to
open/closeBlood gases Click to
Change Password open/closeCSF
3443
Sign Out

© 2011 PasTest Ltd | About Us | Contact Us | Help

file:///E|/Shakhawan/Endocrinology/244a.htm[3/13/2012 7:13:06 PM]


MyPasTest

Main Navigation
Question Browser: MRCP 1
Home
Subscriptions Question Browser Exam Builder Saved Exams
» MRCP 1 Session Progress
• Question Browser
Questions Correct 9
• Timed Test A 25-year-old man is admitted to the A&E having
consumed 20 tablets of propranolol. An infusion of glucagon Questions Incorrect 5
• Mock Exam
is prescribed. What is the main mechanism of action of Questions Total 14
• Past Papers glucagon in this case?
Questions Percentage 64 %
• Random Questions
• My Performance Promotes the formation of cyclic AMP More
• Media Bank Stimulates lipolysis

• New Multimedia Increases glycogenolysis

Online Extras Promotes gluconeogenesis


Reference: Normal Values
Library Alters protein kinase A activity
Click to
Community 3444 open/closeHaematology
Help Click to
open/closeImmunoglobulins
PasTest Store
Click to
open/closeBiochemistry
My Account Click to open/closeDiabetes
Click to
Profile open/closeEndocrinology
Click to open/closeBlood
Newsletters gases Click to
My Career and Exams open/closeCSF
Order History
Learning Goals
Question Filters

Security

Change Password
Sign Out

© 2011 PasTest Ltd | About Us | Contact Us | Help

file:///E|/Shakhawan/Endocrinology/245.htm[3/13/2012 7:13:08 PM]


MyPasTest

Main Navigation
Question Browser: MRCP 1
Home
Subscriptions Question Browser Exam Builder Saved Exams
» MRCP 1 Session Progress
• Question Browser
Questions Correct 9
• Timed Test A 25-year-old man is admitted to the A&E having
consumed 20 tablets of propranolol. An infusion of glucagon Questions Incorrect 6
• Mock Exam
is prescribed. What is the main mechanism of action of Questions Total 15
• Past Papers glucagon in this case?
Questions Percentage 60 %
• Random Questions
• My Performance Promotes the formation of cyclic Correct answer More
AMP
• Media Bank
Stimulates lipolysis
• New Multimedia
Increases glycogenolysis
Online Extras
Promotes gluconeogenesis
Library
Alters protein kinase A activity Your answer
Community
Help
PasTest Store

Glucagon acts by bypassing the blocked β-receptor,


My Account thus activating adenyl cyclase and promoting the
formation of cyclic AMP from ATP. Cyclic AMP in turn
Profile exerts a direct b-stimulant action on the heart.
Newsletters Although all the other options are actions of glucagon,
they are, however, not essential for reversing the effect
My Career and Exams of excess propranolol ingestion.
Order History Reference: Normal Values
Learning Goals
3444 Click to open/closeHaematology
Question Filters Click to open/closeImmunoglobulins
Click to open/closeBiochemistry
Click to open/closeDiabetes Click to
Security open/closeEndocrinology Click to
open/closeBlood gases Click to
Change Password open/closeCSF
Sign Out

© 2011 PasTest Ltd | About Us | Contact Us | Help

file:///E|/Shakhawan/Endocrinology/245a.htm[3/13/2012 7:13:09 PM]


MyPasTest

Main Navigation
Question Browser: MRCP 1
Home
Subscriptions Question Browser Exam Builder Saved Exams
» MRCP 1 Session Progress
• Question Browser
Questions Correct 9
• Timed Test A 50-year-old man presents with a history of panic attacks,
Questions Incorrect 6
• Mock Exam palpitations, sweating, headache and flushing. On
examination his pulse rate is 120 bpm, BP 190/110 mmHg. Questions Total 15
• Past Papers A blood test shows high levels of noradrenaline. Which of Questions Percentage 60 %
• Random Questions the following tests would be most useful in
establishing the diagnosis? More
• My Performance
• Media Bank Ultrasound scan
• New Multimedia CT scan
Online Extras MRI scan Reference: Normal Values
Library Renal arteriography
Click to
Community
[ 131 I]metaiodobenzylguanidine (MIBG) scan open/closeHaematology
Help Click to
3445 open/closeImmunoglobulins
PasTest Store
Click to
open/closeBiochemistry
My Account Click to open/closeDiabetes
Click to
Profile open/closeEndocrinology
Click to open/closeBlood
Newsletters gases Click to
My Career and Exams open/closeCSF
Order History
Learning Goals
Question Filters

Security

Change Password
Sign Out

© 2011 PasTest Ltd | About Us | Contact Us | Help

file:///E|/Shakhawan/Endocrinology/246.htm[3/13/2012 7:13:11 PM]


MyPasTest

Main Navigation
Question Browser: MRCP 1
Home
Subscriptions Question Browser Exam Builder Saved Exams
» MRCP 1 Session Progress
• Question Browser
Questions Correct 10
• Timed Test A 50-year-old man presents with a history of panic attacks,
palpitations, sweating, headache and flushing. On Questions Incorrect 6
• Mock Exam
examination his pulse rate is 120 bpm, BP 190/110 mmHg. Questions Total 16
• Past Papers A blood test shows high levels of noradrenaline. Which of
Questions Percentage 62 %
• Random Questions the following tests would be most useful in
establishing the diagnosis? More
• My Performance
• Media Bank Ultrasound scan
• New Multimedia CT scan
Online Extras MRI scan
Library Renal arteriography
Community Your answer
[ 131 I]metaiodobenzylguanidine (MIBG)
Help scan
PasTest Store

My Account
This patient most probably has a phaeochromocytoma.
Profile The presence of noradrenaline alone usually indicates
Newsletters an extra-adrenal tumour. Scanning with
My Career and Exams [ 131 I]metaiodobenzylguanidine (MIBG) produces
specific uptake in sites of sympathetic activity with
Order History about 90% success. It is particularly useful with extra- Reference: Normal Values
Learning Goals adrenal tumours. CT and MRI scans are useful in
detecting adrenal tumours. An ultrasound scan is the Click to open/closeHaematology
Question Filters least sensitive modality and is thus not preferred. Click to open/closeImmunoglobulins
Secondary hypertension may occur due to unilateral Click to open/closeBiochemistry
renal artery stenosis but is unlikely to be associated Click to open/closeDiabetes Click to
Security open/closeEndocrinology Click to
with these clinical features.
open/closeBlood gases Click to
Change Password open/closeCSF
Sign Out 3445

© 2011 PasTest Ltd | About Us | Contact Us | Help

file:///E|/Shakhawan/Endocrinology/246a.htm[3/13/2012 7:13:12 PM]


MyPasTest

Main Navigation
Question Browser: MRCP 1
Home
Subscriptions Question Browser Exam Builder Saved Exams
» MRCP 1 Session Progress
• Question Browser
Questions Correct 10
• Timed Test A 20-year-old woman with a history of migraine is
prescribed a progestin-only pill. What is the main Questions Incorrect 6
• Mock Exam
contraceptive mechanism of action of this drug? Questions Total 16
• Past Papers
Questions Percentage 62 %
• Random Questions Decreases tubal motility
• My Performance Inhibits ovulation More
• Media Bank Creates an unfavourable endometrium for
• New Multimedia implantation

Online Extras Thickens cervical mucus


Reference: Normal Values
Library Reduces LH secretion
Click to
Community 3446 open/closeHaematology
Help Click to
open/closeImmunoglobulins
PasTest Store
Click to
open/closeBiochemistry
My Account Click to open/closeDiabetes
Click to
Profile open/closeEndocrinology
Click to open/closeBlood
Newsletters gases Click to
My Career and Exams open/closeCSF
Order History
Learning Goals
Question Filters

Security

Change Password
Sign Out

© 2011 PasTest Ltd | About Us | Contact Us | Help

file:///E|/Shakhawan/Endocrinology/247.htm[3/13/2012 7:13:14 PM]


MyPasTest

Main Navigation
Question Browser: MRCP 1
Home
Subscriptions Question Browser Exam Builder Saved Exams
» MRCP 1 Session Progress
• Question Browser
Questions Correct 10
• Timed Test A 20-year-old woman with a history of migraine is
prescribed a progestin-only pill. What is the main Questions Incorrect 7
• Mock Exam
contraceptive mechanism of action of this drug? Questions Total 17
• Past Papers
Questions Percentage 58 %
• Random Questions Decreases tubal motility Your answer
• My Performance Inhibits ovulation More
• Media Bank Creates an unfavourable
• New Multimedia endometrium for implantation

Online Extras Thickens cervical mucus Correct answer

Library Reduces LH secretion

Community
Help
PasTest Store
Progestin-only pills are useful for women who are
unable to take regular oral contraceptives due to their
My Account side-effects or to the exacerbation of existing conditions
like migraine. Daily intake thickens the cervical mucus
Profile thus preventing the entry of sperm. The dose of
progesterone is not sufficient to inhibit ovulation or
Newsletters
reduce luteinising hormone (LH) secretion. Although it
My Career and Exams decreases tubal motility, this is more related to the
Order History occurrence of ectopic pregnancy. It may make the Reference: Normal Values
endometrium out-of-phase but not unfavourable for
Learning Goals implantation. Click to open/closeHaematology
Question Filters Click to open/closeImmunoglobulins
Click to open/closeBiochemistry
3446 Click to open/closeDiabetes Click to
Security open/closeEndocrinology Click to
open/closeBlood gases Click to
Change Password open/closeCSF
Sign Out

© 2011 PasTest Ltd | About Us | Contact Us | Help

file:///E|/Shakhawan/Endocrinology/247a.htm[3/13/2012 7:13:16 PM]


MyPasTest

Main Navigation
Question Browser: MRCP 1
Home
Subscriptions Question Browser Exam Builder Saved Exams
» MRCP 1 Session Progress
• Question Browser
Questions Correct 10
• Timed Test An 11-month old boy weighing 7 kg has polyuria, polydipsia
and delayed motor milestones. Blood tests: creatinine 80 Questions Incorrect 7
• Mock Exam
µmol/l (60–110), urea 6.0 mmol/l (2.5–7.5), potassium 5.0 Questions Total 17
• Past Papers mmol/l (3.5–4.9), sodium 150 mmol/l (137–144), chloride
Questions Percentage 58 %
• Random Questions 100 mmol/l (95–107) and bicarbonate 27 mmol/l (20–28).
• My Performance More
What is the most likely diagnosis?
• Media Bank
• New Multimedia Renal tubular acidosis (type 4)

Online Extras Gordon’s syndrome


Reference: Normal Values
Library Diabetes insipidus
Pseudohypoaldosteronism Click to
Community open/closeHaematology
Help Renal failure Click to
open/closeImmunoglobulins
PasTest Store 3447 Click to
open/closeBiochemistry
My Account Click to open/closeDiabetes
Click to
Profile open/closeEndocrinology
Click to open/closeBlood
Newsletters gases Click to
My Career and Exams open/closeCSF
Order History
Learning Goals
Question Filters

Security

Change Password
Sign Out

© 2011 PasTest Ltd | About Us | Contact Us | Help

file:///E|/Shakhawan/Endocrinology/248.htm[3/13/2012 7:13:17 PM]


MyPasTest

Main Navigation
Question Browser: MRCP 1
Home
Subscriptions Question Browser Exam Builder Saved Exams
» MRCP 1 Session Progress
• Question Browser
Questions Correct 10
• Timed Test An 11-month old boy weighing 7 kg has polyuria, polydipsia
and delayed motor milestones. Blood tests: creatinine 80 Questions Incorrect 8
• Mock Exam
µmol/l (60–110), urea 6.0 mmol/l (2.5–7.5), potassium 5.0 Questions Total 18
• Past Papers mmol/l (3.5–4.9), sodium 150 mmol/l (137–144), chloride
Questions Percentage 55 %
• Random Questions 100 mmol/l (95–107) and bicarbonate 27 mmol/l (20–28).
• My Performance More
What is the most likely diagnosis?
• Media Bank
• New Multimedia Renal tubular acidosis (type 4)

Online Extras Gordon’s syndrome Your answer

Library Diabetes insipidus Correct answer

Community Pseudohypoaldosteronism

Help Renal failure

PasTest Store

My Account
In diabetes insipidus, there is excessive loss of water in
Profile the urine leading to raised sodium and potassium levels.
In type-4 renal tubular acidosis, metabolic acidosis
Newsletters occurs coupled with a low plasma bicarbonate level and
My Career and Exams hyperchloraemia. Gordon’s syndrome is associated with
the renal retention of sodium, causing hypertension, Reference: Normal Values
Order History
volume expansion, low renin/aldosterone,
Learning Goals hyperkalaemia and metabolic acidosis. Decreased Click to open/closeHaematology
Question Filters sodium and increased potassium levels are seen in Click to open/closeImmunoglobulins
patients with pseudohypoaldosteronism. Renal failure is Click to open/closeBiochemistry
ruled out by the presence of normal creatinine and urea Click to open/closeDiabetes Click to
Security levels. open/closeEndocrinology Click to
open/closeBlood gases Click to
Change Password open/closeCSF
Sign Out 3447

© 2011 PasTest Ltd | About Us | Contact Us | Help

file:///E|/Shakhawan/Endocrinology/248a.htm[3/13/2012 7:13:19 PM]


MyPasTest

Main Navigation
Question Browser: MRCP 1
Home
Subscriptions Question Browser Exam Builder Saved Exams
» MRCP 1 Session Progress
• Question Browser
Questions Correct 10
• Timed Test A 15-year-old boy is being treated with ADH for diabetes
insipidus. His plasma glucose level (fasting) is 6 mmol/l (3– Questions Incorrect 8
• Mock Exam
6), sodium 148 mmol/l (137–144), potassium 4.5 mmol/l Questions Total 18
• Past Papers (3.5–4.9) and calcium 2.8 mmol/l (2.2–2.6). He still has
Questions Percentage 55 %
• Random Questions complaints of polyuria, polydipsia and nocturia.
• My Performance More
What could be the most probable cause?
• Media Bank
• New Multimedia Diabetes mellitus

Online Extras Nephrogenic diabetes insipidus


Reference: Normal Values
Library Primary polydipsia
SIADH Click to
Community open/closeHaematology
Help Hypercalcaemia Click to
open/closeImmunoglobulins
PasTest Store 3448 Click to
open/closeBiochemistry
My Account Click to open/closeDiabetes
Click to
Profile open/closeEndocrinology
Click to open/closeBlood
Newsletters gases Click to
My Career and Exams open/closeCSF
Order History
Learning Goals
Question Filters

Security

Change Password
Sign Out

© 2011 PasTest Ltd | About Us | Contact Us | Help

file:///E|/Shakhawan/Endocrinology/249.htm[3/13/2012 7:13:20 PM]


MyPasTest

Main Navigation
Question Browser: MRCP 1
Home
Subscriptions Question Browser Exam Builder Saved Exams
» MRCP 1 Session Progress
• Question Browser
Questions Correct 10
• Timed Test A 15-year-old boy is being treated with ADH for diabetes
insipidus. His plasma glucose level (fasting) is 6 mmol/l (3– Questions Incorrect 9
• Mock Exam
6), sodium 148 mmol/l (137–144), potassium 4.5 mmol/l Questions Total 19
• Past Papers (3.5–4.9) and calcium 2.8 mmol/l (2.2–2.6). He still has
Questions Percentage 52 %
• Random Questions complaints of polyuria, polydipsia and nocturia.
• My Performance More
What could be the most probable cause?
• Media Bank
• New Multimedia Diabetes mellitus

Online Extras Nephrogenic diabetes insipidus Correct answer

Library Primary polydipsia

Community SIADH

Help Hypercalcaemia Your answer

PasTest Store

My Account
This boy most probably has nephrogenic diabetes
Profile insipidus. In this condition, renal tubules are resistant
to antidiuretic hormone (ADH), which is due to a
Newsletters receptor defect of vasopressin-2 receptor or a
My Career and Exams postreceptor defect in an ADH-sensitive water channel
called aquaporin-2. It would appear that he is able to Reference: Normal Values
Order History
drink enough water however to keep up with his losses.
Learning Goals The findings do not suggest diabetes mellitus, which is Click to open/closeHaematology
Question Filters also associated with these symptoms. In patients with Click to open/closeImmunoglobulins
the syndrome of inappropriate secretion of ADH Click to open/closeBiochemistry
(SIADH), sodium levels are usually < 125 mmol/l. Mild Click to open/closeDiabetes Click to
Security hypercalcaemia (< 3 mmol/l) is frequently open/closeEndocrinology Click to
asymptomatic. Primary polydipsia is a psychiatric open/closeBlood gases Click to
Change Password disturbance characterised by the excessive intake of open/closeCSF
water. It is unusual at this age. Sodium levels fall as a
Sign Out
result and the urine produced is appropriately dilute.

3448

© 2011 PasTest Ltd | About Us | Contact Us | Help

file:///E|/Shakhawan/Endocrinology/249a.htm[3/13/2012 7:13:21 PM]


MyPasTest

Main Navigation
Question Browser: MRCP 1
Home
Subscriptions Question Browser Exam Builder Saved Exams
» MRCP 1 Session Progress
• Question Browser
Questions Correct 10
• Timed Test A 70-year-old diabetic man being treated with amiloride for
congestive heart failure is admitted in a comatose state. Questions Incorrect 9
• Mock Exam
Blood tests: sodium 152 mmol/l (137–144), potassium 5.0 Questions Total 19
• Past Papers mmol/l (3.5–4.9), chloride 115 mmol/l (95–107),
Questions Percentage 52 %
• Random Questions bicarbonate 30 mmol/l (20–28), glucose 50 mmol/l (3–6)
and pH 7.35 (7.36–7.44). Creatinine is raised at 190 More
• My Performance
micromol/l. He is taking metformin for his diabetes.
• Media Bank
What is the most probable diagnosis?
• New Multimedia
Online Extras Diabetic ketoacidosis Reference: Normal Values
Library Ventricular arrhythmia
Click to
Community Non-ketotic hyperosmolar coma open/closeHaematology
Help Diabetic nephropathy Click to
open/closeImmunoglobulins
PasTest Store Hyperkalaemia due to amiloride Click to
open/closeBiochemistry
3449 Click to open/closeDiabetes
My Account
Click to
Profile open/closeEndocrinology
Click to open/closeBlood
Newsletters gases Click to
My Career and Exams open/closeCSF
Order History
Learning Goals
Question Filters

Security

Change Password
Sign Out

© 2011 PasTest Ltd | About Us | Contact Us | Help

file:///E|/Shakhawan/Endocrinology/250.htm[3/13/2012 7:13:23 PM]


MyPasTest

Main Navigation
Question Browser: MRCP 1
Home
Subscriptions Question Browser Exam Builder Saved Exams
» MRCP 1 Session Progress
• Question Browser
Questions Correct 11
• Timed Test A 70-year-old diabetic man being treated with amiloride for
congestive heart failure is admitted in a comatose state. Questions Incorrect 9
• Mock Exam
Blood tests: sodium 152 mmol/l (137–144), potassium 5.0 Questions Total 20
• Past Papers mmol/l (3.5–4.9), chloride 115 mmol/l (95–107),
Questions Percentage 55 %
• Random Questions bicarbonate 30 mmol/l (20–28), glucose 50 mmol/l (3–6)
and pH 7.35 (7.36–7.44). Creatinine is raised at 190 More
• My Performance
micromol/l. He is taking metformin for his diabetes.
• Media Bank
What is the most probable diagnosis?
• New Multimedia
Online Extras Diabetic ketoacidosis
Library Ventricular arrhythmia
Community Non-ketotic hyperosmolar coma Your answer
Help Diabetic nephropathy
PasTest Store Hyperkalaemia due to amiloride

My Account

Profile
The investigations suggest that this patient has non-
Newsletters ketotic hyperosmolar coma. The bicarbonate level and
My Career and Exams blood pH would both be low in patients with diabetic
ketoacidosis. The anion gap at the upper limit of normal Reference: Normal Values
Order History
is almost certainly contributed to by metformin and his
Learning Goals degree of renal impairment. Ventricular arrhythmia is Click to open/closeHaematology
Question Filters unlikely at potassium levels of 5 mmol/l. Click to open/closeImmunoglobulins
Click to open/closeBiochemistry
Click to open/closeDiabetes Click to
Security 3449 open/closeEndocrinology Click to
open/closeBlood gases Click to
Change Password open/closeCSF
Sign Out

© 2011 PasTest Ltd | About Us | Contact Us | Help

file:///E|/Shakhawan/Endocrinology/250a.htm[3/13/2012 7:13:24 PM]


MyPasTest

Main Navigation
Question Browser: MRCP 1
Home
Subscriptions Question Browser Exam Builder Saved Exams
» MRCP 1 Session Progress
• Question Browser
Questions Correct 11
• Timed Test A 40-year-old woman is brought to A&E in a semi-
comatose condition. Her husband tells you she has been Questions Incorrect 9
• Mock Exam
thirsty all the time over the past couple of weeks. Blood Questions Total 20
• Past Papers tests: potassium 5.0 mmol/l (3.5–4.9), sodium 140 mmol/l
Questions Percentage 55 %
• Random Questions (137–144), bicarbonate 7 mmol/l (20–28) and pH 7.1
(7.36–7.44). Which of the following blood sugar levels More
• My Performance
is most likely to be found in this case?
• Media Bank
• New Multimedia 11 mmol/l
Online Extras 2.5 mmol/l Reference: Normal Values
Library 20 mmol/l
> 30 mmol/l Click to
Community open/closeHaematology
Help 5.5 mmol/l Click to
open/closeImmunoglobulins
PasTest Store 3450 Click to
open/closeBiochemistry
My Account Click to open/closeDiabetes
Click to
Profile open/closeEndocrinology
Click to open/closeBlood
Newsletters gases Click to
My Career and Exams open/closeCSF
Order History
Learning Goals
Question Filters

Security

Change Password
Sign Out

© 2011 PasTest Ltd | About Us | Contact Us | Help

file:///E|/Shakhawan/Endocrinology/251.htm[3/13/2012 7:13:26 PM]


MyPasTest

Main Navigation
Question Browser: MRCP 1
Home
Subscriptions Question Browser Exam Builder Saved Exams
» MRCP 1 Session Progress
• Question Browser
Questions Correct 12
• Timed Test A 40-year-old woman is brought to A&E in a semi-
comatose condition. Her husband tells you she has been Questions Incorrect 9
• Mock Exam
thirsty all the time over the past couple of weeks. Blood Questions Total 21
• Past Papers tests: potassium 5.0 mmol/l (3.5–4.9), sodium 140 mmol/l
Questions Percentage 57 %
• Random Questions (137–144), bicarbonate 7 mmol/l (20–28) and pH 7.1
(7.36–7.44). Which of the following blood sugar levels More
• My Performance
is most likely to be found in this case?
• Media Bank
• New Multimedia 11 mmol/l
Online Extras 2.5 mmol/l
Library 20 mmol/l
Community > 30 mmol/l Your answer
Help 5.5 mmol/l
PasTest Store

My Account
This patient has severe diabetic ketoacidosis. Plasma
Profile glucose is often elevated to 30 mmol/l or higher. In
non-ketotic hyperosmolar coma, bicarbonate levels are
Newsletters
high (> 30 mmol/l), which differentiates it from diabetic
My Career and Exams ketoacidosis. The blood pH will also be in the normal
Order History range. Reference: Normal Values
Learning Goals
Click to open/closeHaematology
Question Filters 3450 Click to open/closeImmunoglobulins
Click to open/closeBiochemistry
Click to open/closeDiabetes Click to
Security open/closeEndocrinology Click to
open/closeBlood gases Click to
Change Password open/closeCSF
Sign Out

© 2011 PasTest Ltd | About Us | Contact Us | Help

file:///E|/Shakhawan/Endocrinology/251a.htm[3/13/2012 7:13:28 PM]


MyPasTest

Main Navigation
Question Browser: MRCP 1
Home
Subscriptions Question Browser Exam Builder Saved Exams
» MRCP 1 Session Progress
• Question Browser
Questions Correct 12
• Timed Test A 25-year-old woman is being investigated for suspected
MEN-2b syndrome. She is also noted to have marfanoid Questions Incorrect 9
• Mock Exam
features. What will be the most characteristic finding Questions Total 21
• Past Papers in a blood test in this patient?
Questions Percentage 57 %
• Random Questions
• My Performance Elevated serum calcium More
• Media Bank Elevated metanephrines

• New Multimedia Elevated thyroxine

Online Extras Decreased parathyroid hormone


Reference: Normal Values
Library Elevated glucagon
Click to
Community 3451 open/closeHaematology
Help Click to
open/closeImmunoglobulins
PasTest Store
Click to
open/closeBiochemistry
My Account Click to open/closeDiabetes
Click to
Profile open/closeEndocrinology
Click to open/closeBlood
Newsletters gases Click to
My Career and Exams open/closeCSF
Order History
Learning Goals
Question Filters

Security

Change Password
Sign Out

© 2011 PasTest Ltd | About Us | Contact Us | Help

file:///E|/Shakhawan/Endocrinology/252.htm[3/13/2012 7:13:29 PM]


MyPasTest

Main Navigation
Question Browser: MRCP 1
Home
Subscriptions Question Browser Exam Builder Saved Exams
» MRCP 1 Session Progress
• Question Browser
Questions Correct 13
• Timed Test A 25-year-old woman is being investigated for suspected
MEN-2b syndrome. She is also noted to have marfanoid Questions Incorrect 9
• Mock Exam
features. What will be the most characteristic finding Questions Total 22
• Past Papers in a blood test in this patient?
Questions Percentage 59 %
• Random Questions
• My Performance Elevated serum calcium More
• Media Bank Elevated metanephrines Your answer

• New Multimedia Elevated thyroxine

Online Extras Decreased parathyroid hormone

Library Elevated glucagon

Community
Help
PasTest Store
MEN-2b syndrome with a marfanoid phenotype is
associated with phaeochromocytoma and medullary
My Account carcinoma of the thyroid. Parathyroid hyperplasia occurs
in 2A, but is said to be extremely rare in MEN-2b.
Profile Thyroxine remains normal. Glucagon excess is seen in
MEN-1 syndrome.
Newsletters
My Career and Exams
Order History 3451 Reference: Normal Values
Learning Goals
Click to open/closeHaematology
Question Filters Click to open/closeImmunoglobulins
Click to open/closeBiochemistry
Click to open/closeDiabetes Click to
Security open/closeEndocrinology Click to
open/closeBlood gases Click to
Change Password open/closeCSF
Sign Out

© 2011 PasTest Ltd | About Us | Contact Us | Help

file:///E|/Shakhawan/Endocrinology/252a.htm[3/13/2012 7:13:31 PM]


MyPasTest

Main Navigation
Question Browser: MRCP 1
Home
Subscriptions Question Browser Exam Builder Saved Exams
» MRCP 1 Session Progress
• Question Browser
Questions Correct 13
• Timed Test Which of the following investigations is most likely to
indicate that ovulation is imminent? Questions Incorrect 9
• Mock Exam
Questions Total 22
• Past Papers
Follicle size of 20 mm on ultrasound scan Questions Percentage 59 %
• Random Questions
Increase in basal body temperature
• My Performance More
Positive Spinnbarkeit test
• Media Bank
Secretory changes on endometrial biopsy
• New Multimedia
Raised LH level
Online Extras Reference: Normal Values
Library 3452
Click to
Community open/closeHaematology
Help Click to
open/closeImmunoglobulins
PasTest Store
Click to
open/closeBiochemistry
My Account Click to open/closeDiabetes
Click to
Profile open/closeEndocrinology
Click to open/closeBlood
Newsletters gases Click to
My Career and Exams open/closeCSF
Order History
Learning Goals
Question Filters

Security

Change Password
Sign Out

© 2011 PasTest Ltd | About Us | Contact Us | Help

file:///E|/Shakhawan/Endocrinology/253.htm[3/13/2012 7:13:34 PM]


MyPasTest

Main Navigation
Question Browser: MRCP 1
Home
Subscriptions Question Browser Exam Builder Saved Exams
» MRCP 1 Session Progress
• Question Browser
Questions Correct 14
• Timed Test Which of the following investigations is most likely to
indicate that ovulation is imminent? Questions Incorrect 9
• Mock Exam
Questions Total 23
• Past Papers
Follicle size of 20 mm on ultrasound Questions Percentage 60 %
• Random Questions scan
• My Performance Increase in basal body temperature More
• Media Bank Positive Spinnbarkeit test
• New Multimedia Secretory changes on endometrial
Online Extras biopsy

Library Raised LH level Your answer

Community
Help
PasTest Store
Sudden increase in luteinising hormone (LH) – the LH
surge – is of particular significance as it indicates that
My Account ovulation will occur about 24 hours after the LH peak.
Follicle size is usually around 22 mm prior to ovulation
Profile but an ultrasound scan gives no indication that
ovulation is about to occur. An increase in basal body
Newsletters
temperature and secretory changes in the endometrium
My Career and Exams occur after ovulation. A positive Spinnbarkeit test is
Order History used to demonstrate oestrogenic activity on the cervical Reference: Normal Values
mucus. Elasticity of the cervical mucus is greatly
Learning Goals enhanced prior to ovulation, but does not provide a clue Click to open/closeHaematology
Question Filters as to when ovulation will occur. Click to open/closeImmunoglobulins
Click to open/closeBiochemistry
Click to open/closeDiabetes Click to
Security 3452 open/closeEndocrinology Click to
open/closeBlood gases Click to
Change Password open/closeCSF
Sign Out

© 2011 PasTest Ltd | About Us | Contact Us | Help

file:///E|/Shakhawan/Endocrinology/253a.htm[3/13/2012 7:13:35 PM]


MyPasTest

Main Navigation
Question Browser: MRCP 1
Home
Subscriptions Question Browser Exam Builder Saved Exams
» MRCP 1 Session Progress
• Question Browser
Questions Correct 14
• Timed Test A 14-year-old girl is seen at the endocrine clinic because
her mother is concerned that her daughter’s periods have Questions Incorrect 9
• Mock Exam
not yet started. In reviewing her history, which of the Questions Total 23
• Past Papers following would be most typical of the first sign of
Questions Percentage 60 %
• Random Questions onset of puberty?
• My Performance More
Pubic hair growth
• Media Bank
Height spurt
• New Multimedia
Breast-bud development
Online Extras Reference: Normal Values
Axillary hair growth
Library
Menarche Click to
Community open/closeHaematology
Help 3453 Click to
open/closeImmunoglobulins
PasTest Store
Click to
open/closeBiochemistry
My Account Click to open/closeDiabetes
Click to
Profile open/closeEndocrinology
Click to open/closeBlood
Newsletters gases Click to
My Career and Exams open/closeCSF
Order History
Learning Goals
Question Filters

Security

Change Password
Sign Out

© 2011 PasTest Ltd | About Us | Contact Us | Help

file:///E|/Shakhawan/Endocrinology/254.htm[3/13/2012 7:13:38 PM]


MyPasTest

Main Navigation
Question Browser: MRCP 1
Home
Subscriptions Question Browser Exam Builder Saved Exams
» MRCP 1 Session Progress
• Question Browser
Questions Correct 15
• Timed Test A 14-year-old girl is seen at the endocrine clinic because
her mother is concerned that her daughter’s periods have Questions Incorrect 9
• Mock Exam
not yet started. In reviewing her history, which of the Questions Total 24
• Past Papers following would be most typical of the first sign of
Questions Percentage 62 %
• Random Questions onset of puberty?
• My Performance More
Pubic hair growth
• Media Bank
Height spurt
• New Multimedia
Breast-bud development Your answer
Online Extras
Axillary hair growth
Library
Menarche
Community
Help
PasTest Store
Breast-bud enlargement is the first sign of puberty in
My Account girls. This begins between the ages of 9 and 12 years
and continues to 12–18 years. Pubic hair growth occurs
Profile next, at ages 9–14 years, and is complete at 12–16
years. Menarche occurs relatively late (age 11–15
Newsletters
years). Peak height velocity is reached earlier (10–13
My Career and Exams years) and growth is completed much earlier than in
Order History boys. Reference: Normal Values
Learning Goals
Click to open/closeHaematology
Question Filters 3453 Click to open/closeImmunoglobulins
Click to open/closeBiochemistry
Click to open/closeDiabetes Click to
Security open/closeEndocrinology Click to
open/closeBlood gases Click to
Change Password open/closeCSF
Sign Out

© 2011 PasTest Ltd | About Us | Contact Us | Help

file:///E|/Shakhawan/Endocrinology/254a.htm[3/13/2012 7:13:40 PM]


MyPasTest

Main Navigation
Question Browser: MRCP 1
Home
Subscriptions Question Browser Exam Builder Saved Exams
» MRCP 1 Session Progress
• Question Browser
Questions Correct 0
• Timed Test A 55-year-old chronic smoker presents with polyuria,
polydipsia and altered sensorium for the past 2 days. He is Questions Incorrect 0
• Mock Exam
known to be suffering from squamous-cell carcinoma of the Questions Total 0
• Past Papers lung. On examination he is lethargic and confused. ECG
Questions Percentage 0%
• Random Questions shows a narrowed QT interval. What is the most
probable metabolic abnormality? More
• My Performance
• Media Bank Hypernatraemia
• New Multimedia Hypercalcaemia
Online Extras Hypokalaemia Reference: Normal Values
Library Hyponatraemia
Click to
Community Hyperkalaemia open/closeHaematology
Help Click to
3455 open/closeImmunoglobulins
PasTest Store
Click to
open/closeBiochemistry
My Account Click to open/closeDiabetes
Click to
Profile open/closeEndocrinology
Click to open/closeBlood
Newsletters gases Click to
My Career and Exams open/closeCSF
Order History
Learning Goals
Question Filters

Security

Change Password
Sign Out

© 2011 PasTest Ltd | About Us | Contact Us | Help

file:///E|/Shakhawan/Endocrinology/255.htm[3/13/2012 7:13:41 PM]


MyPasTest

Main Navigation
Question Browser: MRCP 1
Home
Subscriptions Question Browser Exam Builder Saved Exams
» MRCP 1 Session Progress
• Question Browser
Questions Correct 1
• Timed Test A 55-year-old chronic smoker presents with polyuria,
polydipsia and altered sensorium for the past 2 days. He is Questions Incorrect 0
• Mock Exam
known to be suffering from squamous-cell carcinoma of the Questions Total 1
• Past Papers lung. On examination he is lethargic and confused. ECG
Questions Percentage 100 %
• Random Questions shows a narrowed QT interval. What is the most
probable metabolic abnormality? More
• My Performance
• Media Bank Hypernatraemia
• New Multimedia Hypercalcaemia Your answer
Online Extras Hypokalaemia
Library Hyponatraemia
Community Hyperkalaemia
Help
PasTest Store

My Account Paraneoplastic syndromes in bronchogenic carcinoma


can cause several electrolyte abnormalities. The
Profile features in this case are suggestive of hypercalcaemia.
This occurs due to parathyroid hormone-related peptide
Newsletters
(PTHrP) production.
My Career and Exams
Order History Reference: Normal Values
3455
Learning Goals
Click to open/closeHaematology
Question Filters Click to open/closeImmunoglobulins
Click to open/closeBiochemistry
Click to open/closeDiabetes Click to
Security open/closeEndocrinology Click to
open/closeBlood gases Click to
Change Password open/closeCSF
Sign Out

© 2011 PasTest Ltd | About Us | Contact Us | Help

file:///E|/Shakhawan/Endocrinology/255a.htm[3/13/2012 7:13:43 PM]


MyPasTest

Main Navigation
Question Browser: MRCP 1
Home
Subscriptions Question Browser Exam Builder Saved Exams
» MRCP 1 Session Progress
• Question Browser
Questions Correct 1
• Timed Test An 8-day-old baby presents with vomiting, poor feeding
and loose stools. On examination, his heart rate is 190 Questions Incorrect 0
• Mock Exam
bpm, BP 50/30 mmHg and respiratory rate 72 breaths/min. Questions Total 1
• Past Papers Blood tests: Hb 15 g/dl (13–18), sodium 120 mmol/l (137–
Questions Percentage 100 %
• Random Questions 144), potassium 6.0 mmol/l (3.5–4.9), chloride 80 mmol/l
(95–107), bicarbonate 15 mmol/l (21-28), urea 7.0 mmol/l More
• My Performance
(2.5–7.5) and creatinine 80 µmol/l (60–110).
• Media Bank
What is the most likely diagnosis?
• New Multimedia
Online Extras Congenital adrenal hyperplasia Reference: Normal Values
Library Acute tubular necrosis
Click to
Community Congenital hypertrophic pyloric stenosis open/closeHaematology
Help Galactosaemia Click to
open/closeImmunoglobulins
PasTest Store Lactose intolerance Click to
open/closeBiochemistry
3456 Click to open/closeDiabetes
My Account
Click to
Profile open/closeEndocrinology
Click to open/closeBlood
Newsletters gases Click to
My Career and Exams open/closeCSF
Order History
Learning Goals
Question Filters

Security

Change Password
Sign Out

© 2011 PasTest Ltd | About Us | Contact Us | Help

file:///E|/Shakhawan/Endocrinology/256.htm[3/13/2012 7:13:44 PM]


MyPasTest

Main Navigation
Question Browser: MRCP 1
Home
Subscriptions Question Browser Exam Builder Saved Exams
» MRCP 1 Session Progress
• Question Browser
Questions Correct 1
• Timed Test An 8-day-old baby presents with vomiting, poor feeding
and loose stools. On examination, his heart rate is 190 Questions Incorrect 1
• Mock Exam
bpm, BP 50/30 mmHg and respiratory rate 72 breaths/min. Questions Total 2
• Past Papers Blood tests: Hb 15 g/dl (13–18), sodium 120 mmol/l (137–
Questions Percentage 50 %
• Random Questions 144), potassium 6.0 mmol/l (3.5–4.9), chloride 80 mmol/l
(95–107), bicarbonate 15 mmol/l (21-28), urea 7.0 mmol/l More
• My Performance
(2.5–7.5) and creatinine 80 µmol/l (60–110).
• Media Bank
What is the most likely diagnosis?
• New Multimedia
Online Extras Congenital adrenal hyperplasia Correct answer
Library Acute tubular necrosis
Community Congenital hypertrophic pyloric
Help stenosis
PasTest Store Galactosaemia Your answer
Lactose intolerance

My Account

Profile
Newsletters The features and findings are highly suggestive of
My Career and Exams congenital adrenal hyperplasia. Absence of jaundice and
hepatomegaly make the diagnosis of galactosaemia Reference: Normal Values
Order History
unlikely. Acute tubular necrosis is ruled out, as urea and
Learning Goals creatinine levels are normal. In congenital hypertrophic Click to open/closeHaematology
Question Filters pyloric stenosis, hypokalaemia is the presenting feature Click to open/closeImmunoglobulins
and constipation (not loose stools) is the chief Click to open/closeBiochemistry
complaint. Lactose intolerance would again present with Click to open/closeDiabetes Click to
Security diarrhoea or vomiting, thus hypokalaemia would be the open/closeEndocrinology Click to
characteristic finding. open/closeBlood gases Click to
Change Password open/closeCSF
Sign Out
3456

© 2011 PasTest Ltd | About Us | Contact Us | Help

file:///E|/Shakhawan/Endocrinology/256a.htm[3/13/2012 7:13:46 PM]


MyPasTest

Main Navigation
Question Browser: MRCP 1
Home
Subscriptions Question Browser Exam Builder Saved Exams
» MRCP 1 Session Progress
• Question Browser
Questions Correct 1
• Timed Test A 10-month-old boy weighing 5 kg has polyuria, polydipsia
and delayed motor milestones. Blood tests: creatinine 75 Questions Incorrect 1
• Mock Exam
µmol/l (60–110), urea 6.5 mmol/l (2.5–7.5), potassium 3.0 Questions Total 2
• Past Papers mmol/l (3.5–4.9), sodium 125 mmol/l (137–144), chloride
Questions Percentage 50 %
• Random Questions 88 mmol/l (95–107), bicarbonate 26 mmol/l (20–28),
calcium 2.3 mmol/l (2.2–2.6) and pH 7.46 (7.36–7.44). More
• My Performance
• Media Bank What is the most likely diagnosis?
• New Multimedia
Renal tubular acidosis
Online Extras Reference: Normal Values
Cranial diabetes insipidus
Library
Bartter’s syndrome Click to
Community open/closeHaematology
Pseudohypoaldosteronism
Help Click to
Liddle’s syndrome open/closeImmunoglobulins
PasTest Store
Click to
3457 open/closeBiochemistry
My Account Click to open/closeDiabetes
Click to
Profile open/closeEndocrinology
Click to open/closeBlood
Newsletters gases Click to
My Career and Exams open/closeCSF
Order History
Learning Goals
Question Filters

Security

Change Password
Sign Out

© 2011 PasTest Ltd | About Us | Contact Us | Help

file:///E|/Shakhawan/Endocrinology/257.htm[3/13/2012 7:13:48 PM]


MyPasTest

Main Navigation
Question Browser: MRCP 1
Home
Subscriptions Question Browser Exam Builder Saved Exams
» MRCP 1 Session Progress
• Question Browser
Questions Correct 2
• Timed Test A 10-month-old boy weighing 5 kg has polyuria, polydipsia
and delayed motor milestones. Blood tests: creatinine 75 Questions Incorrect 1
• Mock Exam
µmol/l (60–110), urea 6.5 mmol/l (2.5–7.5), potassium 3.0 Questions Total 3
• Past Papers mmol/l (3.5–4.9), sodium 125 mmol/l (137–144), chloride
Questions Percentage 66 %
• Random Questions 88 mmol/l (95–107), bicarbonate 26 mmol/l (20–28),
calcium 2.3 mmol/l (2.2–2.6) and pH 7.46 (7.36–7.44). More
• My Performance
• Media Bank What is the most likely diagnosis?
• New Multimedia
Renal tubular acidosis
Online Extras
Cranial diabetes insipidus
Library
Bartter’s syndrome Your answer
Community
Pseudohypoaldosteronism
Help
Liddle’s syndrome
PasTest Store

My Account

Profile This child most probably has Bartter’s syndrome. Renal


tubular acidosis is ruled out as the child has alkalosis.
Newsletters There is loss of water in the urine in those with diabetes
My Career and Exams insipidus, as a consequence of which the concentration
of sodium and potassium is increased. In Reference: Normal Values
Order History
pseudohypoaldosteronism, decreased sodium and
Learning Goals increased potassium levels are usually seen. Potassium Click to open/closeHaematology
Question Filters wasting, hypokalaemia and alkalosis characterise Click to open/closeImmunoglobulins
Liddle’s syndrome. There is unregulated sodium Click to open/closeBiochemistry
reabsorption across the collecting tubule. Click to open/closeDiabetes Click to
Security open/closeEndocrinology Click to
open/closeBlood gases Click to
Change Password 3457 open/closeCSF
Sign Out

© 2011 PasTest Ltd | About Us | Contact Us | Help

file:///E|/Shakhawan/Endocrinology/257a.htm[3/13/2012 7:13:50 PM]


MyPasTest

Main Navigation
Question Browser: MRCP 1
Home
Subscriptions Question Browser Exam Builder Saved Exams
» MRCP 1 Session Progress
• Question Browser
Questions Correct 2
• Timed Test What is the optimum time for the administration of
hydrocortisone to a patient undergoing bilateral Questions Incorrect 1
• Mock Exam
adrenalectomy for Cushing’s disease? Questions Total 3
• Past Papers
Questions Percentage 66 %
• Random Questions At the start of laparotomy
• My Performance At the time of ligation of the left adrenal vein More
• Media Bank Following removal of one adrenal gland
• New Multimedia Following excision of both adrenal glands
Online Extras One week after surgery Reference: Normal Values
Library
3458 Click to
Community open/closeHaematology
Help Click to
open/closeImmunoglobulins
PasTest Store
Click to
open/closeBiochemistry
My Account Click to open/closeDiabetes
Click to
Profile open/closeEndocrinology
Click to open/closeBlood
Newsletters gases Click to
My Career and Exams open/closeCSF
Order History
Learning Goals
Question Filters

Security

Change Password
Sign Out

© 2011 PasTest Ltd | About Us | Contact Us | Help

file:///E|/Shakhawan/Endocrinology/258.htm[3/13/2012 7:13:51 PM]


MyPasTest

Main Navigation
Question Browser: MRCP 1
Home
Subscriptions Question Browser Exam Builder Saved Exams
» MRCP 1 Session Progress
• Question Browser
Questions Correct 2
• Timed Test What is the optimum time for the administration of
hydrocortisone to a patient undergoing bilateral Questions Incorrect 2
• Mock Exam
adrenalectomy for Cushing’s disease? Questions Total 4
• Past Papers
Questions Percentage 50 %
• Random Questions At the start of laparotomy Your answer
• My Performance At the time of ligation of the left More
• Media Bank adrenal vein

• New Multimedia Following removal of one adrenal


gland
Online Extras
Following excision of both adrenal Correct answer
Library glands
Community One week after surgery
Help
PasTest Store

My Account Most surgical texts agree that cortisol should be given


immediately following the removal of both adrenal
Profile glands. This makes sense, as it is at this point that
glucocorticoid and mineralocorticoid supplementation
Newsletters
becomes essential.
My Career and Exams
Order History Reference: Normal Values
3458
Learning Goals
Click to open/closeHaematology
Question Filters Click to open/closeImmunoglobulins
Click to open/closeBiochemistry
Click to open/closeDiabetes Click to
Security open/closeEndocrinology Click to
open/closeBlood gases Click to
Change Password open/closeCSF
Sign Out

© 2011 PasTest Ltd | About Us | Contact Us | Help

file:///E|/Shakhawan/Endocrinology/258a.htm[3/13/2012 7:13:53 PM]


MyPasTest

Main Navigation
Question Browser: MRCP 1
Home
Subscriptions Question Browser Exam Builder Saved Exams
» MRCP 1 Session Progress
• Question Browser
Questions Correct 2
• Timed Test A previously fit 30-year-old man presents with a two
month history of weight loss, tiredness, and nausea. Questions Incorrect 2
• Mock Exam
Investigations show (normal range in brackets): Questions Total 4
• Past Papers
Haemoglobin 10.5 g/dl (13.0–18.0) Questions Percentage 50 %
• Random Questions
• My Performance More
Mean cell volume (MCV) 88 fL (80–96)
• Media Bank
White cell count 6.0 ´ 109/l (4–11)
• New Multimedia
Online Extras Platelet count 450 ´ 109/l (150–400)
Reference: Normal Values
Library Serum sodium 130 mmol/l (137–144)
Click to
Community open/closeHaematology
Serum potassium 5.7 mmol/l (3.5–4.9)
Help Click to
Serum urea 3.0 mmol/l (2.5–7.5) open/closeImmunoglobulins
PasTest Store
Click to
Serum creatinine 78 mmol/l (60–110) open/closeBiochemistry
My Account Click to open/closeDiabetes
Serum total tetra-iodothyronine (T4 ) 55 nmol/l (50– Click to
Profile 150) open/closeEndocrinology
Click to open/closeBlood
Newsletters Serum thyroid-stimulating hormone (TSH) 8 mU/l gases Click to
My Career and Exams (0.2–5.5) open/closeCSF
Order History Which of the following is the most useful diagnostic
Learning Goals investigation?
Question Filters Antithyroid peroxidase antibody titre
Insulin tolerance test
Security Free thyroxine concentration
Change Password Short synacthen test
Sign Out Thyroid-releasing hormone (TRH) test

3889

© 2011 PasTest Ltd | About Us | Contact Us | Help

file:///E|/Shakhawan/Endocrinology/259.htm[3/13/2012 7:13:54 PM]


MyPasTest

Main Navigation
Question Browser: MRCP 1
Home
Subscriptions Question Browser Exam Builder Saved Exams
» MRCP 1 Session Progress
• Question Browser
Questions Correct 1
• Timed Test A previously fit 30-year-old man presents with a two
month history of weight loss, tiredness, and nausea. Questions Incorrect 0
• Mock Exam
Investigations show (normal range in brackets): Questions Total 1
• Past Papers
Haemoglobin 10.5 g/dl (13.0–18.0) Questions Percentage 100 %
• Random Questions
• My Performance More
Mean cell volume (MCV) 88 fL (80–96)
• Media Bank
White cell count 6.0 ´ 109/l (4–11)
• New Multimedia
Online Extras Platelet count 450 ´ 109/l (150–400)

Library Serum sodium 130 mmol/l (137–144)


Community
Serum potassium 5.7 mmol/l (3.5–4.9)
Help
Serum urea 3.0 mmol/l (2.5–7.5)
PasTest Store
Serum creatinine 78 mmol/l (60–110)
My Account Serum total tetra-iodothyronine (T4 ) 55 nmol/l (50–
Profile 150)
Newsletters Serum thyroid-stimulating hormone (TSH) 8 mU/l
My Career and Exams (0.2–5.5)
Order History Which of the following is the most useful diagnostic Reference: Normal Values
Learning Goals investigation?
Click to open/closeHaematology
Question Filters Antithyroid peroxidase antibody titre Click to open/closeImmunoglobulins
Click to open/closeBiochemistry
Insulin tolerance test Click to open/closeDiabetes Click to
Security Free thyroxine concentration open/closeEndocrinology Click to
open/closeBlood gases Click to
Change Password Short synacthen test Your answer
open/closeCSF
Sign Out Thyroid-releasing hormone (TRH) test

There is a clinical suggestion of hypoadrenalism in this


patient’s history, normocytic anaemia, hyperkalaemia,
and hyponatraemia, with deranged thyroid function
tests (raised thyroid-stimulating hormone (TSH)) and a
low and/or normal total tetra-iodothyronine (T4 ). This
picture is strongly suggestive of Addison’s disease. The
most common cause of Addison’s disease is
autoimmune destruction of the glands (80%),
tuberculosis is responsible for around 15% of cases,
and the remainder may be related to haemorrhage,
adrenal infarction, or carcinomatous destruction of the
glands. Diagnostic method of choice is the short
synacthen test, and later measurement of
adrenocorticotrophic (ACTH) confirms the aetiology
(primary or secondary adrenal failure).

3889

file:///E|/Shakhawan/Endocrinology/259a.htm[3/13/2012 7:13:56 PM]


MyPasTest

Main Navigation
Question Browser: MRCP 1
Home
Subscriptions Question Browser Exam Builder Saved Exams
» MRCP 1 Session Progress
• Question Browser
Questions Correct 1
• Timed Test A 16-year-old woman with Addison’s disease is intolerant
of her hydrocortisone treatment, which she takes at a dose Questions Incorrect 0
• Mock Exam
of 20 mg in the morning and 5 mg in the evening. Which Questions Total 1
• Past Papers of the following doses of prednisolone would provide
Questions Percentage 100 %
• Random Questions an approximately equivalent daily dose to her
hydrocortisone? More
• My Performance
• Media Bank 1 mg
• New Multimedia 7.5 mg
Online Extras 10 mg Reference: Normal Values
Library 12.5 mg
Click to
Community 15 mg open/closeHaematology
Help Click to
3890 open/closeImmunoglobulins
PasTest Store
Click to
open/closeBiochemistry
My Account Click to open/closeDiabetes
Click to
Profile open/closeEndocrinology
Click to open/closeBlood
Newsletters gases Click to
My Career and Exams open/closeCSF
Order History
Learning Goals
Question Filters

Security

Change Password
Sign Out

© 2011 PasTest Ltd | About Us | Contact Us | Help

file:///E|/Shakhawan/Endocrinology/260.htm[3/13/2012 7:13:57 PM]


MyPasTest

Main Navigation
Question Browser: MRCP 1
Home
Subscriptions Question Browser Exam Builder Saved Exams
» MRCP 1 Session Progress
• Question Browser
Questions Correct 2
• Timed Test A 16-year-old woman with Addison’s disease is intolerant
of her hydrocortisone treatment, which she takes at a dose Questions Incorrect 0
• Mock Exam
of 20 mg in the morning and 5 mg in the evening. Which Questions Total 2
• Past Papers of the following doses of prednisolone would provide
Questions Percentage 100 %
• Random Questions an approximately equivalent daily dose to her
hydrocortisone? More
• My Performance
• Media Bank 1 mg
• New Multimedia 7.5 mg Your answer
Online Extras 10 mg
Library 12.5 mg
Community 15 mg
Help
PasTest Store

My Account Equivalent dose of prednisolone vs hydrocortisone


therapy is usually about 25% of the hydrocortisone
Profile dose. Adequacy of steroid replacement is normally
assessed by clinical well being and restoration of normal
Newsletters
(not excessive) weight. Cortisol levels during the day
My Career and Exams are only a useful assessment if the patient is on
Order History hydrocortisone. Patients normally require therapy with Reference: Normal Values
mineralocorticoids as well as glucocorticoid therapy.
Learning Goals Standard therapy is with fludrocortisone 50–300μg/day, Click to open/closeHaematology
Question Filters and effectiveness is assessed by serum electrolytes, Click to open/closeImmunoglobulins
postural change in blood pressure, and suppression of Click to open/closeBiochemistry
plasma renin activity to normal levels. Click to open/closeDiabetes Click to
Security open/closeEndocrinology Click to
open/closeBlood gases Click to
Change Password open/closeCSF
3890
Sign Out

© 2011 PasTest Ltd | About Us | Contact Us | Help

file:///E|/Shakhawan/Endocrinology/260a.htm[3/13/2012 7:13:59 PM]


MyPasTest

Main Navigation
Question Browser: MRCP 1
Home
Subscriptions Question Browser Exam Builder Saved Exams
» MRCP 1 Session Progress
• Question Browser
Questions Correct 0
• Timed Test A 40-year-old woman who has been prescribed thyroid
replacement therapy has routine thyroid function tests Questions Incorrect 0
• Mock Exam
(TFTs). On examination she appeared clinically euthyroid Questions Total 0
• Past Papers with no abnormal findings. Her TFTs revealed:
Questions Percentage 0%
• Random Questions
Thyroid-stimulating hormone (TSH) 3.2 mU/l (0. 35– More
• My Performance 5.0)
• Media Bank
Total tetra-iodothyronine (T4 ) 20 nmol/l (55–144)
• New Multimedia
Online Extras Free T 4 2.6 pmol/l (9–24) Reference: Normal Values
Library
Total tri-iodothyronine (T3 ) 2.5 nmol/l (0. 9–2). Click to
Community open/closeHaematology
Help Which one of the following statements is correct? Click to
open/closeImmunoglobulins
PasTest Store
Her thyroid hormone replacement is adequate Click to
Investigation of pituitary function is required open/closeBiochemistry
My Account Click to open/closeDiabetes
She has tertiary hypothyroidism Click to
Profile She has a thyroiditis open/closeEndocrinology
Click to open/closeBlood
Newsletters She has sick euthyroid syndrome gases Click to
My Career and Exams open/closeCSF
3891
Order History
Learning Goals
Question Filters

Security

Change Password
Sign Out

© 2011 PasTest Ltd | About Us | Contact Us | Help

file:///E|/Shakhawan/Endocrinology/261.htm[3/13/2012 7:14:00 PM]


MyPasTest

Main Navigation
Question Browser: MRCP 1
Home
Subscriptions Question Browser Exam Builder Saved Exams
» MRCP 1
• Question Browser
• Timed Test A 40-year-old woman who has been prescribed thyroid replacement therapy has
• Mock Exam routine thyroid function tests (TFTs). On examination she appeared clinically euthyroid
with no abnormal findings. Her TFTs revealed:
• Past Papers
• Random Questions Thyroid-stimulating hormone (TSH) 3.2 mU/l (0. 35–5.0)
• My Performance Total tetra-iodothyronine (T4 ) 20 nmol/l (55–144)
• Media Bank
Free T 4 2.6 pmol/l (9–24)
• New Multimedia
Online Extras Total tri-iodothyronine (T3 ) 2.5 nmol/l (0. 9–2).
Library
Which one of the following statements is correct?
Community
Help Her thyroid hormone replacement is adequate Your answer
PasTest Store Investigation of pituitary function is required
She has tertiary hypothyroidism
My Account She has a thyroiditis
She has sick euthyroid syndrome
Profile
Newsletters
My Career and Exams
Order History
Thyroid-stimulating hormone (TSH) measurements are used as the standard
Learning Goals method to monitor thyroid hormone replacement. She is clinically euthyroid and
Question Filters her TSH is within the normal range. Tri-iodothyronine (T3 ) and tetra-iodothyronine
(T4 ) exist in a state of dynamic equilibrium, so although her free T 4 is low, her T 3
is within the normal range. Many patients who have adequate thyroid hormone
Security replacement do still complain of symptoms of tiredness, lethargy, and mental
slowing. It has been postulated that the addition of some replacement in the form
Change Password
of T 3 may improve these symptoms. A randomised controlled trial of T 4 +T 3
Sign Out replacement vs T 4 replacement alone is ongoing.

3891

file:///E|/Shakhawan/Endocrinology/261a.htm[3/13/2012 7:14:02 PM]


MyPasTest

Main Navigation
Question Browser: MRCP 1
Home
Subscriptions Question Browser Exam Builder Saved Exams
» MRCP 1
• Question Browser
• Timed Test A woman with history of bipolar affective disorder on lithium presented with infertility.
• Mock Exam BMI= 35. Investigation: TSH 13, free T4 4.8, prolactin 800, US pelvis - polycystic
ovaries.
• Past Papers
• Random Questions What is the best management?
• My Performance
Dopamine agonist
• Media Bank
Thyroxine
• New Multimedia
Stop lithium after consultation with psychiatrist
Online Extras
Metformin
Library
Thyroxine and dopamine agonist
Community
Help 5199

PasTest Store

My Account

Profile
Newsletters
My Career and Exams
Order History
Learning Goals
Question Filters

Security

Change Password
Sign Out

file:///E|/Shakhawan/Endocrinology/262.htm[3/13/2012 7:14:03 PM]


MyPasTest

Main Navigation
Question Browser: MRCP 1
Home
Subscriptions Question Browser Exam Builder Saved Exams
» MRCP 1
• Question Browser
• Timed Test A woman with history of bipolar affective disorder on lithium presented with infertility.
• Mock Exam BMI= 35. Investigation: TSH 13, free T4 4.8, prolactin 800, US pelvis - polycystic
ovaries.
• Past Papers
• Random Questions What is the best management?
• My Performance
Dopamine agonist
• Media Bank
Thyroxine
• New Multimedia
Stop lithium after consultation with psychiatrist Your answer
Online Extras
Metformin
Library
Thyroxine and dopamine agonist
Community
Help
PasTest Store
The answer is stop lithium after consultation with psychiatrist. Long-term
My Account treatment with lithium may produce frank hypothyroidism, as in this case. The
raised prolactin is likely to be related to a combination of lithium therapy and
Profile hypothyroidism. Subfertility in this case may be related to a combination of
hypothyroidism and polycystic ovarian syndrome. The best management in this
Newsletters case would be to discontinue lithium therapy and replace it with another agent
My Career and Exams (after consulting the patient’s psychiatrist). Carbamazepine, sodium valproate and
lamotrigine may all be alternative agents for mood stabilisation. The other serious
Order History
side-effect of long-term lithium use that must be considered is nephrogenic
Learning Goals diabetes insipidus.
Question Filters

5199
Security

Change Password
Sign Out

file:///E|/Shakhawan/Endocrinology/262a.htm[3/13/2012 7:14:04 PM]


MyPasTest

Main Navigation
Question Browser: MRCP 1
Home
Subscriptions Question Browser Exam Builder Saved Exams
» MRCP 1 Session Progress
• Question Browser
Questions Correct 2
• Timed Test An elderly woman came with weight loss of 6 kg over 6
months. Investigation showed low free T3, low free T4 and Questions Incorrect 0
• Mock Exam
low TSH. What is the diagnosis? Questions Total 2
• Past Papers
Questions Percentage 100 %
• Random Questions Hypothyroidism
• My Performance Hypothalamic hypothyroidism More
• Media Bank Anterior hypopituitarism
• New Multimedia Primary hypothyroidism
Online Extras Secondary hypothyroidism Reference: Normal Values
Library
5200 Click to
Community open/closeHaematology
Help Click to
open/closeImmunoglobulins
PasTest Store
Click to
open/closeBiochemistry
My Account Click to open/closeDiabetes
Click to
Profile open/closeEndocrinology
Click to open/closeBlood
Newsletters gases Click to
My Career and Exams open/closeCSF
Order History
Learning Goals
Question Filters

Security

Change Password
Sign Out

© 2011 PasTest Ltd | About Us | Contact Us | Help

file:///E|/Shakhawan/Endocrinology/263.htm[3/13/2012 7:14:06 PM]


MyPasTest

Main Navigation
Question Browser: MRCP 1
Home
Subscriptions Question Browser Exam Builder Saved Exams
» MRCP 1 Session Progress
• Question Browser
Questions Correct 2
• Timed Test An elderly woman came with weight loss of 6 kg over 6
months. Investigation showed low free T3, low free T4 and Questions Incorrect 1
• Mock Exam
low TSH. What is the diagnosis? Questions Total 3
• Past Papers
Questions Percentage 66 %
• Random Questions Hypothyroidism
• My Performance Hypothalamic hypothyroidism More
• Media Bank Anterior hypopituitarism Correct answer
• New Multimedia Primary hypothyroidism
Online Extras Secondary hypothyroidism Your answer
Library
Community
Help
PasTest Store The answer is anterior hypopituitarism. There is a
strong suspicion that this patient has suffered anterior
pituitary failure. Isolated secondary hypothyroidism
My Account would not usually be associated with weight loss. The
weight loss of 6 kg over 6 months raises the possibility
Profile of anterior pituitary failure. Thyroid function testing
confirms secondary hypothyroidism, and this patient
Newsletters
should also be screened for Addison’s disease. If 9 am
My Career and Exams cortisol and ACTH are low, this would point towards a
Order History diagnosis of pituitary failure. The investigation of choice Reference: Normal Values
to elucidate the cause is MRI of the pituitary fossa to
Learning Goals exclude a pituitary tumour. Click to open/closeHaematology
Question Filters Click to open/closeImmunoglobulins
Click to open/closeBiochemistry
5200 Click to open/closeDiabetes Click to
Security open/closeEndocrinology Click to
open/closeBlood gases Click to
Change Password open/closeCSF
Sign Out

© 2011 PasTest Ltd | About Us | Contact Us | Help

file:///E|/Shakhawan/Endocrinology/263a.htm[3/13/2012 7:14:07 PM]


MyPasTest

Main Navigation
Question Browser: MRCP 1
Home
Subscriptions Question Browser Exam Builder Saved Exams
» MRCP 1 Session Progress
• Question Browser
Questions Correct 2
• Timed Test A 23-year-old woman presented with painful oral ulcers.
Her mother had history of thyroidectomy. Which of the Questions Incorrect 1
• Mock Exam
following would suggest polyglandular syndrome Questions Total 3
• Past Papers type 1?
Questions Percentage 66 %
• Random Questions
• My Performance Celiac disease More
• Media Bank Type 2 diabetes mellitus

• New Multimedia Hypocalcaemia

Online Extras Hypercalcaemia


Reference: Normal Values
Library Diabetes insipidus
Click to
Community 5201 open/closeHaematology
Help Click to
open/closeImmunoglobulins
PasTest Store
Click to
open/closeBiochemistry
My Account Click to open/closeDiabetes
Click to
Profile open/closeEndocrinology
Click to open/closeBlood
Newsletters gases Click to
My Career and Exams open/closeCSF
Order History
Learning Goals
Question Filters

Security

Change Password
Sign Out

© 2011 PasTest Ltd | About Us | Contact Us | Help

file:///E|/Shakhawan/Endocrinology/264.htm[3/13/2012 7:14:09 PM]


MyPasTest

Main Navigation
Question Browser: MRCP 1
Home
Subscriptions Question Browser Exam Builder Saved Exams
» MRCP 1
• Question Browser
• Timed Test A 23-year-old woman presented with painful oral ulcers. Her mother had history of
• Mock Exam thyroidectomy. Which of the following would suggest polyglandular syndrome
type 1?
• Past Papers
• Random Questions Celiac disease Your answer
• My Performance Type 2 diabetes mellitus
• Media Bank Hypocalcaemia Correct answer
• New Multimedia Hypercalcaemia
Online Extras Diabetes insipidus
Library
Community
Help
PasTest Store The answer is hypocalcaemia. Polyglandular syndrome type 1 is an autosomal
recessive condition associated with hypoparathyroidism in around 90%,
mucocutaneous candidiasis, adrenal insufficiency in around 60%, primary gonadal
My Account failure, primary hypothyroidism, and rarely hypopituitarism or diabetes insipidus.
There may be associated malabsorption, pernicious anaemia, chronic active
Profile hepatitis or vitiligo. Polyglandular syndrome type 2 may be autosomal-recessive,
autosomal-dominant or polygenic. Adrenal insufficiency occurs in all patients, and
Newsletters
there may be associated hypothyroidism, type-1 diabetes, gonadal failure, and
My Career and Exams rarely diabetes insipidus. Associated conditions include vitiligo, myasthenia gravis,
Order History alopecia, immune thrombocytopaenic purpura or pernicious anaemia.

Learning Goals
Question Filters 5201

Security

Change Password
Sign Out

file:///E|/Shakhawan/Endocrinology/264a.htm[3/13/2012 7:14:10 PM]


MyPasTest

Main Navigation
Question Browser: MRCP 1
Home
Subscriptions Question Browser Exam Builder Saved Exams
» MRCP 1 Session Progress
• Question Browser
Questions Correct 2
• Timed Test An obese lady has come for poor diabetic control, despite
being on gliclazide 160 mg bd. Biochemistry: mildly raised Questions Incorrect 2
• Mock Exam
urea, Cr 160, ALT, GGT normal. HbA1c 9.4. Questions Total 4
• Past Papers
Which of the following would you start next? Questions Percentage 50 %
• Random Questions
• My Performance More
Acarbose
• Media Bank
Guar gum
• New Multimedia
Repaglinide
Online Extras Reference: Normal Values
Metformin
Library
Pioglitazone Click to
Community open/closeHaematology
Help 5202 Click to
open/closeImmunoglobulins
PasTest Store
Click to
open/closeBiochemistry
My Account Click to open/closeDiabetes
Click to
Profile open/closeEndocrinology
Click to open/closeBlood
Newsletters gases Click to
My Career and Exams open/closeCSF
Order History
Learning Goals
Question Filters

Security

Change Password
Sign Out

© 2011 PasTest Ltd | About Us | Contact Us | Help

file:///E|/Shakhawan/Endocrinology/265.htm[3/13/2012 7:14:11 PM]


MyPasTest

Main Navigation
Question Browser: MRCP 1
Home
Subscriptions Question Browser Exam Builder Saved Exams
» MRCP 1 Session Progress
• Question Browser
Questions Correct 3
• Timed Test An obese lady has come for poor diabetic control, despite
being on gliclazide 160 mg bd. Biochemistry: mildly raised Questions Incorrect 2
• Mock Exam
urea, Cr 160, ALT, GGT normal. HbA1c 9.4. Questions Total 5
• Past Papers
Which of the following would you start next? Questions Percentage 60 %
• Random Questions
• My Performance More
Acarbose
• Media Bank
Guar gum
• New Multimedia
Repaglinide
Online Extras
Metformin
Library
Pioglitazone Your answer
Community
Help
PasTest Store
The answer is pioglitazone. Her raised creatinine
My Account precludes her from metformin therapy. She is also
obese, which would suggest some insulin resistance. As
Profile such, the best choice for additional therapy would be
pioglitazone. Pioglitazone is a PPAR - γ agonist that acts
Newsletters
to improve glucose uptake into muscle and fat by
My Career and Exams reducing peripheral insulin resistance. It may be used at
Order History doses of 4 or 8 mg. Initial and then periodic monitoring Reference: Normal Values
of liver function tests is recommended. Data suggest
Learning Goals that, by targeting insulin resistance, pioglitazone may Click to open/closeHaematology
Question Filters bring about sustained improvements in glycaemic Click to open/closeImmunoglobulins
control and improvement in other factors, such as Click to open/closeBiochemistry
raised blood pressure, that are associated with the Click to open/closeDiabetes Click to
Security insulin-resistance syndrome. Recent studies suggest open/closeEndocrinology Click to
that congestive cardiac failure due to fluid retention open/closeBlood gases Click to
Change Password occurs in 0.5-0.8% of glitazone patients, but this is open/closeCSF
Sign Out outweighed by the advantages associated with
improving her glycaemic control.

5202

© 2011 PasTest Ltd | About Us | Contact Us | Help

file:///E|/Shakhawan/Endocrinology/265a.htm[3/13/2012 7:14:13 PM]


MyPasTest

Main Navigation
Question Browser: MRCP 1
Home
Subscriptions Question Browser Exam Builder Saved Exams
» MRCP 1
• Question Browser
• Timed Test A 32-year-old patient is being investigated for polydipsia. There was no recent loss of
• Mock Exam weight. Investigation: Na, K, Ca, glucose, urea, Cr normal; serum osmolality 290
(normal), urine osmolality 200 (normal). What is the most likely diagnosis?
• Past Papers
• Random Questions Nephrogenic diabetes insipidus
• My Performance Psychogenic polydipsia
• Media Bank Syndrome of inappropriate antidiuretic hormone secretion (SIADH)
• New Multimedia Cranial diabetes insipidus
Online Extras Water intoxication
Library
5203
Community
Help
PasTest Store

My Account

Profile
Newsletters
My Career and Exams
Order History
Learning Goals
Question Filters

Security

Change Password
Sign Out

file:///E|/Shakhawan/Endocrinology/266.htm[3/13/2012 7:14:14 PM]


MyPasTest

Main Navigation
Question Browser: MRCP 1
Home
Subscriptions Question Browser Exam Builder Saved Exams
» MRCP 1 Session Progress
• Question Browser
Questions Correct 4
• Timed Test A 32-year-old patient is being investigated for polydipsia.
There was no recent loss of weight. Investigation: Na, K, Questions Incorrect 2
• Mock Exam
Ca, glucose, urea, Cr normal; serum osmolality 290 Questions Total 6
• Past Papers (normal), urine osmolality 200 (normal). What is the
Questions Percentage 66 %
• Random Questions most likely diagnosis?
• My Performance More
Nephrogenic diabetes insipidus
• Media Bank
Psychogenic polydipsia Your answer
• New Multimedia
Syndrome of inappropriate antidiuretic
Online Extras hormone secretion (SIADH)
Library Cranial diabetes insipidus
Community Water intoxication
Help
PasTest Store

My Account The answer is psychogenic polydipsia. This patient’s


serum and urine osmolalities are in the normal range. In
Profile patients with polydipsia and polyuria in whom calcium
and glucose levels are normal, diabetes insipidus is
Newsletters
much rarer than ‘psychogenic’ or primary polydipsia.
My Career and Exams Patients with diabetes insipidus normally present with
Order History some degree of dehydration, with urea levels above the Reference: Normal Values
upper limit of the normal range. Diagnosis if diabetes
Learning Goals insipidus (DI) is with the water deprivation test, where Click to open/closeHaematology
Question Filters in normal subjects urine osmolality progressively rises, Click to open/closeImmunoglobulins
although in patients with DI, the urine fails to Click to open/closeBiochemistry
concentrate. Cranial diabetes insipidus responds to Click to open/closeDiabetes Click to
Security vasopressin, although in nephrogenic DI there is little or open/closeEndocrinology Click to
no response to vasopressin. open/closeBlood gases Click to
Change Password open/closeCSF
Sign Out
5203

© 2011 PasTest Ltd | About Us | Contact Us | Help

file:///E|/Shakhawan/Endocrinology/266a.htm[3/13/2012 7:14:15 PM]


MyPasTest

Main Navigation
Question Browser: MRCP 1
Home
Subscriptions Question Browser Exam Builder Saved Exams
» MRCP 1
• Question Browser
• Timed Test You review a 28-year-old girl who has been referred with amenorrhoea. She is noted
• Mock Exam on routine screening to have a raised prolactin level. She has read about her condition
on the Internet and has some questions about prolactin physiology.
• Past Papers
• Random Questions Thinking of hormones in general, which of the following hormones is under
continuous inhibition?
• My Performance
• Media Bank Prolactin
• New Multimedia Growth hormone
Online Extras Adrenocorticotrophic hormone
Library Thyroid-releasing hormone
Community Testosterone
Help
5204
PasTest Store

My Account

Profile
Newsletters
My Career and Exams
Order History
Learning Goals
Question Filters

Security

Change Password
Sign Out

file:///E|/Shakhawan/Endocrinology/267.htm[3/13/2012 7:14:17 PM]


MyPasTest

Main Navigation
Question Browser: MRCP 1
Home
Subscriptions Question Browser Exam Builder Saved Exams
» MRCP 1 Session Progress
• Question Browser
Questions Correct 5
• Timed Test You review a 28-year-old girl who has been referred with
amenorrhoea. She is noted on routine screening to have a Questions Incorrect 2
• Mock Exam
raised prolactin level. She has read about her condition on Questions Total 7
• Past Papers the Internet and has some questions about prolactin
Questions Percentage 71 %
• Random Questions physiology.
• My Performance More
Thinking of hormones in general, which of the
• Media Bank following hormones is under continuous inhibition?
• New Multimedia
Prolactin Your answer
Online Extras
Growth hormone
Library
Adrenocorticotrophic hormone
Community
Thyroid-releasing hormone
Help
Testosterone
PasTest Store

My Account

Profile Prolactin is under predominantly inhibitory control, by


levels of dopamine. Prolactin levels are known to rise
Newsletters during pregnancy, lactation, severe stress, sleep and
My Career and Exams coitus. Mildly increased prolactin levels in the range of
400–600 mU/litre may be physiological, but higher Reference: Normal Values
Order History
levels require a diagnosis. Levels above 1000 imply the
Learning Goals possibility of a microprolactinoma, and levels above Click to open/closeHaematology
Question Filters 5000 imply the presence of a macroprolactinoma. Drug Click to open/closeImmunoglobulins
causes of hyperprolactinaemia are, not surprisingly, the Click to open/closeBiochemistry
dopamine antagonists, which include metoclopramide, Click to open/closeDiabetes Click to
Security domperidone and the phenothiazines. Features of open/closeEndocrinology Click to
hyperprolactinaemia in women include amenorrhoea, open/closeBlood gases Click to
Change Password galactorrhoea, loss of libido, subfertility and features of open/closeCSF
androgen deficiency.
Sign Out

5204

© 2011 PasTest Ltd | About Us | Contact Us | Help

file:///E|/Shakhawan/Endocrinology/267a.htm[3/13/2012 7:14:18 PM]


MyPasTest

Main Navigation
Question Browser: MRCP 1
Home
Subscriptions Question Browser Exam Builder Saved Exams
» MRCP 1 Session Progress
• Question Browser
Questions Correct 5
• Timed Test <font ">An obese patient presented with increased
abdominal striae, and 12-midnight cortisol was elevated. Questions Incorrect 2
• Mock Exam
He has hypertension with a BP of 155/82 mmHg, type 2 Questions Total 7
• Past Papers diabetes and has recently suffered a left Colle's fracture.
Questions Percentage 71 %
• Random Questions
Which of the following would best confirm the More
• My Performance diagnosis of Cushing's disease?
• Media Bank
• New Multimedia <font ">Low-dose dexamethasone suppression test

Online Extras <font ">Synacthen® test


Reference: Normal Values
Library <font ">24-hour urinary cortisol collection
<font ">High-dose dexamethasone suppression test Click to
Community open/closeHaematology
Help <font ">Basal adrenocorticotropic hormone (ACTH) Click to
levels open/closeImmunoglobulins
PasTest Store
Click to
5205 open/closeBiochemistry
My Account Click to open/closeDiabetes
Click to
Profile open/closeEndocrinology
Click to open/closeBlood
Newsletters gases Click to
My Career and Exams open/closeCSF
Order History
Learning Goals
Question Filters

Security

Change Password
Sign Out

© 2011 PasTest Ltd | About Us | Contact Us | Help

file:///E|/Shakhawan/Endocrinology/268.htm[3/13/2012 7:14:20 PM]


MyPasTest

Main Navigation
Question Browser: MRCP 1
Home
Subscriptions Question Browser Exam Builder Saved Exams
» MRCP 1
• Question Browser
• Timed Test <font ">An obese patient presented with increased abdominal striae, and 12-midnight
• Mock Exam cortisol was elevated. He has hypertension with a BP of 155/82 mmHg, type 2
diabetes and has recently suffered a left Colle's fracture.
• Past Papers
• Random Questions Which of the following would best confirm the diagnosis of Cushing's
disease?
• My Performance
• Media Bank <font ">Low-dose dexamethasone suppression test
• New Multimedia <font ">Synacthen® test
Online Extras <font ">24-hour urinary cortisol collection
Library <font ">High-dose dexamethasone suppression test Correct answer
Community <font ">Basal adrenocorticotropic hormone (ACTH) levels Your answer
Help
PasTest Store

My Account The answer is high-dose dexamethasone suppression test. The high-dose


dexamethasone suppression test involves administration of 2 mg dexamethasone
Profile 6-hourly for a 48-hour period. In patients with Cushing’s disease, the cortisol falls
by more than 50% of the basal value, though it should be noted that 10% of
Newsletters patients with Cushing’s disease fail to suppress. Unfortunately, there is
My Career and Exams considerable overlap between basal ACTH levels in pituitary- and non-pituitary-
dependant Cushing’s syndrome, so that this is not a reliable test. Inferior petrosal
Order History
sinus sampling is the gold-standard test to establish a diagnosis of pituitary-
Learning Goals dependent Cushing’s syndrome. This sampling is usually done in conjunction with
Question Filters corticotrophin-releasing hormone. Whilst a low dose dexamethasone test is the
usual first screening investigation, it doesn't add incrementally to our knowledge
versus the high dose alternative, as we are already aware of the raised midnight
Security cortisol.

Change Password
Sign Out 5205

file:///E|/Shakhawan/Endocrinology/268a.htm[3/13/2012 7:14:21 PM]


MyPasTest

Main Navigation
Question Browser: MRCP 1
Home
Subscriptions Question Browser Exam Builder Saved Exams
» MRCP 1 Session Progress
• Question Browser
Questions Correct 5
• Timed Test A 42-year-old woman is diagnosed with
phaeochromocytoma. Screening with pentagastrin testing Questions Incorrect 3
• Mock Exam
suggests that she has medullary thyroid carcinoma, and Questions Total 8
• Past Papers you plan a thyroidectomy.
Questions Percentage 62 %
• Random Questions
What anti-hypertensive medication should be started More
• My Performance before surgery for a patient with
• Media Bank phaeochromocytoma?
• New Multimedia
Phenoxybenzamine
Online Extras Reference: Normal Values
Atenolol
Library
Labetalol Click to
Community open/closeHaematology
Ramipril
Help Click to
Doxazosin open/closeImmunoglobulins
PasTest Store
Click to
5206 open/closeBiochemistry
My Account Click to open/closeDiabetes
Click to
Profile open/closeEndocrinology
Click to open/closeBlood
Newsletters gases Click to
My Career and Exams open/closeCSF
Order History
Learning Goals
Question Filters

Security

Change Password
Sign Out

© 2011 PasTest Ltd | About Us | Contact Us | Help

file:///E|/Shakhawan/Endocrinology/269.htm[3/13/2012 7:14:22 PM]


MyPasTest

Main Navigation
Question Browser: MRCP 1
Home
Subscriptions Question Browser Exam Builder Saved Exams
» MRCP 1 Session Progress
• Question Browser
Questions Correct 6
• Timed Test A 42-year-old woman is diagnosed with
phaeochromocytoma. Screening with pentagastrin testing Questions Incorrect 3
• Mock Exam
suggests that she has medullary thyroid carcinoma, and Questions Total 9
• Past Papers you plan a thyroidectomy.
Questions Percentage 66 %
• Random Questions
What anti-hypertensive medication should be started More
• My Performance before surgery for a patient with
• Media Bank phaeochromocytoma?
• New Multimedia
Phenoxybenzamine Your answer
Online Extras
Atenolol
Library
Labetalol
Community
Ramipril
Help
Doxazosin
PasTest Store

My Account

Profile Phenoxybenzamine is a more potent α -blocker than


Newsletters doxazosin and is the recommended choice for initial
anti-hypertensive medication in phaeochromocytoma
My Career and Exams where surgery is contemplated. It should be given in
Order History divided doses, at 20 mg initially, increasing to 80 mg. Reference: Normal Values
Patients should then be considered for additional β -
Learning Goals
blockade with propranolol, again in divided doses up to Click to open/closeHaematology
Question Filters 240 mg/day. Always start with α -blockade first, as Click to open/closeImmunoglobulins
initial use of β -blockers may worsen hypertension. Click to open/closeBiochemistry
Labetalol is not recommended. An experienced surgeon Click to open/closeDiabetes Click to
Security open/closeEndocrinology Click to
and anaesthetist are crucial to attain the best surgical
result. open/closeBlood gases Click to
Change Password open/closeCSF
Sign Out
5206

© 2011 PasTest Ltd | About Us | Contact Us | Help

file:///E|/Shakhawan/Endocrinology/269a.htm[3/13/2012 7:14:24 PM]


MyPasTest

Main Navigation
Question Browser: MRCP 1
Home
Subscriptions Question Browser Exam Builder Saved Exams
» MRCP 1
• Question Browser
• Timed Test A 51-year-old white man was recently diagnosed with a solitary 2.7- cm papillary
• Mock Exam cancer of the thyroid with no invasion of the capsule, no lymphadenopathy and no
distant metastases. He denies a history of head and neck irradiation, hoarseness,
• Past Papers pain, dysphagia or haemoptysis. His physical exam is otherwise normal, with no lab
• Random Questions abnormailities. Which of the following measures is most appropriate for his
management?
• My Performance
• Media Bank Partial thyroidectomy followed by radioactive iodine (RAI) treatment
• New Multimedia Near-total thyroidectomy followed by RAI treatment
Online Extras Thyroid hormone treatment
Library Partial thyroidectomy, RAI treatment and thyroid hormone treatment
Community Near-total thyroidectomy, RAI treatment and thyroid hormone treatment
Help
5277
PasTest Store

My Account

Profile
Newsletters
My Career and Exams
Order History
Learning Goals
Question Filters

Security

Change Password
Sign Out

file:///E|/Shakhawan/Endocrinology/270.htm[3/13/2012 7:14:25 PM]


MyPasTest

Main Navigation
Question Browser: MRCP 1
Home
Subscriptions Question Browser Exam Builder Saved Exams
» MRCP 1 Session Progress
• Question Browser
Questions Correct 7
• Timed Test A 51-year-old white man was recently diagnosed with a
solitary 2.7- cm papillary cancer of the thyroid with no Questions Incorrect 3
• Mock Exam
invasion of the capsule, no lymphadenopathy and no distant Questions Total 10
• Past Papers metastases. He denies a history of head and neck
Questions Percentage 70 %
• Random Questions irradiation, hoarseness, pain, dysphagia or haemoptysis.
His physical exam is otherwise normal, with no lab More
• My Performance
abnormailities. Which of the following measures is
• Media Bank most appropriate for his management?
• New Multimedia
Partial thyroidectomy followed by
Online Extras radioactive iodine (RAI) treatment
Library Near-total thyroidectomy followed by
Community RAI treatment
Help Thyroid hormone treatment
PasTest Store Partial thyroidectomy, RAI treatment
and thyroid hormone treatment
Near-total thyroidectomy, RAI Your answer
My Account
treatment and thyroid hormone
Profile treatment

Newsletters
My Career and Exams
Order History Reference: Normal Values
Thyroid cancer remains a significant medical problem in
Learning Goals the United States; 12,000 new cases are diagnosed and Click to open/closeHaematology
Question Filters 1000 deaths are reported each year. Differentiated Click to open/closeImmunoglobulins
thyroid cancer is classified into follicular and papillary Click to open/closeBiochemistry
(derived from the follicular cells), medullary thyroid Click to open/closeDiabetes Click to
Security carcinoma (derived from the C cells) and poorly open/closeEndocrinology Click to
differentiated or anaplastic carcinoma. Rarely, the open/closeBlood gases Click to
Change Password thyroid is the site of involvement by lymphoma. The open/closeCSF
Sign Out most common type of thyroid cancer is papillary cancer,
which accounts for approximately 70% of all thyroid
cancers. It is two to three times more common in
females and peaks in the third and fourth decades of
life. Papillary cancer is usually non-encapsulated and
sometimes multi-focal and tends to spread by the
lymphatic route.

Follicular cancer is the second most common form of


thyroid cancer, accounting for 15% of all thyroid
cancers. It affects a slightly older age group and is
more commonly diagnosed in females than in males.
Follicular cancer tends to be encapsulated, is usually
unifocal and tends to spread via the haematogenous
route; early metastases are seen with small lesions.
Thyroid cancer is now diagnosed at an early stage and
its slow rate of growth makes for a favourable outcome
in a majority of cases. Sometimes, however, thyroid
tumours are encountered that display aggressive
features leading to early death despite aggressive
treatment. In papillary cancer, prognosis is affected by
tumour size, presence or absence of metastases,
patient age and degree of differentiation. Generally, the
smaller tumours (< 1.5 cm) carry an excellent
prognosis in the absence of metastasis, whereas larger
tumours (> 2.5 cm) tend to carry a poorer prognosis.

Patient age greater than 40 years at diagnosis tends to


carry a poor prognosis in part because of poor

file:///E|/Shakhawan/Endocrinology/270a.htm[3/13/2012 7:14:27 PM]


MyPasTest

concentration of iodine by most tumours. Poorly


differentiated tumours tend to run a more aggressive
course. The first line of treatment of thyroid cancer
consists of surgical resection. Although the optimum
procedure is not known, the more aggressive tumours
should be managed with more extensive procedures
(near-total or total thyroidectomy with or without
lymph node dissection). RAI ablation should be
considered when residual or metastatic disease is
present. Finally, thyroid hormone treatment should be
used with a goal of keeping the TSH level as low as
possible without causing overt hyperthyroidism. RAI
ablation and thyroid hormone suppression have been
shown to reduce recurrence of thyroid cancer. In this
patient, age and tumour size predict a poor outcome.
Treatment should, therefore, consist of near-total
thyroidectomy, RAI ablation and thyroid hormone
treatment.

5277

© 2011 PasTest Ltd | About Us | Contact Us | Help

file:///E|/Shakhawan/Endocrinology/270a.htm[3/13/2012 7:14:27 PM]


MyPasTest

Main Navigation
Question Browser: MRCP 1
Home
Subscriptions Question Browser Exam Builder Saved Exams
» MRCP 1
• Question Browser
• Timed Test You saw a 71-year-old white woman, a nursing home resident who was brought in by
• Mock Exam her daughter for a complete physical examination. Her complaints include a poor
appetite, weight loss, cramps and weakness. She was diagnosed with Crohn's disease
• Past Papers 10 years ago but is not taking any medications. Five months ago she had a
• Random Questions mammogram and flexible sigmoidoscopy, both of which were normal. Because her
exam was normal, she was given a 1-month return appointment and sent for blood
• My Performance
work. At the end of the day, your lab calls to report a panic value of calcium of 1.4
• Media Bank mmol/l (normal range, 2.2-2.6 mmol/l) with an inorganic phosphate of 0.58 mmol/l
• New Multimedia (normal range, 0.8-1.5 mmol/l). She has a creatinine of 80 μmol/l (normal range, 60-
Online Extras 110 μmol/l), albumin is 35 g/l (normal range, 37-49 g/l) and alkaline phosphatase is
250 U/l (normal range, 42-98 U/l).
Library
Community Which of the following diagnoses is most compatible with these lab data?

Help Hypoparathyroidism
PasTest Store Hypomagnesemia
Vitamin D deficiency
My Account Renal failure
Profile Dietary calcium deficiency
Newsletters
5278
My Career and Exams
Order History
Learning Goals
Question Filters

Security

Change Password
Sign Out

file:///E|/Shakhawan/Endocrinology/271.htm[3/13/2012 7:14:28 PM]


MyPasTest

Main Navigation
Question Browser: MRCP 1
Home
Subscriptions Question Browser Exam Builder Saved Exams
» MRCP 1 Session Progress
• Question Browser
Questions Correct 7
• Timed Test You saw a 71-year-old white woman, a nursing home
resident who was brought in by her daughter for a Questions Incorrect 4
• Mock Exam
complete physical examination. Her complaints include a Questions Total 11
• Past Papers poor appetite, weight loss, cramps and weakness. She was
Questions Percentage 63 %
• Random Questions diagnosed with Crohn's disease 10 years ago but is not
taking any medications. Five months ago she had a More
• My Performance
mammogram and flexible sigmoidoscopy, both of which
• Media Bank were normal. Because her exam was normal, she was given
• New Multimedia a 1-month return appointment and sent for blood work. At
the end of the day, your lab calls to report a panic value of
Online Extras calcium of 1.4 mmol/l (normal range, 2.2-2.6 mmol/l) with
Library an inorganic phosphate of 0.58 mmol/l (normal range, 0.8-
1.5 mmol/l). She has a creatinine of 80 μmol/l (normal
Community
range, 60-110 μmol/l), albumin is 35 g/l (normal range,
Help 37-49 g/l) and alkaline phosphatase is 250 U/l (normal
PasTest Store range, 42-98 U/l).

Which of the following diagnoses is most compatible


My Account with these lab data?

Profile Hypoparathyroidism
Newsletters Hypomagnesemia Your answer
My Career and Exams Vitamin D deficiency Correct answer
Order History Renal failure Reference: Normal Values
Learning Goals Dietary calcium deficiency Click to open/closeHaematology
Question Filters Click to open/closeImmunoglobulins
Click to open/closeBiochemistry
Click to open/closeDiabetes Click to
Security open/closeEndocrinology Click to
The causes of hypocalcemia, an abnormal reduction of open/closeBlood gases Click to
Change Password serum calcium, can quickly be determined by examining open/closeCSF
Sign Out the serum phosphate, creatinine and calcium. In
hypoparathyroidism, there is reduced mobilisation of
calcium from bone, reduced renal reabsorption of
calcium (along with decreased phosphaturia) and
reduced formation of 1,25-hydroxyvitamin D, resulting
in reduced intestinal absorption of calcium.
Consequently, the hypocalcemia is accompanied by
hyperphosphatemia. Hypoparathyroidism can be
congenital or acquired; the latter is accounted for by
transient or permanent disorders. Hypomagnesemia
causes deficient secretion of parathyroid hormone (PTH)
and consequent functional hypoparathyroidism. In
vitamin D deficiency, decreased intestinal absorption of
calcium leads to secondary hyperparathyroidism, which
increases renal tubular loss of phosphate. Vitamin D
deficiency can result from inadequate dietary intake,
lack of sun exposure and malabsorption in this case due
to the long-standing Crohn’s disease. Renal failure
impairs hydroxylation of 25-hydroxyvitamin D, which
results in the malabsorption of calcium. The body
compensates by increased secretion of PTH, leading to
increased mobilisation of calcium from bone. Renal
failure is characterized by an abnormal serum
creatinine, whereas renal dysfunction is not the critical
pathogenetic feature of the other forms of
hypocalcemia.

file:///E|/Shakhawan/Endocrinology/271a.htm[3/13/2012 7:14:29 PM]


MyPasTest

Main Navigation
Question Browser: MRCP 1
Home
Subscriptions Question Browser Exam Builder Saved Exams
» MRCP 1 Session Progress
• Question Browser
Questions Correct 7
• Timed Test A 38-year-old black woman draws your attention to a
swelling in her neck, which she noticed 2 days ago. She Questions Incorrect 4
• Mock Exam
denies palpitations, diaphoresis and weight loss. There is no Questions Total 11
• Past Papers pain, hoarseness or dysphagia. Her medical history is
Questions Percentage 63 %
• Random Questions notable only for hypertension. Medications include only
atenolol 50 mg once daily. On exam, blood pressure is More
• My Performance
150/80 mm Hg; pulse is 70. There is a 2 x 1-cm non-
• Media Bank tender nodule on the right lobe of the thyroid. No
• New Multimedia lymphadenopathy is detected. The remainder of the exam
is unremarkable. Electrolytes, urea, creatinine, liver
Online Extras function tests, calcium, phosphate and FBC are normal. Reference: Normal Values
Library
What would you do next? Click to
Community open/closeHaematology
Help Arrange a thyroid ultrasound scan Click to
Elicit a family history of thyroid cancer open/closeImmunoglobulins
PasTest Store
Click to
Obtain thyroid function tests open/closeBiochemistry
My Account Perform fine-needle aspiration Click to open/closeDiabetes
Click to
All of the above open/closeEndocrinology
Profile
Click to open/closeBlood
Newsletters 5279 gases Click to
My Career and Exams open/closeCSF
Order History
Learning Goals
Question Filters

Security

Change Password
Sign Out

© 2011 PasTest Ltd | About Us | Contact Us | Help

file:///E|/Shakhawan/Endocrinology/272.htm[3/13/2012 7:14:31 PM]


MyPasTest

Main Navigation
Question Browser: MRCP 1
Home
Subscriptions Question Browser Exam Builder Saved Exams
» MRCP 1
• Question Browser
• Timed Test A 38-year-old black woman draws your attention to a swelling in her neck, which she
• Mock Exam noticed 2 days ago. She denies palpitations, diaphoresis and weight loss. There is no
pain, hoarseness or dysphagia. Her medical history is notable only for hypertension.
• Past Papers Medications include only atenolol 50 mg once daily. On exam, blood pressure is
• Random Questions 150/80 mm Hg; pulse is 70. There is a 2 x 1-cm non-tender nodule on the right lobe
of the thyroid. No lymphadenopathy is detected. The remainder of the exam is
• My Performance
unremarkable. Electrolytes, urea, creatinine, liver function tests, calcium, phosphate
• Media Bank and FBC are normal.
• New Multimedia
What would you do next?
Online Extras
Library Arrange a thyroid ultrasound scan Your answer
Community Elicit a family history of thyroid cancer
Help Obtain thyroid function tests
PasTest Store Perform fine-needle aspiration
All of the above Correct answer

My Account

Profile
Newsletters The clinically apparent (>1 cm) thyroid nodule is a common clinical finding; up to
My Career and Exams 5% of the population is affected. It is more common in women than in men and a
majority (85%) are hypofunctional or cold nodules. The likelihood of malignancy in
Order History
a solitary thyroid nodule is low (4%); cold nodules carry a higher risk than hot
Learning Goals nodules (20% vs 1%). Evaluation of a solitary nodule should be aimed at detecting
Question Filters potentially malignant lesions so that as many cancers are removed with as few
operations as possible. A history of head and neck irradiation raises the likelihood
that a thyroid nodule is malignant, as does the presence of a family history of
Security differentiated thyroid cancer or medullary cancer of the thyroid (which can be a
component of multiple endocrine neoplasia (MEN) type IIA or IIB). Fine-needle
Change Password aspiration of the thyroid gland is a cost-effective procedure with a high sensitivity
and specificity for malignancy. Fine-needle aspiration allows the nodule to be
Sign Out
characterized cytologically as benign, malignant, suspicious for malignancy or
indeterminate.

5279

file:///E|/Shakhawan/Endocrinology/272a.htm[3/13/2012 7:14:33 PM]


MyPasTest

Main Navigation
Question Browser: MRCP 1
Home
Subscriptions Question Browser Exam Builder Saved Exams
» MRCP 1 Session Progress
• Question Browser
Questions Correct 7
• Timed Test A 60-year-old white man comes to see you for chronic
back pain, which worsened 1 week ago. He has been Questions Incorrect 5
• Mock Exam
wheelchair bound for 6 months because of severe Questions Total 12
• Past Papers osteoporosis with multiple lumbosacral spine fractures. He
Questions Percentage 58 %
• Random Questions has severe asthma, which has required large doses of
glucocorticoids for many years. The patient reports More
• My Performance
progressive loss of height and kyphosis over the past year.
• Media Bank Other medications include salbutamol and ipratropium
• New Multimedia inhalers and long-acting theophylline 300 mg twice a day.
Significant physical findings include bilateral cataracts,
Online Extras multiple ecchymoses and a prolonged expiratory phase with Reference: Normal Values
Library bilateral wheezes. Which of the following treatments is
first choice in this patient? Click to
Community open/closeHaematology
Help Testosterone replacement Click to
open/closeImmunoglobulins
PasTest Store Vitamin D supplementation
Click to
Calcium supplementation open/closeBiochemistry
My Account Bisphosphonate therapy Click to open/closeDiabetes
Click to
Profile Calcitonin therapy open/closeEndocrinology
Click to open/closeBlood
Newsletters 5280 gases Click to
My Career and Exams open/closeCSF
Order History
Learning Goals
Question Filters

Security

Change Password
Sign Out

© 2011 PasTest Ltd | About Us | Contact Us | Help

file:///E|/Shakhawan/Endocrinology/273.htm[3/13/2012 7:14:34 PM]


MyPasTest

Main Navigation
Question Browser: MRCP 1
Home
Subscriptions Question Browser Exam Builder Saved Exams
» MRCP 1
• Question Browser
• Timed Test A 60-year-old white man comes to see you for chronic back pain, which worsened 1
• Mock Exam week ago. He has been wheelchair bound for 6 months because of severe
osteoporosis with multiple lumbosacral spine fractures. He has severe asthma, which
• Past Papers has required large doses of glucocorticoids for many years. The patient reports
• Random Questions progressive loss of height and kyphosis over the past year. Other medications include
salbutamol and ipratropium inhalers and long-acting theophylline 300 mg twice a day.
• My Performance
Significant physical findings include bilateral cataracts, multiple ecchymoses and a
• Media Bank prolonged expiratory phase with bilateral wheezes. Which of the following
• New Multimedia treatments is first choice in this patient?

Online Extras Testosterone replacement


Library Vitamin D supplementation
Community Calcium supplementation
Help Bisphosphonate therapy Your answer
PasTest Store Calcitonin therapy

My Account

Profile
This patient has significant osteoporosis, almost certainly related to chronic use of
Newsletters
glucocorticoid therapy for his asthma. First-line therapy in this case would be
My Career and Exams bisphosphonates, which improve bone mass in the lumbar spine and the hip. Older
Order History preparations are associated with significant gastrointestinal (GI) side-effects
(predominantly oesophagitis and ulceration), so that newer treatments that may
Learning Goals be dosed weekly over significant advantages in terms of side-effect reduction.
Question Filters Calcium and vitamin D supplementation would be a viable alternative in patients
not tolerating bisphosphonates. Testosterone supplementation is only of use in
patients with low testosterone levels, where it does improve bone mass.
Security

Change Password 5280


Sign Out

file:///E|/Shakhawan/Endocrinology/273a.htm[3/13/2012 7:14:36 PM]


MyPasTest

Main Navigation
Question Browser: MRCP 1
Home
Subscriptions Question Browser Exam Builder Saved Exams
» MRCP 1
• Question Browser
• Timed Test A 60-year-old white man comes to see you for chronic back pain, which worsened 1
• Mock Exam week ago. He has been wheelchair bound for 6 months because of severe
osteoporosis with multiple lumbosacral spine fractures. He has severe asthma, which
• Past Papers has required large doses of glucocorticoids for many years. The patient reports
• Random Questions progressive loss of height and kyphosis over the past year. Other medications include
salbutamol and ipratropium inhalers and long-acting theophylline 300 mg twice a day.
• My Performance
Significant physical findings include bilateral cataracts, multiple ecchymoses and a
• Media Bank prolonged expiratory phase with bilateral wheezes. Which of the following
• New Multimedia mechanisms underlies his osteoporosis?

Online Extras Decreased bone formation


Library Increased bone loss
Community Decreased calcium absorption from the gastrointestinal (GI) tract
Help Increased calcium loss in urine
PasTest Store All of the above

5281
My Account

Profile
Newsletters
My Career and Exams
Order History
Learning Goals
Question Filters

Security

Change Password
Sign Out

file:///E|/Shakhawan/Endocrinology/274.htm[3/13/2012 7:14:37 PM]


MyPasTest

Main Navigation
Question Browser: MRCP 1
Home
Subscriptions Question Browser Exam Builder Saved Exams
» MRCP 1
• Question Browser
• Timed Test A 60-year-old white man comes to see you for chronic back pain, which worsened 1
• Mock Exam week ago. He has been wheelchair bound for 6 months because of severe
osteoporosis with multiple lumbosacral spine fractures. He has severe asthma, which
• Past Papers has required large doses of glucocorticoids for many years. The patient reports
• Random Questions progressive loss of height and kyphosis over the past year. Other medications include
salbutamol and ipratropium inhalers and long-acting theophylline 300 mg twice a day.
• My Performance
Significant physical findings include bilateral cataracts, multiple ecchymoses and a
• Media Bank prolonged expiratory phase with bilateral wheezes. Which of the following
• New Multimedia mechanisms underlies his osteoporosis?

Online Extras Decreased bone formation


Library Increased bone loss
Community Decreased calcium absorption from the gastrointestinal (GI) tract
Help Increased calcium loss in urine
PasTest Store All of the above Your answer

My Account

Profile
Glucocorticoids are used in the treatment of chronic inflammatory diseases of the
Newsletters
lungs, connective tissue and intestines as well as in transplantation because of
My Career and Exams their antiinflammatory effect. When long-term treatment is required, several
Order History complications (eg, cataracts, truncal obesity, skin-thinning, hyperglycemia) may be
seen. A particularly disabling complication is bone loss, which can lead to fracture;
Learning Goals it can occur with or without the other complications of chronic steroid treatment.
Question Filters The incidence of steroid-induced osteoporosis is unknown, but it appears to be
related to the duration of treatment, half-life of the steroid and its dose. Risk
factors associated with increased bone loss include age, body mass index and
Security duration of use. Steroid-induced osteoporosis proceeds rapidly in the first 6
months of steroid use and slows thereafter. Trabecular bone and the cortical rim of
Change Password the vertebral body are most susceptible to the effects of steroids. Steroids induce
Sign Out bone loss by several mechanisms. First, they inhibit calcium absorption in the
gastrointestinal (GI) tract while enhancing calcium loss in the kidneys. These
effects induce secondary hyperparathyroidism, which leads to increased bone
resorption. Second, they lower sex hormone levels through an effect on the
gonadotropin levels and a direct effect at the gonadal level, as well as by
decreasing adrenal sex steroid synthesis by inhibiting ACTH release. Third, they
have a direct inhibitory effect on osteoblast proliferation, activity and half-life,
leading to decreased bone formation. Fourth, they induce proximal muscle
weakness.

5281

file:///E|/Shakhawan/Endocrinology/274a.htm[3/13/2012 7:14:38 PM]


MyPasTest

Main Navigation
Question Browser: MRCP 1
Home
Subscriptions Question Browser Exam Builder Saved Exams
» MRCP 1 Session Progress
• Question Browser
Questions Correct 9
• Timed Test A 35-year-old woman comes to see you for a complete
physical exam. She has experienced cold intolerance, Questions Incorrect 5
• Mock Exam
weakness and constipation for 3 months. Her menses are Questions Total 14
• Past Papers regular but scanty. Her history is significant for
Questions Percentage 64 %
• Random Questions hypertension and peptic ulcer disease and her family
history includes hypertension and diabetes. The patient is More
• My Performance
married but has never been pregnant and takes cimetidine
• Media Bank 400 mg at bedtime, sustained-release nifedipine 60 mg
• New Multimedia daily and docusate sodium 100 mg three times a day. Her
pulse is 58 beats/minute with a blood pressure of 135/90
Online Extras mm Hg. Her skin is dry and scaly and she has hung-up Reference: Normal Values
Library reflexes. The rest of her exam is normal and the following
labs are obtained serum chemistries are normal except for Click to
Community open/closeHaematology
a creatine kinase of 300 U/l (normal range, 26-140 U/l);
Help CBC is normal, free thyroxine (T4) is 6.4 pmol/l (normal Click to
range, 10-22 pmol/l) and thyroid stimulating hormone open/closeImmunoglobulins
PasTest Store
(TSH) is 1.5 mIU (normal range, 0.3-5.0 mIU). Which of Click to
the following is most likely to confirm the underlying open/closeBiochemistry
My Account cause? Click to open/closeDiabetes
Click to
Profile Free tri-iodothyronine (T3) open/closeEndocrinology
Click to open/closeBlood
Newsletters Thyroid ultrasound scan
gases Click to
My Career and Exams Thyroid uptake scan open/closeCSF
Order History Pituitary magnetic resonance imaging (MRI)
Learning Goals Antithyroid antibodies
Question Filters
5282

Security

Change Password
Sign Out

© 2011 PasTest Ltd | About Us | Contact Us | Help

file:///E|/Shakhawan/Endocrinology/275.htm[3/13/2012 7:14:40 PM]


MyPasTest

Main Navigation
Question Browser: MRCP 1
Home
Subscriptions Question Browser Exam Builder Saved Exams
» MRCP 1 Session Progress
• Question Browser
Questions Correct 1
• Timed Test A 35-year-old woman comes to see you for a complete
physical exam. She has experienced cold intolerance, Questions Incorrect 0
• Mock Exam
weakness and constipation for 3 months. Her menses are Questions Total 1
• Past Papers regular but scanty. Her history is significant for
Questions Percentage 100 %
• Random Questions hypertension and peptic ulcer disease and her family
history includes hypertension and diabetes. The patient is More
• My Performance
married but has never been pregnant and takes cimetidine
• Media Bank 400 mg at bedtime, sustained-release nifedipine 60 mg
• New Multimedia daily and docusate sodium 100 mg three times a day. Her
pulse is 58 beats/minute with a blood pressure of 135/90
Online Extras mm Hg. Her skin is dry and scaly and she has hung-up
Library reflexes. The rest of her exam is normal and the following
labs are obtained serum chemistries are normal except for
Community
a creatine kinase of 300 U/l (normal range, 26-140 U/l);
Help CBC is normal, free thyroxine (T4) is 6.4 pmol/l (normal
PasTest Store range, 10-22 pmol/l) and thyroid stimulating hormone
(TSH) is 1.5 mIU (normal range, 0.3-5.0 mIU). Which of
the following is most likely to confirm the underlying
My Account cause?

Profile Free tri-iodothyronine (T3)


Newsletters Thyroid ultrasound scan
My Career and Exams Thyroid uptake scan
Order History Pituitary magnetic resonance imaging Your answer Reference: Normal Values
(MRI)
Learning Goals
Antithyroid antibodies Click to open/closeHaematology
Question Filters Click to open/closeImmunoglobulins
Click to open/closeBiochemistry
Click to open/closeDiabetes Click to
Security open/closeEndocrinology Click to
open/closeBlood gases Click to
Change Password This patient has central hypothyroidism and should be open/closeCSF
Sign Out evaluated for pituitary and end-organ function as well
as the presence of a pituitary tumour. The T4 is low
and a compensatory rise in TSH has not occurred. The
prolactin level should be measured and the pituitary–
adrenal, gonadal and growth hormone axes assessed.
The presence of a pituitary tumour can be determined
by imaging the pituitary gland with MRI or computed
tomography (CT) scan. Where appropriate, this should
be followed by evaluation of the visual fields. Treatment
of choice for most pituitary adenomas is trans-
sphenoidal resection, although microprolactinomas may
be managed successfully with dopamine agonist
therapy.

5282

© 2011 PasTest Ltd | About Us | Contact Us | Help

file:///E|/Shakhawan/Endocrinology/275a.htm[3/13/2012 7:14:41 PM]


MyPasTest

Main Navigation
Question Browser: MRCP 1
Home
Subscriptions Question Browser Exam Builder Saved Exams
» MRCP 1 Session Progress
• Question Browser
Questions Correct 1
• Timed Test A 38-year-old woman comes to you for renewal of her
medications. She has had hypertension since her last Questions Incorrect 0
• Mock Exam
pregnancy at age 30 and has been maintained on clonidine Questions Total 1
• Past Papers 0.2 mg twice a day. She gets headaches, dyspnea on
Questions Percentage 100 %
• Random Questions exertion; swelling of her feet and orthopnea but denies
chest pain. Her father is also being treated for More
• My Performance
hypertension. She is married and does not smoke. She is
• Media Bank five feet seven inches tall and weighs 15 stones. Her blood
• New Multimedia pressure is 180/110 mm Hg; pulse is 92 beats/minute. The
rest of her exam is remarkable for hypertensive
Online Extras retinopathy, bibasilar rales and 1+ pitting edema Reference: Normal Values
Library bilaterally. Initial labs were normal except for serum
potassium of 3.0 mEq/l (normal range, 3.5-5.0 mEq/l) and Click to
Community open/closeHaematology
serum bicarbonate of 33 mEq/l (normal range, 22-28
Help mEq/l). You correct hypokalemia and obtain a random Click to
serum aldosterone level of 25 ng/dl (normal range, 5-30 open/closeImmunoglobulins
PasTest Store
ng/dl) with a plasma renin activity of 0.5 ng/ml/h (normal Click to
range, 1.6-7.4 ng/ml/h) while the patient is on a normal open/closeBiochemistry
My Account diet. Click to open/closeDiabetes
Click to
Profile What additional tests might be appropriate? open/closeEndocrinology
Click to open/closeBlood
Newsletters Adrenal computed tomography (CT) scan gases Click to
My Career and Exams Adrenal vein sampling open/closeCSF
Order History 18-Hydroxycorticosterone
Learning Goals Saline loading test
Question Filters Adrenal CT, adrenal vein sampling and 18-
hydroxycorticosterone testing
Security
5283
Change Password
Sign Out

© 2011 PasTest Ltd | About Us | Contact Us | Help

file:///E|/Shakhawan/Endocrinology/276.htm[3/13/2012 7:14:43 PM]


MyPasTest

Main Navigation
Question Browser: MRCP 1
Home
Subscriptions Question Browser Exam Builder Saved Exams
» MRCP 1 Session Progress
• Question Browser
Questions Correct 2
• Timed Test A 38-year-old woman comes to you for renewal of her
medications. She has had hypertension since her last Questions Incorrect 0
• Mock Exam
pregnancy at age 30 and has been maintained on clonidine Questions Total 2
• Past Papers 0.2 mg twice a day. She gets headaches, dyspnea on
Questions Percentage 100 %
• Random Questions exertion; swelling of her feet and orthopnea but denies
chest pain. Her father is also being treated for More
• My Performance
hypertension. She is married and does not smoke. She is
• Media Bank five feet seven inches tall and weighs 15 stones. Her blood
• New Multimedia pressure is 180/110 mm Hg; pulse is 92 beats/minute. The
rest of her exam is remarkable for hypertensive
Online Extras retinopathy, bibasilar rales and 1+ pitting edema
Library bilaterally. Initial labs were normal except for serum
potassium of 3.0 mEq/l (normal range, 3.5-5.0 mEq/l) and
Community
serum bicarbonate of 33 mEq/l (normal range, 22-28
Help mEq/l). You correct hypokalemia and obtain a random
PasTest Store serum aldosterone level of 25 ng/dl (normal range, 5-30
ng/dl) with a plasma renin activity of 0.5 ng/ml/h (normal
range, 1.6-7.4 ng/ml/h) while the patient is on a normal
My Account diet.

Profile What additional tests might be appropriate?


Newsletters Adrenal computed tomography (CT)
My Career and Exams scan
Order History Adrenal vein sampling Reference: Normal Values
Learning Goals 18-Hydroxycorticosterone
Click to open/closeHaematology
Question Filters Saline loading test Click to open/closeImmunoglobulins
Adrenal CT, adrenal vein sampling and Your answer Click to open/closeBiochemistry
18-hydroxycorticosterone testing Click to open/closeDiabetes Click to
Security open/closeEndocrinology Click to
open/closeBlood gases Click to
Change Password open/closeCSF
Sign Out
Primary aldosteronism, a disorder characterized by
hypertension, hypokalemia, suppressed plasma renin
activity and increased aldosterone secretion, affects
0.05 to 2% of the hypertensive population. This
disorder should be suspected in hypertensive patients in
whom spontaneous or easily provoked hypokalemia
develops that is slow to correct after discontinuation of
diuretics.

As important as recognizing the presence of primary


aldosteronism is the differentiation of lesions that are
surgically curable (60–70% of the cases in some series)
from those that are best treated medically. In this
patient, the presence of hypertension, hypokalemia and
alkalosis appropriately triggered screening for
hyperaldosteronism, which led to the findings of an
aldosterone–renin ratio of greater than 30, which
constitutes a positive screening test.

Aldosteronism can be confirmed by the finding of a


24-h urine aldosterone secretion of 12 µg in the salt
replete state. Adrenal imaging is the next step to
differentiate adrenal adenoma from adrenal hyperplasia,
although adenomas smaller than 1.5 cm can be missed
and so mistaken for hyperplasia. In confusing cases,
adrenal vein sampling for aldosterone measurements is
used to localise adenoma with 95% accuracy.

file:///E|/Shakhawan/Endocrinology/276a.htm[3/13/2012 7:14:44 PM]


MyPasTest

The finding of a literalising 10 : 1 aldosterone ratio in


the presence of a symmetrical adrenocorticotropic
hormone (ACTH)-induced cortisol rise diagnoses and
localizes an adenoma. Other features suggestive of
adenoma include plasma 18-hydroxy corticosterone of
100 ng/dl or more, spontaneous hypokalemia of less
than 3 mEq/l and an anomalous postural decrease of
plasma aldosterone concentration. Saline loading is
inappropriate in this patient because of heart failure and
hypertensive retinopathy.

5283

© 2011 PasTest Ltd | About Us | Contact Us | Help

file:///E|/Shakhawan/Endocrinology/276a.htm[3/13/2012 7:14:44 PM]


MyPasTest

Main Navigation
Question Browser: MRCP 1
Home
Subscriptions Question Browser Exam Builder Saved Exams
» MRCP 1 Session Progress
• Question Browser
Questions Correct 0
• Timed Test A 27-year-old woman was admitted 2 days ago through
the emergency room for seizures. She has a history of Questions Incorrect 0
• Mock Exam
moderate alcohol use. Two weeks ago she received Questions Total 0
• Past Papers benzathine penicillin for secondary syphilis. She is
Questions Percentage 0%
• Random Questions complaining of muscle cramps, weakness and headache.
She received 1 g of phenytoin on the day of admission and More
• My Performance
is now taking 100 mg three times a day. She is also taking
• Media Bank paracetamol, multi-vitamins and tapering doses of
• New Multimedia chlordiazepoxide. There is a history of seizures in her
family. She is 152 cm tall and weighs 55kg. Her blood
Online Extras pressure is 130/80 mm Hg; pulse is 90 beats/minute. The Reference: Normal Values
Library rest of the physical exam is normal except for a round face,
a short neck, short fourth and fifth metacarpals and Click to
Community open/closeHaematology
bilateral cataracts. Abnormal labs include calcium of 1.5
Help mmol/l (normal range, 2.2-2.6 mmol/l), phosphorus of 1.7 Click to
mmol/l (normal range, 0.8-1.4 mmol/l) and an intact open/closeImmunoglobulins
PasTest Store
parathyroid hormone (PTH) of 200 pg/ml (normal range, Click to
15-65). open/closeBiochemistry
My Account Click to open/closeDiabetes
Which of the following is most likely? Click to
Profile open/closeEndocrinology
Hypothyroidism Click to open/closeBlood
Newsletters gases Click to
Pseudohypoparathyroidism
My Career and Exams open/closeCSF
Hypoparathyroidism
Order History
Pseudo-pseudohypoparathyroidism
Learning Goals
Hyperparathyroidism
Question Filters
5284
Security

Change Password
Sign Out

© 2011 PasTest Ltd | About Us | Contact Us | Help

file:///E|/Shakhawan/Endocrinology/277.htm[3/13/2012 7:14:46 PM]


MyPasTest

Main Navigation
Question Browser: MRCP 1
Home
Subscriptions Question Browser Exam Builder Saved Exams
» MRCP 1
• Question Browser
• Timed Test You review a 52-year-old woman in the thyroid clinic. She has a thyroid mass.
• Mock Exam Unfortunately this turns out to be a thyroid lymphoma.
• Past Papers Which of the following is the best choice therapy in this case?
• Random Questions
Suppressive treatment with thyroid hormone
• My Performance
Radioiodine treatment
• Media Bank
Chemotherapy and external beam radiotherapy
• New Multimedia
External beam radiotherapy
Online Extras
Palliative measures
Library
Community 5285
Help
PasTest Store

My Account

Profile
Newsletters
My Career and Exams
Order History
Learning Goals
Question Filters

Security

Change Password
Sign Out

file:///E|/Shakhawan/Endocrinology/278.htm[3/13/2012 7:14:47 PM]


MyPasTest

Main Navigation
Question Browser: MRCP 1
Home
Subscriptions Question Browser Exam Builder Saved Exams
» MRCP 1
• Question Browser
• Timed Test You review a 52-year-old woman in the thyroid clinic. She has a thyroid mass.
• Mock Exam Unfortunately this turns out to be a thyroid lymphoma.
• Past Papers Which of the following is the best choice therapy in this case?
• Random Questions
Suppressive treatment with thyroid hormone
• My Performance
Radioiodine treatment
• Media Bank
Chemotherapy and external beam radiotherapy Your answer
• New Multimedia
External beam radiotherapy
Online Extras
Palliative measures
Library
Community
Help
PasTest Store
Iodine 131 is a radioactive isotope of iodine (RAI) that is selectively concentrated
in the thyroid tissue and metabolized by the same pathways as naturally occurring
My Account iodine. This, together with its long half-life (8 days), allows it to deliver high doses
of radiation to the thyroid gland ( β-radiation) sufficient to destroy thyroid follicular
Profile cells. So, 131I is used in the treatment of Graves' disease, toxic multi-nodular
Newsletters goiter and differentiated thyroid cancer. The doses of RAI used in the treatment of
Graves' disease and toxic multi-nodular goiter are relatively low compared with
My Career and Exams those used in the treatment of thyroid cancer (in which it is used in conjunction
Order History with surgery). RAI has no place in the treatment of thyroid lymphoma because
lymphoma cells do not concentrate iodine. Like all lymphomas, chemotherapy is
Learning Goals extremely useful here. 3-6 courses of combination chemotherapy are usually
Question Filters administered, and trials of additional rituximab look promising, this is usually
combined with external beam radiotherapy.

Security
5285
Change Password
Sign Out

file:///E|/Shakhawan/Endocrinology/278a.htm[3/13/2012 7:14:48 PM]


MyPasTest

Main Navigation
Question Browser: MRCP 1
Home
Subscriptions Question Browser Exam Builder Saved Exams
» MRCP 1 Session Progress
• Question Browser
Questions Correct 2
• Timed Test You review a 54-year-old patient with
hyperparathyroidism. Which of the following Questions Incorrect 0
• Mock Exam
statements is true regarding parathyroid hormone Questions Total 2
• Past Papers (PTH)?
Questions Percentage 100 %
• Random Questions
• My Performance Secretion is inhibited by hypocalcemia More
• Media Bank Secretion is stimulated by hypercalcemia

• New Multimedia The effect of magnesium on secretion is the same as


that of calcium
Online Extras Reference: Normal Values
Secretion is stimulated by low 1,25-hydroxyvitamin D
Library and inhibited by high levels of 1,25-hydroxyvitamin D
Click to
Community Hyperparathyroidism invariably causes open/closeHaematology
Help hypophosphataemia Click to
open/closeImmunoglobulins
PasTest Store 5286 Click to
open/closeBiochemistry
My Account Click to open/closeDiabetes
Click to
Profile open/closeEndocrinology
Click to open/closeBlood
Newsletters gases Click to
My Career and Exams open/closeCSF
Order History
Learning Goals
Question Filters

Security

Change Password
Sign Out

© 2011 PasTest Ltd | About Us | Contact Us | Help

file:///E|/Shakhawan/Endocrinology/279.htm[3/13/2012 7:14:50 PM]


MyPasTest

Main Navigation
Question Browser: MRCP 1
Home
Subscriptions Question Browser Exam Builder Saved Exams
» MRCP 1 Session Progress
• Question Browser
Questions Correct 3
• Timed Test You review a 54-year-old patient with
hyperparathyroidism. Which of the following Questions Incorrect 0
• Mock Exam
statements is true regarding parathyroid hormone Questions Total 3
• Past Papers (PTH)?
Questions Percentage 100 %
• Random Questions
• My Performance Secretion is inhibited by hypocalcemia More
• Media Bank Secretion is stimulated by
hypercalcemia
• New Multimedia
The effect of magnesium on secretion
Online Extras is the same as that of calcium
Library Secretion is stimulated by low 1,25- Your answer
Community hydroxyvitamin D and inhibited by high
levels of 1,25-hydroxyvitamin D
Help
Hyperparathyroidism invariably causes
PasTest Store hypophosphataemia

My Account

Profile
PTH, an 84-amino-acid peptide synthesized and
Newsletters
secreted by the parathyroid gland, is a potent regulator
My Career and Exams of the serum calcium level. Hypocalcemia stimulates the
Order History secretion of PTH acutely (with increased PTH synthesis Reference: Normal Values
and parathyroid cell hypertrophy and hyperplasia after
Learning Goals chronic hypocalcemia), whereas hypercalcemia leads to Click to open/closeHaematology
Question Filters decreased secretion of PTH. Hypomagnesemia inhibits Click to open/closeImmunoglobulins
PTH secretion. Elevated 1,25-dihydroxyvitamin D Click to open/closeBiochemistry
affects PTH synthesis and secretion by directly inhibiting Click to open/closeDiabetes Click to
Security the parathyroid gland and indirectly via hypercalcemia. open/closeEndocrinology Click to
Low levels of 1,25-dihydroxyvitamin D have the open/closeBlood gases Click to
Change Password opposite effect. Primary hyperparathyroidism has a open/closeCSF
Sign Out prevalence of 1 in 1000, but increases in the fifth
decade and is commoner in women, where the
prevalence may be as high as 3% in the post
menopausal period.

5286

© 2011 PasTest Ltd | About Us | Contact Us | Help

file:///E|/Shakhawan/Endocrinology/279a.htm[3/13/2012 7:14:51 PM]


MyPasTest

Main Navigation
Question Browser: MRCP 1
Home
Subscriptions Question Browser Exam Builder Saved Exams
» MRCP 1
• Question Browser
• Timed Test A 40-year-old female complains of feeling tired all the time. The following laboratory
• Mock Exam results have been obtained:
• Past Papers Cholesterol 6.8 mmol/l
• Random Questions High-density lipoprotein (HDL) 0.9 mmol/l
Thyroglobulin (TG) 2.2 mmol/l
• My Performance
Free thyroxine (T4) 10 pmol/l
• Media Bank Thyroid-stimulating hormone (TSH) 22.5 mu/l
• New Multimedia
What is the most appropriate treatment?
Online Extras
Library Atorvastatin
Community Ezetimibe
Help Gemfibrozil
PasTest Store Omega 3 fish oil
Thyroid hormone replacement

My Account 5498
Profile
Newsletters
My Career and Exams
Order History
Learning Goals
Question Filters

Security

Change Password
Sign Out

file:///E|/Shakhawan/Endocrinology/280.htm[3/13/2012 7:14:53 PM]


MyPasTest

Main Navigation
Question Browser: MRCP 1
Home
Subscriptions Question Browser Exam Builder Saved Exams
» MRCP 1
• Question Browser
• Timed Test A 40-year-old female complains of feeling tired all the time. The following laboratory
• Mock Exam results have been obtained:
• Past Papers Cholesterol 6.8 mmol/l
• Random Questions High-density lipoprotein (HDL) 0.9 mmol/l
Thyroglobulin (TG) 2.2 mmol/l
• My Performance
Free thyroxine (T4) 10 pmol/l
• Media Bank Thyroid-stimulating hormone (TSH) 22.5 mu/l
• New Multimedia
What is the most appropriate treatment?
Online Extras
Library Atorvastatin
Community Ezetimibe
Help Gemfibrozil
PasTest Store Omega 3 fish oil
Thyroid hormone replacement Your answer

My Account

Profile
Newsletters In hypothyroidism, serum low-density lipoprotein (LDL) cholesterol and, less
My Career and Exams frequently, serum triglycerides are raised. Levels of high-density lipoprotein (HDL)
tend to be increased. There is decreased receptor-mediated LDL catabolism, and
Order History
lipoprotein lipase activity may be decreased. Hypothyroidism should always be
Learning Goals considered in the diagnosis of hyperlipidaemia, and it is particularly important to
Question Filters exclude it when marked hyperlipidaemia occurs in women and in diabetic patients.

Security 5498

Change Password
Sign Out

file:///E|/Shakhawan/Endocrinology/280a.htm[3/13/2012 7:14:54 PM]


MyPasTest

Main Navigation
Question Browser: MRCP 1
Home
Subscriptions Question Browser Exam Builder Saved Exams
» MRCP 1 Session Progress
• Question Browser
Questions Correct 0
• Timed Test A diabetic heavy goods vehicle (HGV) driver has been
changed from oral medication to insulin. What is the Questions Incorrect 0
• Mock Exam
consequence for his driving licence? Questions Total 0
• Past Papers
Questions Percentage 0%
• Random Questions Keep the licence
• My Performance Suspension for 3 months More
• Media Bank Suspension for 6 month
• New Multimedia Suspension for 1 year
Online Extras Loss of licence Reference: Normal Values
Library
5580 Click to
Community open/closeHaematology
Help Click to
open/closeImmunoglobulins
PasTest Store
Click to
open/closeBiochemistry
My Account Click to open/closeDiabetes
Click to
Profile open/closeEndocrinology
Click to open/closeBlood
Newsletters gases Click to
My Career and Exams open/closeCSF
Order History
Learning Goals
Question Filters

Security

Change Password
Sign Out

© 2011 PasTest Ltd | About Us | Contact Us | Help

file:///E|/Shakhawan/Endocrinology/281.htm[3/13/2012 7:14:55 PM]


MyPasTest

Main Navigation
Question Browser: MRCP 1
Home
Subscriptions Question Browser Exam Builder Saved Exams
» MRCP 1 Session Progress
• Question Browser
Questions Correct 1
• Timed Test A diabetic heavy goods vehicle (HGV) driver has been
changed from oral medication to insulin. What is the Questions Incorrect 0
• Mock Exam
consequence for his driving licence? Questions Total 1
• Past Papers
Questions Percentage 100 %
• Random Questions Keep the licence
• My Performance Suspension for 3 months More
• Media Bank Suspension for 6 month
• New Multimedia Suspension for 1 year
Online Extras Loss of licence Your answer
Library
Community
Help
PasTest Store Drivers with insulin-treated diabetes may NOT obtain a
licence for categories C, C+E, D, D1 and D+E UNLESS
they held a heavy goods vehicle/public service vehicle
My Account (HGV/PSV) licence valid on 1 April 1991 and the traffic
commissioner in whose area they lived, or who issued
Profile the licence, had knowledge of the insulin treatment
before 1 January 1991. Drivers with insulin-treated
Newsletters
diabetes applying for C1, C1+E entitlement (vehicles
My Career and Exams between 3.5 and 7.5 tonnes with a trailer up to a
Order History combined weight of 8.25 tonnes, or 12 tonnes if a Reference: Normal Values
separate test has been passed previously), will need
Learning Goals specialist assessment and must meet strict criteria for Click to open/closeHaematology
Question Filters diabetic control, and meet the other higher medical Click to open/closeImmunoglobulins
standards. Click to open/closeBiochemistry
Click to open/closeDiabetes Click to
Security open/closeEndocrinology Click to
5580 open/closeBlood gases Click to
Change Password open/closeCSF
Sign Out

© 2011 PasTest Ltd | About Us | Contact Us | Help

file:///E|/Shakhawan/Endocrinology/281a.htm[3/13/2012 7:14:57 PM]


MyPasTest

Main Navigation
Question Browser: MRCP 1
Home
Subscriptions Question Browser Exam Builder Saved Exams
» MRCP 1 Session Progress
• Question Browser
Questions Correct 1
• Timed Test A 56-year-old diabetic male had an anterior myocardial
infarction 5 years ago. He is receiving aspirin 150 mg once Questions Incorrect 0
• Mock Exam
daily and twice daily insulin. Baseline screen revealed a Questions Total 1
• Past Papers body mass index (BMI) of 34, blood pressure 150/90
Questions Percentage 100 %
• Random Questions mmHg , haemoglobin A1c (HbA1c) 6.9 %, serum
cholesterol 3.6 mmol/l (normal < 5.1 mmol/l). More
• My Performance
• Media Bank Which of the following measures would delay
deterioration in renal function?
• New Multimedia
Online Extras Orlistat Reference: Normal Values
Library Increase to ×4 daily insulin
Click to
Community Ramipril open/closeHaematology
Help Simvastatin Click to
open/closeImmunoglobulins
PasTest Store Increase aspirin from 150 mg to 300 mg daily Click to
open/closeBiochemistry
5599 Click to open/closeDiabetes
My Account
Click to
Profile open/closeEndocrinology
Click to open/closeBlood
Newsletters gases Click to
My Career and Exams open/closeCSF
Order History
Learning Goals
Question Filters

Security

Change Password
Sign Out

© 2011 PasTest Ltd | About Us | Contact Us | Help

file:///E|/Shakhawan/Endocrinology/282.htm[3/13/2012 7:14:58 PM]


MyPasTest

Main Navigation
Question Browser: MRCP 1
Home
Subscriptions Question Browser Exam Builder Saved Exams
» MRCP 1 Session Progress
• Question Browser
Questions Correct 2
• Timed Test A 56-year-old diabetic male had an anterior myocardial
infarction 5 years ago. He is receiving aspirin 150 mg once Questions Incorrect 0
• Mock Exam
daily and twice daily insulin. Baseline screen revealed a Questions Total 2
• Past Papers body mass index (BMI) of 34, blood pressure 150/90
Questions Percentage 100 %
• Random Questions mmHg , haemoglobin A1c (HbA1c) 6.9 %, serum
cholesterol 3.6 mmol/l (normal < 5.1 mmol/l). More
• My Performance
• Media Bank Which of the following measures would delay
deterioration in renal function?
• New Multimedia
Online Extras Orlistat
Library Increase to ×4 daily insulin
Community Ramipril Your answer
Help Simvastatin
PasTest Store Increase aspirin from 150 mg to 300
mg daily

My Account

Profile
Newsletters Angiotensin-converting enzyme (ACE) inhibitors reduce
My Career and Exams proteinuria, by relaxing the efferent arterioles in the
glomerulus, and slow the development of both Reference: Normal Values
Order History
nephropathy and retinopathy; some evidence points to
Learning Goals specific beneficial effects in nephropathy, in addition to Click to open/closeHaematology
Question Filters the lowering of blood pressure. ACE inhibitors do not Click to open/closeImmunoglobulins
worsen blood glucose or lipids, and may even improve Click to open/closeBiochemistry
insulin sensitivity. Click to open/closeDiabetes Click to
Security open/closeEndocrinology Click to
open/closeBlood gases Click to
Change Password 5599 open/closeCSF
Sign Out

© 2011 PasTest Ltd | About Us | Contact Us | Help

file:///E|/Shakhawan/Endocrinology/282a.htm[3/13/2012 7:15:00 PM]


MyPasTest

Main Navigation
Question Browser: MRCP 1
Home
Subscriptions Question Browser Exam Builder Saved Exams
» MRCP 1 Session Progress
• Question Browser
Questions Correct 2
• Timed Test A patient who has had a subtotal thyroidectomy for
hyperthyroidism is on thyroxine replacement. She has a Questions Incorrect 0
• Mock Exam
normal thyroid-stimulating hormone (TSH), normal tri- Questions Total 2
• Past Papers iodothyronine (T3) and low free thyroxine (T4) levels.
Questions Percentage 100 %
• Random Questions What is the most likely reason for those results?
• My Performance More
Hypothalamic pituitary causes
• Media Bank
Poor compliance with thyroxine supplements
• New Multimedia
The patient’s results are as expected – no change in
Online Extras treatment is required Reference: Normal Values
Library She has sick euthyroid syndrome
Click to
Community Malabsorption open/closeHaematology
Help Click to
5601 open/closeImmunoglobulins
PasTest Store
Click to
open/closeBiochemistry
My Account Click to open/closeDiabetes
Click to
Profile open/closeEndocrinology
Click to open/closeBlood
Newsletters gases Click to
My Career and Exams open/closeCSF
Order History
Learning Goals
Question Filters

Security

Change Password
Sign Out

© 2011 PasTest Ltd | About Us | Contact Us | Help

file:///E|/Shakhawan/Endocrinology/283.htm[3/13/2012 7:15:01 PM]


MyPasTest

Main Navigation
Question Browser: MRCP 1
Home
Subscriptions Question Browser Exam Builder Saved Exams
» MRCP 1 Session Progress
• Question Browser
Questions Correct 3
• Timed Test A patient who has had a subtotal thyroidectomy for
hyperthyroidism is on thyroxine replacement. She has a Questions Incorrect 0
• Mock Exam
normal thyroid-stimulating hormone (TSH), normal tri- Questions Total 3
• Past Papers iodothyronine (T3) and low free thyroxine (T4) levels.
Questions Percentage 100 %
• Random Questions What is the most likely reason for those results?
• My Performance More
Hypothalamic pituitary causes
• Media Bank
Poor compliance with thyroxine
• New Multimedia supplements
Online Extras The patient’s results are as expected – Your answer
Library no change in treatment is required
Community She has sick euthyroid syndrome
Help Malabsorption
PasTest Store

My Account
Once on a full replacement dose, thyroid-stimulating
Profile hormone (TSH) levels should be checked at intervals of
1–3 years, depending on their stability. Fluctuating or
Newsletters
elevated TSH levels in a previously stable patient, or
My Career and Exams thyroxine requirements in excess of 200 μg/day, usually
Order History indicate compliance problems. It is important to rule out Reference: Normal Values
malabsorption or abnormal thyroxine kinetics caused by
Learning Goals
drugs: cholestyramine, ferrous sulphate, lovastatin, Click to open/closeHaematology
Question Filters aluminium hydroxide, rifampicin, amiodarone, Click to open/closeImmunoglobulins
carbamazepine, and phenytoin all alter the absorption Click to open/closeBiochemistry
or clearance of thyroxine. Click to open/closeDiabetes Click to
Security open/closeEndocrinology Click to
open/closeBlood gases Click to
Change Password open/closeCSF
5601
Sign Out

© 2011 PasTest Ltd | About Us | Contact Us | Help

file:///E|/Shakhawan/Endocrinology/283a.htm[3/13/2012 7:15:02 PM]


MyPasTest

Main Navigation
Question Browser: MRCP 1
Home
Subscriptions Question Browser Exam Builder Saved Exams
» MRCP 1
• Question Browser
• Timed Test A patient with Addison's disease presents with an acute onset of diarrhoea and
• Mock Exam vomiting. He currently takes fludrocortisone and hydrocortisone. What is the most
appropriate management?
• Past Papers
• Random Questions Intravenous (iv) cyclizine
• My Performance iv Ondansetron
• Media Bank iv Fluids and iv hydrocortisone
• New Multimedia Increase fludrocortisone
Online Extras iv Antibiotics
Library
5611
Community
Help
PasTest Store

My Account

Profile
Newsletters
My Career and Exams
Order History
Learning Goals
Question Filters

Security

Change Password
Sign Out

file:///E|/Shakhawan/Endocrinology/284.htm[3/13/2012 7:15:04 PM]


MyPasTest

Main Navigation
Question Browser: MRCP 1
Home
Subscriptions Question Browser Exam Builder Saved Exams
» MRCP 1 Session Progress
• Question Browser
Questions Correct 4
• Timed Test A patient with Addison's disease presents with an acute
onset of diarrhoea and vomiting. He currently takes Questions Incorrect 0
• Mock Exam
fludrocortisone and hydrocortisone. What is the most Questions Total 4
• Past Papers appropriate management?
Questions Percentage 100 %
• Random Questions
• My Performance Intravenous (iv) cyclizine More
• Media Bank iv Ondansetron

• New Multimedia iv Fluids and iv hydrocortisone Your answer

Online Extras Increase fludrocortisone

Library iv Antibiotics

Community
Help
PasTest Store
Patients receiving glucocorticoid replacement therapy
should be advised to double the dose in the event of
My Account intercurrent febrile illness, accident, or mental stress
such as an important examination. If the patient is
Profile vomiting and cannot take by mouth, parenteral
hydrocortisone must be given urgently, as indicated
Newsletters
above. For minor surgery, 50–100 mg of hydrocortisone
My Career and Exams hemisuccinate is given with the premedication. For
Order History major procedures this is then followed by the same Reference: Normal Values
regimen as for acute adrenal insufficiency.
Learning Goals
Click to open/closeHaematology
Question Filters Click to open/closeImmunoglobulins
5611 Click to open/closeBiochemistry
Click to open/closeDiabetes Click to
Security open/closeEndocrinology Click to
open/closeBlood gases Click to
Change Password open/closeCSF
Sign Out

© 2011 PasTest Ltd | About Us | Contact Us | Help

file:///E|/Shakhawan/Endocrinology/284a.htm[3/13/2012 7:15:05 PM]


MyPasTest

Main Navigation
Question Browser: MRCP 1
Home
Subscriptions Question Browser Exam Builder Saved Exams
» MRCP 1 Session Progress
• Question Browser
Questions Correct 4
• Timed Test A 65-year-old lady with type 2 diabetes mellitus has had
two episodes of dizziness associated with driving a car. She Questions Incorrect 0
• Mock Exam
currently administers insulin, her glucose was 2.1 mmol/l, Questions Total 4
• Past Papers which improved after intravenous (iv) glucose. Her insulin
Questions Percentage 100 %
• Random Questions dose was reduced.
• My Performance More
What is the next important step?
• Media Bank
• New Multimedia Magnetic resonance imaging (MRI) of the brain

Online Extras 72-h observation in hospital


Reference: Normal Values
Library Advise not to drive
Psychiatric referral Click to
Community open/closeHaematology
Help Better diabetes education Click to
open/closeImmunoglobulins
PasTest Store 5612 Click to
open/closeBiochemistry
My Account Click to open/closeDiabetes
Click to
Profile open/closeEndocrinology
Click to open/closeBlood
Newsletters gases Click to
My Career and Exams open/closeCSF
Order History
Learning Goals
Question Filters

Security

Change Password
Sign Out

© 2011 PasTest Ltd | About Us | Contact Us | Help

file:///E|/Shakhawan/Endocrinology/285.htm[3/13/2012 7:15:07 PM]


MyPasTest

Main Navigation
Question Browser: MRCP 1
Home
Subscriptions Question Browser Exam Builder Saved Exams
» MRCP 1 Session Progress
• Question Browser
Questions Correct 4
• Timed Test A 65-year-old lady with type 2 diabetes mellitus has had
two episodes of dizziness associated with driving a car. She Questions Incorrect 1
• Mock Exam
currently administers insulin, her glucose was 2.1 mmol/l, Questions Total 5
• Past Papers which improved after intravenous (iv) glucose. Her insulin
Questions Percentage 80 %
• Random Questions dose was reduced.
• My Performance More
What is the next important step?
• Media Bank
• New Multimedia Magnetic resonance imaging (MRI)
of the brain
Online Extras
72-h observation in hospital
Library
Advise not to drive Correct answer
Community
Psychiatric referral
Help
Better diabetes education Your answer
PasTest Store

My Account

Profile If loss of awareness of hypoglycaemia is confirmed,


then driving must stop. Although she gives only some
Newsletters reports of dizziness, it is likely that her driving ability
My Career and Exams has been significantly impaired. Driving may resume
provided reports show awareness of hypoglycaemia has Reference: Normal Values
Order History
been regained, confirmed by a general practitioner
Learning Goals (GP)/consultant report. If frequent hypoglycaemic Click to open/closeHaematology
Question Filters episodes continue if diabetes is poorly controlled, then Click to open/closeImmunoglobulins
driving should cease until satisfactory control is re- Click to open/closeBiochemistry
established, with a consultant/GP report. Click to open/closeDiabetes Click to
Security open/closeEndocrinology Click to
open/closeBlood gases Click to
Change Password 5612 open/closeCSF
Sign Out

© 2011 PasTest Ltd | About Us | Contact Us | Help

file:///E|/Shakhawan/Endocrinology/285a.htm[3/13/2012 7:15:08 PM]


MyPasTest

Main Navigation
Question Browser: MRCP 1
Home
Subscriptions Question Browser Exam Builder Saved Exams
» MRCP 1 Session Progress
• Question Browser
Questions Correct 4
• Timed Test A 24-year-old woman is referred by her GP. She is 10
weeks' pregnant and complaining of anxiety and an inability Questions Incorrect 1
• Mock Exam
to sleep Questions Total 5
• Past Papers
total thyroxine (T4) is noted to be 160 nmol/l Questions Percentage 80 %
• Random Questions
(normal 70-140) More
• My Performance free T4 is noted to be 27 pmol/l (normal 9-25)
• Media Bank thyroid-stimulating hormone (TSH) 0.4 mU/l.
• New Multimedia Which of the following is the management of choice
Online Extras in this patient? Reference: Normal Values
Library
Commence low-dose carbimazole therapy Click to
Community open/closeHaematology
Commence propylthiouracil therapy
Help Click to
Observe and repeat thyroid function tests in 1 month open/closeImmunoglobulins
PasTest Store
Start high dose carbimazole and thyroxine Click to
concomitantly open/closeBiochemistry
My Account Click to open/closeDiabetes
Measure antithyroid antibody levels
Click to
Profile open/closeEndocrinology
8272
Click to open/closeBlood
Newsletters gases Click to
My Career and Exams open/closeCSF
Order History
Learning Goals
Question Filters

Security

Change Password
Sign Out

© 2011 PasTest Ltd | About Us | Contact Us | Help

file:///E|/Shakhawan/Endocrinology/286.htm[3/13/2012 7:15:10 PM]


MyPasTest

Main Navigation
Question Browser: MRCP 1
Home
Subscriptions Question Browser Exam Builder Saved Exams
» MRCP 1
• Question Browser
• Timed Test A 24-year-old woman is referred by her GP. She is 10 weeks' pregnant and
• Mock Exam complaining of anxiety and an inability to sleep
• Past Papers total thyroxine (T4) is noted to be 160 nmol/l (normal 70-140)
• Random Questions free T4 is noted to be 27 pmol/l (normal 9-25)
thyroid-stimulating hormone (TSH) 0.4 mU/l.
• My Performance
• Media Bank Which of the following is the management of choice in this patient?
• New Multimedia
Commence low-dose carbimazole therapy
Online Extras
Commence propylthiouracil therapy
Library
Observe and repeat thyroid function tests in 1 month Your answer
Community
Start high dose carbimazole and thyroxine concomitantly
Help
Measure antithyroid antibody levels
PasTest Store

My Account

Profile Suppression of TSH are seen in the first trimester in around 13.5% of women,
small but transient rises in T3 and T4 are also seen, this is physiological and due
Newsletters to the thyroid-stimulating action of b-human chorionic gonadotrophin ( β-HCG).
My Career and Exams Total T4 rises due to increases in thyroid-binding globulin and again is
Order History physiological. A 10–20% increase in thyroid mass also occurs. Physiological
suppression of TSH is also seen in 4–5% of women in the second trimester.
Learning Goals Maternal hyperthyroidism has an incidence of 0.2%, in women who are in
Question Filters remission from Grave’s disease, exacerbation occurs most commonly in the first
trimester.

Security
8272
Change Password
Sign Out

file:///E|/Shakhawan/Endocrinology/286a.htm[3/13/2012 7:15:11 PM]


MyPasTest

Main Navigation
Question Browser: MRCP 1
Home
Subscriptions Question Browser Exam Builder Saved Exams
» MRCP 1 Session Progress
• Question Browser
Questions Correct 5
• Timed Test A 42-year-old woman presents to the Emergency Clinic
with palpitations and shortness of breath. Recent thyroid Questions Incorrect 1
• Mock Exam
function tests on the hospital computer reveal thyroid- Questions Total 6
• Past Papers stimulating hormone (TSH) of <0.05 mU/l and a markedly
Questions Percentage 83 %
• Random Questions elevated T4. You arrange blood gas testing.
• My Performance More
Which of the following findings would be most
• Media Bank consistent with Grave's disease?
• New Multimedia
Respiratory acidosis
Online Extras Reference: Normal Values
Decreased p a (O2 )
Library
Increased p a (CO 2 ) Click to
Community open/closeHaematology
Help Decreased p a (CO 2 ) Click to
Decreased pH open/closeImmunoglobulins
PasTest Store
Click to
open/closeBiochemistry
8273
My Account Click to open/closeDiabetes
Click to
Profile open/closeEndocrinology
Click to open/closeBlood
Newsletters gases Click to
My Career and Exams open/closeCSF
Order History
Learning Goals
Question Filters

Security

Change Password
Sign Out

© 2011 PasTest Ltd | About Us | Contact Us | Help

file:///E|/Shakhawan/Endocrinology/287.htm[3/13/2012 7:15:12 PM]


MyPasTest

Main Navigation
Question Browser: MRCP 1
Home
Subscriptions Question Browser Exam Builder Saved Exams
» MRCP 1 Session Progress
• Question Browser
Questions Correct 6
• Timed Test A 42-year-old woman presents to the Emergency Clinic
with palpitations and shortness of breath. Recent thyroid Questions Incorrect 1
• Mock Exam
function tests on the hospital computer reveal thyroid- Questions Total 7
• Past Papers stimulating hormone (TSH) of <0.05 mU/l and a markedly
Questions Percentage 85 %
• Random Questions elevated T4. You arrange blood gas testing.
• My Performance More
Which of the following findings would be most
• Media Bank consistent with Grave's disease?
• New Multimedia
Respiratory acidosis
Online Extras
Decreased p a (O2 )
Library
Increased p a (CO 2 )
Community
Help Decreased p a (CO 2 ) Your answer
PasTest Store Decreased pH

My Account

Profile
Compared with normal subjects, hyperthyroid patients
Newsletters show significantly lower resting arterial CO2 tension,
My Career and Exams tidal volume and significantly higher mean inspiratory
flow and p a (O2 ). This may of course lead to Reference: Normal Values
Order History
misdiagnosis of patients with hyperthyroidism as having
Learning Goals hyperventilation syndrome. Patients with Click to open/closeHaematology
Question Filters hyperthyroidism also show a greater ventilatory Click to open/closeImmunoglobulins
response to hypoxia than normal patients. These Click to open/closeBiochemistry
ventilatory changes are significantly correlated with T3 Click to open/closeDiabetes Click to
Security levels. open/closeEndocrinology Click to
open/closeBlood gases Click to
Change Password open/closeCSF
Sign Out 8273

© 2011 PasTest Ltd | About Us | Contact Us | Help

file:///E|/Shakhawan/Endocrinology/287a.htm[3/13/2012 7:15:14 PM]


MyPasTest

Main Navigation
Question Browser: MRCP 1
Home
Subscriptions Question Browser Exam Builder Saved Exams
» MRCP 1 Session Progress
• Question Browser
Questions Correct 6
• Timed Test A 28-year-old woman attends the diabetes clinic with her
two children. This is her third pregnancy and although not Questions Incorrect 1
• Mock Exam
normally diabetic she was diagnosed with gestational Questions Total 7
• Past Papers diabetes mellitus. Recent fasting plasma glucose levels
Questions Percentage 85 %
• Random Questions have been in the range of 5-6 mmol/l and she is taking bd
mixed insulins. Later that evening she collapses while More
• My Performance
shopping and is brought to the Emergency Department
• Media Bank with blood glucose of 2.4 mmol/l.
• New Multimedia
Which of the following represents the most
Online Extras appropriate management in this patient? Reference: Normal Values
Library
Reduce her dose of bd mixed insulin by 50% Click to
Community open/closeHaematology
Reduce her dose of bd mixed insulin by 20%
Help Click to
Check a random cortisol to rule out Addison’s disease open/closeImmunoglobulins
PasTest Store
Change her to a basal bolus regime Click to
open/closeBiochemistry
Advise her to increase the size and regularity of her Click to open/closeDiabetes
My Account meals Click to
Profile open/closeEndocrinology
8275 Click to open/closeBlood
Newsletters gases Click to
My Career and Exams open/closeCSF
Order History
Learning Goals
Question Filters

Security

Change Password
Sign Out

© 2011 PasTest Ltd | About Us | Contact Us | Help

file:///E|/Shakhawan/Endocrinology/288.htm[3/13/2012 7:15:15 PM]


MyPasTest

Main Navigation
Question Browser: MRCP 1
Home
Subscriptions Question Browser Exam Builder Saved Exams
» MRCP 1 Session Progress
• Question Browser
Questions Correct 1
• Timed Test A 28-year-old woman attends the diabetes clinic with her
two children. This is her third pregnancy and although not Questions Incorrect 0
• Mock Exam
normally diabetic she was diagnosed with gestational Questions Total 1
• Past Papers diabetes mellitus. Recent fasting plasma glucose levels
Questions Percentage 100 %
• Random Questions have been in the range of 5-6 mmol/l and she is taking bd
mixed insulins. Later that evening she collapses while More
• My Performance
shopping and is brought to the Emergency Department
• Media Bank with blood glucose of 2.4 mmol/l.
• New Multimedia
Which of the following represents the most
Online Extras appropriate management in this patient?
Library
Reduce her dose of bd mixed insulin by
Community
50%
Help
Reduce her dose of bd mixed insulin by
PasTest Store 20%
Check a random cortisol to rule out
Addison’s disease
My Account
Change her to a basal bolus regime Your answer
Profile
Advise her to increase the size and
Newsletters regularity of her meals
My Career and Exams
Order History Reference: Normal Values
Learning Goals
Click to open/closeHaematology
Question Filters This lady’s fasting plasma glucose levels are Click to open/closeImmunoglobulins
appropriate, but taking a bd mixed insulin regime is Click to open/closeBiochemistry
probably not appropriate to her lifestyle where her Click to open/closeDiabetes Click to
Security mealtimes and portion sizes may be slightly irregular open/closeEndocrinology Click to
given the fact that she has two other children to open/closeBlood gases Click to
Change Password manage. A basal bolus regime would give her more open/closeCSF
Sign Out flexibility in varying insulin dose. The coexistence of
Addison’s is unlikely and increasing the size of her
meals would only promote more excess weight gain and
worsening insulin resistance.

8275

© 2011 PasTest Ltd | About Us | Contact Us | Help

file:///E|/Shakhawan/Endocrinology/288a.htm[3/13/2012 7:15:17 PM]


MyPasTest

Main Navigation
Question Browser: MRCP 1
Home
Subscriptions Question Browser Exam Builder Saved Exams
» MRCP 1 Session Progress
• Question Browser
Questions Correct 1
• Timed Test A 76-year-old woman with thyroid cancer comes to see
you. Questions Incorrect 0
• Mock Exam
Questions Total 1
• Past Papers Which of the following has the worst prognosis in
thyroid cancer? Questions Percentage 100 %
• Random Questions
• My Performance More
Papillary carcinoma with lymph node metastases
• Media Bank
Follicular carcinoma with bone metastases
• New Multimedia
Anaplastic carcinoma with long-standing goitre
Online Extras Reference: Normal Values
Thyroid lymphoma
Library
Medullary carcinoma as part of MEN Click to
Community open/closeHaematology
Help 13871 Click to
open/closeImmunoglobulins
PasTest Store
Click to
open/closeBiochemistry
My Account Click to open/closeDiabetes
Click to
Profile open/closeEndocrinology
Click to open/closeBlood
Newsletters gases Click to
My Career and Exams open/closeCSF
Order History
Learning Goals
Question Filters

Security

Change Password
Sign Out

© 2011 PasTest Ltd | About Us | Contact Us | Help

file:///E|/Shakhawan/Endocrinology/289.htm[3/13/2012 7:15:18 PM]


MyPasTest

Main Navigation
Question Browser: MRCP 1
Home
Subscriptions Question Browser Exam Builder Saved Exams
» MRCP 1 Session Progress
• Question Browser
Questions Correct 2
• Timed Test A 76-year-old woman with thyroid cancer comes to see
you. Questions Incorrect 0
• Mock Exam
Questions Total 2
• Past Papers Which of the following has the worst prognosis in
thyroid cancer? Questions Percentage 100 %
• Random Questions
• My Performance More
Papillary carcinoma with lymph node
• Media Bank metastases
• New Multimedia Follicular carcinoma with bone
Online Extras metastases

Library Anaplastic carcinoma with long- Your answer


standing goitre
Community
Thyroid lymphoma
Help
Medullary carcinoma as part of MEN
PasTest Store

My Account

Profile Thyroid lymphoma represents only 5% of all thyroid


malignancy. It is usually in women age 55-75 with
Newsletters chronic lymphocytic thyroiditis. These may be positive
My Career and Exams serum antithyroglobulin or anti TPO antibodies. The
prognosis depends on the cell type and the extent of Reference: Normal Values
Order History
disease beyond the neck. Primary thyroid cancer
Learning Goals involving the C cells is medullary thyroid cancer which Click to open/closeHaematology
Question Filters in 80% of cases is sporadic but in 20% of cases is Click to open/closeImmunoglobulins
familial. Some of the familial forms are part of the Click to open/closeBiochemistry
Multiple Endocrine Neoplasia syndrome. The other forms Click to open/closeDiabetes Click to
Security of thyroid cancer are of the follicular epithelium. open/closeEndocrinology Click to
Papillary carcinoma (70% of thyroid malignancies) is open/closeBlood gases Click to
Change Password slow growing and may be indolent for many years. open/closeCSF
Follicular carcinoma (15% of thyroid malignancies)
Sign Out
carries a prognosis dependant on its invasiveness.

Of all thyroid cancers, even those that have spread,


anaplastic carcinoma (5% of thyroid cancers) carries the
worst prognosis. It is very fast growing and despite
radical surgery the prognosis is only a few months.

13871

© 2011 PasTest Ltd | About Us | Contact Us | Help

file:///E|/Shakhawan/Endocrinology/289a.htm[3/13/2012 7:15:20 PM]


MyPasTest

Main Navigation
Question Browser: MRCP 1
Home
Subscriptions Question Browser Exam Builder Saved Exams
» MRCP 1 Session Progress
• Question Browser
Questions Correct 2
• Timed Test A 30-year-old woman presented with an 18-month history
of polyuria and polydipsia. Her only other history of note Questions Incorrect 0
• Mock Exam
was increasing joint pains, and one visit to the GP with Questions Total 2
• Past Papers depression some 6-months ago. Examination was
Questions Percentage 100 %
• Random Questions unremarkable
• My Performance More
Bloods:
• Media Bank
Glucose 4.1 mmol/l
• New Multimedia
Online Extras Urea 7.2 mmol/l Reference: Normal Values
Library Creatinine 80 micromol/l
Click to
Community open/closeHaematology
Corrected serum calcium 3.1 mmol/l
Help Click to
Serum Phosphate 0.7 mmol/l open/closeImmunoglobulins
PasTest Store
Click to
Parathyroid hormone 6.2 pmol/l (0.9-5.4) open/closeBiochemistry
My Account Click to open/closeDiabetes
Which of the following is the most likely cause of this Click to
Profile clinical picture? open/closeEndocrinology
Click to open/closeBlood
Newsletters Raised levels of calcitonin gases Click to
My Career and Exams Hypervitaminosis D open/closeCSF
Order History Primary Hyperparathryoidism
Learning Goals Secondary Hyperparathryoidism
Question Filters Medullary thyroid carcinoma

14937
Security

Change Password
Sign Out

© 2011 PasTest Ltd | About Us | Contact Us | Help

file:///E|/Shakhawan/Endocrinology/290.htm[3/13/2012 7:15:22 PM]


MyPasTest

Main Navigation
Question Browser: MRCP 1
Home
Subscriptions Question Browser Exam Builder Saved Exams
» MRCP 1 Session Progress
• Question Browser
Questions Correct 3
• Timed Test A 30-year-old woman presented with an 18-month history
of polyuria and polydipsia. Her only other history of note Questions Incorrect 0
• Mock Exam
was increasing joint pains, and one visit to the GP with Questions Total 3
• Past Papers depression some 6-months ago. Examination was
Questions Percentage 100 %
• Random Questions unremarkable
• My Performance More
Bloods:
• Media Bank
Glucose 4.1 mmol/l
• New Multimedia
Online Extras Urea 7.2 mmol/l
Library Creatinine 80 micromol/l
Community
Corrected serum calcium 3.1 mmol/l
Help
PasTest Store Serum Phosphate 0.7 mmol/l

Parathyroid hormone 6.2 pmol/l (0.9-5.4)


My Account
Which of the following is the most likely cause of this
Profile clinical picture?

Newsletters Raised levels of calcitonin


My Career and Exams Hypervitaminosis D
Order History Your answer Reference: Normal Values
Primary Hyperparathryoidism
Learning Goals Secondary Hyperparathryoidism Click to open/closeHaematology
Question Filters Medullary thyroid carcinoma Click to open/closeImmunoglobulins
Click to open/closeBiochemistry
Click to open/closeDiabetes Click to
Security open/closeEndocrinology Click to
open/closeBlood gases Click to
Change Password open/closeCSF
This woman has normal renal function in the presence
Sign Out
of hypercalcaemia and hypophosphataemia. Parathryoid
hormone levels are just above the normal range but any
elevation in this scenario would be suggestive of
primary hyperparathyroidism. 85% of cases are related
to a single parathyroid adenoma, 15% are related to
multiple gland hyperplasia. The condition is twice as
common in women and has increasing prevalence as
age increases. Familial cases may occur as part of MEN-
1 or MEN-2a. Treatment involves identification of the
affected gland by sestamibi technetium scanning and
removal by an experienced surgeon.

14937

© 2011 PasTest Ltd | About Us | Contact Us | Help

file:///E|/Shakhawan/Endocrinology/290a.htm[3/13/2012 7:15:23 PM]


MyPasTest

Main Navigation
Question Browser: MRCP 1
Home
Subscriptions Question Browser Exam Builder Saved Exams
» MRCP 1
• Question Browser
• Timed Test A 32-year-old lawyer presents with non-specific symptoms of tiredness. She is
• Mock Exam dissatisfied with her GP and arranges a private MRI scan of her pituitary. Blood testing
by her private physician reveals normal thyroid function, cortisol, growth hormone and
• Past Papers gonadotrophins. Her pituitary scan reveals a 0.8cm microadenoma.
• Random Questions
Which of the following represents the most appropriate course of action?
• My Performance
• Media Bank Treatment with dopamine agonist therapy
• New Multimedia Somatostatin therapy
Online Extras Bilateral sterotactic pituitary irradiation
Library Observation and reassurance
Community Trans-sphenoidal resection of pituitary
Help
14938
PasTest Store

My Account

Profile
Newsletters
My Career and Exams
Order History
Learning Goals
Question Filters

Security

Change Password
Sign Out

file:///E|/Shakhawan/Endocrinology/291.htm[3/13/2012 7:15:24 PM]


MyPasTest

Main Navigation
Question Browser: MRCP 1
Home
Subscriptions Question Browser Exam Builder Saved Exams
» MRCP 1 Session Progress
• Question Browser
Questions Correct 1
• Timed Test A 32-year-old lawyer presents with non-specific symptoms
of tiredness. She is dissatisfied with her GP and arranges a Questions Incorrect 0
• Mock Exam
private MRI scan of her pituitary. Blood testing by her Questions Total 1
• Past Papers private physician reveals normal thyroid function, cortisol,
Questions Percentage 100 %
• Random Questions growth hormone and gonadotrophins. Her pituitary scan
reveals a 0.8cm microadenoma. More
• My Performance
• Media Bank Which of the following represents the most appropriate
course of action?
• New Multimedia
Online Extras Treatment with dopamine agonist
Library therapy
Community Somatostatin therapy
Help Bilateral sterotactic pituitary irradiation
PasTest Store Observation and reassurance Your answer
Trans-sphenoidal resection of pituitary

My Account

Profile
Newsletters This patient has a microadenoma and a normal pituitary
My Career and Exams screen. Her tiredness is likely to be functional and
completely unrelated to the microadenoma discovered Reference: Normal Values
Order History
on MRI scan. All of the proposed interventions may be
Learning Goals associated with significant morbidity. Some autopsy Click to open/closeHaematology
Question Filters series have suggested that the incidence of pituitary Click to open/closeImmunoglobulins
tumours in those with an unrelated cause of death may Click to open/closeBiochemistry
be as high as 27%. Frequency of follow up is Click to open/closeDiabetes Click to
Security controversial, but this patient should be managed open/closeEndocrinology Click to
conservatively, with repeat MRI scanning and hormonal open/closeBlood gases Click to
Change Password monitoring. open/closeCSF
Sign Out
14938

© 2011 PasTest Ltd | About Us | Contact Us | Help

file:///E|/Shakhawan/Endocrinology/291a.htm[3/13/2012 7:15:26 PM]


MyPasTest

Main Navigation
Question Browser: MRCP 1
Home
Subscriptions Question Browser Exam Builder Saved Exams
» MRCP 1 Session Progress
• Question Browser
Questions Correct 1
• Timed Test A 54-year-old man who has received previous surgery for
acromegaly is receiving lanreotide medical therapy as his Questions Incorrect 0
• Mock Exam
growth hormone is persistently elevated. Questions Total 1
• Past Papers
What is the mode of action of lanreotide? Questions Percentage 100 %
• Random Questions
• My Performance More
Inhibition of the somatostatin receptor
• Media Bank
Inhibition of the growth hormone receptor
• New Multimedia
Stimulation of the somatostatin receptor
Online Extras Reference: Normal Values
Stimulation of the dopamine D2 receptor
Library
Inhibition of the dopamine D2 receptor Click to
Community open/closeHaematology
Help 14939 Click to
open/closeImmunoglobulins
PasTest Store
Click to
open/closeBiochemistry
My Account Click to open/closeDiabetes
Click to
Profile open/closeEndocrinology
Click to open/closeBlood
Newsletters gases Click to
My Career and Exams open/closeCSF
Order History
Learning Goals
Question Filters

Security

Change Password
Sign Out

© 2011 PasTest Ltd | About Us | Contact Us | Help

file:///E|/Shakhawan/Endocrinology/292.htm[3/13/2012 7:15:27 PM]


MyPasTest

Main Navigation
Question Browser: MRCP 1
Home
Subscriptions Question Browser Exam Builder Saved Exams
» MRCP 1 Session Progress
• Question Browser
Questions Correct 2
• Timed Test A 54-year-old man who has received previous surgery for
acromegaly is receiving lanreotide medical therapy as his Questions Incorrect 0
• Mock Exam
growth hormone is persistently elevated. Questions Total 2
• Past Papers
What is the mode of action of lanreotide? Questions Percentage 100 %
• Random Questions
• My Performance More
Inhibition of the somatostatin receptor
• Media Bank
Inhibition of the growth hormone
• New Multimedia receptor
Online Extras Stimulation of the somatostatin Your answer
Library receptor

Community Stimulation of the dopamine D2


receptor
Help
Inhibition of the dopamine D2 receptor
PasTest Store

My Account

Profile The somatostatin receptor is a 7 transmembrane G-


protein receptor. Lanreotide is a long-acting
Newsletters somatostatin analogue Stimulation of the somatostatin
My Career and Exams receptor leads to inhibition of production of a number of
hormones including gastrin, cholecystokinin, glucagon, Reference: Normal Values
Order History
growth hormone, insulin and secretin. Somatostatin
Learning Goals analogues are used for the medical treatment of Click to open/closeHaematology
Question Filters acromegaly, pre-surgery or after surgery of Click to open/closeImmunoglobulins
radiotherapy for persistently elevated growth hormone Click to open/closeBiochemistry
levels. They are also used for symptomatic relief of the Click to open/closeDiabetes Click to
Security carcinoid syndrome. open/closeEndocrinology Click to
open/closeBlood gases Click to
Change Password open/closeCSF
Sign Out 14939

© 2011 PasTest Ltd | About Us | Contact Us | Help

file:///E|/Shakhawan/Endocrinology/292a.htm[3/13/2012 7:15:29 PM]


MyPasTest

Main Navigation
Question Browser: MRCP 1
Home
Subscriptions Question Browser Exam Builder Saved Exams
» MRCP 1
• Question Browser
• Timed Test A 32-year-old man presents with unilateral gynaecomastia. He had breast reduction
• Mock Exam surgery on the other side 10 yrs ago. On examination he has, sparse body hair in the
axilla and pubic regions. Two testicles are identified, both of small volume. On further
• Past Papers questioning you elucidate that he has a normal sense of smell. His blood tests reveal:
• Random Questions
• My Performance Testoterone 4nmol/l (9-35)

• Media Bank LH + FSH elevated


• New Multimedia Prolactin 400mU/l (<360)
Online Extras
Which of the following is the most likely diagnosis?
Library
Community Klinefelter’s syndrome
Help Kallman’s syndrome
PasTest Store Androgen insensitivity
Testicular feminisation
My Account 17-beta hydroxylase deficiency

Profile 18596
Newsletters
My Career and Exams
Order History
Learning Goals
Question Filters

Security

Change Password
Sign Out

file:///E|/Shakhawan/Endocrinology/293.htm[3/13/2012 7:15:30 PM]


MyPasTest

Main Navigation
Question Browser: MRCP 1
Home
Subscriptions Question Browser Exam Builder Saved Exams
» MRCP 1 Session Progress
• Question Browser
Questions Correct 3
• Timed Test A 32-year-old man presents with unilateral gynaecomastia.
He had breast reduction surgery on the other side 10 yrs Questions Incorrect 0
• Mock Exam
ago. On examination he has, sparse body hair in the axilla Questions Total 3
• Past Papers and pubic regions. Two testicles are identified, both of
Questions Percentage 100 %
• Random Questions small volume. On further questioning you elucidate that he
has a normal sense of smell. His blood tests reveal: More
• My Performance
• Media Bank Testoterone 4nmol/l (9-35)
• New Multimedia LH + FSH elevated
Online Extras Prolactin 400mU/l (<360)
Library
Community Which of the following is the most likely diagnosis?
Help
Klinefelter’s syndrome Your answer
PasTest Store
Kallman’s syndrome
Androgen insensitivity
My Account Testicular feminisation
Profile 17-beta hydroxylase deficiency
Newsletters
My Career and Exams
Order History Reference: Normal Values
Learning Goals The elevated LH, FSH and the normal sense of smell
make Kallman’s syndrome less likely versus Click to open/closeHaematology
Question Filters Klinefelter's. Androgen insensitivity would be associated Click to open/closeImmunoglobulins
with a female phenotype and raised testosterone level. Click to open/closeBiochemistry
The subnormal level of testosterone here, coupled with Click to open/closeDiabetes Click to
Security the small volume testicles therefore makes Klinefelter’s open/closeEndocrinology Click to
the most likely diagnosis. open/closeBlood gases Click to
Change Password open/closeCSF
Sign Out
18596

© 2011 PasTest Ltd | About Us | Contact Us | Help

file:///E|/Shakhawan/Endocrinology/293a.htm[3/13/2012 7:15:31 PM]


MyPasTest

Main Navigation
Question Browser: MRCP 1
Home
Subscriptions Question Browser Exam Builder Saved Exams
» MRCP 1 Session Progress
• Question Browser
Questions Correct 3
• Timed Test A 42-year-old woman is referred to the clinic with very
difficult to manage hypertension. She is currently taking Questions Incorrect 0
• Mock Exam
indapamide, ramipril, amlodipine and doxasosin, yet her Questions Total 3
• Past Papers blood pressure is still 155/95 mmHg. On examination she
Questions Percentage 100 %
• Random Questions has a BMI of 25. Ophthalmoscopy reveals evidence of
chronic changes consistent with hypertension. Bloods More
• My Performance
reveal;
• Media Bank
• New Multimedia Hb 14.0g/dl
Online Extras WCC 5.8 x10 9 /l Reference: Normal Values
Library
PLT 190 x10 9 /l Click to
Community open/closeHaematology
Na + 139 mmol/l
Help Click to
K+ 3.3 mmol/l open/closeImmunoglobulins
PasTest Store
Click to
Creatinine 100 μmol/l open/closeBiochemistry
My Account Click to open/closeDiabetes
You suspect Conn’s syndrome. Click to
Profile open/closeEndocrinology
Which of the following is the investigation of choice Click to open/closeBlood
Newsletters gases Click to
to confirm the diagnosis?
My Career and Exams open/closeCSF
Order History CT Abdomen
Learning Goals Iodine (I 131 ) iodocholesterol scanning
Question Filters Aldosterone:renin ratio
MRI abdomen
Security Morning cortisol
Change Password 18597
Sign Out

© 2011 PasTest Ltd | About Us | Contact Us | Help

file:///E|/Shakhawan/Endocrinology/294.htm[3/13/2012 7:15:33 PM]


MyPasTest

Main Navigation
Question Browser: MRCP 1
Home
Subscriptions Question Browser Exam Builder Saved Exams
» MRCP 1
• Question Browser
• Timed Test A 42-year-old woman is referred to the clinic with very difficult to manage
• Mock Exam hypertension. She is currently taking indapamide, ramipril, amlodipine and doxasosin,
yet her blood pressure is still 155/95 mmHg. On examination she has a BMI of 25.
• Past Papers Ophthalmoscopy reveals evidence of chronic changes consistent with hypertension.
• Random Questions Bloods reveal;
• My Performance
Hb 14.0g/dl
• Media Bank
• New Multimedia WCC 5.8 x10 9 /l

Online Extras PLT 190 x10 9 /l


Library Na + 139 mmol/l
Community
K+ 3.3 mmol/l
Help
Creatinine 100 μmol/l
PasTest Store

You suspect Conn’s syndrome.


My Account
Which of the following is the investigation of choice to confirm the diagnosis?
Profile
Newsletters CT Abdomen
My Career and Exams Iodine (I 131 ) iodocholesterol scanning
Order History Aldosterone:renin ratio Your answer
Learning Goals MRI abdomen
Question Filters Morning cortisol

Security

Change Password
Whilst CT/ MRI abdomen is useful in differentiating the underlying cause of primary
Sign Out hyperaldosteronism (bilateral adrenal hyperplasia vs adenoma, aldosterone:renin
ratio is still needed to make the primary diagnosis. Anti-hypertensives can affect
interpretation of the result and ideally the test should be done following a period
off medication, although the test may still be of use if only some of the anti-
hypertensive medications can be discontinued. Iodocholesterol scanning is very
expensive and not a first line investigation. There is no indication of Cushing’s, so
a morning cortisol is not likely to be useful in this case.

18597

file:///E|/Shakhawan/Endocrinology/294a.htm[3/13/2012 7:15:34 PM]


MyPasTest

Main Navigation
Question Browser: MRCP 1
Home
Subscriptions Question Browser Exam Builder Saved Exams
» MRCP 1 Session Progress
• Question Browser
Questions Correct 4
• Timed Test A 38-year-old woman presents to her GP complaining of
palpitations, sweating and weight loss of around 4kg over Questions Incorrect 0
• Mock Exam
the past 6 months. She has a history of thyroid disease in Questions Total 4
• Past Papers the family. On examination she has a blood pressure of
Questions Percentage 100 %
• Random Questions 145/85 mmHg and a pulse of 92/minute.
• My Performance More
Bloods;
• Media Bank
• New Multimedia TSH <0.05 mU/l
Online Extras Hb 13.4 g/dl Reference: Normal Values
Library WCC 5.6 x10 9 /l
Click to
Community open/closeHaematology
PLT 223 x10 9 /l
Help Click to
PasTest Store Na + 140 mmol/l open/closeImmunoglobulins
Click to
K+ 4.0 mmol/l open/closeBiochemistry
My Account Creatinine 100 μmol/l Click to open/closeDiabetes
Click to
Profile open/closeEndocrinology
You suspect that she has thyrotoxicosis. Click to open/closeBlood
Newsletters gases Click to
My Career and Exams Which of the following fits best with the open/closeCSF
action/effects of excess thyroxine?
Order History
Learning Goals Improved insulin sensitivity
Question Filters Decreases myocardial oxygen demand
Increased tissue elasticity
Security Increases prolactin release
Leads to increased bone mass
Change Password
Sign Out 18598

© 2011 PasTest Ltd | About Us | Contact Us | Help

file:///E|/Shakhawan/Endocrinology/295.htm[3/13/2012 7:15:36 PM]


MyPasTest

Main Navigation
Question Browser: MRCP 1
Home
Subscriptions Question Browser Exam Builder Saved Exams
» MRCP 1 Session Progress
• Question Browser
Questions Correct 4
• Timed Test A 38-year-old woman presents to her GP complaining of
palpitations, sweating and weight loss of around 4kg over Questions Incorrect 1
• Mock Exam
the past 6 months. She has a history of thyroid disease in Questions Total 5
• Past Papers the family. On examination she has a blood pressure of
Questions Percentage 80 %
• Random Questions 145/85 mmHg and a pulse of 92/minute.
• My Performance More
Bloods;
• Media Bank
• New Multimedia TSH <0.05 mU/l
Online Extras Hb 13.4 g/dl
Library WCC 5.6 x10 9 /l
Community
PLT 223 x10 9 /l
Help
PasTest Store Na + 140 mmol/l

K+ 4.0 mmol/l
My Account Creatinine 100 μmol/l

Profile
You suspect that she has thyrotoxicosis.
Newsletters
My Career and Exams Which of the following fits best with the
action/effects of excess thyroxine?
Order History Reference: Normal Values
Learning Goals Improved insulin sensitivity Correct answer
Click to open/closeHaematology
Question Filters Decreases myocardial oxygen Click to open/closeImmunoglobulins
demand Click to open/closeBiochemistry
Increased tissue elasticity Your answer Click to open/closeDiabetes Click to
Security open/closeEndocrinology Click to
Increases prolactin release open/closeBlood gases Click to
Change Password open/closeCSF
Leads to increased bone mass
Sign Out

Excess thyroxine leads to improved insulin through an


association with reduced fat mass and increased insulin
sensitivity. It is also associated with increased
myocardial oxygen demand, which leads to presentation
with angina in some patients with thyrotoxicosis.
Increased prolactin occurs in hypothyroidism, and
thyrotoxicosis is associated with osteoporosis.

18598

© 2011 PasTest Ltd | About Us | Contact Us | Help

file:///E|/Shakhawan/Endocrinology/295a.htm[3/13/2012 7:15:38 PM]


MyPasTest

Main Navigation
Question Browser: MRCP 1
Home
Subscriptions Question Browser Exam Builder Saved Exams
» MRCP 1 Session Progress
• Question Browser
Questions Correct 4
• Timed Test A 21-year-old Medical Student presents with recurrent
collapses. These have occurred on a number of occasions in Questions Incorrect 1
• Mock Exam
association with stressful periods on the wards. Most Questions Total 5
• Past Papers recently she has been attached to the Emergency
Questions Percentage 80 %
• Random Questions department and has suffered two collapses during cardiac
arrests of patients on the unit. Her father has a history of More
• My Performance
Type 1 diabetes. You are on call with her overnight and she
• Media Bank collapses again, you collect a blood sample.
• New Multimedia
Bloods;
Online Extras Reference: Normal Values
Library Glucose 1.6 mmol/l
Click to
Community Insulin 1261 pmol/l open/closeHaematology
Help C-Peptide 20 pmol/l(<400) Click to
open/closeImmunoglobulins
PasTest Store
Click to
Which of the following is the most likely diagnosis? open/closeBiochemistry
My Account Click to open/closeDiabetes
Insulinoma Click to
Profile Glucagonoma open/closeEndocrinology
Click to open/closeBlood
Newsletters Phaeochromocytoma
gases Click to
My Career and Exams Illicit use of sulphonylureas open/closeCSF
Order History Illicit use of insulin
Learning Goals
18599
Question Filters

Security

Change Password
Sign Out

© 2011 PasTest Ltd | About Us | Contact Us | Help

file:///E|/Shakhawan/Endocrinology/296.htm[3/13/2012 7:15:39 PM]


MyPasTest

Main Navigation
Question Browser: MRCP 1
Home
Subscriptions Question Browser Exam Builder Saved Exams
» MRCP 1
• Question Browser
• Timed Test A 21-year-old Medical Student presents with recurrent collapses. These have occurred
• Mock Exam on a number of occasions in association with stressful periods on the wards. Most
recently she has been attached to the Emergency department and has suffered two
• Past Papers collapses during cardiac arrests of patients on the unit. Her father has a history of
• Random Questions Type 1 diabetes. You are on call with her overnight and she collapses again, you
collect a blood sample.
• My Performance
• Media Bank Bloods;
• New Multimedia
Glucose 1.6 mmol/l
Online Extras
Library Insulin 1261 pmol/l

Community C-Peptide 20 pmol/l(<400)

Help
Which of the following is the most likely diagnosis?
PasTest Store
Insulinoma

My Account Glucagonoma
Phaeochromocytoma
Profile
Illicit use of sulphonylureas
Newsletters
Illicit use of insulin Your answer
My Career and Exams
Order History
Learning Goals
Question Filters Whilst insulinoma would be a possibility in a patient with this medical history, the
marked elevation in insulin levels in the presence of a normal c-peptide suggests
that illicit use of insulin is the most likely diagnosis. Illicit use of sulphonylureas is
Security
commoner in relatives of patients with Type 2 diabetes. Collapse due
Change Password hypoglycaemia related to insulinoma is precipitated by a period of fasting +/-
exercise.
Sign Out

18599

file:///E|/Shakhawan/Endocrinology/296a.htm[3/13/2012 7:15:40 PM]


MyPasTest

Main Navigation
Question Browser: MRCP 1
Home
Subscriptions Question Browser Exam Builder Saved Exams
» MRCP 1 Session Progress
• Question Browser
Questions Correct 5
• Timed Test A 45-year-old man presents to the clinic complaining of a
lump in the neck which becomes particularly more Questions Incorrect 1
• Mock Exam
prominent when she swallows. On examination there is a Questions Total 6
• Past Papers thyroid nodule at the base of the neck. Leukocyte screening
Questions Percentage 83 %
• Random Questions of two other family members who suffered thyroid
carcinoma has revealed the ret proto-oncogene. More
• My Performance
• Media Bank Which type of thyroid carcinoma is linked to the ret-
• New Multimedia proto-oncogene?

Online Extras Papillary thyroid carcinoma Reference: Normal Values


Library Anaplastic thyroid carcinoma
Click to
Community Follicular thyroid carcinoma open/closeHaematology
Help Medullary thyroid carcinoma Click to
open/closeImmunoglobulins
PasTest Store Thyroid lymphoma Click to
open/closeBiochemistry
18600 Click to open/closeDiabetes
My Account
Click to
Profile open/closeEndocrinology
Click to open/closeBlood
Newsletters gases Click to
My Career and Exams open/closeCSF
Order History
Learning Goals
Question Filters

Security

Change Password
Sign Out

© 2011 PasTest Ltd | About Us | Contact Us | Help

file:///E|/Shakhawan/Endocrinology/297.htm[3/13/2012 7:15:42 PM]


MyPasTest

Main Navigation
Question Browser: MRCP 1
Home
Subscriptions Question Browser Exam Builder Saved Exams
» MRCP 1
• Question Browser
• Timed Test A 45-year-old man presents to the clinic complaining of a lump in the neck which
• Mock Exam becomes particularly more prominent when she swallows. On examination there is a
thyroid nodule at the base of the neck. Leukocyte screening of two other family
• Past Papers members who suffered thyroid carcinoma has revealed the ret proto-oncogene.
• Random Questions
Which type of thyroid carcinoma is linked to the ret-proto-oncogene?
• My Performance
• Media Bank Papillary thyroid carcinoma
• New Multimedia Anaplastic thyroid carcinoma
Online Extras Follicular thyroid carcinoma
Library Medullary thyroid carcinoma Your answer
Community Thyroid lymphoma
Help
PasTest Store

My Account The ret proto-oncogene encodes for a receptor tyrosine kinase. In medullary
thyroid carcinoma the mutation is associated with hyperfunction. Ret is associated
Profile with familial cases of medullary thyroid carcinoma and the MEN-2 syndrome.
Formerly the condition was screened for with the pentagastrin suppression test,
Newsletters
although a DNA probe for ret is now readily available and patients can be screened
My Career and Exams via a leukocyte sample.
Order History
Learning Goals 18600
Question Filters

Security

Change Password
Sign Out

file:///E|/Shakhawan/Endocrinology/297a.htm[3/13/2012 7:15:43 PM]


MyPasTest

Main Navigation
Question Browser: MRCP 1
Home
Subscriptions Question Browser Exam Builder Saved Exams
» MRCP 1 Session Progress
• Question Browser
Questions Correct 6
• Timed Test A 62-year-old man is brought to the Emergency
department with a grand-mal seizure. You are aware from Questions Incorrect 1
• Mock Exam
his notes that he has undergone chemotherapy for small Questions Total 7
• Past Papers cell carcinoma of the bronchus a few months earlier. His
Questions Percentage 85 %
• Random Questions wife tells you that he was drowsy and not himself for a day
or two before the seizure. More
• My Performance
• Media Bank On examination he is very drowsy but you can wake him
• New Multimedia up with stimulation. He is very thin with evidence of muscle
and fat loss consistent with his carcinoma. Neurological
Online Extras examination reveals no focal signs. Reference: Normal Values
Library
Bloods on admission; Click to
Community open/closeHaematology
Help Click to
Na + 123 mmol/l
open/closeImmunoglobulins
PasTest Store
K+ 4.0 mmol/l Click to
open/closeBiochemistry
Creatinine 100 µmol/l Click to open/closeDiabetes
My Account
Urea 3.5 mmol/l Click to
Profile open/closeEndocrinology
Hb 11.1 g/dl Click to open/closeBlood
Newsletters gases Click to
My Career and Exams WCC 4.5 x10 9 /l open/closeCSF
Order History PLT 230 x10 9 /l
Learning Goals
You suspect SIADH and fluid restrict him. Despite no
Question Filters evidence of him having taken in more fluids than
documented his sodium deteriorates to 119 mmol/l the
following day. His drowsiness has increased.
Security

Change Password Which of the following represents the most


appropriate management for him?
Sign Out
Continue fluid restriction
Start dexamethasone
Give normal saline 0.9%
Give saline 0.45%
Start demeclocycline

18686

© 2011 PasTest Ltd | About Us | Contact Us | Help

file:///E|/Shakhawan/Endocrinology/298.htm[3/13/2012 7:15:45 PM]


MyPasTest

Main Navigation
Question Browser: MRCP 1
Home
Subscriptions Question Browser Exam Builder Saved Exams
» MRCP 1 Session Progress
• Question Browser
Questions Correct 7
• Timed Test A 62-year-old man is brought to the Emergency
department with a grand-mal seizure. You are aware from Questions Incorrect 1
• Mock Exam
his notes that he has undergone chemotherapy for small Questions Total 8
• Past Papers cell carcinoma of the bronchus a few months earlier. His
Questions Percentage 87 %
• Random Questions wife tells you that he was drowsy and not himself for a day
or two before the seizure. More
• My Performance
• Media Bank On examination he is very drowsy but you can wake him
• New Multimedia up with stimulation. He is very thin with evidence of muscle
and fat loss consistent with his carcinoma. Neurological
Online Extras examination reveals no focal signs.
Library
Bloods on admission;
Community
Help
Na + 123 mmol/l
PasTest Store
K+ 4.0 mmol/l
Creatinine 100 µmol/l
My Account
Urea 3.5 mmol/l
Profile
Hb 11.1 g/dl
Newsletters
My Career and Exams WCC 4.5 x10 9 /l

Order History PLT 230 x10 9 /l Reference: Normal Values


Learning Goals
You suspect SIADH and fluid restrict him. Despite no Click to open/closeHaematology
Question Filters evidence of him having taken in more fluids than Click to open/closeImmunoglobulins
documented his sodium deteriorates to 119 mmol/l the Click to open/closeBiochemistry
following day. His drowsiness has increased. Click to open/closeDiabetes Click to
Security open/closeEndocrinology Click to
Which of the following represents the most open/closeBlood gases Click to
Change Password open/closeCSF
appropriate management for him?
Sign Out
Continue fluid restriction
Start dexamethasone
Give normal saline 0.9%
Give saline 0.45%
Start demeclocycline Your answer

This man remains drowsy and has suffered a further


dangerous fall in his serum sodium despite fluid
restriction. It seems unlikely on a closely monitored
ward that he would have access to fluids beyond his
restriction, therefore another intervention is required.
Demeclocycline is an efficient oral treatment for SIADH
which fails to respond to intake restriction. Like all
tetracyclines it leads to an increase in excretion of
water, leading to a rise in serum sodium levels.
Dexamethasone may exacerbate salt and water
retention further.

18686

file:///E|/Shakhawan/Endocrinology/298a.htm[3/13/2012 7:15:46 PM]


MyPasTest

Main Navigation
Question Browser: MRCP 1
Home
Subscriptions Question Browser Exam Builder Saved Exams
» MRCP 1 Session Progress
• Question Browser
Questions Correct 7
• Timed Test A 45-year-old builder is admitted to the Emergency room
after falling of his ladder. Past history of note includes Questions Incorrect 1
• Mock Exam
hypertension which is managed with ramipril 10mg daily. Questions Total 8
• Past Papers The Emergency scan reveals a 4cm right sided adrenal
Questions Percentage 87 %
• Random Questions mass. His blood pressure in the Emergency room is 145/90
mmHg. More
• My Performance
• Media Bank Blood results;
• New Multimedia
Hb 12.1 g/dl
Online Extras Reference: Normal Values
Library WCC 6.2 x10 9 /l
Click to
Community PLT 232 x10 9 /l open/closeHaematology
Help Click to
Na + 138 mmol/l open/closeImmunoglobulins
PasTest Store
K+ 3.6 mmol/l Click to
open/closeBiochemistry
Creatinine 100 μmol/l Click to open/closeDiabetes
My Account
Click to
Profile Which of the following would be most likely to reveal open/closeEndocrinology
the underlying diagnosis? Click to open/closeBlood
Newsletters gases Click to
My Career and Exams 24hr Urinary catecholamines open/closeCSF
Order History Plasma renin/aldosterone
Learning Goals MIBG scan
Question Filters MRI adrenals
Urinary sodium
Security
18753
Change Password
Sign Out

© 2011 PasTest Ltd | About Us | Contact Us | Help

file:///E|/Shakhawan/Endocrinology/299.htm[3/13/2012 7:15:48 PM]


MyPasTest

Main Navigation
Question Browser: MRCP 1
Home
Subscriptions Question Browser Exam Builder Saved Exams
» MRCP 1 Session Progress
• Question Browser
Questions Correct 7
• Timed Test A 45-year-old builder is admitted to the Emergency room
after falling of his ladder. Past history of note includes Questions Incorrect 2
• Mock Exam
hypertension which is managed with ramipril 10mg daily. Questions Total 9
• Past Papers The Emergency scan reveals a 4cm right sided adrenal
Questions Percentage 77 %
• Random Questions mass. His blood pressure in the Emergency room is 145/90
mmHg. More
• My Performance
• Media Bank Blood results;
• New Multimedia
Hb 12.1 g/dl
Online Extras
Library WCC 6.2 x10 9 /l
Community PLT 232 x10 9 /l
Help
Na + 138 mmol/l
PasTest Store
K+ 3.6 mmol/l
Creatinine 100 μmol/l
My Account

Profile Which of the following would be most likely to reveal


Newsletters the underlying diagnosis?

My Career and Exams 24hr Urinary catecholamines Your answer


Order History Plasma renin/aldosterone Correct answer Reference: Normal Values
Learning Goals MIBG scan Click to open/closeHaematology
Question Filters MRI adrenals Click to open/closeImmunoglobulins
Click to open/closeBiochemistry
Urinary sodium
Click to open/closeDiabetes Click to
Security open/closeEndocrinology Click to
open/closeBlood gases Click to
Change Password open/closeCSF
Sign Out
Primary hyperaldosteronism is commoner than was first
believed, and may be responsible for 10-15% of cases
of what is thought to be essential hypertension.
Aldosterone producing adenomas are commoner in
women than men, although adrenal hyperplasia is
commoner in men than women. Typically primary
aldosteronism presents in the 30-50 year age group.
Despite this being an incidental finding, the patient
should still be referred for surgical intervention.

18753

© 2011 PasTest Ltd | About Us | Contact Us | Help

file:///E|/Shakhawan/Endocrinology/299a.htm[3/13/2012 7:15:49 PM]


MyPasTest

Main Navigation
Question Browser: MRCP 1
Home
Subscriptions Question Browser Exam Builder Saved Exams
» MRCP 1 Session Progress
• Question Browser
Questions Correct 7
• Timed Test You review a 59-year-old man with Type 2 diabetes. He is
managed with metformin 1g BD and also has a history of Questions Incorrect 2
• Mock Exam
hypertension for which he is treated with ramipril 10mg. Questions Total 9
• Past Papers
Questions Percentage 77 %
• Random Questions Investigations;
• My Performance More
Hb 13.0 g/dl
• Media Bank
• New Multimedia WCC 5.1 x10 9 /l
Online Extras PLT 205 x10 9 /l Reference: Normal Values
Library
Na + 139 mmol/l Click to
Community open/closeHaematology
K+ 4.9 mmol/l
Help Click to
Creatinine 120 μmol/l open/closeImmunoglobulins
PasTest Store
Glucose 8.1 mmol/l Click to
open/closeBiochemistry
HbA1c 8.4% Click to open/closeDiabetes
My Account
Click to
Profile You elect to start a peroxisome proliferator-activated open/closeEndocrinology
receptor (PPAR)gamma agonist. Click to open/closeBlood
Newsletters gases Click to
My Career and Exams What is a main function of peroxisomes? open/closeCSF
Order History
DNA transcription
Learning Goals
Fatty acid metabolism
Question Filters
Protein catabolism
Peptide manufacture
Security
DNA replication
Change Password
18755
Sign Out

© 2011 PasTest Ltd | About Us | Contact Us | Help

file:///E|/Shakhawan/Endocrinology/300.htm[3/13/2012 7:15:51 PM]


MyPasTest

Main Navigation
Question Browser: MRCP 1
Home
Subscriptions Question Browser Exam Builder Saved Exams
» MRCP 1
• Question Browser
• Timed Test You review a 59-year-old man with Type 2 diabetes. He is managed with metformin
• Mock Exam 1g BD and also has a history of hypertension for which he is treated with ramipril
10mg.
• Past Papers
• Random Questions Investigations;
• My Performance
Hb 13.0 g/dl
• Media Bank
• New Multimedia WCC 5.1 x10 9 /l
Online Extras PLT 205 x10 9 /l
Library
Na + 139 mmol/l
Community
K+ 4.9 mmol/l
Help
Creatinine 120 μmol/l
PasTest Store
Glucose 8.1 mmol/l
HbA1c 8.4%
My Account

Profile You elect to start a peroxisome proliferator-activated receptor (PPAR)gamma agonist.


Newsletters
What is a main function of peroxisomes?
My Career and Exams
Order History DNA transcription

Learning Goals Fatty acid metabolism Your answer

Question Filters Protein catabolism


Peptide manufacture

Security DNA replication

Change Password
Sign Out

A main function of peroxisomes is beta-oxidation of fatty acids to produce acetyl


co-A which can be used as an energy source for the cell. PPAR-gamma agonists
increase metabolism of free fatty acids and promote differentiation of fat cells. It is
thought that their glucose lowering effects may well be due to their ability to
improve fatty acid metabolism and reduce visceral obesity and insulin resistance.

18755

file:///E|/Shakhawan/Endocrinology/300a.htm[3/13/2012 7:15:52 PM]


MyPasTest

Main Navigation
Question Browser: MRCP 1
Home
Subscriptions Question Browser Exam Builder Saved Exams
» MRCP 1 Session Progress
• Question Browser
Questions Correct 0
• Timed Test You review a 16-year-old girl with Type 1 diabetes. She is
currently managed with 18U of 30/70 mixed insulin BD. Her Questions Incorrect 0
• Mock Exam
parents tell you that she does not eat properly, and they Questions Total 0
• Past Papers occasionally hear her dreaming in the night and go in to
Questions Percentage 0%
• Random Questions find her suffering a hypoglycaemic attack.
• My Performance More
Investigations;
• Media Bank
• New Multimedia Hb 13.9 g/dl
Online Extras WCC 5.4 x10 9 /l Reference: Normal Values
Library
PLT 234 x10 9 /l Click to
Community open/closeHaematology
Help Na + 140 mmol/l Click to
K+ 5.0 mmol/l open/closeImmunoglobulins
PasTest Store
Click to
Creatinine 105 μmol/l open/closeBiochemistry
My Account HbA1c 9.4% Click to open/closeDiabetes
Click to
Profile open/closeEndocrinology
You decide to make a change to an insulin glargine/ Click to open/closeBlood
Newsletters novorapid basal bolus regime. gases Click to
My Career and Exams open/closeCSF
Which of the following best represents the main
Order History advantage of insulin glargine?
Learning Goals
Reduced post-prandial hypoglycaemia
Question Filters
Ability to miss a meal
Reduced nocturnal hypoglycaemia
Security
Increased ability to undertake exercise more easily
Change Password Ability to vary dosage on a day to day basis
Sign Out
18756

© 2011 PasTest Ltd | About Us | Contact Us | Help

file:///E|/Shakhawan/Endocrinology/301.htm[3/13/2012 7:15:54 PM]


MyPasTest

Main Navigation
Question Browser: MRCP 1
Home
Subscriptions Question Browser Exam Builder Saved Exams
» MRCP 1
• Question Browser
• Timed Test You review a 16-year-old girl with Type 1 diabetes. She is currently managed with
• Mock Exam 18U of 30/70 mixed insulin BD. Her parents tell you that she does not eat properly,
and they occasionally hear her dreaming in the night and go in to find her suffering a
• Past Papers hypoglycaemic attack.
• Random Questions
Investigations;
• My Performance
• Media Bank Hb 13.9 g/dl
• New Multimedia
WCC 5.4 x10 9 /l
Online Extras
Library PLT 234 x10 9 /l

Community Na + 140 mmol/l


Help K+ 5.0 mmol/l
PasTest Store
Creatinine 105 μmol/l
HbA1c 9.4%
My Account
You decide to make a change to an insulin glargine/ novorapid basal bolus regime.
Profile
Newsletters Which of the following best represents the main advantage of insulin
glargine?
My Career and Exams
Order History Reduced post-prandial hypoglycaemia
Learning Goals Ability to miss a meal
Question Filters Reduced nocturnal hypoglycaemia Your answer
Increased ability to undertake exercise more easily
Security Ability to vary dosage on a day to day basis
Change Password
Sign Out

Insulin glargine is an insulin analogue produced by substitution of glycine for


asparagines at A21 and by adding two arginines to the B chain. It is formulated at
pH 4, but, when injected, the transformation to pH 7 causes the insulin crystals to
precipitate leading to a slowly released, “peak less” profile. The main advantage of
insulin glargine is in reduction of nocturnal hypoglycaemia. Short acting analogues
are useful in reducing post-prandial hypoglycaemia, as their profile is more in
keeping with physiological insulin release.

18756

file:///E|/Shakhawan/Endocrinology/301a.htm[3/13/2012 7:15:55 PM]


MyPasTest

Main Navigation
Question Browser: MRCP 1
Home
Subscriptions Question Browser Exam Builder Saved Exams
» MRCP 1 Session Progress
• Question Browser
Questions Correct 1
• Timed Test A 19-year-old woman is admitted to the Emergency room
after a collapse. She is found to be hypoglycaemic. You Questions Incorrect 0
• Mock Exam
understand from colleagues who accompanied her in the Questions Total 1
• Past Papers ambulance that she collapsed at work, and this has
Questions Percentage 100 %
• Random Questions happened at least 3 times in the past 2 months. There is no
past medical history of note apart from the fact that her More
• My Performance
father is obese and was diagnosed with Type 2 diabetes
• Media Bank some 7 years ago.
• New Multimedia
Bloods;
Online Extras Reference: Normal Values
Library Hb 13.9 g/dl
Click to
Community
WCC 5.6 x10 9 /l open/closeHaematology
Help Click to
PLT 231 x10 9 /l open/closeImmunoglobulins
PasTest Store
Click to
Na + 139 mmol/l open/closeBiochemistry
My Account K+ 3.9 mmol/l Click to open/closeDiabetes
Click to
Profile Creatinine 80 µmol/l open/closeEndocrinology
Glucose 2.4 mmol/l Click to open/closeBlood
Newsletters gases Click to
My Career and Exams Insulin 350 IU/l open/closeCSF
Order History C-Peptide 0.1 U/l
Learning Goals
Which of the following is the most likely diagnosis?
Question Filters
Sulphonylurea abuse
Security Metformin abuse

Change Password Retroperitoneal tumour

Sign Out Insulinoma


Insulin abuse

18758

© 2011 PasTest Ltd | About Us | Contact Us | Help

file:///E|/Shakhawan/Endocrinology/302.htm[3/13/2012 7:15:57 PM]


MyPasTest

Main Navigation
Question Browser: MRCP 1
Home
Subscriptions Question Browser Exam Builder Saved Exams
» MRCP 1 Session Progress
• Question Browser
Questions Correct 1
• Timed Test A 19-year-old woman is admitted to the Emergency room
after a collapse. She is found to be hypoglycaemic. You Questions Incorrect 1
• Mock Exam
understand from colleagues who accompanied her in the Questions Total 2
• Past Papers ambulance that she collapsed at work, and this has
Questions Percentage 50 %
• Random Questions happened at least 3 times in the past 2 months. There is no
past medical history of note apart from the fact that her More
• My Performance
father is obese and was diagnosed with Type 2 diabetes
• Media Bank some 7 years ago.
• New Multimedia
Bloods;
Online Extras
Library Hb 13.9 g/dl
Community
WCC 5.6 x10 9 /l
Help
PLT 231 x10 9 /l
PasTest Store
Na + 139 mmol/l

My Account K+ 3.9 mmol/l

Profile Creatinine 80 µmol/l


Newsletters Glucose 2.4 mmol/l
My Career and Exams Insulin 350 IU/l
Order History C-Peptide 0.1 U/l Reference: Normal Values
Learning Goals
Which of the following is the most likely diagnosis? Click to open/closeHaematology
Question Filters Click to open/closeImmunoglobulins
Sulphonylurea abuse Your answer Click to open/closeBiochemistry
Click to open/closeDiabetes Click to
Security Metformin abuse open/closeEndocrinology Click to
Retroperitoneal tumour open/closeBlood gases Click to
Change Password open/closeCSF
Sign Out Insulinoma
Insulin abuse Correct answer

This patient has hypoglycaemia, a marked elevation in


insulin and a c-peptide in the normal range. Coupled
with the fact that her father was diagnosed with Type 2
diabetes some 7 years ago, and is likely to be managed
with insulin, this makes abuse of exogenous insulin the
most likely diagnosis in this case. Had c-peptide levels
also been elevated, then testing of urine for the
metabolites of commonly used sulphonylureas would
have been the logical next step. As metformin merely
reduces hepatic glucose output, it is almost never
associated with symptoms of hypoglycaemia.

18758

© 2011 PasTest Ltd | About Us | Contact Us | Help

file:///E|/Shakhawan/Endocrinology/302a.htm[3/13/2012 7:15:58 PM]


MyPasTest

Main Navigation
Question Browser: MRCP 1
Home
Subscriptions Question Browser Exam Builder Saved Exams
» MRCP 1 Session Progress
• Question Browser
Questions Correct 1
• Timed Test A 20-year-old student presents to the diabetes clinic for
review. She is a keen rower who suffers from Type 1 Questions Incorrect 1
• Mock Exam
diabetes and has suffered two severe hypoglycaemic Questions Total 2
• Past Papers attacks after rowing training. Her current insulin regime is
Questions Percentage 50 %
• Random Questions BD 30/70 mixed insulin.
• My Performance More
Investigations;
• Media Bank
• New Multimedia Hb 13.1 g/dl
Online Extras WCC 4.3 x10 9 /l Reference: Normal Values
Library
PLT 295 x10 9 /l Click to
Community open/closeHaematology
Help Na + 140 mmol/l Click to
K+ 4.4 mmol/l open/closeImmunoglobulins
PasTest Store
Click to
Creatinine 100 μmol/l open/closeBiochemistry
My Account HbA1c 9.1% Click to open/closeDiabetes
Click to
Profile open/closeEndocrinology
You plan to make a change to a basal bolus insulin regime Click to open/closeBlood
Newsletters gases Click to
My Career and Exams Which of the following would be the most appropriate open/closeCSF
short acting insulin for her?
Order History
Learning Goals Insulin glargine
Question Filters Insulin lispro
Insulin detemir
Security Insulotard
Novomix 50/50
Change Password
Sign Out 18759

© 2011 PasTest Ltd | About Us | Contact Us | Help

file:///E|/Shakhawan/Endocrinology/303.htm[3/13/2012 7:16:00 PM]


MyPasTest

Main Navigation
Question Browser: MRCP 1
Home
Subscriptions Question Browser Exam Builder Saved Exams
» MRCP 1 Session Progress
• Question Browser
Questions Correct 2
• Timed Test A 20-year-old student presents to the diabetes clinic for
review. She is a keen rower who suffers from Type 1 Questions Incorrect 1
• Mock Exam
diabetes and has suffered two severe hypoglycaemic Questions Total 3
• Past Papers attacks after rowing training. Her current insulin regime is
Questions Percentage 66 %
• Random Questions BD 30/70 mixed insulin.
• My Performance More
Investigations;
• Media Bank
• New Multimedia Hb 13.1 g/dl
Online Extras WCC 4.3 x10 9 /l
Library
PLT 295 x10 9 /l
Community
Help Na + 140 mmol/l

PasTest Store K+ 4.4 mmol/l


Creatinine 100 μmol/l
My Account HbA1c 9.1%

Profile
You plan to make a change to a basal bolus insulin regime
Newsletters
My Career and Exams Which of the following would be the most appropriate
short acting insulin for her?
Order History Reference: Normal Values
Learning Goals Insulin glargine
Click to open/closeHaematology
Question Filters Insulin lispro Your answer Click to open/closeImmunoglobulins
Insulin detemir Click to open/closeBiochemistry
Click to open/closeDiabetes Click to
Security Insulotard open/closeEndocrinology Click to
Novomix 50/50 open/closeBlood gases Click to
Change Password open/closeCSF
Sign Out

Both insulin glargine and insulin detemir are long-acting


insulin analogues, which have been primarily shown in
Type 1 diabetes to reduce nocturnal hypoglycaemia.
Insulotard is traditional long-acting insulin, novomix
50/50 is unlikely to be of much help in this patient as it
would still necessitate a BD dosing regime. Insulin lispro
was the first insulin analogue to be developed, by
reversing the position of two aminoacids (lysine and
praline) in the B-chain of human insulin. This allows
larger amounts of prandial insulin to be available soon
after injection and reduces the chance of between meal
hypoglycaemia.

18759

© 2011 PasTest Ltd | About Us | Contact Us | Help

file:///E|/Shakhawan/Endocrinology/303a.htm[3/13/2012 7:16:01 PM]


MyPasTest

Main Navigation
Question Browser: MRCP 1
Home
Subscriptions Question Browser Exam Builder Saved Exams
» MRCP 1 Session Progress
• Question Browser
Questions Correct 2
• Timed Test A 39-year-old woman with Hashimoto’s thyroiditis presents
to the clinic for review. Her Hashimoto’s is managed with Questions Incorrect 1
• Mock Exam
thyroxine 125μg/day replacement. She presents to the Questions Total 3
• Past Papers clinic complaining of bilateral loin pain.
Questions Percentage 66 %
• Random Questions
Investigations; More
• My Performance
• Media Bank Hb 12.1 g/dl
• New Multimedia
WCC 5.4 x10 9 /l
Online Extras Reference: Normal Values
Library PLT 294 x10 9 /l
Click to
Community Na + 139 mmol/l open/closeHaematology
Help K+ 3.5 mmol/l Click to
open/closeImmunoglobulins
PasTest Store
Creatinine 140 μmol/l Click to
open/closeBiochemistry
HCO3 - 15 mmol/l Click to open/closeDiabetes
My Account
KUB Evidence of nephrocalcinosis Click to
Profile open/closeEndocrinology
Click to open/closeBlood
Newsletters Which of the following is the most likely diagnosis? gases Click to
My Career and Exams open/closeCSF
Renal tubular acidosis Type 4
Order History
Medullary sponge kidney
Learning Goals
Renal tubular acidosis Type 2
Question Filters
Renal tubular acidosis Type 1
Chronic interstitial nephritis
Security
18767
Change Password
Sign Out

© 2011 PasTest Ltd | About Us | Contact Us | Help

file:///E|/Shakhawan/Endocrinology/304.htm[3/13/2012 7:16:03 PM]


MyPasTest

Main Navigation
Question Browser: MRCP 1
Home
Subscriptions Question Browser Exam Builder Saved Exams
» MRCP 1 Session Progress
• Question Browser
Questions Correct 3
• Timed Test A 39-year-old woman with Hashimoto’s thyroiditis presents
to the clinic for review. Her Hashimoto’s is managed with Questions Incorrect 1
• Mock Exam
thyroxine 125μg/day replacement. She presents to the Questions Total 4
• Past Papers clinic complaining of bilateral loin pain.
Questions Percentage 75 %
• Random Questions
Investigations; More
• My Performance
• Media Bank Hb 12.1 g/dl
• New Multimedia
WCC 5.4 x10 9 /l
Online Extras
Library PLT 294 x10 9 /l

Community Na + 139 mmol/l


Help K+ 3.5 mmol/l
PasTest Store
Creatinine 140 μmol/l

HCO3 - 15 mmol/l
My Account
KUB Evidence of nephrocalcinosis
Profile
Newsletters Which of the following is the most likely diagnosis?
My Career and Exams
Renal tubular acidosis Type 4
Order History Reference: Normal Values
Medullary sponge kidney
Learning Goals
Renal tubular acidosis Type 2 Click to open/closeHaematology
Question Filters Click to open/closeImmunoglobulins
Renal tubular acidosis Type 1 Your answer Click to open/closeBiochemistry
Chronic interstitial nephritis Click to open/closeDiabetes Click to
Security open/closeEndocrinology Click to
open/closeBlood gases Click to
Change Password open/closeCSF
Sign Out
Renal tubular acidosis Type 1 (hypokalaemic distal renal
tubular acidosis) may occur in association with a
number of autoimmune conditions including SLE,
vasulitis and autoimmune hypothyroidism. Other causes
include drugs such as amphotericin B and lithium,
inherited conditions such as Marfan’s and conditions
such as chronic pyelonephritis and obstructive uropathy.
Treatment includes oral supplementation with potassium
and bicarbonate.

18767

© 2011 PasTest Ltd | About Us | Contact Us | Help

file:///E|/Shakhawan/Endocrinology/304a.htm[3/13/2012 7:16:04 PM]


MyPasTest

Main Navigation
Question Browser: MRCP 1
Home
Subscriptions Question Browser Exam Builder Saved Exams
» MRCP 1 Session Progress
• Question Browser
Questions Correct 3
• Timed Test A 42-year-old woman presents with a mass in her neck
and symptoms of choking/ shortness of breath when she Questions Incorrect 1
• Mock Exam
lies down. Past history of note includes a family history of Questions Total 4
• Past Papers goitre, for which her mother and grandmother both
Questions Percentage 75 %
• Random Questions endured partial thyroid resection.
• My Performance More
You examine her and suspect that she has a multinodular
• Media Bank goitre.
• New Multimedia
Investigations;
Online Extras Reference: Normal Values
Library Hb 14.1 g/dl
Click to
Community
WCC 5.4 x10 9 /l open/closeHaematology
Help Click to
PLT 235 x10 9 /l open/closeImmunoglobulins
PasTest Store
Click to
Na + 140 mmol/l open/closeBiochemistry
My Account K+ 4.5 mmol/l Click to open/closeDiabetes
Click to
Profile Creatinine 100 μmol/l open/closeEndocrinology
TSH 0.9 U/l Click to open/closeBlood
Newsletters gases Click to
My Career and Exams open/closeCSF
You suspect that she may have retrosternal thyroid
Order History extension
Learning Goals
Which of the following would be the investigation of
Question Filters choice for this patient to determine the degree of
retrosternal involvement?
Security
CXR
Change Password Thyroid ultrasound
Sign Out X-ray of thoracic inlet
CT Chest
Radioisotope

18768

© 2011 PasTest Ltd | About Us | Contact Us | Help

file:///E|/Shakhawan/Endocrinology/305.htm[3/13/2012 7:16:06 PM]


MyPasTest

Main Navigation
Question Browser: MRCP 1
Home
Subscriptions Question Browser Exam Builder Saved Exams
» MRCP 1 Session Progress
• Question Browser
Questions Correct 3
• Timed Test A 42-year-old woman presents with a mass in her neck
and symptoms of choking/ shortness of breath when she Questions Incorrect 2
• Mock Exam
lies down. Past history of note includes a family history of Questions Total 5
• Past Papers goitre, for which her mother and grandmother both
Questions Percentage 60 %
• Random Questions endured partial thyroid resection.
• My Performance More
You examine her and suspect that she has a multinodular
• Media Bank goitre.
• New Multimedia
Investigations;
Online Extras
Library Hb 14.1 g/dl
Community
WCC 5.4 x10 9 /l
Help
PLT 235 x10 9 /l
PasTest Store
Na + 140 mmol/l

My Account K+ 4.5 mmol/l

Profile Creatinine 100 μmol/l


Newsletters TSH 0.9 U/l
My Career and Exams
You suspect that she may have retrosternal thyroid
Order History extension Reference: Normal Values
Learning Goals
Which of the following would be the investigation of Click to open/closeHaematology
Question Filters choice for this patient to determine the degree of Click to open/closeImmunoglobulins
retrosternal involvement? Click to open/closeBiochemistry
Click to open/closeDiabetes Click to
Security open/closeEndocrinology Click to
CXR
open/closeBlood gases Click to
Change Password Thyroid ultrasound Your answer open/closeCSF
Sign Out X-ray of thoracic inlet
CT Chest Correct answer
Radioisotope

Whilst a plain radiograph may be used to demonstrate


retrosternal thyroid extension and tracheal deviation, a
CT of the chest is more sensitive and specific in
determining the extent of disease. Thyroid ultrasound is
most useful for determining the nature of intra-thyroid
lesions. Radioisotope scanning has a role to play in
determining the uptake of thyroid hormone and as such
the underlying diagnosis behind thyrotoxicosis, or in
identifying cold nodules within the gland which may
raise the suspicion of an underlying neoplasm.

18768

© 2011 PasTest Ltd | About Us | Contact Us | Help

file:///E|/Shakhawan/Endocrinology/305a.htm[3/13/2012 7:16:07 PM]


MyPasTest

Main Navigation
Question Browser: MRCP 1
Home
Subscriptions Question Browser Exam Builder Saved Exams
» MRCP 1 Session Progress
• Question Browser
Questions Correct 3
• Timed Test A 54-year-old man presents with a new diagnosis of Type
2 diabetes. He is also concerned that his fingers seem a Questions Incorrect 2
• Mock Exam
little swollen and he is unable to wear his wedding ring Questions Total 5
• Past Papers anymore. He attends with some pictures from his youth,
Questions Percentage 60 %
• Random Questions which appear to show that his jaw has acquired increasing
prominence over the past few years. On examination his BP More
• My Performance
is 152/90 mmHg.
• Media Bank
• New Multimedia Investigations;

Online Extras Reference: Normal Values


Hb 14.0 g/dl
Library
WCC 5.1 x10 9 /l Click to
Community open/closeHaematology
Help PLT 302 x10 9 /l Click to
open/closeImmunoglobulins
PasTest Store Na + 140 mmol/l Click to
K+ 5.0 mmol/l open/closeBiochemistry
My Account Click to open/closeDiabetes
Creatinine 110 μmol/l Click to
Profile Glucose 9.0 mmol/l open/closeEndocrinology
Click to open/closeBlood
Newsletters gases Click to
My Career and Exams Which of the following would be the most appropriate open/closeCSF
initial screening test for the likely underlying
Order History condition?
Learning Goals
GNRH concentration
Question Filters
Oral glucose tolerance test with growth hormone
measurement
Security IGF-1 measurement
Change Password IGFBP-3 measurement
Sign Out Random growth hormone measurement

20469

© 2011 PasTest Ltd | About Us | Contact Us | Help

file:///E|/Shakhawan/Endocrinology/306.htm[3/13/2012 7:16:09 PM]


MyPasTest

Main Navigation
Question Browser: MRCP 1
Home
Subscriptions Question Browser Exam Builder Saved Exams
» MRCP 1 Session Progress
• Question Browser
Questions Correct 4
• Timed Test A 54-year-old man presents with a new diagnosis of Type
2 diabetes. He is also concerned that his fingers seem a Questions Incorrect 2
• Mock Exam
little swollen and he is unable to wear his wedding ring Questions Total 6
• Past Papers anymore. He attends with some pictures from his youth,
Questions Percentage 66 %
• Random Questions which appear to show that his jaw has acquired increasing
prominence over the past few years. On examination his BP More
• My Performance
is 152/90 mmHg.
• Media Bank
• New Multimedia Investigations;

Online Extras
Hb 14.0 g/dl
Library
WCC 5.1 x10 9 /l
Community
Help PLT 302 x10 9 /l
PasTest Store Na + 140 mmol/l

K+ 5.0 mmol/l
My Account
Creatinine 110 μmol/l
Profile Glucose 9.0 mmol/l
Newsletters
My Career and Exams Which of the following would be the most appropriate
initial screening test for the likely underlying
Order History condition? Reference: Normal Values
Learning Goals
GNRH concentration Click to open/closeHaematology
Question Filters Click to open/closeImmunoglobulins
Oral glucose tolerance test with growth Click to open/closeBiochemistry
hormone measurement Click to open/closeDiabetes Click to
Security IGF-1 measurement Your answer open/closeEndocrinology Click to
open/closeBlood gases Click to
Change Password IGFBP-3 measurement open/closeCSF
Sign Out Random growth hormone
measurement

Growth hormone stimulates the production of IGF-1,


and IGF-1 has a long half life, as such it is the ideal
measure of growth hormone secretion. The oral glucose
tolerance test (OGTT) using 75g is a sensitive test for
acromegaly, with failure of growth hormone to suppress
suggesting the diagnosis, it should be noted however
that 15-20% of patients with acromegaly have a
paradoxical rise in growth hormone secretion post
OGTT. IGFBP-3 is the main circulating binding protein
for IGF and may of utility in the future as a screening
test for acromegaly. Random growth hormone
measurements are not useful as a screening test for
acromegaly because of the episodic nature of secretion.

20469

file:///E|/Shakhawan/Endocrinology/306a.htm[3/13/2012 7:16:10 PM]


MyPasTest

Main Navigation
Question Browser: MRCP 1
Home
Subscriptions Question Browser Exam Builder Saved Exams
» MRCP 1 Session Progress
• Question Browser
Questions Correct 4
• Timed Test A 48-year-old psychiatric nurse with Type 2 diabetes
presents to the clinic for review. He was diagnosed with Questions Incorrect 2
• Mock Exam
Type 2 diabetes following a myocardial infarction 3 years Questions Total 6
• Past Papers ago. Current medication includes metformin 1g bd, ramipril
Questions Percentage 66 %
• Random Questions 10mg, simvastatin 40mg and aspirin 75mg. He aslo takes
bendroflumethiazide started by his GP for mild bilateral More
• My Performance
ankle swelling. On examination his BMI is 33 and he has
• Media Bank gained 7kg since his last appointment 6 months earlier. His
• New Multimedia BP is 135/80 mmHg.

Online Extras Investigations; Reference: Normal Values


Library
Hb 12.9 g/dl Click to
Community open/closeHaematology
Help WCC 4.9 x10 9 /l Click to
open/closeImmunoglobulins
PasTest Store PLT 300 x10 9 /l Click to
open/closeBiochemistry
Na + 140 mmol/l Click to open/closeDiabetes
My Account
K+ 5.4 mmol/l Click to
Profile open/closeEndocrinology
Creatinine 120 µmol/l Click to open/closeBlood
Newsletters gases Click to
HBA1c 8.5%
My Career and Exams open/closeCSF
C-peptide elevated
Order History
Learning Goals Which of the following represents the most
Question Filters appropriate additional medication for him?

Pioglitazone
Security
Gliclazide
Change Password Bedtime insulin glargine
Sign Out BD mixed insulin
Sitagliptin

20470

© 2011 PasTest Ltd | About Us | Contact Us | Help

file:///E|/Shakhawan/Endocrinology/307.htm[3/13/2012 7:16:12 PM]


MyPasTest

Main Navigation
Question Browser: MRCP 1
Home
Subscriptions Question Browser Exam Builder Saved Exams
» MRCP 1 Session Progress
• Question Browser
Questions Correct 4
• Timed Test A 48-year-old psychiatric nurse with Type 2 diabetes
presents to the clinic for review. He was diagnosed with Questions Incorrect 3
• Mock Exam
Type 2 diabetes following a myocardial infarction 3 years Questions Total 7
• Past Papers ago. Current medication includes metformin 1g bd, ramipril
Questions Percentage 57 %
• Random Questions 10mg, simvastatin 40mg and aspirin 75mg. He aslo takes
bendroflumethiazide started by his GP for mild bilateral More
• My Performance
ankle swelling. On examination his BMI is 33 and he has
• Media Bank gained 7kg since his last appointment 6 months earlier. His
• New Multimedia BP is 135/80 mmHg.

Online Extras Investigations;


Library
Community Hb 12.9 g/dl

Help WCC 4.9 x10 9 /l


PasTest Store PLT 300 x10 9 /l

Na + 140 mmol/l
My Account
K+ 5.4 mmol/l
Profile
Creatinine 120 µmol/l
Newsletters
HBA1c 8.5%
My Career and Exams
C-peptide elevated Reference: Normal Values
Order History
Learning Goals Which of the following represents the most Click to open/closeHaematology
Question Filters appropriate additional medication for him? Click to open/closeImmunoglobulins
Click to open/closeBiochemistry
Pioglitazone Click to open/closeDiabetes Click to
Security open/closeEndocrinology Click to
Gliclazide
open/closeBlood gases Click to
Change Password Bedtime insulin glargine Your answer open/closeCSF
Sign Out BD mixed insulin
Sitagliptin Correct answer

The mild ankle swelling may be suggestive of heart


failure, as such pioglitazone is not recommended for this
patient. We are told he works as a psychiatric nurse, as
such he may work shift work, we are also told that his
weight is increasing, so that an SU may not be the most
appropriate option. Insulin should be avoided for the
moment for similar reasons. Sitagliptin in contrast is
weight neutral and so far, evidence suggests a
relatively benign side-effect profile.

20470

© 2011 PasTest Ltd | About Us | Contact Us | Help

file:///E|/Shakhawan/Endocrinology/307a.htm[3/13/2012 7:16:13 PM]


MyPasTest

Main Navigation
Question Browser: MRCP 1
Home
Subscriptions Question Browser Exam Builder Saved Exams
» MRCP 1
• Question Browser
• Timed Test A 19-year-old man presents to his GP complaining of tiredness, polyuria, thirst and
• Mock Exam recurrent thrush infections. He has a history of Type 2 diabetes in the family. On
examination he looks slim, with a BMI of 22.
• Past Papers
• Random Questions Investigations;
• My Performance
Hb 12.9 g/dl
• Media Bank
• New Multimedia WCC 5.4 x10 9 /l
Online Extras PLT 301 x10 9 /l
Library
Na + 140 mmol/l
Community
K+ 5.0 mmol/l
Help
Creatinine 120 µmol/l
PasTest Store
Fasting glucose 8.5 mmol/l
Islet cell, GAD, insulin antibody negative
My Account
HDL 1.8 mmol/l (>1.5)
Profile
Newsletters Which of the following is the most likely diagnosis?
My Career and Exams
Type 1 diabetes
Order History
Type 2 diabetes
Learning Goals
(Maturity Onset Diabetes of the Young) MODY 3
Question Filters
MODY 5
(Latent Autoimmune Diabetes of Adulthood) LADA
Security
20471
Change Password
Sign Out

file:///E|/Shakhawan/Endocrinology/308.htm[3/13/2012 7:16:15 PM]


MyPasTest

Main Navigation
Question Browser: MRCP 1
Home
Subscriptions Question Browser Exam Builder Saved Exams
» MRCP 1 Session Progress
• Question Browser
Questions Correct 5
• Timed Test A 19-year-old man presents to his GP complaining of
tiredness, polyuria, thirst and recurrent thrush infections. Questions Incorrect 3
• Mock Exam
He has a history of Type 2 diabetes in the family. On Questions Total 8
• Past Papers examination he looks slim, with a BMI of 22.
Questions Percentage 62 %
• Random Questions
Investigations; More
• My Performance
• Media Bank Hb 12.9 g/dl
• New Multimedia
WCC 5.4 x10 9 /l
Online Extras
Library PLT 301 x10 9 /l

Community Na + 140 mmol/l


Help K+ 5.0 mmol/l
PasTest Store
Creatinine 120 µmol/l
Fasting glucose 8.5 mmol/l
My Account
Islet cell, GAD, insulin antibody
Profile negative
Newsletters 1.8 mmol/l
HDL (>1.5)
My Career and Exams
Order History Which of the following is the most likely diagnosis? Reference: Normal Values
Learning Goals
Type 1 diabetes Click to open/closeHaematology
Question Filters Click to open/closeImmunoglobulins
Type 2 diabetes Click to open/closeBiochemistry
(Maturity Onset Diabetes of the Young) Your answer Click to open/closeDiabetes Click to
Security MODY 3 open/closeEndocrinology Click to
open/closeBlood gases Click to
Change Password MODY 5 open/closeCSF
Sign Out (Latent Autoimmune Diabetes of
Adulthood) LADA

MODY 3 is characterised by raised HDL cholesterol,


preserved insulin sensitivity and a low renal threshold
(glycosuria). It is the commonest form of MODY, with
age of onset between the 10-30 year age group. MODY
can be managed in many patients with the use of
sulphonylureas only. The sub-acute presentation the
fact that this patient is antibody negative suggests that
a diagnosis of Type 1 diabetes is less likely here.

20471

© 2011 PasTest Ltd | About Us | Contact Us | Help

file:///E|/Shakhawan/Endocrinology/308a.htm[3/13/2012 7:16:16 PM]


MyPasTest

Main Navigation
Question Browser: MRCP 1
Home
Subscriptions Question Browser Exam Builder Saved Exams
» MRCP 1 Session Progress
• Question Browser
Questions Correct 5
• Timed Test You see a poorly controlled 58-year-old type 2 diabetes
patient who is fasting for Ramadan. He has background Questions Incorrect 3
• Mock Exam
diabetic retinopathy, and currently takes metformin 500mg Questions Total 8
• Past Papers TDS as well as ramipril 10mg and aspirin 75mg.
Questions Percentage 62 %
• Random Questions
Investigations More
• My Performance
• Media Bank Hb 12.1 g/dl
• New Multimedia
WCC 5.0 x10 9 /l
Online Extras Reference: Normal Values
Library PLT 212 x10 9 /l
Click to
Community Na + 139 mmol/l open/closeHaematology
Help K+ 4.9 mmol/l Click to
open/closeImmunoglobulins
PasTest Store
Creatinine 120 μmol/l Click to
open/closeBiochemistry
HbA1c 7.4%
My Account

Profile What advice would you give him regarding his


medication while fasting?
Newsletters
My Career and Exams Stop metformin
Order History Give short acting insulin for the evening meal
Learning Goals Switch to glicazide BD
Question Filters Give metformin 500mg before dawn, 1000mg in
evening
Do not fast for health reasons
Security

Change Password 20732


Sign Out

file:///E|/Shakhawan/Endocrinology/309.htm[3/13/2012 7:16:18 PM]


MyPasTest

Main Navigation
Question Browser: MRCP 1
Home
Subscriptions Question Browser Exam Builder Saved Exams
» MRCP 1
• Question Browser
• Timed Test You see a poorly controlled 58-year-old type 2 diabetes patient who is fasting for
• Mock Exam Ramadan. He has background diabetic retinopathy, and currently takes metformin
500mg TDS as well as ramipril 10mg and aspirin 75mg.
• Past Papers
• Random Questions Investigations
• My Performance
Hb 12.1 g/dl
• Media Bank
• New Multimedia WCC 5.0 x10 9 /l
Online Extras PLT 212 x10 9 /l
Library
Na + 139 mmol/l
Community
K+ 4.9 mmol/l
Help
PasTest Store Creatinine 120 μmol/l
HbA1c 7.4%

My Account
What advice would you give him regarding his medication while fasting?
Profile
Stop metformin
Newsletters
Give short acting insulin for the evening meal
My Career and Exams
Switch to glicazide BD
Order History
Give metformin 500mg before dawn, 1000mg in evening Correct answer
Learning Goals
Do not fast for health reasons Your answer
Question Filters

Security

Change Password Metformin is not associated with hypoglycaemia when used as monotherapy. It
Sign Out reduces hepatic glucose output and has its predominant effects on fasting plasma
glucose. Even so, it would seem sensible to divide the dose with the larger portion
taken in the evening after the fast is broken. Giving short acting insulin or
switching to insulin is likely to increase the risk of hypoglycaemia. Whilst diabetes
patients are exempted from the need to fast, if this patient wishes to do so in this
case it would seem unfair to prevent him fasting, as metformin should not
preclude it altogether.

20732

file:///E|/Shakhawan/Endocrinology/309a.htm[3/13/2012 7:16:19 PM]


MyPasTest

Main Navigation
Question Browser: MRCP 1
Home
Subscriptions Question Browser Exam Builder Saved Exams
» MRCP 1
• Question Browser
• Timed Test A 19-year-old female gymnast presents with complaints of headache and fatigue. She
• Mock Exam has had no significant previous medical history, but has been amenorrhoeic for the
past 4 months. On examination her BP is 110/70 mmHg and pulse is 55/min. Her BMI
• Past Papers is 16.
• Random Questions
Investigations
• My Performance
• Media Bank Hb 11.5 g/dl
• New Multimedia
WCC 5.2 x10 9 /l
Online Extras
Library PLT 156 x10 9 /l

Community Na + 140 mmol/l


Help K+ 5.0 mmol/l
PasTest Store
Creatinine 72 μmol/l

My Account The GP requests you to assess her hormone levels. Which of the following
hormone panel do you expect to be most elevated?
Profile
Newsletters Cortisol

My Career and Exams GH

Order History LH

Learning Goals Prolactin

Question Filters Thyroid

20733
Security

Change Password
Sign Out

file:///E|/Shakhawan/Endocrinology/310.htm[3/13/2012 7:16:21 PM]


MyPasTest

Main Navigation
Question Browser: MRCP 1
Home
Subscriptions Question Browser Exam Builder Saved Exams
» MRCP 1
• Question Browser
• Timed Test A 19-year-old female gymnast presents with complaints of headache and fatigue. She
• Mock Exam has had no significant previous medical history, but has been amenorrhoeic for the
past 4 months. On examination her BP is 110/70 mmHg and pulse is 55/min. Her BMI
• Past Papers is 16.
• Random Questions
Investigations
• My Performance
• Media Bank Hb 11.5 g/dl
• New Multimedia
WCC 5.2 x10 9 /l
Online Extras
Library PLT 156 x10 9 /l

Community Na + 140 mmol/l


Help K+ 5.0 mmol/l
PasTest Store
Creatinine 72 μmol/l

My Account The GP requests you to assess her hormone levels. Which of the following
hormone panel do you expect to be most elevated?
Profile
Newsletters Cortisol Your answer

My Career and Exams GH

Order History LH

Learning Goals Prolactin

Question Filters Thyroid

Security

Change Password This woman’s low BMI and amenorrhoea, raises the possibility that she is weight
Sign Out restricting, particularly given her profession as a gymnast. Resting growth
hormone levels are likely to be slightly elevated, LH levels will be low, and
prolactin levels are normal in this case. T3 and T4 are often low normal, with a
normal TSH. Cortisol levels are most elevated, and they often fail to suppress with
dexamethasone. Weight gain leads to a restoration of the normal menstrual cycle.

20733

file:///E|/Shakhawan/Endocrinology/310a.htm[3/13/2012 7:16:22 PM]


MyPasTest

Main Navigation
Question Browser: MRCP 1
Home
Subscriptions Question Browser Exam Builder Saved Exams
» MRCP 1
• Question Browser
• Timed Test A lady presents with amenorrhoea and galactorrhoea. She has normal visual fields.
• Mock Exam You are concerned that she may have a prolactinoma.
Investigations
• Past Papers
• Random Questions Hb 12.5 g/dl
• My Performance
WCC 4.9 x10 9 /l
• Media Bank
PLT 199 x10 9 /l
• New Multimedia
Online Extras Na + 140 mmol/l
Library K+ 4.9 mmol/l
Community Creatinine 90 μmol/l
Help Prolactin 1150 mU/l
PasTest Store
MRI 7 mm pituitary microadenoma

My Account Which of the following hormones would you expect to be low?

Profile ADH
Newsletters Cortisol
My Career and Exams GH
Order History Thyroxine
Learning Goals LH
Question Filters
20734

Security

Change Password
Sign Out

file:///E|/Shakhawan/Endocrinology/311.htm[3/13/2012 7:16:24 PM]


MyPasTest

Main Navigation
Question Browser: MRCP 1
Home
Subscriptions Question Browser Exam Builder Saved Exams
» MRCP 1 Session Progress
• Question Browser
Questions Correct 7
• Timed Test A lady presents with amenorrhoea and galactorrhoea. She
has normal visual fields. You are concerned that she may Questions Incorrect 4
• Mock Exam
have a prolactinoma. Questions Total 11
• Past Papers Investigations
Questions Percentage 63 %
• Random Questions
• My Performance Hb 12.5 g/dl More
• Media Bank 9
WCC 4.9 x10 /l
• New Multimedia
PLT 199 x10 9 /l
Online Extras
Na + 140 mmol/l
Library FusionCharts needs Adobe Flash Player
Community K+ 4.9 mmol/l to run. If you're unable to see the chart
here, it
Help Creatinine 90 μmol/l
PasTest Store Prolactin 1150 mU/l
MRI 7 mm pituitary microadenoma
My Account
Which of the following hormones would you expect to
Profile be low?
Newsletters
ADH
My Career and Exams
Cortisol
Order History
GH
Learning Goals
Thyroxine
Question Filters
LH Your answer

Security

Change Password
Sign Out You would not expect ADH levels to be affected by a
prolactinoma, and production of ACTH, TSH and growth
hormone should not be affected by a
microprolactinoma, as it is usually macroadenomas that
cause pressure effects, suppressing production of other
hormones. Hyperprolactinaemia suppresses release of
gonadotrophin releasing hormone, which leads to
reduced production of LH and FSH. There may also be a
direct effect of prolactin itself on the ovary to disrupt LH
and FSH signalling.

20734

file:///E|/Shakhawan/Endocrinology/311a.htm[3/13/2012 7:16:25 PM]


MyPasTest

Main Navigation
Question Browser: MRCP 1
Home
Subscriptions Question Browser Exam Builder Saved Exams
» MRCP 1
• Question Browser
• Timed Test A patient with poorly controlled Type 2 diabetes comes to the clinic for review. He has
• Mock Exam a history of hypertension for which he takes ramipril 10mg daily, and amlodipine
10mg. Current diabetes medication is metformin 1g BD and gliclazide 160mg BD. On
• Past Papers examination his BP is 145/85 mmHg. His vision is 6/6 bilaterally on clinical
• Random Questions examination.
• My Performance
Investigations
• Media Bank
• New Multimedia Hb 12.0 g/dl
Online Extras WCC 5.0 x10 9 /l
Library
PLT 231 x10 9 /l
Community
Help Na + 139 mmol/l

PasTest Store K+ 4.5 mmol/l

Creatinine 156 μmol/l


My Account HbA1c 7.8%
Profile
Fundoscopy – neovascularisation close to the optic disc.
Newsletters
My Career and Exams Which of the following is the most important step in clinical management?
Order History
Add pioglitazone 30mg to treatment
Learning Goals
Transition to insulin therapy
Question Filters
Referral for laser photocoagulation
Add indapamide to his regime
Security
Add doxazosin to his regime
Change Password
20735
Sign Out

file:///E|/Shakhawan/Endocrinology/312.htm[3/13/2012 7:16:27 PM]


MyPasTest

Main Navigation
Question Browser: MRCP 1
Home
Subscriptions Question Browser Exam Builder Saved Exams
» MRCP 1 Session Progress
• Question Browser
Questions Correct 8
• Timed Test A patient with poorly controlled Type 2 diabetes comes to
the clinic for review. He has a history of hypertension for Questions Incorrect 4
• Mock Exam
which he takes ramipril 10mg daily, and amlodipine 10mg. Questions Total 12
• Past Papers Current diabetes medication is metformin 1g BD and
Questions Percentage 66 %
• Random Questions gliclazide 160mg BD. On examination his BP is 145/85
mmHg. His vision is 6/6 bilaterally on clinical examination. More
• My Performance
• Media Bank Investigations
• New Multimedia
Hb 12.0 g/dl
Online Extras
Library WCC 5.0 x10 9 /l FusionCharts needs Adobe Flash Player
Community to run. If you're unable to see the chart
PLT 231 x10 9 /l
here, it means that your browser does
Help not seem to have the Flash Player
Na + 139 mmol/l
PasTest Store Installed. You can downloaded it here
K+ 4.5 mmol/l for free.

My Account Creatinine 156 μmol/l


HbA1c 7.8%
Profile
Newsletters Fundoscopy – neovascularisation close to the optic disc.
My Career and Exams
Which of the following is the most important step in
Order History
clinical management?
Learning Goals
Question Filters Add pioglitazone 30mg to treatment
Transition to insulin therapy

Security Referral for laser photocoagulation Your answer


Add indapamide to his regime
Change Password
Add doxazosin to his regime
Sign Out

Whilst tighter blood pressure and glucose control may


reduce the risk of further deterioration with respect to
microvascular complications, this patient is at imminent
risk of a retinal haemorrhage as a result of new vessel
formation. As such, laser photocoagulation, to reduce
the development of friable new vessels is clearly the
optimal intervention from those given above.

20735

file:///E|/Shakhawan/Endocrinology/312a.htm[3/13/2012 7:16:28 PM]


MyPasTest

Main Navigation
Question Browser: MRCP 1
Home
Subscriptions Question Browser Exam Builder Saved Exams
» MRCP 1 Session Progress
• Question Browser
Questions Correct 8
• Timed Test A 60-year-old man with a history of Type 2 diabetes and
left ventricular hypertrophy presents to the clinic. He is Questions Incorrect 4
• Mock Exam
currently taking metformin and pioglitazone for his glucose Questions Total 12
• Past Papers control. He is hypertensive with a BP of 160/95 mmHg, but
Questions Percentage 66 %
• Random Questions his GP reports he has suffered various side effects of anti-
hypertensives including a dry, irritating cough, swollen More
• My Performance
ankles, gum hypertrophy, lethargy and urinary frequency.
• Media Bank
• New Multimedia Investigations;

Online Extras Reference: Normal Values


Hb 12.9 g/dl
Library
WCC 5.4 x10 9 /l Click to
Community open/closeHaematology
Help PLT 199 x10 9 /l Click to
open/closeImmunoglobulins
PasTest Store Na + 141 mmol/l Click to
K+ 5.0 mmol/l open/closeBiochemistry
My Account Click to open/closeDiabetes
Creatinine 132 μmol/l Click to
Profile open/closeEndocrinology
HbA1c 8.2% Click to open/closeBlood
Newsletters gases Click to
My Career and Exams Which of the following would be the most appropriate open/closeCSF
therapy choice for him?
Order History
Learning Goals Indapamide
Question Filters Atenolol
Amlodipine
Security Irbesartan

Change Password Losartan

Sign Out 20956

© 2011 PasTest Ltd | About Us | Contact Us | Help

file:///E|/Shakhawan/Endocrinology/313.htm[3/13/2012 7:16:29 PM]


MyPasTest

Main Navigation
Question Browser: MRCP 1
Home
Subscriptions Question Browser Exam Builder Saved Exams
» MRCP 1 Session Progress
• Question Browser
Questions Correct 8
• Timed Test A 60-year-old man with a history of Type 2 diabetes and
left ventricular hypertrophy presents to the clinic. He is Questions Incorrect 5
• Mock Exam
currently taking metformin and pioglitazone for his glucose Questions Total 13
• Past Papers control. He is hypertensive with a BP of 160/95 mmHg, but
Questions Percentage 61 %
• Random Questions his GP reports he has suffered various side effects of anti-
hypertensives including a dry, irritating cough, swollen More
• My Performance
ankles, gum hypertrophy, lethargy and urinary frequency.
• Media Bank
• New Multimedia Investigations;

Online Extras
Hb 12.9 g/dl
Library
WCC 5.4 x10 9 /l
Community
Help PLT 199 x10 9 /l
PasTest Store Na + 141 mmol/l

K+ 5.0 mmol/l
My Account
Creatinine 132 μmol/l
Profile
HbA1c 8.2%
Newsletters
My Career and Exams Which of the following would be the most appropriate
therapy choice for him? Reference: Normal Values
Order History
Learning Goals Indapamide Click to open/closeHaematology
Question Filters Atenolol Click to open/closeImmunoglobulins
Click to open/closeBiochemistry
Amlodipine
Click to open/closeDiabetes Click to
Security Irbesartan Correct answer open/closeEndocrinology Click to
Losartan Your answer open/closeBlood gases Click to
Change Password open/closeCSF
Sign Out

Amlodipine is a calcium antagonist, and hence it may be


associated with gum hypertrophy and swollen ankles,
beta blockers are associated with lethargy and
indapamide may be associated with urinary frequency.
The two angiotensin receptor blockers would thus be
the most appropriate agents to choose from. Losartan
was the first ARB to be launched and has a relatively
flat dose response curve on increasing the dose from 50
to 100mg; hence irbesartan would be the more
appropriate choice here.

20956

© 2011 PasTest Ltd | About Us | Contact Us | Help

file:///E|/Shakhawan/Endocrinology/313a.htm[3/13/2012 7:16:31 PM]


MyPasTest

Main Navigation
Question Browser: MRCP 1
Home
Subscriptions Question Browser Exam Builder Saved Exams
» MRCP 1
• Question Browser
• Timed Test A 56-year-old man with a history of Type 2 diabetes comes to the clinic complaining
• Mock Exam of increased shortness of breath and bilateral ankle swelling. An echocardiogram
arranged by the GP reveals that he has an ejection fraction of 35%. His diabetes is
• Past Papers relatively well controlled, with a recent HbA1c measured at 7.2%. He takes ramipril
• Random Questions and furosemide currently for control of his hypertension and heart failure.
• My Performance
Investigations;
• Media Bank
• New Multimedia Hb 12.3 g/dl
Online Extras WCC 5.0 x10 9 /l
Library
PLT 190 x10 9 /l
Community
Help Na + 141 mmol/l

PasTest Store K+ 4.9 mmol/l


Creatinine 110 µmol/l
My Account HbA1c 7.2%

Profile CXR LVH, increased basal shadowing consistent with mild LVF
Newsletters
Which of the following glucose control strategies would be most suitable in
My Career and Exams this patient with respect to avoiding fluid overload?
Order History
Pioglitazone
Learning Goals
Basal insulin
Question Filters
Gliclazide
Chlorpropamide
Security
BD mixed insulin
Change Password
Sign Out 20957

file:///E|/Shakhawan/Endocrinology/314.htm[3/13/2012 7:16:33 PM]


MyPasTest

Main Navigation
Question Browser: MRCP 1
Home
Subscriptions Question Browser Exam Builder Saved Exams
» MRCP 1 Session Progress
• Question Browser
Questions Correct 9
• Timed Test A 56-year-old man with a history of Type 2 diabetes comes
to the clinic complaining of increased shortness of breath Questions Incorrect 5
• Mock Exam
and bilateral ankle swelling. An echocardiogram arranged Questions Total 14
• Past Papers by the GP reveals that he has an ejection fraction of 35%.
Questions Percentage 64 %
• Random Questions His diabetes is relatively well controlled, with a recent
HbA1c measured at 7.2%. He takes ramipril and More
• My Performance
furosemide currently for control of his hypertension and
• Media Bank heart failure.
• New Multimedia
Investigations;
Online Extras
Library Hb 12.3 g/dl
Community
WCC 5.0 x10 9 /l
Help
PLT 190 x10 9 /l
PasTest Store
Na + 141 mmol/l

My Account K+ 4.9 mmol/l

Profile Creatinine 110 µmol/l


Newsletters HbA1c 7.2%
My Career and Exams LVH, increased basal shadowing consistent
CXR with mild LVF Reference: Normal Values
Order History
Learning Goals Which of the following glucose control strategies Click to open/closeHaematology
Question Filters would be most suitable in this patient with respect to Click to open/closeImmunoglobulins
avoiding fluid overload? Click to open/closeBiochemistry
Click to open/closeDiabetes Click to
Security Pioglitazone open/closeEndocrinology Click to
Basal insulin open/closeBlood gases Click to
Change Password open/closeCSF
Sign Out Gliclazide Your answer
Chlorpropamide
BD mixed insulin

Pioglitazone is associated with fluid retention and


subsequent heart failure, and should be avoided in this
population. Whilst not associated with as great a
propensity to cause heart failure, insulin itself leads to
increased sympathetic drive, sodium and water
retention and heart failure. Out of the sulphonylureas,
chlorpropamide is both long acting, and associated with
more sodium and water retention than other members
of the SU family, hence gliclazide is the obvious choice
here.

20957

© 2011 PasTest Ltd | About Us | Contact Us | Help

file:///E|/Shakhawan/Endocrinology/314a.htm[3/13/2012 7:16:34 PM]


MyPasTest

Main Navigation
Question Browser: MRCP 1
Home
Subscriptions Question Browser Exam Builder Saved Exams
» MRCP 1 Session Progress
• Question Browser
Questions Correct 9
• Timed Test A 61-year-old man with Type 2 diabetes comes to the clinic
with a foot ulcer on the plantar surface of his left foot. He Questions Incorrect 5
• Mock Exam
has had Type 2 diabetes for the past 8 years and is Questions Total 14
• Past Papers currently managed with metformin and sulphonylurea. On
Questions Percentage 64 %
• Random Questions examination his BP is 160/92 mmHg, his pulse 75/min and
regular. He is obese with a BMI of 32. He has loss of More
• My Performance
vibration sense on his big toes and insensitivity to the 10g
• Media Bank monofilament on the soles of both feet. He has loss of the
• New Multimedia arches on both sides.

Online Extras Investigations; Reference: Normal Values


Library
Hb 12.5 g/dl Click to
Community open/closeHaematology
Help WCC 4.9 x10 9 /l Click to
open/closeImmunoglobulins
PasTest Store PLT 180 x10 9 /l Click to
open/closeBiochemistry
Na + 141 mmol/l Click to open/closeDiabetes
My Account
K+ 4.9 mmol/l Click to
Profile open/closeEndocrinology
Creatinine 110 μmol/l Click to open/closeBlood
Newsletters gases Click to
My Career and Exams HbA1c 7.8% open/closeCSF
Order History
Which of the following is the best predictor for his
Learning Goals future risk of ulceration?
Question Filters
Loss of vibration sense
Loss of 10g monofilament sensation
Security
Previous / present ulcer
Change Password Foot deformity
Sign Out His increased BMI

20958

© 2011 PasTest Ltd | About Us | Contact Us | Help

file:///E|/Shakhawan/Endocrinology/315.htm[3/13/2012 7:16:36 PM]


MyPasTest

Main Navigation
Question Browser: MRCP 1
Home
Subscriptions Question Browser Exam Builder Saved Exams
» MRCP 1 Session Progress
• Question Browser
Questions Correct 10
• Timed Test A 61-year-old man with Type 2 diabetes comes to the clinic
with a foot ulcer on the plantar surface of his left foot. He Questions Incorrect 5
• Mock Exam
has had Type 2 diabetes for the past 8 years and is Questions Total 15
• Past Papers currently managed with metformin and sulphonylurea. On
Questions Percentage 66 %
• Random Questions examination his BP is 160/92 mmHg, his pulse 75/min and
regular. He is obese with a BMI of 32. He has loss of More
• My Performance
vibration sense on his big toes and insensitivity to the 10g
• Media Bank monofilament on the soles of both feet. He has loss of the
• New Multimedia arches on both sides.

Online Extras Investigations;


Library
Community Hb 12.5 g/dl

Help WCC 4.9 x10 9 /l


PasTest Store PLT 180 x10 9 /l

Na + 141 mmol/l
My Account
K+ 4.9 mmol/l
Profile
Creatinine 110 μmol/l
Newsletters
My Career and Exams HbA1c 7.8%

Order History Reference: Normal Values


Which of the following is the best predictor for his
Learning Goals future risk of ulceration?
Click to open/closeHaematology
Question Filters Click to open/closeImmunoglobulins
Loss of vibration sense
Click to open/closeBiochemistry
Loss of 10g monofilament sensation Click to open/closeDiabetes Click to
Security open/closeEndocrinology Click to
Previous / present ulcer Your answer
open/closeBlood gases Click to
Change Password Foot deformity open/closeCSF
Sign Out His increased BMI

The North-West Diabetes foot care study confirmed that


neuropathy disability score, loss of 10g monofilament
sensation and foot deformity are all associated with
increased risk of plantar ulceration, but the strongest
predictive factor with respect to future risk of ulceration
is either active ulceration or a history of a previous
ulcer. Bearing in mind that 2% of patients with diabetes
in the community develop new foot ulcers each year, it
is clearly very important to closely follow up those
patients with documented neuropathy.

20958

© 2011 PasTest Ltd | About Us | Contact Us | Help

file:///E|/Shakhawan/Endocrinology/315a.htm[3/13/2012 7:16:37 PM]


MyPasTest

Main Navigation
Question Browser: MRCP 1
Home
Subscriptions Question Browser Exam Builder Saved Exams
» MRCP 1 Session Progress
• Question Browser
Questions Correct 10
• Timed Test A 42-year-old man who works as a security guard and
suffers from morbid obesity (BMI 34) attends the clinic Questions Incorrect 5
• Mock Exam
because his GP has documented fasting blood glucose of Questions Total 15
• Past Papers 8.2 mmol/l, and he has symptoms of frequent nocturia and
Questions Percentage 66 %
• Random Questions lethargy. He has a history of hypertension which is
managed with ramipril 10mg daily but is currently taking no More
• My Performance
other medication.
• Media Bank
• New Multimedia Investigations;

Online Extras Reference: Normal Values


Hb 12.1 g/dl
Library
WCC 6.9 x10 9 /l Click to
Community open/closeHaematology
Help PLT 309 x10 9 /l Click to
open/closeImmunoglobulins
PasTest Store Na + 139 mmol/l Click to
K+ 4.4 mmol/l open/closeBiochemistry
My Account Click to open/closeDiabetes
Creatinine 110 µmol/l Click to
Profile open/closeEndocrinology
Which of the following would be the most appropriate Click to open/closeBlood
Newsletters gases Click to
initial medication for him?
My Career and Exams open/closeCSF
Order History Pioglitazone
Learning Goals Sitagliptin
Question Filters Gliclazide
Basal bolus insulin
Security Metformin

Change Password 20959


Sign Out

© 2011 PasTest Ltd | About Us | Contact Us | Help

file:///E|/Shakhawan/Endocrinology/316.htm[3/13/2012 7:16:39 PM]


MyPasTest

Main Navigation
Question Browser: MRCP 1
Home
Subscriptions Question Browser Exam Builder Saved Exams
» MRCP 1 Session Progress
• Question Browser
Questions Correct 11
• Timed Test A 42-year-old man who works as a security guard and
suffers from morbid obesity (BMI 34) attends the clinic Questions Incorrect 5
• Mock Exam
because his GP has documented fasting blood glucose of Questions Total 16
• Past Papers 8.2 mmol/l, and he has symptoms of frequent nocturia and
Questions Percentage 68 %
• Random Questions lethargy. He has a history of hypertension which is
managed with ramipril 10mg daily but is currently taking no More
• My Performance
other medication.
• Media Bank
• New Multimedia Investigations;

Online Extras
Hb 12.1 g/dl
Library
WCC 6.9 x10 9 /l
Community
Help PLT 309 x10 9 /l
PasTest Store Na + 139 mmol/l

K+ 4.4 mmol/l
My Account
Creatinine 110 µmol/l
Profile
Newsletters Which of the following would be the most appropriate
initial medication for him?
My Career and Exams
Order History Pioglitazone Reference: Normal Values
Learning Goals Sitagliptin
Click to open/closeHaematology
Question Filters Gliclazide Click to open/closeImmunoglobulins
Basal bolus insulin Click to open/closeBiochemistry
Click to open/closeDiabetes Click to
Security Metformin Your answer
open/closeEndocrinology Click to
open/closeBlood gases Click to
Change Password open/closeCSF
Sign Out

Pioglitazone, whilst effective in controlling this man’s


blood sugar, promotes fat differentiation and is likely to
promote progressive weight gain as the consequence of
hitting the glucose target. Gliclazide and insulin will both
promote weight gain, and given that he works as a
security guard would not be the first option due to risk
of hypoglycaemia. On the other hand, metformin, one
of the primary actions of which is through AMP kinase
activity, reduces hepatic glucose output and doesn’t
increase hyperinsulinaemia. For this reason it achieves
glucose control but does not promote hypoglycaemia or
weight gain.

20959

© 2011 PasTest Ltd | About Us | Contact Us | Help

file:///E|/Shakhawan/Endocrinology/316a.htm[3/13/2012 7:16:40 PM]


MyPasTest

Main Navigation
Question Browser: MRCP 1
Home
Subscriptions Question Browser Exam Builder Saved Exams
» MRCP 1 Session Progress
• Question Browser
Questions Correct 11
• Timed Test You are drawing up guidelines for the management of oral
blood glucose lowering agents in patients post myocardial Questions Incorrect 5
• Mock Exam
infarction. Questions Total 16
• Past Papers
Looking at a synthesis of the available evidence, Questions Percentage 68 %
• Random Questions
which of the following pieces of advice would you More
• My Performance give?
• Media Bank
• New Multimedia Pioglitazone should be started in patients post
myocardial infarction
Online Extras Reference: Normal Values
Metformin should be stopped in all patients post
Library myocardial infarction
Click to
Community All patients should be transitioned to permanent open/closeHaematology
Help insulin therapy Click to
Metformin should be stopped in patients in those open/closeImmunoglobulins
PasTest Store
patients who have an unstable circulation post infarct Click to
open/closeBiochemistry
Insulin should be given IV for the first 48hrs Click to open/closeDiabetes
My Account
Click to
Profile 20960 open/closeEndocrinology
Click to open/closeBlood
Newsletters gases Click to
My Career and Exams open/closeCSF
Order History
Learning Goals
Question Filters

Security

Change Password
Sign Out

© 2011 PasTest Ltd | About Us | Contact Us | Help

file:///E|/Shakhawan/Endocrinology/317.htm[3/13/2012 7:16:42 PM]


MyPasTest

Main Navigation
Question Browser: MRCP 1
Home
Subscriptions Question Browser Exam Builder Saved Exams
» MRCP 1 Session Progress
• Question Browser
Questions Correct 11
• Timed Test You are drawing up guidelines for the management of oral
blood glucose lowering agents in patients post myocardial Questions Incorrect 6
• Mock Exam
infarction. Questions Total 17
• Past Papers
Looking at a synthesis of the available evidence, Questions Percentage 64 %
• Random Questions
which of the following pieces of advice would you More
• My Performance give?
• Media Bank
• New Multimedia Pioglitazone should be started in
patients post myocardial infarction
Online Extras
Metformin should be stopped in all Your answer
Library patients post myocardial infarction
Community All patients should be transitioned to
Help permanent insulin therapy
PasTest Store Metformin should be stopped in Correct answer
patients in those patients who have
an unstable circulation post infarct
My Account Insulin should be given IV for the
first 48hrs
Profile
Newsletters
My Career and Exams
Order History Reference: Normal Values
Glitazones are generally not used in patients at risk of
Learning Goals fluid retention due to the risk of heart failure, which Click to open/closeHaematology
Question Filters includes some of those patients who have suffered a Click to open/closeImmunoglobulins
myocardial infarction. The DIGAMI-2 study was Click to open/closeBiochemistry
generally disappointing with respect to demonstrating Click to open/closeDiabetes Click to
Security any benefits of insulin therapy, and for this reason open/closeEndocrinology Click to
permanent insulin therapy is no longer recommended open/closeBlood gases Click to
Change Password post infarct if pre-MI glycaemic control was adequate. open/closeCSF
Metformin was originally ruled out in all patients who
Sign Out
suffered an MI, but is continued by many clinicians now
unless there is evidence of significant circulatory
impairment or raised creatinine.

20960

© 2011 PasTest Ltd | About Us | Contact Us | Help

file:///E|/Shakhawan/Endocrinology/317a.htm[3/13/2012 7:16:43 PM]


MyPasTest

Main Navigation
Question Browser: MRCP 1
Home
Subscriptions Question Browser Exam Builder Saved Exams
» MRCP 1 Session Progress
• Question Browser
Questions Correct 11
• Timed Test A 45-year-old man presents to the clinic after being
referred by his GP. He complains of recurrent episodes of Questions Incorrect 6
• Mock Exam
sweating and light headedness which are relieved by eating Questions Total 17
• Past Papers snacks. Tiredness and poor concentration have affected his
Questions Percentage 64 %
• Random Questions ability to work and he has gained 12kg in weight over the
past 3 months. On examination he has a BMI of 31, his BP More
• My Performance
is 145/89 mmHg, pulse 75/min regular. He is obese but
• Media Bank there are no other clinical findings of note.
• New Multimedia
Investigations;
Online Extras Reference: Normal Values
Library Hb 13.9 g/dl
Click to
Community
WCC 5.8 x10 9 /l open/closeHaematology
Help Click to
PLT 204 x10 9 /l open/closeImmunoglobulins
PasTest Store
Click to
Na + 141 mmol/l open/closeBiochemistry
My Account K+ 4.9 mmol/l Click to open/closeDiabetes
Click to
Profile Creatinine 102 μmol/l open/closeEndocrinology
Click to open/closeBlood
Newsletters Fasting glucose 4.1 mmol/l gases Click to
My Career and Exams open/closeCSF
Which of the following is the most appropriate next
Order History
investigation?
Learning Goals
Question Filters TSH
Barium swallow

Security In-patient 72hr fast


3x 15hr fasts with outpatient glucose estimation
Change Password
Food diary
Sign Out
20961

© 2011 PasTest Ltd | About Us | Contact Us | Help

file:///E|/Shakhawan/Endocrinology/318.htm[3/13/2012 7:16:45 PM]


MyPasTest

Main Navigation
Question Browser: MRCP 1
Home
Subscriptions Question Browser Exam Builder Saved Exams
» MRCP 1 Session Progress
• Question Browser
Questions Correct 12
• Timed Test A 45-year-old man presents to the clinic after being
referred by his GP. He complains of recurrent episodes of Questions Incorrect 6
• Mock Exam
sweating and light headedness which are relieved by eating Questions Total 18
• Past Papers snacks. Tiredness and poor concentration have affected his
Questions Percentage 66 %
• Random Questions ability to work and he has gained 12kg in weight over the
past 3 months. On examination he has a BMI of 31, his BP More
• My Performance
is 145/89 mmHg, pulse 75/min regular. He is obese but
• Media Bank there are no other clinical findings of note.
• New Multimedia
Investigations;
Online Extras
Library Hb 13.9 g/dl
Community
WCC 5.8 x10 9 /l
Help
PLT 204 x10 9 /l
PasTest Store
Na + 141 mmol/l

My Account K+ 4.9 mmol/l

Profile Creatinine 102 μmol/l


Newsletters Fasting glucose 4.1 mmol/l
My Career and Exams
Which of the following is the most appropriate next Reference: Normal Values
Order History
investigation?
Learning Goals
Click to open/closeHaematology
Question Filters TSH Click to open/closeImmunoglobulins
Barium swallow Click to open/closeBiochemistry
Click to open/closeDiabetes Click to
Security In-patient 72hr fast
open/closeEndocrinology Click to
3x 15hr fasts with outpatient glucose Your answer open/closeBlood gases Click to
Change Password estimation open/closeCSF
Sign Out Food diary

The episodes of sweating, tiredness, light headedness


and loss of concentration raise the possibility of
recurrent hypoglycaemia. Around 30% of patients with
insulinoma also suffer from rapid weight gain, as has
been seen here. The most appropriate initial
investigation is 3 glucose measurements after 15hr
fasts, if these are negative but clinical suspicion is high,
hypoglycaemia may be precipitated either by a period of
exercise post fasting, or by a longer in patient fast of
72hrs. Estimation of insulin and C peptide during
symptoms of hypoglycaemia will prove diagnostic.
Whilst hypothyroidism may cause weight gain and
tiredness, the symptoms of hypoglycaemia point more
towards ruling out hypoglycaemia initially. Gastric
dumping is a possibility, but he reports no GI symptoms
going along with rapid transit which might guide us to
this as a diagnosis.

20961

file:///E|/Shakhawan/Endocrinology/318a.htm[3/13/2012 7:16:46 PM]


MyPasTest

Main Navigation
Question Browser: MRCP 1
Home
Subscriptions Question Browser Exam Builder Saved Exams
» MRCP 1
• Question Browser
• Timed Test A 35-year-old man comes to the clinic complaining of tiredness, lethargy and an
• Mock Exam increasingly hoarse voice. He also tells you that he has been losing his hair over the
past few months and has noticed fullness in his neck; he has gained 7kg in the past 3
• Past Papers months. On examination he has a puffy face and periorbital oedema and there is a
• Random Questions firm, rubbery goitre. His pulse is 48/min and regular, his BP 142/73 mmHg.
• My Performance
Investigations;
• Media Bank
• New Multimedia Hb 11.4 g/dl
Online Extras WCC 5.4 x10 9 /l
Library
PLT 183 x10 9 /l
Community
Help Na + 141 mmol/l

PasTest Store K+ 4.7 mmol/l

Creatinine 120 μmol/l


My Account TSH 10.1 U/l
Profile Total cholesterol 8.2 mmol/l
Newsletters Triglycerides 3.2 mmol/l
My Career and Exams LDL 3.9 mmol/l
Order History FNA thyroid Diffuse lymphocytic and plasma cell infiltration
Learning Goals
Question Filters Which of the following is the most likely diagnosis?

De Quervain’s thyroiditis
Security Atrophic hypothyroidism
Change Password Hashimoto’s thyroiditis
Sign Out Grave’s disease
Thyroid lymphoma

21072

file:///E|/Shakhawan/Endocrinology/319.htm[3/13/2012 7:16:48 PM]


MyPasTest

Main Navigation
Question Browser: MRCP 1
Home
Subscriptions Question Browser Exam Builder Saved Exams
» MRCP 1 Session Progress
• Question Browser
Questions Correct 13
• Timed Test A 35-year-old man comes to the clinic complaining of
tiredness, lethargy and an increasingly hoarse voice. He Questions Incorrect 6
• Mock Exam
also tells you that he has been losing his hair over the past Questions Total 19
• Past Papers few months and has noticed fullness in his neck; he has
Questions Percentage 68 %
• Random Questions gained 7kg in the past 3 months. On examination he has a
puffy face and periorbital oedema and there is a firm, More
• My Performance
rubbery goitre. His pulse is 48/min and regular, his BP
• Media Bank 142/73 mmHg.
• New Multimedia
Investigations;
Online Extras
Library Hb 11.4 g/dl
Community
WCC 5.4 x10 9 /l
Help
PLT 183 x10 9 /l
PasTest Store
Na + 141 mmol/l

My Account K+ 4.7 mmol/l

Profile Creatinine 120 μmol/l


Newsletters TSH 10.1 U/l
My Career and Exams Total
Order History cholesterol 8.2 mmol/l Reference: Normal Values
Learning Goals Triglycerides 3.2 mmol/l
Click to open/closeHaematology
Question Filters LDL 3.9 mmol/l Click to open/closeImmunoglobulins
Diffuse lymphocytic and plasma cell Click to open/closeBiochemistry
FNA thyroid infiltration Click to open/closeDiabetes Click to
Security open/closeEndocrinology Click to
open/closeBlood gases Click to
Change Password Which of the following is the most likely diagnosis?
open/closeCSF
Sign Out
De Quervain’s thyroiditis
Atrophic hypothyroidism
Hashimoto’s thyroiditis Your answer
Grave’s disease
Thyroid lymphoma

The features seen here are typical of Hashimoto’s


thyroiditis. Treatment is of course with T4 replacement;
most physicians tailor the dose of T4 to keep the TSH in
the lower half of the normal reference range. Where
there is a history of cardiovascular disease, it is
recommended to slowly up-titrate the dose of T4,
although in a patient of this age, it should be possible
to begin at a full replacement dose. Over-replacement
can lead to accelerated bone loss and osteoporosis,
particularly in women, for this reason regular monitoring
of TSH is recommended.

21072

file:///E|/Shakhawan/Endocrinology/319a.htm[3/13/2012 7:16:49 PM]


MyPasTest

Main Navigation
Question Browser: MRCP 1
Home
Subscriptions Question Browser Exam Builder Saved Exams
» MRCP 1 Session Progress
• Question Browser
Questions Correct 13
• Timed Test A 27-year-old pharmacist is admitted to the Emergency
room with a panic attack. On further questioning it Questions Incorrect 6
• Mock Exam
transpires that she has been suffering from palpitations and Questions Total 19
• Past Papers has lost weight over the past 6 months. Her periods
Questions Percentage 68 %
• Random Questions stopped 3 months ago. On examination her BP is 145/90
mmHg, pulse is 92/min at rest. Her BMI is 20; the More
• My Performance
remainder of her clinical examination is unremarkable.
• Media Bank
• New Multimedia Investigations;

Online Extras Hb 13.1 g/dl Reference: Normal Values


Library
WCC 4.9 x10 9 /l Click to
Community open/closeHaematology
Help PLT 145 x10 9 /l Click to
TSH <0.05 U/l open/closeImmunoglobulins
PasTest Store
Click to
Free T4 32 pmol/l open/closeBiochemistry
My Account Thyroglobulin low Click to open/closeDiabetes
Click to
Scintography scan decreased thyroid uptake open/closeEndocrinology
Profile
Click to open/closeBlood
Newsletters Which of the following is the most likely diagnosis? gases Click to
My Career and Exams open/closeCSF
Order History Hashimoto’s thyroiditis

Learning Goals Graves'disease

Question Filters Toxic multinodular goitre


Thyrotoxicosis factitia
Struma ovarii
Security

Change Password 21253


Sign Out

© 2011 PasTest Ltd | About Us | Contact Us | Help

file:///E|/Shakhawan/Endocrinology/320.htm[3/13/2012 7:16:51 PM]


MyPasTest

Main Navigation
Question Browser: MRCP 1
Home
Subscriptions Question Browser Exam Builder Saved Exams
» MRCP 1 Session Progress
• Question Browser
Questions Correct 14
• Timed Test A 27-year-old pharmacist is admitted to the Emergency
room with a panic attack. On further questioning it Questions Incorrect 6
• Mock Exam
transpires that she has been suffering from palpitations and Questions Total 20
• Past Papers has lost weight over the past 6 months. Her periods
Questions Percentage 70 %
• Random Questions stopped 3 months ago. On examination her BP is 145/90
mmHg, pulse is 92/min at rest. Her BMI is 20; the More
• My Performance
remainder of her clinical examination is unremarkable.
• Media Bank
• New Multimedia Investigations;

Online Extras Hb 13.1 g/dl


Library
WCC 4.9 x10 9 /l
Community
Help PLT 145 x10 9 /l

PasTest Store TSH <0.05 U/l


Free T4 32 pmol/l
My Account Thyroglobulin low
Scintography scan decreased thyroid uptake
Profile
Newsletters Which of the following is the most likely diagnosis?
My Career and Exams
Order History Hashimoto’s thyroiditis Reference: Normal Values
Learning Goals Graves'disease
Click to open/closeHaematology
Question Filters Toxic multinodular goitre
Click to open/closeImmunoglobulins
Thyrotoxicosis factitia Your answer Click to open/closeBiochemistry
Struma ovarii Click to open/closeDiabetes Click to
Security open/closeEndocrinology Click to
open/closeBlood gases Click to
Change Password open/closeCSF
Sign Out
The combination of low thyroglobulin, decreased uptake
on scintography and raised T4 can only really be
thyrotoxicosis factitia. Given the fact that this patient is
a pharmacist she is likely to have access to thyroid
hormone. Management is likely to be challenging, with
appropriate psychological support and counselling the
mainstay of therapy.

21253

© 2011 PasTest Ltd | About Us | Contact Us | Help

file:///E|/Shakhawan/Endocrinology/320a.htm[3/13/2012 7:16:52 PM]


MyPasTest

Main Navigation
Question Browser: MRCP 1
Home
Subscriptions Question Browser Exam Builder Saved Exams
» MRCP 1 Session Progress
• Question Browser
Questions Correct 14
• Timed Test A 19-year-old girl with a history of Type 1 diabetes
presents with confusion, hyperventilation and dehydration. Questions Incorrect 6
• Mock Exam
On examination she is hyperventilating, smells of pear Questions Total 20
• Past Papers drops and has a BP of 95/50 mmHg with a pulse of
Questions Percentage 70 %
• Random Questions 105/min. Arterial blood gas measurement reveals a pH of
7.2. You suspect that she has diabetic ketoacidosis (DKA). More
• My Performance
• Media Bank What is the primary cause of ketoacidosis in Type 1
diabetes?
• New Multimedia
Online Extras Lipogenesis Reference: Normal Values
Library Lipolysis
Click to
Community Gluconeogenesis open/closeHaematology
Help Glycolysis Click to
open/closeImmunoglobulins
PasTest Store Glycogenolysis Click to
open/closeBiochemistry
21354 Click to open/closeDiabetes
My Account
Click to
Profile open/closeEndocrinology
Click to open/closeBlood
Newsletters gases Click to
My Career and Exams open/closeCSF
Order History
Learning Goals
Question Filters

Security

Change Password
Sign Out

© 2011 PasTest Ltd | About Us | Contact Us | Help

file:///E|/Shakhawan/Endocrinology/321.htm[3/13/2012 7:16:54 PM]


MyPasTest

Main Navigation
Question Browser: MRCP 1
Home
Subscriptions Question Browser Exam Builder Saved Exams
» MRCP 1 Session Progress
• Question Browser
Questions Correct 15
• Timed Test A 19-year-old girl with a history of Type 1 diabetes
presents with confusion, hyperventilation and dehydration. Questions Incorrect 6
• Mock Exam
On examination she is hyperventilating, smells of pear Questions Total 21
• Past Papers drops and has a BP of 95/50 mmHg with a pulse of
Questions Percentage 71 %
• Random Questions 105/min. Arterial blood gas measurement reveals a pH of
7.2. You suspect that she has diabetic ketoacidosis (DKA). More
• My Performance
• Media Bank What is the primary cause of ketoacidosis in Type 1
diabetes?
• New Multimedia
Online Extras Lipogenesis
Library Lipolysis Your answer
Community Gluconeogenesis
Help Glycolysis
PasTest Store Glycogenolysis

My Account

Profile Glycogenolysis and gluconeogenesis lead to severe


hyperglycaemia, which accentuates dehydration and
Newsletters
hypotension. Lipolysis increases the availability of free
My Career and Exams fatty acid substrate for hepatic metabolism which leads
Order History to accumulation of acid intermediate and end products Reference: Normal Values
(ketoacids and ketones such as acetoacetate, beta-
Learning Goals hydroxybutyrate and acetone). Ketones are also seen in Click to open/closeHaematology
Question Filters some situations of acute energy deficit. Click to open/closeImmunoglobulins
Click to open/closeBiochemistry
21354 Click to open/closeDiabetes Click to
Security open/closeEndocrinology Click to
open/closeBlood gases Click to
Change Password open/closeCSF
Sign Out

© 2011 PasTest Ltd | About Us | Contact Us | Help

file:///E|/Shakhawan/Endocrinology/321a.htm[3/13/2012 7:16:55 PM]


MyPasTest

Main Navigation
Question Browser: MRCP 1
Home
Subscriptions Question Browser Exam Builder Saved Exams
» MRCP 1 Session Progress
• Question Browser
Questions Correct 15
• Timed Test A 32-year-old woman presents to the clinic with tiredness.
She has 3 children and a full time job and is finding it very Questions Incorrect 6
• Mock Exam
difficult to hold everything together. There is no significant Questions Total 21
• Past Papers past medical history. On examination her BP is 145/80
Questions Percentage 71 %
• Random Questions mmHg, her BMI is 28.
• My Performance More
Investigations;
• Media Bank
• New Multimedia Hb 12.4 g/dl

Online Extras WCC 6.7 x10 9 /l Reference: Normal Values


Library PLT 204 x10 9 /l
Click to
Community open/closeHaematology
Na + 141 mmol/l
Help Click to
PasTest Store K+ 4.9 mmol/l open/closeImmunoglobulins
Click to
Creatinine 120 µmol/l open/closeBiochemistry
My Account Total cholesterol 5.0 mmol/l Click to open/closeDiabetes
Click to
TSH 7.8 U/l open/closeEndocrinology
Profile
Free T4 10.0 pmol/l (10-22) Click to open/closeBlood
Newsletters gases Click to
My Career and Exams Free T3 4.9 pmol/l (5-10) open/closeCSF
Order History
Which of the following is the most likely diagnosis?
Learning Goals
Question Filters Hypothyroidism
Thyrotoxicosis

Security Thyroid hormone resistance


Subclinical hypothyroidism
Change Password
TSH secreting tumour
Sign Out
21355

© 2011 PasTest Ltd | About Us | Contact Us | Help

file:///E|/Shakhawan/Endocrinology/322.htm[3/13/2012 7:16:57 PM]


MyPasTest

Main Navigation
Question Browser: MRCP 1
Home
Subscriptions Question Browser Exam Builder Saved Exams
» MRCP 1 Session Progress
• Question Browser
Questions Correct 15
• Timed Test A 32-year-old woman presents to the clinic with tiredness.
She has 3 children and a full time job and is finding it very Questions Incorrect 7
• Mock Exam
difficult to hold everything together. There is no significant Questions Total 22
• Past Papers past medical history. On examination her BP is 145/80
Questions Percentage 68 %
• Random Questions mmHg, her BMI is 28.
• My Performance More
Investigations;
• Media Bank
• New Multimedia Hb 12.4 g/dl

Online Extras WCC 6.7 x10 9 /l


Library PLT 204 x10 9 /l
Community
Na + 141 mmol/l
Help
PasTest Store K+ 4.9 mmol/l
Creatinine 120 µmol/l

My Account Total cholesterol 5.0 mmol/l


TSH 7.8 U/l
Profile
Free T4 10.0 pmol/l (10-22)
Newsletters
My Career and Exams Free T3 4.9 pmol/l (5-10)

Order History Reference: Normal Values


Which of the following is the most likely diagnosis?
Learning Goals
Click to open/closeHaematology
Question Filters Hypothyroidism Your answer
Click to open/closeImmunoglobulins
Thyrotoxicosis Click to open/closeBiochemistry
Thyroid hormone resistance Click to open/closeDiabetes Click to
Security open/closeEndocrinology Click to
Subclinical hypothyroidism Correct answer open/closeBlood gases Click to
Change Password open/closeCSF
TSH secreting tumour
Sign Out

The TSH here is outside the upper limit of normal, and


the T3 and T4 levels are right at the lower end of the
normal range. The suspicion is that her pituitary is
having to work extra hard to drive a failing thyroid, so
called subclinical hypothyroidism. Treatment with T4
replacement is controversial, with some clinicians
treating based on symptom scoring, presence of
autoantibodies /other autoimmune pathology, although
no randomised controlled trials exist to support this
approach.

21355

© 2011 PasTest Ltd | About Us | Contact Us | Help

file:///E|/Shakhawan/Endocrinology/322a.htm[3/13/2012 7:16:58 PM]


MyPasTest

Main Navigation
Question Browser: MRCP 1
Home
Subscriptions Question Browser Exam Builder Saved Exams
» MRCP 1 Session Progress
• Question Browser
Questions Correct 15
• Timed Test A 23-year-old South African woman who has recently
started the oral contraceptive pill comes to the dermatology Questions Incorrect 7
• Mock Exam
clinic. She is concerned as the skin on her hands and Questions Total 22
• Past Papers forearms has become increasingly fragile with a bullous
Questions Percentage 68 %
• Random Questions rash. In addition she has increased pigmentation and some
hair growth on her face. More
• My Performance
• Media Bank Investigations;
• New Multimedia
Hb 12.2 g/dl
Online Extras Reference: Normal Values
Library WCC 8.1 x10 9 /l
Click to
Community PLT 284 x10 9 /l
open/closeHaematology
Help Na + 141 mmol/l Click to
open/closeImmunoglobulins
PasTest Store
K+ 4.9 mmol/l Click to
open/closeBiochemistry
Creatinine 110 µmol/l Click to open/closeDiabetes
My Account
ANA positive Click to
Profile open/closeEndocrinology
Which of the following is the most likely diagnosis? Click to open/closeBlood
Newsletters gases Click to
My Career and Exams open/closeCSF
Polycystic ovarian syndrome
Order History
Erythema multiforme
Learning Goals
Hereditary coproporphyria
Question Filters
Porphyria cutanea tarda
SLE
Security
21356
Change Password
Sign Out

© 2011 PasTest Ltd | About Us | Contact Us | Help

file:///E|/Shakhawan/Endocrinology/323.htm[3/13/2012 7:17:00 PM]


MyPasTest

Main Navigation
Question Browser: MRCP 1
Home
Subscriptions Question Browser Exam Builder Saved Exams
» MRCP 1 Session Progress
• Question Browser
Questions Correct 16
• Timed Test A 23-year-old South African woman who has recently
started the oral contraceptive pill comes to the dermatology Questions Incorrect 7
• Mock Exam
clinic. She is concerned as the skin on her hands and Questions Total 23
• Past Papers forearms has become increasingly fragile with a bullous
Questions Percentage 69 %
• Random Questions rash. In addition she has increased pigmentation and some
hair growth on her face. More
• My Performance
• Media Bank Investigations;
• New Multimedia
Hb 12.2 g/dl
Online Extras
Library WCC 8.1 x10 9 /l

Community PLT 284 x10 9 /l


Help Na + 141 mmol/l
PasTest Store
K+ 4.9 mmol/l
Creatinine 110 µmol/l
My Account
ANA positive
Profile
Newsletters Which of the following is the most likely diagnosis?
My Career and Exams
Polycystic ovarian syndrome
Order History Reference: Normal Values
Erythema multiforme
Learning Goals
Hereditary coproporphyria Click to open/closeHaematology
Question Filters Click to open/closeImmunoglobulins
Porphyria cutanea tarda Your answer
Click to open/closeBiochemistry
SLE Click to open/closeDiabetes Click to
Security open/closeEndocrinology Click to
open/closeBlood gases Click to
Change Password open/closeCSF
Sign Out This patient’s clinical picture is very typical of porphyria
cutanea tarda. Anti-nuclear antibodies are frequently
seen in patients with the condition. Use of oestrogens
may precipitate development of the condition, hence her
presentation shortly after commencing the oral
contraceptive pill. Urinary porphyrins are raised in
porphyria cutanea tarda; the cause is congenital
deficiency of uroporphyrinogen decarboxylase (UROD).
Assay of red blood cells for UROD activity is now
available in many hospital laboratories. She should be
encouraged to find another form of contraception apart
from the oestrogen containing pill.

21356

© 2011 PasTest Ltd | About Us | Contact Us | Help

file:///E|/Shakhawan/Endocrinology/323a.htm[3/13/2012 7:17:01 PM]


MyPasTest

Main Navigation
Question Browser: MRCP 1
Home
Subscriptions Question Browser Exam Builder Saved Exams
» MRCP 1 Session Progress
• Question Browser
Questions Correct 16
• Timed Test A 31-year-old woman who is 22 weeks pregnant is referred
to the diabetes clinic with glycosuria. It is her first Questions Incorrect 7
• Mock Exam
pregnancy. On examination her BP is 139/80 mmHg, her Questions Total 23
• Past Papers BMI is 32. She has no other past medical history of note.
Questions Percentage 69 %
• Random Questions
Investigations; More
• My Performance
• Media Bank Hb 11.9 g/dl
• New Multimedia
WCC 5.9 x10 9 /l
Online Extras Reference: Normal Values
PLT 178 x10 9 /l
Library
Na + 140 mmol/l Click to
Community open/closeHaematology
Help K+ 4.9 mmol/l Click to
open/closeImmunoglobulins
PasTest Store Creatinine 95 µmol/l Click to
Fasting glucose 9.2 mmol/l open/closeBiochemistry
My Account Click to open/closeDiabetes
She monitors her post-prandial glucoses and you decide Click to
Profile that dietary intervention alone is unlikely to be sufficient for open/closeEndocrinology
her. She is not keen on insulin therapy. Click to open/closeBlood
Newsletters gases Click to
My Career and Exams How would you plan to manage her sugars initially? open/closeCSF
Order History
Persuade her to accept BD mixed insulin
Learning Goals
Persuade her that a basal bolus regime is the best
Question Filters
thing for her
Start low dose gliclazide
Security Start low dose glibenclamide
Change Password Start metformin 500mg BD
Sign Out
21357

© 2011 PasTest Ltd | About Us | Contact Us | Help

file:///E|/Shakhawan/Endocrinology/324.htm[3/13/2012 7:17:03 PM]


MyPasTest

Main Navigation
Question Browser: MRCP 1
Home
Subscriptions Question Browser Exam Builder Saved Exams
» MRCP 1 Session Progress
• Question Browser
Questions Correct 17
• Timed Test A 31-year-old woman who is 22 weeks pregnant is referred
to the diabetes clinic with glycosuria. It is her first Questions Incorrect 7
• Mock Exam
pregnancy. On examination her BP is 139/80 mmHg, her Questions Total 24
• Past Papers BMI is 32. She has no other past medical history of note.
Questions Percentage 70 %
• Random Questions
Investigations; More
• My Performance
• Media Bank Hb 11.9 g/dl
• New Multimedia
WCC 5.9 x10 9 /l
Online Extras
PLT 178 x10 9 /l
Library
Community Na + 140 mmol/l
Help K+ 4.9 mmol/l
PasTest Store Creatinine 95 µmol/l
Fasting glucose 9.2 mmol/l
My Account
She monitors her post-prandial glucoses and you decide
Profile that dietary intervention alone is unlikely to be sufficient for
Newsletters her. She is not keen on insulin therapy.
My Career and Exams How would you plan to manage her sugars initially?
Order History Reference: Normal Values
Persuade her to accept BD mixed
Learning Goals
insulin Click to open/closeHaematology
Question Filters Click to open/closeImmunoglobulins
Persuade her that a basal bolus regime
is the best thing for her Click to open/closeBiochemistry
Click to open/closeDiabetes Click to
Security Start low dose gliclazide open/closeEndocrinology Click to
Start low dose glibenclamide open/closeBlood gases Click to
Change Password open/closeCSF
Sign Out Start metformin 500mg BD Your answer

This patient has proven gestational diabetes and


obesity. Traditionally management of gestational
diabetes has centred on dietary management and
insulin initiation, however two studies suggested that
glibenclamide and metformin may be viable alternatives
in appropriate patients. One very large trial of
metformin +/- insulin versus insulin alone reported
similar outcomes with respect to a composite of various
measures of fetal distress. Given the fact that she is
obese, it would therefore seem reasonable to begin with
metformin in this case.

NEJM Vol 358 2003-2015

21357

© 2011 PasTest Ltd | About Us | Contact Us | Help

file:///E|/Shakhawan/Endocrinology/324a.htm[3/13/2012 7:17:04 PM]


MyPasTest

Main Navigation
Question Browser: MRCP 1
Home
Subscriptions Question Browser Exam Builder Saved Exams
» MRCP 1 Session Progress
• Question Browser
Questions Correct 17
• Timed Test A 34-year-old obese woman with a history of polycystic
ovarian syndrome (PCOS) comes to the clinic with Questions Incorrect 7
• Mock Exam
tiredness, thirst and polyuria. She complains that she is Questions Total 24
• Past Papers unable to get pregnant, and that she has been trying for a
Questions Percentage 70 %
• Random Questions baby with her partner for the past 3 years. On examination
her BMI is 31, her BP is 155/90 mmHg. She also has acne More
• My Performance
and a pattern of hirsutism consistent with PCOS.
• Media Bank
Investigations;
• New Multimedia
Online Extras Hb 13.4 g/dl Reference: Normal Values
Library
WCC 5.6 x10 9 /l Click to
Community open/closeHaematology
PLT 230 x10 9 /l
Help Click to
Na + 139 mmol/l open/closeImmunoglobulins
PasTest Store
Click to
K+ 4.9 mmol/l open/closeBiochemistry
My Account Click to open/closeDiabetes
Creatinine 110 µmol/l Click to
Profile Fasting glucose 9.1 mmol/l open/closeEndocrinology
Click to open/closeBlood
Newsletters gases Click to
Which of the following is the most appropriate
My Career and Exams therapy with respect to both her fertility and Type 2 open/closeCSF
Order History diabetes?
Learning Goals
Dietary advice
Question Filters
Metformin monotherapy
Pioglitazone monotherapy
Security
Insulin
Change Password Gliclazide
Sign Out
21358

© 2011 PasTest Ltd | About Us | Contact Us | Help

file:///E|/Shakhawan/Endocrinology/325.htm[3/13/2012 7:17:06 PM]


MyPasTest

Main Navigation
Question Browser: MRCP 1
Home
Subscriptions Question Browser Exam Builder Saved Exams
» MRCP 1 Session Progress
• Question Browser
Questions Correct 18
• Timed Test A 34-year-old obese woman with a history of polycystic
ovarian syndrome (PCOS) comes to the clinic with Questions Incorrect 7
• Mock Exam
tiredness, thirst and polyuria. She complains that she is Questions Total 25
• Past Papers unable to get pregnant, and that she has been trying for a
Questions Percentage 72 %
• Random Questions baby with her partner for the past 3 years. On examination
her BMI is 31, her BP is 155/90 mmHg. She also has acne More
• My Performance
and a pattern of hirsutism consistent with PCOS.
• Media Bank
Investigations;
• New Multimedia
Online Extras Hb 13.4 g/dl
Library
WCC 5.6 x10 9 /l
Community
PLT 230 x10 9 /l
Help
PasTest Store Na + 139 mmol/l

K+ 4.9 mmol/l
My Account Creatinine 110 µmol/l
Profile Fasting glucose 9.1 mmol/l
Newsletters
Which of the following is the most appropriate
My Career and Exams therapy with respect to both her fertility and Type 2
Order History diabetes? Reference: Normal Values
Learning Goals
Dietary advice Click to open/closeHaematology
Question Filters Click to open/closeImmunoglobulins
Metformin monotherapy Your answer
Click to open/closeBiochemistry
Pioglitazone monotherapy Click to open/closeDiabetes Click to
Security open/closeEndocrinology Click to
Insulin
open/closeBlood gases Click to
Change Password Gliclazide open/closeCSF
Sign Out

The lack of ovulation in PCOS may be related to ovarian


insulin resistance; as such metformin is an effective
therapy both for controlling blood glucose and inducing
ovulation. Indeed, one study suggested that metformin
is potentially as effective as clomiphene when used in
PCOS. Pioglitazone reduces insulin resistance more than
metformin and is also associated in case series with
improvements in ovulation, but its association with
fractures reduces its applicability in this population.
Trials of metformin and glibenclamide in gestational
diabetes have also suggested that these may be viable
alternatives to insulin in appropriate patients.

21358

© 2011 PasTest Ltd | About Us | Contact Us | Help

file:///E|/Shakhawan/Endocrinology/325a.htm[3/13/2012 7:17:07 PM]


MyPasTest

Main Navigation
Question Browser: MRCP 1
Home
Subscriptions Question Browser Exam Builder Saved Exams
» MRCP 1 Session Progress
• Question Browser
Questions Correct 18
• Timed Test An 18-year-old girl is referred to the clinic because of
secondary amenorrhoea. She had two periods at the age of Questions Incorrect 7
• Mock Exam
14 and had been taking the combined oral contraceptive Questions Total 25
• Past Papers pill since the age of 16 but stopped it 6 months ago,
Questions Percentage 72 %
• Random Questions because as she wasn’t having any periods she didn’t see
any need to continue it. Her mother’s periods apparently More
• My Performance
stopped at the age of 26. On examination her height is 152
• Media Bank cm and her BMI is 18. She admits to training for long
• New Multimedia distance running when you question her about her weight.
Other physical examination is unremarkable. She has a
Online Extras normal pattern of secondary sexual hair. Reference: Normal Values
Library
Investigations; Click to
Community open/closeHaematology
Help Hb 12.9 g/dl Click to
open/closeImmunoglobulins
PasTest Store WCC 8.0 x10 9 /l Click to
open/closeBiochemistry
PLT 172 x10 9 /l
My Account Click to open/closeDiabetes
Na + 139 mmol/l Click to
Profile open/closeEndocrinology
K+ 4.3 mmol/l Click to open/closeBlood
Newsletters gases Click to
My Career and Exams Creatinine 90 µmol/l open/closeCSF
Order History TSH 2.3 U/l (<0.5-4.5)

Learning Goals FSH markedly elevated


Question Filters LH markedly elevated

What is the likely diagnosis?


Security
Turners syndrome
Change Password
Polycystic ovarian syndrome (PCOS)
Sign Out
Premature ovarian failure
Pregnancy
Sheehan syndrome

22453

© 2011 PasTest Ltd | About Us | Contact Us | Help

file:///E|/Shakhawan/Endocrinology/326.htm[3/13/2012 7:17:09 PM]


MyPasTest

Main Navigation
Question Browser: MRCP 1
Home
Subscriptions Question Browser Exam Builder Saved Exams
» MRCP 1 Session Progress
• Question Browser
Questions Correct 19
• Timed Test An 18-year-old girl is referred to the clinic because of
secondary amenorrhoea. She had two periods at the age of Questions Incorrect 7
• Mock Exam
14 and had been taking the combined oral contraceptive Questions Total 26
• Past Papers pill since the age of 16 but stopped it 6 months ago,
Questions Percentage 73 %
• Random Questions because as she wasn’t having any periods she didn’t see
any need to continue it. Her mother’s periods apparently More
• My Performance
stopped at the age of 26. On examination her height is 152
• Media Bank cm and her BMI is 18. She admits to training for long
• New Multimedia distance running when you question her about her weight.
Other physical examination is unremarkable. She has a
Online Extras normal pattern of secondary sexual hair.
Library
Investigations;
Community
Help Hb 12.9 g/dl
PasTest Store WCC 8.0 x10 9 /l

PLT 172 x10 9 /l


My Account
Na + 139 mmol/l
Profile
K+ 4.3 mmol/l
Newsletters
My Career and Exams Creatinine 90 µmol/l

Order History TSH 2.3 U/l (<0.5-4.5) Reference: Normal Values


Learning Goals FSH markedly elevated
Click to open/closeHaematology
Question Filters LH markedly elevated Click to open/closeImmunoglobulins
Click to open/closeBiochemistry
What is the likely diagnosis? Click to open/closeDiabetes Click to
Security open/closeEndocrinology Click to
Turners syndrome open/closeBlood gases Click to
Change Password open/closeCSF
Polycystic ovarian syndrome (PCOS)
Sign Out
Premature ovarian failure Your answer
Pregnancy
Sheehan syndrome

This woman has had at least 2 periods that she reports;


hence this is by definition secondary amenorrhoea. The
raised FSH and LH levels are suggestive of premature
ovarian failure. The two most likely causes include low
percentage of body fat, and autoimmune ovarian
failure. Hormone replacement is essential to avoid
osteoporosis in the longer term. Sheehan syndrome is
associated with failure to produce GnRH, hence FSH and
LH levels are low, Turners is associated with raised
levels of gonadotrophins, but menses never occur.

22453

© 2011 PasTest Ltd | About Us | Contact Us | Help

file:///E|/Shakhawan/Endocrinology/326a.htm[3/13/2012 7:17:10 PM]


MyPasTest

Main Navigation
Question Browser: MRCP 1
Home
Subscriptions Question Browser Exam Builder Saved Exams
» MRCP 1 Session Progress
• Question Browser
Questions Correct 19
• Timed Test A 25-year-old woman goes to her GP complaining of
tiredness and thirst. She tells you that she has no Questions Incorrect 7
• Mock Exam
significant history of previous medical problems, apart from Questions Total 26
• Past Papers two episodes with a kidney stone. In terms of family
Questions Percentage 73 %
• Random Questions history, her father had two of his parathyroid glands
removed for high calcium levels and recurrent pancreatitis More
• My Performance
when he was in his thirties but this didn’t seem to cure the
• Media Bank problem. On examination she looks well. Her BP is 118/72
• New Multimedia mmHg, pulse 75/min, her BMI is 22.

Online Extras Investigations; Reference: Normal Values


Library
Hb 12.3 g/dl Click to
Community open/closeHaematology
Help WCC 5.9 x10 9 /l Click to
open/closeImmunoglobulins
PasTest Store PLT 183 x10 9 /l
Click to
Na + 141 mmol/l open/closeBiochemistry
My Account Click to open/closeDiabetes
K+ 4.5 mmol/l Click to
Profile open/closeEndocrinology
Creatinine 105 µmol/l Click to open/closeBlood
Newsletters gases Click to
Ca ++ 2.83 mmol/l
My Career and Exams open/closeCSF
PTH 9 (normal<10)
Order History
Alkaline phosphatase 82 U/l
Learning Goals
Question Filters Urine calcium excretion decreased

Which of the following is the most likely diagnosis?


Security
Primary hyperparathyroidism
Change Password
Secondary hyperparathyroidism
Sign Out
Hypoparathyroidism
Familial hypercalcaemic hypocalcicuria
Osteomalacia

22501

© 2011 PasTest Ltd | About Us | Contact Us | Help

file:///E|/Shakhawan/Endocrinology/327.htm[3/13/2012 7:17:12 PM]


MyPasTest

Main Navigation
Question Browser: MRCP 1
Home
Subscriptions Question Browser Exam Builder Saved Exams
» MRCP 1 Session Progress
• Question Browser
Questions Correct 20
• Timed Test A 25-year-old woman goes to her GP complaining of
tiredness and thirst. She tells you that she has no Questions Incorrect 7
• Mock Exam
significant history of previous medical problems, apart from Questions Total 27
• Past Papers two episodes with a kidney stone. In terms of family
Questions Percentage 74 %
• Random Questions history, her father had two of his parathyroid glands
removed for high calcium levels and recurrent pancreatitis More
• My Performance
when he was in his thirties but this didn’t seem to cure the
• Media Bank problem. On examination she looks well. Her BP is 118/72
• New Multimedia mmHg, pulse 75/min, her BMI is 22.

Online Extras Investigations;


Library
Hb 12.3 g/dl
Community
Help WCC 5.9 x10 9 /l
PasTest Store PLT 183 x10 9 /l

Na + 141 mmol/l
My Account
K+ 4.5 mmol/l
Profile
Creatinine 105 µmol/l
Newsletters
Ca ++ 2.83 mmol/l
My Career and Exams
PTH 9 (normal<10) Reference: Normal Values
Order History
Alkaline phosphatase 82 U/l
Learning Goals
Click to open/closeHaematology
Question Filters Urine calcium excretion decreased
Click to open/closeImmunoglobulins
Click to open/closeBiochemistry
Which of the following is the most likely diagnosis? Click to open/closeDiabetes Click to
Security open/closeEndocrinology Click to
Primary hyperparathyroidism open/closeBlood gases Click to
Change Password open/closeCSF
Secondary hyperparathyroidism
Sign Out
Hypoparathyroidism
Familial hypercalcaemic hypocalcicuria Your answer
Osteomalacia

FHH is an autosomal dominant mutation of the calcium


receptor sensing gene. It leads to reduced calcium
excretion and consequent mild to moderate
hypercalcaemia. Patients can be asymptomatic or
present with renal stones, rarely the condition can be
associated with episodes of acute pancreatitis. Patients
with mild FHH are recommended to maintain adequate
hydration to reduce the risk of stone formation. For
recurrent severe pancreatitis, total parathyroidectomy
appears to be effective.

http://pmj.bmj.com/content/63/744/835.full.pdf

22501

file:///E|/Shakhawan/Endocrinology/327a.htm[3/13/2012 7:17:13 PM]


MyPasTest

Main Navigation
Question Browser: MRCP 1
Home
Subscriptions Question Browser Exam Builder Saved Exams
» MRCP 1 Session Progress
• Question Browser
Questions Correct 20
• Timed Test A 26-year-old woman presents to the clinic with nausea,
thirst and dehydration. She explains that she has an uncle Questions Incorrect 7
• Mock Exam
with a previous parathyroid gland excision and a cousin Questions Total 27
• Past Papers who has recently been diagnosed with an insulinoma. On
Questions Percentage 74 %
• Random Questions examination her BP is 135/72 mmHg, pulse is 70/min and
regular, her BMI is 20. Cardiovascular, respiratory and More
• My Performance
abdominal examinations are unremarkable.
• Media Bank
Investigations;
• New Multimedia
Online Extras Hb 12.6 g/dl Reference: Normal Values
Library
WCC 5.4 x10 9 /l Click to
Community open/closeHaematology
PLT 299 x10 9 /l
Help Click to
Na + 139 mmol/l open/closeImmunoglobulins
PasTest Store
Click to
K+ 4.4 mmol/l open/closeBiochemistry
My Account Click to open/closeDiabetes
Creatinine 121 µmol/l Click to
Profile Albumin 37 g/l open/closeEndocrinology
Click to open/closeBlood
Newsletters Ca ++ 2.95 mmol/l gases Click to
My Career and Exams PTH 18 (normal<10) open/closeCSF
Order History
Learning Goals Which of the following is the most likely cause of her
raised calcium?
Question Filters
MEN 2a
Security Familial hypercalcaemic hypocalciuria

Change Password Parathyroid hyperplasia

Sign Out Parathyroid adenoma


MEN 2b

22504

© 2011 PasTest Ltd | About Us | Contact Us | Help

file:///E|/Shakhawan/Endocrinology/328.htm[3/13/2012 7:17:15 PM]


MyPasTest

Main Navigation
Question Browser: MRCP 1
Home
Subscriptions Question Browser Exam Builder Saved Exams
» MRCP 1 Session Progress
• Question Browser
Questions Correct 20
• Timed Test A 26-year-old woman presents to the clinic with nausea,
thirst and dehydration. She explains that she has an uncle Questions Incorrect 8
• Mock Exam
with a previous parathyroid gland excision and a cousin Questions Total 28
• Past Papers who has recently been diagnosed with an insulinoma. On
Questions Percentage 71 %
• Random Questions examination her BP is 135/72 mmHg, pulse is 70/min and
regular, her BMI is 20. Cardiovascular, respiratory and More
• My Performance
abdominal examinations are unremarkable.
• Media Bank
Investigations;
• New Multimedia
Online Extras Hb 12.6 g/dl
Library
WCC 5.4 x10 9 /l
Community
PLT 299 x10 9 /l
Help
PasTest Store Na + 139 mmol/l

K+ 4.4 mmol/l
My Account Creatinine 121 µmol/l
Profile Albumin 37 g/l
Newsletters Ca ++ 2.95 mmol/l
My Career and Exams PTH 18 (normal<10)
Order History Reference: Normal Values
Learning Goals Which of the following is the most likely cause of her
raised calcium? Click to open/closeHaematology
Question Filters Click to open/closeImmunoglobulins
MEN 2a Your answer Click to open/closeBiochemistry
Click to open/closeDiabetes Click to
Security Familial hypercalcaemic open/closeEndocrinology Click to
hypocalciuria open/closeBlood gases Click to
Change Password open/closeCSF
Parathyroid hyperplasia Correct answer
Sign Out
Parathyroid adenoma
MEN 2b

The combination of parathyroid disease and insulinoma


is suggestive of MEN-1. Parathyroid hyperplasia is
associated with MEN-1, whereas MEN-2 is associated
with parathyroid adenomas. Gastrinomas are the
commonest pancreatic tumours found in MEN-1,
accounting for 60% of functional tumours, insulinomas
account for 30-35% of pancreatic tumours,
glucagonomas occurring in around 3% of patients.
Pituitary hyperplasia is also seen in MEN-1.

22504

© 2011 PasTest Ltd | About Us | Contact Us | Help

file:///E|/Shakhawan/Endocrinology/328a.htm[3/13/2012 7:17:16 PM]


MyPasTest

Main Navigation
Question Browser: MRCP 1
Home
Subscriptions Question Browser Exam Builder Saved Exams
» MRCP 1 Session Progress
• Question Browser
Questions Correct 20
• Timed Test A 34-year-old man presents with tiredness and postural
hypotension. He works nights in a factory and suffers from Questions Incorrect 8
• Mock Exam
both asthma for which he takes inhaled fluticasone, and Questions Total 28
• Past Papers allergic rhinitis for which he takes an over the counter nasal
Questions Percentage 71 %
• Random Questions spray. On examination he has a BP of 130/80 mmHg,
which drops to 95/60 mmHg on standing. His pulse also More
• My Performance
increases from 65/min to 92/min. Otherwise physical
• Media Bank examination is unremarkable.
• New Multimedia
Investigations;
Online Extras Reference: Normal Values
Library Hb 12.9 g/dl
Click to
Community WCC 7.8 x10 9 /l open/closeHaematology
Help Click to
PLT 210 x10 9 /l
open/closeImmunoglobulins
PasTest Store
Na + 132 mmol/l Click to
open/closeBiochemistry
My Account K+ 5.3 mmol/l Click to open/closeDiabetes
Click to
Creatinine 129 µmol/l open/closeEndocrinology
Profile
9am cortisol: 200 nmol/l (200-700) Click to open/closeBlood
Newsletters gases Click to
My Career and Exams 30 minutes into synacthen test: 570 open/closeCSF
Order History
Which of the following is the most likely diagnosis?
Learning Goals
Question Filters ACTH deficiency
Primary adrenal failure

Security Normal adrenal function


Corticosteroid related adrenal suppression
Change Password
SIADH
Sign Out
22514

© 2011 PasTest Ltd | About Us | Contact Us | Help

file:///E|/Shakhawan/Endocrinology/329.htm[3/13/2012 7:17:18 PM]


MyPasTest

Main Navigation
Question Browser: MRCP 1
Home
Subscriptions Question Browser Exam Builder Saved Exams
» MRCP 1 Session Progress
• Question Browser
Questions Correct 21
• Timed Test A 34-year-old man presents with tiredness and postural
hypotension. He works nights in a factory and suffers from Questions Incorrect 8
• Mock Exam
both asthma for which he takes inhaled fluticasone, and Questions Total 29
• Past Papers allergic rhinitis for which he takes an over the counter nasal
Questions Percentage 72 %
• Random Questions spray. On examination he has a BP of 130/80 mmHg,
which drops to 95/60 mmHg on standing. His pulse also More
• My Performance
increases from 65/min to 92/min. Otherwise physical
• Media Bank examination is unremarkable.
• New Multimedia
Investigations;
Online Extras
Library Hb 12.9 g/dl
Community WCC 7.8 x10 9 /l
Help
PLT 210 x10 9 /l
PasTest Store
Na + 132 mmol/l

My Account K+ 5.3 mmol/l


Creatinine 129 µmol/l
Profile
9am cortisol: 200 nmol/l (200-700)
Newsletters
My Career and Exams 30 minutes into synacthen test: 570

Order History Reference: Normal Values


Which of the following is the most likely diagnosis?
Learning Goals
Click to open/closeHaematology
Question Filters ACTH deficiency Click to open/closeImmunoglobulins
Primary adrenal failure Click to open/closeBiochemistry
Click to open/closeDiabetes Click to
Security Normal adrenal function
open/closeEndocrinology Click to
Corticosteroid related adrenal Your answer open/closeBlood gases Click to
Change Password suppression open/closeCSF
Sign Out SIADH

It is likely he is using an over the counter inhaled nasal


corticosteroid preparation, as well as his inhaled
corticosteroids for asthma. The combination is likely to
have led to a degree of adrenal suppression as
evidenced by hyponatraemia, hyperkalaemia and
postural hypotension, coupled with a low baseline
cortisol which returns to the normal range after
administration of Synacthen. Treatment of choice is
cessation of inhaled nasal steroids coupled with a
strategy to reduce his asthma therapy corticosteroids,
for instance by adding in a long acting beta 2 agonist
and then stepping down the level of inhaled steroids.
Once these strategies are in place, the Synacthen test
should be repeated.

22514

© 2011 PasTest Ltd | About Us | Contact Us | Help

file:///E|/Shakhawan/Endocrinology/329a.htm[3/13/2012 7:17:19 PM]


MyPasTest

Main Navigation
Question Browser: MRCP 1
Home
Subscriptions Question Browser Exam Builder Saved Exams
» MRCP 1 Session Progress
• Question Browser
Questions Correct 21
• Timed Test An 18-year-old man comes to the endocrine clinic. He
complains that his weight has increased very significantly Questions Incorrect 8
• Mock Exam
over the last 6 months. He works hard as a computer Questions Total 29
• Past Papers operator and has a sedentary job, but tells you that he at
Questions Percentage 72 %
• Random Questions least tries to get out for a walk most lunchtimes. On
examination his BP is 142/84 mmHg, his pulse 75/min and More
• My Performance
regular. His BMI is 28 and he has abdominal striae.
• Media Bank
Investigations;
• New Multimedia
Online Extras Hb 12.8 g/dl Reference: Normal Values
Library
WCC 6.1 x10 9 /l Click to
Community open/closeHaematology
PLT 217 x10 9 /l
Help Click to
Na + 141 mmol/l open/closeImmunoglobulins
PasTest Store
Click to
K+ 3.4 mmol/l open/closeBiochemistry
My Account Click to open/closeDiabetes
Creatinine 110 µmol/l Click to
Profile open/closeEndocrinology
Given the suspected diagnosis, which of the following Click to open/closeBlood
Newsletters is the most appropriate next investigation? gases Click to
My Career and Exams open/closeCSF
Aldosterone levels
Order History
ACTH levels
Learning Goals
Short synacthen test
Question Filters
High dose dexamethasone suppression test
24hr urinary free cortisol
Security

Change Password 22515


Sign Out

© 2011 PasTest Ltd | About Us | Contact Us | Help

file:///E|/Shakhawan/Endocrinology/330.htm[3/13/2012 7:17:21 PM]


MyPasTest

Main Navigation
Question Browser: MRCP 1
Home
Subscriptions Question Browser Exam Builder Saved Exams
» MRCP 1 Session Progress
• Question Browser
Questions Correct 22
• Timed Test An 18-year-old man comes to the endocrine clinic. He
complains that his weight has increased very significantly Questions Incorrect 8
• Mock Exam
over the last 6 months. He works hard as a computer Questions Total 30
• Past Papers operator and has a sedentary job, but tells you that he at
Questions Percentage 73 %
• Random Questions least tries to get out for a walk most lunchtimes. On
examination his BP is 142/84 mmHg, his pulse 75/min and More
• My Performance
regular. His BMI is 28 and he has abdominal striae.
• Media Bank
Investigations;
• New Multimedia
Online Extras Hb 12.8 g/dl
Library
WCC 6.1 x10 9 /l
Community
PLT 217 x10 9 /l
Help
PasTest Store Na + 141 mmol/l

K+ 3.4 mmol/l
My Account Creatinine 110 µmol/l
Profile
Given the suspected diagnosis, which of the following
Newsletters is the most appropriate next investigation?
My Career and Exams
Aldosterone levels Reference: Normal Values
Order History
ACTH levels
Learning Goals
Click to open/closeHaematology
Short synacthen test
Question Filters Click to open/closeImmunoglobulins
High dose dexamethasone suppression Click to open/closeBiochemistry
test Click to open/closeDiabetes Click to
Security open/closeEndocrinology Click to
24hr urinary free cortisol Your answer
open/closeBlood gases Click to
Change Password open/closeCSF
Sign Out

The diagnosis to be excluded here is Cushing’s


syndrome. 24hr urinary free cortisol is an ideal initial
test as it can be done as an outpatient and is non-
invasive. The false positive rate for the test is 1%, with
the false negative rate put at 5-10%. Because of the
false negative rate approaching 10%, it is never used
alone as a screening test for Cushing’s and should be
followed by an overnight dexamethasone suppression
test. If both of these tests are normal then Cushing’s
can effectively be ruled out. Midnight cortisol levels may
be an additional useful test in some cases to
demonstrate the loss of circadian rhythm of cortisol
secretion.

22515

© 2011 PasTest Ltd | About Us | Contact Us | Help

file:///E|/Shakhawan/Endocrinology/330a.htm[3/13/2012 7:17:22 PM]


MyPasTest

Main Navigation
Question Browser: MRCP 1
Home
Subscriptions Question Browser Exam Builder Saved Exams
» MRCP 1
• Question Browser
• Timed Test A 19-year-old woman presents with nausea, vomiting and dehydration. She has been
• Mock Exam unwell for some weeks and passed out whilst shopping and was brought to hospital by
ambulance. The only other past history of note is that her periods stopped a few
• Past Papers months earlier. On examination her BP is 110/80 mmHg lying, and drops to 90/60
• Random Questions mmHg on standing; her pulse is 90/min and regular. Her BMI is 19 and her skin looks
quite tanned. Other examination is unremarkable.
• My Performance
• Media Bank Investigations;
• New Multimedia
Hb 11.0 g/dl
Online Extras
Library WCC 8.3 x10 9 /l

Community PLT 191 x10 9 /l


Help Na + 131 mmol/l
PasTest Store
K+ 4.9 mmol/l
Creatinine 130 µmol/l
My Account

Profile She is diagnosed with Addison’s disease.


Newsletters Change in which hormone is likely to be responsible for her amenorrhoea?
My Career and Exams
Aldosterone
Order History
Cortisol
Learning Goals
17-alpha OH progesterone
Question Filters
Dehydroepiandrosterone
Prolactin
Security

Change Password 22516

Sign Out

file:///E|/Shakhawan/Endocrinology/331.htm[3/13/2012 7:17:24 PM]


MyPasTest

Main Navigation
Question Browser: MRCP 1
Home
Subscriptions Question Browser Exam Builder Saved Exams
» MRCP 1 Session Progress
• Question Browser
Questions Correct 22
• Timed Test A 19-year-old woman presents with nausea, vomiting and
dehydration. She has been unwell for some weeks and Questions Incorrect 9
• Mock Exam
passed out whilst shopping and was brought to hospital by Questions Total 31
• Past Papers ambulance. The only other past history of note is that her
Questions Percentage 70 %
• Random Questions periods stopped a few months earlier. On examination her
BP is 110/80 mmHg lying, and drops to 90/60 mmHg on More
• My Performance
standing; her pulse is 90/min and regular. Her BMI is 19
• Media Bank and her skin looks quite tanned. Other examination is
• New Multimedia unremarkable.

Online Extras Investigations;


Library
Hb 11.0 g/dl
Community
Help WCC 8.3 x10 9 /l
PasTest Store PLT 191 x10 9 /l

Na + 131 mmol/l
My Account
K+ 4.9 mmol/l
Profile
Creatinine 130 µmol/l
Newsletters
My Career and Exams She is diagnosed with Addison’s disease.
Order History Reference: Normal Values
Change in which hormone is likely to be responsible
Learning Goals for her amenorrhoea?
Click to open/closeHaematology
Question Filters Click to open/closeImmunoglobulins
Aldosterone
Click to open/closeBiochemistry
Cortisol Click to open/closeDiabetes Click to
Security open/closeEndocrinology Click to
17-alpha OH progesterone
open/closeBlood gases Click to
Change Password Dehydroepiandrosterone Your answer open/closeCSF
Sign Out Prolactin Correct answer

Modest hyperprolactinaemia is reported in cases of


Addison’s and is glucocorticoid responsive. The absence
of steroid enhanced hypothalamic dopaminergic tone is
thought to lead to hyper-responsiveness of the
lactotroph to TRH, which leads to the elevated prolactin
levels. If prolactin is elevated enough, this may lead to
symptoms of hyperprolactinaemia which include
cessation of menses.

22516

© 2011 PasTest Ltd | About Us | Contact Us | Help

file:///E|/Shakhawan/Endocrinology/331a.htm[3/13/2012 7:17:25 PM]


MyPasTest

Main Navigation
Question Browser: MRCP 1
Home
Subscriptions Question Browser Exam Builder Saved Exams
» MRCP 1 Session Progress
• Question Browser
Questions Correct 22
• Timed Test A 41-year-old woman presents to the clinic because she is
concerned about recent weight gain. In total she has put on Questions Incorrect 9
• Mock Exam
around 5kg over the past 4 months. She is also feeling Questions Total 31
• Past Papers rather tired and has asked to reduce her hours at work
Questions Percentage 70 %
• Random Questions because of this. There is no history of medication use apart
from the progesterone-only pill. On examination her BMI is More
• My Performance
28, her BP is 155/90 mmHg, pulse 72/min regular. Her
• Media Bank heart sounds are normal, chest is clear and abdominal
• New Multimedia examination is unremarkable. There is a slight, smooth
goitre on examination of the neck.
Online Extras Reference: Normal Values
Library Investigations;
Click to
Community Hb 12.1 g/dl open/closeHaematology
Help Click to
WCC 5.7 x10 9 /l open/closeImmunoglobulins
PasTest Store
Click to
PLT 164 x10 9 /l
open/closeBiochemistry
My Account Na + 137 mmol/l Click to open/closeDiabetes
Click to
Profile K+ 4.5 mmol/l open/closeEndocrinology
Click to open/closeBlood
Newsletters Creatinine 110 µmol/l gases Click to
My Career and Exams Total cholesterol 6.5 mmol/l open/closeCSF
Order History Triglycerides 3.1 mmol/l
Learning Goals TSH 7.5 mU/l (0.4-5.0)
Question Filters Free T4 9.2 pmol/l (10-22)
Free T3 3.0 pmol/l (5-10)
Security Anti TPO antibody positive
Change Password
Which of the following is the most likely diagnosis?
Sign Out
Hashimoto’s thyroiditis
Grave’s disease
Viral thyroiditis
Sick euthyroid syndrome
Weight gain due to lifestyle changes

22517

© 2011 PasTest Ltd | About Us | Contact Us | Help

file:///E|/Shakhawan/Endocrinology/332.htm[3/13/2012 7:17:27 PM]


MyPasTest

Main Navigation
Question Browser: MRCP 1
Home
Subscriptions Question Browser Exam Builder Saved Exams
» MRCP 1 Session Progress
• Question Browser
Questions Correct 23
• Timed Test A 41-year-old woman presents to the clinic because she is
concerned about recent weight gain. In total she has put on Questions Incorrect 9
• Mock Exam
around 5kg over the past 4 months. She is also feeling Questions Total 32
• Past Papers rather tired and has asked to reduce her hours at work
Questions Percentage 71 %
• Random Questions because of this. There is no history of medication use apart
from the progesterone-only pill. On examination her BMI is More
• My Performance
28, her BP is 155/90 mmHg, pulse 72/min regular. Her
• Media Bank heart sounds are normal, chest is clear and abdominal
• New Multimedia examination is unremarkable. There is a slight, smooth
goitre on examination of the neck.
Online Extras
Library Investigations;

Community Hb 12.1 g/dl


Help
WCC 5.7 x10 9 /l
PasTest Store
PLT 164 x10 9 /l

My Account Na + 137 mmol/l

Profile K+ 4.5 mmol/l


Newsletters Creatinine 110 µmol/l
My Career and Exams Total cholesterol 6.5 mmol/l
Order History Triglycerides 3.1 mmol/l Reference: Normal Values
Learning Goals TSH 7.5 mU/l (0.4-5.0) Click to open/closeHaematology
Question Filters Free T4 9.2 pmol/l (10-22) Click to open/closeImmunoglobulins
Click to open/closeBiochemistry
Free T3 3.0 pmol/l (5-10) Click to open/closeDiabetes Click to
Security Anti TPO antibody positive open/closeEndocrinology Click to
open/closeBlood gases Click to
Change Password open/closeCSF
Which of the following is the most likely diagnosis?
Sign Out
Hashimoto’s thyroiditis Your answer
Grave’s disease
Viral thyroiditis
Sick euthyroid syndrome
Weight gain due to lifestyle changes

This patient has recent weight gain, increased total


cholesterol and triglycerides, a raised TSH with low
normal T3 and T4 and a small goitre. This picture is
entirely consistent with Hashimoto’s thyroiditis. Thyroid
hormone replacement is probably indicated in this case
with the aim of improving symptoms and restoring TSH
to the normal range. It is likely in her case that her
hypothyroidism will develop over coming months and
her dose of thyroxine may increase.

22517

file:///E|/Shakhawan/Endocrinology/332a.htm[3/13/2012 7:17:28 PM]


MyPasTest

Main Navigation
Question Browser: MRCP 1
Home
Subscriptions Question Browser Exam Builder Saved Exams
» MRCP 1 Session Progress
• Question Browser
Questions Correct 23
• Timed Test A 72-year-old woman is admitted to the Emergency room
with confusion. She is known to have both hypothyroidism Questions Incorrect 9
• Mock Exam
and hypertension and takes ramipril and 150mcg daily of Questions Total 32
• Past Papers thyroid hormone replacement. Her thyroxine dose was
Questions Percentage 71 %
• Random Questions increased a few weeks earlier because her T4 was still
slightly below the normal range. On examination her BP is More
• My Performance
142/82 mmHg. Her pulse is 74/min and regular, her BMI is
• Media Bank 29. There are no abnormal heart sounds and her chest is
• New Multimedia clear. Abdominal examination reveals obesity but nil else of
note.
Online Extras Reference: Normal Values
Library Investigations;
Click to
Community Hb 12.1 g/dl open/closeHaematology
Help Click to
WCC 6.7 x10 9 /l open/closeImmunoglobulins
PasTest Store
Click to
PLT 191 x10 9 /l
open/closeBiochemistry
My Account Na + 120 mmol/l Click to open/closeDiabetes
Click to
Profile K+ 4.5 mmol/l open/closeEndocrinology
Click to open/closeBlood
Newsletters Creatinine 121 µmol/l gases Click to
My Career and Exams TSH 7.4 mU/l (0.4-5.0) open/closeCSF
Order History Free T4 6.7 pmol/l (10-22)
Learning Goals
Question Filters Which of the following is the most likely diagnosis?

Thyroid hormone resistance


Security Sick euthyroid syndrome
Change Password Viral thyroiditis
Sign Out Non-compliance with medication
Poor thyroxine absorption

22518

© 2011 PasTest Ltd | About Us | Contact Us | Help

file:///E|/Shakhawan/Endocrinology/333.htm[3/13/2012 7:17:30 PM]


MyPasTest

Main Navigation
Question Browser: MRCP 1
Home
Subscriptions Question Browser Exam Builder Saved Exams
» MRCP 1 Session Progress
• Question Browser
Questions Correct 24
• Timed Test A 72-year-old woman is admitted to the Emergency room
with confusion. She is known to have both hypothyroidism Questions Incorrect 9
• Mock Exam
and hypertension and takes ramipril and 150mcg daily of Questions Total 33
• Past Papers thyroid hormone replacement. Her thyroxine dose was
Questions Percentage 72 %
• Random Questions increased a few weeks earlier because her T4 was still
slightly below the normal range. On examination her BP is More
• My Performance
142/82 mmHg. Her pulse is 74/min and regular, her BMI is
• Media Bank 29. There are no abnormal heart sounds and her chest is
• New Multimedia clear. Abdominal examination reveals obesity but nil else of
note.
Online Extras
Library Investigations;

Community Hb 12.1 g/dl


Help
WCC 6.7 x10 9 /l
PasTest Store
PLT 191 x10 9 /l

My Account Na + 120 mmol/l

Profile K+ 4.5 mmol/l


Newsletters Creatinine 121 µmol/l
My Career and Exams TSH 7.4 mU/l (0.4-5.0)
Order History Free T4 6.7 pmol/l (10-22) Reference: Normal Values
Learning Goals
Click to open/closeHaematology
Question Filters Which of the following is the most likely diagnosis?
Click to open/closeImmunoglobulins
Click to open/closeBiochemistry
Thyroid hormone resistance
Click to open/closeDiabetes Click to
Security Sick euthyroid syndrome open/closeEndocrinology Click to
Viral thyroiditis open/closeBlood gases Click to
Change Password open/closeCSF
Sign Out Non-compliance with medication Your answer
Poor thyroxine absorption

The most likely problem here is that this lady has


suffered a precipitating illness leading to confusion and
non-compliance with thyroxine therapy. This has been
further exacerbated by consequent SIADH as evidenced
by the hyponatraemia seen here. The elevation of TSH
in the presence of a low free T4 is entirely consistent
with this picture. The increase to 150 mcg of thyroxine
may itself have been precipitated by poor compliance or
just a slow fall off in endogenous thyroxine production
due to underlying Hashimoto’s.

22518

© 2011 PasTest Ltd | About Us | Contact Us | Help

file:///E|/Shakhawan/Endocrinology/333a.htm[3/13/2012 7:17:31 PM]


MyPasTest

Main Navigation
Question Browser: MRCP 1
Home
Subscriptions Question Browser Exam Builder Saved Exams
» MRCP 1 Session Progress
• Question Browser
Questions Correct 24
• Timed Test A 32-year-old woman attends the GP complaining of
anxiety and tiredness. She has no other past medical Questions Incorrect 9
• Mock Exam
history of note. On examination she has a diffusely Questions Total 33
• Past Papers enlarged smooth non-tender goitre. Her BP is 142/85
Questions Percentage 72 %
• Random Questions mmHg and her pulse is 90/min.
• My Performance More
Investigations;
• Media Bank
• New Multimedia Hb 12.4 g/dl

Online Extras WCC 5.9 x10 9 /l Reference: Normal Values


Library PLT 182 x10 9 /l
Click to
Community open/closeHaematology
Na + 139 mmol/l
Help Click to
PasTest Store K+ 4.5 mmol/l open/closeImmunoglobulins
Click to
TSH 7.5 mU/l (0.4-5.0) open/closeBiochemistry
My Account Free T4 24 pmol/l (normal range 9-25) Click to open/closeDiabetes
Click to
Free T3 8.1 pmol/l (normal range 3.4-7.2) open/closeEndocrinology
Profile
Click to open/closeBlood
Newsletters How would you best investigate this patient? gases Click to
My Career and Exams open/closeCSF
Thyroid ultrasound
Order History
Fine needle aspiration of thyroid
Learning Goals
MRI pituitary
Question Filters
Thyroid isotope scan
Thyroid autoantibodies
Security

Change Password 22519


Sign Out

© 2011 PasTest Ltd | About Us | Contact Us | Help

file:///E|/Shakhawan/Endocrinology/334.htm[3/13/2012 7:17:33 PM]


MyPasTest

Main Navigation
Question Browser: MRCP 1
Home
Subscriptions Question Browser Exam Builder Saved Exams
» MRCP 1 Session Progress
• Question Browser
Questions Correct 24
• Timed Test A 32-year-old woman attends the GP complaining of
anxiety and tiredness. She has no other past medical Questions Incorrect 10
• Mock Exam
history of note. On examination she has a diffusely Questions Total 34
• Past Papers enlarged smooth non-tender goitre. Her BP is 142/85
Questions Percentage 70 %
• Random Questions mmHg and her pulse is 90/min.
• My Performance More
Investigations;
• Media Bank
• New Multimedia Hb 12.4 g/dl

Online Extras WCC 5.9 x10 9 /l


Library PLT 182 x10 9 /l
Community
Na + 139 mmol/l
Help
PasTest Store K+ 4.5 mmol/l
TSH 7.5 mU/l (0.4-5.0)

My Account Free T4 24 pmol/l (normal range 9-25)


Free T3 8.1 pmol/l (normal range 3.4-7.2)
Profile
Newsletters How would you best investigate this patient?
My Career and Exams
Thyroid ultrasound Reference: Normal Values
Order History
Fine needle aspiration of thyroid
Learning Goals
Correct answer Click to open/closeHaematology
MRI pituitary
Question Filters Click to open/closeImmunoglobulins
Thyroid isotope scan Your answer Click to open/closeBiochemistry
Thyroid autoantibodies Click to open/closeDiabetes Click to
Security open/closeEndocrinology Click to
open/closeBlood gases Click to
Change Password open/closeCSF
Sign Out
Rarely, thyrotoxicosis may be due to a TSH-producing
pituitary tumour. It is associated with raised TSH and
elevated T3 and T4 as seen here. There is a diffuse
smoothly enlarged thyroid due to TSH stimulation. MRI
pituitary is the next investigation of choice, with
pituitary function testing. Transphenoidal resection of
the tumour is the therapy of choice.

22519

© 2011 PasTest Ltd | About Us | Contact Us | Help

file:///E|/Shakhawan/Endocrinology/334a.htm[3/13/2012 7:17:34 PM]


MyPasTest

Main Navigation
Question Browser: MRCP 1
Home
Subscriptions Question Browser Exam Builder Saved Exams
» MRCP 1 Session Progress
• Question Browser
Questions Correct 24
• Timed Test A 34-year-old man presents to the Emergency room. His
wife says that he complained of sudden onset severe Questions Incorrect 10
• Mock Exam
headache, nausea and vomiting and double vision and then Questions Total 34
• Past Papers passed out on the hall floor. He works as a lawyer and has
Questions Percentage 70 %
• Random Questions no significant past medical history of note, but has recently
been undergoing a trial of sildenafil for erectile dysfunction. More
• My Performance
On examination he looks pale, his BP is 105/70 mmHg, his
• Media Bank pulse is 98/min. Heart sounds are normal, his chest is
• New Multimedia clear, and abdominal examination is unremarkable. He
begins to wake up more as you are examining him and is
Online Extras orientated, able to speak and obey commands. He Reference: Normal Values
Library complains of severe retro-orbital headache. He has a
bitemporal superior quadrantinopia on visual field testing. Click to
Community open/closeHaematology
You give him some IV cyclizine, as he continues to vomit.
Help Click to
Investigations; open/closeImmunoglobulins
PasTest Store
Click to
Hb 12.4 g/dl open/closeBiochemistry
My Account Click to open/closeDiabetes
WCC 5.4 x10 9 /l Click to
Profile open/closeEndocrinology
PLT 203 x10 9 /l
Click to open/closeBlood
Newsletters gases Click to
Na + 132 mmol/l
My Career and Exams open/closeCSF
Order History K+ 5.4 mmol/l

Learning Goals Creatinine 119 µmol/l


Question Filters TSH 1.5 mU/l (0.4-5.0)
Free T4 7.7 pmol/l (low)
Security Testosterone 8.0 nmol/l (low)

Change Password Which of the following is the most likely diagnosis


Sign Out leading to his acute presentation?

Subarachnoid haemorrhage
Pituitary apoplexy
Migraine
Parietal lobe infarct
Non-functioning pituitary adenoma

22520

© 2011 PasTest Ltd | About Us | Contact Us | Help

file:///E|/Shakhawan/Endocrinology/335.htm[3/13/2012 7:17:36 PM]


MyPasTest

Main Navigation
Question Browser: MRCP 1
Home
Subscriptions Question Browser Exam Builder Saved Exams
» MRCP 1 Session Progress
• Question Browser
Questions Correct 25
• Timed Test A 34-year-old man presents to the Emergency room. His
wife says that he complained of sudden onset severe Questions Incorrect 10
• Mock Exam
headache, nausea and vomiting and double vision and then Questions Total 35
• Past Papers passed out on the hall floor. He works as a lawyer and has
Questions Percentage 71 %
• Random Questions no significant past medical history of note, but has recently
been undergoing a trial of sildenafil for erectile dysfunction. More
• My Performance
On examination he looks pale, his BP is 105/70 mmHg, his
• Media Bank pulse is 98/min. Heart sounds are normal, his chest is
• New Multimedia clear, and abdominal examination is unremarkable. He
begins to wake up more as you are examining him and is
Online Extras orientated, able to speak and obey commands. He
Library complains of severe retro-orbital headache. He has a
bitemporal superior quadrantinopia on visual field testing.
Community
You give him some IV cyclizine, as he continues to vomit.
Help
Investigations;
PasTest Store
Hb 12.4 g/dl
My Account WCC 5.4 x10 9 /l
Profile PLT 203 x10 9 /l
Newsletters
Na + 132 mmol/l
My Career and Exams
Order History K+ 5.4 mmol/l Reference: Normal Values
Learning Goals Creatinine 119 µmol/l
Click to open/closeHaematology
Question Filters TSH 1.5 mU/l (0.4-5.0) Click to open/closeImmunoglobulins
Free T4 7.7 pmol/l (low) Click to open/closeBiochemistry
Click to open/closeDiabetes Click to
Security Testosterone 8.0 nmol/l (low) open/closeEndocrinology Click to
open/closeBlood gases Click to
Change Password Which of the following is the most likely diagnosis open/closeCSF
Sign Out leading to his acute presentation?

Subarachnoid haemorrhage
Pituitary apoplexy Your answer
Migraine
Parietal lobe infarct
Non-functioning pituitary adenoma

The hormone tests with which we are presented indicate


hypothyroidism, hypoandrogenism, and probable
hypocortisolism. The most likely underlying cause is a
pituitary adenoma, but the history leading to his acute
presentation is typical of pituitary apoplexy,
characterised by sudden onset retro-orbital headache,
vomiting and relative hypotension, with diplopia and
visual field defects on CNS examination. MRI is the best
way to image the pituitary to support the diagnosis.
Over the long term he is likely to require corticosteroid,
testosterone and thyroid hormone replacement.

22520

file:///E|/Shakhawan/Endocrinology/335a.htm[3/13/2012 7:17:37 PM]


MyPasTest

Main Navigation
Question Browser: MRCP 1
Home
Subscriptions Question Browser Exam Builder Saved Exams
» MRCP 1 Session Progress
• Question Browser
Questions Correct 25
• Timed Test A 16-year-old Jewish girl comes to the GP complaining of
primary amenorrhoea. On examination she looks a little Questions Incorrect 10
• Mock Exam
hirsute, and has evidence of facial acne. She is within her Questions Total 35
• Past Papers predicted adult height and has normal breast and external
Questions Percentage 71 %
• Random Questions genitalia development, although there is excess hair over
her lower abdomen and around her nipple area. More
• My Performance
• Media Bank Investigations;
• New Multimedia
Hb 13.1 g/dl
Online Extras Reference: Normal Values
Library WCC 8.6 x10 9 /l
Click to
Community PLT 201 x10 9 /l
open/closeHaematology
Help Na + 139 mmol/l Click to
open/closeImmunoglobulins
PasTest Store
K+ 4.5 mmol/l Click to
open/closeBiochemistry
Creatinine 110 µmol/l Click to open/closeDiabetes
My Account
bilateral ovaries and uterus Click to
Profile Pelvic ultrasound visualised open/closeEndocrinology
Click to open/closeBlood
Newsletters 17-OH gases Click to
My Career and Exams progesterone 1.4 times the upper limit of normal open/closeCSF
Order History
Which of the following is the most likely diagnosis?
Learning Goals
Question Filters Polycystic ovarian syndrome
Classical congenital adrenal hyperplasia

Security Non-classical congenital adrenal hyperplasia


Turner’s syndrome
Change Password
Testicular femininisation
Sign Out
22531

© 2011 PasTest Ltd | About Us | Contact Us | Help

file:///E|/Shakhawan/Endocrinology/336.htm[3/13/2012 7:17:39 PM]


MyPasTest

Main Navigation
Question Browser: MRCP 1
Home
Subscriptions Question Browser Exam Builder Saved Exams
» MRCP 1 Session Progress
• Question Browser
Questions Correct 25
• Timed Test A 16-year-old Jewish girl comes to the GP complaining of
primary amenorrhoea. On examination she looks a little Questions Incorrect 11
• Mock Exam
hirsute, and has evidence of facial acne. She is within her Questions Total 36
• Past Papers predicted adult height and has normal breast and external
Questions Percentage 69 %
• Random Questions genitalia development, although there is excess hair over
her lower abdomen and around her nipple area. More
• My Performance
• Media Bank Investigations;
• New Multimedia
Hb 13.1 g/dl
Online Extras
Library WCC 8.6 x10 9 /l

Community PLT 201 x10 9 /l


Help Na + 139 mmol/l
PasTest Store
K+ 4.5 mmol/l
Creatinine 110 µmol/l
My Account
bilateral ovaries and uterus
Profile Pelvic ultrasound visualised
Newsletters 17-OH
My Career and Exams progesterone 1.4 times the upper limit of normal

Order History Reference: Normal Values


Which of the following is the most likely diagnosis?
Learning Goals
Click to open/closeHaematology
Question Filters Polycystic ovarian syndrome Click to open/closeImmunoglobulins
Classical congenital adrenal Your answer Click to open/closeBiochemistry
hyperplasia Click to open/closeDiabetes Click to
Security open/closeEndocrinology Click to
Non-classical congenital adrenal Correct answer
hyperplasia open/closeBlood gases Click to
Change Password open/closeCSF
Sign Out Turner’s syndrome
Testicular femininisation

Non classical CAH is a cause of hyperandrogenism in up


to 1:1000 females, particularly those of Hispanic,
Yugoslavian, or Eastern European Jewish descent. The
clinical picture may be confusing, as females may
complain of no other symptoms apart from primary
amenorrhoea. Management involves glucocorticoid
supplementation in juveniles, although in adults with
mild symptoms, flutamide as an anti-androgen may be
the only treatment required. The normal ultrasound
scan rules out both Turner’s and testicular
femininisation.

22531

© 2011 PasTest Ltd | About Us | Contact Us | Help

file:///E|/Shakhawan/Endocrinology/336a.htm[3/13/2012 7:17:40 PM]

Вам также может понравиться